algebra superior murray r spiegel

324

Click here to load reader

Upload: jose-oviedo

Post on 10-Aug-2015

535 views

Category:

Documents


57 download

TRANSCRIPT

Page 1: Algebra Superior Murray R Spiegel

,ALGEBRASUPERIOR

MUf¡ray R. Spiegel

E

I

I

, ALGEBRA SUPERIOR

601 problemas resueltos con so completamente detalladas.

Incluye 487 problemas propuestos con solución.

Abarca los aspectos teóricos y prácticos del álgebra.

Murray R. Spiegel

Entre los problemas resueltos figura la deducción de algunas fórmulas y teoremas.

, -, http://carlos2524.jimdo.com/

Page 2: Algebra Superior Murray R Spiegel

http://carlos2524.jimdo.com/

Page 3: Algebra Superior Murray R Spiegel

ALGEBRA SUPERIOR

MURRA y R. SPIEGEL, Ph. D.Professor of Mathematics

Rensse/aer Polytechnic Institute

TRADUCCIÓN Y ADAPTACIÓN

LUIS GUTIÉRREZ DIEZ

Ingeniero de Armamento

ANGEL GUTIÉRREZ VÁZQUEZ

Ingeniero de ArmamentoLicenciado en Ciencias Físicas

Diplomado en Ingeniería Nuclear

McGRAW-HILL

MÉXICO. BUENOS AIRES. CARACAS • GUATEMALA. LISBOA. MADRID. NUEVA YORKSAN JUAN. SANTAFÉ DE BOGOTÁ. SANTIAGO. SAO PAULO. AUCKLAND

LONDRES. MILÁN • MONTREAL • NUEVA DELHI • SAN FRANCISCO. SINGAPURSTo LOUIS • SIDNEY • TORONTO

ALGEBRA SUPERIOR

MURRAY R. SPIEGEL, Ph. D.

Professor of Mathematics Rensselaer Polytechnic Institute

TRADUCCIÓN Y ADAPTACIÓN

LUIS GUTIÉRREZ DiEZ

Ingeniero de Armamento

ANGEL GUTIÉRREZ V ÁZQUEZ

Ingeniero de Armamento Licenciado en Ciencias Físicas

Diplomado en Ingeniería Nuclear

McGRAW-HILL

MÉXICO. BUENOS AIRES. CARACAS. GUATEMALA. LISBOA. MADRID. NUEVA YORK SAN JUAN. SANTAFÉ DE BOGOTÁ. SANTIAGO. SAO PAULO. AUCKLAND

LONDRES. MILÁN. MONTREAL • NUEVA DELHI • SAN FRANCISCO. SINGAPUR STo LOUIS • SIDNEY • TORONTO

/ http://carlos2524.jimdo.com/

Page 4: Algebra Superior Murray R Spiegel

SPIEGEL

Gerente de Producto: Carlos Granados IslasSupervisorade edición: Leticia Medina VigilSupervisor de producción: Zeferino García García

ÁLGEBRA SUPERIOR

1402356789 G.A.91 09876543201

deteaall

qua~nese

paaldede

Ccreilecqucoenge

1, prch.libelday I

Prohibida la reproducción total o parcial de esta obra,por cualquier medio, sin autorización escrita del editor.

DERECHOS RESERVADOS © 1998, 1991, 1956 respecto a la primera edición en español porMcGRA W-HILLlINTERAMERICANA EDITORES, S.A. DE C.V.Una División de The McGraw-Hill Companies Inc.

Cedro Num 512, Col. AtlampaDelegación Cuauhtémoc06450 México, D.F.Miembro de la Cámara Nacional de la Industria Editotial Mexicana, Reg. Núm. 736

ISBN 970-10-2172-X(ISBN 968-422-925-9 1991-:'l9561 "

Translated ofthe firts edition in English ofSCHAUM'S OUTLINE OF COLLEGE ALGEBRACopyrigh © MCML VI, by McGraw-Hill, Inc., U.S.A.

ISBN 0-07-060-226-3

Impreso en México Printed in Mexico

Estaobrase lenninó deImprimiren Enerodel 2001 enLitográfica IngramexCenteno Núm. 162-1Col. Granjas EsmeraldaDelegación Iztapalapa09810 México, D.F.

'Se tiraron 1500 ejemplares

http://carlos2524.jimdo.com/

Page 5: Algebra Superior Murray R Spiegel

Prólogo

El álgebra, de brillante historia, con más de tres mil años de antígüedad, muy bien pudiera consi-derarse como el idioma universal de la civilización. Constituye la base sobre la que se apoya la alta ma-temática y es el lenguaje en que se expresan la ciencia y técnica modernas. Problemas de dificil solucióna partir de un planteamiento aritmético se resuelven mucho más fácilmente si se plantean en términosalgebraicos.

Igual que ocurre con los idiomas, el álgebra también exige muchas horas de dedicación antes deque el estudioso pueda considerarse versado en ella. El viejo adagio de que «no existe un camino deaprendizaje corto» no es una excepción en este caso. Para llegar a «hablar» con soltura este idioma esnecesario adquirir, ante todo, una idea clara y concisa de sus principios fundamentales y, después, po-seer una gran dosis de práctica.

El propósito de este libro es, en esencia, proporcionar al alumno los conocimientos necesariospara llegar a dominar este campo fundamental de la matemática. Además de servir como libro de textoa los alumnos de un curso medio de álgebra, puede ser de considerable utilidad para aquellos otros quedeseen repasar sus principios fundamentales y aplicaciones como introducción a ulteriores estudiosde matemáticas, ciencias o ingeniería.

El contenido del libro se divide en capítulos que abarcan todos los conceptos clásicos de la teoría.Cada uno de ellos comienza con un resumen, a modo de formulario, de las definiciones, principios y teo-remas correspondientes, junto con ejemplos ilustrativos de los mismos. A continuación, figura una co-lección de problemas resueltos y otra de problemas propuestos. Los primeros se han elegido de formaque proporcionen una visión clara de la aplicación correcta de los principios enunciados. Ilustran ycomplementan la teoría, ya que la repetición de los teoremas es de importancia vital para conseguir unaenseñanza eficaz e iluminan con potente foco aquellos conceptos que por su especial dificultad escapangeneralmente al alumno, y cuya ignorancia se traduce siempre en sentimiento de inseguridad. Entre losproblemas resueltos figura la deducción de algunas fórmulas y teoremas. El estudio que se hace de mu-chas de las materias tratadas es más profundo y completo que el que se encuentra en la mayoría de loslibros de texto; su exposición incluye el número complejo, la teoría de ecuaciones, la combinatoria,el cálculo de probabilidades, los determinantes y las series infinitas. La finalidad de la presente obra esdar respuesta a cualquier selección de temas propuesta por el profesor, servir como libro de consultay estimular un ulterior interés del alumno por la materia que aquí se trata.

l'

M. R. Spiegel

Prólogo

El álgebra, de brillante historia, con más de tres mil años de antígüedad, muy bien pudiera consi­derarse como el idioma universal de la civilización. Constituye la base sobre la que se apoya la alta ma­temática y es el lenguaje en que se expresan la ciencia y técnica modernas. Problemas de dificil solución a partir de un planteamiento aritmético se resuelven mucho más fácilmente si se plantean en ténninos algebraicos.

Igual que ocurre con los idiomas, el álgebra también exige muchas horas de dedicación antes de que el estudioso pueda considerarse versado en ella. El viejo adagio de que «no existe un camino de aprendizaje corto» no es una excepción en este caso. Para llegar a «hablar» con soltura este idioma es necesario adquirir, ante todo, una idea clara y concisa de sus principios fundamentales y, después, po­seer una gran dosis de práctica.

El propósito de este libro es, en esencia, proporcionar al alumno los conocimientos necesarios para llegar a dominar este campo fundamental de la matemática. Además de servir como libro de texto a los alumnos de un curso medio de álgebra, puede ser de considerable utilidad para aquellos otros que deseen repasar sus principios fundamentales y aplicaciones como introducción a ulteriores estudios de matemáticas, ciencias o ingeniería.

El contenido del libro se divide en capítulos que abarcan todos los conceptos clásicos de la teoría. Cada uno de ellos comienza con un resumen, a modo de fonnulario, de las definiciones, principios y teo­remas correspondientes, junto con ejemplos ilustrativos de los mismos. A continuación, figura una co­lección de problemas resueltos y otra de problemas propuestos. Los primeros se han eiegido de fonna que proporcionen una visión clara de la aplicación correcta de los principios enunciados. Ilustran y complementan la teoría, ya que la repetición de los teoremas es de importancia vital para conseguir una enseñanza eficaz e iluminan con potente foco aquellos conceptos que por su especial dificultad escapan generalmente al alumno, y cuya ignorancia se traduce siempre en sentimiento de inseguridad. Entre los problemas resueltos figura la deducción de algunas fórmulas y teoremas. El estudio que se hace de mu­chas de las materias tratadas es más profundo y completo que el que se encuentra en la mayoría de los libros de texto; su exposición incluye el número complejo, la teoría de ecuaciones, la combinatoria, el cálcu lo de probabilidades, los determinantes y las series infinitas . La finalidad de la presente obra es dar respuesta a cualquier selección de temas propuesta por el profesor, servir como libro de consulta y estimular un ulterior interés del alumno por la materia que aquí se trata.

M. R. Spiegel

http://carlos2524.jimdo.com/

Page 6: Algebra Superior Murray R Spiegel

e,1

23,

~5,

6.

~8.9.

10.1!.

12.

13.

1~

15.

16.

17.

18.

19.

n21.

nn.24.

25.

~2~28.m.AP

INI

http://carlos2524.jimdo.com/

Page 7: Algebra Superior Murray R Spiegel

Tabla de materias

CAPITULO PAGINA

1. OPERACIONES FUNDAMENTALES CON LOS NUMEROS...................... 12. 'OPERACIONES FUNDAMENTALES CON EXPRESIONES ALGEBRAICAS. . . . . . . 113. PRODUCTOS DE INTERES PRACTICO. . 214. DESCOMPOSICION EN FACTORES o o •••••••••••••••••• o • • • 265. FRACCIONES o •••••••••••••• o • o •• o •••••••••••••••••• o o •••••••••••• o o o o o o 356. POTENCIACION y RADICACION o •••••••••••••••• o •••••••••••••••••• o • • • 427. RADICALES o ••• o • • • • • • • • 538. OPERACIONES CON NUMEROS COMPLEJOS. . . . . . . . . . . . . . . . . . . . . . . . . . . . . . . . . . 639. ECUACIONES EN GENERAL. o • o • • • • • • • • • • • • • • • • • • • • • • • • • 67

10. FUNCIONES Y GRAFICAS. . . . . . . . . . . . . . . . . . . . . . . . . . . . . . . . . . . . . . . . . . . . . . . . . . . . . 7511. ECUACIONES LINEALES CON UNA INCOGNITA.............................. 8712. SISTEMAS DE ECUACIONES LINEALES. . . . . . 10013. ECUACIONES DE SEGUNDO GRADO CON UNA INCOGNITA................ 11014. ECUACIONES DE SEGUNDO GRADO CON DOS INCOGNITAS o........... 12715. RAZON, PROPORCION y PROPORCIONALIDAD o........... 13516. PROGRESIONES............................................................... 14017. TEOREMA DEL BINOMIO DE NEWTON....................................... 15518. PRINCIPIO MATEMATICO DE INDUCCION COMPLETA....................... 16319. DESIGUALDADES.............................................................. 16720. FORMA POLAR DE LOS NUMEROS COMPLEJOS............................. 17221. TEORIA DE ECUACIONES. . . 18222. LOGARITMOS o • • • • • • • • • • • •• 20923. INTERESES Y ANUALIDADES. . . 22124. COMBINATORIA o •••• o •••••••• '. 22925. PROBABILIDADES............................................................. 24226. DETERMINANTES Y SISTEMAS DE ECUACIONES LINEALES. . . 25227. DETERMINANTES DE ORDEN n. . . . . . . . . . . . . . . . . . . . . . . . . . . . . . . . . . . . . . . . . . . . .. 26028. FRACCIONES SIMPLES : o • o •• o • o •• o • o •••••• ~ • • • • • • • • • • • •• 27529. SERIES INFINITAS. . . . . . . . . . . . . . . . . . . . . . . . . . . . . . . . . . . . . . . . . . . . . . . . . . . . . . . . . . . .. 280APENDICE

TABLA DE LOGARITMOS DECIMALES ... o •••••• o ••••••••• o •••• o •• o • o • • • • • • • •• 300TABLA DEL INTERES COMPUESTO. . . . . . . . . . . . . . . . . . . . . . . . . . . . . . . . . . . . . . . . . .. 302VALOR ACTUAL DESPUES DE n PERIODOS..... . . . . . . . . . . . . . . . . . . . . . . . . . . . . .. 303CAPITAL DE UNA ANUALIDAD o o...... 304VALOR ACTUAL DE UNA ANUALIDAD...................................... 305

INDICE '0·................................... 307

Tabla de materias

CAPITULO PAGINA

1. OPERACIONES FUNDAMENTALES CON LOS NUMEROS .. .. ...... .. . ... ..... .

2. 'OPERACIONES FUNDAMENTALES CON EXPRESIONES ALGEBRAICAS. . . . . . . 11

3. PRODUCTOS DE INTERES PRACTICO. . .. . .................. . .......... . . . .... 21

4. DESCOMPOSICION EN FACTORES.. . ......... . .... . . ....... . .. . .. .. .......... . 26

5. FRACCIONES............................. .. ................... . ..... . .. . ... .. . 35

6. POTENCIACION y RADICACION. . . . . . . . . . . . . . . . . . . . . . . . . . . . . . . . . . . . . . . . . . . . . . 42

7. RADICALES........... . .. . .... .. ................. .. .. . ... .... . ... ..... ..... . . .. 53

8. OPERACIONES CON NUMEROS COMPLEJOS. . . . . . . . . . . . . . . . . . . . . . . . . . . . . . . . . . 63

9. ECUACIONES EN GENERAL....... . . . . . . . . . . . . . . . . . . . . . . . . . . . . .. . . . . . .. . . . .. . . 67

lO. FUNCIONES Y GRAFICAS . . . . . . . . . . . . . . . . . . . . . . . . . . . . . . . . . . . . . . . . . . . . . . . . . . . . . 75

11. ECUACIONES LINEALES CON UNA INCOGNITA.. .. .. .. .. .............. . . . ... 87

12. SISTEMAS DE ECUACIONES LINEALES . . . . . . .. .... ....... .. ... .. .. . .. . . . . .. . . 100

13. ECUACIONES DE SEGUNDO GRADO CON UNA INCOGNITA.... .... ........ 110

14. ECUACIONES DE SEGUNDO GRADO CON DOS INCOGNITAS... . .. . . ....... . 127

15. RAZON, PROPORCION y PROPORCIONALIDAD . ............ . ... . . ........ .. . 135

16. PROGRESIONES......... .. .... . ......... . . ... . . . .............................. 140

17. TEOREMA DEL BINOMIO DE NEWTON .... . .. ...... . ........ .. . . . .. . . . . . . .... 155

18. PRINCIPIO MATEMATICO DE INDUCCION COMPLETA......... .. .. .. . . ... . . . 163

19. DESIGUALDADES.... .. .. . ... . ........ .. .......... .... .. . .. . .. . .. .. . ........... 167

20. FORMA POLAR DE LOS NUMEROS COMPLEJOS. . .. .... . . .... ... .. .. ..... ... 172

21. TEORIA DE ECUACIONES. . . ............ . . .. .. . ......... ....... ... ..... ....... 182

22. LOGARITMOS........... . .. . ......... . . ..................... . ... ... .. . . . ....... 209

23. INTERESES Y ANUALIDADES. . . . . .. .... . .. . ..... ... . ....... .. ..... .. ....... .. 221

24. COMBINATORIA...... . ......... . ............ ... .. ... ... ...... . . ... . .... ....... 229

25. PROBABILIDADES.. . ....................... . .................... .. . . . . .. ... ... 242

26. DETERMINANTES Y SISTEMAS DE ECUACIONES LINEALES. . . . ......... . ... 252

27. DETERMINANTES DE ORDEN n. . . . . . . . . . . . . . . . . . . . . . . . . . . . . . . . . . . . . . . . . . . . .. 260

28 . FRACCIONES SIMPLES ......... : . ......... . ..... . .... ... ... . .... ~ . . . . . . . . . . . . . 275

29. SERIES INFINITAS. .. . . ..... . . .... . .... ....... . ........ .... . .. . .. . . . .. . . ....... 280

APENDICE

TABLA DE LOGARITMOS DECIMALES....... . ...... . . . . ....... .. . .. .......... 300

TABLA DEL INTERES COMPUESTO. . . . . . . . . . . . . . . . . . . . . . . . . . . . . . . . . . . . . . . . . .. 302

VALOR ACTUAL DESPUES DE n PERIODOS.... . . . . . . . . . . . . . . . . . . . . . . . . . . . . . . . 303

CAPITAL DE UNA ANUALIDAD....... .. .. .. . . ..... ... .. .... . . . .... . ... ... .. . 304

VALOR ACTUAL DE UNA ANUALIDAD.. . .......... . . . ....... ...... . ...... .. 305

INDICE... . .... . . ..................... ... ...... . ....... . ............. .. ............... 307

http://carlos2524.jimdo.com/

Page 8: Algebra Superior Murray R Spiegel

LAS

Sm.

RES

MUI

DIVi

EL<

http://carlos2524.jimdo.com/

Page 9: Algebra Superior Murray R Spiegel

CAPITULO 1

Operaciones fundamentales con los números

LAS CUATRO OPERACIONES fundamentales del álgebra y la aritmética son la suma, la resta,la multiplicación y la división.

SUMA. La suma, o adición, de dos números a y b se representa por a + b. Por ejemplo, 3 más 2, obien 3 y 2, se escribe 3 + 2 = 5.

RESTA. La resta, sustracción o diferencia de un número b de otro a se representa por a-b. Porejemplo, 6 menos 2, o bien, de 2 a 6, se escribe 6 - 2 = 4.

La resta es un caso particular de la suma. Esto es, la diferencia a - b es un número x tal quex más b proporciona el número a: x + b = a. Por ejemplo, 8 - 3 es un número x tal que suma-do a 3 da 8, es decir, x + 3 = 8, de donde 8 - 3 = 5.

MUL TIPLICACION. El producto de dos números a y bes otro número cy se representa así: a x b = c.La operación de multiplicar se puede indicar mediante una cruz, un punto o un paréntesis. Porejemplo, 5 x 3 = 5·3 = 5(3) = (5)(3) = 15, en donde los números 5 y 3 son los factores y 15el producto. Cuando se utilicen letras para representar números, se debe evitar la notación p x q,ya que el símbolo x se puede confundir con una letra que pudiera representar a otro número.

DIVISION. Cuando se divide un número a entre, o por, otro b, el cociente se representa así: a -:- b, a: bo alb, en donde a recibe el nombre de dividendo y b el de divisor. La expresión ajb también se de-nomina fracción, siendo. a el numerador y b el denominador.

La división por cero carece de sentido. [Véanse Probs. l(b), l(e).]

La división es un caso particular de la multiplicación. Esto es, el cociente afb es un número xtal que multiplicado por b da lugar al número a : bx = a. Por ejemplo, 6/3 es un número x tal quemultiplicado por 3 da 6, es decir, 3x = 6, de donde 6/3 = 2.

EL CONJUNTO DE NUMEROS REALES se establece, hoy en día, como resultado de un procesogradual de aplicación de otros conjuntos que vamos a reseñar a continuación.

1) Números naturales 1, 2, 3, 4, ... , (los puntos suspensivos significan «y así sucesivamente»)que se utilizan para contar y que se denominan también números enteros positivos. La suma ola multiplicación de dos números naturales es siempre otro número natural.

2) Números racionales positivos ofracciones positivas. Son los cocientes de dos enteros po-sitivos; por ejemplo, 2/3, 8/5, 121/17. El conjunto dé los números naturales está incluido en el delos números racionales positivos. Esto es, el número racional 3/1 es el número natural 3.

3) Números irracionales positivos. Son números no racionales como, por ejemplo, j2, tt.

CAPITULO 1

Operaciones fundamentales con los números

LAS CUATRO OPERACIONES fundamentales del álgebra y la aritmética son la suma, la resta, la multiplicación y la división.

SUMA. La suma, o adición, de dos números a y b se representa por a + b. Por ejemplo, 3 más 2, o bien 3 y 2, se escribe 3 + 2 = 5.

RESTA. La resta, sustracción o diferencia de un número b de otro a se representa por a-b. Por ejemplo, 6 menos 2, o bien, de 2 a 6, se escribe 6 - 2 = 4.

La resta es un caso particular de la suma. Esto es, la diferencia a - b es un número x tal que x más b proporciona el número a : x + b = a. Por ejemplo, 8 - 3 es un número x tal que suma­do a 3 da 8, es decir, x + 3 = 8, de donde 8 - 3 = 5.

MUL TIPLICACION. El producto de dos números a y bes otro número cy se representa así: a x b = c. La operación de multiplicar se puede indicar mediante una cruz, un punto o un paréntesis. Por ejemplo, 5 x 3 = 5·3 = 5(3) = (5)(3) = 15, en donde los números 5 y 3 son los factores y 15 el producto. Cuando se utilicen letras para representar números, se debe evitar la notación p x q, ya que el símbolo x se puede confundir con una letra que pudiera representar a otro número.

DIVISION. Cuando se divide un número a entre, o por, otro b, el cociente se representa así: a -:- b, a: b o a/b, en donde a recibe el nombre de dividendo y b el de divisor. La expresión a/b también se de­nomina fracción, siendo. a el numerador y b el denominador.

La división por cero carece de sentido. [Véanse Probs. l(b), l(e).]

La división es un caso particular de la multiplicación. Esto es, el cociente a/b es un número x tal que multiplicado por b da lugar al número a : bx = a. Por ejemplo, 6/3 es un número x tal que multiplicado por 3 da 6, es decir, 3x = 6, de donde 6/3 = 2.

EL CONJUNTO DE NUMEROS REALES se establece, hoy en día, como resultado de un proceso gradual de aplicación de otros conjuntos que vamos a reseñar a continuación.

1) Números naturales 1, 2, 3, 4, . .. , (los puntos suspensivos significan «y así sucesivamente») que se utilizan para contar y que se denominan también números enteros positivos. La suma o la multiplicación de dos números naturales es siempre otro número natural.

2) Números racionales positivos ofracciones positivas. Son los cocientes de dos enteros po­sitivos; por ejemplo, 2/3, 8/5, 121/17. El conjunto dé los números naturales está incluido en el de los números racionales positivos. Esto es, el número racional 3/1 es el número natural 3.

3) Números irracionales positivos. Son números no racionales como, por ejemplo, j2, 1t.

http://carlos2524.jimdo.com/

Page 10: Algebra Superior Murray R Spiegel

2 OPERACIONES FUNDAMENTALES CON LOS NUMEROS

4) Cero. Se presenta por O y se introduce para ampliar el sistema numérico, de forma quese puedan realizar operaciones tales como 6 - 6, 10 - 10, etc. El cero tiene la propiedad de quecualquier número multiplicado por él da cero. Cero dividido por cualquier número distinto decero (fO) es igual a cero.

5) Números negativos. Son los enteros, racionales e irracionales antepuestos del signo menosde la resta como, por ejemplo, - 3, - 2/3 Y - fi. Se introducen para ampliar el sistema numé-rico de forma que se puedan realizar operaciones tales como 2 - 8, ¡¡ - 3¡¡, 2 - 2fi, ete.

Cuando a un número no se le antepone signo alguno, se sobrentiende que es positivo. Así,pues, 5 quiere decir + 5, fi es + fi. El cero se considera como un número racional carente designo.

EL CONJUNTO DE NUMEROS REALES está formado por los números racionales e irracionales,tanto positivos como negativos, y el número cero.

Nota. La palabra real se emplea para distinguir estos números de otros que se caracterizanpor contener el 'término ¡=¡y que reciben el nombre de imaginarios. Aunque estos últimos apa-recen con suma frecuencia en las matemáticas y ciencias, en general, mientras no se diga lo con-trario, trataremos con números reales.

REPRESENT ACION GRAFICA DE LOS NUMEROS REALES. Los números reales se puedenrepresentar mediante los infinitos puntos de una recta. Para ello, se elige un punto de la mismaque represente al cero y que se toma como origen. Los enteros positivos, + 1, + 2, + 3, ... , seasocian con los puntos de la recta situados a distancias 1, 2, 3, ... , unidades, respectivamente,a la derecha del origen (véase figura), mientras que los enteros negativos - 1, - 2, - 3, ... , seasocian con los puntos de la recta situados a 1,2,3, ... , unidades, respectivamente, a la izquierdadel mismo.

RE(

RI I I

PI i I

-5 -4 -3 -2 "'-1MI

o S +1+

+2 +3 +4 +5

El número racional 1/2 se representa, en esta escala, por un punto P equidistante de los corres-pondientes al O y + 1. El número negativo -3/2, o -li, se representa por un punto R situado unaunidad y media a la izquierda del origen.

Se puede decir, pues, que a cada número real le corresponde un solo puntq de la recta y, re-cíprocamente, que a cada punto de la recta le corresponde un solo número real.

LA POSICION DE LOS NUMEROS REALES sobre una recta establece un orden en el conjuntode dichos números. Si un punto A está situado a la derecha de otro B de la recta el número corres-pondiente a A es mayor que el correspondiente a B, o bien, el número correspondiente a B es menorque el correspondiente a A. Las expresiones «mayor que» y «menor que» se 'representan por lossímbolos > y -c , respectivamente, y son «signos de desigualdad».

Por ejemplo, como 5 está a la derecha de 3, 5 es mayor que 3, es decir, 5 > 3; también se de-duce que 3 es menor que 5, y se escribe 3 < 5. Análogarnente, como - 6 está a la izquierda de - 4,-6 es más pequeño que -4, es decir. -6 < -4; también se deduce en este caso que -4 > -6.

POl

EL VALOR ABSOLUTO de un número es el correspondiente al número prescindiendo del signoque le afecte. El valor absoluto se representa encerrando el número entre dos barras verticales.Por ejemplo, 1-61 = 6, 1+41 = 4, 1-3/41 = 3/4.

íPROPIEDADES DE LA SUMA Y DE LA MULTlPLlCACION

1) Propiedad conmutativa de la suma. El orden de los sumandos no altera el valor dela suma.

Es decir: a + b = b + a, 5 + 3 = 3 + 5 = 8.

2 OPERACIONES FUNDAMENTALES CON LOS NUMEROS

4) Cero . Se presenta por ° y se introduce para ampliar el sistema numérico, de forma que se puedan realizar operaciones tales como 6 - 6, 10 - 10, etc. El cero tiene la propiedad de que cualquier número multiplicado por él da cero. Cero dividido por cualquier número distinto de cero (+0) es igual a cero.

5) Números negativos. Son los enteros, racionales e irracionales antepuestos del signo menos

de la resta como, por ejemplo, - 3, - 2/3 Y -)2. Se introducen para ampliar el sistema numé­

rico de forma que se puedan realizar operaciones tales como 2 - 8, ¡¡ - 3¡¡, 2 - 2)2, etc.

Cuando a un número no se le antepone signo alguno, se sobrentiende que es positivo . Así,

pues, 5 quiere decir + 5, )2 es +)2. El cero se considera como un número racional carente de signo.

EL CONJUNTO DE NUMEROS REALES está formado por los números racionales e irracionales, tanto positivos como negativos, y el número cero.

Nota . La palabra real se emplea para distinguir estos números de otros que se caracterizan

por contener el término J=l y que reciben el nombre de imaginarios. Aunque estos últimos apa­recen con suma frecuencia en las matemáticas y ciencias, en general, mientras no se diga lo con­trario, trataremos con números reales.

REPRESENT ACION GRAFICA DE LOS NUMEROS REALES. Los números reales se pueden representar mediante los infinitos puntos de una recta. Para ello, se elige un punto de la misma que represente al cero y que se toma como origen. Los enteros positivos, + 1, + 2, + 3, ... , se asocian con los puntos de la recta situados a distancias 1, 2, 3, ... , unidades, respectivamente , a la derecha del origen (véase figura), mientras que los enteros negativos - 1, - 2, - 3, . . . , se asocian con los puntos de la recta situados a 1,2, 3, . . . , unidades, respectivamente, a la izquierda del mismo.

-5 -4 -3

R I I I

-2 N-1 'M

I

o P i I S +1 +2 +3 +4 +5

+

El número racional 1/2 se representa, en esta escala , por un punto P equidistante de los corres­pondientes al ° y + 1. El número negativo - 3/2, o - 1 t, se representa por un punto R situado una unidad y media a la izquierda del origen.

Se puede decir, pues, que a cada número real le corresponde un solo punt~ de la recta y, re­cíprocamente, que a cada punto de la recta le corresponde un solo número real.

LA POSICION DE LOS NUMEROS REALES sobre una recta establece un orden en el conjunto de dichos números. Si un punto A está situado a la derecha de otro B de la recta el númeró corres­pondiente a A es mayor que el correspondiente a B, o bien, el número correspondiente a B es menor que el correspondiente a A. Las expresiones «mayor que» y «menor que» se 'representan por los símbolos > y <, respectivamente, y son «signos de desigualdad».

Por ejemplo, como 5 está a la derecha de 3, 5 es mayor que 3, es decir, 5 > 3; también se de­duce que 3 es menor que 5, y se escribe 3 < 5. Análogamente, como - 6 está a la izquierda de - 4, -6 es más pequeño que -4, es decir, -6 < - 4; también se deduce en este caso que -4> -6.

EL VALOR ABSOLUTO de un número es el correspondiente al número prescindiendo del signo que le afecte. El valor absoluto se representa encerrando el número entre dos barras vertica les . Por ejemplo, 1-61 = 6, 1+41 = 4, 1-3/41 = 3/4.

í PROPIEDADES DE LA SUMA Y DE LA MUL TIPLICACION

1) Propiedad conmutativa de la suma. El orden de los sumandos no altera el valor de la suma.

Es decir: a + b = b + a, 5 + 3 = 3 + 5 = 8.

http://carlos2524.jimdo.com/

Page 11: Algebra Superior Murray R Spiegel

OPERACIONES FUNDAMENTALES CON LOS NUMEROS

2) Propiedad asociativa de la suma. Se pueden agrupar los sumandos de cualquier formasin que se modifique el valor de la suma.

a + b + e = a + (b + e) = (a + b) + e. 3 + 4 + 1 = 3 + (4 + 1) = (3 + 4) + 1 = 8

3) Propiedad conmutativa de la multiplicación. El orden de los factores no altera el valordel producto.

a' b = b : a, 2· 5 = 5' 2 = 10

4) Propiedad aso. .ativa de la multiplicación. Se pueden agrupar los factores de cualquierforma sin que se modifique el valor del producto.

abe = a(be) = ab(e), 3' 4· 6 = 3(4' 6) = (3 . 4)6 = 72

5) Propiedad distributiva de la multiplicación. El producto de un número a por la sumade otros dos (b + e) es igual a la suma de los productos ab y ac,

a(}:. .¡. e) = ab + ae, 4(3 + 2) = 4' 3 + 4' 2 = 20

Estas propiedades son susceptibles de una generalización. Quiere esto decir que se puedensumar los números a, b, e, d, e, agrupándolos en un orden cualquiera como, por ejemplo,(a + b) + e + (d + e), a + (b + e) + (d + e), etc. Análogamente, en la multiplicación, se puedeponer (ab)c(de), o bien, a(bc)(de), siendo el resultado independiente de la forma en que se realiceel agrupamiento.

REGLAS DE LOS SIGNOS

1) Para sumar dos números del mismo signo se suman sus valores absolutos y se anteponeal resultado dicho signo común. Por ejemplo, 3 + 4 = 7, (- 3) + (- 4) = - 7.

2) Para sumar dos números de signos diferentes se efectúa la diferencia entre sus valoresabsolutos y se antepone al resultado el signo del sumando de mayor valor absoluto.

Ejemplos. 17 + (-8) = 9, (-6) + 4 = -2, (-18) + 15 = -3

3) Para restar un número b de otro a, se cambia el signo de b y se le suma a.Ejemplos. 12 - (7) = 12+ (-7)= 5, (-9) - (4) = -9 + (-4) = -13, 2 - (-8) = 2 + 8 = 10

4) Para multiplicar (o dividir) dos números del mismo signo se multiplican (o dividen) susvalores absolutos y se antepone al resultado el signo más (o no se pone signo).

-6Ejemplos. (5)(3) = 15, (-5)(-3) = 15, -3= 2

5) Para multiplicar (o dividir) dos números de signos diferentes, se multiplican (o dividen)sus valores absolutos y se antepone al resultado el signo menos.

Ejemplos. ( - 3)(6) = - 18, (3)( -6) = -18,-12-= -3

4

POTENCIAS Y EXPONENTES. Cuando un número a se multiplica consigo mismo n veces, elproducto a . a . a ... a (n veces) se representa por el símbolo a" que se lee «potencia enésima de a»o bien «a elevado a la potencia n» o todavía «a a la n».

Ejemplos. 2'2'2'2'2 = 25 = 32, (-w = (-5)(-5)(-5) = -1252' X' X' x = 2x3, a' a' a' b· b = a'b", (a - b)(a - b)(a - b) = (a - b)3

En la potencia a", el número a recibe el nombre de base y el número positivo y entero n el deexponente.

3OPERACIONES FUNDAMENTALES CON LOS NUMEROS 3

2) Propiedad asociativa de la suma. Se pueden agrupar los sumandos de cualquier forma sin que se modifique el valor de la suma.

a + b + e = a + (b + e) = ((1 + b) + e. 3 + 4 + 1 = 3 + (4 + 1) = (3 + 4) + 1 = 8

3) Propiedad conmutativa de la multiplicación. El orden de los factores no altera el valor del producto.

(l' b = b' a, 2'5=5'2=\0

4) Propiedad aso( ,ativa de la multiplicación. Se pueden agrupar los factores de cualquier forma sin que se modihque el valor del producto.

abe = a(be) = abre), 3' 4· 6 = 3(4' 6) = (3' 4)6 = 72

5) Propiedad distributiva de la multiplicación. El producto de un número a por la suma de otros dos (b + e) es igual a la suma de los productos ab yac.

(I(b .¡. e) = ab + ae, 4(3 + 2) = 4' 3 + 4' 2 = 20

Estas propiedades son susceptibles de una generalización . Quiere esto decir que se pueden sumar los números a, b, e, d, e, agrupándolos en un orden cualquiera como, por ejemplo, (a + b) + e + (d + e), a + (b + e) + (d + e), etc. Análogamente, en la multiplicación, se puede poner (ab)c(de), o bien, a(be)(de), siendo el resultado independiente de la forma en que se realice el agrupamiento.

REGLAS DE LOS SIGNOS

1) Para sumar dos números del mismo signo se suman sus valores absolutos y se antepone al resultado dicho signo común. Por ejemplo, 3 + 4 = 7, (-3) + (-4) = -7.

2) Para sumar dos números de signos diferentes se efectúa la diferencia entre sus valores absol utos y se antepone al resultado el signo del sumando de mayor valor absoluto.

Ejemplos. 17 + (- 8) = 9, (-6) + 4 = -2, (-18)+ 15=-3

3) Para restar un número b de otro a, se cambia el signo de b y se le suma a.

Ejemplos. 12 - (7) = 12 + ( - 7) = 5, ( - 9) - (4) = - 9 + (- 4) = - 13, 2 - ( - 8) = 2 + 8 = 10

4) Para multiplicar (o dividir) dos números del mismo signo se multiplican (o dividen) sus valores absolutos y se antepone al resultado el signo más (o no se pone signo).

-6 Ejemplos. (5)(3) = 15, (-5)(-3) = 15, -3= 2

5) Para multiplicar (o dividir) dos números de signos diferentes, se multiplican (o dividen) sus valores absolutos y se antepone al resultado el signo menos.

Ejemplos. ( - 3)(6) = - 18, (3)( -6) = -18, -12 -= -3

4

POTENCIAS Y EXPONENTES. Cuando un número a se multiplica consigo mismo n veces, el producto a . a . a .•. a (n veces) se representa por el símbolo an que se lee «potencia enésima de a»

o bien «a elevado a la potencia n» o todavía «a a la n».

Ejemplos. 2'2'2'2'2 = 25 = 32, (-w = (-5)(-5)(-5) = -125 2' X' X' x = 2x3, a' a' a' b· b = a3b2

, (a - b)(a - bita - b) = (a - b)3

En la potencia an, el número a recibe el nombre de base y el número positivo y entero n el de

exponente.

http://carlos2524.jimdo.com/

Page 12: Algebra Superior Murray R Spiegel

4 OPERACIONES FUNDAMENTALES CON LOS NUMEROS

PROPIEDADES DE LAS POTENCIAS. Si p Y q son enteros y positivos, se verifica:

34 1 1jb = 36-4 = 32 1. 1-

b

a

b

OPERACIONES CON FRACCIONES. Se efectúan teniendo en cuenta las reglas siguientes:

1) El valor de una fracción no se altera si se multiplican numerador y denominador por unmismo número distinto de cero.

e:IT

Ejemplos. ~3,24,2

68'

1518

15 -7- 318 -7- 3

5'6

d

e

2) Si se cambia el signo del numerador, o el del denominador, de una fracción, ésta cambiade signo.

x

Ejemplo. -35

3) La suma de dos fracciones del mismo denominador es igual a una fracción que tiene pornumerador la suma de los numeradores y por denominador dicho denominador común.

2. E

Ejemplo. 3 4 3 + 4 75" + 5" = -5- = 5"

ab

d

4) La suma de dos fracciones de distinto denominador se efectúa como en 3) una vez quese hayan transformado las fracciones a un denominador común.

l'

a

Ejemplo. b

5) El producto de dos fracciones es otra fracción cuyo numerador es igual al producto delos numeradores y el denominador igual al producto di! los denominadores. d

e

E· I 2 4~empos. "3' 5" 2'43, 5

815'

3 8 3, 8 24 2¡.9 = 4 . 9 = 36 = "3 j

6) El recíproco de una fracción es la fracción cuyos numerador y denominador son, res-pectivamente, el denominador y numerador de la fracción dada. Así, pues, el recíproco de 3 (esdecir, 3/1) es 1/3. Análogamente, los recíprocos de 5/8 y - 4/3 son 8/5 y - 3/4, respectivamente.

7) Para dividir dos fracciones se multiplica la primera por el recíproco de la segunda. a e a d ad 2 4 2 5 10 5

EJemplos. b -7- d = b . e = be' "3 -7- 5" = "3 • ¡= 12 = '6h

pa e afb alb . bd ad

El resultado se puede expresar como sigue: b -7- -¡j= c/d = c/d, bd = be

k

4 OPERACIONES FUNDAMENTALES CON LOS NUMEROS

PROPIEDADES DE LAS POTENCIAS. Si p Y q son enteros y positivos, se verifica:

1) ú"a" = ap +q

aP 1 2) - = aP -

q = -- si a ..t. O a" a"- P -r 34 1 1 ~ = 36 - 4 = 32

OPERACIONES CON FRACCIONES. Se efectúan teniendo en cuenta las reglas siguientes :

1) El valor de una fracción no se altera si se multiplican numerador y denominador por un mismo número distinto de cero.

Ejemplos. i 3,2 4,2

6 8'

15 18

15 -7- 3 18 -7- 3

5 '6

2) Si se cambia el signo del numerador, o el del denominador, de una fracción , ésta cambia de signo.

Ejemplo. -3 5

3 5

3 -5

3) La suma de dos fracciones del mismo denominador es igual a una fracción que tiene por numerador la suma de los numeradores y por denominador dicho denominador común.

Ejemplo. 3 4 3 + 4 7 5' + 5' = -5- = 5'

4) La suma de dos fracciones de distinto denominador se efectúa como en 3) una vez que se hayan transformado las fracciones a un denominador común.

Ejemplo.

5) El producto de dos fracciones es otra fracción cuyo numerador es igual al producto de los numeradores y el denominador igual al producto d~ los denominadores.

E· l 2 4 ~emp os. "3 ' 5'

2, 4 8 3, 5 15'

3 8 3, 8 24 2 ¡ . 9" = 4 . 9 = 36 = "3

6) El recíproco de una fracción es la fracción cuyos numerador y denominador son, res­pectivamente, el denominador y numerador de la fracción dada. Así, pues, el recíproco de 3 (es decir, 3/ 1) es 1/3. Análogamente, los recíprocos de 5/8 y - 4/3 son 8/5 y - 3/4, respectivamente.

7) Para dividir dos fracciones se multiplica la primera por el recíproco de la segunda . a cad ad 2 4 2 5 10 5

Ejemplos. b -7- d = b . e = bc' "3 -7- 5' = "3 • 4' = 12 = '6

a c a/b a/b . bd ad El resultado se puede expresar como sigue : b -7- {¡= c/d = c/d , bd = bc

http://carlos2524.jimdo.com/

Page 13: Algebra Superior Murray R Spiegel

OPERACIONES. FUNDAMENTALES CON LOS NUMEROS 5

PROBLEMAS RESUELTOS

I. Hallar la suma S. diferencia D. producto P y cociente Q de cada uno de los siguientes pares de números: a) 48.12;

b) 8, O: e) O, 12; d) 10. 20; e) O, O.48

a) S = 48 + 12 = 60. D = 48 - 12 = 36. P = 48(12) = 576. Q = 48 -7 12 = 12 = 4

b) S = 8 + O = 8. D = 8 - O = 8. P = 8(0) = o. Q = 8 -7 0.0 bien ~8

Pero. por definición. -O

es un número x (si existe) de forma que x(O) = 8. Ahora bien, un número tal no existe, ya que todo númeromultiplicado por O da cero.

e) S = O + 12 = 12, D = O - 12 = -12, P = 0(12) = O,O

Q = - = O12

10 1d) S = 10 + 20 = 30, D = 10 - 20 = -10, P = 10(20) = 200, Q = 10 -7 20 = 20 = "2

Oe) S = O + O = O, D = O - O = O, P = 0(0) = O, Q = O -7 O, o bien O es por definición un número

. Ox (si existe) tal que x(O) = O. Como esto ocurre para Iodos los números x, el valor del cociente O es inde-

terminado.

De b) y e) se deduce que la división por cero es una operación que carece de sentido.

a

2. Efectuar las siguientes operaciones indicadas.

¡r) 756 -7 21

i) 72 -7 24 + 64 -7 16

j) 4 -7 2 + 6 -7 3 - 2 -7 2 + 3 . 4

k) 128 -7 (2' 4), (128 -7 2)' 4

e

a) 42 + 23, 23 + 42h) 27 + (48 + 11). (27 + 48) + 12c) 125 - (38 + 27)d) 6'8, 8·6 h) (40+21)(72-38)e) 4(7' 6). (4' 7)6 (32 - 15)

a) 42 + 23 = 65. 23 + 42 = 65. Luego 42 + 23 = 23 + 42.Este es un ejemplo de la propiedad conmutativa de la suma.

f) 35·28

h) 27 + (48 + 12) = 27 + 60 = 87. (27 + 48) + 12 = 75 + 12 = 87. Luego 27 + (48 + 12) = (27 + 48) + 12.Este es un ejemplo de la propiedad asociativa de la suma.'

ee) 125 - (38 + 27) = 125 - 65 = 60.

d) 6' 8 = ?8. 8 . 6 = 48. Luego 6 . 8 = 8 . 6. ejemplo de la propiedad conmutativa de la multiplicación.

e) 4(7' 6) = 4(42) = 168. (4' 7)6 = (28)6 = 168. Luego 4(7' 6) = (4' 7)6.Este es un ejemplo de la propiedad asociativa de la multiplicación.

/) (35)(28) = 35(20 + 8) = 35(20) + 35(8) = 700 + 280 = 980 por la propiedad distributiva de la multi-plicación.

756¡r) TI = 36 Comprobacion : 11 . 36 = 756

2

h) (40 + 21)(72 - 38) = (61)(34) = ~~ = 61.2 = 122(32 - 15) 17 JlI

i) Los cálculos aritméticos. por convenio. obedecen a la siguiente regla: las operaciones de multiplicar y dividirpreceden a las de sumar y restar.

Luego 72 -7 24 + 64 -7 16 = 3 + 4 = 7.

j) Aplicando la regla i) tendremos 4 -72 + 6 -7 3 - 2 -7 2 + 3-.:.4 = 2 + 2 - 1 + 12 = 15.

k) 128 -7 (2'4) = 128 -7 8 = 16. (128 -7 2)'4 = 64·4 = 256.Si se escribe 128 -7 2 . 4 sin paréntesis, no se sabría el orden de las operaciones.

OPERACIONES. FUNDAMENTALES CON LOS NUMEROS 5

PROBLEMAS RESUELTOS

1. Hallar la suma S. diferencia D. producto P y cociente Q de cada uno de los siguientes pares de números: a) 48.12;

2.

b) 8, O: e) O, 12; d) 10. 20; e) O, O.

a) S = 48 + 12 = 60. D = 48 - 12 = 36. P = 48(12) = 576. 48

Q = 48.;- 12 = - = 4 12

b) S = 8 + O = 8. D = 8 - O = 8, P = 8(0) = O, . . 8

Q = 8 .;- O, o bien O 8 Pero, por definición. O

es un número x (si existe) de forma que x(O) = 8. Ahora bien, un número tal no existe, ya que todo número multiplicado por O da cero.

e) S = O + 12 = 12, D = O - 12 = -12, P = 0(12) = O, O

Q = - = O 12

10 1 d) S = 10 + 20 = 30, D = 10 - 20 = -10, P = 10(20) = 200, Q = 10 .;- 20 = 20 = "2

e) S = O + O = O, O

D = O - O = O, P = 0(0) = O, Q = O .;- O, o bien O es por definición un número

O x (si existe) tal que x(O) = O. Como esto ocurre para lodos los números x, el valor del cociente O es inde-

terminado.

De b) y e) se deduce que la división por cero es una operación que carece de sentido.

Efectuar las siguientes operaciones indicadas.

a) 42 + 23. 23 + 42 f) 35·28 i) 72.;- 24 + 64 .;- 16 h) 27 + (48 + 12). (27 + 48) + 12

¡r) 756 .;- 21 c) 125 - (38 + 27)

j) 4.;- 2 + 6 .;- 3 - 2 .;- 2 + 3 . 4

d) 6' 8, 8·6 h)

(40 + 21)(72 - 38) e) 4(7' 6). (4' 7)6 (32 - 15)

k) 128.;- (2'4), (128.;- 2)'4

a) 42 + 23 = 65. 23 + 42 = 65. Luego 42 + 23 = 23 + 42. Este es un ejemplo de la propiedad conmutativa de la suma.

h) 27 + (48 + 12) = 27 + 60 = 87, (27 + 48) + 12 = 75 + 12 = 87. Luego 27 + (48 + 12) = (27 + 48) + 12. Este es un ejemplo de la propiedad asociativa de la suma. '

e) 125 - (38 + 27) = 125 - 65 = 60.

d) 6' 8 = ~8. 8 . 6 = 48 . Luego 6 . 8 = 8 . 6. ejemplo de la propiedad conmutativa de la multiplicación.

e) 4(7' 6) = 4(42) = 168. (4' 7)6 = (28)6 = 168. Luego 4(7' 6) = (4' 7)6. Es~ es un ejemplo de la propiedad asociativa de la multiplicación.

f) (35)(28) = 35(20 + 8) = 35(20) + 35(8) = 700 + 280 = 980 por la propiedad distributiva de la multi­plicación.

756 ¡r) TI = 36 Comprohación: 21 . 36 = 756

2

h) (40 + 21)(72 - 38) = (61)(34) = ~~ = 61.2 = 122 (32 - 15) 17 YI

i) Los cálculos aritméticos. por convenio. obedecen a la siguiente regla: las operaciones de multiplicar y dividir preceden a las de sumar y restar.

Luego 72 .;- 24 + 64 .;- 16 = 3 + 4 = 7.

j) Aplicando la regla i) tendremos 4.;- 2 + 6 .;- 3 - 2 .;- 2 + }..:. 4 = 2 + 2 - 1 + 12 = 15 .

k) 128.;- (2'4) = 128.;- 8 = 16. (128 .;- 2)'4 = 64·4 = 256. Si se escribe 128 .;- 2 . 4 sin páréntesis. no se sabría d orden de las operaciones.

http://carlos2524.jimdo.com/

Page 14: Algebra Superior Murray R Spiegel

6 OPERACIONES FUNDAMENTALES CON LOS NUMEROS

3. Clasificar los siguientes números según las categorías: número real, entero positivo, entero negativo, número ra-cional, número irracional, ninguna de las anteriores.

- 5, 3/5, 3n, 2, -1/4, 6,3, O, .)5, F, 0,3782, )4, -18/7 y

Si el número pertenece a una o más categorías, indíquese por medio de una señal.

Número Entero Entero Número Número Ninguno dereal positivo negativo racional irracional los anter.

-5 .¡ ,¡ ,¡

3/5 ,¡ .¡

37t ,¡ .¡

2 ,¡ .¡ .¡

-1/4 .¡ ,¡

6,3 ,¡ ,¡

O .¡ ,¡

.¡5 ,¡ ,¡

r-I ,¡

0,3782 ,¡ .¡

/4 ,¡ ,¡ ,¡

-18/7 ,¡ ,¡

8. E

ablel

t-

~~ ~ ." '"." ..• ..• <,Ñ <D ~

I I I '" '"I ~ I 1 I I 1 11,1 I I I I I 11 I 1 I ! I

-6 -5 -4 -3 -2 -1 O +1 +21 +3 +4 +5 +6 +7 +8 +9 +10N ~00e--,..,o

9. H:b)

a)

b)

e)

d)

10. Ef,

a)

b)

e)

d)

n. Ca

a)

b)

e)

d)

e)

i)

j)

12. Tncae

a)

4. Representar (aproximadamente) los números reales del Problema 3 por medio de puntos sobre una escala gráfica.

No/a. 3n es, aproximadamente, 3(3,14) = 9,42; por tanto, el punto correspondiente está comprendidoentre +9 y + 10. El valor de .)5 con tres cifras decimales es 2,236 y, por tanto, está comprendido entre 2 y 3.

5. Colocar correctamente el signo de desigualdad « o » entre cada uno de los pares de números reales siguientes:a) 2, 5 e( 3, -1 e) -4, -3 g) J7, 3 i) -3/5, -1/2b) O, 2 d) -4, +2 J) n, 3 h) -fi,-1

a) 2 < 5 (65) 2), es decir, 2 es menor que 5 (o 5 es mayor que 2)b) 0<2 (o 2> O) e) -4 < -3 (o -3> -4) h) -.j2 < -1 (-1> -.j2)e) 3> -1 (o -1 < 3) f) n > 3 (o 3 < n) i) -3/5 < -1/2, ya que -0,6 < -0,5d) -4 < +2 (o +2 > -4) g) 3 > J7 (o J7 < 3)

6. Ordenar los siguientes grupos de números reales disponiéndolos de menor a mayor.a) -3,22/7,.)5, -3,2, O; b) -.j2, -)3, -1,6, -3/2.

a) -3,2 < -3 < O <.)5 < 22/7 b) -)3 < -1,6 < -3/2 < -.j2

7. Escribir el valor absoluto de los siguientes números reales.-1, +3,2/5, -.j2, -3,14,2,83, -3/8, -l!, +5/7

http://carlos2524.jimdo.com/

Page 15: Algebra Superior Murray R Spiegel

OPERACIONES FUNDAMENTALES CON LOS NUMEROS 7

Los valores absolutos de los números dados se representan por

1-11, 1+31, 12/51, I-fil, 1-3,141, 12,831, 1-3/81, I-nl, 1+5/71

y su valor es igual a 1, 3, 2/5, .)2, 3,14, 2,83, 3/8, n, 5/7, respectivamente.

8. Efectuar las sumas y restas de los números reales siguientes:

a) (-3) + (-8) = -11b) (-2)+3=1e) (-6) + 3 = -3

d) -2 + 5 = 3e) -15+8=-7f) (-32)+48+(-10)=6

g) 50 - 23 - 27 = O11) -3-(-4)=-3+4=1i) -(-14) + (-2) = 14 - 2 = 12

9. Hallar la suma S, diferencia D, producto P y cociente Q de los siguientes pares de números reales: a) -2,2;b) -3,6; e) O, -5; d) -5, O.

a) S=-2+2=0, D=(-2)-2=-4, P=(-2)(2)=-4, Q=-2/2=-1

b) S=(-3)+6=3, D=(-3)-6=-9, P=(-3)(6)=-18, Q=-3/6=-1/2

e) S = O + (- 5) = - 5, D = O - (- 5) = 5, P = (0)( - 5) = O, Q = O/ - 5 = O

d) S = (-5) + 0= -5, D = (-5) - O = -5, P = (-5)(0) = O, Q = -5/0 no está definida la operación.

10. Efectuar las siguientes operaciones:

a) (5)(-3)(-2) = [(5)(-3)](-2) = (-15)(-2) = 30= (5)[(-3)(-2)] = (5)(6) = 30

El orden de los factores no altera el producto.

b) 8(-3)(10) = -240

e) 8~42) + (-4~-2) = -=-~6+ ~ = 4 + 4 = 8

12(-40)(-12) = 12(-40)(-12) = 12(-40)(-12) = -960d) 5(-3) _ 3(-3) -15 - (-9) -6

ll. Calcular:

a) 23 = 2·2·2 = 8

b) 5(3)' = 5' 3 . 3 = 45

e) 24. 26 = 24+6 = 210 = 1024

52 . 53 55 1f) 5' = 57 = 57-5 = 52

l25

d) 25. 52 = (32)(25) = 800

12. Transformar las funciones siguientes en otras equivalentes cuyo denominador sea el número que se indica encada caso.

a) 1/3; 6 b) 3/4; 20 e) 5/8; 48 d) -3/7; 63 e) -12/5; 75

OPERACIONES FUNDAMENTALES CON LOS NUMEROS

Los valores absolutos de los números dados se representan por

1-11, 1+ 31, 12/51, I-fil· 1-3,141, 12,831, 1- 3/81, I-nl. 1+ 5/71

y su valor es igual a 1, 3, 2/5, .j2, 3,14, 2,83 , 3/8, n, 5/7. respectivamente.

8. Efectuar las sumas y restas de los números reales siguientes :

a) (-3) + (-8) = -11 d) -2 + 5 = 3 g) 50 - 23 - 27 = O b) (-2)+3=1 e) -15+8=-7 11) -3-(-4)=-3+4=1

7

e) (-6) + 3 = -3 f) (-32) + 48 + (-JO) = 6 i) -(-14) + (-2) = 14 - 2 = 12

9. Hallar la suma S, diferencia D, producto P y cociente Q de los siguientes pares de números reales: a) - 2, 2; b) -3,6; e) O, -5; d) -5, O.

a) S=-2+2=0, D=(-2)-2=-4, P=(-2)(2)=-4, Q=-2/2=-1

b) S=(-3)+6=3, D=(-3)-6=-9, P=(-3)(6)=-18, Q=-3/6=-1 /2

e) S=0+(-5)=-5, D=0-(-5)=5, P=(0)(-5)=0, Q=0/ -5=0

d) S = (-5) + 0= -5, D = (-5) - O = -5, P = (-5)(0) = O, Q = -5/0 no está definida la operación.

10. Efectuar las siguientes operaciones:

a) (5)(-3)(-2) = [(5)(-3)](-2) = (-15)(-2) = 30 = (5)[(-3)(-2)] = (5)(6) = 30

El orden de los factores no altera el producto.

b) 8(-3)(10) = -240

) 8(-2)+(-4)(-2)=-16+~=4+4=8 e -4 2 -4 2

d 12(-40)(-12) = 12(-40)(-12) = 12(-40)(-12) = -960 ) 5(-3) - 3(-3) -15 - (-9) -6

11. Calcular:

a) 23 = 2 . 2 . 2 = 8 52 . 53 5s I

f) ---s" = 5' = 57 - S = 52 = 25 b) 5(3)2 = 5' 3 . 3 = 45

e) 24 . 26 = 24+6 = 2'0 = 1024

d) 2s , 52 = (32)(25) = 800

34 . 33 37

e) -- = - = 37-

2 = 3s = 243 32 32

i) (34)3. (32)4 3'2. 38 320

(-3)'s . 34 _3's, 34 = - 3'9 = -3' = -3

38 42 • 24 .. 42

j) JS - 26 + 3(-2)3 = 33 - 22 + 3(-8) = 27 - 4 - 24 = -1

12. Transformar las funciones siguientes en otras equivalentes cuyo denominador sea el número que se indica en cada caso.

a) 1/3; 6 b) 3/4; 20 e) 5/8; 48 d) -3/7; 63 e) -12/5 ; 75

http://carlos2524.jimdo.com/

Page 16: Algebra Superior Murray R Spiegel

8 OPERACIONES FUNDAMENTALES CON LOS NUMEROS

a) Para obtener el denominador 6. multiplicamos el numerador de la fracción 1/3 por 2.l 1 2 2

Tendremos - = -' - = -3 3 2 6

3·5 3'9IS.

3 15 3 27b) -=-=- d)

4 4'5 20 7 7·9 63

5 5·6 30 12 12' 15 180 16.e)8 8'6 48

e)5 -T15= 75

13. Calcular la suma S. diferencia D. producto P y cociente Q de cada uno de los pares de números racionales si-guientes: a) 1/3, 1/6; b) 2/5. 3/4; e) -4/15. -11/24.

a) 1/3 se puede sustituir por la fracción equivalente 2/6.

1 1 2 1 3 lS=3+6=6+6=6='2

1 1 2 1 1D=---=---=-

3 666 6

jg

1 1 1P = (3)(6) = 18

1/3 1 6 6Q=-=-'-=-=2

1/6 3 1 317.

b) 2/5 Y 3/4 se pueden expresar con denominador 20: 2/5 = 8/20. 3/4 = 15/20

2 3 8 15 23S = S + 4 = 20 + 20 = 20

2 3 8 15 7D = S - 4 = 20 - 20 = - 20

2 3 6 3P = (S)(4)= 20 = \O

2/5 2 4 8Q = 3/4 = "5. 3 = 15

18.

e) -4/15 Y -11/24 tienen por mínimo común denominador 120: -4/15 = -32/120. -11/24 = -55/120.

41111P = (- 15)( - 2"4) = 90

19.

2940

20.

87120

4 11 32 55 23D = (- 15) - (- 2"4) = - 120 + 120 = 120

-4/15 4 24 32Q = -=11/24 = 1-15)( -il) = 55

14. Calcular las siguientes expresiones. siendo .r = 2. )' = - 3. z = 5. (J = 1/2. b = - 2/3. 21.a

a) 2x+y=2(2)+(-3)=4-3= 1

b) 3x - 2y - 4z = 3(2) - 2(-3) - 4(5) = 6 + 6 - 20 = -8

e) 4x2y=4(2)2(-3)=4'4'(-3)= -48

x' + 4y 2' + 4( - 3) 8 - 12 4d) 2a _ 3b = 2(1/2.) - 3(-2/3) = 1+2' = 3

e) (::)2 _ 3(~)' = (2.)2 _ 3(-2/3)3 = (_ ~)2 _ 3(- ~)3 = ~ _ 3(- ~~) = ~ + ~4 = ~y a - 3 1/2 3 3 9 27 9 9 9

22.

a

b

23. C

d

24. 1

http://carlos2524.jimdo.com/

Page 17: Algebra Superior Murray R Spiegel

es si-

OPERACIONES FUNDAMENTALES CON LOS NUMEROS 9

PROBLEMAS PROPUESTOS15. Hallar la suma S, diferencia D, producto P y cociente Q de cada uno de los siguientes pares de números:

a) 54, 18; b) 4, O; e) 0, 4; d) 12, 24; e) 50, 75.

16. Efectuar las operaciones indicadas siguientes:

a) 38 + 57, 57 + 38b) 15 + (33 + 8), (15 + 33) + 8e) (23 + 64) - (41 + 12)d) 12· 8, 8· 12e) 6(4' 8), (6' 4)8f) 42·68g) 1296 -;- 36

(35 - 23)(28 + 17)h) 43 _ 25

i) 45 -;- 15 + 84 -i- 12j) 10 -i- 5 - 4 -i- 2 + 15 -r- 3 + 2· 5k) 112 -i- (4' 7), (112 -i- 4)· 7

15 + 3' 21) 9 _ 4 -i- 2

1120.

17. Colocar adecuadamente el signo de desigualdad « o » entre cada uno de los siguientes pares de números reales:

a) 4, 3

b) -2, ° e) -1,2

d) 3, -2e) -8,-7

f) 1, J2g) -3,-JIIh) -1/3, -2/5

18. Ordenar de menor a mayor los números de los grupos de números reales siguientes:

a) -)3, -2, .j6, -2,8, 4, 7/2 b) 2n, -6, )8, -3n, 4,8, 19/3

19. Escribir el valor absoluto de los siguientes números reales: 2, -3/2, -.j6, +3,14,0,5/3, j4, -0,001,-n - 1.

3255

20. Calcular:

a) 6 + 5b) (-4) + (-6)

e) (-4) + 3d) 6 + (-4)

e) -8 + 4f) -4 + 8

g) (-18) + (-3) + 22h) 40 - 12 + 4

i) -12 - (-8)j) -(-16) - (-12) + (-5) - 15

21. Hallar la suma S, diferencia D, producto P y cociente Q de cada uno de los siguientes pares de números reales:a) 12,4; b) -6, -3; e) -8,4; d) 0, -4; e) 3, -2.

22. Efectuar las operaciones indicadas siguientes:

a) (-3)(2)(-6)

b) (6)(-8)(-2)

23. Calcular:

a) 33

b) 3(4)2

e) 24. 23

d) 42 . 32

56. 53e) -5-'-

e) 4(-1)(5) + (-3)(2)(-4)

d) (-4)(6) (-16)(-9)--=-3 + 12

e) (-8) -;- (-4) + (-3)(2)

(-3)(8)(-2)f) (-4)(-6) - (2)(-12)

f)34. 38

j)(_2)3'(2)3

36. 3' 3(22)2

7'k)

3(-3)2 + 4(-2)3g)

73. 74 23 _ 32

h) (3')3 57 2101) - + - 4(-3)4

i) <!l6 . 25 54 82'(_2)3

24. Transformar las fracciones siguientes en otras equivalentes cuyo denominador sea el indicado en cada caso.

a) 2/5; 15b) -4/7; 28

e) 5/16; 64d) -10/3; 42

e) 11/12; 132f) 17/18; 90

http://carlos2524.jimdo.com/

Page 18: Algebra Superior Murray R Spiegel

10 OPERACIONES FUNDAMENTALES CON LOS NUMEROS

25. Hallar la suma S, diferencia D, producto P y cociente Q de cada uno de los pares de números racionales siguien-tes: a) 1/4,3/8; b) 1/3, 2/5; e) -4,2/3; d) -2/3, -3/2.

26. Calcular las expresiones siguientes, siendo x = - 2. Y = 4, Z = 1/3, a = - 1, b = 1/2:

a) 3.• - 2y + 6z

b) 2xr + Sa: d)3)'2 - 4x

e)x2y(x + y) f) (~)3 _ 4(~)2 _ xy

ax + by 3x + 4y x b Z2

e) 4b2x3

o]SOLUCIONES DE LOS PROBLEMAS PROPUESTOS

15. a) S = 72, D = 36, P = 972, Q = 3b) S = 4, D = 4, P = O, Q no está definidoe) S = 4, D = -4, P = O, Q = O

d) S = 36, D = -12, P = 288, Q = 1/2e) S = 125, D = -25, P = 3750, Q = 2/3

16. a) 95, 95b) 56-; 56

e) 34d) 96, 96

e) 192, 192f) 2856

g) 36h) 30

i) 10j) 15

k) 4, 1961) 3

EXPIn

17. a) 3 < 4 o 4 > 3b) -2 < O o 0>-2e) - 1 < 2 o 2 > -1

d) -2 < 3 o 3 > -2e) -8 < -7 o -7>-8f) 1<.J2 o .J2>1

g) -fo < -3 o -3>-foh) -2/5 < -1/3 o -1/3 > -2/5

TERII

18. a) -2,8 < -2 < -.j3 < )6 < 7/2 < 4

19. 2, 3/2, )6, 3,14, O, 5/3, )4, 0,001, n + 1

b) -3n < -6 < J8 < 4,8 < 2n < 19/3

20, a) 11b) - \O

e) -1d) 2

e) -4f) 4

g) 1h) 32

i) -4j) 8

d) S = -4, D = 4, P = O, Q =_0el S = 1, D = 5, P = -6, Q = -3/2

BINC21. a) S = 16, D = 8, P = 48, Q = 3

b) S = -9, D = -3, P = 18, Q = 2e) S= -4, D= -12, P= -32, Q=-2 TRIJI

22. a) 36 b) 96 e) 4 d) 20 e) -4 f)

23. a) 27b) 48

e) 128d) 144

e) 54 = 625f) 3

k) 5J) -201

MUlg) 1/49h) 36 = 729

i) 1/2j) -4/3

24. a) 6/15 b) -16/28 e) 20/64 d) -140/42 e) 121/132 f) 85/90COEI

It

COEl'25. a) S = 5/8, D = - 1/8, P = 3/32, Q = 2/3

b) S = 11/15, D = -1/15, P = 2/15, Q = 5/6e) S = -10/3, D = -14/3, P = -8/3, Q = -6d) S = - 13/6, D = 5/6, P = 1, Q = 4/9

26. a) -12 b) -18 e) -8 d) 14 e) 16/5 f) 48 TER

UN"

http://carlos2524.jimdo.com/

Page 19: Algebra Superior Murray R Spiegel

CAPITULO 2

Operaciones fundamentales con expresiones algebraicas

EXPRESION ALGEBRAICA. Es una combinación de números y de letras que representannúmeros cualesquiera.

Por ejemplo, 3x2 - 5xy + 2y4, son expresiones algebraicas.

TERMINO. Es una expresión que solo contiene productos y cocientes de números y de letras. Así,pues, 6x2 y3, 5x/3 y4, - 3x 7, son términos de una expresión algebraica.

Sin embargo, 6x2 + 7xy es una expresión algebraica que consta de dos términos.

MONOMIO. Es una expresión algebraica de un solo término.

Así, pues, 7X3y4, 3xyz2, 4x2/y son monomios.

A causa de esta definición, los monomios se denominan con frecuencia términos simplemente.

BINOMIO. Es una expresión algebraica de dos términos.

Por ejemplo, 2x + 4y, 3x4 - 4xyz3 son binomios.

TRINOMIO. Es una expresión algebraica de tres términos.

Por ejemplo, 3x2 - 5x + 2, 2x + 6y - 3z, x3 - 3xy/z - 2X3Z7 son trinomios.MULTINOMIO. Es una expresión algebraica de más de un término.

Por ejemplo, 7x + 6y, 3x3 + 6x2y - 7xy + 6, 7x + 5x2/y - 3x3/16 son multinomios.

COEFICIENTE. Cualquier factor de un término se llama coeficiente del resto de dicho término. Así,pues, en el término 5X3y2, 5x3 es el coeficiente de y2, 5y2 es el coeficiente de x3 y 5 es el coeficien-te de X3y2.

COEFICIENTE NUMERICO. Si un término es el producto de un número por una o varias letras,dicho número es el coeficiente numérico (o simplemente coeficiente) del término.

Por ejemplo, en el término - 5X3y2, el coeficiente numérico o coeficiente es - 5.

TERMINOS SEMEJANTES. Son aquellos que solo se diferencian en su coeficiente numérico.

Por ejemplo, 7xy y -2xy son términos semejantes; 3x2y4 y _iX2y4 son asimismo términossemejantes; sin embargo, - 2a2b3 y - 3a2b 7 no son semejantes.

Se pueden reducir dos o más términos semejantes a uno solo. Por ejemplo 7x2y - 4x2y + 2x2yse pueden reducir a 5x2y.

UN TERMINO ES ENTERO Y RACIONAL con respecto a ciertas letras (que representan a núme-ros cualesquiera), si está formado por:

11

CAPITULO 2

Operaciones fundamentales con expresiones algebraicas

EXPRESION ALGEBRAICA. Es una combinación de números y de letras que representan números cualesquiera.

Por ejemplo, 3X2 - 5xy + 2y 4, son expresiones algebraicas.

TERMINO. Es una expresión que solo contiene productos y cocientes de números y de letras. Así, pues, 6X2 y3, 5x/3 y4, - 3x 7 , son términos de una expresión algebraica.

Sin embargo, 6X2 + 7xy es una expresión algebraica que consta de dos términos.

MONOMIO. Es una expresión algebraica de un solo término.

Así, pues, 7X3y 4, 3xyz2, 4x2/y son monomios.

A causa de esta definición, los monomios se denominan con frecuencia términos simplemente.

BINOMIO. Es una expresión algebraica de dos términos.

Por ejemplo, 2x + 4y, 3x4 - 4xyz3 son binomios.

TRINOMIO. Es una expresión algebraica de tres términos.

Por ejemplo, 3X2 - 5x + 2, 2x + 6y - 3z, x 3 - 3xy/z - 2X3Z

7 son trinomios.

MULTINOMIO. Es una expresión algebraica de más de un término. Por ejemplo, 7x + 6y, 3x3 + 6x2y - 7xy + 6, 7x + 5x2/y - 3x3/16 son multinomios.

COEFICIENTE. Cualquier factor de un término se llama coeficiente del resto de dicho término. Así, pues, en el término 5x 3y 2, 5x3 es el coeficiente de y2, 5y2 es el coeficiente de x 3 y 5 es el coeficien­te de X3y 2.

COEFICIENTE NUMERICO. Si un término es el producto de un número por una o varias letras, dicho número es el coeficiente numérico (o simplemente coeficiente) del término.

Por ejemplo, en el término - 5x 3y 2, el coeficiente numérico o coeficiente es - 5.

TERMINOS SEMEJANTES. Son aquellos que solo se diferencian en su coeficiente numérico.

Por ejemplo, 7xy y -2xy son términos semejantes; 3x2y4 y _iX2y 4 son asimismo términos semejantes; sin embargo, - 2a2b3 y - 3a2b 7 no son semejantes.

Se pueden reducir dos o más términos semejantes a uno solo. Por ejemplo 7x2y - 4x2y + 2x2y se pueden reducir a 5x2y.

UN TERMINO ES ENTERO Y RACIONAL con respecto a ciertas letras (que representan a núme­ros cualesquiera), si está formado por:

11

http://carlos2524.jimdo.com/

Page 20: Algebra Superior Murray R Spiegel

12 OPERACIONES FUNDAMENTALES CON EXPRESIONES ALGEBRAICAS

a) Potencias enteras y positivas de letras multiplicadas por un factor numérico.b) Un número.

Por ejemplo, los términos 6X2y3, _5y4, 7, -4x, .J3 X3y6, son enteros y racionales con res-pecto a las letras que figuran en ellos. Sin embargo, 3); no es racional con respecto ax y 4/x no es entero con respecto a x.

POLINOMIO. Es un monomio, o un multinomio, en el que cada término es entero y racional conrespecto a las letras.

Por ejemplo, 3X2y3 - 5x4y + 2, 2X4 - 7x3 + 3x2 - 5x + 2, 4xy + Z, 3X2, son polino-

mios. Sin embargo, 3x2 - 4/x, 4.JY + 3, no son polinomios.

RE

GRADO DE UN MONOMIO. Es la suma de todos los exponentes de la parte literal del término.Por ejemplo, el grado de 4x3y2z es 3 + 2 + 1 = 6. El grado de una constante, como porejemplo, 6, O, -.J3, 71:, es cero.

Ml

GRADO DE UN POLINOMIO. Es el correspondiente al término de mayor grado cuyo coe-ficiente sea distinto de cero.

Los grados de los términos del polinomio 7x3y2 - 4xz5 + 2x3y son 5, 6 y 4, respectivamente;por consiguiente, el grado del polinomio es 6.

SIMBOLOS DE AGRUPAMIENTO. Son los paréntesis ( ), los corchetes [ ] o las llaves { };se emplean para indicar que los términos encerrados en ellos se consideran como una sola cantidad.

Por ejemplo, la suma de las dos expresiones algebraicas, 5x2 - 3x + y y 2x - 3y, se puederepresentar por (5x2 - 3x + y) + (2x - 3y), su diferencia por (5x2 - 3x + y) - (2x - 3y), y suproducto por (5x2 - 3x + y)(2x - 3y).

Algunas veces se emplea como símbolo de agrupamiento una barra encima de los términosa asociar. Por ejemplo, 5x - 3y es lo mismo que escribir (5x - 3y).

SUPRESION DE LOS SIMBOLOS DE AGRUPAMIENTO. Está regida por las normas siguientes:

1) Si un signo + precede al símbolo de agrupamiento, dicho símbolo se puede suprimir sin modi-ficar los términos que contiene.

Por ejemplo, (3x + 7y) + (4xy - 3x3) = 3x + 7y + 4 xy - 3x3.

2) Si un signo - precede al símbolo de agrupamiento, dicho símbolo se puede suprimir cambian-do el signo de cada uno de los términos que contiene.

Por ejemplo, (3x + 7y) - (4xy - 3x3) = 3x +.7y - 4xy + 3x3.

3) Si en una expresión figura más de un símbolo de agrupamiento, para suprimirlos se comien-za por los interiores.

Por ejemplo, 2x - {4x3 - (3x2 - 5y)} = 2x - {4x3 - 3x2 + 5y} = 2x - 4x3 + 3x2 - 5y.

SJ)MA DE EXPRESIONES ALGEBRAICAS. Se efectúa agrupando los términos semejantes. Parallevar a cabo la suma se pueden disponer las expresiones en filas, con los términos semejantes enla misma columna, y, a continuación, se suman los términos de cada columna.

Ejemplo. Sumar 7x + 3y3 - 4xy, 3x - 2y3 + 7xy Y 2xy - 5x - 6y3.

Disponemos el cálculo así: 7x3x

-5x5xy. El resultado es 5x - 5y3 + 5xy

-4xy7xy2xy

Suma: 5x

12 OPERACIONES FUNDAMENTALES CON EXPRESIONES ALGEBRAICAS

a) Potencias enteras y positivas de letras multiplicadas por un factor numérico. b) Un número.

Por ejemplo, los términos 6X2y 3, _5y4, 7, -4x, .)3 X3 },6, son enteros y racionales con res­pecto a las letras que figuran en ellos. Sin embargo, 3J; no es racional con respecto a x y 4/x no es entero con respecto a x.

POLINOMIO. Es un monomio, o un multinomio, en el que cada término es entero y racional con respecto a las letras.

Por ejemplo, 3X2y 3 - 5x4y + 2, 2X4 - 7x3 + 3X2 - 5x + 2, 4xy + Z , 3X2, son polino-

mios. Sin embargo, 3X2 - 4/x, 4JY + 3, no son polinomios.

GRADO DE UN MONOMIO. Es la suma de todos los exponentes de la parte literal del término. Por ejemplo, el grado de 4x3y2z es 3 + 2 + 1 = 6. El grado de una constante, como por ejemplo, 6, O, - .)3, 71:, es cero.

GRADO DE UN POLINOMIO. Es el correspondiente al término de mayor grado cuyo coe­ficiente sea distinto de cero.

Los grados de los términos del polinomio 7x 3y2 - 4xz5 + 2x3y son 5, 6 y 4, respectivamente; por consiguiente, el grado del polinomio es 6.

SIMBOLOS DE AGRUPAMIENTO. Son los paréntesis ( ), los corchetes [ ] o las llaves { }; se emplean para indicar que los términos encerrados en ellos se consideran como una sola cantidad.

Por ejemplo, la suma de las dos expresiones algebraicas, 5X2 - 3x + y Y 2x - 3y, se puede representar por (5x2 - 3x + y) + (2x - 3y), su diferencia por (5x2 - 3x + y) - (2x - 3y), y su producto por (5x2 - 3x + y)(2x - 3y).

Algunas veces se emplea como símbolo de agrupamiento una barra encima de los términos a asociar. Por ejemplo, 5x - 3y es lo mismo que escribir (5x - 3y).

SUPRESION DE LOS SIMBOLOS DE AGRUPAMIENTO. Está regida por las normas siguientes:

1) Si un signo + precede al símbolo de agrupamiento, dicho símbolo se puede suprimir sin modi­ficar los términos que contiene.

~or ejemplo, (3x + 7y) + (4xy - 3x3) = 3x + 7y + 4 xy - 3x3

2) Si un signo - precede al símbolo de agrupamiento, dicho símbolo se puede suprimir cambian­do el signo de cada uno de los términos que contiene.

Por ejemplo, (3x + 7y) - (4xy - 3x3) = 3x + .7y - 4xy + 3x3

3) Si en una expresión figura más de un símbolo de agrupamiento, para suprimirlos se comien­za por los interiores.

Por ejemplo, 2x - {4x3 - (3x2 - 5y)} = 2x - {4x3 - 3X2 + 5y} = 2x - 4x3 + 3X2 - 5y.

SIi MA DE EXPRESIONES ALGEBRAICAS. Se efectúa agrupando los términos semejantes. Para llevar a cabo la suma se pueden disponer las expresiones en filas, con los términos semejantes en la misma columna, y, a continuación, se suman los términos de cada columna.

Ejemplo. Sumar 7x + 3y 3 - 4xy, 3x - 2y 3 + 7xy Y 2xy - 5x - 6y 3.

Disponemos el cálculo así: 7x 3y 3 -4xy 3x _2y 3 7xy

-5x _6y 3 2xy Suma: 5x _5y 3 5xy. El resultado es 5x - 5y 3 + 5xy

http://carlos2524.jimdo.com/

Page 21: Algebra Superior Murray R Spiegel

con

OPERACIONES FUNDAMENTALES CON EXPRESIONES ALGEBRAICAS 13

res-o a

RESTA DE DOS EXPRESIONES ALGEBRAICAS. Se lleva a cabo efectuando la suma de la ex-presión rninuendo con la opuesta del sustraendo, la cual se obtiene cambiando el signo de todossus términos.

Ejemplo. Restar 2x2 - 3xy + 5y2 de 10x2 - 2xy - 3y2.

Resta:

10x2 - 2xy - 3/2X2 - 3xy + 5.1'28x2 + xy - 8.1'2

lino-También se puede hacer así: (lOx2 - 2xy - 3y2) - (2x2 - 3xy + 5y2)

= IOx2 - 2xy - 3y2 - 2X2 + 3xy - 5y2 = 8x2 + xy _ 8y2.

ino.por

MUL T1PLICACION DE EXPRESIONES ALGEBRAICAS

coe-

1) Multiplicación de dos o más monomios. Se efectúa aplicando las reglas de la potencia-ción y de los signos y las propiedades asociativa y conmutativa del producto.

Ejemplo. Multiplicar -3X2y3z, 2x4y y _4xy4Z2.

uedey su

Escribimos (- 3X2y3Z)(2x4y)( - 4xy4Z2).

Aplicando las propiedades conmutativa y asociativa, tendremos,

{( - 3 )(2)( - 4)}{ (x2 )(x4)(x)}{ (y3 )(y)(y4)) {(Z)(Z2)}

De acuerdo con las reglas de los signos y exponentes se deduce

24x 7y8z3

La operación (1) se puede realizar mentalmente cuando se haya adquirido ciertasoltura.

(1)

nte;

{ };idad.

2) Multiplicación de un monomio por un polinomio. Se efectúa multiplicando el monomiopor todos y cada uno de los términos del polinomio, sumando los productos obtenidos.

Ejemplo. Multiplicar 3xy - 4x3 + 2xy2 por 5x2y4.

Escribimos (5x2y4)(3xy - 4x3 + 2xy2)= (5x2y4)(3xy) + (5x2y4)(_4x3) + (5x2y4)(2xy2)= 15x3y5 - 20X5y4 + IOx3y6

3) Multiplicación de dos polinomios. Se efectúa multiplicando todos y cada uno de lostérminos de uno de ellos por todos y cada uno de los términos del otro, sumando los productosobtenidos.

Es conveniente ordenar los polinomios según las potencias crecientes (o decrecientes) de unade las letras.

5y.Ejemplo. Multiplicar - 3x + 9 + x2 por 3 - x.

Paraes en Ordenando según las potencias decrecientes de x,

x2 - 3x + 9x (2)-x + 3

Multiplicando (2) por - x,Multiplicando (2) por 3,

Sumando,

_x3 + 3x2 - 9 X3x2 - 9x + 27

- x3 + 6x2 - 18x + 27

5xy

OPERACIONES FU NDAMENTALES CON EXPRESIONES ALGEBRAICAS 13

RESTA DE DOS EXPRESIONES ALGEBRAICAS. Se lleva a cabo efectuando la suma de la ex­presión minuendo con la opuesta del sustraendo, la cual se obtiene cambiando el signo de todos sus términos.

Ejemplo. Restar 2X2 - 3xy + 5y2 de IOx2 - 2xy - 3y 2.

Resta:

IOx 2 - 2xy - 3y2 2X2 - 3xy + 51'2

8X2 + xy - 8.1'2

También se puede hacer así: (IOx2 - 2xy - 3y2) - (2x2 - 3xy + 5y2)

= IOx2 - 2xy - 3y2 - 2X2 + 3xy - 5y2 = 8X2 + xy _ 8y2.

MUL TIPLICACION DE EXPRESIONES ALGEBRAICAS

1) Multiplicación de dos o más monomios. Se efectúa aplicando las reglas de la potencia­ción y de los signos y las propiedades asociativa y conmutativa del producto.

Ejemplo. Multiplicar -3X2y 3z , 2x4y y _4xy4Z2.

Escribimos (- 3X2y 3Z)(2x4y)( - 4xy4Z2).

Aplicando las propiedades conmutativa y asociativa, tendremos,

{( - 3 )(2)( - 4)}{ (x2 )(x4)(x)}{ (y3 )(y)(y4)) {(Z)(Z2)}

De acuerdo con las reglas de los signos y exponentes se deduce

24x 7y 8z3

(1 )

La operación (1) se puede realizar mentalmente cuando se haya adquirido cierta soltura.

2) Multiplicación de un monomio por un polinomio. Se efectúa multiplicando el monomio por todos y cada uno de los términos del polinomio, sumando los productos obtenidos.

Ejemplo. Multiplicar 3xy - 4x3 + 2xy 2 por 5x2y4.

Escribimos (5x2y4)(3xy - 4x3 + 2xy2) = (5x2y4)(3xy) + (5x2y4)(_4x3) + (5x2y4)(2xy2) = 15x 3y s - 20X5y4 + IOX3y 6

3) Multiplicación de dos polinomios. Se efectúa multiplicando todos y cada uno de los términos de uno de ellos por todos y cada uno de los términos del otro, sumando los productos obtenidos.

Es conveniente ordenar los polinomios según las potencias crecientes (o decrecientes) de una de las letras.

Ejemplo. Multiplicar - 3x + 9 + X2 por 3 - x .

Ordenando según las potencias decrecientes de x,

Multiplicando (2) por - x , Multiplicando (2) por 3,

Sumando,

X2 - 3x + 9x (2) - x + 3 _ x 3 + 3X2 - 9 X

3X2 - 9x + 27 - x 3 + 6X2 - 18x + 27

http://carlos2524.jimdo.com/

Page 22: Algebra Superior Murray R Spiegel

14 OPERACIONES FUNDAMENTALES CON EXPRESIONES ALGEBRAICAS

DIVISION DE EXPRESIONES ALGEBRAICAS.

1) División de dos monomios. Se efectúa hallando el cociente de los coeficientes y el delos factores literales, multiplicando después dichos cocientes.

Ponemos24x4y2z3 24 x4 y2 Z3 1 8xz2

= (-)( )(-)(-) = (-8)(x)( )(Z2) =-_-3=-X"';3'y'4-Z - 3 Xl y4 Z ? -72.

2) División de dos polinomios.

a) Se ordenan los términos de ambos polinomios según las potencias decrecientes (o crecien-tes) de una de las letras comunes a los dos polinomios.

b) Se divide el primer término del dividendo por el primero del divisor, con lo que resulta el pri-mer término del cociente.

e) Se multiplica el primer término del cociente por el divisor y se resta del dividendo, obtenién-dose un nuevo dividendo.

d) Con el dividendo de e), se repiten las operaciones b) y e) hasta que se obtenga un resto igual acero o de grado menor que el del dividendo.

dividendo . restoe) El resultado es: divisor = cociente + divisor'

Ejemplo. Dividir x2 + 2X4 - 3x3 + X - 2 por x2 - 3x + 2.

Se ordenan los polinomios según las potencias decrecientes de x y se dispone elcálculo de la forma siguiente:

2X4 - 3x3 + x2 +2X4 - 6x3 + 4x2

x + 2 x2 - 3x + 22X2 + 3x + 6 3.

3x3 - 3x2 +3x3 - 9x2 +

x - 26x5x - 2

18x + 1213x - 14

2X4 - 3x3 + x2 + X - 2 2 2 3 6 I3x - 14Por tanto, 2 = X + X + + 2 3 2x - 3x + 2 x - x +

4.

PROBLEMAS RESUELTOS1. Hallar el valor de las expresiones algebraicas siguientes, siendo x = 2, Y = -1, Z = 3, a = O, b = 4, e = 1/3.

a) 2X2 - 3yz = 2(2)2 - 3(-1)(3) = 8 + 9 = 17

b) 2z4- 3z3 + 4z2

- 2z + 3 = 2(3)4 - 3(3)3 + 4(3)2 - 2(3) + 3 = 162 - 81 + 36 - 6 + 3 = 114

e) 4a2 - 3ab + 6c = 4(W - 3(0)(4) + 6(1/3) = O - O + 2 = 2

14 OPERACIONES FUNDAMENTALES CON EXPRESIONES ALGEBRAICAS

DIVISION DE EXPRESIONES ALGEBRAICAS.

1) División de dos monomios. Se efectúa hallando el cociente de los coeficientes y el de los factores literales, multiplicando después dichos cocientes.

Ejemplo.

Ponemos 24x4y2z 3 24 x4 y2 Z3 1 8xz2

= (-)( )(-)(-) = (-8)(x)( )(Z2) = -_-3=-X--'3'-y-'4-Z - 3 ? y4 Z ? - 7

2) División de dos polinomios.

a) Se ordenan los términos de ambos polinomios según las potencias decrecientes (o crecien­tes) de una de las letras comunes a los dos polinomios.

b) Se divide el primer término del dividendo por el primero del divisor, con lo que resulta el pri­mer término del cociente.

e) Se multiplica el primer término del cociente por el divisor y se resta del dividendo, obtenién­dose un nuevo dividendo.

d) Con el dividendo de e), se repiten las operaciones b) y e) hasta que se obtenga un resto igual a cero o de grado menor que el del dividendo.

dividendo . resto e) El resultado es: divisor = cocIente + divisor'

Ejemplo. Dividir X2 + 2X4 - 3x3 + X - 2 por X2 - 3x + 2.

Se ordenan los polinomios según las potencias decrecientes de x y se dispone el cálculo de la forma siguiente :

2X4 - 3x3 + X2 + X + 2 X2 - 3x + 2 2X4 - 6x3 + 4X2 2X2 + 3x + 6

3x3 - 3X2 + X - 2 3x3 - 9X2 + 6x

6X2 - 5x - 2 6X2 - 18x + 12

13x - 14

2X4 - 3x3 + X2 + X - 2 2 I3x - 14 Por tanto, 2 3 2 = 2x + 3x + 6 + 2 3 2 x - x+ x-x +

PROBLEMAS RESUELTOS

1. Hallar el valor de las expresiones algebraicas siguientes, siendo x = 2, Y = -1, Z = 3, a = O, b = 4, e = 1/3.

a) 2X2 - 3yz = 2(2)2 - 3(-1)(3) = 8 + 9 = 17

b) 2z4 - 3z3 + 422 - 2z + 3 = 2(3)4 - 3(3)3 + 4(3)2 - 2(3) + 3 = 162 - 81 + 36 - 6 + 3 = 114

e) 4a2 - 3ab + 6c = 4(W - 3(0)(4) + 6(1 /3) = O - O + 2 = 2

http://carlos2524.jimdo.com/

Page 23: Algebra Superior Murray R Spiegel

OPERACIONES FUNDAMENTALES CON EXPRESIONES ALGEBRAICAS

d) 5xy + 3z = 5(2)(-1) + 3(3) = -lO + 9 = --=-'- = 9203 - e2 2(o¡J - (1/3)2 -1/9 -1/9

e) 3x2y _ ~ = 3(2)2 (-1) _ 4(1/3) = -4 _ 4/9 = -40/9

z x+1 3 3

4x2y(z - 1) 4(2)2 (-1)(3 - 1) 4(4)(-1)(2)f) a + b - 3e O + 4 - 3(1/3) 4 - 1

323

2. Clasificar las expresiones algebraicas siguientes según las categorías: término o monomio, binomio, trinomio,multinomio, polinomio.

a) x3 + 3y2z

b) 2X2 - 5x + 3

e) 4x2y/z

d) Y + 3

e) 4z2 + 3z - 2Jz

f) 5x3 + 4/y

g) Jx2 + y2 + Z2

h) Jy + Jz

i) a3 + b3 + e3 - 3abe

Señálese con una indicación la(s) categoria(s) a que pertenece cada expresión.

Término omonomio Binomio Trinomio Multinomio Polinomio

%~ + 3/z .¡ .¡ .¡

2%2 - 5% + 3 .¡ .¡ .¡

4%2y/z .¡

y + 3 .¡ .¡ .¡

4z2 + 3z - 2vz v' f

5%~ + 4/y .¡ .¡

1%2 + y2 + z2 .¡

vY+vZ .¡ .¡

a~ + b~ + c~ - 3abc .¡ .¡

3. Hallar el grado de los siguientes polinomios:

a) 2x3y + 4xyz4. El grado de 2x3y es 4, y el de 4xyz4 es 6; luego el polinomio es de grado 6.

b) x2 + 3x3 - 4. El grado de x2 es 2, el de 3x3 es 3, y el de -4 es O; luego el grado del polinomio es 3.

e) y3 - 3y2 + 4y - 2 es de grado 3.

d) xz3 + 3X2Z2 - 4x3z + x4. Cada término es de grado 4; luego el polinomio es de grado 4.

e) x2 - lOs es de grado 2. (El grado de la constante lOs es cero.)

4. Suprimir los símbolos de aproximadamente en cada una de las expresiones siguientes y simplificar los resultadosreduciendo los términos semejantes:

a) 3x2 + (y2 _ 4z) ....: (2x - 3y + 4z) = 3x2 + y2 - 4z - 2x + 3y - 4z = 3x2 + y2 - 2x + 3y - 8z

b) 2(4xy + 3z) + 3(x - 2xy) - 4(z - 2xy) = 8xy + 6z + 3x - 6xy - 4z + 8xy = 10xy + 3x + 2z

e) x - 3 - 2{2 - 3(x - y)} = x - 3 - 2{2 - 3x + 3y} = x - 3 - 4 + 6x - 6y = 7x - 6y - 7

d) 4x2 - {3x2 - 2[y - 3(x2 - y)] + 4} = 4x2 - {3x2 - 2[y - 3x2 + 3y] + 4}

= 4x2 - {3x2 - 2y + 6x2 - 6y + 4} = 4x2 - {9x2 - 8y + 4}

= 4x2 - 9x2 + 8y - 4 = -5x2 + 8y - 4

15

http://carlos2524.jimdo.com/

Page 24: Algebra Superior Murray R Spiegel

16 OPERACIONES FUNDAMENTALES CON EXPRESIONES ALGEBRAICAS

5. Sumar las expresiones algebraicas de cada uno de los grupos siguientes: e)

a) x2 + y2 _ Z2 + 2xy _ 2yz, y2 + Z2 _ x2 + 2yz _ 2zx, Z2 + x2 _ y2 + 2zx - 2xy. I _ x2 _ y2 _ ;:2

Ordenando, x2 + y2 _ Z2 + 2xy - 2yz

_ x2 + y2 + Z2 + 2yz - 2zx

x2 _ y2 + Z2 - 2xy + 2;:x

_x2 _ y2 _ ;:2 + I

0+0+0+0+0+0+1Sumando, El resultado de la suma es 1.

g)

Ordenando, 5x3y - 4ab + e2

-3x2y + 2ab + 3e2

x2y + X3y - 3ab _ 4e2

- 2x2y - 3ab + 4e2 + ab2

-4x2y + 6x3y - 8ab + 4e2 + ab2Sumando,

6. Restar la segunda expresión de la primera en los siguientes casos: 8. El

a) a - b + e - d, e - a + d - b.Ponemos a-b+e- d

-a - b + e + d2a+0+0-2d El resultado es 2a - 2d.

a)

Restando,

De otra forma: (a - b + e - d) - (e - a + d - b) = a - b + e - d - e + a - d + b = 2a - 2d. b)

b) 4.\"2y - 3ab + 2a2 - xy, 4xy + ab" - 3a2 + 2ab.e)

Escribimos 4x2y - 3ab + 2a2 - xy

2ab - 3a2 + 4xy + ab2

Restando, 4x2)' - 5ab + 5a2 - 5.\")' - ab"d)

De otra forma. (4x2)' - 3ab + 2a2 - xy) - (4xy + ab2 - 3a2 + 2ab)

= 4x2y _ 3ab + 2a2 - xy - 4xy - ab" + 3a2 - 2ab

= 4x2y - 5ab + 5a2 - 5xy - ab2

e)

7. Efectuar los productos indicados en los casos siguientes:

a) ( -2ab3)(4a2b') el (x2 - 3x + 9)(x + 3)

b) (-3x2y)(4x/)( _2X3y4) f) (X4 + X3y + X2)'2 + xy3 + y4)(X _ y)

e) (3ab2 )(2ab + b2) g) (x2 _ xy + y2)(X2 + xy + y2)

d) (x2 - 3xy + y2)(4x/) h) (2x + y - z)(3x - z + )')

a) (-2ah3)(4a2b') = {(-2)(4)}{(a)(a2)1{(b')(b')} = -8a'bB

b) (_3x2y)(4xy2)(_2x'y4) = {(-3)(4)(-2)}{(X2)(X)(X')}{lJ'llv2)lv4)} = 24x6y7

e) (3ab2)(2ah + b2) = (3ab2)(2ab) + (3ab2)(h2) = 6a2b' + 3ah·

el) (x2 - 3xy +.y2)(4xy2) = (x2)(4xy2)+ (-3xy)(4xy2) + (y2)(4xy2)

= 4X'.1'2 - 12x2y' + 4xy·

http://carlos2524.jimdo.com/

Page 25: Algebra Superior Murray R Spiegel

OPERACIONES FUNDAMENTALES CON EXPRESIONES ALGEBRAICAS 17

e) x2 - 3x + 9

x+3

f) x4 + X3y + x2y2 + xy3 + y4

x-yx3 - 3x2 + 9x

3x2 - 9x + 27

x! + x4y + X3y2 + X2y3 + xy4

_ x·y - X3y2 _ X2y3 _ xy· _ y!

x! + O + O + O + O - y!

Sol. x! - y!

x3 + O + O + 27

Sol. x3 + 27

g) x2 _ xy + y2

x2 + xy + y2x4 _ X3y + x2y2

X3y _ x2y2 + xy3

x2y2 _ xy3 + y4

x4 + O + x2y2 + O + y4

Sol. x4 + x2y2 + y4

h) 2x + y - z

3x + y - Z6x2 + 3xy - 3xz

2xy + y2 - yz

- 2xz - yz + Z2

6x2 + 5xy - 5xz + y2 - 2yz + Z2

8. Efectuar las divisiones.

24x3y2z 24 x3 y2 z 1 6x2ya) --2 = (-)(-)(-)(2) = (6)(x2)(y)(_) = -

4xyz 4 x y z z z

-16a4b6 -16 a4 b6 1 2a3b4b) -8ab2e = ( -8 )(~)(b2)(~) = -e-

3x3y + 16xy2 - 12x4yz4 3x3y 16xy2 -12x4yz4 3x 8ye) = (--)+ (--) + ( ) = _ + __ 6X2Z3

2x2yz 2x2yz 2x2yz 2x2yz 2z xz

4a3b2 + 16ab - 4a2 4a3b2 16ab _4a2 8 2d) -2a2b = (-2a2b) + (-2a2b) + (-2a2b) = -2ab - ~ + b

2X4 + 3x3 _ x2 _ 1e) x _ 2

16y4 _ 1f) 2y-=-t

2X4 + 3x3 _ x2

2x4 _ 4x3

- 1 x-2 -1 2y-1

8y3 + 4y2 + 2y + 12X3 + 7x2 + I3x + 26

-1

4y2

4y2 - 2y

2y - 1

2y- 1

O

-1

26x -

26x - 52

51

2X4 + 3x3 - x2 - 1 51 16y4 - 1Por tanto, = 2X3 + 7x2 + I3x + 26 + --2 Y --- = 8y3 + 4y2 + 2v .•. 1x - 2 x - 2y - 1 .

OPERACIONES FUNDAMENTALES CON EXPRESIONES ALGEBRAICAS

e) X2 - 3x + 9

x + 3 Xl - 3X2 + 9x

f) x4 + xly + x2y2 + xyl + y4

x-y

x~ + x·y + xly2 + x2yl + xy4

17

3X2 - 9x + 27

Xl + O + O + 27

Sol. Xl + 27

_ x4y _ xly2 _ x2yl _ xy4 _ y~

x~ + O + O + O + O - y~

Sol. x~ - y~

g) X2 _ xy + y2

X2 + xy + y2

x4 _ xly + x2y2

xly _ x2y2 + xyl

h) 2x + y - z

3x + y - Z

6X2 + 3xy - 3xz

2xy + y2 _ yz

x2y2 _ xyl + y4

x4 + O + x2y2 + O + y4

- 2xz - yz + Z2

6X2 + 5xy - 5xz + y2 - 2yz + Z2

Sol. x4 + x2y2 + y4

8. Efectuar las divisiones.

24xly2z 24 Xl y2 z 2 l 6x2y a) 4xyz2 = (4)(~)(y)(zZ> = (6)(x )(y)(~) = -z-

_16a4b6 - 16 a4 b6 1 2al b4

b) -8ab2e = (-=S)(~)(b2)(-;;-) = -e-

3xly + 16xy2 - 12x4yz4 3xly 16xy2 -12x4yz4 3x 8y e) = (--) + (--) + ( ) = _ + __ 6X2Zl

2x2yz 2x2yz 2x2yz 2x2yz 2z XZ

4alb2 + 16ab - 4a2 4alb2 16ab _ 4a2 8 2 d) -2a2b = (-2a2b) + (-2a2b) + ( - 2a2b) = -2ab -; + b

2X4 + 3Xl - X2 - 1 e) X _ 2

16y 4 - 1 f)--

2y - 1

2X 4 + 3Xl _ X2

2X4 _ 4Xl

- I x - 2 -1 2y-1

- 1

26x -

26x - 52

51

2Xl + 7 X2 + 13x + 26

-1

-1

4y2 - 2y

2y- 1

2y- 1

O

8yl + 4y2 + 2y + 1

2X 4 + 3Xl - X2 - 1 51 16y 4 - 1 Por tanto, = 2Xl + 7X2 + 13x + 26 + --2 Y --- = 8yl + 4y2 + 2y + 1

X - 2 x - 2y - 1

http://carlos2524.jimdo.com/

Page 26: Algebra Superior Murray R Spiegel

18 OPERACIONES FUNDAMENTALES CON EXPRESIONES ALGEBRAICAS

g)2X6 + 5x4 _ Xl +

_x2 + X +2X6

10. (

Ordenando según potencias decrecientes de x.

+ 1 _x2 + X + 1

-2x4 - 2X3 - 9x2 - 10x - 19

2x' + 7x4- Xl

2x' - 2X4 - 2X3

+ 1

9x4 + x3

9x4 - 9x3 - 9x2+ 1

11.10xJ - IOx2 - 10x

19x2 + IOx +19x2 - 19x - 19

29x + 20 12.

2X6 + 5x4 _ x3 +De donde _x2 + x + 1

29x + 20-2x4 - 2xJ - 9x2 - 10x - 19 + -~---_x2 + X +

13.

Ordenando según las potencias decrecientes de una letra, porejemplo x.

14. I

2x2y _ 2xy2 + 2yJ

2x2y _ 2xy2 + 2y3

o

15.

16.

9. Comprobar los Problemas 7h) y 8g) con los valores X = 1, Y = -1, z = 2.

Del Problema 7h), (2x + y - z)(3x - z + y) = 6x2 + 5xy - 5xz - 2yz + Z2 + y2.

Sustituyendo x = 1, .v = - 1, z = 2 obtenemos

[2(1) + (-1) - 2][3(1) - (2) - 1] = 611)2 + 5(1)(-1) - 5(1)(2) - 2(-1)(2) + (2)2 + (-If

o sea [ -1][0] = 6 - 5 - 10 + 4 + 4 + 1, es decir, 0= O.17.

Del Problema 8g).2X6 + 5x4 _ xJ +

_x2 + X + 1

29x + 20_2x4 - 2xJ - 9x2 - IOx - 19 + --=----

_x2 + X +2 + 5 - 1 + 1 29 + 20

Haciendo x = 1 obtenemos = -2 - 2 - 9 - 10 - 19 + o sea 7 = 7.-1+1+1 -1+1+1

La comprobación de una operación sustituyendo números por letras no es definitiva; se puede utilizar paraindicar posibles errores.

18

g)

OPERACIONES FUNDAMENTALES CON EXPRESIONES ALGEBRAICAS

2X 6 + 5x4 _ X 3 + _X2 + X +

2X 6

Ordenando según potencias decrecientes de x .

+ 1 _x2 -+- X + 1

-2x4 - 2X3 - 9X2 - 10x - 19

2x' + 7x4 - x 3

2x' - 2X 4 - 2X3

+ 1

9x4 + x 3 + 1

9x4 - 9x3 - 9X2

IOx3 + 9X2 +

IOx3 - IOx2 - 10x

19x2 + IOx + 19x2 - 19x - 19

29x + 20

29x + 20 -2x4

- 2x3 - 9X2 - 10x - 19 + --:---~ _x2 + X +

Ordenando según las potencias decrecientes de una letra. por ejemplo x.

2x2y _ 2xy2 + 2y 3

2x2y _ 2xy2 + 2y 3

o

9. Comprobar los Problemas 7h) y Sg) con los valores x = 1, Y = -1, Z = 2.

Del Problema 7h). (2x + y - z)(3x - z + y) = 6X2 + 5xy - 5xz - 2yz + Z2 + y2.

Sustituyendo x = 1, Y = - l . z = 2 obtenemos

[2(1) + (-1) - 2][3(1) - (2) - 1] = 6(1)2 + 5(1)(-1) - 5(1)(2) - 2(-1)(2) + (2)2 + (_1)2

o sea [ -1][0] = 6 - 5 - 10 + 4 + 4 + 1, es decir, 0= O.

Del Problema Sg). 2X6 + 5x4 _ x 3 +

_x2 + X + 1 29x + 20

- 2X 4 - 2x3 - 9X2 - IOx - 19 + ---,-----, _ x 2 + X +

2 + 5 - 1 + 1 29 + 20 Haciendo x = 1 obtenemos = -2 - 2 - 9 - 10 - 19 + o sea 7 = 7.

-1+1+1 -1+1+1

La comprobación de una operación sustituyendo números por letras no es definitiva ; se puede utilizar para indicar posibles errores.

http://carlos2524.jimdo.com/

Page 27: Algebra Superior Murray R Spiegel

OPERACIONES FUNDAMENTALES CON EXPRESIONES ALGEBRAICAS

PROBLEMAS PROPUESTOS

10. Calcular las siguientes expresiones. siendo x = +}, Y = 3. z = 2. a = 1/2. b = -2/3.

a) 4x3y2 - 3xyz2 z(x + y) 3ab I I I(x - y)(y - z)(z - .v}

e) -8T- y - x.+ Ig) -+-+-

h) x Y z

e) 9ab2 + 6ah - 4a2

d)xy2 - 3z f) (x - y)2 + 2z

11)(x - I){y - I)(z - 1)

---a + h ax + by (a - I){h - 1)

11. Determinar el grado de los polinomios siguientes:

a) 3x4 - 2X3 + x2 - 5 e) x5 + y5 + Z5 - 5xyz e) -103

b) 4xy4 _ 3X3y3 d) .j3 xyz - 5 f) y2 _ 3)'5 _ Y + 2y3 - 4

12. Suprimir los símbolos de agrupamiento y simplificar los resultados reduciendo términos semejantes:

a) (x + 3y - z) - (2y - x + 3z) + (4z - 3x + 2y)

b) 3(x2 - 2yz + y2) _ 4(x2 _ y2 _ 3yz) + x2 + y2

e) 3x + 4y + 3{x - 2(y - x) - y}

d) 3 - {2x - [1 - (x + y)] + [x - 2y]:

13. Sumar las expresiones algebraicas de cada uno de los grupos siguientes:

a) 2X2 + )'2 - X + y. 3y2 + X - x2, X - 2y + x2 _ 4y2

h) a2 - ab + 2hc + 3e2• 2ab + b2 - 3hc - 4e2, ab - 4be + e2 - a2, a2 + 2e2 + 5be - 2ab

e) 2a2bc - 2aeb2 + 5c2ab, 4b2ae + 4bea2 - Tac'b, 4abe2 - 3a2be - 3ab2e, b2ae - abc? - 3a2be

14. Restar la segunda expresión de la primera de los grupos siguientes:

a) 3xy - 2yz + 4zx, 3zx + yz - 2xy

b) 4x2 + 3.1'2- 6x + 4y - 2, 2x - y2 + 3x2 - 4y + 3

e) r3 - 3r2s + 4rs2 - S3, 2s3 + 3s2r - 2sr2 - 3r3

15. Restar xy - 3yz + 4xz del doble de la suma de las siguientes expresiones: 3xy - 4yz + 2xz y 3yz - 4zx - 2xy.

16. Efectuar el producto de las expresiones algebraicas de los grupos siguientes:

a) 4X2y5, _3x3y2

b) 3abe2• _ 2a3h2e4• 6a2b2

e) _4x2y, 3X)'2 - 4xy

d) r2s + 3rs3 - 4rs + S3. 2r2s4

e) y - 4, Y + 3

17. Efectuar las divisiones:

f) y2 - 4y + 16. Y + 4

g) x3 + x2)' + xy2 + y3, X _ Y

h) x2 + 4x + 8. x2- 4x + 8

i) 3r - s - /2, 2s + r + 3/2

j) 3 - x - y, 2x + y + 1, x - y

4ab3 - 3a2he + l2a3 b2e4

e) -2ah2e34x3 - 5x2 + 3x - 2

d)x+l

ra

19

http://carlos2524.jimdo.com/

Page 28: Algebra Superior Murray R Spiegel

20 OPERACIONES FUNDAMENTALES CON EXPRESIONES ALGEBRAICAS

18. Efectuar las divisiones:

27s3 - 64a) 3s _ 4

2y3 + y5 - 3y - 2e) y2 _ 3y + 1

19. Efectuar las operaciones indicadas y comprobarlas con los valores x = 1. y = 2:

x4 + xy3 + X3y + 2x2y2 + y4b) xy + x2 + y2

SOLUCIONES DE LOS PROBLEMAS PROPUESTOS

10. a) -24 b) -12 e) -1 d) 90 e) 11/5 f) -8 g) -1/6 h) -24/5

11. a) 4 b) 6 e) 5 d) 3 e) O f) 5

12. a) 3y - x b) 8y2 + 6yz e) l2x - 5y d) y - 4x + 4

13. a) 2x2+x-y b) a2+b2+2e2 e) abc"

14. a) 5xy - 3yz + zx b) x2 + 4y2 - 8x + 8y - 5 e) 4r3 - r2s + rs2 - 3s3

PRO]ned

15. xy + yz - 8xz

16. a) _12x5y7

b) _ 36a6b5e6

e) - l2x3y3 + 16x3y2

d) 2r4s5 + 6r3s7 _ 8r3s5 + 2r2s7

e) y2 _ Y - 12

4X2Z2

b)9s

17. a) -7 2r21

f) y3 + 64

g) x4 _ y4

h) x4 + 64

i) 3r2 + 5rs + 8rl2 - 2s2 - 5S12 - 314

j) y3 _ 2y2 _ 3y + 3x + 5x2 - 3xy - 2X3 - .t2y + 2xy2

2b 3a 2e) - - + - - 6a e

. e3 2be2-14

d) 4x2 - 9x + 12 + --x+l

18. a) 9s2 + l2s + 16 3 2 1b) -x - x + 1 _ x

68y - 29e) y3 + 3y2 + lOy + 27 + ~2~---:---

y - 3y + 1d) x2 + xy

19. al x8 + x4y4 + y8. Comprobación: 21(13) = 273. b) x2 + y2 Comprobación: 35/7 = 5.

http://carlos2524.jimdo.com/

Page 29: Algebra Superior Murray R Spiegel

Productos de interés práctico

xy

CAPITULO 3

PRODUCTOS DE INTERES PRACTICO. Las fórmulas que se exponen a continuación son elresultado de algunos de los productos que con mayor frecuencia se presentan en el cálculo algebrai-co y con los que el alumno debe procurar familiarizarse en todo lo posible. La comprobación dedichos resultados se puede realizar efectuando las multiplicaciones correspondientes.

I a(c + d) = ac + ad11 (a + b )(a - b) = a2 - b2

III (a + b)(a + b) = (a + b)2 = a2 + 2ab + b2

IV (a - b)(a - b) = (a - b)2 = a2 - 2ab + b2

V (x + a)(x + b) = x2 + (a + b)x + abVI (ax + b)(cx + d) = acx? + (ad + bc)x + bd

VII (a + b)(e + d) = ac + be + ad + bd

Otros productos muy utilizados son:14

+1 VIII (a + b)(a + b)(a + b) = (a + b)3 = a3 + 3a2b + 'sab? + b3

IX (a - b)(a - b)(a - b) = (a - b)3 = a3 - 3a2b + 'sab? - b3

X (a - b)(a2 + ab + b2) = a3 - b3

XI (a + b)(a2 - ab + b2) = a3 + b3

XII (a + b + C)2 = a2 + b2 + e2 + 2ab + 2ae + 2bc

Se puede comprobar, efectuando las multiplicaciones, que

(a - b)(a2 + ab + b2) = a3 - b3 (X)(a - b)(a3 + a2b + ab? + b3) = a4 - b4

(a - b)(a4 + a'b + a2b2 + ab3 + b4) = aS - b5

(a - b)(a5 + a4b + a3b2 + a2b3 + ab" + b5) = a6 _ b6

etc. Generalizando, tendremos

XIII (a - b)(an-1 + an-2b + an-3b2 + ... + abn-2 + bn-1) = a" _ bn

siendo n un entero positivo cualquiera (1, 2, 3. 4, ... ).Análogamente, se puede comprobar que

(a + b)(a2 - ab + b2) = a3 + b3 (Xl)(a + b)(a4 - a+b + a'b? - ab? + h4) = aS -+ h5

21

CAPITULO 3

Productos de interés práctico

PRODUCTOS DE INTERES PRACTICO. Las fórmulas que se exponen a continuación son el resultado de algunos de los productos que con mayor frecuencia se presentan en el cálculo algebrai­co y con los que el alumno debe procurar familiarizarse en todo lo posible. La comprobación de dichos resultados se puede realizar efectuando las multiplicaciones correspondientes.

a(e + d) = ae + ad

11 (a + b )(a - b) = a2 - b2

III (a + b)(a + b) = (a + b)2 = a2 + 2ab + b2

IV (a - b)(a - b) = (a - b)2 = a2 - 2ab + b2

V (x + a)(x + b) = X2 + (a + b)x + ab

VI (ax + b)(ex + d) = aex2 + (ad + be)x + bd

VII (a + b)(e + d) = ae + be + ad + bd

Otros productos muy utilizados son:

VIII (a + b)(a + b)(a + b) = (a + b)3 = a3 + 3a2b + 3ab2 + b3

IX (a - b)(a - b)(a - b) = (a - b)3 = a3 - 3a2b + 3ab2 - b3

X (a - b)(a2 + ab + b2) = a3 - b3

XI (a + b )(a2 - ab + b2) = a3 + b3

XII (a + b + e)2 = a2 + b2 + e2 + 2ab + 2ae + 2bc

Se puede comprobar, efectuando las multiplicaciones, que

(a - b)(a2 + ab + b2) = a3 - b3 (X)

(a - b)(a3 + a2b + ab2 + b3) = a4 - b4

(a - b)(a4 + a3b + a2b2 + ab3 + b4) = a5

- b5

(a - b)(a 5 + a4b + a3b2 + a2b3 + ab4 + b5) = a6

_ h6

etc. Generalizando, tendremos

XIII (a - b)(an - I + an- 2b + an

- 3b2 + ... + abn- 2 + bn

-I

) = an _ bn

siendo n un entero positivo cualquiera (1, 2, 3. 4, ... ).

Análogamente, se puede comprobar que

(a + b)(a2 - ab + bZ) = (13 + h3 (XI)

(a + b)(a4 - a3b + a2hz - ab3 + b4

) = a5 + h5

21

http://carlos2524.jimdo.com/

Page 30: Algebra Superior Murray R Spiegel

22 PRODUCTOS DE INTERES PRACTICO

(a + h)(a6 - a5h + a4h2 - a3b3 + a2h4 - ab5 + b6) = a7 + b7

etc. Generalizando tendremos

w)

x)

XIV (a + b)(an-I - an-2b + an-3b2 - - ab"-2 + b"-I) = a" + b"siendo n un entero positivo impar (1, 3, 5, 7, ).

y)

PRODU

PROBLEMAS RESUELTOS2. a)

Efectuar los productos que se indican.b)

e)

PRODUCTOS I-VII

1. a)

b)

e)

d)

e)

f)g)

11)

i)

j)

k)

1)

111)

n)

o)

p)

q)

r)

s)

1)

u)

L')

3x(2x + 3y) = (3x)(2x) + (3x)(3y) = 6x2 + 9xy. aplicando I con a = 3x, e = 2x, d = 3y

x2y(3x3 - 2y + 4) = (x2.1')(3x3) + (x2y)(-2y) + (x2y)(4) = 3x'y _ 2x2y2 + 4x2y

(3x3y2 + 2xy - 5)(X2y3) = (3x3y2)(X2y3) + (2xY)(X2y3) + (_5)(X2y3)

= 3x' y' + 2X3.1'4 _ 5x2 y3

(2x + 3y)(2x - 3)') = (2X)2 - (3.1')2 = 4x2 - 9y2. aplicando 11 con a = 2x, b = 3y

(1 - 5x3)(1 + 5x3) = (1)2 - (5X3)2 = 1 - 25x6

(5x + X3y2)(5x _ X3y2) = (5X)2 _ (X3y2)2 = 25x2 _ X6y4

(3x + 5y)2 = (3X)2 + 2(3x)(5y) + (5y)2 = 9x2 + 30xy + 25y2. aplicando III con a = 3x, b = 5y

(x + 2)2 = x2 + 2(x)(2) + 22 = x2 + 4x + 4

(7x2 - 2xy)2 = (7X2)2 - 2(7x2)(2xy) + (2xy)2

= 49x4 - 28x]r + 4x2y2. aplicando IV con a = 7x2, b = 2"1

(ax - 211y)2 = (ax)2 - 2(ax)(2I1y) + (211.1')2= a2x2 - 4axby + 4b2y2

(x4 + 6)2 = (X4)2 + 2(x4)(6) + (6)2 = x8 + 12x4 + 36

(3y2 _ 2)2 = (3y2)2 _ 2(3y2)(2) + (2)2 = 9y4 - 12y2 + 4

(x + 3)(x + 5) = x2 + (3 + 5)x + (3)(5) = x2 + 8x + 15. aplicando V con a = 3, b' = 5

(x - 2)(x + 8) = x2+ (-2 + 8)x + (-2)(8) = x2 + 6x - 16

(x + 2)(x - 8) = x2 + (2 - 8)x + (2)(-8) = x2 - 6x - 16

(t2 + 10)(12 - 12) = (t2)2 + (10 - 12)12 + (10)( -12) = 14 - 212 - 120

(3x + 4)(2x - 3) = (3)(2)x2 + [(3)(-3) + (4)(2)]x + (4)(-3)

= 6x2 - X - 12. aplicando VI con a = 3, b ~ 4, e = 2, d = -3

(2x + 5)(4x - 1) = (2)(4)x2 + [(2)(-1) + (5)(4)]x + (5)(-1) = 8x2 + 18x - 5

(3x + y)(4x - 2y) = (3x)(4x) + (y)(4x) + (3x)(-2y) + (y)(-2y)

= 12x2 - 2xy - 2y2. aplicando VII con a = 3x, b = y. e = 4x, d = -2y

(312s - 2)(41 - 35) = (3/2s)(4/) + (-2)(4/) + (312s)(-3s) + (-2)(-3s)

= 1213S _ 81 - 912S2 + 6s

d)

e)

f)

g)

h)

i)

j)

k)

PRODl

3. a)

b)

e)

OTRO!

4. a)

(3xy + 1)(2x2 - 3y) = (3xy)(2x2) + (3xy)(-3y) + (I)(2X2) + (I)(-3y)

= 6x3y - 9xy2 + 2X2 - 3.1'

(x + y + 3)(x + y - 3) = (x + y)2 - 32 = x2 + 2xy + y2 - 9

http://carlos2524.jimdo.com/

Page 31: Algebra Superior Murray R Spiegel

PRODUCTOS DE INTERES PRACTICO

w) (2x - y - 1)(2x - y + 1) = (2x - y)' - (1)' = 4x' - 4xy + y' - 1

x) (x' + 2xy + y')(x' - 2xy + y') = (x' + y' + 2xy)(x' + y' - 2xy)

= (x' + y')' - (2xy)' = x4 + 2x'y' + y4 _ 4x'y' = x4 _ 2x'y' + y4

y) (x) + 2 + xy)(x) - 2 + xy) = (x) + xy + 2)(x) + xy - 2)

= (x) + xy)' - 2' = x6 + 2(x))(xy) + (xy)' - 4 = x6 + 2x4y +-..r'y' - 4

PRODUCTOS VIII~XII

2. a) (x + 2y)) = x) + 3(xf (2y) + 3(x)(2y)' + (2y¡J

= x) + 6x'y + 12xy' + 8y), aplicando VIII con a = x, b = 2y

b) (3x + 2)) = (3x)) + 3(3x)' (2) + 3(3x)(2)' + (2)) = 27x3. + 54x' + 36x + 8

e) (2y - 5)) = (2y)3 - 3(2y)' (5) + 3(2y)(5)' - (W= 8y) - 60y' + 150y - 125, aplicando IX con a = 2y, b = 5

d) (xy - 2)) = (xy)3 - 3(xy)' (2) + 3(xy)(2)' - (2)3 = X3y3 - 6x'y' + 12xy - 8

e) (x'y - y')3 = (x'y)) _ 3(x'y)' O.') + 3(x'y)(y')' _ (y')) = X6y3 _ 3x4y4 + 3x'y5 _ )'6

1) (x - l)(x' + x + 1) = x) - 1, aplicando X con a = x, b = 1

Si se reconoce la forma, se multiplica como sigue:

(x - 1)(x' + x + 1) = x(x' + x + 1) - 1(x' + x + 1) = x3 + x' + x - x' - x - 1 = x) - 1

g) (x - 2y)(x' + 2xy + 4y') = x) - (2y)) = x) - 8y3, aplicando X con a = x, b = 2y

h) (xy + 2)(x'y' - 2xy + 4) = (xy)3 + (2)3 = X3y3 + 8, aplicando XI con a = xy, b = 2

i) (2x + 1)(4x' - 2x + 1) = (2X)3 + 1 = 8x) + 1

j) (2x + 3y + z)' = (2x)' + (3y)' + (d + 2(2x)(3y) + 2(2x)(z) + 2(3y)(z)

= 4x' + 9y' + z' + 12xy + 4xz + 6)'z, aplicando XII con a = 2x, b = 3y, e = z

k) (u' - v' + 2w)' = (u))' + (-v')' + (2w)' + 2(u))(-v') + 2(u)'(2w) + 2(-v')(2w)

= u6 + v4 + 4w' - 2u)v' + 4u)w - 4v'w

PRODUCTOS XIII-XIV

3. a) (x - l)(x5 + x4 + x) + x' + x + 1) = x6 - 1, aplicando XIII con a = x. b = 1, n = 6

b) (x - 2y)(x4 + 2x)y + 4x'y' + 8xy) + 16y4) = x5 - (2y)5

= x5 - 32.1'5, aplicando XIII con a = .v, b = 2y

e) (3y + x)(81y4 - 27y)x + 9),'x' - 3yx3 + x") = (3.1')5 + x5

= 243)'5 + x5, aplicando XIV con a = 3y, b = x

OTROS TIPOS DE PRODUCTOS

4. a) (x + y + z)(x + y - z)(x - y + z)(x - y - z). El producto de los dos primeros factores es

(x + y + z)(x + y - z) = (x + y)' - z' = x' + 2xy + y' - z'

y el de los dos últimos

(x - y + z)(x - y - z) = (x - y)' - z' = x' - 2xy + y' - z,

El resultado es que

23PRODUCTOS DE INTERES PRACTICO 23

w) (2x - y - 1 )(2x - y + 1) = (2x - y)2 - (1 ¡> = 4X2 _ 4xy + y2 _ 1

x) (x2 + 2xy + y2)(X2 _ 2xy + y2) = (x2 + .1'2 + 2xy)(x2 + .1'2 - 2xy)

= (x2 + y2)2 _ (2xy)2 = x4 + 2x2y2 + y4 _ 4x2y2 = x4 _ 2x2y2 + y4

y ) (X' + 2 + xy)(x' - 2 + x)') = (x' + xy + 2)(x' + xy - 2)

= (x' + xy)2 - 22 = x 6 + 2(x')(xy) + (xy)2 - 4 = x 6 + 2x4y +-..r2y2 - 4

PRODUCTOS VIII :XII

2. a) (x + 2y)' = x' + 3(X)2 (2y) + 3(x)(2y)2 + (2y)'

= x' + 6x2y + 12xy2 + 8y', aplicando VIII con a = x, b = 2y

b) (3x + 2)3 = (3X)3 + 3(3x)2 (2) + 3(3x)(2)2 + (2)3 = 27x3. + 54x2 + 36x + 8

e) (2y - W = (2y)3 - 3(2y)2 (5) + 3(2y)(5)2 - (5)3

= 8y 3 - 60y2 + 150y - 125, aplicando IX con a = 2y, b = 5

d) (xy - 2)3 = (xy)' - 3(xy)2 (2) + 3(xy)(2)2 - (2)3 = X 3)'3 - 6x2y2 + I2xy - 8

e) (x2y _ y2)3 = (X2y)3 _ 3(X2y)2 ()'2) + 3(x2y)(v2)2 _ (v2)3 = x 6)" _ 3x4y4 + 3x2ys _ y6

J) (x - 1 )(x2 + X + 1) = x3 - 1, aplicando X con a = x, b = 1

Si se reconoce la forma, se multiplica como sigue:

(x - 1 )(x2 + X + 1) = x(x2 + X + 1) - 1 (x2 + X + 1) = x 3 + X2 + X - X2 - X - 1 = x 3 - 1

g) (x - 2y)(x2 + 2xy + 4y2) = x 3 - (2)')' = x' - 8y', aplicando X con a = x, b = 2)'

h) (xy + 2)(X2)'2 - 2xy + 4) = (xy)' + (2)3 = X3y 3 + 8, aplicando XI con a = xy, b = 2

i) (2x + 1 )(4x2 - 2x + 1) = (2X)3 + 1 = 8x3 + 1

j) (2x + 3y + Z)2 = (2X)2 + (3y )2 + (Z)2 + 2(2x)(3y) + 2(2x)(z) + 2(3y)(z)

= 4X2 + 9y2 t Z2 + 12xy + 4xz + 6yz, aplicando XII con a = 2x, b = 3)', e = z

k) (u j - v2 + 2W)2 = (U3)2 + (- V2)2 + (2W)2 + 2(u3)( - v2) + 2(u"(2u:) + 2( - v2)(2w)

= u6 + v4 + 4w2 _ 2U3V2 + 4u3u: - 4v2u;

PRODUCTOS XIII -XIV

3. a) (x - 1 )(xs + x 4 + x' + X2 + X + 1) = x 6 - l. aplicando XIII con a = x, h = 1, n = 6

b) (x - 2y)(x4 + 2x3y + 4x2y2 + 8xy' + 16y4) = x5 - (2y)s

= x S - 32y s, aplicando XIII con a = x, h = 2y

e) (3y + x)(81y4 - 27y'x + 9),2X2 - 3yx' + X4) = (3.1')5 + XS

= 243)'s + x S, aplicando XIV con a = 3y, h = x

OTROS TIPOS DE PRODUCTOS

4. a) (x + )' + z)(x + y - z)(x - y + z)(x - y - z). El producto de los dos primeros factores es

(x + y + z)(x + y - z) = (x + y)2 - Z2 = X2 + 2xy + y2 - Z2

Y el de los dos últimos

(x - y + z)(x - y - z) = (x - y)2 - Z2 = X2 _ 2xy + y2 - Z2

El resultado es que

http://carlos2524.jimdo.com/

Page 32: Algebra Superior Murray R Spiegel

24

s. a)

b)

e)

d)

e)

f)

g)

h)

i)

j)

k)

1)

m)

n)

o)

p)

q)

r)

s)

()

PRODUCTOS DE INTERES PRACTICO

(x2 + )'2 _ =2 + 2xy)(x2 + y2 _ 2xy) = (x2 + )'2 _ Z2)2 _ (2xy)2

= (X2)2 + (y2)2 + (_ Z2)2 + 2(x2 )(y2) + 2(x2)( _ Z2) + 2(y2)( _ Z2) _ 4x2 y2

= X' + y' + z' - 2x2y2 - 2X2Z2 - 2y2z2

u)

v)

w)

x)

y)

6. a)

b)

e)

d)

e)

f)g)

h)

7. a)

b)

e)

d)

e)

f)g)

b) (x + y + 'z + 1)2 = [(x + y) + (z + 1)]2 = (x + y)2 + 2(x + y)(z + 1) + (z + 1¡>

= x2 + 2xy + y2 + 2xz + 2x + 2yz + 2y + Z2 + 2z + 1

e) (u - V)3 (u + V)3 = [(u - v)(u + vJ]3 = (u2 - V2)3

= (u2)3 _ 3(U2)2V2 + 3(U2)(V2)2 _ (V2)3 = u6 _ 3u'v2 + 3u2v' _ 116

d) (x2 - X + I)¿ (x2 + X + 1)2 = [(x2 - X + 1)(x2 + X + 1)]2 = [(x2 + 1 - X)(X2 + 1 + xJ]2

= [(x2 + 1)2 _ X2]2 = [x' + 2X2 + 1 _ X2]2 = (x" + x2 + 1)2

= (X')2 + (X2)2 + 12 + 2(x')(x2') + 2(x')(I) + 2(x2)(1)

= XS + x' + 1 + 2X6 + 2x' + 2X2 = x8 + 2X6 + 3x' + 2X2 + 1

e) (e' + l)(eY - l)(e2y + l)(éY + I)(esy + 1) = (e2y - 1)(e2y + l)(e'Y + I)(esy + 1)

= (e'Y - l)(e'Y + I)(esy + 1) = (eS Y - l)(e8y + 1) = el6y - 1

PROBLEMAS PROPUESTOS

Efectuar los siguientes productos:

2xy(3x2y - 4y3) = 6x3y2 - 8xy'

3X2y3 (2xy - x - 2y) = 6x3y' - 3X3y3 - 6x2y'

(2S(3 _ 4rs2 + 3s3()(5rs(2) = 10rs2(5 - 20r2s3(2 + 15rs'(3

(3a + 5b)(3a - 5b) = 9a2 - 25b'

(5xy + 4)(5xy - 4) = 25x2y2 - 16

(2 - 5y2)(2 + 5y2) = 4 - 25y'

(3a + 5a2b)(3a - 5a2b) = 9a2 - 25a'b2

(x + 6 ¡> = x2 + 12x + 36

(y + 3X)2 = y2 + 6x)' + 9x2

(z - 4)2 = Z2 - 8z + 16

(3 - 2X2)2 = 9 - 12x2 + 4x'

(x2y _ 2Z)2 = x'y2 - 4x2yz + 4z2

(x + 2)(x + 4) = x2 + 6x + 8

(x - 4 )(x + 7) = x2 + 3x - 28

(y + 3)lv - 5) = y2 - 2y - 15

(xy + 6)(xy - 4) = x2y2 + 2x)' - 24

(2x - 3)( 4x + 1) = 8x2 - 10x - 3

(4 + 3r)(2 - r) = 8 + 2r - 3r2

(5x + 3y)(2x - 3y) = IOx2 - 9xy - 9y2

(2(2 + S)(3(2 + 4s) = 6(' + 11(2S + 4s2

http://carlos2524.jimdo.com/

Page 33: Algebra Superior Murray R Spiegel

PRODUCTOS DE INTERES PRACTICO 25

u) (x2 + 4y)(2x2y _ y2) = 2x4)' + 7X2)'2 _ 4)'3

v) x(2x - 3)(3x + 4) = 6x3 - x2 - 12x

w) (r + S - l)(r + s + 1) = r2 + 2rs + .1'2- 1

x) (x - 2y + z)(x - 2y - z) = x2 - 4x)' + 4y2 _ Z2

y) (x2 + 2x + 4)(x2 - 2x + 4) = x4 + 4x2 + 16

6. a) (2x + 1)3 = 8x3 + 12x2 + 6x + 1

b) (3x + 2y)3 = 27x3 + 54x2y + 36xy2 + 8y3

e) (r - 2S)3 = r3 - 6r2s + 12rs2 _ 8.1'3

d) (x2 - 1)3 = x6 _ 3x4 + 3x2 - 1

e) (ab2 - 2b)3 = a3b6 - 6a2b' + 12ab4 _ 8b3

f) (1 - 2)(12 + 2¡ + 4) = ¡3 - 8

g) (z - X)(x2 + xz + Z2) = Z3 _ x3

h) (x + 3y)(x2 - 3xy + 9y2) = x3 + 27y3

7. a) (x - 2)' + Z)2 = x2 - 4xy + 4y2 + 2zx - 4zy + Z2

b) (s - 1)(S3 + S2 + S + 1) = S4 - 1

e) (1 + ¡2)(1 - ¡2 + ¡4 _ ¡6) = 1 _ ¡8

d) (3x + 2y)2 (3x - 2)')2 = 81x4 - 72x2y2 + 16y4

e) (x2 + 2x + 1)2 (x2 - 2x + 1)2 = x8 - 4x6 + 6x4 _ 4x2 + 1

f) (y - 1)3 (y + 1)3 = y6 _ 3y4 + 3y2 _ 1

g) (u + 2)(u - 2)(u2 + 4)(u4 + 16) = u8 - 256

http://carlos2524.jimdo.com/

Page 34: Algebra Superior Murray R Spiegel

LOS PF

A) Fac

CAPITULO 4

B) Difi

Descomposición en factores

LOS FACTORES de una expresión algebraica dada son dos o más expresiones algebraicas quemultiplicadas entre sí originan la primera.

Por ejemplo, la expresión algebraica x2 - 7x + 6 se puede expresar como producto de losdos factores (x - 1)(x - 6).

Análogamente, x2 + 'f.xy - 8y2 = (x + 4y)(,'i: - 2)')

C) tw

PROCESO DE DESCOMPOSICION EN FACTORES. Se aplica, generalmente, a polinomios decoeficientes enteros. En este caso, se requiere que los factores sean también polinomios de coefi-cientes enteros. Mientras no se advierta lo contrario, supondremos estas condiciones.

D) 011

Por ejemplo, (x - 1) no lo consideraremos descompuesto en los factores (Jx + I)(Jx - 1),ya que éstos no son polinomios. Igualmente, (x2 - 3y2) no' lo consideramos descompuestoen los factores (x - j3y)(x + j3y), ya que éstos no son polinomios de coeficientes enteros.

Asimismo, aunque 3x + 2y se pueda expresar por 3(x + ~y), no lo consideraremos así, porque

x + ~y no es un polinomio de coeficientes enteros.

Un polinomio de coeficientes enteros es primo cuando no se puede descomponer en factoressiguiendo los criterios expuestos anteriormente. Por ejemplo, x2 - 7x + 6 = (x - l)(x - 6) estáexpresado como producto de los factores primos x - 1 Y x - 6.

Un polinomio se puede descomponer totalmente en factores cuando se pueda expresar comoproducto de factores primos.

E) SU!

Nota J. En la descomposición en factores se pueden efectuar cambios de signo; Por ejem-plo, x2 - 7x + 6 se puede descomponer en (x - 1)(x - 6), o bien en (1 - x)(6 - x). Se demues-tra que la descomposición en factores primos, prescindiendo de los cambios de signo o del ordende los factores, es única. Este es el teorema fundamental de la descomposición en factores.

Nota 2. Un polinomio es primo cuando no admite más factores (o divisores) que él mismo,con signo más o menos, y la unidad, ± 1. Esta definición es análoga a la de números primos, comoson 2, 3, 5, 7, 11, ...

Nota 3. Algunas veces se descomponen en factores polinomios de coeficientes racionales;por ejemplo, x2 - 9/4 = (x + 3/2)(x - 3/2). En estos casos, los factores son también polinomiosde coeficientes racionales.

F) Ag

G) Fa

EN LA DESCOMPOSICION EN FACTORES son de gran aplicación las fórmulas I-XIV del Ca-pítulo 3. De la misma forma que leídas de izquierda a derecha dan el resultado de un producto, cuan-do se leen de derecha a izquierda constituyen la descomposición en factores.

26

CAPITULO 4

Descomposición en factores

LOS FACTORES de una expresión algebraica dada son dos o más expresiones algebraicas que multiplicadas entre sí originan la primera.

Por ejemplo, la expresión algebraica X2 - 7x + 6 se puede expresar como producto de los dos factores (x - 1 )(x - 6).

Análogamente, X2 + ,2xy - 8y2 = (x + 4yKx- - 2y)

PROCESO DE DESCOMPOSICION EN FACTORES. Se aplica, generalmente, a polinomios de coeficientes enteros. En este caso, se requiere que los factores sean también polinomios de coefi­cientes enteros. Mientras no se advierta lo contrario, supondremos estas condiciones.

Por ejemplo, (x - 1) no lo consideraremos descompuesto en los factores (Jx + l)(Jx - 1), ya que éstos no son polinomios. Igualmente, (x2 - 3y2) no' lo consideramos descompuesto en los factores (x - j3y)(x + j3y), ya que éstos no son polinomios de coeficientes enteros.

Asimismo, aunque 3x + 2y se pueda expresar por 3(x + ~y), no lo consideraremos así, porque

x + ~y no es un polinomio de coeficientes enteros.

Un polinomio de coeficientes enteros es primo cuando no se puede desQOmponer en factores siguiendo los criterios expuestos anteriormente. Por ejemplo, X2 - 7x + 6 = (x - l)(x - 6) está expresado como producto de los factores primos x - 1 Y x - 6.

Un polinomio se puede descomponer totalmente en factores cuan.do se pueda expresar como producto de factores primos.

Nota J. En la descomposición en factores se pueden efectuar cambios de signo; Por ejem­plo, X2 - 7x + 6 se puede descomponer en (x - l)(x - 6), o bien en (1 - x)(6 - x). Se demues­tra que la descomposición en factores primos, prescindiendo de los cambios de signo o del orden de los factores, es única. Este es el teorema fundamental de la descomposición en factores.

Nota 2. Un polinomio es primo cuando no admite más factores (o divisores) que él mismo, con signo más o menos, y la unidad, ± 1. Esta definición es análoga a la de números primos, como son 2, 3, 5, 7, 11, ...

Nota 3. Algunas veces se descomponen en factores polinomios de coeficientes racionales; por ejemplo, X2 - 9/4 = (x + 3/2)(x - 3/2). En estos casos, los factores son también polinomios de coeficientes racionales.

EN LA DESCOMPOSICION EN FACTORES son de gran aplicación las fórmulas I-XIV del Ca­pítulo 3. De la misma forma que leídas de izquierda a derecha dan el resultado de un producto, cuan­do se leen de derecha a izquierda constituyen la descomposición en factores.

26

http://carlos2524.jimdo.com/

Page 35: Algebra Superior Murray R Spiegel

que

los

deoefi-

1),estoros.

que

oresestá

amo

jern-ues-rden

smo,amo

ales;mios

l ea-cuan-

DESCOMPOSICION EN FACTORES 27LOS PROCEDIMIENTOS SIGUIENTES son de gran utilidad en la descomposición en factores.

A) Factor monomio común. Tipo: ac + ad = a(c + d)

Ejemplos. 1) 6x2y - 2X3 = 2x2(3y - x)2) 2x3y - xy2 + 3x2y = xy(2x2 - y + 3x)

B) Diferencia de los cuadrados. Tipo: a2 - b2 = (a + b)(a - b)

Ejemplos. 1) x2 - 25 = x2 - 52 = (x + 5)(x - 5) donde a = x, b = 52) 4x2 - 9y2 = (2X)2 - (3y)2 = (2x + 3y)(2x - 3y) donde a = 2x, b = 3y

C) Trinomio cuadrado perfecto. T. a2 + 2ab + b2 = (a + b)2IPOS' . a2 _ 2ab + b2 = (a - b)2

Un trinomio es un cuadrado perfecto si dos términos son cuadrados perfectos y el terceroes igual al duplo de la raíz cuadrada del producto de aquéllos.

Ejemplos. 1) x2 + 6x + 9 = (x + 3)22) 9x2 - 12xy + 4y2 = (3x - 2y)2

D) Otros trinomios.. x2 + (a + b)x + ab = (x + a)(x + b)

TIpos: dacx" + (ad + bc)x + bd = (ax + b)(cx + )

Ejemplos. 1) x2 - 5x + 4 = (x - 4)(x - 1) siendo a = -4, b = -1 su suma es iguala (a + b) = -5 Y su producto ab = 4.

2) x2 + xy - 12y2 = (x - 3y)(x + 4y) siendo a = - 3y, b = 4y3) 3x2-5x-2=(x-2)(3x+ 1).Enestecasoac=3,bd= -2.ad+bc= -5;

por tanteos se obtiene que a = 1, e = 3, b = - 2, d = 1 satisface ad + be = - 5.

4) 6x2 + X - 12 = (3x - 4)(2x + 3)5) 8 - 14x + 5x2 = (4 - 5x)(2 - x)

. '. . . a3 + b3 = (a + b )(a2 - ab + b2)E) Suma, diferencia de dos cubos. TIpos: 3 b3 ( b)( 2 b b2)

• a- =a- a+a+

Ejemplos. 1) 8x3 + 27y3 = (2x)3 + (3y)3= (2x + 3y)[(2x)2 - (2x)(3y) + (3y)2]= (2x + 3y)(4x2 - 6xy + 9y2)

2) 8X3y3 - 1 = (2xy)3 - 13 = (2xy - 1)(4x2y2 + 2xy + 1)

F) Agrupamiento de términos. Tipo: ac + be + ad + bd = c(a + b) + dia + b) = (a + b)(c + d)

Ejemplo. 2ax - 4bx + ay - 2by = 2x(a - 2b) + y(a - 2b) = (a - 2b)(2x + y)G) Factores de o" ± b". Aplicamos la fórmulas XIII y XIV del Capítulo 3.

Ejemplos. 1) 32x5 + 1 = (2X)5 + 15 = (2x + 1)[(2X)4 - (2X)3 + (2X)2 - 2x + 1]= (2x + l)(16x4 - 8x3 + 4x2 - 2x + 1)

2) X 7 - 1 = (x - 1)(x6 + x5 + x4 + x.3 + x2 + X + 1)

DESCOMPOSICION EN FACTORES 27

LOS PROCEDIMIENTOS SIGUIENTES son de gran utilidad en la descomposición en factores.

A) Factor monomio común. Tipo: ac + ad = a(c + d)

Ejemplos. 1) 6x2y - 2X3 = 2x2(3y - x) 2) 2x3y - xy2 + 3x2y = xy(2x2 - y + 3x)

B) Diferencia de los cuadrados. Tipo: a2 - b2 = (a + b)(a - b)

Ejemplos. 1) X2 - 25 = X2 - 52 = (x + 5)(x - 5) donde a = x, b = 5 2) 4X2 - 9y2 = (2X)2 - (3y)2 = (2x + 3y)(2x - 3y) donde a = 2x, b = 3y

C) Trinomio cuadrado perfecto. T a2 + 2ab + b2 = (a + b)2

IpOS: a2 _ 2ab + b2 = (a _ b)2

Un trinomio es un cuadrado perfecto si dos términos son cuadrados perfectos y el tercero es igual al duplo de la raíz cuadrada del producto de aquéllos.

Ejemplos. 1) X2 + 6x + 9 = (x + 3)2 2) 9X2 - 12xy + 4y2 = (3x - 2y)2

D) Otros trinomios. T. X2 + (a + b)x + ab = (x + a)(x + b) IpOS: acx2 + (ad + bc)x + bd = (ax + b)(cx + d)

Ejemplos. 1) X2 - 5x + 4 = (x - 4)(x - 1) siendo a = - 4, b = -1 su suma es igual a (a + b) = -5 Y su producto ab = 4.

2) X2 + xy - 12y 2 = (x - 3y)(x + 4y) siendo a = - 3y, b = 4y 3) 3x2 -5x-2=(x-2)(3x+ 1).Enestecasoac=3,bd= -2,ad+bc= -5 ;

por tanteos se obtiene que a = 1, c = 3, b = -2, d = 1 satisface ad + bc = - 5.

4) 6X2 + X - 12 = (3x - 4)(2x + 3) 5) 8 - 14x + 5X2 = (4 - 5x)(2 - x)

. . . . a3 + b3 = (a + b )(a2 - ab + b2) E) Suma, diferencw de dos cubos. TIpos: 3 b3 ( b)( 2 b b2)

• a- =a- a+a+

Ejemplos. 1) 8x3 + 27y 3 = (h)3 + (3y)3 = (2x + 3y)[(2x)2 - (2x)(3y) + (3y)2J = (2x + 3y)(4x2

- 6xy + 9y2)

2) 8X3y 3 - 1 = (2xy)3 - 13 = (2xy - \)(4x2y2 + 2xy + 1)

F) Agrupamiento de términos. Tipo: ac + bc + ad + bd = c(a + b) + d(a + b) = (a + b)(c + d)

Ejemplo. 2ax - 4bx + ay - 2by = 2x(a - 2b) + y(a - 2b) = (a - 2b)(2x + y) G) Factores de cl' ± b". Aplicamos la fórmulas XIII y XIV del Capítulo 3.

Ejemplos. \) 32x5 + 1 = (2X)5 + 15 = (2x + 1)[(2X)4 - (2X)3 + (2X)2 - 2x + 1 J = (2x + 1)(16x4 - 8x3 + 4X2 - 2x + 1)

http://carlos2524.jimdo.com/

Page 36: Algebra Superior Murray R Spiegel

28 DESCOMPOSICION EN FACTORES

H) Suma y resta de términos.

Ejemplo. Factor x4 + 4.DIF

2.

Sumando y restando 4x2 (doble del producto de las raíces cuadradas de x4 y 4),obtenemosx4 + 4 = (x4 + 4x2 + 4) - 4x2 = (x2 + 2)2 - (2X)2

= (x2 + 2 + 2x)(x2 + 2 - 2x) = (x2 + 2x + 2)(x2 - 2x + 2)

1) Combinación de los métodos anteriores.Ejemplo. x4 _ xy3 _ X3y + y4 = (x4 _ xy3) _ (x3y _ y4)

= x(x3 _ y3) _ y(x3 _ y3)

= (x3 _ y3)(X - y) = (x - y)(x2 + xy + y2)(X - y)= (x - y)2(X2 + xy + y2)

EL MAXIMO COMUN DIVISOR (M.C.D.)de dos o más polinomios es el polinomio de mayorgrado y mayor coeficiente numérico (prescindiendo de los signos) que es factor (o divisor) de lospolinomios dados.

Para hallar el M.C.D. de varios polinomios se procede de la forma siguiente: a) Se descom-pone cada polinomio en el producto de sus factores primos. b) El M.C.D. es el producto obtenidoal tomar todos los factores comunes elevados a la menor potencia con la que entran a formar parteen cada uno de los polinomios.

Ejemplo. El M.C.D. de2332(x _ y)3(X + 2y)2, 2233(x - y)2(X + 2y)3, 32(x - y)2(X + 2y)es 32(x - y)2(X + 2y).

TRI

3.

Dos o más polinomios son primos entre sí si su M.C.D. es la unidad ± 1.

EL MINIMO COMUN MULTIPLO (M.C.M.) de dos o más polinomios es el polinomio de menorgrado y menor coeficiente (prescindiendo de los signos) del cual es factor (o divisor) cada uno delos polinomios dados.

Para hallar el M.C.M. de varios polinomios se procede de la forma siguiente: a) Se descom-pone cada polinomio en el producto de sus factores primos. b) El M.C.M. es el producto obtenidoal tomar todos los factores, comunes y no comunes, elevados a la mayor potencia con la que entrana formar parte en cada uno de los polinomios.

Ejemplo. El M.C.M. de2332(x - y)3(X + 2y)2, 2233(x _ y)2(X + 2y)3, 32(x - y)2(X + 2y)es 2333(x - y)3(X + 2y)3.

OT!

4.

PROBLEMAS RESUELTOS

FAcrOR BINOMIO COMUN. Tipo: ac + ad = a(e + d)

1. al 2x2 - 3xy = x(2x - 3yl e) 3x2 + 6x3 + 12x4 = 3x2 (1 + 2x + 4x2)

b) 4x + 8y + 12z = 4(x + 2y + 3z) d) 9s3[ + ISs2[3 - 3S2[2 = 3s2[(3s + S[2 - [)

e) 10a2b3e4 - ISa3b2c4 + 30a4b3e2 = Sa2b2c2 (2be2 - 3ac2 + 6a2b)f) 40"+ I - Sa20 = 40"+ I (1 - 20"- 1)

28 DESCOMPOSICION EN FACTORES

H) Suma y resta de términos.

Ejemplo. Factor x 4 + 4.

Sumando y restando 4X2 (doble del producto de las raíces cuadradas de x4 y 4), obtenemos

x4 + 4 = (x4 + 4X2 + 4) - 4X2 = (x2 + 2)2 - (2X)2 = (x2 + 2 + 2x)(x2 + 2 - 2x) = (x2 + 2x + 2)(x2 - 2x + 2)

1) Combinación de los métodos anteriores.

Ejemplo. x4 _ xy3 _ X3y + y4 = (x4 _ xy3) _ (x3y _ y4) = x(x3 _ y3) _ y(x3 _ y3)

= (x3 _ y3)(X - y) = (x - y)(x2 + xy + y2)(X - y)

= (x - y)2(X2 + xy + y2)

EL MAXIMO COMUN DIVISOR (M.C.D.)de dos o más polinomios es el polinomio de mayor grado y mayor coeficiente numérico (prescindiendo de los signos) que es factor (o divisor) de los polinomios dados.

Para hallar el M.C.D. de varios polinomios se procede de la forma siguiente: a) Se descom­pone cada polinomio en el producto de sus factores primos. b) El M.C.D. es el producto obtenido al tomar todos los factores comunes elevados a la menor potencia con la que entran a formar parte en cada uno de los polinomios.

Ejemplo. El M.C.D. de 2332(x _ y)3(X + 2y)2, 2233(x - y)2(X + 2y)3, 32(x - y)2(X + 2y)

es 32(x - y)2(X + 2y).

Dos o más polinomios son primos entre sí si su M.C.D. es la unidad ± 1.

EL MINIMO COMUN MULTIPLO (M.C.M.) de dos o más polinomios es el polinomio de menor grado y menor coeficiente (prescindiendo de los signos) del cual es factor (o divisor) cada uno de los polinomios dados.

Para hallar el M.C.M. de varios polinomios se procede de la forma siguiente: a) Se descom­pone cada polinomio en el producto de sus factores primos. b) El M.C.M. es el producto obtenido al tomar todos los factores, comunes y no comunes, elevados a la mayor potencia con la que entran a formar parte en cada uno de los polinomios.

Ejemplo. El M.C.M. de 2332(x - y)3(X + 2y)2, 2233(x _ y)2(X + 2y)3, 32(x - y)2(X + 2y)

es 2333(x - y)3(X + 2y)3.

PROBLEMAS RESUELTOS

FAcrOR BINOMIO COMUN. Tipo: ac + ad = a(e + d)

1. a) 2x2 - 3xy = x(2x - 3y)

b) 4x + By + 12z = 4(x + 2y + 3z)

e) 3X2 + 6x3 + 12x 4 = 3X2 (1 + 2x + 4x2)

d) 9s3 [ + 15s2[3 - 3S2[2 = 3s2[(3s + 5[2 - [)

e) 10a2b3e4 - 15a3b2c4 + 30a4b3c2 = 5a2b2e2 (2be2 - 3ae2 + 6a2b)

f) 40"+ I - 8020 = 40"+ I (1 - 20"- 1)

http://carlos2524.jimdo.com/

Page 37: Algebra Superior Murray R Spiegel

y)

descom-obtenidoar parte

de menorda uno de

e descom-obtenidoue entran

DESCOMPOSICION EN FACTORES

DIFERENCIA DE DOS CUADRADOS. Tipo: a2 - b2 = (a + b)(a - b)

2. a)

b)

e)

d)

e)

f)g)

11)

i)

j)

k)

1)

4. a)

b)

e)

d)

i)

j)

k)

/)2) m)-1) n)

o)

x2 - 9 = x2 - 32 = (x + 3)(x - 3)

25x2 - 4y2 = (5X)2 - (2y)2 = (5x + 2y)(5x - 2y)

9x2y2 - 16a2 = (3xy)2 - (4of = (3xy + 4o)(3xy - 4a)

1 - m2n4 = 12 - (mn2f = (1 + mn2)(1 - mn2)

3x2 - 12 = 3(x2 - 4) = 3(x + 2)(x - 2)

x2y2 _ 36y4 = y2 [X2 - (6.d] = y2 (x + 6y)(x - 6y)

x4 _ y4 = (X2)2 _ lV2)2 = (x2 + y2)(X2 _ y2) = (x2 + y2)(X + y)(x _ y)

1 - x8 = (1 + x4)(1 - x4) = (1 + x4)(1 + X2)(I - X2) = (1 + x4)(I + x2)(1 + x)(1 - x)

32a4b - 162bs = 2b(l6a4 - 81b4) = 2b(4a2 + 9b2)(402 _ 9b2)= 2b(402 + 9b2)(2a + 3b)(2a - 3b)

X3y - y3x = xy(x2 - y2) = xy(x + y)(x _ y)

(x + 1)2 - 36y2 = [(x + 1) + (6y)][(x + 1) - (6y)] = (x + 6y + I)(x - 6y + 1)

(5x + 2y)2 - (3x - 7y)2 = [(5x + 2y) + (3x - 7y)][(5x + 2y) - (3x - 7y)] = (8x - 5y)(2x + 9y)

TRINOMIO CUADRADO PERFECTO. a2 + 2ab + b2 = (a + b)2Tipos: a2 _ 2ab + b2 = (a _ b)2

3. a) x2 + 8x + 16 = x2 + 2(x)(4) + 42 = (x + 4)2

b) 1 + 4y + 4y2 = (1 + 2y)2

e) 12 - 41 + 4 = 12 - 2(1)(2) + 22 = (l - 2)2

g) 9x4 -- 24x2y + 16y2 = (3x2 - 4y)2

11) 2x3y3 + 16x2y4 + 32xys = 2xy3 (x2 + 8xy + 16y2) = 2xy3 (x + 4y)2

i) 16a4- 72a2b2 + 81b4 = (402 - 9b2)2 = [(2a + 3b)(2a - 3b)]2 = (2a + 3W (20 - 3W

j) (x + 2y)2 + 10(x + 2y) + 25 = (x + 2y + W k) a2x2 - 2abxy + b2y2 = (ax - by)2

/) 4m6n6 + 32m4n4 + 64m2n2 = 4m2n2 (m4n4 + 8m2n2 + 16) = 4m2n2 (m2n2 + 4)2

d) x2 - 16xy + 64y2 = (x - 8y)2

e) 25x2 + 60xy + 36y2 = (5x + 6y)2

f) 16m2 - 40mn + 25n2 = (4m - 5n)2

OTROS TRINOMIOS. Ti os' x2 + (a + b)x + ab = (x + a)(x + b)p . aex2 + (ad + be)x + bd = (ax + b)(ex + d)

x2 + 6x + 8 = (x + 4)(x + 2) e) x2 - 7xy + 12y2 = (x - 3y)(x - 4y)

x2 - 6x + 8 = (x - 4)(x - 2) f) x2 + xy - 12y2 = (x + 4y)(x - 3y)

x2 + 2x - 8 = (x + 4)(x - 2) g) 16 - 10x + x2 = (8 - x)(2 - x)

x2 - 2x - 8 = (x - 4)(x + 2) h) 20 - x - x2 = (5 + x)(4 - x)

3x3 - 3x2 - 18x = 3x(x2 - X - 6) = 3x(x - 3)(x + 2)

y4 + 7y2 + 12 = (y2 + 4)(y2 + 3)

m" + m2 - 2 = (m2 + 2)(m2 - 1) = (m2 + 2)(m + 1)(m - 1)

(x + 1)2 + 3(x + 1) + 2 = [(x + 1) + 2][(x + 1) + 1] = (x + 3)(x + 2)

S2/2 _ 2S13 _ 63/4 = t2(S2 - 2st - 6312) = 12(S - 91)(s + 71)

Z4 - IOz2 + 9 = (Z2 - l)(z2 - 9) = (z + I)(z - I)(z + 3)(z - 3)

2x6y _ 6X4y3 _ 8x2ys = 2x2Y(X4 _ 3x2y2 _ 4y4)

=2x2Y(X2 + y2)(X2 _ 4y2) = 2x2Y(X2 + y2)(X + 2y)(x - 2y)

29

http://carlos2524.jimdo.com/

Page 38: Algebra Superior Murray R Spiegel

DESCOMPOSICION EN FACTORES

p)

q)

r)

s)

1)

s, a)

b)

e)

g)

h)

i)

j)

k)

1)

x2 - 2xy + y2 + 10(x - y) + 9 = (x - y)2 + lO(x - y) + 9

= [(x - y) + 1][(x - y) + 9] = (x -)' + l)(x - y + 9)

4X8y'0 _ 40X5y7 + 84x2)'4 = 4X2y4 (X6y6 _ 10x'y' + 21) = 4X2y4 (x')" - 7)(x'y' - 3)

x2• - x· - 30 = (x· - 6)(x" + 5)

x •••+2" + 7x"'+O + IOx'" = x"'(x2" + 7x" + 10) = x"'(x" + 2)(.>.:"+ 5)

a2(,-I) - 5a,-1 + 6 = (a,-I - 3)(arl - 2)

k)

1)

AG3x2 + IOx + 3 = (3x + l)(x + 3) d) 10s2+11s-6 = (5s - 2)(2s + 3)

= (3x + 4y)(2x - 3)')

= (5 - 3x)(2 + x)

7.2X2 - Tx + 3 = (2x - l)(x - 3) e) 6x2 - xy - 12y2

2y2 - y - 6 = (2)' + 3)(y - 2) f) 10 - x - 3x2

4z4 - 9z2 + 2 = (Z2 - 2)(4z2 - 1) = (=2 - 2)(2z + 1)(2z - 1)

16x'y + 28x2y2 - 30xy' = 2x)'(8x2 + 14xy - 15)'2)= 2xy(4x - 3y)(2x + 5)')

12(x + y)2 + 8(x + y) - 15 = [6(x + y) - 5][2(x + y) + 3] = (6x + 6y - 5)(2x + 2y + 3)

6b2"+1 + 5b"+1 - 6b = b(6b2" + 5b" - 6) = b(2b" + 3)(3b" - 2)

SUMA O DIFERENCIA DE DOS CUBOS.

18x4.+'" _ 66x2'+"'y2 _ 24x"'y4 = 6x"'(3x4• _ 11.\,2.)'2- 4y4) = 6x"'(3x2• + y2)(X2• _ 4)'2)

= 6x"'(3x2p + y2)(X' + 2y)(x' - 2y)

64X'2)" _ 68x8y7 + 4x4y" = 4x4y' (16x8 _ 17x4y4 + y8) = 4x4y' (16x4 _ y4)(X4 _ y4)

= 4x4y' (4x2 + y2)(4x2 _ y2)(X2 + y2)(X2 _ y2)

= 4x4y' (4x2 + y2)(2x + )')(2x - y)(x2 + y2)(X + y)(x - y)

6. a) x' + 8 = x' + 2' = (x + 2)(x2 - 2x + 22) = (x + 2)(x2 - 2x + 4)

b) a' - 27 = a' - 3' = (a - 3)(a2 + 3a + 32) = (a - 3)(a2 + 3a + 9)

e) a6 + b6 = (a2)' + (b2)' = (a2 + b2)[(a2)2 _ a2b2 + (b2)2]

= (a2 + b2)(a4 _ a2b2 + b4)

ti) a6 - b6 = (a' + b')(a' - b') = (a + h)(a2 - ab + b2)(a - b)(a2 + ab + b2)

e) aO + b9 = (a')' + (b')' = (a' + h')[(a')2 - a'b' + (h')2]

= (ti + b)(a2- ab + b2)(a6

- a'h' + b6)

f) a'2 + b'? = (a4)' + (b4)' = (a4 + b4)(a8 _ a4h4 + b8)

g) 64x' + 125v' = (4x)' + (5y)' = (4x + 5y)[(4x)2 - (4x)(5y) + (5yf]

= (4x + 5y)(16x2 - 20xy + 25y2)

h) (x + y)' - z' = (x + y - z)[(x + y)2 + (x + y)z + Z2]

= (x + y - z)(x2 + 2xy + .1'2 + x= + )'Z + Z2)

i) (x - 2)' + 8y' = (x - 2)' + (2y)3 = (x - 2 + 2)')[(x - 2)2 - (x - 2)(2)') + (2y)2]

= (x - 2 + 2y)(x2 - 4x + 4 - 2x)' + 4)' + 4)'2)

FA

8.

j) x6 - 7x3 - 8 = (x' - 8 )(x' + 1)

= (x' - 2' )(x' + 1) = (x - 2)(x2 + 2x -~ 4)(x + 1)(x2 - X + 1)

http://carlos2524.jimdo.com/

Page 39: Algebra Superior Murray R Spiegel

OESCOMPOSICION EN FACTORES

k) X8y - 64X2y7 = x2Y(X6 - 64y6) = x2Y(X3 + 8y3)(X3 _ 8y3) = X2)'[X3 + (2y)3][X3 _ (2y)3]

= x2y(x + 2v)(x2 - 2xy + 4y2)(X -- 2y)(x2 + 2x)' + 4y2)

1) 54x·y2 - 38x3y2 - 16y' = 2y2(27x· - 19x3 - 8) = 2y2 (27x3 + 8)(x3 - 1)

= 2)'2 [(3X)3 + 23](X3 - 1) = 2y2 (3x + 2)(9x2 - 6x + 4)(x - l)(x2 + X + 1)

AGRUPAMIENTO DE TERMINOS_ Tipo: ac + be + ad + bd = c(a + b) + dia + b) = (a + b)(e + d)

7. a) bx - ab + x2 - ax = b(x - a) + xIx - a) = (x - a)(b + x) = (x - a)(x + b)

b) 3ax - ay - 3bx + by = a(3x - y) - b(3x - y) = (3x - y)(a - b)

e) 6x2 - 4ax - 9bx + 6ab = 2x(3x - 2a) - 3b(3x - 2a) = (3x - 2a)(2x - 3b)

d) ax + ay + x + y = a(x + y) + (x + y) = (x + y)(a + 1)

e) x2 - 4y2 + X + 2y = (x + 2y)(x - 2y) + (x + 2.1') = (x + 2y)(x - 2y + 1)

f) x3 + x2y + xy2 + y3 = x2 (x + y) + y2 (x + y) = (x + )')(x2 + y2)

g) x7 + 27x4- x3

- 27 = x4 (x3 + 27) - (x3 + 27) = (x3 + 27)(x4 - 1)

= (x3 + 33)(X2 + l)(x2 - 1) = (x + 3)(x2 - 3x + 9)(x2 + I)(x + I)(x - 1)

11) X3y3 - y3 + 8x3 _ 8 = y3(X3 _ 1) + 8(x3 _ 1) = (x3 _ l)(y3 + 8)

= (x - 1)(x2 + X + I)(y + 2 )(y2 - 2y + 4)

i) a· + b6 - a2b4 - a4b2 = a6 _ a2h4 + b· _ a4b2 = a2(a4 _ b4) _ b2 (a4 _ b4)

= (a4- h4)(a2 _ h2) = (a' + b2)(a2 -- b2)(a + b)(a - b)

= (a2 + b2)(a + h)(a - b)(a + h)(a - b) = (a2 + b2)(a + b)2 (a _ b)2

j) a3 + 3a2 - 5ab + 2b2 - b3 = (a3 - h3) + (3a2 - 5ab + 2b2)

= (a - b)(a2 + ab + b2) + (a - b)(3a - 2b)

= (a - b)(a2 + ab + b2 + 3a - 2b)

FACTORES DE a" ± bn

8. a" + b" tiene a + b como factor (o divisor) solamente cuando n es un número entero, impar y positivo. En estascondiciones,

a) a3 + b3 = (a + b)(a2 - ab + b2)

b) 64 + y3 = 43 + y3 = (4 + y)W - 4y + y2) = (4 + y)(16 _ 4y + y2)

e) x3 + 8y6 = x3 + (2y2)3 = (x + 2y2)[X2 _ X(2)'2) + (2y2)2]

= (x + 2y2)(X2 - 2xy2 + 4y4)

d) aS + b5 = (a + b)(a4 - a'b + a2b2 _ ab3 + b4)

e) 1 + X5)'5 = l' + (xy)s = (1 + xy)(1 _ x)' + x2y2 _ X3y3 + x4y4)

f) Z5 + 32 = Z5 + 2' = (z + 2)(Z4 -2z3 + 22Z2 _ 23Z + 24)

= (z + 2)(Z4 - 2z3 + 4z2 - 8z + 16)

g) a10 + x10 = (a2)5 + (x2

)' = (a2 + x2)[(a2)4 _ (a')3x2 + (a2)2 (X2)2 _ (a2)(x2)3 + (X2)4]

= (a2 + x2)(a8 _ a6x2 + a4x4 _ a2x6 + x8)

h) u7 + v7 = (u + v)(u· - U'¡, + U4V2 _ U3V3 + U2V4 _ uv' + v6)

i) x9 + 1 = (X3)3 + 13 = (x3 + 1)(x6 - x3 + 1) = (x + 1)(x2 - X + 1)(x· _ x3 + 1)

31OESCOMPOSICION EN FACTORES

k) X8y _ 64X2y 7 = x 2y(x· - 64y 6) = x 2Y(X3 + 8y 3)(X3 _ 8y 3) = X2)'[X3 + (2y)3][X3 _ (2.1')3]

= x 2y(x + 2v)(x2 - 2xy + 4y2)(X -- 2y)(x2 + 2.\)' + 4y2)

1) 54x 6y 2 - 38x 3y 2 - 16y' = 2y2(27x 6 - 19x 3 - 8) = 2y 2 (27x 3 + 8)(x3 - 1)

= 2)'2 [(3X)3 + 23](X3 - 1) = 2y 2 (3x + 2)(9x2 - 6x + 4)(x - l)(x 2 + X + 1)

31

AGRUPAMIENTO DE TERMINOS. Tipo : ae + be + (Id + bd = c(a + b) + d(a + b) = (a + b)(c + d)

7. a) bx - ah + X2 - ax = h(x - a) + xIx - a) = (x - a)(h + x) = (x - a)(x + b)

b) 3ax - ay - 3bx + by = a(3x - y) - h(3x - y) = (3x - y)(a - h)

e) 6X2 - 4ax - 9bx + 6ah = 2x(3x - 2a) - 3b(3x - 2a) = (3x - 2a)(2x - 3b)

d) ax + ay -t- x + y = a(x + y) + (x + y) = (x + y)(a + 1)

e) X2 - 4.1'2 + X + 2y = (x + 2y)(x - 2y) + (x + 2.1') = (x + 2y)(x - 2y + 1)

f) x 3 + x2y + xy2 + y3 = X2 (x + y) + y2 (x + y) = (x + y)(x2 + y2)

g) x 7 + 27x4 - x 3 - 27 = x 4 (x3 + 27) - (x3 + 27) = (x) + 27)(x4 - 1)

= (x) + 3))(x2 + l)(x2 - 1) = (x + 3)(x2 - 3x + 9)(x2 + I)(x + I)(x - 1)

h) x)y) - y3 + 8x) - 8 = y3(X) - 1) + 8(x) - 1) = (x) - 1 )(y) + 8)

= (x - 1 )(x2 + X + I)(y + 2 )(y2 - 2y + 4)

i) (16 + b. - a2b4 _ a4b2 = a6 _ a2b4 + b6 _ a4b2 = a2(a4 _ b4) _ b2 (a4 _ b4)

= (a4 _ h4)(a2 - b2) = (a2 + b2)(a2 -- b2)(a + b)(a - b)

= (a2 + b2)(a + b)(a - b)(a + b)(a - b) = (a2 + b2)(a + b)2 (a - b)'

j) a) + 3a2 - 5ab + 2b2 - b3 = (a) - b)) + (3a2 - 5ab + 2b2)

FACTORES DE a" ± b"

= (a - b)(a2 + ab + b2) + (a - b)(3a - 2b)

= (a - b)(a2 + ab + b2 + 3a - 2b)

8. a" + b" tiene a + b como factor (o divisor) solamente cuando n es un número entero, impar y positivo . En estas condiciones.

a) a) + b) = (a + b)(a2 - ab + b2)

b) 64 + y) = 4) + y) = (4 + y)W - 4y + y2) = (4 + y )(16 - 4y + y2)

e) x) + 8y 6 = x 3 + (2y 2)) = (x + 2y2)[X2 _ X(2y 2) + (2.1'2)2]

= (x + 2y2)(X2 - 2xy2 + 4y4)

d) a5 + bS = (a + b)(a4 - a)b + a2b2 - ab) + b4)

e) 1 + XSys = 1 s + (xy)s = (\ + xy)(1 _ xy + X2),2 _ x)y) + x 4y4)

.n ZS + 32 = ZS + 25 = (z + 2)(Z4 - 2z) + 22Z2 - 23Z + 24)

= (z + 2)(Z4 - 2z3 + 4z2 - 8z + 16)

g) a 10 + x 10 = (a2)s + (x2)s = (a2 + x 2)[(a2)4 _ (a'»)x2 + (a2)2 (X2)2 _ (a2)(x2)3 + (X2)4]

= (al + X2)((18 _ a·x2 + a4x 4 _ a2x 6 + x 8)

h) u7 + v7 = (u + v)(u6 - uSt' + U41,2 _ u3v) + U2V4 _ uvs + v6)

i) x 9 + 1 = (x3)) + 1) = (x) + 1 )(x6 - x 3 + 1) = (x + 1 )(x2 - X + 1 )(x6 _ x) + 1)

http://carlos2524.jimdo.com/

Page 40: Algebra Superior Murray R Spiegel

32 DESCOMPOS1C10N EN FACTORES

9. a" - b" tiene a - b como factor (o divisor) si 11 es un número entero y positivo. En estas condiciones,

a" - h" = (a - b)(a"-1 + a"-2b + a"-3b2 + ... + ab"-2 + b"-I)

Si 11 es un entero par y positivo. a" - b" también tiene a + b como factor.

a) a2 - b2 = (a - b)(a + b)

b) a3 - b3 = (a - b )(a2 + ah + b2)

e) 27x3 _ y3 = (3X)3 _ y3 = (3x _ y)[(3X)2 + (3x)y + y2] = (3x - y)(9x2 + 3xy + y2)

d) 1- x3 = (1 - x)(12 + Ix + X2) = (1 - x)(l + x + X2)

e) aS _ 32 = aS - 2s = (a - 2)(a4 + a3 • 2 + a2 . 22 + a' 23 + 24)

= (a - 2)(a4 + 2a3 + 4é + 8a + 16)

f) y' _ z' = (y _ Z)U,6 + ySz + y4z2 + ),3Z3 + y2z4 + yzs + Z6)

g) x6 _ a6 = (x3 + a3)(x3 _ a3) = (x + a)(x2 - ax + a2)(x - a)(x2 + ax + a2)

h) u· - v. = (u4 + V4)(U4 - v4) = (u4 + ¡,4)(U2 + V2)(U2 - v2)

= (u4 + (4)(u2 + v2)(u + v)(u - 11)

i) x9 _ I = (X3)3 _ I = (x3 - I )(x6 + x3 + 1) = (x - I )(x2 + X + I )(x6 + x3 + 1)

j) x10 _ yl0 = (xs + yS)(xs _ yS)

= (x + y)(x4 _ x3y + x2y2 _ xy3 + ),4)(X _ y)(x4 + xly + x2y2 + xy3 + y4)

MA:li

12. ,

SUMA Y RESTA DE TERMINOS

(sumando y restando a2b2)

= (a4 + 2a2b2 + b4) _ a2b2 = (a2 + b2)2 - (ab)2

= (a2 + b2 + ab)(a2 + b2 - ab)

(sumando y restando 9x2)

= (36x4 + 24x2 + 4) - 9x2 = (6x2 + 2f - (3X)2

= [(6x2 + 2) + 3x][(6x2 + 2) - 3x] = (6x2 + 3x + 2)(6x2 - 3x + 2)

(sumando y restando 16x2y2)

= (64x4 + 16x2y2 + y4) _ 16x2)'2 = (8x2 + y2)2 _ (4xy)2

= (8x2 + y2 + 4xy)(8x2 + y2 - 4xy)

b) 36x4 + 15x2 + 4

(sumando y restando 4u4)

= (u. - IOu4 + 25) - 4u4 = (u4 - 5)2 - (2U2)2

= (u4 _ 5 + 2u2)(u· - 5 - 2u2) = (u· + 2u2 - 5)(u4 - 2u2 - 5)

Deseo

13. a

b

e

d

e

fg

14. a

b

e

diJ

15. a)

b)

e)

PROBLEMAS DIVERSOS

n. a) x2 - 4z2 + 9y2 - 6xy = (x2 - 6xy + 9y2) - 4:2

= (x - 3y)2 - (2=)2 = (x - 3y + 2=)(x - 3y - 2=)

b) 1602 + IObe - 25e2 - b2 = 16a2 - (b2 - IObe + 25e2)

= (4a)2 - (b - 5e)2 = (4a + b - 5e)(4o - b + 5e)

e) x2 + 7x + y2 - 7y - 2xy - 8 = (x2 - 2xy + .1'2) + 7(x - y) - 8

= (x - y)2 + 7(x - y) - 8 = (x - y + 8)(x - y - 1)

http://carlos2524.jimdo.com/

Page 41: Algebra Superior Murray R Spiegel

DESCOMPOSICION EN FACTORES

d) a2 - 8ab - 2ae + 16b2 + 8be - 15e2 = (a2 - 8ab + 16b2) _ (2ae _ 8be) _ 15e2

= (a - 4W - 2e(a - 4b) - 15e2 = (a - 4b - 5e)(a - 4b + 3e)e) m4

- n4 + m3 - mn" - n3 + m3n = (m4 _ mn3) + (m3n _ n4) + (m3 _ n3)

= m(m3 - n3) + n(m3 _ n3) + (m3 _ n3)

= (m3 - n3)(m + n + 1) = (m - n)(m2 + mn + n2)(m + n + 1)

MAXIMO COMUN DIVISOR y MINIMO COMUN MULTIPLO

12. a) 9x4y2 = 32x4y2, 12x3y3 = 22• 3X3y3

M.C.D. = 3X3y2, M.C.M. = 22• 32x4y3 = 36x4y3

b) 48,3(4 = 24. 3,3(\ 54~(6 = 2· 33,2(6, 60,4(2 = 22• 3 • 5,4(2

M.C.D. = 2· 3,2(2 = 6,2(2, M.C.M. = 24• 33• 5,4(6 = 2160,4(6

e) 6x - 6y = 2· 3(x - y), 4x2 - 4y2 = 22 (x2 _ y2) = 22 (x + y)(x _ y)

M.C.D. = 2(x - y), M.C.M. = 22 • 3(x + y)(x - y)

d) y4 - 16 = (y2 + 4)(y + 2)(y - 2), y2 - 4 = (y + 2)(y _ 2), y2 - 3y + 2 = (y _ I)(y _ 2)

M.C.D. = y - 2, M.C.M. = (y2 + 4)(y + 2)(y - 2)(y _ 1)

e) 3' 52 (x + 3y)2 (2x - y)4, 23. 32 • 5(x + 3y)3 (2x _ y)2, 22• 3 • 5(x + 3y)4 (2x _ y)5

M.C.D. = 3· 5(x + 3y)2 (2x - y)2, M.C.M. = 23• 32• 52 (x + 3y)4 (2x _ y)5

PROBLEMAS PROPUESTOSDescomponer en factores.

13. a) 3x2y4 + 6X3y3 h) 18x3y - 8xy3 o) 3a4 + 6a2b2 + 3b4b) 12s2(2 - 6SS(4 + 4S4( i) (2x + y)2 - (3y _ Z)2 p) (m2 - n2)2 + 8(m2 _ n2) + 16e) 2x2yz - 4xyz2 + 8xy2z3 j) 4(x + 3y)2 - 9(2x _ y)2 q) x2 + 7x + 12d) 4y2 - 100 k) x2 + 4x + 4 r) y2 _ 4y - 5e) 1 - a4 1) 4 - 12y + 9y2 s) x2 - 8xy + 15y2f) 64x - x3 m) x2y2 - 8xy + 16 () 2z3 + IOz2 - 28zg) 8x4 - 128 n) 4x3y + 12x2y2 + 9xy3 u) 15 + 2x - x2

14. a) m" - 4m2 - 21 e) 2X2 + 3x + 1 i) 36z6 - I3z4 + Z2

b) a4 - 20a2 + 64 f) 3y2 - Ily + 6 j) 12(x - y)2 + 7(x - y) - 12e) 4s4

í - 4s3(2 _ 24s2(3 g) 5m3 - 3m2 - 2m k) 4x2.+2 _ 4x"+2 _ 3x2

d) x2m+4 + 5x'"+4 _ 50x4 h) 6x2 + 5xy - 6y2

15. a) y3 + 27 d) 8z4 _ 27z7 g) y6 + 1b) x3 _ 1 e) 8x4y +; 64xy4 h) (x _ 2)3 + (y + 1)3e) X3y3 + 8 f) m9 _ n9 i) 8x6 + 7x3 _ 1

33

http://carlos2524.jimdo.com/

Page 42: Algebra Superior Murray R Spiegel

34 DESCOMPOSICION EN FACTORES

16. a) xy + 3y - 2x - 6 e) ax' + bx - ax - b e) z 7 - 2z6 + Z4 - 2z3

h) Lpr - po5 + 6qr - 3qo5 d) x3 _ xy2 _ x2y + y3 f) m3 _ mn2 + m2n _ n.l + m2 - /l"

17. a) ZS + 1 b) x' + 32y' e) 32 - US d) m'" _ 1 e) 1 - Z7

18. a) z· + 64 e) x' - 12x4 + 16 e) 6ab + 4 - a2 - 9h2

b) 4x. + 3x2y2 + y4 d) m2 _ 4p2 + 4mn + 4n2 f) 9x2 _ X2)'2 + 4y2 + 12xy

g) x2 + y2 _ 4z2 + 2xy + 3xz + 3yz

19. Hallar el M.C.O. y el M.C.M. de los polinomios siguientes:

a) 16y2z·, 24y3z2

b) 9r3052(S. 12,2S·(3, 21,s52

e) x2 - 3xy + 2y2, 4x2 - 16xy + 16y2

d) 6y) + 12y2z. 6y2 - 24z2. 4y2 - 4yz - 24z2

FI

SOLUCIONES DE LOS PROBLEMAS PROPUESTOS

13. a) 3x2y) (y + 2x) h) 2S2((6( - 353(3 + 2052) e) 2xyz(x - 2z + 4yz2) d) 4(y + 5)(y - 5)

e) (1 + a2)(1 + a)(1 - a) f) x(8 + x)(8 - x) g) 8(x2 + 4)(x + 2)(x - 2)/¡) 2xy(3x + 2y)(3x - 2y) i) (2x + 4y - z)(2x - 2y + z) j) (8x + 3y)(9y - 4x)k) (x + 2)2 1) (2 - 3y)2 m) (xy - 4)2 n) xy(2x + 3y)2 o) 3(a2 + b2¡Ip) (m2 _ n2 + 4)2 q) (x + 3)(x + 4) r] (y - 5)(y + 1) s) (x - 3y)(x - 5y)

1) 2z(z + 7l(z - 2) u) (5 - x)(3 + x)

u

14. a) (m2 - 7)(m2 + 3) h) (a + 2)(a - 2)(a + 4)(a - 4) e) 4S21(5 - 31)(05 + 2()d) x· (x" - 5)(x"' + 10) e) (2x + I)(x + 1) f) (3y - 2)(y - 3)¡¡) m(5m + 2)(m - 1) h) (2x + 3y)(3x - 2y) i) Z2 (22 + 1)(2z - 1)(3z + 1)(3z - 1)j) (4x - 4y - 3)(3x - 3y + 4) k) x2 (2x· + 1)(2x· - 3)

15. a) (y + 3)(y2 - 3y + 9) h) (x - l)(x2 + X + 1) e) (x)' + 2)(X2y2 - 2x)' + 4)d) z· (2 - 3z)(4 + 6z + 9z2) e) 8xy(x - 2y)(x2 + 2x)' + 4y2)f) (m - n)(m2 + mn + n2)(m6 + m3,,3 + n6) g) (y2 + 1)(y4 - )'2 + 1)h) (x + y - 1)(x2 - xy + y2 - 5x + 4y + 7) i) (2x - 1)(4x2 + 2x + I)(x + 1)(x2 - X + 1)

16. a) (x + 3)(y - 2) hj (2, - s)(p + 3q) e) (ax + b)(x - 1)e) z'(z-2)(z+I)(z2-z+l) f) (m+n)(m-n)(m+n+l)

d) (x - .1')2 (x + y)

17. a) (2 + J )(Z4 - z' + Z2 - Z + 1) h) (x + 2y)(x4- 2x3y + 4x2y2 - 8xy' + 16y4)

e) (2-u)(16+8u+4//2+2u'+u4) d) (m+l)(m4_m3+m2_m+l)(m_l)(m4+m3+m2+m+l)el (1 - z)(I + z + Z2 + Z3 + Z4 + Z5 + Z6)

18. a) (z2+4z+8)(Z2_4z+8)

ti) (m + 2" + 2p)(m + 2n - 2p)

If) (x + y + 4zJ(x + Y - z)

h) (2x2 + xy + y2)(2x2 - xy + y2)

el (2 + a - 3b)(2 - a + 3b)

e) (x4 + 2X2 - 4)(x4 - 2X2 - 4)f) (3x + xy + 2)')(3.\ - xy + 2y)

SI

19. a) M.C.O. = 2'/Z2 = 8),2Z2.b) M.C.D. = 3,.2S2.e) M.C.O. = x - 2)'.ti) M.C.O. = 2(y + 2z),e) M.C.O. = xIx - 1).

M.C.M. = 24• 3Y)Z4 = 48y3=4M.C.M. = 252,.5S41'M.C.M. = 4(x - y)(x - 2y)2

M.C.M. = 12y2 (y + 2z)(y - 2z)lv - 3z)M.C.M. = x) (x + I)(x - l)(x2 + J)(x2 + X + 1)

TF

http://carlos2524.jimdo.com/

Page 43: Algebra Superior Murray R Spiegel

xy

CAPITULO 5

Fracciones

FRACCION ALGEBRAICA RACIONAL. Es una expresión que se puede escribir como cocien-te de dos polinomios PIQ. El polinomio P es el numerador y Q el denominador de la fracción.

5)(y - 5)

3x - 4 x3 + 2y2Por ejemplo, x2 _ 6x-+8 y x4 _ 3xy + 2y3 son fracciones algebraicas racionales.

1)

LAS REGLAS para el cálculo con fracciones algebraicas son las mismas que las correspondientesde las fracciones en aritmética. Una de las fundamentales es: El valor de una fracción no se alterasi se multiplican, o dividen, el numerador y el denominador por una misma cantidad, siempre queésta sea distinta de cero. En estas condiciones las fracciones se llaman equivalentes.

Por ejemplo, si se multiplica el numerador y denominador de :: ~ ~ por (x - 1), se obtie-

l f . , . l (x + 2 )(x - 1) x2 + X - 2 . ( dne a raccion equiva ente (x _ 3 )(x _ 1) = x2 _ 4x + 3 siempre que x - 1) sea istinto de

cero, es decir, x 1= 1.x2 + 3x + 2 (x + 2)(x + 1)

Análogarnente, la fracción 2 4 3 se puede expresar por ( 3)( 1) Y dividir,x + x+ x+ x+entonces, su numerador y denominador por (x + 1), siempre que (x + 1) sea distinto de cero, o

bien, x 1= 1, obteniéndose x + 23' La operación de dividir por un factor común al numeradorx+

y denominador recibe el nombre de simplificación y se indica tachando el término común; por. (x+2)(~) (x+2)

ejemplo, (x + 3)(~) = (x + 3)''+m+ 1)

-2x'-4)- x)' + 2)')

SIMPLIFICAR una fracción es transformarla en otra equivalente cuyo numerador y denominadorno tengan más factores comunes que la unidad, ± l. La fracción que resulta es irreducible. Estareducción se lleva a cabo descomponiendo en factores el numerador y el denominador, simplifican-do, seguidamente, los factores comunes siempre que sean distintos de cero.

. x2 - 4xy + 3y2 (x - 3y)(x.---y) x - 3yPor ejemplo, 2 2 siempre que (x - y) 1= O

x - y (x + y)(.x--:r) x + y

TRES SIGNOS están asociados a una fracción: el correspondiente al numerador, el del denomi-nador y el de la fracción. Se pueden alterar dos cualesquiera de ellos, simultáneamente, sin quevaríe el valor de la fracción. Si a una fracción no se le antepone signo alguno, se sobrentiende queéste es positivo (más).

35

CAPITULO 5

Fracciones

FRACCION ALGEBRAICA RACIONAL. Es una expresión que se puede escribir como cocien­te de dos polinomios PIQ. El polinomio P es el numerador y Q el denominador de la fracción .

3x - 4 x 3 + 2y 2 Por ejemplo, ~.2 _ 6x-+8' y x4 _ 3xy + 2y 3 son fracciones algebraicas racionales.

LAS REGLAS para el cálculo con fracciones algebraicas son las mismas que las correspondientes de las fracciones en aritmética. Una de las fundamentales es: El valor de una fracción no se altera si se multiplican, o dividen , el numerador y el denominador por una misma cantidad, siempre que ésta sea distinta de cero. En estas condiciones las fracciones se llaman equivalentes.

Por ejemplo, si se multiplica el numerador y denominador de :: ~ ~ por (x - 1), se obtie-

I f . , . I (x + 2 )(x - 1) X2 + X - 2 . ( d

ne a racclOn eqUlva ente (x _ 3)(x _ 1) = X2 _ 4x + 3 siempre que x - 1) sea· istinto de

cero, es decir, x =1= 1. X2 + 3x + 2 (x + 2)(x + 1)

Análogamente, la fracción 2 4 3 se puede expresar por ( 3)( 1) Y dividir, x + x+ x+ x+

entonces, su numerador y denominador por (x + 1), siempre que (x + 1) sea distinto de cero, o

bien, x =1= 1, obteniéndose x + 23' La operación de dividir por un factor común al numerador x+

y denominador recibe el nombre de simplificación y se indica tachando el término común; por . (x+2)(~) (x+2)

ejemplo, (x + 3)(~) = (x + 3)"

SIMPLIFICAR una fracción es transformarla en otra equivalente cuyo numerador y denominador no tengan más factores comunes que la unidad , ± 1. La fracción que resulta es irreducible. Esta reducción se \leva a cabo descomponiendo en factores el numerador y el denominador, simplifican­do, seguidamente, los factores comunes siempre que sean distintos de cero.

. X2 - 4xy + 3y2 (x - 3y)(x.---y) x - 3y Por ejemplo, 2 2 siempre que (x - y) =1= O

x - y (x + y )(.x--:r) x + y

TRES SIGNOS están asociados a una fracción : el correspondiente al numerador, el del denomi­nador y el de la fracción . Se pueden alterar dos cualesquiera de ellos, simultáneamente, sin que varíe el valor de la fracción. Si a una fracción no se le antepone signo alguno, se sobrentiende que éste es positivo (más).

35

http://carlos2524.jimdo.com/

Page 44: Algebra Superior Murray R Spiegel

36 FRACCIONES

-a a a -a a -a aEjemplos. b = -b = - b' -b b' -(-b) = b

EL coder

Muchas veces la simplificación consiste en un cambio de signo. Por ejemplo,

_x_2..".-_3_x_+_2= (x - 2)(x - 1) = (x - 2)(x - 1) = x_-_l = 1 _ x2 - x 2 - x - (x - 2) -1

LA SUMA ALGEBRAICA DE FRACCIONES que tienen el mismo denominador es otra fraccióncuyo numerador es la suma algebraica de los numeradores de las fracciones dadas, y cuyo deno-minador es el denominador común.

UNA Fen1)2)

3 4 2 1 3 - 4 - 2 + 1 -2 2Ejemplos. 5-5-5+5= 5 =T=-5

2 3x + 4 x2 + 5 2 - (3x + 4) + (x2 + 5)-x---3 - -x---3- + -x---3 = ----''---x---'---;;3:-'-----'- x2 - 3x + 3x-3

PARA SUMAR Y RESTAR FRACCIONES de distinto denominador, se transforman éstas enotras equivalentes que tengan un denominador común.

El denominador común mínimo (D.C.M.) de varias fracciones es el mínimo común múltiplo(M.C.M.) de sus denominadores.

REDUO

Por ejemplo, el D.C.M. de i,~,170es el M.C.M. de 4, 5, 10 que es 20, y1. a)

2 3 xel D.C.M. de ?' 2J( "1 es 14x2 b)

3 4 7 15 16 14 15 - 16 + 14 13Ejemplos. ¡- 5 + 10 = 20 - 20 + 20 = 20 20

e) -

2 3 x 2(14) - 3(7x) - x(2x2)

?- 2x - "1 = 14x228 - 21x - 2X3

14x2

f) -2x + 1

x(x + 2)3 (2x + 1)(x - 1) - 3x

x(x + 2)(x - 1)2X2 - 4x - 1

x(x + 2)(x - 1)(x+2)(x-l)

g) -

EL PRODUCTO de dos o más fracciones es otra fracción cuyo numerador es el producto de losnumeradores, y cuyo denominador es el producto de los denominadores.

. 2 4 15 2· 4 . 15 1Ejemplos. 3". 5 . 16 = 3 . 5 . 16 2"

h) -

x2 - 9 x - 5 (x + 3)(x - 3)x2 - 6x + 5 . x + 3 = (x - 5)(x - 1)

x - 5x + 3

i)

(x--r3)(x - 3)(~) x - 3= (.x.---5)(x - 1)(..v-rJ) = x - 1 j) -

36 FRACCIONES

-a a a -a a -a a Ejemplos. b = -b = - b' -b = b' -(-b) = b

Muchas veces la simplificación consiste en un cambio de signo. Por ejemplo,

Xl - 3x + 2 = (x - 2)(x - 1) = (x - 2)(x - 1) = x_-_l = 1 _ x --;;-2---x - 2 - x -(x - 2) -1

LA SUMA ALGEBRAICA DE FRACCIONES que tienen el mismo denominador es otra fracción cuyo numerador es la suma algebraica de los numeradores de las fracciones dadas, y cuyo deno­minador es el denominador común.

2 3x + 4 Xl + 5 2 - (3x + 4) + (Xl + 5) -x---3 - -x---3- + -x---3 = ---'.--x----'-----,3~--'--...:.

Xl - 3x + 3 x-3

PARA SUMAR Y RESTAR FRACCIONES de distinto denominador, se transforman éstas en otras equivalentes que tengan un denominador común.

El denominador común mínimo (D.C.M.) de varias fracciones es el mínimo común múltiplo (M.C.M.) de sus denominadores.

Por ejemplo, el D.C.M. de ~, ~, 170 es el M.C.M. de 4, 5, 10 que es 20, y

2 3 x el D.C.M. de ?' 2:i 7 es 14xl

3 4 7 15 16 14 15 - 16 + 14 13 Ejemplos. ¡ - "5 + 10 = 20 - 20 + 20 = 20 20

2 3 x 2(14) - 3(7x) - x(2x2 ) 28 - 21x - 2X 3

? - 2x - 7 = 14x2 14xl

2x + 1 x(x + 2)

3 (x+2)(x-l)

(2x + 1 )(x - 1) - 3x x(x + 2)(x - 1)

2X2 - 4x - 1 x(x + 2)(x - 1)

EL PRODUCTO de dos o más fracciones es otra fracción cuyo numerador es el producto de los numeradores, y cuyo denominador es el producto de los denominadores.

Ejemplos. ~ 4 "5

15 16

2 . 4 . 15 3 . 5 . 16

1 2"

X2 - 9 x - 5 (x + 3)(x - 3) x - 5 X2 - 6x + 5 . x + 3 = (x - 5)(x - 1) . x + 3

(.v-r3)(x - 3)(~) x - 3 = (x---S")(x - 1 )(~) = x - 1

http://carlos2524.jimdo.com/

Page 45: Algebra Superior Murray R Spiegel

no-

en

lo

os

FRACCIONES 37

EL COCIENTE de dos fracciones es otra fracción que se obtiene multiplicando la fracción divi-dendo (o fracción numerador) por el recíproco de la fracción divisor (o fracción denominador).

Ejemplos. ~ -i- ~ o 3/8 _ ~ . ~ -1.5/4 - 8 5 - 10

7 . xy 7 x + 2 7Xl"'=-¡ -t- x + 2 - (x + 2)(x - 2) . -x-y- = xy(x - 2)

UNA FRACCION COMPUESTA es aquella que tiene una o más fracciones en el numerador oen el denominador. Para simplificarla:1) Se reducen el numerador y denominador a fracciones simples.2) Se dividen las dos fracciones que resultan.

Ejemplo.

1 x2 - 1x - x --x- x2 - 1 X x2 - 1--= ---=---' --=---= x-l1 1 x+l x x+l x+l+-x x

PROBLEMAS RESUELTOS

REDUCCION DE FRACCIONES

15x2 3'5'x'x 5x 14a3b3e2 2a1. a)12xy 3'4'x'Y 4y

e)_7a2b4e2 b

4x2y 2'2'x'x'Y 2xd)

8x - 8y 8(x..--y) 1(siendo x - y ,¡ O)b) 18xy3 = 2·9·x·y·y2 = 9j? 16x - 16y 16(x--rJ 2

(x - 3y)(x - y)

(y - x)(y + x)

(x - 3y)(x..--y)

(.:L--)')(y + x)_ x - 3y = 3y -:-~

y+x y+x

3x(2x - y)

2xy(y - 2x)

3x(~)

2xy(~)

32y

h)r3s + 3r2s + 9/'s rs(/,2 + 3r + 9) rs(r2 + 3r + 9) rs

r3 - 27 r3 _ 33 (r - 3)(r2 + 3r + 9) r - 3

i)(8xy + 4y2)2 (4y[2x + y]J2 16y2 (2.-.: + y)' 16y(2x + y)

8x3y + y4 y(8x3 + y3) y(2x + y)(4x2 - 2xy + y2) 4x2 _ 2xy + y2

X2n+ 1 _ X2/1y X2"(X - y) X2"(X - y) x"j) x"+3 _ x"y3 X"(X3 _ y3) x"(X - Y)(X2 + xy + y2) X2 + xy + y2

http://carlos2524.jimdo.com/

Page 46: Algebra Superior Murray R Spiegel

38 . FRACCIONES

MULTIPLlCACION DE FRACCIONES SUMA

9 x, - 1 9 (x + 1)(x - 1) x - 1b) 3x + 3'-6- = 3(x + 1)' 6 = -2-

4. a)

x2 - 4 2x)'e) --:ry2 x2 _ 4x + 4

(x+2)(x-2) 2xy 2(x+2)

xy2 • (x - 2¡>=y(x - 2)

b)

e)

6x - 12 y2 - 1ti) ~y + 4x' 2 _ 3x + x2

6(x - 2) . (y + I)(r - 1)4x(y+ 1) (2 - x)(1 - x)

g)

6(~)(~1)()' - 1)

4x{)l--n)(~)(1 - x)

3(y-1) 3(y-1)

2x(1 - x) = 2x(x - 1) h)

ax + ab + ex + be x2 - 2ax + a2 (a + e)(x + b) (x - a)(x - a)e) ( .)( ) '= . -'-----:--,-,

a2 _ x2 x2 + (b + a)x + ab (a - x)(a + x) (x + a)(x + b) i)

(a + e)(x + b) . (x - a)(x - a)

(x - a){a + x) (x + a)(x + b)

(a + e){x - a)

(x + a)2

(a + e)(a - x)

(x + a)2j)

k)

1)DIVISION DE FRACCIONES

5 3 5 11 553. a) 4 ~ 11= 4' 3 = 12

3x 6x2 3x 4e) 2 ~4 = 2.6x2 = :;

m)

IOxy2 5xy IOxy2 6z3 2d) --~-=--'-=4yz

3z 6z3 3z 5xy

n)

x + 2xy 2y + 1 x + 2xy 6x x(1 + 2y) 6xe) ----'---=---.---= ,---=2

3x2 . 6x 3x2 2y + 1 3x2 (2y + 1)

9 - x2 x3 - 2X2 - 3x

f) x4 + 6x3"" x2 + 7x + 69 - x2 x2 + 7x + 6

x4 + 6x3 • x3 - 2X2 - 3x

(3-x)(3+x) (x+l)(x+6) 3+x

x3 (x + 6) • x(x - 3)(x + 1) - ~

FRACC

5. a)

2X2 - 5x + 2 3 3g) 2 = (2x2 - 5x+ 2)'--- = (2x - I)(x - 2).-- = 3(x - 2)

x - J 2x - 1 2x - 1

3 d)

x2- 5x + 6

x2 + Tx - 8h) 9 - x2

64 - x2

x2 - 5x + 6 64 - x2 (x - 3)(x - 2) . (8 - x)(8 + x)

x2 + Tx - 8' 9 - x2 = (x + 8)(x - 1) (3 - x)(3 + x)

(x - 2)(8 - x)

(x - 1)(3 + x) e)

38 FRACCIONES

MUL T1PLICACION DE FRACCIONES

9 x~ - 1 9 (x + 1 )(x - 1) x - 1 b) 3x+3'-6-=3(x+I)' 6 =-2-

X2 - 4 2xy

e) ~2 ' X2 _ 4x + 4

6x - 12 y2 - 1 ti) ~y + 4x ' 2 _ 3x + X2

(x + 2)(x - 2) 2xy 2(x + 2) xy2 . (x - 2)2 = y(x - 2)

6(x - 2) • (y + l)(r - 1)

4x(y + 1) (2 - x)(I - x)

6(~)(.lYV1)()' - 1)

4x()l--Y1)(~)(1 - x)

3(y - 1) = 3(y - 1 L 2x(l - x) 2x(x - 1)

ax + ab + ex + be X2 - 2ax + a2 (a + e)(x + b) (x - a)(x - a) ) ( )( p= . -:------:-:--------;-;

e a2 _ X2 X2 + (b + a)x + ab (a - x)(a + x) (x + a)(x + b)

(a + e)(x + b) . (x - a)(x - a)

(x - a)(a + x) (x + a)(x + b)

(a + e)(x - a) (a + e)(a - x)

(x + a)2 (x + a)2

DIVISION DE FRACCIONES

5 3 5 11 55 3x 6X2 3x 4 3. a) ¡ ~ 11 = ¡ . "3 = 12 e) 2" ~ 4 = 2"' 6X2 = ~

10xy2 . 5xy IOxy2 6z3 2 d) - -""" - = - -' - = 4yz

3z 6z3 3z 5xy

x + 2xy 2y + 1 x + 2xy 6x x(l + 2y) 6x e) --- ~ -- ----'-_.- · ----2

3X2 . 6x - 3X2 2y + 1 - 3X2 (2y + 1) -

9 - X2 x 3 - 2X2 - 3x 9 - X2 X2 + 7x + 6

f) x 4 + 6x3 """ X2 + 7x + 6 x 4 + 6x3 x 3 - 2X2 - 3x

(3 - x)(3 + x) . (x + 1)(x + 6) 3 + X

x 3 (x + 6) xIx - 3)(x + 1) = - ~

2X2 - 5x + 2 3 3 g) 2 = (2x2 - 5x + 2) • --- = (2x - 1 )(x - 2) • -- = 3(x - 2)

x - J 2.-.: - 1 2x - 1 3

X2 - 5x + 6

9 - X2

X2 - 5x + 6 64 - X2 (x - 3)(x - 2) . (8 - x)(8 + x)

X2 + 7x - 8 • 9 - X2 = (x + 8)(x - 1) (3 - x)(3 + x)

(x - 2)(8 - x)

(x - 1)(3 + x)

http://carlos2524.jimdo.com/

Page 47: Algebra Superior Murray R Spiegel

x-I2

)(a - x)

+ a)2

FRACCIONES 39

SUMA Y RESTA DE FRACCIONES

d)3t2 4t2 3t2 (3) - 4t2 (1) 5t2 t2---= =-=-5 15 15 15 3

1 1 y+xe) -+-=--

x y xy

3 4 3(3y) + 4(x) 9y + 4xf) -+ -= =hYx 3)' 3xy

x 5x x(7) + 5x(2) 17xe) 6 + 2t = --4-2-- = 42

5 3 5(2x) - 3(1) 10x - 3g) 2x - 4x2 = 4x2 = ~

3a 2b 3a(a) + 2b(b) 3a2 + 2b2

h) be + -;;;;= abe = abe

i) 3t - 1 5 - 2t (3t - 1)3 + (5 - 2t)2 9t - 3 + lO - 4t 5t + 710 + -1-5- = 30 = 30 = 30

j) ~ __ 2 _ + ~ = 3x(x + 1) - 2x2 + 2(x + 1) = x2 + 5x + 2x x + 1 x2 x2 (x + 1) x2 (x + 1)

5 lO 5(x2 - 9) - 5(x - 3) + lO 5(x2 - X - 4)k) 5 - x + 3 + x2 _ 9 = . x2 _ 9 = x2 - 9

3 2 Y 3(y + 2) - 2(y - 2) - Y lO1) Y _ 2 - Y + 2 - y2 - 4 = y2 - 4 = y2 - 4

5 3 s 5 3 sm) -- - + = --- - + -----,---

2s+4 s2+3s+2 s2-s-2 2(s+2) (s+I)(s+2) (s-2)(s+l)

5(s + I)(s - 2) - 3(2)(s - 2) + s(2)(s + 2)2(s + 2)(s + l)(s - 2)

7s2 - 7s + 22(s + 2)(s + I)(s - 2)

3x - 6 2x - 5 3x2 + 3 3x - 6 2x - 5 3x2 + 3n) - --- + = - + ::-:--~--:-:-

4x2 + 12x - 16 6x2 - 6 8x2 + 40x + 32 4(x + 4)(x - 1) 6(x + I)(x - 1) 8(x + 4)(~ + 1)

(3x - 6)(6)(x + 1) - (2x - 5)(4)(x + 4) + (3x2 + 3)(3)(x - 1) 9x3 + x2 - 21x + 35

24(x + 4)(x - I)(x + 1) 24(x + 4)(x - I)(x + 1)

FRACCIONES COMPUESTAS

lO 6 60e) - = 10" - = - = 12

5/6 5 5

2 5 4 5 9"3+6 6+6 "6 9 8

d) --- = -- = - = -" - = 43 3 3 6 3- - -8 8 8

2a-b 2 1 2

f) ---;;-::¡; = a - b " a - b = (a - W

x+y

3T x+y x x+ye) x-y =3T"x-y=3x(x-y)

x

2a x + 1g) -- = 2a"-- = 2(x+ 1)

a ax+1

http://carlos2524.jimdo.com/

Page 48: Algebra Superior Murray R Spiegel

40 FRACCIONES

a + b a - b (a + W - (a - W------a - b a + b (a - b)(a + b)

h) ---;;=T- = ~(a-+--:-b'---)-+---'(-a---b:-:-)-

l+a+b a s- b

4ab(a - b)(a + b)

2aa + b

4ab a + b 2b(a - b)(a + b) . ~ = a - b

7

2 2x - 3

2(x - 3) - 2(x + h - 3)

(x + h - 3)(x - 3)h

-2h

i)x+h-3

h

(x + h - 3)(x - 3)

h-2

(x + h - 3)(x - 3)

2 Y + 21 +- ---

l' Y Y + 2 Y - 2 y - 2 3y2 + y - 2j) 3v + --' = 3y + -- = 3y + (--)(--) = 3y + -- = -'-_-c-_

- ~~ y+2 Y y+2 Y Yy-2 y-2

8.

9.

x\---

x + 1

1x+l-x =-\-=x+l

x+l x+l

k)1

1---1

1+-x

11---

x+lx

10.a a a a

1)a + b a + b b abra + b) ab

a-b+-- a - b + a2 _ b2 a- + (a- b)+--a b (a + b)(a - b) a - b

b a ab

a a ara - b)a2-ab+b2(a - W + ab

-----a-b

1m) \ - ----,-- = 1 - -------,---- = 1 - ----,,-----,-

2- 2- 1 2_2a+l3 __2a_-_l 3(2a + 1) - (2a - 1) 4a + 4

2a+l 2a+1 11.

=1- 1 = 1 _ 4a + 4 = 6a + 7 - (4a + 4) = 2a + 32(4a + 4) - (2a + 1) 6a + 7 6a + 7 6a + 7

4a + 4

PROBLEMAS PROPUESTOS

Comprobar que:

4x2 - 16 4(x + 2)d) x2 _ 2x = --x-

x(x + 2a)

a(x + 3a)

e) y2 - 5y + 6 = 3 - y4_y2 y+2

5a2 - 10abe) a _ 2b = 5a

(x2 + 4X)2f) x2+6x+8

x2 (x + 4)x+2

3a2 2b4 bi) 4b3· 9a3 = ~

http://carlos2524.jimdo.com/

Page 49: Algebra Superior Murray R Spiegel

+ 20)+ 3a)

FRACCIONES

8xyz2 9xy2z 6y7. a) 3x3y2Z· 4xzs = X2Z3

x2 - 4y2 2y2 - 2d) • ::--.;---'--------.

3xy + 3x 2y2 + xy - x22(x + 2y)(y - 1)

3x(x + y)

xy2 x2 _ y2 X + Yb) 2x _ 2y· X3y2 = 2x2

y2 _ y _ 6 y2 + 3y - 4e) y2 _ 2y + 1· 9y _ y3

(y + 2)(y + 4)y(y - 1)(y + 3)

x2 + 3x 2x2 + 2x x2 - 4x + 3e) 4x2 _ 4· x2 _ 9· x2

12

t3 + 3t2 + t + 3 8 - t3 (t + 3)(t2 + 2t + 4)f) 4t2 _ 16t + 16 . t3 + t = 4t(2 - t)

x2 _ 4y2 x2 - xy - 6y2 y(x - 2y)e) x2 + xy + y2 + xy = x(x - 3y)

9. a) 6x2

- x - 2 = (2x + 1)23x - 22x + 1

y2 _ 3y + 2y2 + 4y - 21

b) 4 _ 4y + y29_ y2

e)

ry + xy2x-yx+y

(y - l)(y + 3)

(y - 2)(y + 7)=~

x-y

1 1 x xe) --+-----=--

x+2 x-2 x2_4 x2-4

r - 1 , + 2 1 ,2 + 4, + 12f) ,2 + , _ 6 - ,2 + 4, + 3 + 3, - 6 = 3(, + 3)(, - 2)(, + 1)

3 8 3 - 16ye) 2y2-y=~

3x2 + xy) x _ x-y + yg 2X2 + 3xy + y2 y2 _ 4x2 2X2 + xy _ y2 (2x + y)(2x - y)(x + y)

X+y2 x-1 y2d) --+---1 =-

x2 x x2 h) a b e O-,----:-:--~ + + =(e - a)(a - b) (a - b)(b - e) (b - e)(e - a)

x+y11. a) -1-1 = xy

-+-x y

x+1 x-1-----d) x-1 x+1 =2

1 1x+1+x-1

xe) ---:-1-=x+y

1---x

1 +-y

e)

2yY + -y---2-----.:...-4:-- = y + 21+--

y2 _ 4

f) 2- 2 = 2x2

1__ 2_

2-~x2

41FRACCIONES

X2 _ 4y2 2y2 - 2 d) • ----=--=----~

3xy + 3x 2y2 + xy - X2

y2 _ y _ 6 y2 + 3y - 4 e) y2 _ 2y + 1· 9y _ y3

41

2(x + 2y)(y - 1)

3x(x + y)

(y + 2)(y + 4)

y(y - 1)(y + 3)

X2 + 3x 2x2 + 2x X2 - 4x + 3 e) 4X2 _ 4· X2 _ 9 X2

t3 + 3t2 + t + 3 8 - t3 (t + 3)(r + 2t + 4) f) 4t2 _ 16t + 16 . t3 + t = 4t(2 - t)

9. a) 6X2 - x - 2 = (2x + 1)2 3x - 2 2x + 1

4 5 b) 3x - 4x = 12x

3 8 3 - 16y e) 2y2 -y=~

x+y 11. a) -1 -1 = xy

-+-x Y

2+1 x

b) 2X2 + X = X2

e)

2y y + -y---2 ----=---4,- = y + 2 1+--

y2 _ 4

2

X2 _ 4y2 X2 - xy - 6y2 y(x - 2y) e) X2 + xy + y2 + xy = x(x - 3y)

y2 _ 3y + 2

y2 + 4y - 21 b) 4 _ 4y + y2

9 _ y2

(y - 1)(y + 3)

(y - 2)(y + 7)

1 1 x x e) --+-----=--

x+2 x-2 x2_4 x2-4

ry + xy2

e) x-y =~ x+y x-y

, - 1 , + 2 1 ,2 + 4, + 12 f) ,2 + , _ 6 - ,2 + 4, + 3 + 3, - 6 = 3(, + 3)(, - 2)(, + 1)

x x - y Y 3X2 + xy g) 2X2 + 3xy + y2 - y2 _ 4X2 + 2X2 + xy - y2 = (2x + y)(2x - y)(x + y)

h) a b e O .,.--.,-,---:-:- + + = (e - a)(a - b) (a - b)(b - e) (b - c)(c - a)

x+1 x-1

x-1 x+1 d) 1 1 = 2

x+1+x-1

e) x

1 =x+y 1---

x 1 +-

y

2 f) 2 - -----::---= 2X2

1 __ 2_

2 - 2. X2

http://carlos2524.jimdo.com/

Page 50: Algebra Superior Murray R Spiegel

pond

CAPITULO 6

Potenciación y radicación

ponp

POTENCIA DE EXPONENTE POSITIVO. Si n es un entero positivo, a" representa el productode n factores iguales a a. Así, pues, a4 = a • a . a • a. En la expresión a", a recibe el nombre de basey n el de exponente o índice de la potencia. a" se lee «potencia enésima de a», o bien «a a la m). Sin = 2, a2 se lee «a al cuadrado»; a3 se lee «a al cubo».

Ejemplos. x3 = X·X·X, 25 = 2'2'2'2'2 = 32, (-w = (-3)(-3)(-3) = -27

POTENCIA DE EXPONENTE ENTERO NEGATIVO. Si n es un entero positivo, por de-finición POTE

_" 1a =-a"

suponiendo a 9= O.

-4 I IEjemplos. 2 = 24 = 16'

I- = 33 ="73 -3 - ,

-2 -4-4x = -2'X

(a+b)-¡ 1(a + b)

PROPri

RADICACION. Si n es un entero positivo y a y b son tales que a" = b, por definición, a es la raíz ené-sima de b.

Si b es positivo, solamente hay un número positivo tal que a" = b. Dicho número se representapor jb y recibe el nombre de la raíz enésima principal de b.

Ejemplo 1. 116 es un número positivo que, elevado a la cuarta potencia, da lugar al nú-mero 16. Es evidente que dicho número es +2 y, por tanto, ~ = +2.

Ejemplo 2. El número - 2 elevado a la cuarta potencia también da lugar a 16.En estas con-diciones, - 2 es una raíz cuarta de 16, pues no es la raíz cuarta principal de 16.

Si b es negativo, no existe una raíz enésima positiva de b, pero sí existe una raíz enésima nega-tiva de h siempre que n sea impar. Este número negativo recibe el nombre de raíz enésima principal

de b y se representa por jb.Ejemplo 3. 1-27 es un número que, elevado al cubo (o tercera potencia), da lugar a -27.

Se ve fácilmente que dicho número es - 3 y, por tanto, 1-27 = - 3 es la raíz cúbica prin-cipal de - 27.

Ejemplo 4. Siempre que n sea par, por ejemplo, 1-16, la raíz enésima principal no sepuede representar por medio de un número real.

Nota. En matemáticas superiores se demuestra que hay exactamente n valores tales quea" = b, b 9= O, siempre que se introduzcan los números imaginarios (o complejos).

42

CAPITULO 6

Potenciación y radicación

POTENCIA DE EXPONENTE POSITIVO. Si n es un entero positivo, a" representa el producto de n factores iguales a a. Así, pues, a4 = a • a . a • a. En la expresión a", a recibe el nombre de base y n el de exponente o índice de la potencia. a" se lee «potencia enésima de a», o bien «a a la m). Si n = 2, a2 se lee «a al cuadrado»; a3 se lee «a al cubo».

Ejemplos. x 3 = X·X·X, 25 = 2'2'2'2'2 = 32, (-w = (-3)(-3)(-3) = -27

POTENCIA DE EXPONENTE ENTERO NEGATIVO. Si n es un entero positivo, por de­finición

-4 I I Ejemplos. 2 = 24 = 16'

_" 1 a =­a"

I - = 33 ="7 3 -3 - ,

suponiendo a 9= O.

-2 -4 -4x = -2'

X (a+b)-l

(a + b)

RADICACION. Si n es un entero positivo y a y b son tales que a" = b, por definición, a es la raíz ené­sima de b.

Si b es positivo, solamente hay un número positivo tal que a" = b. Dicho número se representa

por fi y recibe el nombre de la raíz enésima principal de b.

Ejemplo 1. 116 es un número positivo que, elevado a la cuarta potencia, da lugar al nú­

mero 16. Es evidente que dicho número es +2 y, por tanto, ~ = +2.

Ejemplo 2. El número - 2 elevado a la cuarta potencia también da lugar a 16. En estas con­diciones, - 2 es una raíz cuarta de 16, pues no es la raíz cuarta principal de 16.

Si b es negativo, no existe una raíz enésima positiva de b, pero sí existe una raíz enésima nega­tiva de h siempre que n sea impar. Este número negativo recibe el nombre de raíz enésima principal

de b y se representa por fi. Ejemplo 3. 1-27 es un número que, elevado al cubo (o tercera potencia), da lugar a -27.

Se ve fácilmente que dicho número es - 3 y, por tanto, 1-27 = - 3 es la raíz cúbica prin­cipal de -27.

Ejemplo 4. Siempre que n sea par, por ejemplo, 1-16, la raíz enésima principal no se puede representar por medio de un número real.

Nota. En matemáticas superiores se demuestra que hay exactamente n valores tales que a" = b, b 9= O, siempre que se introduzcan los números imaginarios (o complejos).

42

http://carlos2524.jimdo.com/

Page 51: Algebra Superior Murray R Spiegel

o

e-

a

se

e

POTENCIACION y RADICACION 43

POTENCIA DE EXPONENTE FRACCIONARIO POSITIVO. Si m y n son enteros positivos, pordefinición

ami" = .y;;; (suprimiendo a ~ O si n es par)

Ejemplos. 43/2 = J43 = J64 = 8, (27)213 = -1(27)2 = 9

POTENCIA DE EXPONENTE FRACCIONARIO NEGATIVO. Si m y n son enteros positivos,por definición

-mi" 1a = ami"

Ejemplos. 8-2/3 = _1_ = _1_ = _1_ = !82/3 .fi2 164 4'

X-5/2 = _1_ = _1_X5/2 P

POTENCIA DE EXPONENTE CERO. Por definición, aO = 1 si a4: O.

Ejemplos 10° = 1, (_3)° = 1, (ax)o = 1 (si ax 4: O)

PROPIEDADES GENERALES DE LA POTENCIACION. Si p Y q son números reales, se ve-rifica

A)

Ejemplos. 23'22 = 23+2 = 25, 5-3'57 = 5-3+7 = 54, 21/2'25/2 = 23 = 8

31/3• 31/6 = 31/3+1/6 = 31/2 = )3, 39• 3-2. 3-3 = 34 = 81

B)

Ejemplos. (24)3 = 212, (51/3)-3 = 5(1/3)(-3) = 5-1 = 1/5,

(X5)-4 = X-20, (a2/3)314 = d2/3)(3/4) = a1/2

e) a4:0

E· l 26

- 26-4 - 22 - 4jempros. 14- - -,

(x + 15)413 ~(x + 15)5/6 = (x + 15)4/3-5/6 = (x + 15)1/2 = yX + 15

D)

POTENCIACION y RADlCACION 43

POTENCIA DE EXPONENTE FRACCIONARIO POSITIVO. Si m y n son enteros positivos, por definición

(suprimiendo a ~ O si n es par)

Ejemplos. 43/2 = J43 = J64 = 8, (27)2 /3 = {/(27)2 = 9

POTENCIA DE EXPONENTE FRACCIONARIO NEGATIVO. Si m y n son enteros positivos, por definición

-m/n __ l_ a - am/n

Ejemplos. 8 - 2/3 = _1_ = _1_ = _1_ = ! 82 /3 ,ygz .,y64 4'

X- 5/2 = _1_ = _1_ X

5/2 P

POTENCIA DE EXPONENTE CERO. Por definición, aO = 1 si a=l= O.

Ejemplos 10° = 1, (_3)° = 1, (ax)o = 1 (si ax =1= O)

PROPIEDADES GENERALES DE LA POTENCIACION. Si p Y q son números reales, se ve­rifica

A)

31/3 • 31/6 = 31/3+1/6 = 31/2 = J3, B)

Ejemplos. (24)3 = 212

,

(X 5 )-4 = x- 20 ,

e)

(5 1/3)-3 = 5(1 /3)(-3) = 5- 1 = 1/5,

(a 2/ 3 )3 /4 = 0<2 /3)(3 /4) = al/2

a =1= O

Ejemplos. ;: = 26-

4 = 22 = 4, 3 -2

-4 = 3- 2 - 4 = 3- 6

3 ' X

I/2

_ 1/2-(-1) _ 3/2 x-I - X -x

(x + 15)4/3 (x + 15)5/6 = (x + 15)4/3- 5/6 = (x + 15)1 /2 = F+15

D)

(3 )- ~ - r 2 - 2 _ 1 a - a -9(i2

http://carlos2524.jimdo.com/

Page 52: Algebra Superior Murray R Spiegel

44 POTENCIACION y RADICACION

E) b=l=O

2 ( 2)- 3 - 6 9eX" )-3 = X = ~ = L? (y3)-3 y-9 X6

E

4.

PROBLEMAS RESUELTOS

EXPONENTE ENTERO POSITIVO

d) (3y)2 (2y)3 = (3y)(3y)(2y)(2y)(2y) = 72y5

e) (_ 3xy2)3 = (_ 3xy2)( - 3xy2)( - 3xy2)= _27x3y6

E

5.

4h) __ = 4x2y2

x-2y-2

i)3 -3 1 43 64

(4) = (3/4)3 = (3) = 27

J) (~)-3 = _1_ = (~)3 = ~y (X/y)3 X x3

k) (0,02)-1 = (1~)-1 = 1~ = 50

ab-4 a' a2 a31) a-2b = b'b4 b5

x2"+ 1p

m) __ = X2n+lyl-311y3.-1 6.

n)(x - 1)-2 (x + 3)-1 (2x - 4)(x + s¡J

(2x - 4) 1 (x + 5)-3 - (x - 1)2 (x + 3)

l. a) 23 = 2' 2 . 2 = 8

b) (_3)4 = (-3)(-3)(-3)(-3) = 81

EXPONENTE ENTERO NEGATIVO

-2 1 1e) -4(4) = -4(42) = - 4

2 1 2d) -2b- = -2(b2) = - b2

-2 1 1e) (-2b) = (_2b)2 = 4b2

1 5 1f) 5' 10-3 = 5(W) = 1000 = 200

g) _8_ = 8' 102 = 80010-2

EXPONENTES FRACCIONARIOS POSITIVOS

3. a) (8)2/3 = ft = ~ = 4 b) (_8)2/3 = .y(_8)2 = ~ = 4

http://carlos2524.jimdo.com/

Page 53: Algebra Superior Murray R Spiegel

POTENCIACION y RADlCACION 4S

d) (~)1/2 = IT 116 ~16= 4 e) (_~)213= V(_~)2= 31I=~

8 8 ~64 4

EXPONENTE FRACCIONARIO NEGATIVO

X-1/3 = _1_xl/3

1

y;4. a)

b) (8)-2/3 = _1- = _1_ = ~82/3 j82 4

e) (_1)-213 = _1_ = __ 1_ = 1(-1)2/3 .y( _1)2

1 1 1e) (_8)-2/3 = -- = --- =-

(_8)2/3 .y(_8)2 42/3 1f) -(n- = --=-1

12/3

g)' _(_1)-3/5 = __ 1_= 1_= __ 1_= _~= 1(_I)3/S 1(_1)3 FI -1

EXPONENTE CERO

5. a) 7° = 1, (_3)° = 1, (-2/3)° = 1 e) 4'10° = 4'1 = 4

f) (4' 10)° = (40)° = 1

g) -(1)0 = -1

h) (_1)0 = 1

b) (x - y)o = 1, si x - y *- O

e) 3xo = 3 • 1 = 3, si x *- O

d) (3x)0 = 1, si 3x *- O, es decir, si x *- O

i) (3x)0 (4y)0 = 1 • 1 = 1, si 3x 'f. O Y 4y *- O, es decir, si x *- O, Y *- O

j) -2(3x + 2y - 4)° = -2(1) = -2, si 3x + 2y - 4 *- O

(5x + 3y) 5x + 3y .k) ° = --- = 5x + 3y, SI 5x + 3y *- O

(5x + 3y) I

PROPIEDADES GENERALES DE LA POTENCIACION

6. a) aP • a" = ap+q g) 10"10-3= 10'-3 = 104

b) a3 • aS = a3 + 5 = a8 11) (4' 10-6)(2' 104) = 8· 104-6 = 8' 10-2

e) 34 • 3' = 39 i) a": a)'. a-z = tr+Y-Z

d) a"+ 1 • a"-2 = a2,.-1 j) (Jx"+Y)(x + y) = (x + y)I/2 (x + y)1 = (x + .1')3/2

e) X1/2 • xl/3 = x112 + 1/3 = XS/6 k) 101.1. 102.6= 104.3

f) XI/2. X-I/3 = XI/2-1/3 = .'(1/6 1) 10-4.1. 103.5. 10-0.1 = 10-4.1+3.5-0.1 = 10-0.1

http://carlos2524.jimdo.com/

Page 54: Algebra Superior Murray R Spiegel

46 POTENCIACION y RADlCACION

n) (~)-1 (_X_)1I2 = (_X_)-1/2 = (" + Y)I/2 =X+'Y X+y X+y x

~x + yx

10

7. a) (aP)" = apq f) (49)3/2 = (72)3/2 = 72.3/2 = 73 = 343

b) (X3)4 = X3·4 = X12 g) (r"2)-2 = 31 = 3

e) (a'"+2f = a(",+2f = a",·+2. h) (U-2¡-3 = U(-2X-3) = u6

d) (103)2 = 103.2 = 106 i) (81)3/4 = (34)3/4 = 33 = 27

e) (10-3)2 = 10-3.2 = 10-6 j) (F+Y)5 = [(x + y)I/2]5 = (x + y)5/2

aP 2/38. a) - = aP-q g) ~ = y2/3-1/3 = yl/3

a" 1'/3

b)a5

5 _ 3= a2 h)

ZI/2 _ .112_ 3/4 = Z-114;:? = a Z3/4 - "

74 (x + y)3.+ 1 .-4e) _ = 74-3 = 71 = 7 i) (x + y)2.+' = (x + y)73

2,,+ 3 8· 102 8d) P__ = p(2n+3J-<n+l) = pn+2 j) ___ = _ . 102+ 6 = 4 . 108

r" 1 2.10-6 2

102

= 102-5 =9.10-2 9

e) 10-3 k) ___ = _.10-2-4 = 3.10-6105 3.104 3

Ell

ll.

x"'+3f) '-- = x4

,x"'-1

8X2/3yl/4Z-1/2

_2X1/3y5/2Z1I2

9. a) (ab)" = arb"

http://carlos2524.jimdo.com/

Page 55: Algebra Superior Murray R Spiegel

POTENCIACION y RADlCACION 47

a aP u" + 1 am2+m10. a) (-)"=- e) (-b-¡m = ¡;.;-b bP

24 24 16 f)a2 (a2)3/2 a3

b) (3) =? = 81 (¡;¡)3/2 = (b4)l/2 = b6 (siendo a ~ 0, b +- O)

3a 3 (3a)3 27a3 2 5 125e) (4b) = (4W = 64b3 g) (-)-3 = (-)3 =_5 2 8

x2 x2"h) (~)-1/3 = (~)1/3 = ~ = ~d) (-)"=-y3 y30 26 53 5 5

i)3 8x30 8x30 13 (8x30)1/3 81/3x" 2x"

27y6 = (27y6) / = (27y6)1/3 = 271/3y2 = 3y2

1)

EJEMPLOS DIVERSOS

1 1 1 711. a) 23 + 22 + 21 + 2° + 2-1 + 2-2 + 2-3 = 8 + 4 + 2 + 1 + - + - + - = 15-

2 4 8 8

1 1 5b) 43/2 + 41/2 + 4-1/2 + 4-3/2 = 8 + 2 + - + - = 10-

2 8 8

4xOe) 2-4 = 4(1)(24) = 4· 16 = 64

d) 104 + 103 + 102 + 101 + 10° + 10-1 + 10-2 = 10.000 + 1.000 + 100 + 10 + 1 + 0,1 + 0,01

= 11.111,11

e) 3· 103 + 5 . 102 + 2 • 101 + 4· 10° = 3.524

430

= (22)30 = 2

60= 260-0 = 250

f) 20 20 20

http://carlos2524.jimdo.com/

Page 56: Algebra Superior Murray R Spiegel

48 POTENCIACION y RADICAClON

g) (0,125)1/3 (0,25)-1/2 = .y'O,125 = 0,5 = 1JO,2s 0,5

15.

12. a) Calcular 4x-2/3 + 3Xl/3 + 2xo para x = 8.

4 44' 8-213 + 3' 8113 + 2' 8° = - + 3' 81/3 + 2' 8° = - + 3' 2 + 2' 1 = 982/3 4

(-w (-2X)-3b) Calcular (x + 1)-2 para x = 2.

9( 1 )3(-w (_4)-3 -4 l-'---'-3--'---=-2-'--= -1- = 9(-'¡l' (9)=

32

8164

2°-2-21-1/221-1/43/4113. a) - - --2 - 2(2)-2 - 2 - 2/22 - 2 - 2/4 - 6/4 - 2

2 2 + a-+1 --

2a-1 +ao a a 2+ab) -2 = -- = -- = --' a' = (2 + aja = 2a + a2

a 1 1 a;;r ;;r

16.

o

d) (~)-2 _ (_3)-2 = (3)2 _ (_ ~)2 = 9 - ~ = ~3 3 9 9

1 1 1 1e) ( __ )-2/3 + (__ )2/S = (_27)2/3 + (__ )2/S = [(_3)3]2/3 + [( __ )S]2/S27 32 2s 2

1 37= (_ 3)2 + (__ )2 = _2 4

(-3a)3. 3a-2/314. a) (2a)-2 • al/3

(- 3a)3 . 3 . (2a)2a2/3 • al/3

-27a3 . 3' 4a2a2/3 +1/3 a

(xy + l xy - 1--) ... (--)"

Y Y(xy + 1r .~'CJ - ~

y" y.

17.

e)

1 1(x + -r' (x - -)"

y y

1 l(y + -1'" (y - _1"

x x(y+ I)••.(Y-I)"

x x(xy + 1)". (x)' - 1)"

xm x"

(xy + 1)'" (xy - 1)"vm+n m+n•. = '-- = (~)"+"

(xy+I)"(xy-I)" y'"+n yxm+n

3,q+q 3" 3,q+q+"d) --' - - --- - 3(,q+q+2,)-(pq+,+2q) - 3,-q

3""+' 32q - 3,q+,+2q - -

48 POTENC1AC10N y RAD1CAClON

g) (0,125)" 3 (0,25)-" 2 = ,yO,125 = 0,5 = 1 jOi5 0,5

12. a) Calcular 4X- 213 + 3x"3 + 2xo para x = 8.

4 4 4. 8- 213 + 3 ' 81/3 + 2' 8° = - + 3 ' 8" 3 + 2 ' 8° = - + 3' 2 + 2' 1 = 9

8213 4

(-3)' (-2X)-3 9( 1 )3

(-3)' (_4)-3 -4 1 b) Calcular para x = 2.

(x + 1)- 2 3- 2 = -1- = 9(- 'i (9) =

32

2°-2- 2 1-1/22 1-1/43/4 13. a) 2 _ 2(2)-2 = 2 _ 2/22 = 2 - 2í4 = 6/4 = 2

2 2 + a -+1 --

2a-'+ao a a 2+a b) _ = -- = -- = -- . a2 = (2 + aja = 2a + a2

a 2 1 1 a

~ ~

o

d ) (~) - 2 _ (_3)-2 = (J)' _ (_ ~)2 = 9 - ~ = ~ 3 3 9 9

1 1 1 1 e) ( _ _ )-213 + (- _)2IS = (_27)213 + ( _ _ )2IS = [(-3¡J]213 + [( _ _ )SJ2'S

27 32 2s 2

3)2 1 2 37

=(- +(- - ) = -2 4

(-3a)3. 3a- 213 14. a)

(2a) 2. a' ,3

(- 3a)3 • 3 . (2a)2

02' 3 • a'/3

-27a3 • 3 ' 4a2

a2 / 3 + 113 a

b) (X- 2)-3. (X-'/3)9 x6 • x- 3 X6- 3 x 3 '2 (x " 2) 3. (x 312)S - X 312 . X 1512 - x-312 'SI2 = x-9 = X

1 1 (x + - )'"' (x - -r

e) y y

1 1 (y + -)'" ' (y - -r

x x

(xy + 1)," xy - 1", -- . (--,

y y

(V + 1)," . (V - 1)" x x

(xy + 1)'" (xy - 1)"

(xv + 1)'" • ~'(y --.!L y'" y"

(xy + 1)'" . (xy - 1)"

xlrt X'

ym+n .'('"+" X --~:::---- = -- = (-)'"+. (xy + 1)'" (xy - 1)" y'"+" Y

x,"+n

3pq +. 32p 3P'+'+ 2p d) --' - = --- = 3(p·+·+2p,-(pq+p+2.' - 3P-'

3pq + p 32• 3pq + p + 2• -

81

64

http://carlos2524.jimdo.com/

Page 57: Algebra Superior Murray R Spiegel

POTENCIACION y RADICACION ~9

(X314)213 _ (ySI4)2IS Xl12 _ yl12 (XIl4)2 _ (y114)2

b) (X314)113 + (y2/3)318 = XiII, + yl14 = XiII, + yl14

(X"I, + y114)(X114 _ y"4)

XiII, + yl14

1 1 1 ~ ~ ~+~e) 1 + x. 9 + 1 + ~ • = ~ + -----.;4 = ~ + x. + x. + ~ = ~ + x. = 1

1+- 1+-~ x·

= 36 x lOs o 3.600.000

48.000.000b) 1.200

48 X 106-:-::----:-= = 4 X 106-2 = 4 X 104 o 40.00012 x 102

0,078e)

0,00012

78 X 10-3~-.,.-:-. - 65 X 1O-3+S = 6,5 X 102 o 65012 x 10 s - ,

d) (80.000.000)2 (0,000003)(600.000)(0,0002)4

e)3 (0,004)4 (0,0036)

(120.000)2

3 256(36) 10-12. 10-4~. WS

3 (4 x 10-' 3)" (36 X 10-4)

(12 X 10")2

= ,y64 x 10 24 = 4 X 10-8

17. Hallar los valores reales de las letras que figuran en las expresiones siguientes para las cuales son válidas las ope-raciones que conduzcan a números reales.

b) Ja2 + 2a + I = fo+1iI = a + 1

x-I (X-I)I ~e) --- = = (x - 1)1-1/2 = (x - 1)1/2 = VX - 1

~ (x_1)112

POTENCIACION y RADICACION

(X l /4)2/l _ (y5 /4)2/5 _ Xl /2 _ y1l2 _ (X1l4)2 _ (y1 /4)2 (X1 /4 + y1 /4)(X1 /4 _ / 14)

b) (Xl /4)1 /3 + (y2 /3)3/8 - X l /4 + yl /4 - X l /4 + yl /4 XI /4 + yl /4

1 1 1 ~ ~ ~+~

e) 1 + xp q + 1 + ~ p = ------;p + ~ = ~ + x p + xp + ~ = ~ + xp = 1 1+- 1+-

~ xp

= 36 X 105 o 3.600.000

48.000.000 b)

1.200

48 X 106

-,-,--~ = 4 X 106 - 2 = 4 X 104 o 40.000 12 x 102

0,078 78 x 10- 3

e) 0,00012 = 12 x 10- 5 = 6,5 X 10-3+5 = 6,5 X 102 o 650

d) (80.000.000f (0,000003) (600.000)(0,0002)4

e) 3 (0,004)4 (0,0036)

(120.000)2

3 (4 x 10- 3)4 (36 X 10- 4 )

(12 X lQ4)2

= ,y64 x 10 24 = 4 X 10- 8

3 256(36) . 10- 12 • 10- 4

144 108

~9

17. Hallar los valores reales de las letras que figuran en las expresiones siguientes para las cuales son válidas las ope­raciones que conduzcan a números reales.

b) Ja2 + 2a + 1 = J(a+Ti2 = a + 1

x-I (x-l)1 ~ e) - -- = = (x - 1)1 - 1/2 = (x - 1)1 /2 =.,¡x - 1 ~ (x-l)1 /2

http://carlos2524.jimdo.com/

Page 58: Algebra Superior Murray R Spiegel

50 POTENCIAClON y RADICACION

a) Siendo x un número real, p debe ser positivo o cero. Suponiendo que y/-¡i = x para todos los valoresde x. si x = - 1 tendríamos J<=I)l = - 1, o sea fi = - 1, con lo que 1 = - 1, lo cual es un absurdo. Así;pues.. J-¡i = x no es cierto para todos los valores de x. Entonces, p = x siempre que x ;¡; O. Si x ~ O,yÍxi = -x. Un resultado válido para ambos casos, x ;¡; OYx ~ O.esp = Ixl (valor absoluto de x).

h) Ja2+2-;;-¡J debe ser positivo o cero y, por tanto, será igual a a + 1 siempre que a + 1 ;¡; O, es decir.a ;¡; - L Un resultado válido para todos los valores de a viene dado por J a2 + 2a + 1 = la + 11.

19

e) a-: - h~: carece de sentido si a y b son ambos iguales a cero. Tampoco tiene sentido si el denominadora -.b

a-2 _ b-2a-I - b-' = O.es decir, si a-I = s:', o bien, a = b. Por consiguiente, el resultado _1 = a- 1 + h- 1a - b 1

solo es válido para a '" O. b '" O y a '" b.

d) #+ 2X2 + 1 debe ser positivo o cero y será igual a x2 + 1 siempre que x2 + 1 ;¡; O. Como x2 + 1 esmayor que cero para todos los números reales x, el resultado es válido para todos los valores reales de x.

x-le) fi-=I no es un número real si x - 1 < O,es decir, si x < 1. Asimismo ~ carece de sentido si el

yx-Ix-I ~

denominador es cero, es decir, si x = l. Luego, r::-t = Y x - 1 únicamente para x > l.yx-l

20

18. Interrogado un alumno sobre el resultado de la expresión x + 2y + J(x - 2y)2 para x = 2, Y = 4, hizo losiguiente:

x + 2y + J(x - 2y)2 = X + 2y + x - 2y = 2x

obteniendo el valor 2x = 2(2) = 4. ¿Es correcta la respuesta? 21

Haciendo x = 2, .1' = 4 en la expresión dada se obtiene

x + 2y + J(x - 2y)2 = 2 + 2(4) + ~ = 2 + 8 + ../36 = 2 + 8 + 6 = 16

El alumno comete la equivocación de escribir ~ 2y)2 = X - 2y, lo cual es cierto, únicamente, cuandox ;¡; 2y. Si x ~ 2y, J(x - 2y)2 = 2.1'- x. En todos los casos, J(x - 2y)2 = jx - 2yl. La operación efectua-da correctamente habrá sido .r + 2y + 2y - x = 4y, que es igual a 16 para y = 4.

22

50

18.

POTENCIAClON y RADICACION

a) Siendo x un número real, p debe ser positivo o cero. Suponiendo que .j? = x para todos los valores

de x. si x = - 1 tendríamos J<=I)l = - 1, o sea Ji = - 1, con lo que 1 = - 1, lo cual es un absurdo. Así;

pues .. /;? = x no es cierto para todos los valores de x. Entonces, p = x siempre que x ;¡; O. Si x ~ O.

'¡;i = -x. Un resultado válido para ambos casos, x ;¡; O Y x ~ O, es P = Ixl (valor absoluto de x).

h) Ja2+2-;;:¡:¡- debe ser positivo o cero y, por tanto, será igual a a + 1 siempre que a + 1 ;¡; O, es decir. a ;¡; - L Un resultado válido para todos los valores de a viene dado por J a2 + 2a + 1 = la + 11.

e) a-: - h-: carece de sentido si a y b son ambos iguales a cero. Tampoco tiene sentido si el denominadClr a - .b

a - 2 _ b- 2

a- I - b- ' = O, es decir, si a- I = h- ' , o bien, a = b. Por consiguiente, el resultado I = a - I + h- I a - h I

solo es válido para a '" O, b '" O y a '" b.

d) F+ 2X2 + 1 debe ser positivo o cero y será igual a X2 + 1 siempre que X2 + 1 ;¡; O. Como X2 + 1 es mayor que cero para todos los números reales x, el resultado es válido para todos los valores reales de x.

x-I e) F=1 no es un número real si x - 1 < O, es decir, si x < 1. Asimismo ~ carece de sentido si el

yx-1 x -I ~

denominador es cero, es decir, si x = 1. Luego, c-t = Y x - 1 únicamente para x > 1. yx -1

Interrogado un alumno sobre el resultado de la expresión x + 2y + J(x - 2y)2 para x = 2, Y = 4, hizo lo siguiente:

x + 2y + J(x - 2y)2 = X + 2y + x - 2y = 2x

obteniendo el valor 2x = 2(2) = 4. ¿ Es correcta la respuesta?

Haciendo x = 2, Y = 4 en la expresión dada se obtiene

x + 2y + J(x - 2y)2 = 2 + 2(4) + ~ = 2 + 8 + .. /36 = 2 + 8 + 6 = 16

El alumno comete la equivocación de escribir ~ 2y)2 = X - 2y, lo cual es cierto, únicamente, cuando x ;¡; 2y. Si x ~ 2}', J(x - 2y)2 = 2.1' - x. En todos los casos, J(x - 2y)2 = jx - 2yl. La operación efectua­da correctamente habrá sido x + 2y + 2y - x = 4y, que es igual a 16 para y = 4.

http://carlos2524.jimdo.com/

Page 59: Algebra Superior Murray R Spiegel

alores. Así;

x s 0,

decir.

inador

+ 1 es

o si el

hizo lo

POTENCI¡\CION y RADICACION 51

PROBLEMAS PROPUESTOS

Calcular las siguientes expresiones:

19. a) 34 e) (-4X)-2

h) ( -2x)' f) (2y-l )-1

e)3y,

Ir)3-1x2r-4

(4"1 2-2".-3)'3

d) 4-3 h) (16)1/4

S-2/3 (_S)2/3i) 81/3

k) _3(_1)-1/5 (4)-1/2

(x + yf/3 (x + y)-1/6d)

[(x + y)2J'/4

4- 1/2a2/3b-1/6C- 3/2Ir) S2/3a-l/3b-2/3c5/2

10·d,·. I()Y-x. I()y+ 1

b)2- 8 • 34

h) ( )-1/4

5-4

31/2. 3-2/3e) r1/2. 31/3 i)

21. a) J27- 2/3 + 52/3 . 51/3 g) x3/2 + 4x- 1 _ 5xO para x = 4

1 y2/3 + 3y- 1 _ 2yo para y = liSh) 4(_·)° + r1 - 16-1/2 '4' 3° h)2

cuando e) 82/3 + r 2 - -~(10)Ú i) 64 - 2/3 . 165/4 . 2° . (J3)49

fectua-

272:3 - 3(3x)0 + 251/2Ja. a-2/3 a-5/6

d) j) ---+P .;¡ a2 • a-t/2

e) S213-16-3/4-2° _ 8-2/3

/72v2"

f) ,y('x - 2)-2 -6k) (~. 9O)(2y"+2)-1

para x = 3

22. a) 250 + 0.251/2 _ SI/3 - 4-112 + 0.027113 f) (64)-2/3 _ 3(150)0 + 12(2)-2

1 3h) ___ 3ao + (3a)0 + (27)- 1{3_ 13/2 g) (O 125)-2/3 + ---

S-2{3 • 2 + r1

r2 + 5(2)0 y;;.e) 3- 4(3)-1

h) 25+n

d)30x + 4x-1

si x = S(60.000)' (0.00002)4

2/3 i)1002 (72.000.000)(0.0002)5X

2 + ?-1e) __ -_ + (-So) _ 43/2

5

http://carlos2524.jimdo.com/

Page 60: Algebra Superior Murray R Spiegel

52 POTENCIACION y RADlCACION

(x2 + 3x + 4)1/3 [ -t(5 - X)-1/2] - (5 - X)1/2 [(X2 + 3x + 4)-2/3 (2x + 3)/3]23. a) (x2 + 3x + 4)2/3 si x = 1

(9x2 - 5y)1/4 (2x) - x2 [!(9x2 - 5y)-3/4 (18x)] .b) (9x2 _ 5y)1/2 SI X = 2, Y = 4

d) x - 1 + J x2 + 2x + 1

e) 3x - 2y - J4X2 - 4xy + y2RAr

SOLUCIONES DE LOS PROBLEMAS PROPUESTOS

4xS19. a) 81 d) 1/64 g) 3y7

j)a2b2

-8x3 1b) e)

16x2h) 2 k) 3/2

e)27y3

f) y/2 i) 1/2 /) 164

1m) (x _ y)2

PRCn) XSy p) 7.200

29 10-4 x-6 aJb 4 a1/4bsI6e20. a) b) l/lO e) 1 d) 1 e) f) g)

8e4 h) i)15 d

16 7 1 1 89 2 fi21. a) b) e) 4 d) 11 e) f) g) 4 h) i) 18 j) - k) -:73 2 4 4 4 a

22. a) 0,84

b) 346

e) 15 d) 34 e)132

1f) 16

26g) 5

1h) 2 i) 150

23. a)1

3

7b) 8

7 - xe)

6(x - 1 )1/2 (x + 1 )S/3

d) 2x si x ~ - 1, - 2 si x ~ -1

e) x - y si 2x ~ y, 5x - 3y si 2x ~ Y

Nm

http://carlos2524.jimdo.com/

Page 61: Algebra Superior Murray R Spiegel

.200

j27"

150

CAPITULO 7

Radicales

RADICAL. Es una expresión de la forma ~ que representa la raíz enésima principal de a. Elentero positivo n es el índice u orden del radical, y el número a es el subradical. El índice nose suele escribir si n = 2.

Por ejemplo, 15, ~,hX3'- 2y2, ~, son radicales de índices 3, 4, 2, Y subradicales5, 7x3 - 2y2, X + 10, respectivamente.

PROPIEDADES DE LOS RADICALES. Son las mismas que las correspondientes de las poten-cias, ya que ~ = a'!". A continuación se exponen las propiedades utilizadas con más fre-cuencia. Nota. Si n es par, se supone a, b ~ O.

A)

Ejemplos. (.y6)3 = 6,

B)

Ejemplos.

e) b+ o

Ejemplos.fs_.j5_~32 - ~ - 2 '

3 (x + 1 )3 _ .y (x + 1 )3 _ X + 1(y - 2)6 - .J (y - 2)6 - (y - 2)2

D)

Ejemplo

E)

3

Ejemplos. ns = ,ys, #> = ifi,

NUMERO RACIONAL. Es un número real que se puede escribir en la forma rt«. siendo p y qenteros.

53

CAPITULO 7

Radicales

RADICAL. Es una expresión de la forma ~ que representa la raíz enésima principal de a. El entero positivo n es el índice u orden del radical, y el número a es el subradical. El índice no se suele escribir si n = 2.

Por ejemplo, 15, ~hX3 ' - 2y 2, fi+"lO, son radicales de índices 3, 4, 2, Y subradicales 5, 7x 3 - 2y 2, X + 10, respectivamente.

PROPIEDADES DE LOS RADICALES. Son las mismas que las correspondientes de las poten­

cias, ya que ~ = a l /" . A continuación se exponen las propiedades utilizadas con más fre­cuencia . Nota. Si n es par, se supone a, b ~ O.

A)

Ejemplos. (~)3 = 6,

B)

Ejemplos.

e)

Ejemplos. fs_ys_~

32 - ~ - 2 '

D)

Ejemplo

E)

b+ o

3 (x + 1 )3 _ .y (x + 1 )3 _ X + 1 (y _ 2)6 - ~(y _ 2)6 - (y _ 2)2

3

Ejemplos. ns = ys, #> = ifi,

NUMERO RACIONAL. Es un número real que se puede escribir en la forma p/q. siendo p y q enteros.

53

http://carlos2524.jimdo.com/

Page 62: Algebra Superior Murray R Spiegel

54 RADICALES

Por ejemplo, 4, 2/3, 3/8, 0,36, -2,4, JTIi, ,/36/49, ~~27, son números racionales, pues-to que se pueden expresar como cociente de dos enteros de la forma siguiente:4 2 3 36 9 - 24 - 12 4 6 - 3r 3' 8' 100 o 25' 10 o -5-' l' 7' -1-

NUMERO IRRACIONAL. Es un número real que no se puede escribir en la forma rt«. siendop y q enteros.

Por ejemplo, j2, "Í3, 12, iIS, j2i3, ';;=4/5 son números irracionales.

La raíz cuadrada irracional de un número racional, como, por ejemplo, J5 ó JIi6, recibeel nombre de irracional cuadrático.

LA FORMA DE UN RADICAL se puede modificar con alguno de los siguientes métodos:

a) Sacando fuera de la raíz las potencias enésimas de la cantidad subradical.

Ejemplos. 132 = -123(4) = ft .,y¡ = 2-14

b) Reduciendo el índice del radical.

Ejemplos. -Y64 = ft = 26/4 = 23p =p = fi, habiéndose reducido el índice de 4 a 2.

~25x6 = ~(5X3)2 = (5X3)2/6 = (5X3)1/3 = ft;? = x.y5, habiéndose reducido el ín-dice de 6 a 3.

Nota. i( -4¡2 = i16 = 2.

Es incorrecto escribir 1(_4)2 = (_4)2/4 = (_4)1/2 = p.e) Racionalizando el denominador en el subradical.

Ejemplo 1. Racionalizar el denominador de .j9ji.Se multiplica el numerador y denominador del subradical (9/2) por un número quetransforme al denominador en una potencia enésima perfecta (en este caso, n = 3)Y se saca dicho denominador fuera de la raíz. El número, en este caso, es 22. Así, pues,

2. Racionalizar el denominador de

Para transformar 8b6x3 en una cuarta potencia perfecta, multiplicamos el numera-dor y el denominador por 2b2x, con lo cual,

4 14a3y2b2x _ 114a3y2b2x16b8x4 - 2b2:r:

UN

RAD

PAR.)

MUl

I

I

DIVI,

54 RADICAL ES

Por ejemplo, 4, 2/ 3, 3i8, 0,36, -2,4, -/1-6, -/36/49, .(!~27, son números racionales, pues­to que se pueden expresar como cociente de dos enteros de la forma siguiente: 4 2 3 36 9 - 24 - 12 4 6 - 3 l' 3' 8' 100 o 25' lo o -5-' l' 7' - 1-

NUMERO IRRACIONAL. Es un número real que no se puede escribir en la forma p/q, siendo p y q enteros.

Por ejemplo, )2, \/ 3, 12, fo, J2j3, .y - 4/5 son números irracionales.

La raíz cuadrada irracional de un número racional, como, por ejemplo, fi ó JIj6, recibe el nombre de irracional cuadrático.

LA FORMA DE UN RADICAL se puede modificar con alguno de los siguientes métodos:

a) Sacando fuera de la raíz las potencias enésimas de la cantidad subradical.

Ejemplos. .y32 = .y2 3 (4) = j23 . .y4 = 2.y4

b) Reduciendo el índice del radical.

Ejemplos. .y'64 = ft = 26/

4 = 23p =p = j8, habiéndose reducido el índice de 4 a 2.

~25x6 = ~(5X3)2 = (5X3)2/6 = (5X3)1 /3 = .ys;? = x-YS, habiéndose reducido el ín­dice de 6 a 3.

Nota. i( -4¡2 = i16 = 2.

Es incorrecto escribir .i( _4)2 = (_4)214 = ( _ 4)1 12 = p .

e) Racionalizando el denominador en el subradical.

Ejemplo 1. Racionalizar el denominador de .j9ji. Se multiplica el numerador y denominador del subradical (9/2) por un número que transforme al denominador en una potencia enésima perfecta (en este caso, n = 3) Y se saca dicho denominador fuera de la raíz. El número, en este caso, es 22

. Así, pues,

2. Racionalizar el denominador de

Para transformar 866\"3 en una cuarta potencia perfecta, multiplicamos el numera­

dor y el denominador por 2b2x , con lo cual,

4 14a3y 2b2x _ ~14a3y2b2x 16b8x 4 - 2b2x

http://carlos2524.jimdo.com/

Page 63: Algebra Superior Murray R Spiegel

pues-

iendo

recibe

a 2.

el in-

ro quen = 3), pues,

umera-

RADICALES 55

UN RADICAL está en su forma más simple cuando:

a) Se han sacado fuera de la raíz todas las potencias enésimas perfectasb) El índice de la raíz es el menor posible.e) Se ha racionalizado el denominador. es decir. cuando no existan fracciones en el subradical.

RADICALES SEMEJANTES. Dos o más radicales son semejantes cuando. reducidos a su formamás simple, tienen el mismo índice y el mismo subradical.

a: ¡-;-;;:. - F- J21=Por ejemplo, v 32, v 1/2, J8 son semejantes. ya que v 32 = JI6 :-2 = 4)2.

nfi'1/2 . "2 = 2' fi = ~ = 2fi· Todos los subradicales son 2 y todos los índices son 2.

Sin embargo, ,y32 y fi no son semejantes, ya que ,y32 = ~ = 214

PARA SUMAR ALGEBRAICAMENTE dos o más radicales se reducen a su forma más simpley se combinan los términos con radicales semejantes.

Por ejemplo, fo - fii2 - fi = 4fi - 4- - 2fi = (4 - ~ - 2)fi = ~fi

MUL TIPLlCACION DE RADICALES

a) Para multiplicar dos o más radicales del mismo índice se aplica la propiedad B:

.::/~ jb = fab

Ejemplos. (214)(3116) = 2' 314 116 = 6164 = 6' 4 = 24

(3M)(,y'X3y2) = 3.j(X2Y)(X3y2) = 3.jX5y3 = 3xU

b) Para multiplicar radicales de índices distintos conviene utilizar exponentes fraccionarios yaplicar las propiedades de la potenciación.

Ejemplos. 15fi = 51/3• 21/2 = 52/6. 23/6 = (52 . 23)1/6 = (25' 8)1/6 = .y2Oo14 fi = ft fi = 22/3• 21/2 = 24/6. 23/6 = 27/6 = .:f2' = 2,y2

DIVISION DE RADICALES .:.raa) Para dividir dos radicales del mismo índice se aplica la propiedad C. :,¡-b =

plifica a continuación.Aa .b y se sim-

Ejemplo.

También se puede racionalizar directamente el denominador.

b) Para dividir dos radicales de índices distintos conviene utilizar exponentes fraccionarios yaplicar las propiedades de la potenciación.

RADICALES 55

UN RADICAL está en su forma más simple cuando :

a) Se han sacado fuera de la raíz todas las potencias enésimas perfectas b) El índice de la raíz es el menor posible. e) Se ha racionalizado el denominador. es decir. cuando no existan fracciones en el subradical.

RADICALES SEMEJANTES. Dos o más radicales son semejantes cuando. reducidos a su forma más simple, tienen el mismo índice y el mismo subradical.

Por ejemplo. fo, J1ii, J8 son semejantes, ya que ,/32 = ,/16:2 = 4,/2. J~ =

nfi ..)2 . "2 = l' fi = ~ = 2fi· Todos los subradicales son 2 y todos los índices son 2.

Sin embargo, ~ y 12 no son semejantes, ya que ~ = ~4 = 214

PARA SUMAR ALGEBRAICAMENTE dos o más radicales se reducen a su forma más simple y se combinan los términos con radicales semejantes.

Por ejemplo, fo - J1ii - fi = 4fi - -4 -2fi = (4 - ~ - 2)fi = ~fi

MUL TlPLlCACION DE RADICALES

a) Para multiplicar dos o más radicales del mismo índice se aplica la propiedad B:

.::/ ~ .:jb = yIab

Ejemplos. (214)(3116) = 2' 314 116 = 6.y64 = 6' 4 = 24

(3M)(,ix 3y 2) = 3,i(x2Y)(X

3y2) = 3,iX 5y 3 = 3xU

b) Para multiplicar radicales de índices distintos conviene utilizar exponentes fraccionarios y aplicar las propiedades de la potenciación.

Ejemplos. 15 fi = 51/3 • 21/2 = 52/6• 23/6 = (52 . 23 )1 /6 = (25' 8)1 16 = .y2Oo

14 fi = fi fi = 22 /3• 21/2 = 24 /6

• 23/6 = 27/6 = .:ji' = 2,y2

DIVISION DE RADICALES .;¡;

a) Para dividir dos radicales del mismo índice se aplica la propiedad C, :¡b =

plifica a continuación.

Ejemplo.

También se puede racionalizar directamente el denominador.

A a . b y se slm-

b) Para dividir dos radicales de índices dislintos conviene utilizar exponentes fraccionarios y aplicar las propiedades de la potenciación.

http://carlos2524.jimdo.com/

Page 64: Algebra Superior Murray R Spiegel

Ejemplos.

IRRACIONALES CONJUGADOS. Los binomios irracionales cuadráticos Ja + Jb y Ja-Jb se denominan conjugados entre sí. Por ejemplo, 2)3 + J2 y 2)3 - J2 son conjugados.

TI

2.

Para racionalizar una fracción cuyo denominador es un binomio irracional cuadrático, semultiplican sus dos términos por el conjugado del denominador.

Ejemplo.5 5 2)3 - J2 _ 5(2)3 - J2) _ 2)3 - J2

2)3 - J2 - 12- 2 - 2

3.

PROBLEMAS RESUELTOS

REDUCCION DE UNA EXPRESION RADICAL A SU FORMA MAS SIMPLE 4.

1. a) fo=~=~=3j2 d) j648 = ~ = ;)23'33'3 = 6.y3

b) fo = 18-10 = ~ = 2.yJO e) aJ9b4c3 = aJ32b4c2 . e = 3ab2cJce) 5J243 = 5;)27' 9 = 5;)Y' 9 = 1519 f) .y343 = ft = 73/6 = 71/2 = ..fi

g) ~ = ~ = 34/6a2/6 = 32/3al/3 = 1% Obsérvese que a ;¡:; O. Véase k).

s.= (2' 22/5)(3' 33/5) = 22• 3122• 33 = 12.yi08

j) (7.y;¡;;b)2 = 49(4ab)2!3 = 49;)16a2b2 = 98;)2a2b2

k) 2aJa2 + 6a + 9 = 2a~ = 2a(a + 3). Hay que tener en cuenta que j{a + 3)2 es un positivoo cero; por tanto. ~ = a + 3 solo si a + 3 ;¡:; O. Si se quiere hacer extensión a valores de a tales quea + 3 < O. tendremos ~ = la + 31.

1) x - 25 = (Jx + 5)(Jx - 5) = Jx _ 5Jx+5 fi+5

6.Obsérvese que esto es cierto si 2.\'2 ;¡:; 3.1'2 Véase k).

http://carlos2524.jimdo.com/

Page 65: Algebra Superior Murray R Spiegel

RADICALES 57o) ;¡.j256 = ,yi6 = ~ = 212

4

p) J.y¡;;;i} = [(6ab2)"3]1/4 = (6ab2)1/12 = ~5

q) ;;729# = .;j729aJ/2 = (36a3/2)1/5 = 312/10a3/10 = 3~

TRANSFORMACION DE UN RADICAL

2. Expresar como radicales de orden 12 los siguientes:

b) fo = (ab)"2 = (ab)6/12 = !J(ab)6 = !J?h6

3. Expresar como radicales del menor orden posible los siguientes:

a) fi = 91/4 = (32)1/4 = 31/2 = .j3

b) ~ = ~ = (2xy2)3/12 = (2xy2)1/4 = 12xy2

e) :Ya2 + 2ab + b2 = :Y(a + W = (a + b)2/B = (a + b)I/4 = 1a + b

4. Transformar en radicales enteros, es decir, en radicales de coeficiente 1, los siguientes:

a) 6.j3 = .j36=3 = JtOs

d) a - b Ja + b =a+b a-b

5. Determinar cuál es el mayor de los siguientes números irracionales:

a) 12, 13 b) fi, .y1T e) 2,j5, 3Jia) 12 = 21/3 = 24/12 = (24)1/12 = (16)1/12; 13= 31/4 = 33/12 = (33)1/12 = (27)1/12

Como (27)1/12 > (16)1/12, 13 > 12

b) fi = 51/2 = 53/6 = (53)1/6 = (125)1/6; fo = (11)"3 = (11)2/6 = (112)1/6 = (121)1/6

Como 125> 121, fi> foe) 2fi = J2T:5 = fo; 3Ji = .j32:2 = j18. Por tanto, 2fi > 3Ji

6. Racionalizar el denominador de las expresiones siguientes:

RADICALES

4

p) J .y6ab2 = [(6ab2)"3]1 /4 = (6ab2)"12 = !,J6cJ} S

q) ;;729# = .y729a3/2 = (36a3/2) I /S = 312/10a3/10 = 3~

TRANSFORMACION DE UN RADICAL

2. Expresar como radicales de orden 12 los siguientes:

a) .j5 = 51/3 = 54/12 = if54 = if625

b) fo = (ab)"2 = (ab)6/12 = 1(ab)6 = ~

e) Y7' = X·/6 = X2 0 /12 = ~

3. Expresar como radicales del menor orden posible los siguientes:

a) j9 = 9"4 = (3 2)1 /4 = 31/2 = .)3

b) !J8?Y6 = ~ = (2xy2)3il2 = (2XJ?)1 /4 = 12xy2

e) ~a2 + 2ab + b2 = -Y(a + b)2 = (a + b)2/B = (a + b)1 /4 = 1a + b

4. Transformar en radicales enteros, es decir, en fadicales de coeficiente 1, los siguientes:

a) 6.)3 = .j36-3 = jiOs

d) a - b Ja + b = a+b a-b

(a - b)2 • a + b = ja - b (a + b)2 a - b a + b

S. Determinar cuál es el mayor de los siguientes números irracionales:

a) fi, 13 b) J5, .yII e) 2,fi, 3fi

a) fi = 2"3 = 24 /12 = (24 )1/12 = (16)1 /12; 13 = 31/4 = 33/12 = (3 3)1 /12 = (27)1112

Como (27)1 /12 > (16)"12 , 13 > fi b) J5 = 51/2 = 53/6 = (5 3)1 /6 = (125)1 /6; .yII = (11)"3 = (11)216 = (11 2)"6 = (121)1 /"

Como 125 > 121, J5 > .yII

e) 2,fi = ~ = fiO; 3fi = ~ = J18. Por tanto, 2J5 > 3fi

6. Racionalizar el denominador de las expresiones siguientes:

57

http://carlos2524.jimdo.com/

Page 66: Algebra Superior Murray R Spiegel

58 RADICALES

MULl3 3 % 3% 3136 1b) - = -'- = -- = -- =-136

16 16 % 16' 62 6 2 8. a)

011'0 método:3 3 62/3 3.62/3 3% 1 3-=-·-=--=-=-136.y6 61/3 62/3 6' 6 2

h)

e)

e) 3x4fY = 3x4 y(2x)3 = 3.•,4/ y(8x3) = 3x,j/8x3y = ~180

V h 2x(2x)3 V (2X)4 2x. 2

d)

e)

d)

.fl

g)

f¡)

by 2J27 - 4J12 = 2,,/9'-3 - 4,,/4-3 = 2' 3J3 - 4' 2J3 = 6J3 - 8)3 = -2J'1

e) 4fo + 3J4j3 - 2fo = 4' S)3 + 3 J~.~- 2' 4.j3 = (20 + 3 . ~ - 8)/3 = 14)3v 3 3 3

i)SUMA Y RESTA DE RADICALES SEMEJANTES

7. a) Ji8 + J50- fo = ~ + %2 - )36' 2 = 3fi + S,,/2 - 6fi = (3 + S - 6)fi = 2,/2j)

k)

3'-1--2 1 - Sd) 314)2 - 31250+ 3!jj32 = 163. 2 - ,3153:2 + 1-,· - = (6 - S + -)';;2 = - 3/2-r: y"-JV y'/J"- v V 2' 2 4 4"

/)

11I)

el )3 + fil - fo + s13 = J3 + jf¡-:} - ../9-:'3+ S,:,/3

= J3 + 313- 3J3 + s13 = -2J3 + 813

11

01

f) 2a127x3y + 3h18x3y - 6e.j'-x3y = 6axj;' + 6bxjY + 6ex~/;' = 6x(a + h + e) 1-;p)

q)

2 1 S r: S=(--+4'---) /6=-.)63 4 12 \, 4

1')

f¡) fi +~= - JC6 = ,fiJ. + Ji3 , - jiO,16)¡ü¡) = !v'IO + J,/ú5 - OA"lIo= 3.IJ10fi JO.I V 2 2 1(10 2

9. a

2 G 3 r::í. 4 e 2 3 4 1:- la - 3h + 4 G= -vab - -y·ah + - ab = (.. _ .. + --) Lab = (-----)yah

h a ab v b a ab v ab

58 RADICALES

Olro mélodo: 3 3 62/3 3 ' 62/3 3.y6i 1 -= -'-= - - =--=-fi 16 61/3 6213 6' 6 2

e) 3x 4fy = 3x 4 y(2X)3 = 3.\" :1 y(8x3

) = ~18x3y = ~18x3y V b- 2x(2.\")3 V (2.\")4 2x. 2

d)

4xy2 4xy2.j (2xy2 )2 4xy2 .j (2xy2 )2 3IA""::D 3¡-;-¡-e) -- = -- ' = = 2v4x y = 2yV'4x-y

.j2x/ .j2xy2 .j(2xy2 )2 2.\")'2

SUMA Y RESTA DE RADICALES SEMEJANTES

7, a) ji8 + fi - fo = ~ + %2 - )36'2 = 3.fi + s.fi - 6.fi = (3 + 5 - 6).fi = 2J2

b} 2J'ii - 4J12 = 2.)9,-5 - 4J4-3 = 2' 3)3 - 4· 2j j = 6j'3 - 8)3 = -2/ i

- r4") 2 e) 4J7s + 3J 4/3 - 2fo = 4' 5)3 + 3 1/-' -_. - 2· 4)3 = (20 + 3· - - 8) J3 = 14)3

v 3 3 3

"'- 1 - ;; 1 5 d) 31432 - 312sO + 31J/32 = .j63 , 2 - 3/ 5" 2 + I -- ' : = (6 - 5 + - ) 3/2 = - 3.12 V .. v"-JV V 11.)"- " V2' 2 4 Y 4"

el )3 + fil - fo + 5-13 = Ji + -127---:'3 - ,,19:'3 + S,Y3

= )3 + 3,y3 - 3j3 + 5-13 = - 2j 3 + 8.j3

f) 2a.j27x3y + 3h.j8x3y - 6e.j _ X3 y = 6ax,y'y + 6bxY; + 6cx~(';; = 6x(a + b + e) .j';;

2 Cl. 3 C. 4 C ' 2 3 4 r- 2a - 3b + 4 C. = -v ab - -· y ·ah + - yab = (- - -- + - - ) ¡ah = ( - )v ah h a ah b a ab" ab

http://carlos2524.jimdo.com/

Page 67: Algebra Superior Murray R Spiegel

RADICALES

MULTIPLlCACION DE RADICALES

8. a) (2)7)(3/S) = (2' 3).j'l-:s = 6J35

h) (3.y2)(516)(8j4) = (3' 5 . 8)FH = 120j48 = 24016

e) (.1i8.;>)(j2.) = j36x3 = x.y36

d) jah-1e' ·ja3h3c-1 = ja4h2c4 = Ja2hc2 = acjh

el fi·.y2 = 31/2• 21/3 = 33/0• 22{0 = ,y33 • 22 = iJ08

f) (j14)(j686) = (~)(~) = (71/3. 21/3)(73/4. 21/4) = (74/12. 24/12)(79/12. 23/12)

= 7(71/12. 27/12) = 7if7~ = 7'.]8%

g) (-fi .y:;:)6 = 5612x6!3 = 5)~2 = 125x2

11) (Fx10-0)(J8,l x 103)(JO.0016) = (J4 x 1O-6)(J81 x W)(JI6---;-¡<r4"j

= (2 x 10-3)(9 x 10)(4 x 10-2) = 72 x 10-4 = 0.0072

i) (fi + fi)(fi - 2fi) = fifi + fifi + (fi)(-2fi) + (fi){-2fi)

= 6 + fo - 2fig - 2 . 3 = - fo = - 3fi

j) (v/S + fi)2 = (fi)2 + 2(fi)(fi) + (fi)2 = 5 + 2..flO + 2 = 7 + 2fio

k) ofi - 4fi)2 = (7fi)2 - 2(7fi){4fi) + (4fi)2

= 7' . 5 - 2 . 7 . 4j15 + 4' . 3 = 245 - 56)15 + 48 = 293 - 56j15

/) (Ji + l)(fi - 1) = (fi)2 - (1)2 = 3 - 1 = 2

/11) (2/3 - fi){2fi + fi) = (2fi)' - (fi)2 = 4' 3 - 5 = 12 - 5 = 7

11) (2fi - 3fi){2fi + 3fi) = (2fi)2 - (3fi)1 = 4' 5 - 9' 2 = 20 - 18 = 2

o) (2 + .f3)(2 - .f3) = 4 - .j9

q) (3J:i - 4fi)(2fi + 3fi) = (3fi)(2,/3) - (4fi)(2j3) + (3,/2)(3,/6) - (4fi)(3,/6)

= 6J6 - 8j15 + 9ft - 12ft = 6ft - sfo + 18y'3 - 12)30

r) (J;-¡-Y - zH,¡;-:tY + z) = x + y - Z2

s) (2/;-:::-1 - xfi)(3F--"¡ + 2x_,/2) = 6(x - 1) - 3xJ"2(;-:'::1) + 4xJi(x - 1) - 4x'

= 61~- 1) + xJi(-~--=-li- 4.\2

9. a) (v12 + ,,/3 + ./5)(,,/2 + ,/3 - ,/S) = [(,/2 + -,./3) + Js][(.j:i + Jj) - "ts]= (J2 + ,fl)2 - (,,'S)l = 2 + 2,/6 + 3 - 5 = 2,~6

59RADie ALES

MUL T1PLICACION DE RADICALES

8. a) (2yÍ7 )(3J'5 ) = (2' 3) F5 = 6.fi5

h) (3,y2)(516)(s.j4) = (3' 5 . S) ~ = 1201 48 = 24016

e) (fo~)(~) = 1 36.\3 = x~136

d) .yah- l e5 • .ya3h'c·- 1 = .ya4h2c4 = J a2he2 = acfi

el .j3 . ,y2 = 31/2 . 21 13 = 33/6 . 22t6 = ,133 • 22 = ~

.fl (.yJ4)(,y686¡ = (~)(.y73. 2) = (7 113 • 2113 )(73/4. 21 /4) = (74/1 2. 24/12)(79/12. 23/1 2)

= 7(7 11 12 . 27 / 12 ) = 7!j7? = 7!j8%

g) (_Js,J'v )6 = 5612x6!3 = 53x2 = 125x 2

11) (J 4 x lO 6)(JS. I x 103)(JO.0016) = (J 4 x 1O - 6 )(JSI x 102 )( J I6-;;-¡0"4¡

= (2 x 10- 3 )(9 x 10)(4 x 10- 2 ) = 72 X 10 - 4 = 0.0072

i) (fi + .j3 )( fi - 2.j3 ) = fifi +.j3fi + (fi)(-2.j3 ) + (.j3)(-2.j3 )

= 6 + Jt8 - 2JlS - 2 . 3 = - Jt8 = - 3fi

j) (Js + fi )2 = (fi)2 + 2(fi)(fi) + (fi)2 = 5 + 2fo + 2 = 7 + 2fo

k) (7 fi - 4.j3)2 = (7 fi)2 - 2(7fi)(4.j3 ) + (4.j3 )'

= 7' . 5 - 2 . 7 . 4fo + 42 • 3 = 245 - 56J15 + 4S = 293 - 56 fo

/) (J~ + 1 )(J'3 - 1) = (.j3)2 - (1)' = 3 - 1 = 2

111) (2/3 - fi )(2.j3 + Js ) = (2 .j3 )2 - (fi )' = 4 ' 3 - 5 = 12 - 5 = 7

11 ) (2 Js - 3fiH2fi + 3fi ) = (2 fi )2 - (3 fi )2 = 4' S - 9· 2 = 20 - 18 = 2

(J) (2 + 13)(2 - 13) = 4 - ·f 9

q) (3 J 2 - 4Js )(2.j3 + 3fi ) = (3 fi )(2.j3 ) - (4Js )(2J3 ) + 13 J 2)(3)6) - (4Js)(3.J6 )

59

= 6J 6 - sfo + 9fo - 12.fiO = 6ft - sJls + ISJ 3 - 12fio

r) (J;--:;Y - Z)(P+Y + z) = x + y - z,

s ) (2J';-=:'-¡ - xJ2)(3'¡;-':"'-'1 + 2x .. ,/2) = 6(x - 1) - 3xJ2(;-~ + 4xJ i¡;--::::-¡¡ - 4x'

= 6(y - 1) + \Jii~-"::' Ii - 4.\ '

9. a) (vf2 + ./ , + ,,/ 5)(,,/2 + ,,/ 3 - v'S) = [( J 2 + / 3) + fi ][( J 2 + J 3) - /5]

= (/2 + / 3)' - (,,: 5 )2 = 2 + 2/ {; + 3 - S = 2,, 6

http://carlos2524.jimdo.com/

Page 68: Algebra Superior Murray R Spiegel

60 RADICALES

b) (2j3 + 3.Ji + 1)(2j3 - 3.Ji - 1) = (2,j3 + (3.Ji + l)][2j3 - (3.Ji + 1)]

= (2J3)2 - (3.Ji + 1)2= 12- (9·2 + 6.Ji + 1)= -7 - 6.Jie) (.Ji + ,j3 + J5)2 = (.Ji)2 + (,j3)2 + (J5)2 + 2(.Ji)(,j3) + 2(,j3)(J5) + 2(.Ji)(J5)

= 2 + 3 + S + 2J6 + 2jl5 + 2Jlo = 10+ 2J6 + 2Ji5 + 2Jlo

d) 06 + 3,j3-)(J6 - 3,j3) =J(6 + 3,j3)(6 - 3,j3) = J36 - 9·3 = J9 = 3

e) (Ja+"b - ~)2 = a + b - 2J(a + b)(a - b) + a - b = 2a - 2Ja2 - b2

De

11.

DIVISION DE RADICALES. RACIONALIZACION DE DENOMINADORES

10. a) 1OJ6 = 10~ = 2j3 d) ~= A= J~.~= A=~J6s.Ji S 2

b) 2fiO=~~=~~ e) ~ = ~= J~.~= f!;=~~3.~ís 3 S 3 13 3 3 9 27 312.

e) 4x . .{/x2y2 = 4x :f?!- = 4x fxY f) ~= 1~·16= ~=~fiy fxY y xy y 2 2 16 32 2

g) ,j3 + 4.Ji - sJ8 ,j3 + 4.Ji - sJ8.Ji J6 + 4·2 - sji6 J6 - 12.Ji = .Ji . .Ji= 2 = 2

3h) J5+.Ji

i) 1 + .Ji = 1 + .Ji. 1 + .Ji = 1 + 2.j2 + 2 = -(3 + 2.j2)1-.Ji 1-.Ji 1+.Ji 1-2

13.

j)x - JX2 - y2 X + JX2 _ y2

(x + P7) - (x - P7)(x - JX2 - y2)(X+ JX2 _ y2)

1) Jx""=l- Jx"+l. Jx""=l- Jx"+l = (x - 1) - 2J(."{- I)(x + 1) + (x + 1) = p--=-t _ xJx""=l + Jx"+l Jx""=l - Jx"+l (x - 1) - (x + 1)

m) x + Jx _ x + Jx . 1 + x - Jx _ x2 + Jx _ x2 + Jx1 + Jx + x - 1 + x + Jx 1 + x - Jx - (1 + X)2- X - 1 + x + x2

1 x2 - xy + y2 x2 _ xy + y2X + Y . x2 - xy + y2 = x3 + y3

32/3_ 31/341/3+ 42/3(31/3)3+ (41/3)3

http://carlos2524.jimdo.com/

Page 69: Algebra Superior Murray R Spiegel

RADICALES 61

PROBLEMAS PROPUESTOSDemostrar que:

r;;¡,11. a) ,/72 = 6Ji i) Rh=L, h

h) ./27 = 3)3 j) 14Jii7 = 2fo

e) 3,/20 = 6./5 k) 3.y213= fo

el) 2 rso;? ~ (tomando a ~ O) 1) 3a ~ 3Yi2a'"5\. SOa2= 2a,j2 - - = - 12a4 2a 8

a --- Fa fzfle) h ,j7SaJh2 = Sa2 3a m) xyz -2- = "2JIOyz2-, yz4 ,---- ,'_o

60j4¡45 = 8./51) --- /98a2hJ = 28V2h n)ab v

g) Y640 = 4.y¡Ü o) 314;9 = fi

11) ,J'88xJY"Z5 = 2xy2zjil?

12. a) v!fj + J48 - fo = s)3 11) .::/x"+ ly2.- 1Zl. = xy2zl fx1Yh) s.fi - 3JlS = fi i) 319 - 2j27 = fi

e) 2,/I5Ó - 4-/54 + 6J48 = 24fi - 2fi

el) sfi - 3-JsO + 7,/288 = 74fi j) 6J8a3/3 - 2J24ab2 + a~ = (7a - 4b) Fa a. b ~ O

e) y'l6aJ- 48a2b = 4a~ (a ~ O)

fl 3,j1fu? + 8.y;?74 = 10afi

g) J%s = 2.fi

13. a) (3/8)(6./5) = 36,jlo

h) ,/4-8x5 .,¡T;s = 12-,4

e) fi if32 = 4el) fi (fi + vli8) = 8el (S + fi)(S - fi) = 23

x f) (x - ,jJ)(-, + ,jJ) = x2 - )'

g) (2fi - fi)(3fi + 3fi) = 9fi

k) (x + 1),J'16x2 - 4x~ = 2~1) 2-/54 - 6.j2j3 - .j% = O

rr: 3 6r.31.3 4x - SY + 3 ~m) 4yxly + 4r.T3 - 5yY [x = yxy,j X2)'2 xy

k) (,/3 + ./5 + .Ji)(fi + ./5 - .Ji) = I + 2fi5

1) .J8 - 2.Ji.J8 + 2.Ji = 6

m) 4 +2.fi = 2 + fi

n) 6 - JI8 = 2 - fi3

o) ~-A=fi

p) ~+ 4J48 = I + 2fi8

/¡) (3fi - 2fi)(4,/2 + 3fi) = 6 + fi q) 36 - 2,fi\ = 6 _ 136

i) (fi - fi)2 + (fi + fi)2 = lO

j) (2';;+ 5~)(';; + ~)

= 7a - 5b + 7Ja2 - ab

http://carlos2524.jimdo.com/

Page 70: Algebra Superior Murray R Spiegel

6214. a) 2J24x

3= 4xfi (x> O)

J3x

ajb fob) --=-

bJa b

e) .fi + fi = 1+ ~ J6fi 2

J6 - JIO - .fi2 _ .fi - J5 - J6d) JI8 - 3

f) 3~ = a,yls112

~3a7b6e5 abg) ~ =2,y4c4

h)3 -2 -3 -1 1

x y Z = --fil4xyz2 2xy2z

4 3 1415. a) 2 + J5 + 5 + 2J5 = 5J5 - 5

3.fi + 2J5 47 + I2fob)

3.fi - 2J5 7

e) fi + .fi + J6 = 1+ 3fi - 2.fifi+.fi

ajb - bJa a + b - 2fod)

ajb + bJa a - b

x + JY x - JY 2X2+ 2ye) -- + -- = ---'-

x - JY x + JY x2 - y

RADICALES

i) fi fij9 ~13 3 3

120 - ,yIs = ~ 145 _~ 112Yi2 3 2

j)

k) _1_ = .fi + 2.fi-2 3

/) _5_ = ~ (3- fi)3 + fi 7

m) ~= -4-2.j32 -.fi

s.fi 3s s.fin) .fi _ 1 2 -2-

2.fi - 1 ¡;;o) = 5....; 3 - 8

.fi + 2

1- fi+I 2fi+1 - x - 2p)

l+fi+I x

Nl

f2:x - y Jxy(x - y)f) - -"----'--'-----'--'X3y _ 2x2y2 + xy3 - xy(x - y)

2+.fi+J5g)

2+.fi-J56 + 10.fi + 4J5 + 3fo

11

1h) --

2 +,y24 - 2,y2 + .y¡

10

3 3n ~x~yx + y ., JX2 - 2xy + y2 2y

EL3

si x ~ y2x

2 OP

62

14. a) 2J24x3

= 4xfi (x> O)

J3x ajb #

b) --=-bJa b

e) .)3 + fi = 1 + ~ J6 fi 2

J6 - jIO - .fi2 _ .)3 - .j5 - J6 d) Jl8 - 3

~3a7b6e5 ab g) ~=2j4c4

h)

4 3 14 15. a) 2 + .j5 + 5 + 2.j5 = 5 J5 - 5

3.)3 + 2.j5 47 + 12J15 b)

3.)3 - 2.j5 7

e) fi + .)3 + J6 = 1 + 3fi _ 2.)3 fi+.)3

ajb - bJa a + b - 2# d)

ajb + bJa a - b

x + JY x - JY 2X2 + 2y e) --+ - - = ---'-

x - JY x + JY X2 - y

RADICALES

i) fi fi.y9 ~ 13 3 3

j) ,y2o - ,y¡s = ~ fo -~ ~ Yi2 3 2

k) _1_ .fi + 2 .fi-2 3

/) _ 5_ = ~ (3 - fi) 3 + fi 7

m) ~= -4- 2.)3 2 -.)3

s.)3 3s s.)3 n) --

.)3-1 2 2

2.)3 - 1 ¡;; o) = 5....; 3 - 8

.)3 + 2

1-F+\ 2F+\-x-2 p)

1 +F+\ x

¡;;x - y Jxy(x - y) f) - ~---'-

X3 y _ 2x2y2 + xy3 - xy(x - y)

2 + .)3 +.j5 6 + 10.)3 + 4.j5 + 3J15 g)

2+.)3-.j5 11

1 4 - 2.y2+.y4 h) --

2+.y2 10

3 3 O ~x~y x + y ~ J X2 - 2xy + y2 2y

3 si x ~ y

2x

http://carlos2524.jimdo.com/

Page 71: Algebra Superior Murray R Spiegel

CAPITULO 8

Operaciones con números complejos

LA UNIDAD DE LOS NUMEROS IMAGINARIOS es Fl y se representa, en general, porla letra i. Muchas de las propiedades de los números reales son válidas también en los númerosimaginarios.

Por ejemplo, p = )(4)( -1) = 2Fl = 2i, J=l8 = )(18)( -1) = Ji8 Fl = 3ft¡También como i = Fl, tenemos i2 = -1, i3 = i2. i = (-I)i = -i, i4 = (i2f = (_1)2

1, i5 = i4• i = 1 . i = i, Y análogamente para cualquier potencia entera de i.

Nota. Se debe tener sumo cuidado al aplicar algunas de las propiedades de los números rea-les. Por ejemplo, se puede pensar que

p p = )( -4)( -4) = Ji6 = 4, lo cual es incorrecto

Para salvar tales dificultades, expresaremos siempre ~, siendo m un número positivo, porJm i; siendo ¡2 = -1. Así, pues,

p p = (2i)(2i) = 4i2 = -4, que es correcto

NUMERO COMPLEJO. Es de la forma a + bi, siendo a y b números reales, e i = Fl.En el número complejo a + bi, a recibe el nombre de parte real y bi el de parte imaginaria. «Sia = O, el número complejo se llama imaginario puro.» Si b = O, el número complejo se reduceal número real a. Por consiguiente, en los números complejos están incluidos todos los númerosreales y todos los imaginarios puros.

La condición necesaria y suficiente para que los números complejos a + bi Y e + di seaniguales es que a = e y b = d. Así, pues, a + bi = O si, y solo si, a = O, b = O. Si e + di = 3, setendrá e = 3, d = O.

EL CONJUGADO DE UN NUMERO COMPLEJO a + bi es a - bi, Y recíprocamente. Porejemplo, 5 - 3i Y 5 + 3i son conjugados.

OPERACIONES ALGEBRAICAS CON NUMEROS COMPLEJOS

1) Para sumar dos números complejos se suman, por una parte, las partes reales y, por otra,las imaginarias. Por ejemplo,

(a + bi) + (e + di) = (a + e) + (b + d)i(5 + 4i) + (3 + 2i) = (5 + 3) + (4 + 2)i = 8 + 6i(-6 + 2i) + (4 - 5i)= (-6 + 4) + (2- 5)i= -2 - 3i

2) Para restar dos números complejos se restan, por una parte, las partes reales y, por otra,las imaginarias. Por ejemplo,

(a + bi) - (e + di) = (a - e) + (b - d)i

63

CAPITULO 8

Operaciones con números complejos

LA UNIDAD DE LOS NUMEROS IMAGINARIOS es Fl y se representa, en general, por la letra i. Muchas de las propiedades de los números reales son válidas también en los números imaginarios.

Por ejemplo, p = )(4)( -1) = 2Fl = 2i, J=l8 = )(18)( -1) = Ji8 Fl = 3ft¡ También como i = Fl, tenemos i 2 = -1, i 3 = i 2

• i = (-I)i = -i, i4 = (¡Zf = (_1)2

1, i5 = i4• i = 1 . i = i, Y análogamente para cualquier potencia entera de i.

Nota. Se debe tener sumo cuidado al aplicar algunas de las propiedades de los números rea­les. Por ejemplo, se puede pensar que

p p = )( -4)( -4) = Ji6 = 4, lo cual es incorrecto

Para salvar tales dificultades, expresaremos siempre ~, siendo m un número positivo, por Jm i; siendo i 2 = -1. Así, pues,

p p = (2i)(2i) = 4i2 = - 4, que es correcto

NUMERO COMPLEJO. Es de la forma a + bi, siendo a y b números reales, e i = Fl. En el número complejo a + bi, a recibe el nombre de parte real y bi el de parte imaginaria. «Si a = O, el número complejo se llama imaginario puro.)) Si b = O, el número complejo se reduce al número real a. Por consiguiente, en los números complejos están incluidos todos los números reales y todos los imaginarios puros.

La condición necesaria y suficiente para que los números complejos a + bi Y e + di sean iguales es que a = e y b = d. Así, pues, a + bi = O si, y solo si, a = O, b = o. Si e + di = 3, se tendrá e = 3, d = O.

EL CONJUGADO DE UN NUMERO COMPLEJO a + bi es a - bi, Y recíprocamente. Por ejemplo, 5 - 3i Y 5 + 3i son conjugados.

OPERACIONES ALGEBRAICAS CON NUMEROS COMPLEJOS

1) Para sumar dos números complejos se suman, por una parte, las partes reales y, por otra, las imaginarias. Por ejemplo,

(a + bi) + (e + di) = (a + e) + (b + d)i

(5 + 4i) + (3 + 2i) = (5 + 3) + (4 + 2)i = 8 + 6i

(-6 + 2i) + (4 - 5i) = (-6 + 4) + (2 - 5)i = -2 - 3i

2) Para restar dos números complejos se restan, por una parte, las partes reales y, por otra, las imaginarias. Por ejemplo,

(a + bi) - (e + di) = (a - e) + (b - d)i

63

http://carlos2524.jimdo.com/

Page 72: Algebra Superior Murray R Spiegel

64 OPERACIONES CON NUMEROS COMPLEJOS

(3 + 2i) - (5 - 3i) = (3 - 5) + (2 + 3)i = -2 + 5i( - 1 + i) - (- 3 + 2i) = (- 1 + 3) + (1 - 2)i = 2 - i

3. (

3) Para multiplicar dos números complejos se efectúa la operación como si se tratase de dosbinomios sustituyendo i2 por - l. Por ejemplo,

(a + bi)(e + di) = ac + adi + bci + bdi2~= (ac - bd) + (ad + be)i(5 + 3i)(2 - 2i) = 10 - 10i + 6i - 6i2 = 10 - 4i - 6( -1) = 16 - 4i

4) Para dividir dos números complejos se multiplican el numerador y denominador por elconjugado del denominador y se sustituye i2 por - l. Por ejemplo,

2 + i 2 + i 3 + 4i 6 + 8i + 3i + 4iz 2 + 11i 2 11 .3 - 4i = 3 - 4i . 3 + 4i = 9 - 16;2 = -2-5- = 25 + 25/

En el Capítulo 20 se trata de cálculos más complicados con números complejos.

PROBLEMAS RESUELTOS

1. Expresar en función de i.

a) F25 = J(25)(-I) =.fi5p = 5i

b) 3J"=36 = 3)36 P = 3 . 6 . i = 18i

e) -4J=8t = -4J8t P = -4·9' i = -36i

Dd) R=flp=jt=fi

. FI6 (=i9 4 . 7. 8. 21. 16. 21. 1 .e) 2 V 25 - 3V 100= 2' 51 - 3'10 1 = 51 -lo / = lo1 -lo / = - "2/

k

m

f) P2 -P = Ji2 i - j3 i = 2j3i - j3 i = j3 i

g) 3J -50 + 5J=18 - 6J -200 = 15)2 i + 15)2 i - 60)2 i = -30)2i

h) - 2 + J=4 = - 2 + J4 i = - 2 + 2i j) J8 + J=8 = J8 + J8 i = 2)2 + 2)2 i

n

o

i) 6 - J - 50= 6 - J50 i = 6 - 5)2 i k) ~(-IO + piS) = ~(-IO + 5)5 i) = -2 +)5 ip

1) ~ (J32 + J=I28) = ~ (4../2 + 8)2 i) = )2 + 2)2 i

m) R + J=8 = -2 + 2)2 i = _1 + )2 i2 2

4. al

b)

e)

2. Efectuar las operaciones indicadas y simplificar.

a) (5 - 2i) + (6 + 3i) = 11+ i h) (6+ 3i) - (4 - 2i) = 6 + 3i - 4 + 2i = 2 + 5i

64 OPERACIONES CON NUMEROS COMPLEJOS

(3 + 2i) - (5 - 3i) = (3 - 5) + (2 + 3)i = -2 + 5i

( - 1 + i) - (- 3 + 2i) = (- 1 + 3) + (1 - 2)i = 2 - i

3) Para multiplicar dos números complejos se efectúa la operación como si se tratase de dos binomios sustituyendo i2 por - l. Por ejemplo,

(a + bi)(c + di) = ac + adi + bci + bdi21 = (ac - bd) + (ad + bC)i

(5 + 3i)(2 - 2i) = 10 - 10i + 6i - 6~2 = 10 - 4i - 6( -1) = 16 - 4i

4) Para dividir dos números complejos se multiplican el numerador y denominador por el conjugado del denominador y se sustituye i2 por - 1. Por ejemplo,

2 + i 2 + i 3 + 4i 6 + 8i + 3i + 4i l 2 + 11 i 2 11 . -3---4i = -3---4i . -3-+-4-i = 9 - 16i2 = - 2-5- = 25 + 251

En el Capítulo 20 se trata de cálculos más complicados con números complejos.

PROBLEMAS RESUELTOS

l. Expresar en función de ;.

a) ) -2S = )(2S)(-I) = foP = S;

b) 3)=36 = 3)36 P = 3 ' 6' ; = 18;

e) -4J=8I = -4J81 P = -4,9'; = -36;

- d) R=flp=fi~=~; . FI6 F49 4 . 7. 8. 21. 16 . 21. 1 .

e) 2 V 25 - 3 V 100 = 2· 5' - 3'\0 1 = 5' -\o 1 = \o 1 - \o 1 = - "2 '

f) P2 - P = Ji2 ; - Ji ; = 2Ji; - Ji; = Ji ; g) 3.J=50 + sJ=Ts - 6) -200 = IS)2 ; + IS)2; - 60)2; = -3012;

h) -2 + J=4 = -2 + J4; = -2 + 2; j) fi + J=8 = fi + fi; = 2)2 + 2)2;

;) 6 - ) - SO = 6 - J50 ; = 6 - s)2 ; k) ~(-10 + piS) = ~(-IO + s)5 ;) = -2 +)5; S S

1) ~ (ft + F-I2s) = ~ (4 .. / "2 + 8)2;) = )2 + 2)2 ; 4 4

y=s + J=8 -2 + 2)2 ; F. . m) = = -1 + ...;21

2 2

2. Efectuar las operaciones indicadas y simplificar.

a) (S - 2;) + (6 + 3;) = 11 + ; h) (6 + 3;) - (4 - 2;) = 6 + 3; - 4 + 2; = 2 + S;

http://carlos2524.jimdo.com/

Page 73: Algebra Superior Murray R Spiegel

dos

r el

¡Si

5i

OPERACIONES CON NUMEROS COMPLEJOS 65

e) (S - 3i) - (- 2 + Si) = S - 3i + 2 - Si = 7 - 8i

3S 1131 SI S7d) (2 + 8 i) + (- 4 + 4 i) = 2 - 4 + (8 + 4) i = 4 + 8 i

'"e) (a + bi) + (a - bi) = 2a

f) (a + bi) - (a - bi) = a + bi - a + bi = 2bi

g) (S - J-12S) - (4'.- J-20) = (S - syÍS i) - (4 - 2yÍS i) = 1 - 3yÍS i

3. a) ,J=2y'=32 = (J2 i)(fo i) = J2 fo ¡2 = j64 (-1) = -8

b) -3J=5J-20= -3(yÍSi)(foi)= -3(J5fo)i2= -3J!OO(-I) = 30

e) (4i)(-3i) = -12¡2 = 12 e) (2P)3 = (2i)3 = 8i3 = 8i(i2) = -8i

f) 3i(i + 2) = 3¡2 + 6i = -3 + 6id) (6i)2 = 36i2 = - 36

g) (3 - 2i)(4 + i) = 3 . 4 + 3 . i - (2i)4 - (2i)i = 12 + 3i - 8i + 2 = 14 - Si

h) (5 - 3i)(i + 2) = 5i + 10 - 3i2 - 6i = 5i + 10 + 3 - 6i = 13 - i

i) (5 + 3i)2 = 52 + 2(5)3i + (3i)2 = 25 + 30i + 9i2 = 16 + 30i

j) (2 - i)(3 + 2i)(1 - 4i) = (6 + 4i - 3i - 2i2)(I - 4i) = (8 + i)(1 - 4i)

= 8 - 32i + i - 4i2 = 12 - 31i

1) (1 + i)3 = 1 + 3i + 3i2 + 3i3 = 1 + 3i - 3 - i = - 2 + 2i

m) (3 - 2i)3 = 33 + 3p2)(-2i) + 3(3)(-2i)2 + (-2i)3

= 27 + 3(9)( -,2i) + 3(3)(4i2) - 8i3 = 27 - S4i - 36 + 8i = -9 - 46i

n) (3 + 2i)3 = 33 + 3(32)(2i) + 3(3)(2i)2 + (2i)3

= 27 + S4i + 36i2 + 8i3 = 27 + S4i - 36 - 8i = - 9 + 46i

o) (1 + 2i)4 = [(1 + 2i)2]2 = (1 + 4i + 4i2)2 = (-3 + 4i)2 = 9 - 24i + 16i2 = -7 - 24i

p) (-1 + i)8 = [(-1 + i)2]4 = (1 - 2i + ¡2)4 = (_2i)4 = 16¡4 = 16

1 + i 1 + i 3 + i 3 + 3i + i + i2 2 + 4i 1 24. a) 3 _ i = 3 _ i' 3 + i = 32 _ i2 = -10- = "5 + "5 i

1 1 -i -i -ib) i = i (_i) = _ i2 = T = - i

e) 2.j3 + J2 i 2.j3 + J2 i . 3J2 + 4.j3 i _ (2.j3 + J2 i)(3J2 + 4)3 i)

3J2 - 4.j3 i 3J2 - 4.j3 i 3J2 + 4.j3 i (3J2)2 - (4.j3)2i2

_ 6)6 + 8J9 i + 3J4i + 4)4 i2-2)6 + 30 i .)6 S.- 18 + 48 = 66 = 33 + 11'

OPERACIONES CON NUMEROS COMPLEJOS

e) (S - 3i) - (- 2 + Si) = S - 3i + 2 - Si = 7 - Si

3S 1131 SI S7 d) (2 + g i) + (- 4 + 4 i) = 2 - 4 + (g + 4) i = 4 + g i

'" e) (a + bi) + (a - bi) = 2a

f) (a + bi) - (a - bi) = a + bi - a + bi = 2bi

g) (S - J -12S) - (4'.- J - 20) = (S - sfi i) - (4 - 2j5 i) = 1 - 3fi i

3. a) R~ = (J2 i)(J32 i) = J2 J32 i2 = J64 (-1) = -S

b) - 3P J -20 = - 3(fi i)(J20 i) = -3(jS J20)i2 = - 3JiOO (-1) = 30

e) (4i)(-3i) = -12¡2 = 12 e) (2p)3 = (2i)3 = Si3 = Si(i2) = -Si

d) (6i)2 = 36¡2 = - 36 f) 3i(i + 2) = 3i2 + 6i = -3 + 6i

g) (3 - 2i)(4 + i) = 3 . 4 + 3 . i - (2i)4 - (2i)i = 12 + 3i - Si + 2 = 14 - Si

h) (S - 3i)(i + 2) = Si + lO - 3i2 - 6i = Si + lO + 3 - 6i = 13 - i

i) (S + 3i)2 = S2 + 2(S)3i + (3i)2 = 2S + 30i + 9i2 = 16 + 30i

j) (2 - i)(3 + 2i)(1 - 4i) = (6 + 4i - 3i - 2i2)(1 - 4i) = (S + i)(1 - 4i)

= S - 32i + i - 4i2 = 12 - 31i

1) (1 + i)3 = 1 + 3i + 3i2 + 3i3 = 1 + 3i - 3 - i = -2 + 2i

m) (3 - 2i)3 = 33 + 3(32)(-2i) + 3(3)(-2i)2 + (-2i)3

= 27 + 3(9)( - ,2i) + 3(3)(4i2) - Si3 = 27 - S4i - 36 + Si = -9 - 46i

n) (3 + 2i)3 = 33 + 3(32)(2i) + 3(3)(2i)2 + (2i)3

= 27 + S4i + 36i2 + Si3 = 27 + S4i - 36 - Si = - 9 + 46i

o) (1 + 2i)4 = [(1 + 2i)2]2 = (1 + 4i + 4¡2)2 = (-3 + 4i)2 = 9 - 24i + 16i2 = -7 - 24i

1 + i 1 + i 3 + i 3 + 3i + i + i2 2 + 4i 1 2 4. a) -- = --' -- = - - + i

3 - i 3 - i 3 + i 32 - ¡2 - ---¡o - 5 5

1 1 -i -i -i b) i = i (_ i) = _ i2 = T = - i

e) 2fi + J2 i 2.fi + J2 i , 3J2 + 4.fi i _ (2.fi + J2 i)(3J2 + 4fi i)

3J2 - 4.fi i 3J2 - 4.fi i 3J2 + 4.fi i - (3J2)2 - (4.fi)2i2

_ 6)6 + s)9 i + 3)4 i + 4)4 i2 -2)6 + 30 i ,)6 S. - IS + 48 = 66 = 33 + 11'

65

http://carlos2524.jimdo.com/

Page 74: Algebra Superior Murray R Spiegel

66 OPERACIONES CON NUMEROS COMPLEJOS

PROBLEMAS PROPUESTOS5. 'Expresar en función de i.

a) 2}"=49 d) 4F1i8h) -4)-64 e) 3J=2s - 5FtOO

e) 6FJi9 fl 2J=72 + 3J - 32

Efectuar las operaciones indicadas y simplificar.

h) ~ ( - 12 - ) - 288 )6

i) 4)=81 - 3) - 36 + 4~25

j) 3Ji2 - 3F12

6. a) (3 + 4i) + (- I - 6i) g) (2i)4 m) (3 - 4i)1

b) (- 2 + 5i) - (3 - 2i) h) (íJ':"3)" 11) (1 + i)(2 + 2i)(3 - i)

2 1 1 15i(2 - i) (i - 1)'e) (3 - 2 i) - (- 3 + 2 i) i) o)

d) (3 + J=8) - (2 - J-32) j) (2 + i)(2 - i) p) (2 + 3i)3

e) P~-12 k) (- 3 + 4i)( - 3 - 4i) q) (1 - i)·

f) (-ifi)(ifl) 1) (2 - 5i)(3 + 2i) r) (i + 2)5

2 - 5i 3 - fli i + i1 + i-1 + i·7. a) d)

./2ig) ---------~-_.-

4 + 3i 1 + i

h)-1 1 3 ¡2b _ i

2 - 2ie) --+-- h)

1 - 2i 1 + 4i i - I

3fl + 2)3 i 5 10 4-" .1 - 1e)

3fl - 2)3 i.fl --+--- i) (____ )1

3 - 4i 4 + 3i 1 + 2i

SOLUCIONES DE LOS PROBLEMAS PROPUESTOS

5. a) 14i e) 2i e) -35i

h) -32i d) fli O/) 24fl i

6. a) 2 - 2i d) 1 + 6fl i g) 16

h) -5+7i h) -27/64e) -6

c) 1-i fl 2 i) 5 + 10i

7. a)7 26

- - - -- i25 251 2)-

e) 5 + 5 6 i

1 1h) - 4 - 4 i

3 ¡::,d) -1 - -v2 i

2

g) -2 + i i) 18i + 20

11) -2 - 2fl i j) 6)3 - 6)3 i

j) 5

k) 25

1) 16 - 11i

32 26e) Ss - 85 i

11 2fl S-Si

m) -7 - 24i p) -46 + 9i

11) 4 + 12i q) -4

o) 2 +: 2i r) -38 + 41i

g) O

h) i

i) 3 + 4i

UNA

n

LASeeId1,

t

OPEI

http://carlos2524.jimdo.com/

Page 75: Algebra Superior Murray R Spiegel

9i

67

CAPITULO 9

Ecuaciones en general

UNA ECUACION es una igualdad entre dos expresiones que se denominan miembros de la misma.

Una ecuación que solo se verifique para ciertos valores de las letras (o incógnitas) recibe elnombre de ecuación condicional o, simplemente, ecuación.

Una ecuación que se verifique para todos los valores permitidos de sus letras (o incógnitas)recibe el nombre de identidad. Valores permitidos son aquellos para los que están definidos losmiembros de la ecuación.

Por ejemplo:

1) x + 5 = 8 se verifica solo para x = 3; es una ecuación condicional.

2) x2 - y2 (x - y)(x + y) se verifica para todos los valores de x e y; es una identidad.

2x - 5 ifi d3) x _ 2 + x _ 3 = (x _ 2)(x _ 3) se ven ea para to os los valores excepto para los no per-

mitidos x = 2, x = 3; para estos valores, la operación se reduce a una división por cero, lo cualcarece de sentido. Como la ecuación se verifica para todos los valores permitidos de x, es unaidentidad.

Para representar una identidad se emplea el símbolo == en lugar del símbolo =.

3 i

LAS SOLUCIONES de una ecuación son los valores de las incógnitas que transforman la ecua-ción en una identidad, es decir, se igualan ambos miembros. Las soluciones satisfacen a laecuación. Si la ecuación solo contiene una incógnita, las soluciones se denominan raíces de la ecuación.Resolver una ecuación es hallar todas sus soluciones. Por ejemplo, x = 2 es una raíz, o solución,de la ecuación 2x + 3 = 7, ya que sustituyendo x = 2 en ésta, se obtiene 2(2) + 3 = 7, es decir,los dos miembros se hacen iguales y la ecuación se convierte en una identidad. Análogamente,tres soluciones de la ecuación 2x + y = 4 son: x = O, Y = 4; x = 1, Y = 2; x = 5, Y = -6.

OPERACIONES APLICADAS EN LA TRANSFORMACION DE ECUACIONES

41i

a) Si se suman miembro a miembro varias igualdades, se obtiene otra igualdad.

Por ejemplo, en la igualdad x - y = z, podemos sumar y a ambos miembros, con lo que re-sulta x = y + z .

b) Si se restan miembro a miembro varias igualdades se obtiene otra igualdad.

Por ejemplo, en la igualdad x + 2 = 5, podemos restar 2 a ambos miembros, con lo que seobtiene x = 3.

Nota. Como consecuencia de a) y b) se deduce que para trasponer un término de una ecua-ción de un miembro a otro no hay más que cambiarlo de signo. Por ejemplo,

CAPITULO 9

Ecuaciones en general

UNA ECUACION es una igualdad entre dos expresiones que se denominan miembros de la misma.

Una ecuación que solo se verifique para ciertos valores de las letras (o incógnitas) recibe el nombre de ecuación condicional o, simplemente, ecuación.

Una ecuación que se verifique para todos los valores permitidos de sus letras (o incógnitas) recibe el nombre de identidad. Valores permitidos son aquellos para los que están definidos los miembros de la ecuación.

Por ejemplo:

1) x + 5 = 8 se verifica solo para x = 3; es una ecuación condicional.

2) X2 - y2 (x - y)(x + y) se verifica para todos los valores de x e y; es una identidad .

I l 2x - 5 'fi d 3) x _ 2 + x _ 3 = (x _ 2)(x _ 3) se .ven ca para to os los valores excepto para los no per-

mitidos x = 2, x = 3 ; para estos valores, la operación se reduce a una división por cero, lo cual carece de sentido. Como la ecuación se verifica para todos los valores permitidos de x, es una identidad.

Para representar una identidad se emplea el símbolo == en lugar del símbolo = .

LAS SOLUCIONES de una ecuación son los valores de las incógnitas que transforman la ecua­ción en una identidad, es decir, se igualan ambos miembros. Las soluciones satisfacen a la ecuación. Si la ecuación solo contiene una incógnita, las soluciones se denominan raíces de la ecuación. Resolver una ecuación es hallar todas sus soluciones. Por ejemplo, x = 2 es una raíz, o solución, de la ecuación 2x + 3 = 7, ya que sustituyendo x = 2 en ésta, se obtiene 2(2) + 3 = 7, es decir, los dos miembros se hacen iguales y la ecuación se convierte en una identidad. Análogamente, tres soluciones de la ecuación 2x + y = 4 son : x = O, Y = 4; x = 1, Y = 2; x = 5, Y = -6.

OPERACIONES APLICADAS EN LA TRANSFORMACION DE ECUACIONES

a) Si se suman miembro a miembro varias igualdades, se obtiene otra igualdad.

Por ejemplo , en la igualdad x - y = z, podemos sumar y a ambos miembros, con 10 que re­

sulta x = y + z.

b) Si se restan miembro a miembro varias igualdades se obtiene otra igualdad.

Por ejemplo, en la igualdad x + 2 = 5, podemos restar 2 a ambos miembros, con lo que se obtiene x = 3.

Nota. Como consecuencia de a) y b) se deduce que para trasponer un término de una ecua­ción de un miembro a otro no hay más que cambiarlo de signo. Por ejemplo,

67

http://carlos2524.jimdo.com/

Page 76: Algebra Superior Murray R Spiegel

68 ECUACIONES EN GENERAL

si 3x + 2y - 5 = x - 3y + 2, tendremos 3x - x + 2y + 3y = 5 + 2 o 2x + 5y = 7.e) Si se multiplican miembro a miembro varias igualdades se obtiene otra igualdad.

Por ejemplo, se se multiplican por 4 los dos miembros de la igualdad iY = 2X2 se obtieney = 8x2•

Análogamente, si los dos miembros de ~C = F - 32 se multiplican por ~ se obtiene5e = 9(F - 32).

d) Si se dividen miembro a miembro varias igualdades se obtiene otra igualdad, siempre que nose divida por cero.

Por ejemplo, si se dividen los dos miembros de la igualdad -4x = -12 por -4, se obtie-ne x = 3.

Análogamente, en la igualdad V = RI se pueden dividir los dos miembros por R =1= O, ob-teniéndose I = VIR.e) Si se elevan al mismo exponente los dos miembros de una igualdad se obtiene otra igualdad.

Por ejemplo, si T = 21tJTji, tendremos ~ = (21tJTji)2 = 41t21/g

f) Si se extrae la raíz enésima de los dos miembros de una igualdad se obtiene otra igualdad.

P . 1 . J 3V 1 JJ3Vor ejemp o, SI r = 41t ' resu ta r = V ~ .

g) Los recíprocos de los miembros de una igualdad dan lugar a otra igualdad, siempre queno tenga lugar la división por cero.

Por ejemplo, si xl -31, tendremos x = 3. Análogamente, si I - R¡ + R2 se verificaR - R¡R2

R = R¡R2

R¡ + R2'

Las operaciones a) a f) se llaman axiomas de la igualdad.

ECUACIONES EQUIVALENTES. Son las que tienen las mismas soluciones.

Por ejemplo, x - 2 = O Y 2x = 4 tienen la solución común x = 2 y, por tanto, son equiva-lentes. Sin embargo, x - 2 = O Y x2 - 2x = O no son equivalentes, ya que x2 - 2x = O tiene,además, la solución x = O.

Las operaciones anteriores aplicadas a la transformación de ecuaciones no dan lugar, en to-dos los casos, a ecuaciones equivalentes a las primitivas. La aplicación de estas operaciones puedeconducir a ecuaciones derivadas que tengan distintas soluciones que la ecuación original.

Si se \lega a una ecuación con más soluciones que la original, las soluciones nuevas se deno-minan extrañas y la ecuación derivada se \lama redundante con respecto a la original. Si se \legaa una ecuación con menos soluciones que la original, la ecuación derivada recibe el nombre dedefeetiva con respecto a la original.

Las operaciones a) y b) siempre conducen a ecuaciones equivalentes. Sin embargo, e) y e) pue-den dar lugar a eeuaciones redundantes y soluciones extrañas y d) y f) a ecuaciones defectivas.

UNA FORMULA es una ecuación que expresa un hecho general, una regla o un principio.Por ejemplo, la fórmula de geometría A = 1tr2 expresa el área A de un círculo en función de

su radio r.

La fórmula fisica s = Ígt2, en la que g es la aceleración de la gravedad y que vale, aproxima-damente, 9,81 metros por segundo en cada segundo, expresa la relación que existe entre el espa-cio s, en metros, que recorre un cuerpo que cae libremente partiendo del reposo, y el tiempo t, ensegundos, que emplea en el movimiento.

maunt

deori

UN TItieivo:+

res

UNAnopo

UNAer

lo,de

UNAde

68 ECUACIONES EN GENERAL

si 3x + 2y - 5 = x - 3y + 2, tendremos 3x - x + 2y + 3y = 5 + 2 o 2x + 5y = 7.

c) Si se multiplican miembro a miembro varias igualdades se obtiene otra igualdad.

Por ejemplo, se se multiplican por 4 los dos miembros de la igualdad iY = 2X2 se obtiene y = 8X2.

Análogamente, si los dos miembros de ~C = F - 32 se multiplican por ~ se obtiene 5 e = 9(F - 32).

d) Si se dividen miembro a miembro varias igualdades se obtiene otra igualdad, siempre que no se divida por cero.

Por ejemplo, si se dividen los dos miembros de la igualdad -4x = -12 por -4, se obtie­ne x = 3.

Análogamente, en la igualdad V = RI se pueden dividir los dos miembros por R =1= O, ob­teniéndose 1 = VI R . e) Si se elevan al mismo exponente los dos miembros de una igualdad se obtiene otra igualdad.

Por ejemplo, si T = 21t$g, tendremos ~ = (21t$g)2 = 41t21/g

f) Si se extrae la raíz enésima de los dos miembros de una igualdad se obtiene otra igualdad.

P . l . 3 3V 1 313V or eJemp o, SI r = 41t ' resu ta r = y ~ .

g) Los recíprocos de los miembros de una igualdad dan lugar a otra igualdad, siempre que no tenga lugar la división por cero.

Por ejemplo, si xl -31

, tendremos x = 3. Análogamente, si I - R¡ + R2 se verifica R - R ¡R2

R = R¡R2

R¡ + R 2 '

Las operaciones a) a f) se llaman axiomas de la igualdad.

ECUACIONES EQUIVALENTES. Son las que tienen las mismas soluciones.

Por ejemplo, x - 2 = O Y 2x = 4 tienen la solución común x = 2 y, por tanto, son equiva­lentes. Sin embargo, x - 2 = O Y X2 - 2x = O no son equivalentes, ya que X2 - 2x = O tiene, además, la solución x = O.

Las operaciones anteriores aplicadas a la transformación de ecuaciones no dan lugar, en to­dos los casos, a ecuaciones equivalentes a las primitivas. La aplicación de estas operaciones puede conducir a ecuaciones derivadas que tengan distintas solucioJ;les que la ecuación original.

Si se llega a una ecuación con más soluciones que la original, las soluciones nuevas se deno­minan extrañas y la ecuación derivada se llama redundante con respecto a la original. Si se llega a una ecuación con menos soluciones que la original, la ecuación derivada recibe el nombre de defectiva con respecto a la original.

Las operaciones a) y b) siempre conducen a ecuaciones equivalentes. Sin embargo, c) y e) pue~ den dar lugar a ecuaciones redundantes y soluciones extrañas y d) y f) a ecuaciones defectivas.

UNA FORMULA es una ecuación que expresa un hecho general, una regla o un principio.

Por ejemplo, la fórmula de geometría A = 1tr2 expresa el área A de un círculo en función de su radio r.

La fórmula fisica s = Ígt 2, en la que g es la aceleración de la gravedad y que vale, aproxima­

damente, 9,81 metros por segundo en cada segundo, expresa la relación que existe entre el espa­cio s, en metros, que recorre un cuerpo que cae libremente partiendo del reposo, y el tiempo t, en segundos, que emplea en el movimiento.

http://carlos2524.jimdo.com/

Page 77: Algebra Superior Murray R Spiegel

ECUACIONES EN GENERAL

tiene

Resolver una fórmula con respecto a una de las letras que figuran en ella es efectuar las mis-mas operaciones en ambos miembros de la misma hasta que aparezca la letra deseada aislada enuno de ellos.

Por ejemplo, si F == ma, se puede dividir por m obteniéndose a = Flm, con lo cual quedadespejada a en función de F y de m. Como comprobación, si se sustituye a = Flm en la ecuaciónoriginal se obtiene F = m(F/m), que es una identidad.tiene

e noUN TERMINO RACIONAL ENTERO con respecto a cierto número de incógnitas, x, y, z, ... ,

tiene la forma axPyqz' ... , en donde "los exponentes, p, q, r, ... , son números enteros y positi-vos, o cero, y el coeficiente a es independiente de las incógnitas. La suma de los exponentes, p + q+ r + ... , se denomina grado del término con respecto a las incógnitas x, y, z ,btie-

, ob-Ejemplos. 3x2 Z3, tx4, 6 son términos racionales enteros.

3X2Z3 es de grado 2 en x, 3 en z y 5 en x y ~.

!-X4 es de cuarto grado. 6 es de grado cero.Idad.

Idad.

4y í..- = 4yx-1 no es entero en x; 3xy' Y Z3 no es racional en y.x

Si al hablar del grado no se especifica a qué incógnitas se refiere, se sobren tiende que es conrespecto a todas las que figuran en el término.

e queUNA EXPRESION RACIONAL ENTERA, o polinomio de varias incógnitas, consta de térmi-

nos, cada uno de los cuales es racional y entero. El grado de la expresión viene dado por el corres-pondiente al término de mayor grado.rifica

Ejemplo. 3x3y4z + xy2z5 - 8x + 3 es una expresión racional entera de grado 3 en x, 4en y, 5 en z, 7 en x e y, 7 en y y z, 6 en x y Z y 8 en x, y y z.

UNA ECUACION RACIONAL ENTERA es una igualdad entre dos expresiones racionalesenteras. El grado de una ecuación es el correspondiente al término de mayor grado.

uiva-tiene, Ejemplo. xyz2 + 3xz = 2x3y + 3z2 es de grado 3 en x, 1 en y, 2 en z, 4 en x e y, 3 en)' y z,

3 en x y z y 4 en x, y y z.en to-puede1.

En una ecuación se pueden reducir los términos semejantes. Por ejemplo, 4x3)' + xZz - xy2= 4x3y + z, se puede escribir en la forma x2z - xy2 = Z.

deno-llegare de

Una ecuación se llama lineal si es de primer grado, y cuadrática si es de segundo grado. Aná-logamente, las de grados 3, 4 Y 5, reciben el nombre de ecuaciones de tercero, cuarto y quinto gra-do, respectivamente.

) pue-s.

Ejemplos. 2x + 3y = 7z es una ecuación lineal en x, y y Z.

x2 - 4xy + 5l = 10 es una ecuación cuadrática en x e y.

x3 + 3x2 - 4x - 6 = O es una ecuación de tercer grado en x.

ión de UNA ECUACION RACIONAL ENTERA DE GRADO 11 con respecto a la incógnita x, se pue-de escribir en la forma

xima-espa-t, en

aoxn + alxn-I + a2xn-Z + ... + an_Ix + an = O

siendo ao, al' ... , a; constantes y n un entero positivo.

69ECUACIONES EN GENERAL 69

Resolver una fórmula con respecto a una de las letras que figuran en ella es efectuar las mis­mas operaciones en ambos miembros de la misma hasta que aparezca la letra deseada aislada en uno de ellos.

Por ejemplo, si F == ma, se puede dividir por m obteniéndose a = F/m, con lo cual queda despejada a en función de F y de m. Como comprobación, si se sustituye a = F/m en la ecuación original se obtiene F = m(F/m), que es una identidad.

UN TERMINO RACIONAL ENTERO con respecto a cierto número de incógnitas, x, y, z, ... , tiene la forma axPyqz' ... , en donde ,los exponentes, p, q, r, ... , son números enteros y positi­vos, o cero, y el coeficiente a es independiente de las incógnitas. La suma de los exponentes, p + q + r + . .. , se denomina grado del término con respecto a las incógnitas x, y, z,

Ejemplos. 3X2 Z3, tx4, 6 son términos racionales enteros.

3X2Z

3 es de grado 2 en x, 3 en z y 5 en x y ~.

!-X4 es de cuarto grado. 6 es de grado cero.

4y í.. - = 4yx-¡ no es entero en x; 3xy' Y Z3 no es racional en y. x

Si al hablar del grado no se especifica a qué incógnitas se refiere, se sobrentiende que es con respecto a todas las que figuran en el término.

UNA EXPRESION RACIONAL ENTERA, o polinomio de varias incógnitas, consta de térmi­nos, cada uno de los cuales es racional y entero. El grado de la expresión viene dado por el corres­pondiente al término de mayor grado.

Ejemplo. 3x3y4z + xy2z 5 - 8x + 3 es una expreslOn racional entera de grado 3 en x, 4 en y, 5 en z, 7 en x e y, 7 en y y z, 6 en x y z y 8 en x, y y z.

UNA ECUACION RACIONAL ENTERA es una igualdad entre dos expresiones racionales enteras. El grado de una ecuación es el correspondiente al término de mayor grado.

Ejemplo. xyz2 + 3xz = 2x3y + 3z2 es de grado 3 en x, 1 en y, 2 en z, 4 en x e y, 3 en y y z, 3 en x y z y 4 en x, y y z.

En una ecuación se pueden reducir los términos semejantes. Por ejemplo, 4x3y + x 2 z - xy2 = 4x3y + z, se puede escribir en la forma x 2z - xy2 = Z.

Una ecuación se llama lineal si es de primer grado, y cuadrática si es de segundo grado. Aná­logamente, las de grados 3, 4 Y 5, reciben el nombre de ecuaciones de tercero, cuarto y quinto gra­do, respectivamente.

Ejemplos. 2x + 3y = 7z es una ecuación lineal en x, y y Z.

X2 - 4xy + 5l = 10 es una ecuación cuadrática en x e y.

x 3 + 3X2 - 4x - 6 = O es una ecuación de tercer grado en x.

UNA ECUACION RACIONAL ENTERA DE GRADO 11 con respecto a la incógnita x, se pue­de escribir en la forma

aoxn + a¡xn-¡ + a2xn- 2 + ... + an _ ¡x + an = O

siendo ao, a ¡, ... , an constantes y n un entero positivo.

http://carlos2524.jimdo.com/

Page 78: Algebra Superior Murray R Spiegel

70 ECUAC10NES EN GENERAL

Como casos particulares tendremos

aox + a¡ = O o ax + b = O

aox2 + a¡x + a2 = O o ax" + bx + e = O

aox3 + a¡x2 + a2x + a3 = O

aox4 + a¡x3 + a2x2 + a3x + a¿ = O

es de grado (ecuación lineal)

es de grado 2 (ecuación cuadrática)

es de grado 3 (ecuación cúbica)

es de grado 4 (ecuación cuártica)

PROBLEMAS RESUELTOS1. Determinar cuáles de las expresiones siguientes son ecuaciones y cuáles son identidades:

a) 3x - (x + 4) = 2(x - 2), 2x - 4 = 2x - 4; identidad.

b) (x - 1)(x + 1) = (x - 1)2, x2 - 1 = x2 - 2x + 1; ecuación.

e) (y - 3)' + 3(2y - 3) = y(y + 1) - y, y2 _ 6y + 9 + 6y - 9 = y2 + Y - y, y2 = y2; identidad

d) x + 3y - 5 = 2(x + 2y) + 3, x + 3y - 5 = 2x + 4y + 3; ecuación.

2. Comprobar si la solución o soluciones indicadas satisfacen las ecuaciones siguientes:

x xa) "2 + '3 = lO; x = 12.

_12+ _12__ ro2 3 ,6 + 4 = lO, Y x = 12 es solución.

x2 + 6xb) ~ = 3x - 2; x = 2, x = -1. 2

2 + 6(2) = 3(2) _ 22 + 2 '

Hí ..4 = 4, Y x = 2 es solución.

( - 1)2 + 6( - 1) - 5--.,------:-- = 3( - 1) - 2, - = - 5, Y x = - 1 es solución.-1 + 2 1

e) x2 - xy + y2 = 19; x = -2, y = 3; x = 4, y = 2 + fi; x = 2, y = -1.

x = -2, y = 3: (_2)2 - (-2)3 + 32 = 19 19 = 19, Y x = -2, y = 3 es solución.

; = 4, y = 2 + fi: 42 - 4(2 + fi) + (2 + fi)2 = 19, 16 - 8 - 4fi + (4 + 4fi + 7) = 19,

19 = 19 Y x = 4, Y = 2 + fi es solución.

x = 2, y = -1: 22 - 2( -1) + (_1)2 = 19. 7 = 19 Y x = 2, y = -1 no es solución.

3. Aplicar los axiomas de la igualdad para resolver las ecuaciones siguientes:

a) 2(x + 3) = 3(x - 1), 2x + 6 = 3x - 3.

Transponiendo términos: 2x - 3x = -6 - 3, -x = -9. Multiplicando por -1: x = 9.

Comprobación: 2(9 + 3) = 3(9 - 1), 24 = 24.

x xb) '3 + 6 = 1. Multiplicando por 6: 2x + x = 6, 3x = 6. Dividiendo por 3: x = 2.

Comprobación: 2/3 + 2/6 = 1, 1 = 1.

e) 3y - 2(y - 1) = 4(y + 2), 3y - 2y + 2 = 4y + & y + 2 = 4y + 8.

Transponiendo: y - 4y = 8 - 2, - 3y = 6.6

Dividiendo por - 3 : y = -=3 = - 2.

Comprobación: 3(-2) - 2(-2 - 1) = 4(-2 + 2), 0=0.

d)

cer

tipect

e)

raí

f)

g)

4. D

a)

b)

el

dl

5. H

a

b

70 ECUACIONES EN GENERAL

Como casos particulares tendremos

aox + a¡ = O o ax + b = O

aox2 + a¡x + a2 = O o ax2 + bx + e = O

aox3 + a¡x2 + a2x + a3 = O

aox4 + a¡x3 + a2x 2 + a3x + a4 = O

es de grado 1

es de grado 2

es de grado 3

es de grado 4

(ecuación

(ecuación

(ecuación

(ecuación

PROBLEMAS RESUELTOS

lineal)

cuadrática)

cúbica)

cuártica)

l. Detenninar cuáles de las expresiones siguientes son ecuaciones y cuáles son identidades:

a) 3x - (x + 4) = 2(x - 2), 2x - 4 = 2x - 4; identidad.

b) (x - I)(x + 1) = (x - 1)2, X2 - I = X2 - 2x + 1; ecuación.

e) (y - W + 3(2y - 3) = y(y + 1) - y, y2 _ 6y + 9 + 6y _ 9 = y2 + Y - y, y2 = y2; identidad

d) x + 3y - 5 = 2(x + 2y) + 3, x + 3y - 5 = 2x + 4y + 3; ecuación.

2. Comprobar si la solución o soluciones indicadas satisfacen las ecuaciones siguientes:

x x a) "2 + "3 = lO; x = 12.

_12 + _12 __ 10 2 3 ,6 + 4 = lO, Y x = 12 es solución.

X2 + 6x b) --- = 3x - 2' x = 2, x = - 1.

x + 2 '

22 + 6(2) 16' = 3(2) - 2 .. 2 + 2 ' 4" = 4, Y x = 2 es soluclOn.

( - 1)2 + 6( - 1) - 5 - ---:---::--- = 3( -1) - 2 - = - 5 Y x = - I es solución.

-1 + 2 '1 '

e) X2 - xy + y2 = 19; x = -2, y = 3; x = 4, Y = 2 + fi; x = 2, y = -1.

x = -2, y = 3: (_2)2 - (-2)3 + 32 = 19 19 = 19, Y x = - 2, y = 3 es solución.

; = 4, y = 2 + fi: 42 - 4(2 + fi) + (2 + fi)2 = 19, 16 - 8 - 4fi + (4 + 4fi + 7) = 19,

19 = 19 Y x = 4, Y = 2 + fi es solución.

x = 2, y = -1: 22 - 2(-1) + (_1)2 = 19, 7 = 19 Y x = 2, y = -1 no es solución.

3. Aplicar los axiomas de la igualdad para resolver las ecuaciones siguientes :

a) 2(x + 3) = 3(x - 1), 2x + 6 = 3x - 3.

Transponiendo ténninos: 2x - 3x = -6 - 3, -x = -9.

Comprobación: 2(9 + 3) = 3(9 - 1), 24 = 24.

x x b) "3 + 6 = 1. Multiplicando por 6: 2x + x = 6, 3x = 6.

Comprobación: 2/3 + 2/6 = 1, 1 = 1.

Multiplicando por - 1 : x = 9.

Dividiendo por 3: x = 2.

e) 3y - 2(y - 1) = 4(y + 2), 3y - 2y + 2 = 4y + ~ Y + 2 = 4y + 8.

Transponiendo: y - 4y = 8 - 2, - 3y = 6. 6

Dividiendo por -3: y = -3 = -2.

Comprobación: 3(-2) - 2(-2 - 1) = 4(-2 + 2), 0=0.

http://carlos2524.jimdo.com/

Page 79: Algebra Superior Murray R Spiegel

entidad

7) = 19,

ECUACIONES .EN GENERAL 71

2x - 3d) x _ 1

4x - 5x - 1

Multiplicando por x - 1, 2x - 3 = 4x - 5 o x = 1.

Comprobación: Sustituyendo x = 1 en la ecuación dada se obtiene -1/0 = -1/0. Como la división porcero carece de sentido, la ecuación dada no tiene solución.

2x - 3 4x - 5Obsérvese que 1) --- = --- y 2) 2x - 3 = 4x - 5 no son ecuaciones equivalentes. Si 1) se mul-

x - 1 x - 1

tiplica por x - 1 se introduce la solución extraña x = 1, y la ecuación 2) es redundante con respecto a laecuación 1).

e) x(x - 3) = 2(x - 3). Dividiendo ambos miembros por x - 3 se obtiene la solución x = 2.

Ahora bien, x - 3 = O, o x = 3, también es solución de la ecuación dada pero se ha perdido al dividir. Lasraíces buscadas son x = 2 y x = 3.

La ecuación x = 2 es defectiva con respecto a la dada.

fl F+2 = - 1. Elevando al cuadrado los dos miembros, x + 2 = 1, o sea x = - 1.

Comprobación: Sustituyendo x = -1 en la ecuación dada, fi = -1, es decir, 1= -1, que es un absurdo.

Por consiguiente, x = -1 es una solución extraña. La ecuación dada no tiene solución.

g) fo--=4 = 6. Elevando al cuadrado los dos miembros, 2x - 4 = 36, es decir, x = 20.

Comprobación: Si x = 20, J2(20) - 4 = 6, o sea J36 = 6.

Por tanto, x = 20 es una solución. En este caso no se han introducido soluciones extrañas.

4. Despejar, en las fórmulas siguientes, las incógnitas que se indican.

a) E = RI. Despejar R. Dividiendo los dos miembros por 1+ O, obtenemos R = E/l.

b) s = vot + !at2, despejar a.

Transponiendo, tat2 = s - vot. Multiplicando por 2, at? = 2(s- vot).

Dividiendo por t2 + O,2(s - vol)

a = -----,t2,---'--

1 1 1 .e) - = - + -, despejar- p.

f p q. 1 1 1 q-f

Transponiendo - = - - - = --'pfq fq'

fqTomando los recíprocos, p = -- (suponiendo q + f).

q-f

d) T = 21[.¡¡¡g, despejar g. Elevando al cuadrado los dos miembros,41[21

T2 =--g

Multiplicando por g, g'I? = 41[2/. Dividiendo por T2, g = 41[21/T2.

S. Hallar el valor de la incógnita que se indica conocidos los valores de las restantes.

9 9 9 5a) F = '5 C + 32, F = 68; hallar C. 68 = '5 C + 32, 36 = '5 C, C = 9 (36) = 20.

~ C = F _ 32 5 5. 55 ' C = 9 (F - 32) = 9(68 - 32) = 9 (36) = 20.Otro método.

1 1 1 1 1 1 1 1 1 5 - 2 1b) R = R, + R

2' R = 6, R, = 15; hallar R2· (; = 15+ R

2' R

2= (; -15 = ~ = 10' R2 = 10.

2x - 3 d) x _ 1

4x - 5 x - 1

ECUACIONES .EN GENERAL 71

Multiplicando por x - 1, 2x - 3 = 4x - 5 o x = 1.

Comprobación: Sustituyendo x = 1 en la ecuación dada se obtiene -1/0 = -1/0. Como la división por cero carece de sentido, la ecuación dada no tiene solución.

2x - 3 4x - 5 Obsérvese que 1) --- = --- y 2) 2x - 3 = 4x - 5 no son ecuaciones equivalentes. Si 1) se mul­

x -l x -l

tiplica por x - 1 se introduce la solución extraña x = 1, y la ecuación 2) es redundante con respecto a la ecuación 1).

e) x(x - 3) = 2(x - 3). Dividiendo ambos miembros por x - 3 se obtiene la solución x = 2.

Ahora bien, x - 3 = O, o x = 3, también es solución de la ecuación dada pero se ha perdido al dividir. Las raíces buscadas son x = 2 y x = 3.

La ecuación x = 2 es defectiva con respecto a la dada.

f) F+2 = - 1. Elevando al cuadrado los dos miembros, x + 2 = 1, o sea x = -1.

Comprobación: Sustituyendo x = -1 en la ecuación dada, ji = -1 , es decir, 1 = -1 , que es un absurdo.

Por consiguiente, x = - 1 es una solución extraña. La ecuación dada no tiene solución.

g) fo-=4 = 6. Elevando al cuadrado los dos miembros, 2x - 4 = 36, es decir, x = 20.

Comprobación: Si x = 20, J2(20) - 4 = 6, o sea J36 = 6.

Por tanto, x = 20 es una solución. En este caso no se han introducido soluciones extrañas.

4. Despejar, en las fórmulas siguientes, las incógnitas que se indican.

a) E = RI. Despejar R. Dividiendo los dos miembros por 1+ O, obtenemos R = E/l .

b) s = vot + !at2, despejar a.

Transponiendo, 1at2 = s - voto Multiplicando por 2, at2 = 2(s - vot).

2 ..1. O _ 2(s - vol} Dividiendo por t -r , a - t2 •

1 1 1. 1 1 1 q - f e) - = - + - , despejar· p. Transponiendo, - = - - - = --.

fpq pfq fq

Tomando los recíprocos, p = ~ (suponiendo q + f) . q-f

d) T = 2n.¡¡¡g, despejar g. Elevando al cuadrado los dos miembros,

Multiplicando por g, g]'1 = 4n2/. Dividiendo por T2

, g = 4n2 1/ T 2•

S. Hallar el valor de la incógnita que se indica conocidos los valores de las restantes.

9 9 9 5 a) F = "5 C + 32, F = 68; hallar C. 68 = "5 C + 32, 36 = "5 c, C = 9 (36) = 20.

Otro método. 9 5 5 . 5 "5 C = F - 32, C = 9 (F - 32) = 9(68 - 32) = 9 (36) = 20.

1 1 1 b) Ji. = R, + R

2' R = 6, R,

1 1 1 1 1 1 5 - 2 1 15 ; hallar R 2 • (; = 15 + R

2 ' R

2 = (; -15 = ~ = 10' R 2 = 10.

http://carlos2524.jimdo.com/

Page 80: Algebra Superior Murray R Spiegel

72 ECUACIONES EN GENERAL

Otro método.RR, 6(15)

R2 = --- = -- = 10.R, - R 15 - 6

28811: = ~m·33 '

,.3 = 28811: = 216.411:(3

v = 28811:; hallar r. r = 6.e)

Otro método.3 3V

r =-,411:

_ ~3V _ V3(28811:) _ 3f)i(;6 _ 6r - - - v 21 - .411: 411:

6. Determinar el grado de las ecuaciones siguientes con respecto a las incógnitas que se indican:

a) 2X2 + xy - 3 = O: x; y; x e y.

Grado 2 en x, 1 en y, 2 en x e y.

b) 3xy2 - 4y2z + 5x - 3y = x4 + 2: x; z; y y z; x, .l' y •.

Grado 4 en x, 1 en z, 3 en y y Z, 4 en x, y y z:

2 3e) x = -- x; x y z; x, y y z.

y + z

La ecuación, escrita en esta forma, no es racional entera. Sin embargo, se puede transformar en dicho tiposi se multiplica por y + z, obteniéndose x2(}, + z) = 3, o sea x2y + x2z = 3. Esta ecuación derivada es racionalentera de segundo grado en x, de tercero en x y z y de tercero, también, en x, y y z .

d) Jx+3 = x + y: y; x e y.

Igualmente, esta ecuación se puede transformar en racional entera elevando al cuadrado sus dos miembros.Así, pues, x + 3 = x2 + 2xy + y2, que es de segundo grado en y y de segundo, también, en x e y.

Hay que tener en cuenta, sin embargo, que las ecuaciones no son equivalentes, ya que x2 + 2xy + y2 = X + 3incluye tanto a Jx+3 = x + y como a -Jx+3 = x + y.

7. Hallar los valores de x para los cuales a) x2 = 81, b) (x - 1)2 = 4.

a) Como nada se dice sobre si debe ser positivo o negativo, tendremos que considerar las dos posibilidades. Ex-trayendo la raíz cuadrada de los dos miembros de la ecuación se obtiene P = J8l = 9. Ahora bien, prepresenta un número positivo (o cero) si x es real. Por consiguiente, p = x si x es positivo, mientras quep = -x si x es negativo. En resumen, cuando se escribe p debemos considerar que es igual a x (si x> O)

o -x (si x < O). Por ejemplo, la ecuación p = 9 equivale a x = 9 o a -x = 9 (es decir, x = -9). Las dossoluciones se representan por x = ±9.

b) (x - 1)2 = 4, ± (x - 1) = 2 o bien (x - 1) = ± 2, las dos raíces son x = 3 Y x = - 1.

8. Encontrar el error cometido en el siguiente razonamiento:

a) Sea x = y:

b) Se multiplican los dos miembros por x:e) Se resta y2 a ambos miembros:d) Se escribe el resultado en la forma siguiente:e) Se dividen los dos miembros por x - y:f) Se sustituye x por su igual, y:g) De aquí resulta:h) Dividiendo por y:

x=yx2 = xy

x2 _ y2 = xy _ y2(x - y)(x + y) = y(x - y)

x+y=yy+y=y

2y = Y2 = 1

No hay nada que objetar a las operaciones efectuadas en .a), b), e), d).

Sin embargo, en e) se divide por x - y y esto no es válido, ya que, por hipótesis, el divisor es igual a cero.Como la división por cero carece de sentido, todo lo que se haga a partir de e) es falso.

9. Dde

a

10. r

a

b

e

d

11. e

a

b

d

12. ¡.

a

13. [

http://carlos2524.jimdo.com/

Page 81: Algebra Superior Murray R Spiegel

icho tiporacional

iembros.

ades. Ex-ien,pntras que(si x > O)). Las dos

al a cero.

ECUACIONES EN GENERAL 73

9. Demostrar que yÍ2 es un número irracional, es decir, un número que no se puede representar por el cociente dedos enteros.

Supongamos que yÍ2 = pjq siendo p y q dos números enteros que no tengan más divisores comunes que launidad, ± 1, es decir,pjq es una fracción irreducible. Elevando al cuadrado los dos miembros se obtienep2jq2 = 2,o sea p2 = 2q2. Como 2q2 es un número par, p2 será par y p también (si p fuera impar, p2 también lo seria);por tanto, p = 2k, siendo k un número entero. Así, pues, p2 = 2q2 también se puede escribir en la forma(2k)2 = 2q2, o sea q2 = 2k2; en consecuencia, q y q2 también son números pares. Pero si p y q son ambos pares,deberán tener el divisor común 2, en contra de la hipótesis hecha de que solo admitían como divisores comunesa la unidad, ± 1. Por consiguiente, yÍ2 es irracional.

PROBLEMAS PROPUESTOS10. Determinar cuáles de las expresiones siguientes son ecuaciones y cuáles son identidades:

a) 2x + 3 - (2 - x) = 4x - 1 e)9x2 _ 4y2

= 2x + 3y3x - 2y

b) (2y - 1)2 + (2y + 1)2 = (2y)2 + 6 J) (x - 3)(x2 + 3x + 9) = x3 - 27

x2 x2e) 2{x + 4 - 3(2x - I)} = 3(4 - 3x) + 2 - x g) _ + _ = x2

4 12

d) (x + 2y)(x - 2y) - (x - 2y)2 + 4y(2y - x) = O h) (x2 _ y2)2 + (2xy)2 = (x2 + y2)2

11. Comprobar si la solución o soluciones indicadas satisfacen las ecuaciones.

y2 _ 4a) -- = 2y - l : Y = 3

y-2 '

1 1 1e) - + - = --; x = 3

x 2x x-I

b) x2 - 3x = 4; -1,-4 J) y3 + y2 - 5y - 5 = O; ±j5,-1

e) Fx-=2 - Fx'+2 = 4; 34, 2 g) x2 - 2y = 3y2; X = 4, Y = 2; x = 1, Y = -1

d) x3 - 6x2 + llx - 6 = O; 1, 2, 3 h) (x + y)2 + (x - y)2 = 2(x2 + y2); cualquier valor de x, y

12. Aplicar los axiomas de la igualdad para resolver las ecuaciones siguientes. Comprobar las soluciones obtenidas.

a) 5(x - 4) = 2(x + 1) - 7x+1 x-I

d) --=--x-I x-2

g) fo+l + 5 = O

b) 2y _ ~ = 23 6

3x-2 x+2e) x _ 2 = x - 2

i) (y + 1)2 = 161 3

e) - = 8 --y y

J) Fx-=2 = 4j) (2x + 1)2 + (2x - 1)2 = 34

13. Despejar la incógnita que se indica en las fórmulas siguientes:

a) PIVI = P2V2; T2 d) v2 = v~ + 2as; a

TI T2

b) t = ~;s e) T= 2n ~'kk'

1 S = ~ [2a + (n - l)d]; de) m = - J2a2 + 2b2 - e2; e J)2

http://carlos2524.jimdo.com/

Page 82: Algebra Superior Murray R Spiegel

74 ECUACIONES EN GENERAL

14. Hallar el valor de la incógnita que se indica conocidos los valores de las restantes.

a) v = Vo + al; hallar a si v = 20, Vo = 30, I = 5.

b) S = ~ (a + d); hallar d si S = 570, n = 20, a = 40.

1 1 1e) - = - + -; hallar q si 1= 30, p = lO.I p q

d) Fs = imv2; hallar v si F = 100, s= 5, m = 2,5.1

e) 1= rr;;; hallar C con cuatro cifras decimales si 1=21tvLC

1000, L = 4· 10-6.

15. Determinar el grado de las ecuaciones siguientes con respecto a las incógnitas que se indican:

a) x3 - 3x + 2 = O : xb) x2 + xy + 3y4 = 6: x; y; x e ye) 2xy3 - 3x2y2 + 4xy = 2X3 : x; y; x e yd) xy + yz + xz + z2x = y4: x; y; z ; x y z; y y z; x, y y z

16. Clasificar las ecuaciones siguientes (transformándolas si es preciso) según sean lineales, cuadráticas, de tercero,cuarto o quinto grado con respecto a todas las incógnitas que figuran en ellas.

a) 2X4 + 3x3 - X - 5 = O

b) x - 2y = 4

e) 2X2 + 3xy + y2 = lOd) X2y3 - 2xyz = 4 + y5

e) JX2 + y2 - 1 = x + y

1) 2x+Y=4x - 3y

g) 3),.2 - 4y + 2 = 2lv - Wh) (z + 1)2 (z - 2) = O

17. La ecuación J(x + 4)2 = X + 4, ¿es una identidad? Razónese la respuesta.

18. Demostrar que J3 es un número irracional.

SOLUCIONES DE LOS PROBLEMAS PROPUESTOS

10. a) Ecuaciónb) Ecuación

e) Ecuación1) Identidad

g) Ecuación11) Identidad

e) Identidadd) Identidad

11. a) y = 3 es soluciónb) x = -1 es solución, x = -4 no lo ese) x = 34 es solución, x = 2 no lo esd) x = 1, 2, 3 son todas soluciones

e) x = 3 es solución1) y = ± j5, - 1 son todas solucionesg) x = 4, Y = 2; x = 1, Y = -1 son solucionesh) La ecuación es una identidad; luego todos los va-

lores de x e y son soluciones

12. a) x = 5b) Y = 4

e) y = 1/2d) x = 3

e) no tiene solución1) .v = 6

i) y=3,-5j) x = ±2

g) no tiene solución11) x = I

P2V2T, 41t2m13. a) T2 = --- e) e = ±J2a2 + 2b2 - 4m2 e) k=--

P,V, T2

S = !gt2 d)['2 - V5 f) d=

2S - 2anb) a=--- 2.1 n(n - l)

14. a) a= -2 b) d= 17 e) q= -15 d) t' = ±20 e) C = 0,0063

15. a) 3 b) 2, 4, 4 el 3, 3, 4 d) 1, 4. 2. 3, 4, 4

16. a) cuarto grado e) cuadrática e) cuadrática g) cuadráticab) lineal d) de quinto grado f) lineal 11) de tercer grado

17, J(x + 4)2 = X + 4 solo si x + 4 ~ O; J(x + 4)2 = - (x + 4) si x + 4 ;::¡; O.La ecuación dada no es una identidad.

UNA \tan;

las

FUNCIIentreste

tom

siónte, .

repr

[ort.

cadciór

porsi -'

VARIAresdenDel

http://carlos2524.jimdo.com/

Page 83: Algebra Superior Murray R Spiegel

CAPITULO 10

Funciones y gráficas

UNA VARIABLE es un símbolo al que se le puede asignar un conjunto de valores. Una cons-tante es un símbolo al que solo se le puede asignar un valor.

Para representar las variables se emplean las letras finales del alfabeto, x, y, z, u, v, W, y paralas constantes se emplean las primeras, a, b, c.

FUNCION DE UNA VARIABLE. Una variable y es función de otra x si existe una relaciónentre ambas, x e y, de forma que a cada valor de x le corresponda uno, o más, de y. Eneste capítulo, solo consideramos números y funciones reales.

Ejemplo 1. )' = x2 - 5x + 2 establece una relación entre las variables x e y. Cuando xtoma los valores x = O, 1, 2, - 1, los correspondientes de y son y = 2, - 2, - 4, 8, respectivamente.

Ejemplo 2. La longitud e de la circunferencia es una función del radio r dada por la expre-sión e = Znr. Las longitudes de las circunferencias de radios 1, 3, 5 (metros) son, respectivamen-te, 2n, 6n, IOn (metros).

Ejemplo 3. La población y de una nación es función del año x. En el cuadro siguiente serepresenta la población de los Estados Unidos con intervalos de diez años entre 1880-1950.

Año x 1880 1890 1900 1910 1920 1930 1940 1950Población y

50,2 62,9 76,0 92,0 105,7 122,8 131,7 150,7en millones

Cuando a cada valor de x le corresponde un solo valor de y, se dice que y es una función uni-forme de x; en caso contrario, y es una función multiforme de x.

Por consiguiente, en los Ejemplos 1 y 3 anteriores, y es una función uniforme de x, ya que acada valor de x le corresponde uno, y solo uno, de y. Análogamente, en el Ejemplo 2, e es una fun-ción uniforme de r,

Sin embargo, y = ±Jx es una función multiforme de x, ya que a cada valor de x le corres-ponden dos valores de y (excepto para la solución trivial x = O). Por ejemplo, si x = 4, Y = ± 2;si x = 5, y = ± fi, etc.

VARIABLES INDEPENDIENTES Y DEPENDIENTES. La variable a la que se asignan valo-res (x en los ejemplos anteriores) se denomina variable independiente; la.variable cuyo valor vienedeterminado por el que toma aquélla (ven los ejemplos) se llama cariable dependiente o función.Decir que y es función de x equivale a decir que y depende de x.

75

http://carlos2524.jimdo.com/

Page 84: Algebra Superior Murray R Spiegel

11 1

2 --------,P(3.2)II

1 I

-2 II

X'O

X GRA-~ -~ -5 ¡ -1 1 2 ~I -1 dIII -2II

Q(-2.-3)L----- -5nI IV-~

y'

76 FUNCIONES Y GRAFICAS

El conjunto de valores que puede tomar la variable independiente recibe el nombre de cam-po de variación de la variable.

En el Ejemplo 1, x puede ser cualquier número real.

En el Ejemplo 2, la variable independiente es el radio r de la circunferencia. El campo de va-riación de r es el conjunto de todos los números positivos y el cero.

En el Ejemplo 3, el campo de variación de x está formado por los años 1880, ... , 1950.

En la función y = ±Jx, si se quiere que y sea siempre real, los valores que se pueden asig-nar a la variable independiente x es el conjunto de todos los números mayores o igual a cero, locual se representa por x ;?; O.

En la función y = (x _ l~x + 2) la variable x puede tomar cualquier valor real excepto

x = 1 y x = - 2, para los cuales la función y no está definida. Por consiguiente, el campo de va-riación está constituido por el conjunto de los números reales excepto 1 y - 2.

LA NOTACION FUNCIONAL y = f(x), que se lee «y igual a f de x», es la que se utiliza pararepresentar que y es una función de x. Según esta notación, f(a) significa el valor de la varia-ble dependiente y cuando x = a (siempre que dicho valor exista).

Así, pues, y = x2 - 5x + 2 se puede escribir f(x) = x2 - 5x + 2. Por tanto, f(2), que esel valordef(x) o y cuando x = 2, esf(2) = 22 - 5(2) + 2 = -4. Análogamente,J( -1) = (_1)2- 5(-1) + 2 =8.

En la notación funcional se puede emplear una letra cualquiera; esto es, g(x), h(x), F(x), etc., re-presentan, asimismo, funciones de x.

SISTEMA DE COORDENADAS RECTANGULARES. Se utiliza para representar, gráficamen-te, una relación entre dos variables.

y

Sean X'X e Y'Y dos rectas perpendiculares entre sí que se cortan en el punto O, como indicala figura.

La recta X'X, denominada eje x, se sitúa normalmente en posición horizontal.

La recta Y' Y, denominada eje y, se sitúa normalmente en posición vertical.

El punto O recibe el nombre de origen del sistema.

Empleando una unidad de longitud adecuada se pueden situar sobre el eje x, a la derecha e

LA

FUN

j

1. E:el

al

bl

el

http://carlos2524.jimdo.com/

Page 85: Algebra Superior Murray R Spiegel

va-

va-

araría-

., re-

en-

FUNCIONES Y GRAFICAS

izquierda del origen O. los puntos 1,2,3.4, ... ,y -1, -2, -3, -4, ... , sin más que ir toman-do, sucesivamente, dicha unidad de longitud. En la figura hemos tomado arbitrariamente OX comosemieje positivo; esto es lo más corriente, aunque no obligatorio.

Asimismo, hemos tomado O Y como semieje positivo. Es también normal utilizar la mismaunidad de longitud sobre ambos ejes, aunque tampoco es obligatorio.

Los ejes x e y dividen al plano en 4 regiones o cuadrantes, denominados 1, I1, III YIV, como seindica en la figura.

Sea P un punto cualquiera del plano xy. Trazando desde P las perpendiculares a los ejes xe y, los valores de x e y de los puntos de intersección de dichas perpendiculares con los ejes deter-minan, respectivamente, la coordenada x (abscisa) y la coordenada y (ordenada) del punto P. Estascoordenadas se representan por el símbolo (x, y).

Recíprocamente, dadas las coordenadas de un punto, se puede situar éste en el plano xy.

Por ejemplo, las coordenadas del punto P de la figura son (3, 2). El punto cuyas coordena-das son (-2, -3) es Q.

LA GRAFICA DE UNA FUNCION y = f(x) es el lugar geométrico de los puntos (x, y) -quesatisfacen a la ecuación y = f(x) .

FUNCION DE DOS VARIABLES. Se dice que una variable z es función de las variables x e ysi existe una relación tal que a cada par de valores de x e y le corresponde uno, o más, valores dez. En este caso, x e y son variables independientes, y z es la variable dependiente o función.

La notación funcional que se utiliza es z = f(x, y), que se lee «z igual af de x e y». Entonces,fia, b) representa el valor de z cuando x = a e y = b, siempre que la función esté definida paradichos valores.

Por ejemplo, si f(x, y) = x3 + xy2 - 2x, tendremos f(2, 3) = 23 + 2 - 32 - 2 - 3 = 20.

De igual forma se definirían las funciones de más de dos variables.

GRAFICOS ESTADISTICOS. Son aquellos que representan una relación entre datos obtenidosde observaciones efectuadas en experimentos científicos, censos, operaciones comerciales, etc.

PROBLEMAS RESUELTOS

l. Expresar el área A de un cuadrado en función de su a) lado x, b) perímetro P,e) diagonal D. ,,,,,,

',D,,-, -,,,,

a) A = x2

Pb) P = 4-, o sea x = 4'

P p2

Por tanto, A = x2 = (4)2 o bien A = \6%

D D ~e) D = ';X2 + x2 = xfi o sea x = ¡;:" Por tanto. A = x2 = ( ¡;:,)2 o bien A =-

y2 ",2 2

77FUNCIONES Y GRAFICAS 77

izquierda del origen O, los puntos 1, 2, 3, 4, ... , y - 1, - 2, - 3, - 4, . .. , sin más que ir toman­do, sucesivamente, dicha unidad de longitud . En la figura hemos tomado arbitrariamente OX como semieje positivo; esto es lo más corriente, aunque no obligatorio.

Asimismo, hemos tomado O Y como semieje positivo. Es también normal utilizar la misma unidad de longitud sobre ambos ejes, aunque tampoco es obligatorio.

Los ejes x e y dividen al plano en 4 regiones o cuadrantes, denominados 1, 1I, 11I Y IV, como se indica en la figura.

Sea P un punto cualquiera del plano xy. Trazando desde P las perpendiculares a los ejes x e y, los valores de x e y de los puntos de intersección de dichas perpendiculares con los ejes deter­minan, respectivamente, la coordenada x (abscisa) y la coordenada y (ordenada) del punto P. Estas coordenadas se representan por el símbolo (x, y).

Recíprocamente, dadas las coordenadas de un punto, se puede situar éste en el plano xy.

Por ejemplo, las coordenadas del punto P de la figura son (3 , 2). El punto cuyas coordena­das son (- 2, - 3) es Q.

LA GRAFICA DE UNA FUNCION y = f(x) es el lugar geométrico de los puntos (x, y) ·que satisfacen a la ecuación y = f(x).

FUNCION DE DOS VARIABLES. Se dice que una variable z es función de las variables x e y si existe una relación tal que a cada par de valores de x e y le corresponde uno, o más, valores de z. En este caso , x e y son variables independientes, y z es la variable dependiente o función.

La notación funcional que se utiliza es z = f(x, y), que se lee «z igual af de x e y». Entonces, f(a, b) representa el valor de z cuando x = a e y = b, siempre que la función esté definida para dichos valores.

Por ejemplo, ,si f(x, y) = x 3 + xy2 - 2x, tendremos f(2 , 3) = 23 + 2' 32 - 2' 3 = 20.

De igual forma se definirían las funciones de más de dos variables.

GRAFICOS ESTADISTICOS. Son aquellos que representan una relación entre datos obtenidos de observaciones efectuadas en experimentos científicos, censos, operaciones comerciales, etc.

PROBLEMAS RESUELTOS

1. Expresar el área A de un cuadrado en función de su a) lado x, b) perímetro P, e) diagonal D.

a) A = X2

P b) P = 4.\" o sea x = 4'

P p 2

Por tanto, A = X2 = (_ )2 o bien A = -4 16

D D ~ e) D = ';X2 + X2 = xfi o sea x = ¡;:" Por tanto. A = X2 = ( ¡;:,)2 o bien A =-

y2 ..,;2 2

%

"

http://carlos2524.jimdo.com/

Page 86: Algebra Superior Murray R Spiegel

78 FUNCIONES Y GRAFICAS

2. Expresar a) el área A, b) el perímetro P y e) la diagonal D de un rectángulo en función de sus lados x e y.Véase Figura (a).

a) A = xy, b) P = 2x + 2y, e) D = JX2 + y2

y

!sFig. (b) Probo 3

r

Fig. (a) Probo 2

3. Expresar a) la altura h y b) el área A de un triángulo equilátero en función de su lado S. Véase Figura (b).

b) A = ~hs = ~Fj3)s= s2j32 2 2 4

4. El área de una superficie esférica es S = 4nr2, y su volumen, V = -inr3.

Expresar a) r en función de S y de V, b) Ven función de S y e) S en función de V.

a) De S = 4nr2 se obtiene r = . rs _ 1 • rsy¡; - '2 y-;.4 ~ [3v

De V = 3nr3 se deduce r = y¡;

b) S· d 1 ~ V -_ 4nr3 resulta V 4 1 /f; 3 S ~ustituyen o r = -2 -n en -3 = -n(- -) = - -3 2 . n 6 n

e) Sustituyendo r = ,3f3V en S = 4nr2 se deduce S = 4n V(3V)2 = 4n 3 90 . 4n = .,y36nV2

V ¡; 4n 16n2 4n

5. Dada la función y = 3.'(2- 4x + 1, hallar los valores de y correspondientes a x = -2, -1, O, 1,2.

Para ~ = -2, Y = 3(-2)2 - 4(-2) + 1 = 21; para x = -1, Y = 3(-lf - 4(-1) + 1 = 8; para x = O,y=3(W-4(0)+I=I; para x=l, y=3(1)2-4(1)+I=,0; para x=2, y=3(2)2-4(2)+1=5.Estos valores de x e y figuran en la tabla siguiente:

x -2 -1 O 1 2

Y 21 8 1 O 5

6. Ampliar la tabla de valores del Problema 5 calculando los valores de y correspondientes a x = - 3/2, -1/2,1/2, 3/2, '

Para x = - 3/2, Y = 3(- 3/2)2 - 4(- 3/2) + 1 = 13i, etc. En la tabla siguiente se resumen los resultados,

x -2 -~ -1 _!. O !. 1 '!. 22 2 2 2

Y 21 13~ 8 3~ 1 _1- O 1:! 54 4 4

NOl

7.

8.

9.

10.

http://carlos2524.jimdo.com/

Page 87: Algebra Superior Murray R Spiegel

y.

(b).

O,5.

1/2,

dos.

FUNCIONES Y GRAFICAS 79NOTACION FUNCIONAL

7. Siendo f(x) = x' - 5x - 2, hallar f( - 2), f( - 3/2), f( -1), feO), f(I), f(2).

f(-2) = (_2)' - 5(-2) - 2 = O

f(-3/2) = (-3/2)' - 5(-3/2) - 2 = 17/8

f( -1) = (-1)' - 5(- 1) - 2 = 2

f(O) = O' - 5(0) - 2 = -2

f(l) = 13 - 5(1) - 2 = -6

f(2) = 23 - 5(2) - 2 = -4

Los valores se pueden disponer en forma de tabla:

% -2 -3/2 -1 O 1 2f(%) O 17/8 2 -2 -6 -4

t3 + 2t8. Siendo F(t) = --¡-=-¡, hallar F(-2), F(x), F(-x).

(-2)3+2(-2) -8-4F(-2) = =--=4

-2 - 1 -3

F(x)

(_X)3 + 2(-x) _x3 - 2x x3 + 2xF(-x) = = =--

-x - 1 -x - 1 x + 1

3x - 1 x - 19. Dada la función R(x) = 4x + 2' hallar a) R(x + 2)' b) R(x + h) - R(x)

he) R[R(x)].

x-l 2x-53(--) - 1

a) R(x - 1) = x + 2 x + 2 2x - 5x+2 4(x-l)+2 =~=~

x+2 x+2

b) R(x + h) - R(x) = ~{R(x + h) _ R(x)} = ~{3(x + h) - 1 _ 3x - 1}h h h 4(x + h) + 2 4x + 2

= ~P(x + h) - 1][4x + 2] - [3x - 1][4(x + h) + 21} = 5h [4(x + h) + 2][4x + 2] 2(2x + 2h + 1)(2x + 1)

3x - 13x - 1 3(4x + 2) - 1

e) R[R(x)] = R(4x + 2) = 3x _ 1

4(4x + 2) + 2

5x - 5 x - 1~=---¡;-

F(x, y + k) - F(x, y)10. Siendo F(x, y) = x3 - 3xy + y2, hallar a) F(2, 3), b) F(-3, O), e) k .

a) F(2, 3) = 2' - 3(2)(3) + 32 = -1

b) F(-3, O) = (_3)3 - 3(-3)(0) + 02 = -27

e) F(x, y + k) - F(x, y) x3 - 3x(y + k) + (y + k¡Z - [x' - 3xy + y2] 3 2 kk = k =-x+y:+

http://carlos2524.jimdo.com/

Page 88: Algebra Superior Murray R Spiegel

80 FUNCIONES Y GRAFICAS

COORDENADAS RECTANGULARES Y GRAFICAS

11. Representar en un sistema de coordenadas rectangulares los puntos siguientes: (2,1), (4, 3), (-2.4). (-4, 2),

(-4, -2), (-5/2, -9/2), (4, -3), (2, -)2). Véase la Figura (e).

y,.(-2.4) ~

5.(-4.2) 2

1

y

-~ -~ -5 -2 -l

.(-4.-2)

-H~r-HfH+++X

.(4.3)

(2.1)~-+~--+-~~~--~+-~-+x

_ll 2 5 ~ ~-2 (2.:12)-5 .(4.-3)-~-~(-5/2.-9/2).

Fig. (e) Probo 11 Fig. (d) Probo 12 Fig. (e) Probo 13

12. Siendo y = 2x - 1, calcular los valores de y correspondientes a x = - 3, - 2, - 1, 0, 1, 2, 3 Y representar lospuntos (x, y) obtenidos.

La tabla siguiente contiene los valores de y correspondientes a los valores dados de X.

x -3 -2 -1 O 1 2 3 Iy -7 -5 -3 -1 1 3 5 I

Los puntos (-3, -7), (-2, -5), (-1, -3), (0, -1), (1,1),(2,3), (3,5)se han representado en la Figura (d).

Obsérvese que todos los puntos que satisfacen a la ecuación y = 2x - 1 están situados sobre una recta. Laecuación de una recta, en general, viene dada por y = ax + b, siendo a y b constantes; por esta razón, y = ax + b,o f(x) = ax + b, recibe el nombre de función lineal. Como una recta queda definida por dos puntos, solo habráque situar dos puntos y unidos entre sí para obtener la representación de aquélla.

13, Representar gráficamente la función definida por y = x2- 2x - 8 o f(x) = x2

- 2x - 8.

En la tabla siguiente figuran los valores de y o f(x) para varios valores de X.

-2 -1 O 5 67 16o -5 -8

Los puntos representados en la gráfica son: (- 4, 16), (- 3, 7), (- 2, O), (- 1, - 5), etc.

Para representar estos puntos es conveniente utilizar escalas diferentes sobre los ejes x e y, como apareceen la Fig. (e). Los puntos señalados con una cruz, x , se han añadido a los ya calculados con objeto deobtener una representación más precisa.

La curva obtenida es una parábola. Su punto más bajo, P. que es un mínimo de la función, recibe el nombrede vértice de la parábola.

14, Dibujar la función definida por y = 3 - 2x - x2•

x -5 -4 -3 -2 -1 O 1 2 3 4

Y -12 -5 O 3 4 3 O -5 -12 -21

m~qumi

15. Re

ha)

resel,

16. Re

tar

(-

rac

cin

un:res

http://carlos2524.jimdo.com/

Page 89: Algebra Superior Murray R Spiegel

b,rá

ede

re

FUNCIONES Y GRAFICAS 81

La curva obtenida es una parábola, como indica la Fig. if). El punto Q( -1,4), vértice de la parábola, es unmáximo. En general, y = ax" + bx + e representa una parábola cuyo vértice es un máximo o un mínimo segúnque el coeficiente a sea negativo o positivo, respectivamente. La función ¡(x) = ax2 + bx + e se llama trino-mio de segundo grado o función cuadrática.

y

Fig. (f) Probo 14

15. Representar la función y = x3 + 2X2 - 7x - 3.

y

Fig. (g)

x -4 -3 -2 -1 O 1 2 3

Y -7 9 11 5 -3 -7 -1 21

La gráfica se representa en la Fig. (g). Los puntos señalados con una cruz, x, no figuran en la tabla; sehan añadido a aquéllos para conseguir una representación más precisa.

El punto A se denomina máximo relativo; no es el punto más alto de toda la curva, pero sí lo es conrespecto a los puntos situados a ambos lados de él. El punto B recibe el nombre de mínimo relativo. Medianteel cálculo diferencial se pueden determinar los máximos y mínimos relativos de una función.

16. Representar gráficamente la función x2 + y2 = 36.

Se puede escribir y2 = 36 - X2, de donde y = ±J36 - x2. Para que y sea real, los valores de x deben es-tar comprendidos entre - 6 Y + 6.

x -6 -5 -4 -3 -2 -1 O 1 2 3 4 5 6

Y O ± m ±120 ± /27 ±132 ±V35 ±6 ±V35 ±132 =/27 ±120 ±m O

Los puntos a representar son (-6, O), (-5, fo),(-5, -fo), (-4, fo), (-4, -fo), etc.

La figura adjunta representa una circunferencia deradio 6.

En general, la ecuación x2 + y2 = a2 es la de unacircunferencia de centro el origen y radio a.

Se debe tener en cuenta que, si no se toma la mismaunidad de longitud sobre los ejes x e y, la gráfica queresulta no es la correspondiente a una circunferencia.

y

http://carlos2524.jimdo.com/

Page 90: Algebra Superior Murray R Spiegel

82 FUNCIONES Y GRAFICAS

ALGUNAS APLICACIONES DE LAS REPRESENTACIONES GRAFICAS

17. Un hombre dispone de 40 m de alambrada para cercar un jardín rectangular. Sabiendo que solo debe colocarlasobre tres lados, porque el cuarto limita con su casa, determinar el área máxima que puede proteger.

Sea x = longitud de dos de los lados del rectángulo; la longitud del tercero será 40 - 2x.

El área A del jardín es A = x(40 - 2x) = 40x - 2X2. Se trata de hallar el valor máximo de A. Se hace unatabla de valores y se representa, gráficamente, la función. Es fácil de ver que los valores de x deben estar com-prendidos entre O y 20 m para que A sea positivo.

A'00

Casa p

40-2%

Las coordenadas del máximo P de la curva son (lO, 2(0), con lo cual las dimensiones del jardín han de serlO m y 20 m y su área resulta de 200 ml.

18. Las dimensiones de una placa rectangular de estaño son 12 por 18 cm. Con ella se desea construir un cajón sintapa cortando en sus esquinas unos cuadrados y doblando los lados. Calcular el lado de los cuadrados que sedeben cortar para que el cajón que resulte sea de volumen máximo.

Sea x el lado del cuadrado que se ha de cortar en cada esquina. El volumen V del cajón que así resulta esV = x(l2 - 2x)(18 - 2x). Es fácil de ver que x debe estar comprendido entre O y 6 cm para que se puedarealizar.

x O 1 2 21'z 3 31'z 4 5 6

V O 160 224 2271'z 216 1921'z 160 80 O

D-_o,

: %:x '}12-2%

, ,. .\. .•• - 4

10-18 - 2.&-oj

V

19.

20.

http://carlos2524.jimdo.com/

Page 91: Algebra Superior Murray R Spiegel

rla

unaITI-

FUNCIONES Y GRAFICAS

De la gráfica se deduce que el valor de x que corresponde al máximo valor de V está comprendido entre 2y 2,5 cm. Dibujando más puntos, se ve que es, aproximadamente, 2,4 cm.

Todos los problemas de este tipo y los análogos al 17 se pueden resolver más fácilmente, y de forma exacta,acudiendo al cálculo diferencial.

19. Se quiere fabricar un bote cilíndrico de 200 cm" de volumen. Calcular las dimensiones que debe tener para emplearla menor cantidad posible de material.

Sean x e y el radio y la altura, respectivamente, del cilindro.

El área de las bases superior e inferior es 1tX2, y el de la superficie lateral, 21txy; por consiguiente, el áreatotal es S = 21tx2 + 21txy.

200El volumen del cilindro es 1tX2y, luego 1tX2y = 200 e y = -2' Por tanto,

1tX

. 2 200S = 21tx + 21tx (-2)

1tXo

400S = 21tx2 +-

X

Se hace una labia de valores y se representa, gráficamente, la función S. Se ha tomado 1t = 3,14.

x 1 2 3 3,2 3,5 4 4,5 5 6 7 I 8S 406 225 190 189 191 200 216 237 293 I 365 I 452

S

5~0

500

2~0

200y

1~0

100--------

,, ,, ~Ota esueda O 1

----~~----~---L----~--~----~----~--~----~--x

De la gráfica se deduce que el mínimo de S = 189 cm? se obtiene para x = 3,2 cm, aproximadamente; de.. 200

la expresion y = 1tX2 se deduce el valor y = 6,2 cm, aproximadamente.

20. Hallar, aproximadamente, los valores de x para los cuales x3 + 2X2 - 7x - 3 = O.

Consideremos la función y = x3 + 2X2 - 7x - 3. Se deben hallar los valores de x para los cuales y = O.

De la gráfica de y = x3 + 2X2 - 7x - 3, que es la del Problema 15, se deduce que existen tres valores rea-les de x para los cuales y = O (los valores de x correspondientes a la intersección con el eje x). Estos valoresson x = -3,7, x = -0,4, x = 2,1, aproximadamente. Se puede obtener un resultado más exacto mediante elcálculo diferencial.

83

http://carlos2524.jimdo.com/

Page 92: Algebra Superior Murray R Spiegel

84 FUNCIONES Y GRAFICAS

GRAFICOS ESTADISTICOS

21. La tabla siguiente representa la población (en millones de habitantes) de los EE. UU. en los años 1840, 1850, ... ,1950. Representar estos datos gráficamente.

Años 1840 1850 1860 1870 1880 1890 1900 1910 1920 1930 1940 1950

Población17,1 23,2 31,4 39,8 50,2 62,9 76,0 92.0 105.7 122.8 131.7 150,7(millones)

160

'"" 140c:,gg 120

:i 100~¡¡j¡¡.¡ 80"-ec: 60'0'ü'":ooe,

o.•.~

Años

22. El tiempo T (segundos) empleado por un péndulo de longitud 1 (centímetros) en efectuar una oscilación comple-ta viene dado por las siguientes observaciones obtenidas en un laboratorio de fisica experimental. Representargráficamente T en función de l.

1 16,2 22,2 33,8 42,0 53,4 66,7 74,5 86,6 100,0

T 0,81 0,95 1,17 1,30 1,47 1,65 1,74 1,87 2,01

Los puntos se han unido por medio de una línea a sentimiento, como, normalmente, se hace en ciencias eingeniería.

T2,0

1.5

1,0

0,5

o 5040 70 90 10010 8030 6020

23. Laa)

24. E~

25. Ex

Y

26. LaS·

27. Do

28. Al

29. Do

30. Do

31. D,

32. Do

33. R,al

34. Sipt

35. R

36. R

37. R.

38. S,Flo

39. HV

http://carlos2524.jimdo.com/

Page 93: Algebra Superior Murray R Spiegel

e-r

FUNCIONES Y GRAFICAS 85

PROBLEMAS PROPUESTOS

23. Las longitudes de los lados de un rectángulo son x y 2x. Expresar el área A del rectángulo en función dea) el lado x, b) el perímetro P, e) la diagonal D.

24. Expresar el área S de un círculo en función de a) el radio r, b) el diámetro d, e) el perímetro P.

25. Expresar el área A de un triángulo isósceles en función de x e y, siendo x la longitud de los lados iguales ey la del tercer lado.

26. La longitud de la arista de un cubo es x. Expresar a) x en función del volumen V del cubo, b) la superficieS del cubo en función de x, e) el volumen Ven función de la superficie S.

27, Dada la función y = 5 + 3x - 2x2, hallar los valores de y correspondientes a x = -3, -2, -1, 0.1,2.3.

28. Ampliar la tabla de valores del Problema 27 hallando los valores de y correspondientes a x = - 5/2. - 3/2,- li2, 1/2, 3/2, 5/2.

29. Dada f(x) = 2X2 + 6x - 1, hallar f(-3), f(-2), f(O), f(l/2), f(3).

a2 - 2a30. Dada F(a) = ---, hallar a) F(I), b) F(2), e) F(x), d) F(-x).

1 + a

.v - 1 x G(x + h) - G(x) 231. Dada G(x) = x + ¡' hallar a) G(x + 1)' b) h ' e) G(x + 1).

32. Dada F(x, y) = 2X2 + 4x.I' - ),2, hallar a) F(I, 2), b) F(-2, -3), e) F(x + 1, )' - 1).

33. Representar, en un sistema de coordenadas rectangulares, los puntos siguientes:a) (1,3), b) (-2,1), e) (-1/2, -2), d) (-3,2/3), e) (-fi,3).

34. Siendo)' = 3x + 2. a) obtener los valores de y correspondientes a x = -2, = t , O. 1, 2 Y b) representar lospuntos (x, y) obtenidos.

35. Representar las funciones a) f(x) = 1 - 2x, b) f(x) = x2 - 4x + 3. e) f(x) = 4 - 3x - x2

36, Representar y = x3 - 6x2 + l l.v - 6.

37, Representar a) x2 + y2 = 16, b) x2 + 4)'2 = 16.

38. Se dispone de 120 m de alambrada para cercar dos jardines rectangulares iguales, A y B, como se indica en laFig. (a). Sabiendo que no es necesario proteger los lados que limitan con la casa, determinar el área máxima delos jardines que se puede cercar.

39. Hallar el área del rectángulo más largo que se puede inscribir en un triángulo rectángulo de catetos 6 y 8 cm.Véase Figura (h).

A B

Fig. lal Probo 38 Fig. Ibl Prob. 39

FUNCIONES Y GRAFICAS 85

PROBLEMAS PROPUESTOS

23. Las longitudes de los lados de un rectángulo son x y 2x. Expresar el área A del rectángulo en función de a) el lado x, b) el perímetro P, e) la diagonal D.

24. Expresar el área S de un círculo en función de a) el radio r , b) el diámetro d, e) el perímetro P.

25. Expresar el área A de un triángulo isósceles en función de x e y , siendo x la longitud de los lados iguales e y la del tercer lado .

26. La longitud de la arista de un cubo es x. Expresar a) x en función dei volumen V del cubo, b) la superficie S del cubo en función de x, e) el volumen Ven función de la superficie S.

27. Dada la función y = 5 + 3x - 2x' , hallar los valores de y correspondientes a x = -3, -2, -1, O. l. 2. 3.

28. Ampliar la tabla de valores del Problema 27 hallando los valores de y correspondientes a x = - 5/2. - 3/2. - l i 2, 1/2, 3/2, 5/2.

29. Dada f(x) = 2x' + 6x - 1, hallar f(-3) , f(-2) , feO), f(l/2), f(3).

u' - 2u JO. Dada F(u) = - - - , hallar a) F(I), b) F(2) , e) F(x) , d) F(-x).

I + u

x - I x G(x + h) - G(x) , 31. Dada G(x) = x + l' hallar a) G(x + 1)' b) h ' e) G(x + 1).

32. Dada F(x, y ) = 2x' + 4xr - )", hallar a) F(I. 2), b) F(-2 , -3), e) F(x + 1, )' - 1).

33. Representar, en un sistema de coordenadas rectangulares, los puntos siguientes:

a) (1,3), b) (-2.1), e) (-1 /2. -2), d) (-3.2/3), e) (-)3,3).

34. Siendo)' = 3x + 2. a) obtener los valores de y correspondientes a x = -2, -1 , O. 1, 2 Y b) representar los puntos (x , y) obtenidos.

35. Representar las funciones a) f(x) = 1 - 2x, b) f(x) = x' - 4x + 3. e) f(x) = 4 - 3x - X2

36. Representar y = x 3 - 6x' + Ilx - 6.

37. Representar a) x' + y' = 16, b) x' + 4)" = 16.

38. Se dispone de 120 m de alambrada para cercar dos jardines rectangulares iguales, A y B. como se indica en la Fig. (a) . Sabiendo que no es necesario proteger los lados que limitan con la casa. determinar el área máxima de los jardines que se puede cercar.

39. Hallar el área del rectángulo más largo que se puede inscribir en un triángulo rectángulo de catetos 6 y 8 cm. Véase Figura (h).

Casa / %

A B

Fig. la) Probo 38 Fig . lb) Probo 39

http://carlos2524.jimdo.com/

Page 94: Algebra Superior Murray R Spiegel

86 FUNCIONES Y GRAFICAS

40. Hallar los máximos y mínimos relativos de la función f(x) = 2X3 - 15x2 + 36x - 23.

41. De la gráfica de la función y = x3 - 7x + 6, obtener las raíces de la ecuación x3- 7x + 6 = O.

42. Demostrar que la ecuación x3 - x2 + 2x - 3 = O solo tiene una raíz real.

43. Demostrar que la ecuación x4 - x2 + 1 = O no tiene raíces reales.

44. El porcentaje de trabajadores agrícolas en EE. UU., en los años 1860, 1870, ... , 1950, viene dado en la tablasiguiente. Representar estos datos gráficamente.

Años 1860 1870 1880 1890 1900 1910 1920 1930 1940 1950Porcentaje de tra- 58,9 53,0 49,4 42,6 37,5 31,0 27,0 21,4 18,0 12,8bajadores agrícolas

45. El tiempo total empleado en detener un automóvil desde el momento en que el conductor se da cuenta de un pe-ligro, se compone delliempo de reacción (tiempo transcurrido desde el apercibimiento hasta que acciona el pedaldel freno) y delliempo de frenado (tiempo que tarda el coche en detenerse desde que se presiona el pedal corres-pondiente). La tabla que figura a continuación relaciona la distancia d (metros) que recorre hasta detenerse unautomóvil que marcha a una velocidad v (kilómetros por hora) en el instante en que se da cuenta del peligro. Re-presentar gráficamente d en función de v.

Velocidad v (kmjh) 30 45 60 75 90 105

Distancia d (m) 18 30 46 68 97 132

46. Los tiempos 1 que tarda un objeto en caer libremente partiendo del reposo desde distintas alturas h vienen dadosen la tabla siguiente.

Tiempo t (seg) 1 2 3 4 5 6

Altura h (m) 5 20 45 80 125 180

a) Representar gráficamente h en función de l.b) Hallar el tiempo que tardará un objeto en caer libremente, partiendo del reposo, desde una altura de 25 m

y de 150 m.e) Hallar la distancia que recorre un objeto que cae libremente, partiendo del reposo, durante un tiempo de 3,6 seg.

SOLUCIONES DE LOS PROBLEMAS PROPUESTOS

p2 2D223. A = 2x2, A = - A =-

18' 525. A = !'..Jx2 - y2/4 = !'..J4x2 _ y2

2 4

~

3 S26. x =.y¡;, S = 6x2, V = - =-.j6S

216 36

27. x -3 -2 -1 O 1 2 3Y -22 -9 O 5 6 3 -4

28. x -5/2 -3/2 -1/2 1/2 3/2 5/2Y -15 -4 3 6 5 O

29. f(-3) = -1, f(-2) = -5, f(O) = -1, f(I/2) = 5/2, f(3) = 35

30. a) -1/2,x2 - 2x

e) ---1 + x' -x

x2 + 2xd)b) O,

-131. a) 2x + 1 '

x2

e) x2 + 2

38. 1200 m2 39. 12 cm"

b) 2(x + I)(x + h + 1)'

32. a) 6, b) 23, e) 2X2 + 4xy - y2 + 6y - 3

40. El máximo de f(x) es 5 (en x = 2); el mínimo de f(x) es 4 (en x = 3).

41. Las raíces son x = -3, x = 1, x = 2. 46. b) 2,24 s, 5,48 s; e) 64,80 m

ECU

1. R

a

b

e

<, d

e

f

e.

h

I

k

http://carlos2524.jimdo.com/

Page 95: Algebra Superior Murray R Spiegel

a tabla

un pe-Ipedalcorres-erse uno. Re-

n dados

de 25 ID

CAPITULO 11

Ecuaciones lineales con una incógnita

ECUACION LINEAL CON UNA INCOGNITA. Es de la. forma ax + b = O, siendo a =F O Ybb constante. Su soiución viene dada por x =a

PROBLEMAS RESUELTOS

l. Resolver las ecuaciones siguientes:

a) x + 1 = 5, x = 5 - 1, x = 4.Comprobación: Haciendo x = 4 en la ecuación dada se obtiene 4 + 1 = 5, 5 = 5.

b) 3x - 7 = 14, 3x = 14 + 7, 3x = 21, x = 7. Comprobación: 3(7) - 7 = 14, 14 = 14.

e) 3x + 2 = 6x - 4, 3x - 6x = -4 - 2, -3x = -6, x = 2.

d) x + 3(x - 2) = 2x - 4, x + 3x - 6 = 2x - 4, 4x - 2x = 6 - 4, 2x = 2, x = 1.

e) 3x - 2 = 7 - 2x, 3x + 2x = 7 + 2, 5x = 9, x = 9/5.

J) 2(/ + 3) = 5(/ - 1) - 7(1 - 3), 2/ + 6 = 5/ - 5 - 7/ + 21, 4/ = 10, / = 10/4 = 5/2.

g) 3x + 4(x - 2) = x - 5 + 3(2x - 1), 3x + 4x - 8 = x - 5 + 6x - 3, 7x - 8 = 7x - 8.Esta es una identidad y se verifica para todos los valores de x.

x - 3h)

2x + 45

5(x - 3) = 2(2x + 4), 5x - 15 = 4x + 8, x = 23.2

i) 3 + 2[y - (2y + 2)] = 2[y + (3y - 1)), 3 + 2[y - 2y - 2] = 2[y + 31. - 1],3 + 2y - 4y - 4 = 2y + 6y - 2, -2y - 1 = 8y - 2, -lOy = -1, Y = 1/10.

j) (s + 3)2 = (s - 2)2 - 5, S2 + 6s + 9 = S2 - 4s + 4 - 5, 6s + 4s = -9 - 1, s = -1.

x-2 x-4k) (x - 2)(x + 4) = (x - 4)(x + 2), x2 + 2x - 8 = x2 - 2x - 8, 4x = O, x = O.x+2=x+4'

0-2 0-4Comprobación: O + 2 = O + 4 ' -1=-1.

3x + 1 3x - 21) (x + 1)(3x + 1) = (x + 2)(3x - 2), 3x2 + 4x + 1 = 3x2 + 4x - 4 o 1 = - 4.x + 2 = -:;-:¡:-¡ ,

No hay ningún valor de x que satisfaga esta ecuación.

87

CAPITULO 11

Ecuaciones lineales con una incógnita

ECUACION LINEAL CON UNA INCOGNITA. Es de la . forma ax + b = O, siendo a =1= O Y b

b constante. Su sOlución viene dada por x = a

PROBLEMAS RESUELTOS

1. Resolver las ecuaciones siguientes :

a) x + 1 = 5, x = 5 - 1, x = 4. Comprobación: Haciendo x = 4 en la ecuación dada se obtiene 4 + 1 = 5, 5 = 5.

b) 3x - 7 = 14, 3x = 14 + 7, 3x = 21 , x = 7. Comprobación: 3(7) - 7 = 14, 14 = 14.

e) 3x + 2 = 6x - 4, 3x - 6x = -4 - 2, -3x = -6, x = 2.

d) x + 3(x - 2) = 2x - 4, x + 3x - 6 = 2x - 4, 4x - 2x = 6 - 4, 2x = 2, x = 1.

e) 3x - 2 = 7 - 2x, 3x + 2x = 7 + 2, 5x = 9, x = 9/ 5.

J) 2(t + 3) = 5(1 - 1) - 7(t - 3), 21 + 6 = 51 - 5 - 71 + 21 , 41 = 10, 1 = 10/4 = 5/2.

g) 3x + 4(x - 2) = x - 5 + 3(2x - 1), 3x + 4x - 8 = x - 5 + 6x - 3, 7x - 8 = 7x - 8. Esta es una identidad y se verifica para todos los valores de x .

x - 3 h)

2 2x + 4

5 5(x - 3) = 2(2x + 4), 5x - 15 = 4x + 8, x = 23.

i) 3 + 2[y - (2y + 2)] = 2[y + (3y - 1)], 3 + 2[y - 2y - 2] = 2[y + 3y - 1]. 3 + 2y - 4y - 4 = 2y + 6y - 2, - 2y - 1 = 8y - 2, -10y = -1, Y = 1/ 10.

j) (s + 3)2 = (s - 2)2 - 5, S2 + 6s + 9 = S2 - 4s + 4 - 5, 6s + 4s = -9 - 1, s = -1.

x-2 k)

x+2

x-4

x + 4' (x - 2)(x + 4) = (x - 4)(x + 2),

0-2 0-4 Comprobación: - - = - - , - 1 = - 1.

0+2 0+4

X2 + 2x - 8 = X2 - 2x - 8, 4x = O, x = O.

3x + 1 /)

3x - 2 (x + 1 )(3x + 1) = (x + 2)(3x - 2). 3X2 + 4x + 1 = 3X2 + 4x - 4 o 1 = -4.

No hay ningún valor de x que satisfaga esta ecuación.

87

http://carlos2524.jimdo.com/

Page 96: Algebra Superior Murray R Spiegel

88 ECUACIONES LINEALES CON UNA INCOGNITA

5 5n) - + - = 6. Multiplicando por 2x, 5(2) + 5 = 12x, 12x = 15, x = 5/4.

x 2x

x + 3 5 1o) ~ + x _ 1 = "2' Multiplicando por 2x(x - 1), que es el denominador común mínimo de las fracciones.

(x+3)(x-1)+5(2x)=x(x-1), x2+2x-3+IOx=x2-x. I3x=3, x=3/13.

2 4 16p) -- - -- - -- Multiplicando por (x - 3)(x + 3) o x2 - 9,

x-3 x+3-x2-9'

2(x + 3) - 4(x - 3) = 16, 2x + 6 - 4x + 12 = 16, -2x = -2, x = 1.

1 1 1 (}'+3)-y (y+5)-(y+2)q) Y - y + 3 - Y + 2 - Y + 5' y(y + 3) Lv + 2)(}' + 5) ,

(y + 2)(y + 5) = y(y + 3), / + 7y + 10 = y2 + 3y, 4y = -lO,

3 3

YÚ' + 3) t~, + 2)t1' + 5)'

Y = -5/2.

3 2 9 3 2 9r) x2 _ 4x - 2X2 _ 5x _ 12 2X2 + 3x o x(x - 4) - (2x + 3)(x - 4) x(2x + 3)'

Multiplicando por x(x - 4)(2x + 3), que es el denominador común mínimo de las fracciones.

3(2x + 3) - 2x = 9(x - 4), 6x + 9 - 2x = 9x - 36, 45 = 5x, x = 9.

ECUACIONES LITERALES

2. Despejar x.

a) 2x - 4p = 3x + 2p, 2x - 3x = 2p + 4p, -x = 6p, x = -6p.

b - ax' = a _ b = - l siempre que a '" b.b) ax + a = bx + b, ax - bx = b - a, x(a - b) = b - a,

Si a = b la ecuación es una identidad y se verifica para todo valor de x,

-4b-4d 4b+4dx = 2 3 = --2- siempre que 3a '" 2e.e - a 3a - e

Si 3a = 2e no hay solución a menos que d = =b, en cuyo caso la ecuación dada es una identidad.

e) 2ex + 4d = 3ax - 4b, 2ex - 3ax = -4b - 4d,

3x + a 4x + b 2 2 b2 - a2d) --b- = --a-' 3ax + a = 4bx + b, 3ax - 4bx = b2 - a2, x = ---b (siempre que 3a '" 4b).

3a - 4

REPRESENTACION DE PALABRAS POR SIMBOLOS

3. Expresar por medio de símbolos algebraicos.

a) El doble de un número más uno.

Sea x = el número. Entonces 2x = doble del número, y el doble del número más uno es = 2.'" + l.

b) El quíntuplo de un número menos tres.

Sea x = el número. El quíntuplo del número inenos tres es = 5x - 3.

e) Dos números cuya suma es 100.

Si x = uno de los números. entonces 100 - x = al otro número.

d)

e)

f)

g)

h)

i)

j)

k)

1)

m)

n)

o)

PROBL

4. Hal

88 ECUACIONES LINEALES CON UNA INCOGNITA

5 5 n) ; + 2x = 6. Multiplicando por 2x, 5(2) + 5 = 12x, 12x = 15, x = 5/4.

x + 3 5 1 o) 2:;- + x _ 1 = "2' Multiplicando por 2x(x - 1), que es el denominador común mínimo de las fracciones ,

(x + 3)(x - 1) + 5(2x) = x(x - 1), X2 + 2x - 3 + IOx = X2 - x, I3x = 3. x = 3/ 13.

2 4 16 p) x _ 3 - x + 3 = X2 _ 9' Multiplicando por (x - 3)(x + 3) o X2 - 9,

2(x + 3) - 4(x - 3) = 16, 2x + 6 - 4x + 12 = 16, -2x = -2, x = 1.

3 3 q) y - y + 3 = Y + 2 - Y + 5 '

(y + 2)(y + 5) = y(y + 3),

(¡., + 3) - Y

y (y + 3)

(y + 5) - (y + 2)

(y + 2)(¡.' + 5) , - -- =-----y(y + 3) (1' + 2)(1' + 5)'

y2 + 7y + 10 = y2 + 3y, 4} = -10, )' = - 5/2.

3 2 9 3 2 9 r) X2 _ 4x - 2X2 _ 5x - 12 2X2 + 3x o x(x - 4) - (2x + 3)(x - 4) x(2x + 3)

Multiplicando por x(x - 4)(2x + 3), que es el denominador común mínimo de las fracciones,

3(2x + 3) - 2x = 9(x - 4), 6x + 9 - 2x = 9x - 36, 45 = 5x, x = 9.

ECUACIONES LITERALES

2. Despejar x.

a) 2x - 4p = 3x + 2p, 2x - 3x = 2p + 4p, -x = 6p, x = -6p.

b - a b) ax + a = bx + b, ax - bx = b - a, x(a - b) = b - a, x = a _ b = - l siempre que a -J b.

Si a = b la ecuación es una identidad y se verifica para todo valor de x.

e) 2ex + 4d = 3ax - 4b, 2ex - 3ax = - 4b - 4d, -4b-4d 4b+4d

x = 2c _ 3a = 3a _ 2e siempre que 3a -J 2c.

Si 3a = 2e no hay solución a menos que d = -b, en cuyo caso la ecuación dada es una identidad .

3x + a 4x + b b2 - a2

d) --b- = - -a- ' 3ax + a2 = 4bx + b2, 3ax - 4bx = b2 - a2, x = ---h (siempre que 3a -J 4b). 3a - 4

REPRESENTACION DE PALABRAS POR SIMBOLOS

3. Expre.s¡¡r por medio de símbolos algebraicos.

a) El doble de un número más uno.

Sea x = el número. Entonces 2x = doble del número, y el doble del número más uno es = 2." + 1.

b) El quíntuplo de un número menos tres .

Sea x = el número. El quíntuplo del número menos tres es = 5x - 3.

e) Dos números cuya suma es 100.

Si x = uno de los números, entonces 100 - x = al otro número.

http://carlos2524.jimdo.com/

Page 97: Algebra Superior Murray R Spiegel

es.

4b).

ECUACIONES LINEALES CON UNA INCOGNITA 89d) Tres enteros consecutivos (por ejemplo, 5, 6, 7).

Si x es el menor de los enteros, entonces (x + 1) Y (x + 2) serán los otros dos.

e) Dos números cuya diferencia sea lO.

Sea x = el número menor; tendremos (x + lO) = el número mayor.

f) El exceso de 100 sobre el triplo de un número.

Sea x = al número dado. El exceso de 100 sobre 3x es (100 - 3x).

g) Un entero impar.

Sea x = un entero ·cualquiera. Entonces 2x es siempre un número par, y (2x + 1) es un entero impar.

h) Cuatro enteros impares consecutivos (por ejemplo, 1, 3, 5, 7; 17, 19, 21, 23).

La diferencia entre dos enteros impares consecutivos es 2.Sea 2x + 1 = el entero impar más pequeño. Los números pedidos serán 2x + 1, 2x + 3, 2x + 5, 2x + 7.

i) Los céntimos que hay en x pesetas.

Como 1 peseta = 100 céntimos, x pesetas = 100x céntimos.

j) La edad de Juan es el doble que la de Leticia y la de ésta el triple que la de Fernando. Expresar cada una deestas edades en función de una de ellas.

Sea x = edad de Fernando. La de Leticia será 3x y la de Juan 2(3x) = 6x.

Otro método. Sea y = edad de Juan; la de Leticia será ~y, y la de Fernando, ~(~y) = ~y.

k) Los tres ángulos A, B, C de un triángulo sabiendo que A es igual al doble de C más 10°.

Sea C = x"; entonces A = (2x + IOt. Como A + B + C = 180°, B = 180° - (A + C) = (170 - 3xt.

1) El tiempo invertido por un móvil en recorrer una distancia de x kilómetros a una velocidad de 20 km/h.

distancia x km xDistancia = velocidad x tiempo. Por tanto, tiempo = 'd d = -20 k h = 20 h.

veloci a mi

m) El perímetro y el área de un rectángulo uno de cuyos lados es 4 m más largo que el otro.

Sea x = longitud del lado menor, entonces (2x + 4) = longitud del lado mayor.El perímetro = 2(x) + 2(2x + 4) = (6x + 8), Y el área = x(2x + 4).

n) La fracción cuyo numerador es igual a 4 veces el denominador menos tres unidades.

4x - 3Sea x = denominador; el numerador será = 4x - 3. La fracción es ---o. x

o) El número de litros de alcohol de un recipiente que contiene x litros de una mezcla al 40 % de alcohol envolumen.

En x litros de mezcla habrá 0,40x litros de alcohol.

PROBLEMAS DE NUMEROS

4. Hallar dos números sabiendo que su suma es igual a 21 y que uno de ellos es igual al doble del otro.

Sean x y 2x los dos números pedidos.En estas condiciones, x + 2x = 21, o sea x = 7; por tanto, los números pedidos son x = 7 Y 2x= 14.

Comprobación. 7 + 14 = 21 Y 14 = 2(7).

ECUACIONES LINEALES CON UNA INCOGNITA 89

d) Tres enteros consecutivos (por ejemplo, 5, 6, 7).

Si x es el menor de los enteros, entonces (x + 1) Y (x + 2) serán los otros dos.

e) Dos números cuya diferencia sea 10.

Sea x = el número menor; ten4remos (x + 10) = el número mayor.

f) El exceso de 100 sobre el triplo de un número.

Sea x = al número dado. El · exceso de 100 sobre 3x es (100 - 3x).

g) Un entero impar.

Sea x = un entero ·cualquiera. Entonces 2x es siempre un número par, y (2x + 1) es un entero impar.

h) Cuatro enteros impares consecutivos (por ejemplo, 1, 3, 5, 7; 17, 19, 21, 23).

La diferencia entre dos enteros impares consecutivos es 2. Sea 2x + 1 = el entero impar más pequeño. Los números pedidos serán 2x + 1, 2x + 3, 2x + 5, 2x + 7.

i) Los céntimos que hay en x pesetas.

Como 1 peseta = 100 céntimos, x pesetas = lOOx céntimos.

j) La edad de Juan es el doble que la de Leticia y la de ésta el triple que la de Fernando. Expresar cada una de estas edades en función de una de ellas.

Sea x = edad de Fernando. La de Leticia será 3x y la de Juan 2(3x) = 6x. I 1 I 1

Otro método. Sea y = edad de Juan; la de Leticia será 2Y' y la de Fernando, 3(2Y ) = '(/.

k) Los tres ángulos A , B, C de un triángulo sabiendo que A es igual al doble de C más 10°.

Sea C = xO; entonces A = (2x + 10)". Como A + B + C = 180°, B = 180° - (A + C) = (170 - 3x)".

1) El tiempo invertido por un móvil en recorrer una distancia de x kilómetros a una velocidad de 20 km/h.

distancia x km x Distancia = velocidad x tiempo. Por tanto, tiempo = 'd d = -20 k h = 20 h.

velocl a mi

m) El perímetro y el área de un rectángulo uno de cuyos lados es 4 m más largo que el otro.

Sea x = longitud del lado menor, entonces (2x + 4) = longitud del lado mayor. El perímetro = 2(x) + 2(2x + 4) = (6x + 8), y el área = x(2x + 4).

n) La fracción cuyo numerador es igual a 4 veces el denominador menos tres unidades.

4x - 3 Sea x = denominador; el numerador será = 4x - 3. La fracción es --x-o

o) El número de litros de alcohol de un recipiente que contiene x litros de una mezcla al 40 % de alcohol en volumen.

En x litros de mezcla habrá 0,40x litros de alcohol.

PROBLEMAS DE NUMEROS

4. Hallar dos números sabiendo que su suma es igual a 21 y que uno de ellos es igual al doble del otro.

Sean x y 2x los dos números pedidos. En estas condiciones. x + 2x = 21 , o sea x = 7; por tanto, los números pedidos son x = 7 Y 2x= 14.

Comprobación. 7 + 14 = 21 Y 14 = 2(7).

http://carlos2524.jimdo.com/

Page 98: Algebra Superior Murray R Spiegel

90 ECUACIONES LINEALES CON UNA INCOGNITA

S. Hallar un número sabiendo que si se multiplica por cuatro y se le resta diez se obtiene 14.

Sea x = el número buscado. Tendremos 4x - 10 = 14, 4x = 24, Y x = 6.

Comprobación. Cuatro veces 6 menos diez es 4(6) - 10 = 14.

6. Hallar tres números enteros consecutivos cuya suma sea 24.

Sean los tres números consecutivos x, x + 1, x + 2.

Tendremos x + (x + 1) + (x + 2) = 24, o sea x = 7: por tanto. los números son 7, 8.9.

7. Hallar dos números sabiendo que su suma es 37 y que si se divide el mayor por el menor, el cociente vale 3y el resto 5.

Sea x el número menor. 37 - .r = el número mayor.

número mayor 5Tendremos . = 3 + ------

numero menor número menor

37 - x 5o sea ---=3+-

x x

Resolviendo, 37 - x = 3x + 5, 4x = 32, x = 8. Los números buscados son 8, 29.

PROBLEMAS DE EDADES

8. La edad de una persona es 41 años y la de su hijo es 9. Hallar al cabo de cuántos años la edad del padretriplica la del hijo.

Sea x = el número de años buscado.

La edad del padre después de x años = 3(edad del hijo después de x años)41+x=3(91-x) yx=7años

9. Hace diez años. la edad de Carlos era cuatro veces mayor que la de Javier y. hoy en día. es solamente del doble.Hallar las edades actuales de ambos.

Sea x'= la edad actual de Javier; será 2x = edad actual de Carlos.

Edad de Carlos hace diez años = 4(edad de Javier hace diez años)2x - 10 = 4(x - 10) y .r = 15 años

Luego la edad actual de Javier x = 15 años y la edad actual de CarlosZx ..= 30 años.

Comprobación. Hace diez años Javier tenía 5 y Carlos 20, es decir, la edad de Carlos era cuatro veces mayorque la de Javier.

PROBLEMAS DE MONEDAS

10. Pablo tiene 350 pts en monedas de 5 y 25 pts. Sabiendo que posee 50 monedas. calcular el número de ellasde 5 pts.

Sea x = número de monedas de 5 pts: 50 - x = número de monedas de 25 pts.Pesetas en monedas de 5 + pesetas en monedas de 25 = 350 pts

5x pts + 25(50 - .x) pts = 350 pts, de donde .v = 45 monedas de 5 pts.

11. En una bolsa hay 230 pts en monedas de 5. 25 y 50 pts. Sabiendo que el número de monedas de 25 es igual aldoble del de 50. y que el número de monedas de 5 es igual al doble del de 25 menos 2, hallar las monedas queexisten de cada clase.

Sea x = número de monedas de 50. 2x = número de monedas de 25 y 2(2x) - 2 = 4x - 2 = número demonedas de 5.

Pesetas en monedas de 50 + pesetas en monedas de 25 + pesetas en monedas de 5 = 230 pts50(x) pts + 25(2x) pts + 5(4x - 2) pts = 230 pts, de donde x = 2.

Por tanto. hay x = 2 monedas de 50, 2x = 4 de 25 y 4x - 2 = 6 de 5 pts.

Comprobacián, 2 monedas de 50 = 100 pts, 4 de 25 = 100 pts, 6 de 5 = 30 pts, y su suma = 230 pts.

PROBL

12. Hauni

13. Hales i

Res

PROBLI

14. Un:aeeune

Res

15. Haldel

16. Halconven

PROBLl

17. Halen

90 ECUACIONES LINEALES CON UNA INCOGNITA

S. Hallar un número sabiendo que si se multiplica por cuatro y se le resta diez se obtiene 14.

Sea x = el número buscado. Tendremos 4x - 10 = 14, 4x = 24, Y x = 6.

Comprobación. Cuatro veces 6 menos diez es 4(6) - 10 = 14.

6. Hallar tres números enteros consecutivos cuya suma sea 24.

Sean los tres números consecutivos x, x + 1, x + 2.

Tendremos x + (x + 1) + (x + 2) = 24, o sea x = 7: por tanto. los números son 7, 8. 9.

7. Hallar dos números sabiendo que su suma es 37 y que si se divide el mayor por el menor, el cociente vale 3 y el resto 5.

Sea x el número menor, 37 - x = el número mayor.

número mayor 5 Tendremos . = 3 + -,.-----

numero menor número menor

37 - x 5 osea ---=3+-

x x

Resolviendo. 37 - x = 3x + 5, 4x = 32, x = 8. Los números buscados son 8, 29.

PROBLEMAS DE EDADES

8. La edad de una persona es 41 años y la de su hijo es 9. Hallar al cabo de cuántos años la edad del padre triplica la del hijo.

Sea x = el número de años buscado.

La edad del padre después de x años = 3(edad del hijo después de x años) 41+x=3(9-¡-x) yx=7años

9. Hace diez años. la edad de Carlos era cuatro veces mayor que la de Javier y. hoy en día. es solamente del doble. Hallar las edades actuales de ambos.

Sea x '= la edad actual de Javier ; será 2" = edad actual de Carlos.

Edad de Carlos hace diez años = 4(edad de Javier hace diez años) 2x - 10 = 4(x - 10)

Luego la edad actual de Javier x = 15 años y la edad actual de Carlos 2x .=. 30 años.

y x = 15 años

Comprobación. Hace diez años Javier tenía 5 y Carlos 20, es decir, la edad de Carlos era cuatro veces mayor que la de Javier.

PROBLEMAS DE MONEDAS

10. Pablo tiene 350 pts en monedas de 5 y 25 pts. Sabiendo que posee 50 monedas, calcular el número de ellas de 5 pts.

Sea x = número de monedas de 5 pts : 50 - x = número de monedas de 25 pts. Pesetas en monedas de 5 + pesetas en monedas de 25 = 350 pts

5x pts + 25(50 - x) pts = 350 pts. de donde x = 45 monedas de 5 pts.

11. En una bolsa hay 230 pts en monedas de 5. 25 y 50 pts. Sabiendo que el número de monedas de 25 es igual al doble del de 50, y que el número de monedas de 5 es igual al doble del de 25 menos 2, hallar las monedas que existen de cada clase.

Sea x = número de monedas de 50. 2x = número de monedas de 25 y 2(2x) - 2 = 4x - 2 = número de monedas de 5.

Pesetas en monedas de 50 + pesetas en monedas de 25 + pesetas en monedas de 5 = 230 pts 50(x) pIS + 25(2x) pts + 5(4x - 2) pts = 230 pts, de donde x = 2.

Por tanto. hay x = 2 monedas de 50. 2x = 4 de 25 y 4x - 2 = 6 de 5 pts.

Comprobaciún . 2 monedas de 50 = 100 pts. 4 de 25 = 100 pts. 6 de 5 = 30 pts, y su suma = 230 pts.

http://carlos2524.jimdo.com/

Page 99: Algebra Superior Murray R Spiegel

le 3

dre

os

ble.

años

ayor

ellas

pts.

al alque

ro de

= 2.

pis.

ECUACIONES LINEALES CON UNA INCOGNITA

PROBLEMAS DE DIGITOS

12. Hallar un número de dos cifras sabiendo que la correspondiente a las decenas excede en 4 a la cifra de lasunidades y, por otra parte, es igual al doble de ésta menos 1.

Sea x = cifra de las unidades; x + 4 = cifra de las decenas.

Como la cifra de las decenas = 2(cifra de unidades) - 1, tendremos x + 4 = 2(x) - 1, o sea x = 5.

Por tanto, x = 5, x + 4 = 9, Y el número pedido es 95.

13. Hallar un número de dos cifras sabiendo que la suma de éstas es 12 y que si se invierten el número que resultaes igual a 4/7 del primitivo.

Sea x = cifra de las unidades; 12 - x = cifra de las decenas.

Número original = 10(12 - x) + x; invirtiendo el orden de las cifras resulta el número = 10x + (12 - x).Ahora bien, como

4 4número nuevo = "1 (número original), tendremos IOx + (12 - x) = "1 [10(12 - x) + x]

Resolviendo esta ecuación resulta x = 4, 12 - x = 8 Y el número pedido es 84.

PROBLEMAS COMERCIALES

14. Una persona invierte 300 000 pts en acciones y recibe, anualmente, 10 000 pts de intereses. Sabiendo que unasacciones le producen el 5 % y las restantes el 3 %, a interés simple, hallar el dinero que ha invertido en cadauno de los dos tipos de acciones.

Sea x = cantidad invertida al 5 %; 300 000 - x = cantidad invertida al 3 %.Intereses al 5 % + intereses al 3 % = 10 000

0,05x + 0,03(300 000 - x) = 10 000

Resolviendo esta ecuación resulta x = 50 000 pts al 5 lo, 300 000 - x = 250 000 pts al 3 %.

15. Hallar el sueldo de un empleado sabiendo que después de deducido el 14 % de impuestos sobre el rendimientodel trabajo personal la cantidad que percibe es de 26 800 pts mensuales.

Sea x = sueldo. Según el enunciado, sueldo - impuestos = 26 800 pts,o sea x - 0,14 = 26800 pts, de donde x = 30000 pts.

16. Hallar el precio que un vendedor debe poner a un articulo que a él le cuesta 12 000 pts, para poder ofrecerlocon un descuento del 20 % sobre el precio señalado y. todavía. ganar en la operación un 25 % sobre el precio deventa.

Sea x = precio marcado del articulo; el precio de venta = x - 0.20x = 0.80x.

Como la ganancia = 25 % del precio de venta. el precio de coste será = 75 % del precio de venta. Por tanto.

coste = 0,75(precio de venta)12000 = 0,75(O,8x), 12 000 = 0.6x. de donde x = 20000 pts.

PROBLEMAS DE MEDIDAS

17. Hallar la longitud del lado de un cuadrado sabiendo que si se aumenta ésta en 4 m. su área se incrementaen 64 m'-

Sea x = lado del cuadrado; x + 4 = lado del nuevo cuadrado.

Area nueva = área primitiva + 64(X+4)2 =.~2+64 dedondex=6m.

91ECUACIONES LINEALES CON UNA INCOGNITA 91

PROBLEMAS DE DIGITOS

12. Hallar un número de dos cifras sabiendo que la correspondiente a las decenas excede en 4 a la cifra de las unidades y, por otra parte, es igual al doble de ésta menos 1.

Sea x = cifra de las unidades; x + 4 = cifra de las decenas.

Como la cifra de las decenas = 2(cifra de unidades) - 1, tendremos x + 4 = 2(x) - 1, o sea x = 5.

Por tanto, x = 5, x + 4 = 9, Y el número pedido es 95.

13. Hallar un número de dos cifras sabiendo que la suma de éstas es 12 y que si se invierten el número que resulta es igual a 4/7 del primitivo.

Sea x = cifra de las unidades ; 12 - x = cifra de las decenas.

Número original = \0(12 - x) + x; invirtiendo el orden de las cifras resulta el número = 10x + (12 - x). Ahora bien, como

4 4 número nuevo = '7 (número original), tendremos \Ox + (12 - x) = '7 [\0(12 - x) + x]

Resolviendo esta ecuación resulta x = 4, 12 - x = 8 Y el número pedido es 84.

PROBLEMAS COMERCIALES

14. Una persona invierte 300 000 pts en acciones y recibe, anualmente, \O 000 pts de intereses. Sabiendo que unas acciones le producen el 5 % y las restantes el 3 %, a interés simple, hallar el dinero que ha invertido en cada uno de los dos tipos de acciones.

Sea x = cantidad invertida al 5 %; 300 000 - x = cantidad invertida al 3 %.

Intereses al 5 % + intereses al 3 % = \O 000 0,05x + 0,03(300 000 - x) = \O 000

Resolviendo esta ecuación resulta x = 50000 pts al 5 lo, 300 000 - x = 250 000 pts al 3 %.

15. Hallar el sueldo de un empleado sabiendo que después de deducido el 14 % de impuestos sobre el rendimiento del trabajo personal la cantidad que percibe es de 26 800 pts mensuales.

Sea x = suelc;lo. Según el enunciado, sueldo - impuestos = 26 800 pts, o sea x - 0,14 = 26800 pts. de donde x = 30000 pts.

16. Hallar el precio que un vendedor debe poner a un artículo que a él le cuesta 12 000 pts, para poder ofrecerlo con un descuento del 20 % sobre el precio señalado y, todavía, ganar en la operación un 25 % sobre el precio de venta.

Sea x = precio marcado del artículo; el precio de venta = x - 0.20x = 0.80x.

Como la ganancia = 25 % del precio de venta. el precio de coste será = 75 % del precio de venta. Por tanto.

coste = O,75(precio de venta) 12000 = O,75(0.8x). 12000= 0.6x. de donde x = 20000 pts.

PROBLEMAS DE MEDIDAS

17. Hallar la longitud del lado de un cuadrado sabiendo que si se aumenta ésta en 4 m. su área se incrementa en 64 m2

.

Sea x = lado del cuadrado; x + 4 = lado del nuevo cuadrado.

Area nueva = área primitiva + 64 (x+4)2 = .,2+64 dedondex=6m.

http://carlos2524.jimdo.com/

Page 100: Algebra Superior Murray R Spiegel

92 ECUACIONES LINEALES CON UNA INCOGNITA

18. Un cateto de un triángulo rectángulo mide 20 cm y la hipotenusa es 10 cm mayor que el otro cateto. Hallarlas longitudes de los lados desconocidos.

Sea x = longitud del cateto desconocido; x + 10 = longitud de la hipotenusa.

Cuadrado de la hipotenusa = suma de los cuadrados de los catetos(x + 10)2 = x2 + (20)2 de donde x = 15 cm.

Los lados pedidos son x = 15 cm y x + 10 = 25 cm.

19. Hallar la temperatura a la que coinciden las indicaciones de dos termómetros graduados, uno en la escala cen-9

tígrada y otro en la fahrenheit. Se sabe que: Temperatura fahrenheit = "5 (temperatura centígrada) + 32.

Sea x = temperatura buscada = temperatura fahrenheit = temperatura centígrada.

9Tendremos x = "5x + 32; de donde x = _40°. Por tanto, _40° F = _40° C.

PROBLEMAS DE MEZCLAS

20. Hallar el número de kilogramos (kg) que se deben tomar de dos ingredientes cuyos precios son 45 y 85 pts/kg,respectivamente, para obtener un producto de 40 kg a un precio de 60 pts/kg,

Sea x = masa del de 45 pts; 40 - x = masa del de 85 pts.

Valor del ingrediente de 45 ptsjkg + valor del ingrediente dees decir, 45x + 85(40 - x)

86 pts/kg = valor de la mezcla60 x 40

Resolviendo esta ecuación resulta x = 25 kg del de 45 pts/kg ; 40 - x = 15 kg del de 85 pts/kg.

21. Un depósito contiene 20 litros de una mezcla de alcohol yagua al 40 % de alcohol en volumen. Hallar el númerode litros de mezcla que se deben sustituir por un volumen igual de agua para que la solución que resulte seadel 25 % de alcohol en volumen.

Sea x = volumen que se extrae de la solución al 40 %.Volumen de alcohol en la solución final = volumen de alcohol en 20 litros de solución al 25 %Es decir, 0,40(20 - x) = 0,25(20) de donde x = 7.5 litros.

22. Hallar la masa de agua que se debe evaporar de 40 kg de una solución salina al 20 %, para que resulte unasolución al 50 %. Los porcentajes son en masa.

Sea x = masa de agua que se debe evaporar.

Masa de sal en la solución al 20 % = masa de sal en la solución al 50 %0,20(40 kg] = 0.50(40 kg - x] de donde x = 24 kg.es decir,

23. Calcular el número de litros de una solución de alcohol al 60 % que se deben añadir a 40 litros de otrasolución de alcohol al 20 % para obtener una mezcla al 30 I~. Los porcentajes son en volumen.

Sea x = número de litros de solución al 60 % que se deben añadir.

Alcohol en la solución al 60 % + alcohol en la solución al 20 % = alcohol en la solución al 30 %es decir, 0,60x + 0,20(0,40) 0,30(x + 40)

1 .de donde x = 13"3 litros.

24. Dos minerales de manganeso [Mn] contienen el 4O:%; y el 25/~ de dicho metal, respectivamente. Calcular lastoneladas de cada uno de ellos que se deben mezclar para obtener 100 ton de mineral con una riqueza del 35 %.Todos los porcentajes son en peso.

PRO25.

26.

27.

28

92 ECUACIONES LINEALES CON UNA INCOGNITA

18. Un cateto de un triángulo rectángulo mide 20 cm y la hipotenusa es 10 cm mayor que-el otro cateto. Hallar las longitudes de los lados desconocidos.

Sea x = longitud del cateto desconocido; x + 10 = longitud de la hipotenusa.

Cuadrado de la hipotenusa = suma de los cuadrados de los catetos (x + 10)2 = X2 + (20)2 de donde x = 15 cm.

Los lados pedidos son x = 15 cm y x + 10 = 25 cm.

19. Hallar la temperatura a la que coinciden las indicaciones de dos termómetros graduados, uno en la escala cen-9

tígrada y otro en la fahrenheit. Se sabe que: Temperatura fahrenheit = "5 (temperatura centígrada) + 32.

Sea x = temperatura buscada = temperatura fahrenheit = temperatura centígrada.

9 Tendremos x = "5x + 32; de donde x = _40°. Por tanto , _40° F = _40° C.

PROBLEMAS DE MEZCLAS

20. Hallar el número de kilogramos (kg) que se deben tomar de dos ingredientes cuyos precios son 45 y 85 pts/kg, respectivamente, para obtener un producto de 40 kg a un precio de 60 pts/kg.

Sea x = masa del de 45 pts; 40 - x = masa del de 85 pts.

Valor del ingrediente de 45 ptsjkg + valor del ingrediente de 86 pts/kg = valor de la mezcla es decir, 45x + 85(40 - x) 60 x 40

Resolviendo esta ecuación resulta x = 25 kg del de 45 ptsfkg; 40 - x = 15 kg del de 85 pts/kg.

21. Un depósito contiene 20 litros de una mezcla de alcohol yagua al 40 % de alcohol en volumen . Hallar el número de litros de mezcla que se deben sustituir por un volumen igual de agua para que la solución que resulte sea del 25 % de alcohol en volumen.

Sea x = volumen que se extrae de la solución al 40 %.

Volumen de alcohol en la solución final = volumen de alcohol en 20 litros de solución al 25 % Es decir, 0,40(20 - x) = 0,25(20) de donde x = 7.5 litros.

22. Hallar la masa de agua que se debe evaporar de 40 kg de una solución salina al 20 %, para que resulte una solución al 50 %. Los porcentajes son en masa.

Sea x = masa de agua que se debe evaporar.

Masa de sal en la solución al 20 % = masa de sal en la solución al 50 % es decir, 0,20(40 kg) = 0.50(40 kg - x) de donde x = 24 kg.

23. Calcular el número de litros de una solución de alcohol al 60 % que se deben añadir a 40 litros de otra solución de alcohol al 20 % para obtener una mezcla al 30 I~. Los porcentajes son en volumen.

Sea x = número de litros de solución al 60 % que se deben añadir.

Alcohol en la solución al 60 % + alcohol en la solución al 20 % = alcohol en la solución al 30 % es decir. 0,60x + 0,20(0,40) 0,30(x + 40)

1 . de donde x = 13"3 lItros.

24. Dos minerales de manganeso (Mn) contienen el 4O:%; y el 25 /~ de dicho metal, respectivamente. Calcular las toneladas de cada uno de ellos que se deben mezclar para obtener 100 ton de mineral con una riqueza del 35 %. Todos los porcentajes son en peso.

http://carlos2524.jimdo.com/

Page 101: Algebra Superior Murray R Spiegel

Hallar

5 cm.

a ceno

pts/kg,

úmerohe sea

litros.

he una

%

1.'3 litros.

ular las135 %.

ECUACIONES LINEALES CON UNA lNCOGNITA 93

Sea x = peso necesario de mineral del 40 %; 100 - x = peso necesario de mineral del 25 %.Mn del 40 % + Mn del 25 % = Mn total en las 100 ton

0,4Ox + 0,25(100 - x) = 0,35(100)

De esta ecuación resulta x = 67 ton de mineral del 40 % y 100 - x = 33 ton de mineral del 25 %.

PROBLEMAS DE MOVILES

25. Dos automóviles, A y B, cuyas. velocidades medias son de 30 y 40 km/h, respectivamente, distan 280 km. Hallara qué hora se encontrarán sabiendo que a las tres de la tarde empiezan a moverse el uno hacia el otro.

Sea I = tiempo, en horas, que tardan en encontrarse. Distancia = velocidad x tiempo.

Distancia recorrida por A + distancia recorrida por B = 280 km~ + .~ =~ de donde I = 4 h

Se encuentran a las 7 de la tarde a una distancia 301 = 120 km de la posición inicial de A, o bien a unadistancia de 401 = 160 km de la correspondiente a B.

26. Dos automóviles, A y B, parten de un mismo punto y recorren un trayecto rectilíneo con velocidades medias de30 y 50 km/h, respectivamente. Sabiendo que B parte 3 h después que A, hallar a) el tiempo y b) la distanciarecorrida, hasta que se encuentran.

Sean I y (1 - 3) el tiempo, en horas, que A y B viajan hasta que se encuentran.

a) Distancia (km) = velocidad media (krn/h] x tiempo (h). Cuando se encuentran:

distancia recorrida por A = distancia recorrida por B301 = 50(1 - 3)es decir, de donde I = 7!/, h.

Por tanto, A viaja durante I = 7!/, h y B viaja (1 - 3) = 4!/, h.

b) Distancia = 301 = 30(7!/,) = 225 km, o bien distancia = 50(1 - 3) = 50(4!/,) = 225 km.

27. Dos automóviles, A y B, recorren una pista circular de 1 km de longitud en 6 'j lO minutos, respectivamente. Su-poniendo que parten en el mismo instante y lugar, hallar al cabo de cuánto tiempo se encontrarán si se muevenalrededor de la pista a) en la misma dirección, b) en direcciones opuestas.

Sea I = el tiempo pedido en minutos.

a) Se encontrarán cuando A recorra 1 km más que B. Las velocidades de A y B son 1/6 y l/lO km/mrespectivamente. Por tanto, de la expresión espacio = velocidad x tiempo resulta:

Distancia recorrida por A - distancia recorrida por B=I km

1 1-1 -1 =1 de donde 1=15 mino6 10

b) Distancia recorrida por A + distancia recorrida por B = 1 km1 1-1 + -1 = 1 de donde 1= 15/4 mino6 lO

28. La velocidad, en aguas de reposo, de una motora es de 25 km/h. Sabiendo que cuando avanza contra corrienterecorre 4,2 km en el mismo tiempo que recorre a favor de ella 5,8 km, calcular la velocidad de la corriente.

Sea v = velocidad de la corriente. Tiempo = espacio/velocidad.

Tiempo contra la corriente = tiempo a favor de la corriente

4,2 km 5,8 kmde donde v = 4 krn,1l,es decir,

(25 -v) km/h (25 + v) krn/h

http://carlos2524.jimdo.com/

Page 102: Algebra Superior Murray R Spiegel

94 ECUACIONES LINEALES CON UNA INCOGNITA

PROBLEMAS DE TIEMPOS DE TRABAJO- 29. Un obrero A puede realizar un trabajo en 3 días y otro B lo puede hacer en 6 días. Hallar el tiempo que.tardarán en realizar dicho trabajo los dos juntos.

Sea 11 = número de días que tardan trabajando A y B.

En l día, A realiza 1/3 del trabajo y B hace 1/6 del mismo. Trabajando juntos realizarán 1/11 en l día.

l 1 l}+6=;; de donde 11 = 2 días.

Otro método. En 11 días, A y B realizan el trabajo completo.

l lII(} + 6) = l (trabajo completo). De aquí. resulta 11 = 2 días.

30. Tres grifos llenan un depósito en 20, 30 Y 60 minutos, respectivamente. Calcular el tiempo que tarda en llenarsedicho depósito cuando se utilizan los tres tubos simultáneamente.

Sea 1 = el tiempo (min) necesario.

l l lEn l minuto, los tres grifos juntos llenarán (20 + 30 + 60) del depósito. Por tanto, en 1 minutos llenarán

l l l1(20 + 30 + 60) = l (depósito completo). De aquí resulta 1 = 10 mino

31. Actuando juntos los operarios A y B realizan un trabajo en 6 días. El operario A trabaja dos veces más deprisa que B. Hallar los días que tardarán en realizar dicho trabajo trabajando cada uno por separado.

Sea 11. 211 = número de días que necesitan A y B. respectivamente, trabajando por separado.

En l día, A realiza l/n del trabajo y B hace 1/211 del mismo. Como en 6 días completan el trabajo. tendremos:

1 16(- + -) = l (trabajando completo).

11 211De aquí resulta 11 = 9 días. 211 = 18 días.

32. La velocidad a que trabaja A es tres veces mayor que la de B. Los operarios A y B empiezan a trabajar juntosdurante 4 h, al cabo de las cuales A se retira y continúa solo B. que termina el trabajo en 2 h. Hallar el tiempoque tardará B en realizar todo el trabajo si actuara él solo.

Sean l. 31 = tiempos. en horas, que tardarían A y B, respectivamente. trabajando solos.

En 1 h, A realiza 1/1 del trabajo y B hace 1/31 del mismo. Por tanto.

l l l4(- + -) + 2(-) = l (trabajo completo).1 31 1 De aquí resulta 31 = 22 h.

33. Un empleado cobra l 200 pts diarias cuando acude al trabajo y cuando no lo hace sufre una penalización de400 pts. Sabiendo que al cabo de 40 días la cantidad que percibió fue de 32000 pts, hallar el número de días quefaltó al trabajo.

Sea x = número de días que faltó al trabajo; 40 - x = número de días que trabajó.

Cantidad ganada - cantidad descontada = 32 000 ptsl 200(40 - x) 400 = 32000 de donde .v = 10 días faltó al trabajo.o sea.

34.

j

g

35.

36.

J

37.

94 ECUACIONES LINEALES CON UNA INCOGNITA

PROBLEMAS DE TIEMPOS DE TRABAJO

29. Un obrero A puede realizar un trabajo en 3 días y otro B lo puede hacer en 6 días. Hallar el tiempo que tardarán en realizar dicho trabajo los dos juntos.

Sea n = número de días que tardan trabajando A y B.

En 1 día , A realiza 1/3 del trabajo y B hace 1/6 del mismo. Trabajando juntos realizarán I /n en l día.

1 1 1 3+(;=;; de donde n = 2 días.

aIrO mélodo. En n días, A y B realizan el trabajo completo.

1 1 n(3 + (;) = l (trabajo completo). De aqu í. resulta n = 2 días.

30. Tres grifos llenan un depósito en 20, 30 Y 60 minutos, respectivamente. Calcular el tiempo que tarda en llenarse dicho depósito cuando se utilizan los tres tubos simultáneamente.

Sea 1 = el tiempo (min) necesario.

1 1 1 En 1 minuto, los tres grifos juntos llenarán (20 + 30 + 60) del depósito. Por tanto, en 1 minutos llenarán

1 1 1 1(20 + 30 + 60) = 1 (depósito completo). De aquí resulta 1 = 10 mino

31. Actuando juntos los operarios A y B realizan un trabajo en 6 días. El operario A trabaja dos veces más de prisa que B. Hallar los días que tardarán en realizar dicho trabajo trabajando cada uno por separado.

Sea n, 2n = número de días que necesitan A y B, respectivamente, trabajando por separado.

En 1 día , A realiza I /n del trabajo y B hace 1/211 del mismo. Como en 6 días completan el trabajo , tendremos:

1 1 6(- + - ) = l (trabajando completo).

11 211 De aquí resulta 11 = 9 días, 211 = 18 días.

32. La velocidad a q ue trabaja A es tres veces mayor que la de B. Los operarios A y B empiezan a trabajar juntos durante 4 h, a l cabo de las cua les A se retira y continúa solo B, que termina el trabajo en 2 h. Hallar el tiempo que tardará B en rea lizar todo el trabajo si actuara él so lo.

Sean 1, 31 = tiempos. en horas. que tardarían A y B, respectivamente, trabajando solos.

En l h, A realiza 1/1 del trabajo y B hace 1/ 31 del mismo. Por tanto,

1 l I 4(/ + 3/) + 2(/) = l (trabajo completo). De aquí resu lta 31 = 22 h.

33. Un empleado cobra l 200 pts diarias cuando acude al trabajo y cuando no lo hace sufre una penalización de 400 pts. Sabiendo que al cabo de 40 días la cantidad que percibió fue de 32000 pts, hallar el número de días que faltó al trabajo .

Sea x = número de días que faltó al trabajo: 40 - x = número de días que trabajó .

Cantidad ganada - cantidad descontada = 32 000 pts o sea, l 200(40 - x l 400 = 32000 de donde x = lO días fa ltó a l trabajo .

http://carlos2524.jimdo.com/

Page 103: Algebra Superior Murray R Spiegel

ECUACIONES LINEALES CON UNA INCOGNITA 95

PROBLEMAS PROPUESTOS

34. Resolver las ecuaciones siguientes:

a) 3x - 2 = 7

b) Y + 3(y - 4) = 4

e) 4x - 3 = 5 - 2x

d) x - 3 - 2(6 - 2x) = 2(2x - 5)

21 - 9 31 + 4e) -3-=-2-

h) (2x + 1)2 = (x - 1)' + 3x(x + 2)

3 4 1i) - - - =-

z 5z 10

2x+ 1 x-4j) -x-+x+I=3

5 522k) -- - -- = -- ---

y-I y+1 y-2 y+3

7 2 4/) x2 - 4 + x2 - 3x + 2 = x2 + X - 2

2x+3 x-If) --=-

2x-4 x+1

g) (x - 3)' + (x + 1)2 = (x - 2)2 + (x + 3)2

35. Despejar la incógnita indicada.

a) 2(x - p) = 3(6p - x) : x

b) 2by - 2a = ay - 4b :y

x-a x-ed) x _ b = x _ d: x

2x-a 2x-be) --b- = --a- :x

1 1 1e) - + - = -:y

ay by e

36. Representar las expresiones siguientes por medio de símbolos algebraicos.

a) Cinco veces un cierto número más dos.

b) Dos veces un cierto número menos seis.

e) Dos números cuya diferencia sea 25.

d) Los cuadrados de tres enteros consecutivos.

e) El exceso del quíntuplo de un cierto número sobre 40.

f) El cuadrado de un entero impar cualquiera.

g) El exceso del cuadrado de un número sobre el doble del mismo.

/¡) El número de centímetros cúbicos correspondientes a x litros.

i) La diferencia entre los cuadrados de dos enteros pares consecutivos.

j) Carlos es seis años mayor que Javier, y éste tiene la mitad de años que Pablo. Expresar sus edades en funciónde una sola de ellas.

k) Los tres ángulos, A, B, C, de un triángulo ABC si el ángulo A excede en 20 al doble del ángulo B.

1) El perímetro y el área de un rectángulo si uno de los lados es 3 m más pequeño que el triple del otro.

m) La fracción cuyo denominador es igual al cuadrado del numerador más cuatro.

n) La cantidad de sal en un depósito que contiene x litros de agua si la concentración es de 2 kg de sal porlitro.

37. Problemas de números.

a) Hallar un número sabiendo que su mitad es igual a su sexta parte más 10.

b) Hallar dos números cuya diferencia es 20 y su suma 48.

ECUACIONES LINEALES CON UNA INCOGNITA

PROBLEMAS PROPUESTOS

34. Resolver las ecuaciones siguientes:

a) 3x - 2 = 7

b) Y + 3(y - 4) = 4

e) 4x - 3 = 5 - 2x

d) x - 3 - 2(6 - 2x) = 2(2x - 5)

2/ - 9 3/ + 4 e) - 3- = - 2-

2x + 3 f)

x-I

2x - 4 x+1

g) (x - 3)' + (x + 1)2 = (x - 2)2 + (x + 3)'

35. Despejar la incógnita indicada.

a) 2(x - p) = 3(6p - x) : x

b) 2by - 2a = ay - 4b : y

2x-a 2x-b e) --b- = - -a- : x

h) (2x + 1)2 = (x - 1)2 + 3x(x + 2)

2x + 1 x-4 j) - x-+x+I=3

5 522 k) - - - -- = -- --­

y-I y+1 y -2 y+3

7 2 4 1) X2 _ 4 + X2 _ 3x + 2 = X2 + X - 2

x-a x-e d) x _ b = x _ d: x

1 1 1 e) - + - = -:y

ay by e

36. Representar las expresiones siguientes por medio de símbolos algebraicos.

a) Cinco veces un cierto número más dos .

b) Dos veces un cierto número menos seis.

e) Dos números cuya diferencia sea 25.

d) Los cuadrados de tres enteros consecutivos.

e) El exceso del quíntuplo de un cierto número sobre 40.

f) El cuadrado de un entero impar cualquiera.

g) El exceso del cuadrado de un número sobre el doble del mismo.

11) El número de centímetros cúbicos correspondientes a x litros.

i) La diferencia entre los cuadrados de dos enteros pares consecutivos.

95

j) Carlos es seis años mayor que Javier, y éste tiene la mitad de años que Pablo. Expresar sus edades en función de una sola de ellas.

k) Los tres ángulos, A, B, C, de un triángulo ABC si el ángulo A excede en 20 al doble del ángulo B.

1) El perímetro y el área de un rectángulo si uno de los lados es 3 m más pequeño que el triple del otro.

m) La fracción cuyo denominador es igual al cuadrado del numerador más cuatro.

n) La cantidad de sal en un depósito que contiene x litros de agua si la concentración es de 2 kg de sal por litro.

37. Problemas de números.

a) Hallar un número sabiendo que su mitad es igual a su sexta parte más 10.

b) Hallar dos números cuya diferencia es 20 y su suma 48.

http://carlos2524.jimdo.com/

Page 104: Algebra Superior Murray R Spiegel

96 ECUACIONES LINEALES CON UNA INCOGNITA

e) Hallar dos enteros pares consecutivos sabiendo que el doble del menor excede al mayor en 18.

d) Hallar dos números sabiendo que su suma es 36 y que al dividir el mayor por el menor el cociente es2 y el resto 3.

e) Hallar los enteros impares consecutivos sabiendo que la diferencia de sus cuadrados es igual a 64.

f) Hallar tres números cuya suma es 54 sabiendo que el primero es igual al doble del segundo más 4 y queel tercero es igual al doble del primero.

38. Problema de edades.

a) Un padre tiene 24 años más que su hijo. Determinar sus edades actuales sabiendo que dentro de 8 años laedad del padre es el doble que la del hijo.

b) Leticia tiene quince años más que su hermana Begoña. Hace seis años la edad de Leticia era seis veces la deBegoña. Hallar sus edades actuales.

e) La edad actual de Juan es el doble de la de Fernando. Hace cinco años Juan era tres veces mayor queFernando. Hallar sus edades actuales.

39. Problemas de monedas.

a) Una bolsa contiene 215 pts en monedas de 5 y 25. Sabiendo que hay 19 monedas más de 5 que de 25,hallar el número de monedas de cada clase.

b) Un muchacho tiene 500 pts en monedas de 25 y de 50 pts. Sabiendo que el número de las de 25 es igualal doble de las de 50, hallar el número de monedas de cada clase.

e) Las entradas de un teatro valen 50 pts para los adultos y 20 pts para los niños. Sabiendo que asistieron 280personas y que la recaudación fue de 8 000 pts, hallar el número de niños que asistieron a la función.

40. Problemas de dígitos.

a) Hallar un número de dos cifras sabiendo que la suma de éstas es igual a 1/7 del número y que la cifra delas decenas excede en 3 a la correspondiente de las unidades.

b) Hallar un número de dos cifras sabiendo que la suma de éstas es igual a 10 y que, si se invierten, el nú-mero que resulta es una unidad menor que el número original.

e) Hallar un número de dos cifras sabiendo que' la de las decenas es igual a 1/3 de la correspondiente de lasunidades y que, si se invierten, el número que resulta es igual al doble del primitivo más la suma de lascifras de éste más 2 unidades.

41. Problemas comerciales.

a) Un comerciante adquiere una mercancía a un precio de 72 pts. Hallar el precio a que la debe poner en ventapara que, haciendo un descuento del lO% sobre éste, gane en la operación un 20 % sobre el precio deventa.

b) Un empleado cobra 200 pts diarias cuando acude al trabajo y cuando no lo hace le descuentan 50 pts. Sa-biendo que al cabo de 25 días la cantidad de dinero que recibe es de 4 500 pts, hallar el número de días queacudió al trabajo.

e) En cierta factoría trabajan 400 empleados entre hombres y mujeres. Cada hombre percibe, diariamente,160 pts, y cada mujer, 120 pts. Calcular el número de mujeres empleadas sabiendo que la nómina diaria delpersonal asciende a 57 200 pts.

d) Una persona tiene invertidas 45000 pts, una parte al 2 % y la otra al 3 % de interés simple. Sabiendo quelos intereses que percibe anualmente ascienden a 1 100 pts, hallar las cantidades que tiene colocadas a losreferidos tipos de interés.

e) Una persona ha invertido 20 000 pts al 7 % y 50 ooo pts al 4 % de interés simple. Hallar la cantidad que debecolocar al 6 % para que el total invertido le resulte a un interés del 5 %.

42. Pn

a)

b}

e}

d}

e}

43. Pn

a}

b}

e}

d}

e}

fl

g)

h)

i)

j)

k)

1}

http://carlos2524.jimdo.com/

Page 105: Algebra Superior Murray R Spiegel

es

que

s la

lade

que

25,

gual

280

'a de

nú-

e lasle las

entao de

. Sa-que

ente,a del

quea los

debe

ECUACIONES LINEALES CON UNA INCOGNITA 97

42. Problemas de medidas.

a) Hallar las dimensiones de un rectángulo sabiendo que su perímetro es igual a 110 cm y que su longitud es5 cm más pequeña, que el doble de su altura.

b) Hallar las dimensiones de una puerta rectangular sabiendo que su altura es 80 cm mayor que su anchura yque, si se aumentan sus dimensiones en 20 cm, el área se incrementa en 6000 cm"

e) El área de un cuadrado excede a la de un rectángulo en 3 cm" Hallar el lado del cuadrado sabiendo quela anchura del rectángulo es 3 cm más pequeña que el lado del cuadrado, y que la altura de aquél es 4 cmmayor que éste.

d) El perímetro de un triángulo rectángulo es igual a 40 cm. Sabiendo que uno de los catetos mide 15 cm, ha-llar la longitud de los otros dos lados.

e) La longitud de una piscina es igual al doble de su anchura. Determinar sus dimensiones sabiendo que susparedes tienen 4 m de altura y un área de 720 m'.

43. Problemas de mezclas.

a) Para limpiar manchas de grasa en tejidos de lana o de cuero se puede emplear un disolvente a base de 80 %de tetracloruro de carbono (en volumen), 16 % de ligroin y 4 % de alcohol. Calcular los litros que se debentomar de cada componente para formar 75 litros de disolvente.

b) Mezclando un aceite de 28 pts el litro con otro de 33 pts el litro se quieren obtener 45 litros de un productoal precio de 30 pts el litro. Calcular las cantidades que se deben tomar de cada uno de los tipos de aceite.

e) Hallar la masa de agua que se debe añadir a 50 kg de una solución de ácido sulfúrico al 36 % paraobtener una solución al 20 %. Los tantos por ciento son en masa.

d) Hallar el número de litros de alcohol puro que se deben añadir a 10 litros de una solución al 15 % paraobtener una solución de alcohol al 25 %. Los tantos por ciento son en volumen.

e) Se dispone de 60 litros de una solución de glicerina yagua al 50 %. Hallar el volumen de agua que sedebe añadir para reducir la concentración de glicerina al 12 %. Los tantos por ciento son en volumen.

f) Se dispone de 4 litros de una solución anticongelante de agua y glicerina al \O %. Hallar el número de litrosde solución que se deben remplazar por igual de glicerina para que la solución resultante sea del 25 ~;.;.Lostantos por ciento son en volumen.

g) Se tienen I 000 litros de leche con un 4 % de nata. Determinar cuántos litros de leche, con un contenido ennata de un 23 %, se deben separar de los anteriores para obtener una leche cuyo porcentaje de nata sea deun 3 %. Los tantos por ciento son en volumen.

h) Se dispone de 10 ton de un carbón con un contenido en azufre del 2,5 %, y de otros dos tipos de carbóncuyos contenidos en azufre son 0,8 :~~y 1,10 %' respectivamente. Hallar las cantidades de estos últimos quese deben mezclar con las 10 ton del primero para obtener 20 ton de carbón con un contenido en azufre del 1,7 %.

i) Una arcilla contiene un 45 % de sílice y un 10 % de agua. Hallar el tanto por ciento de sílice en una arcillaseca (sin agua). Los tantos por ciento son en peso .

j) Un mineral de oro y cuarzo tiene una masa de 100 g. La densidad del oro es 19,3, la del cuarzo 2,6 y la delmineral 6,4 (gramos por centímetro cúbico). Hallar la masa de oro que contiene el mineral.Ind. Sea x = masa de oro en el mineral; (100 - x) = masa de cuarzo en el mineral.

Volumen del mineral = volumen de oro en el mineral + volumen de cuarzo en el mineral.

k) Se toma una muestra de 8.41 g de un aceite y se la calienta hasta una temperatura de 1\00 C evaporándose5,83 g de agua. Disolviendo en agua y evaporando después, se separaron 1,27 g de glicerina y el resto era aceite.Hallar la composición de la muestra.

1) Un carbón contiene un 2,4 ~,~de agua. Una vez seco, el contenido en carbón del residuo fue del 71,0 ~;.;.Ha-llar el tanto por ciento de carbón. Calcular el tanto por ciento de carbón en la muestra húmeda. Todos losporcentajes son en peso.

ECUACIONES LINEALES CON UNA INCOGNITA 97

42. Problemas de medidas.

a) Hallar las dimensio"nes de un rectángulo sabiendo que su perímetro es igual a 110 cm y que su longitud es 5 cm más pequeña . que el doble de su altura.

b) Hallar las dimensiones de una puerta rectangular sabiendo que su altura es 80 cm mayor que su anchura y que, si se aumentan sus dimensiones en 20 cm, el área se incrementa en 6000 cm 2

.

e) El área de un cuadrado excede a la de un rectángulo en 3 cm 2 Hallar el lado del cuadrado sabiendo que la anchura del rectángulo es 3 cm más pequeña que el lado del cuadrado, y que la altura de aquél es 4 cm mayor que éste.

d) El perímetro de un triángulo rectángulo es igual a 40 cm. Sabiendo que uno de los catetos mide 15 cm, ha­llar la longitud de los otros dos lados.

e) La longitud de una piscina es igual al doble de su anchura. Determinar sus dimensiones sabiendo que sus paredes tienen 4 m de altura y un área de 720 m2

43. Problemas de mezclas.

a) Para limpiar manchas de grasa en tejidos de lana o de cuero se puede emplear un disolvente a base de 80 % de tetracloruro de carbono (en volumen), 16 % de ligroin y 4 % de alcohol. Calcular los litros que se deben tomar de cada componente para formar 75 litros de disolvente.

b) Mezclando un aceite de 28 pts el litro con otro de 33 pts el litro se quieren obtener 45 litros de un producto al precio de 30 pts el litro. Calcular las cantidades que se deben tomar de cada uno de los tipos de aceite .

e) Hallar la masa de agua que se debe añadir a 50 kg de una solución de ácido sulfúrico al 36 % para obtener una solución al 20 %. Los tantos por ciento son en masa.

d) Hallar el número de litros de alcohol puro que se deben añadir a 10 litros de una solución al 15 % para obtener una solución de alcohol al 25 %. Los tantos por ciento son en volumen.

e) Se dispone de 60 litros de una solución de glicerina yagua al 50 %. Hallar el volumen de agua que se debe añadir para reducir la concentración de glicerina al 12 %. Los tantos por ciento son en volumen.

f) Se dispone de 4 litros de una solución anticongelante de agua y glicerina al lO %. Hallar el número de litros de solución que se deben remplazar por igual de glicerina para que la solución resultante sea del 25 ~;.;. Los tantos por ciento son en volumen.

g) Se tienen 1 000 litros de leche con un 4 % de nata. Determinar cuántos litros de leche, con un contenido en nata de un 23 %, se deben separar de los anteriores para obtener una leche cuyo porcentaje de nata sea de un 3 %. Los tantos por ciento son en volumen.

h) Se dispone de 10 ton de un carbón con un contenido en azufre del 2,5 %, y de otros dos tipos de carbón cuyos contenidos en azufre son 0,8 :~~ y 1,10 %' respectivamente. Hallar las cantidades de estos últimos que se deben mezclar con las 10 ton del primero para obtener 20 ton de carbón con un contenido en azufre del 1,7 %.

i) Una arcilla contiene un 45 ~~ de sílice y un lO % de agua. Hallar el tanto por ciento de sílice en una arcilla seca (sin agua). Los tantos por ciento son en peso.

j) Un mineral de oro y cuarzo tiene una masa de 100 g. La densidad del oro es 19,3 , la del cuarzo 2,6 y la del mineral 6,4 (gramos por centímetro cúbico). Hallar la masa de oro que contiene el mineral. Ind. Sea x = masa de oro en el mineral ; (100 - x) = masa de cuarzo en el mineral.

Volumen del minera l = volumen de oro en el mineral + volumen de cuarzo en el mineral.

k) Se toma una muestra de 8,41 g de un aceite y se la calienta hasta una temperatura de 110° C evaporándose 5,83 g de agua. Disolviendo en agua y evaporando después, se separaron 1,27 g de glicerina y el resto era aceite. Hallar la composición de la muestra.

1) Un carbón contiene un 2,4 ~,~ de agua. Una vez seco, el contenido en carbón del residuo fue del 71,0 ~;.;. Ha­llar el tanto por ciento de carbón. Calcular el tanto por ciento de carbón en la muestra húmeda. Todos los porcentajes son en peso .

http://carlos2524.jimdo.com/

Page 106: Algebra Superior Murray R Spiegel

98 ECUAClONES LINEALES CON UNA INCOGNITA

44. Problemas de móviles.

a) Dos motoristas. a una distancia uno del otro de 225 km. empiezan a moverse a las 4 h 30' de la tarde ensentido contrario. Sabiendo que sus velocidades medias son de 40 y 45 km/h. hallar a qué hora se encontrarán.

b) Dos. aviones parten del mismo lugar y a la misma hora volando en direcciones opuestas. La velocidad deuno de ellos es 40 km/h mayor que la del otro. Sabiendo que al cabo de 5 h se encuentran a 2 000 kmde distancia. hallar sus velocidades medias.

e) Hallar la velocidad a que debe viajar un motorista A para alcanzar a otro B. que marcha a una velocidadde 20 km/h. sabiendo que A. partiendo 2 h después que B. desea alcanzarlo en 4 h.

d) Un motorista parte de una ciudad A a las 2 h de la tarde y viaja hacia la ciudad B a una velocidadde 30 km/h. Después de permanecer en B durante l h. regresa por el mismo camino a una velocidad de40 km/h y llega a A a las 6 h 30' de la tarde. Hallar la distancia entre A y B.

e) Un automovilista recorre una distancia de 265 km. Durante la primera parte del viaje marcha a una velocidadde 40 km/h y el resto lo hace a 35 km/h. Sabiendo que la duración del viaje fue de 7 h. hallar el tiempo queestuvo marchando a la velocidad de 40 km/h.

f) La velocidad de una canoa. en aguas en reposo. es de 8 km/h. Sabiendo que recorre 20 km a favor de lacorriente en el mismo tiempo que recorre 12 km en contra de ella. hallar la velocidad de la corriente.

g) La velocidad de un avión. en aire en reposo. es de 120 km/h. Cuando marcha a favor del viento recorre unacierta distancia en 4 h. pero cuando va en contra de él recorre solamente los 3/5 de la misma en igualtiempo. Hallar la velocidad del viento.

h) Una columna de soldados marcha a una velocidad de 5 km/h. Un enlace a caballo va desde la cabeza de lacolumna hasta el final de la misma y regresa inmediatamente. empleando un tiempo total de lO minutos.Suponiendo que la velocidad del enlace es de 10 km/h. hallar la longitud de la columna.

i) Un tren recorre una distancia en h horas a una velocidad de l' kilómetros por hora. Hallar en cuántos kiló-metros debe incrementar su velocidad para efectuar el rnjsmo recorrido en l h menos.

45. Problemas de tiempos de trabajo.

a) Un granjero puede trabajar un cierto terreno a una velocidad tres veces mayor que la de su hijo. Trabajandojuntos invierten 6 h en realizar la labor. Hallar el tiempo que tardarían en realizarlo trabajando por separado.

b) Un pintor puede realizar un trabajo en 6 h y su ayudante puede hacerlo en lO h. El pintor comienza atrabajar y al cabo de 2 h se incorpora al trabajo su ayudante. Hallar el tiempo que tardarán en completarel trabajo en cuestión.

e) Un grupo de operarios puede realizar un trabajo en 8 días. Después de que este grupo ha estado trabajando3 días. se incorpora un segundo grupo y. juntos. terminan el trabajo en otros 3 días más. Hallar el tiempo quetardaría en realizar dicho trabajo el segundo grupo trabajando por sí solo.

d) Dos grifos llenan un depósito en 10 y 15 minutos. respectivamente. Los 2 grifos anteriores y un tercero. ac-tuando todos simultáneamente. llenan el depósito en 4 minutos. Hallar el tiempo que tardaría en llenarseel depósito empleando solamente el tercer grifo.

SOLUCIONES DE LOS PROBLEMAS PROPUESTOS

34. a) x = 3b) Y = 4e) x = 4/3

x) x = -1/2/¡) todos los valores de x (identidad)i) z = 22

j) x = 1/4k) Y = 51) x = -1

dI' x = 5e) 1=-6fl x = 1/11

35. a) x = 4p

b) Y = -2 si 1I ,J, 2b

a + be) x = -2- si 1I 4= b

he - add) x =

h+e-a-d

36. a)h)e)d)

e)f)g)

h)i)

37. a)

38. a)

39. a)

40. a)

41. a)

42. a)h)e)

43. a)h)e)d)

elf)

44. a)b)

45. a)

http://carlos2524.jimdo.com/

Page 107: Algebra Superior Murray R Spiegel

locidadpo que

r de la

re unan igual

a de lainutos.

f + heah

ECUAClONES LINEALES CON UNA INC()(jNITA 99

36. a) 5x + 2h) 2x - 6e) x + 25. .vd¡ x2• (x + 1)2. (x + 2)2el 5x - 40fl (2x + 1)2 siendo .v = entero/?) x2 - 2x/¡) 1000xi) (2x + 2)2 - (2X)2. X ., entero

j) Edad de Javier x. Edad de Carlos x + 6. Edad de Pablo 2xk) B = x. A = (2x + 20)'. C = (160 - 3x)1) Un lado es .v, el lado adyacente es 3x - 3

Perímetro = 8x - 6. Area = 3x2 - 3x

xm) 2.\'2 + 4

n) 2 x kg de sal

37. a) 30 h) 34. 14 e) 20. 22 d) 25. 11 el 15. 17 f) 16. 6. 32

38. a) Padre 40. hijo 16 e) Juan 20. Fernando 10h) Leticia 24. Begoña 9,39. a) 4 de 25. 23 de 5 e] 200 niños. 80 adultosh) 5 de 50. 10 de 25

40. a) 63 h) 37 e) 26

41. a) 100 pts e) 170 mujeres <'l \O 000 ptsd) 20000 pts al 3 '.•.••.25000 pts al 2 ".~h) 23 días

42. a) ancho 20 cm. altura 35 cmh) ancho 100 cm. altura 180 cme) 9 cm

d) otro cateto 8 cm. hipotenusa 17 cmel 30 m por 60 m

43. a) 60. 12. 3 litrosh) 18 1del de 33 pts. 27 l del de 28 ptsc) 40 kgd) 4/3 lel 1901f) 2.'31

/?) 50 l/¡) 6.7 ton de 0.80 ";.;. 3.3 ton de 1.1O~..~i) 50 '~~sílicej) 69 g de orok) 69.3 ~~~humedad. 15.1 ~\ glicerina. 15.6:';~ aceite1) 69.3 % carbón

44. a) 7 h 30' de la tardeh) 180. 220 kmfh

e) 60 kmfhd) 60 km

e) 4 hf) 2 kmfh

/?) 30 krn/h/¡) 5i8 km i) h=I

45. a) Padre 8 h. hijo 24 h h) 2~ h e) 12 días d) 12 minutos

ECUACIONES LINEALES CON UNA INC()(jNITA 99

36. a) Sx + 2 j) Edad de Javier x. Edad de Carlos x + 6. Edad de Pablo 2x h) 2x - 6 k) B= x. A = (2x + 20r. C = (160 - 3x) e) x + 25. x 1) Un lado es x . el lado adyacente es 3x - 3 d) X2, (x + 1)2, (x + 2)2 Perímetro = 8x - 6. Area = 3X2 - 3x

el Sx - 40

fl (2x + 1)2 siendo x = entero m) x

g) X2 - 2x 2.\'2 + 4

h) 1000x i) (2x + 2)2 - (2X)2. X ' .' entero n) 2 x kg de sal

37. a) 30 h) 34. 14 e) 20. 22 d) 25. I1 e) 15. 17 f) 16. 6. 32

38. al Padre 40. hijo 16 h) Leticia 24. Begoña 9 e) Juan 20. Fernando 10 '1

39. a) 4 de 25. 23 de 5 h) 5 de 50. 10 de 25 e) 200 niños. 80 adultos

40. al 63 hl 37 e) 26

41. al 100 pts h) 23 días e) 170 mujeres d) 20 000 pts al 3 '\ , 25 000 pts al 2 ~',~ el \O 000 pts

42. al ancho 20 cm. altura 35 cm hl ancho 100 cm. altura 180 cm el 9 cm

43. al 60. 12. 3 litros hl 18 I del de 33 pts. 27 I del de 28 pts el 40 kg d) 4/3 I el 190 I f) 2.'31

44. (/l 7 h 3D' de la tarde hl 180, 220 kmfh

45. a) Padre 8 h. hijo 24 h

e) 60 kmfh dI 60 km

hl 2~ h

d) otro cateto 8 cm. hipotenusa 17 cm el 30 m por 60 m

gl 50 I hl 6.7 ton de 0.80 '::' ,3.3 ton de 1,10 ~ .. ~ il 50 '~~ sílice JI 69 g de oro k) 69,3 ~~~ humedad. 15.1 ~\ glicerina, 15,6 :'t~ aceite 1) 69.3 :%; carbón

el 4 h fl 2 kmfh

gl 30 km/h /¡) 5i8 km

el 12 días d) 12 minutos

il 7l=I

http://carlos2524.jimdo.com/

Page 108: Algebra Superior Murray R Spiegel

CAPITULO 12

Sistemas de ecuaciones lineales

UNA ECUACION LINEAL CON DOS INCOGNITAS (o variables) x e y es de la formaax + by = c, en donde a, b, e son constantes y a, b distintos de cero. Dos ecuaciones de este tipo

a¡x + b¡y = C¡

a2x + h2y = C2

constituyen un sistema de ecuaciones lineales, en este caso de dos ecuaciones con dos incógnitas.Todo par de valores de x e y que satisfagan ambas ecuaciones, simultáneamente, recibe el nombrede solución del sistema.

Por ejemplo, la solución del sistema x + y = 7 y x - y = 3 es x = 5, Y = 2.

SISTEMA DE ECUACIONES LINEALES CON DOS INCOGNIT AS. A continuación, se ex-ponen tres métodos para resolver un sistema de ecuaciones lineales.

A) Método de reducción. Cuando sea necesario, se pueden multiplicar las ecuaciones dadaspor números, de manera que los coeficientes de una de las incógnitas en ambas ecuaciones sea elmismo. Si los signos de los términos de igual coeficiente son distintos, se suman las ecuaciones ;en caso contrario, se restan. Consideremos

(1) 2x - y = 4(2) x + 2y = -3.

Para eliminar y, se multiplica (1) por 2 y se suma con (2), obteniendo

2 x (1): 4x '+ 2y = 8(2): x + 2y = - 3

Suma: 5x = 5 o sea x = 1.

Sustituyendo x = 1 en (1), se obtiene 2 - Y = 4, o sea y = - 2.Por tanto, la solución del sistema formado por (1) y (2) es x = 1, y = - 2.

Comprobación: Sustituyendo x = 1, Y = -2 en (2)se obtiene 1 + 2( -2) = -3, -3 = -3.

B) Método de sustitución. Consiste en despejar una incógnita en una de las ecuaciones y susti-tuir su valor en la otra.

Por ejemplo, consideremos el sistema formado por las ecuaciones (1) y (2) anteriores. De (1)se obtiene y = 2x -4 y sustituyendo este valor en (2) resulta x + 2(2x -4) = -3, de la que sededuce la solución x = 1. Sustituyendo x = I en (1), o en (2), se obtiene y = - 2.

C) Método gráfico. Consiste en trazar, en un sistema de coordenadas dado, las dos rectas qUe:representan las ecuaciones. La solución del sistema viene dada por las coordenadas (x, y) del punto

100

de in:=4

Por¡dicaevide

...

a)

los¡nes.si se

SISTEMmin:incó

SISTEMP

Resolver

1.

sS

ex +;

CAPITULO 12

Sistemas de ecuaciones lineales

UNA ECUACION LINEAL CON DOS INCOGNITAS (o variables) x e y es de la forma ax + by = c, en donde a, b, c son constantes y a, b distintos de cero. Dos ecuaciones de este tipo

alx + bly = C I

a2x + b2y = C2

constituyen un sistema de ecuaciones lineales, en este caso de dos ecuaciones con dos incógnitas. Todo par de valores de x e y que satisfagan ambas ecuaciones, simultáneamente, recibe el nombre de solución del sistema.

Por ejemplo, la solución del sistema x + y = 7 Y x - y = 3 es x = 5. Y = 2.

SISTEMA DE ECUACIONES LINEALES CON DOS INCOGNITAS. A continuación , se ex­ponen tres métodos para resolver un sistema de ecuaciones lineales.

A) Método de reducción . Cuando sea necesario, se pueden multiplicar las ecuaciones dadas por números, de manera que los coeficientes de una de las incógnitas en ambas ecuaciones sea el mismo. Si los sIgnos de los términos de igual coeficiente son distintos, se suman las ecuaciones; en caso contrario, se restan. Consideremos

(1) 2x - y = 4 (2) x + 2y = - 3.

Para eliminar y, se multiplica (1) por 2 y se suma con (2), obteniendo

2 x (1): 4x"+ 2v = 8 (2): x + 2y = - 3

Suma: 5x = 5 o sea x = 1.

Sustituyendo x = I en (1), se obtiene 2 - y = 4, o sea y = - 2. Por tanto, la solución del sistema formado por (1) y (2) es x = 1, y = -2.

Comprobación: Sustituyendo x = 1, y = -2 en (2) se obtiene 1 + 2( -2) = -3, -3 = - 3.

B) Método de sustitución. Consiste en despejar una incógnita en una de las ecuaciones y susti­tuir su valor en la otra.

Por ejemplo, consideremos el sistema formado por las ecuaciones (l) y (2) anteriores. De (1) se obtiene y = 2x -4 y sustituyendo este valor en (2) resulta x + 2(2x -4) = -3, de la que se deduce la solución x = 1. Sustituyendo x = 1 en (1), o en (2), se obtiene y = - 2.

C) Método gráfico. Consiste en trazar, en un sistema de coordenadas dado , las dos rectas qUe: representan las ecuaciones. La solución del sistema viene dada por las coordenadas (x, y ) del punto

100

http://carlos2524.jimdo.com/

Page 109: Algebra Superior Murray R Spiegel

itas.bre

ex-

adasea elnes;

-3.

usti-

SISTEMAS DE ECUACIONES LINEALES 101

de intersección de ambas. De la Fig. (a) se deduce que la solución del sistema formado por (1) 2x - y= 4 Y (2) x + 2y = -3 es x = 1, Y = -2, o bien (1, -2).

Si las rectas son paralelas, el sistema de ecuaciones es incompatible, es decir, no tiene solución.Por ejemplo, el sistema formado por (3) x + y = 2 Y (4) 2x + 2y = 8 es incompatible, como in-dica la Fig. (b). Obsérvese que si se multiplica la ecuación (3) por 2 se obtiene 2x + 2y = 4 que,evidentemente, es incompatible con (4).

a) Ecuaciones compatibles b) Ecuaciones incompatibles e) Ecuaciones dependientes

(1) 2x-y=4(2) x+2y=-3

(3) x + y = 2(4) 2x + 2y = 8

(5) x + y = t(6) 4x + 4y = 4

Las ecuaciones dependientes están representadas por una misma recta. Por consiguiente, todoslos puntos de la recta constituyen una solución y, en definitiva, el sistema tendrá infinitas solucio-nes. Por ejemplo, (5) x + y = 1 y (6) 4x + 4y = 4 son ecuaciones dependientes; obsérvese quesi se multiplica por 4 la ecuación (5) se obtiene la ecuación (6).

SISTEMA DE TRES ECUACIONES LINEALES CON TRES INCOGNITAS. Se resuelve eli-minando una incógnita en dos cualesquiera de las ecuaciones y a continuación eliminando la mismaincógnita en otras dos.

PROBLEMAS RESUELTOS

SISTEMAS DE ECUACIONES LINEALES CON DOS INCOGNITAS

Resolver los sistemas siguientes:

l. (1) 2x-y=4(2) x + y = 5

Sumando (1) Y (2) se obtiene 3x = 9, x = 3.

Sustituyendo x = 3 en (1) o en (2) se obtiene y = 2. La solución es x = 3, Y = 2 o (3, 2).

Otro método. De (1) se obtiene y = 2x - 4 Y sustituyendo este valor en la ecuación (2) se llega ax + 2x - 4 = 5, 3x = 9, x = 3. Sustituyendo x = 3 en (1) o en (2) se obtiene y = 2.

Comprobación: 2x - y = 2(3) - 2 = 4 Y x + y = 3 + 2 = 5.

Solución gráfica. La representación de una ecuación lineal es una línea recta. Como una recta queda de-

SISTEMAS DE ECUACIONES LINEALES \0\

de intersección de ambas. De la Fig. (a) se deduce que la solución del sistema formado por (\) 2x - y = 4 Y (2) x + 2y = -3 es x = \, Y = -2, o bien (\, -2).

Si las rectas son paralelas, el sistema de ecuaciones es incompatible, es decir, no tiene solución. Por ejemplo, el sistema formado por (3) x + y = 2 Y (4) 2x + 2y = 8 es incompatible, como in­dica la Fig. (b). Obsérvese que si se multiplica la ecuación (3) por 2 se obtiene 2x + 2y = 4 que, evidentemente, es incompatible con (4).

a) Ecuaciones compatibles

(1) 2x-y=4 (2) x+2y=-3

b) Ecuaciones incompatibles

(3) x + y = 2 (4) 2x + 2y = 8

e) Ecuaciones dependientes

(5) x + y = 1 (6) 4x + 4y = 4

Las ecuaciones dependientes están representadas por una misma recta. Por consiguiente, todos los puntos de la recta constituyen una solución y, en definitiva, el sistema tendrá infinitas solucio­nes. Por ejemplo, (5) x + y = \ y (6) 4x + 4y = 4 son ecuaciones dependientes; obsérvese que si se multiplica por 4 la ecuación (5) se obtiene la ecuación (6).

SISTEMA DE TRES ECUACIONES LINEALES CON TRES INCOGNITAS. Se resuelve eli­minando una incógnita en dos cualesquiera de las ecuaciones y a continuación eliminando la misma incógnita en otras dos.

PROBLEMAS RESUELTOS

SISTEMAS DE ECUACIONES LINEALES CON DOS INCOGNITAS

Resolver los sistemas siguientes:

l. (1) 2x-y=4 (2) x + y = 5

Sumando (1) Y (2) se obtiene 3x = 9, x = 3.

Sustituyendo x = 3 en (1) o en (2) se obtiene y = 2. La solución es x = 3, Y = 2 o (3, 2).

Olro método. De (1) se obtiene y = 2x - 4 Y sustituyendo este valor en la ecuación (2) se llega a x + 2x - 4 = 5, 3x = 9, x = 3. Sustituyendo x = 3 en (1) o en (2) se obtiene y = 2.

Comprobación: 2x - y = 2(3) - 2 = 4 Y x + y = 3 + 2 = 5.

Solución gráfica. La representación de una ecuación lineal es una línea recta. Como una recta queda de-

http://carlos2524.jimdo.com/

Page 110: Algebra Superior Murray R Spiegel

102 SISTEMAS DE ECUACIONES LINEALES

terminada por dos puntos. basta con representar dos puntos decada ecuación. Sin embargo, para obtener una precisión mayor sepueden representar tres puntos de cada recta.

Para 2x - y = 4: ~~

Para .v +)' = S: ~~

La solución del sistema es el punto de intersección (3, 2) delas rectas.

2. (1) Sx + 2)' = 3(2) 2x + 3y = - 1

Para eliminar y, se multiplica (1) por 3 y (2) por 2 y se restan los resultados.

ISx + 6y = 94x + 6y = -2

IIx 11

3 x (1):2 x (2):

Resta: o sea x = 1.

Sustituyendo x = 1 en (1) o en (2) se obtiene y = - l. La solución del sistema es (1, - 1).

(1) 2x+3)'=3(2) 6y - 6x = 1 Reordenando (2),

(1) 2x + 3.1' = 3(2) -6x + 6y = 1

3.

Para eliminar x, se multiplica (1) por 3 y se suma el resultado con (2) obteniéndose

3 x (1):(2):

6x + 9y = 9-6x + 6.1'= I

ISy = 10 o sea y = 2/3.

Sustituyendo y = 2/3 en (1) o en (2) se obtiene x = 1/2. La solución es (1/2, 2/3).

4. (1) Sy = 3 - 2x(2) 3x = 2.1'+ I Aplicamos el método de sustitución

3-h 3-2x 11De (1), y = --S-' . Sustituyendo este valor en (2) se obtiene 3x = 2(--S-) + 1 o sea x = 19'

Luego l' = 3 - 2x = 3 - 2(11/19) = 2. y 11 7, S S 19 la solución es (19' 19)'

s, x-2 y+1(1) -3-+-6-=2

x + 3 2y - 1(2)-4- - --2- =

Para quitar denominadores, se multiplica (1) por 6 y (2) por 4 y simplificando se obtiene

(I¡) 2x+y= IS(2¡) x - 4y = - 1

. S9 17Resolviendo, se obtiene x = 9' y = 9'

6.

7.

8.

9.

10.

11.

102

2.

3.

4.

s.

SISTEMAS DE ECUAC IONES LINEALES

terminada por dos puntos. basta con representar dos puntos de cada ecuación. Sin embargo, para obtener una precisión mayor se pueden representar tres puntos de cada recta .

Para 2x - y = 4 : 1: I =; 1_: I -21

Para .\ + )' = S: 1; 1-61 I ~ I 4 1

La solución del sistema es el punto de intersección (3 , 2) de las rectas.

(1) Sx + 2)' = 3 (2) 2x + 3y = - I

Para eliminar y, se multiplica (1) por 3 y (2) por 2 y se restan los resultados.

3 x (1): 2 x (2):

ISx + 6y = 9 4x + 6y = -2

Resta: Ilx 11 o sea x = 1.

Sustituyendo x = 1 en (1) o en (2) se obtiene y = - 1. La solución del sistema es (1, - 1).

(1) 2x+3y=3 (2) 6y - 6x = 1

Reordenando (2), (1) 2x + 3.1' = 3 (2) -6x + 6y = 1

Para eliminar x, se multiplica (1) por 3 y se suma el resultado con (2) obteniéndose

3 x (1): (2):

6x + 9y = 9 -6x + 6y = I

ISy = 10 o sea y = 2/3.

Sustituyendo y = 2/3 en (1) o en (2) se obtiene x = 1/2. La solución es (1/2, 2/3).

(1) Sy=3-2x (2) 3x = 2)' + I

Aplicamos el método de sustitución

3-2x S ' di ' 3-2x 11 De (1), )' = --S-o ustltuyen o este va or en (2) se obtiene 3x = 2(--S-) + o sea x = 19'

Luego l' = 3 - 2x = 3 - 2(11 / 19) = 2. y 11 7 . S S 19 la solución es (19 ' 19)'

x-2 y+1 (1) -3-+ - 6-=2

x + 3 2)' - 1 (2) -4- - --2- =

Para quitar denominadores, se multiplica (1) por 6 y (2) por 4 y simplificando se obtiene

(1 tl 2x + y = IS (2tl x - 4y = - 1

. S9 17 Resolviendo, se obtiene x = 9' y = 9'

http://carlos2524.jimdo.com/

Page 111: Algebra Superior Murray R Spiegel

SISTEMAS DE ECUACIONES LINEALES

6. (1) x - 3y = 2a(2) 2x + y = 5a

Para eliminar x, se multiplica (1) por 2 y se resta (2); se obtiene y = af]:Para eliminar y, se multiplica (2) por 3 y se suma con (1); se obtiene x = 17a/7.La solución es x '= 17a/7, y = afl .

7. (1) 3u + 2v= 7r+ s(2) 2u - v = 3s Se despeja u y v en función de r y s.

Para eliminar u, se multiplica (2) por 2 y se suma con (1); luego Tu = 7r + Ts o sea u = r + s.Para eliminar u, se multiplica (1) por 2, (2) por - 3, Y se suman los resultados; luego v = 2r - s.La solución es u = r + s, v = 2r - s.

8. (1) ax+by=2a2-3b2

(2) x + 2y = 2a - 6b

Se multiplica (2) por a y se resta de (1); tendremos by - 2ay = 6ab - 3b2, yib - 2a) = 3b(2a - b),3b(2a - b) -3b(b - 2a)

e y = b _ 2a = b _ 2a = - 3b siempre que b - 2a + O.

Análogamente, se obtiene x = 2a siempre que b - 2a + O.

Comprobación: (1) a(2a) + b(-3b) = 2a2 - 3b2, (2) 2a + 2(-3b) = 2a - 6b.

No/a. Si b - 2a = O, o sea b = 2a, las ecuaciones dadas se transforman en

(1.) ax + 2a)' = - lOa2

(2.) x + 2y = - lOa

que son dependientes, ya que (1.) se deduce de (2.) multiplicándola por a. Por tanto. si b = 2a. el sistema tieneinfinitas soluciones. es decir. todos los valores de x e y satisfacen a .r + 2y = -lOa.

9. Hallar dos números cuya suma es 28 y su diferencia 12.

Sean x e y los dos números pedidos. Tendremos (1) x + y = 28 y (2) .r - y = 12.

Sumando (I) y (2) se obtiene 2x = 40. x = 20. Restando (2) de (1) se obtiene 2y = 16. Y = 8.

No/a. Este problema también se puede resolver empleando solo una incógnita. Sean los números n y28 - n. Tendremos n - (28 - n) = 12. o sea n = 20 y 28 - n = 8.

10. Hallar una fracción sabiendo que si el numerador se aumenta en 2 y el denominador en I se obtiene 1/2. yque si el numerador se aumenta en I y el denominador se disminuye en 2. se obtiene 3/5.

Sea x = numerador. y = denominador y x/y = la fracción pedida. Tendremos

x + 2 1(1) -- = - o sea 2x - y = -3 y

r + I 2x+1

(2)Y - 2

35 o sea 5x - 3y -11

Del sistema formado por (1) y (2) resulta u = 2, Y = 7. La fracción pedida es 217·

11. Hace 2 años un padre era 6 veces mayor que su hijo. Hallar sus edades actuales sabiendo que dentro de 18 añosla edad del padre será el doble que la del hijo.

Sea x = edad actual del padre. y = edad actual del hijo.

Ecuación para la condición de hace 2 años:Ecuación para la condición de dentro de 18 años:

( 1) (x - 2) = 6(y - 2).(2) (x + 18) = 2(y + IS).

Resolviendo el sistema formado por (1) y (2) se obtiene x = 32. Y = 7.

103

6.

7.

8.

SISTEMAS DE ECUACIONES LINEALES

(1) x - 3y = 2a (i) 2x + y = 5a

Para eliminar x, se multiplica (1) por 2 y se resta (2); se obtiene y = a/7. Para eliminar y , se multiplica (2) por 3 y se suma con (1); se obtiene x = l7a/7. La solución es X'= 17aj7, y = a/7.

(1) 3u + 2v= 7r+ s (2) 2u - v = 3s Se despeja u y v en función de r y s.

Para eliminar v, se multiplica (2) por 2 y se suma con (1); luego 7u = 7r + 7s o sea u = r + s. Para eliminar u, se multiplica (1) por 2, (2) por - 3, Y se suman los resultados; luego v = 21' - s. La solución es u = r + s, v = 2r - s.

(1 ) ax + by = 2a2 - 3b2

(2) x + 2y = 2a - 6b

103

Se multiplica (2) por a y se resta de (1) ; tendremos by - 2ay = 6ab - 3b2 , y(b - 2a) = 3b(2a - b), 3b(2a - b) -3b(b - 2a)

e y = b _ 2a = b _ 2a = - 3b siempre que b - 2a * O.

Análogamente. se obtiene x = 2a siempre que b - 2a * O.

Comprobación: (1) a(2a) + b( - 3b) = 2a2 - 3b2 , (2) 2a+2(-3b)=2a-6b.

Nota. Si b - 2a = O, o sea b = 2a, las ecuaciones dadas se transforman en

(1,) ax + 2a)' = -lOa2

(2¡) x + 2y = -lOa

que son dependientes, ya que (1 ¡) se deduce de (2¡) multiplicándola por a. Por tanto. si b = 2a. el sistema tiene infinitas soluciones. es decir, todos los valores de x e)' satisfacen a x + 2y = -lOa.

9. Hallar dos números cuya suma es 28 y su diferencia 12.

Sean x e y los dos números pedidos. Tendremos (1) x + y = 28 Y (2) x - y = 12.

Sumando (1) Y (2) se obtiene 2x = 40, :r: = 20. Restando (2) de (1) se obtiene 2.1' = 16, r = 8.

Nota. Este problema también se puede resolver empleando solo una incógnita. Sean los números n y 28 - n. Tendremos n - (28 - n) = 12, o sea n = 20 Y 28 - n = 8.

10. Hallar una fracción sabiendo que si el numerador se aumenta en 2 y el denc;>minador en 1 se obtiene 1/2. y que si el numerador se aumenta en 1 y el denominador se disminuye en 2. se obtiene 3/5.

Sea x = numerador. )' = denominador y x /y = la fracción pedida. Tendremos

x + 2 1 (1) -- = - o sea 2x-y = -3 y

Y + 1 2

x + 1 3 (2) - - - - o sea 5x - 3y

y - 2 - 5

Del sistema formado por (1) Y (2) resulta x = 2, Y = 7. La fracción pedida es 2/7.

-11

11. Hace 2 años un padre era 6 veces mayor que su hijo. Hallar sus edades actuales sabiendo que dentro de 18 años la edad del padre será el doble que la del hijo.

Sea x = edad actual del padre. y = edad actual del hijo.

Ecuación para la condición de hace 2 años: Ecuación para la condición de dentro de 18 años:

(1) (x- 2)=6(y- 2). (2) (x + 18) = 2(y + Ul).

Resolviendo el sistema formado por (1) Y (2) se obtiene :r: = 32. Y = 7.

http://carlos2524.jimdo.com/

Page 112: Algebra Superior Murray R Spiegel

104 SISTEMAS DE ECUACIONES LINEALES

12. Hallar un número de 2 cifras que satisfaga las 2 condiciones siguientes: (1) el cuádruplo de la cifra de las unida-des es igual al doble de la correspondiente a las decenas menos 6, (2) el número es igual al triplo del que seobtiene invirtiendo sus cifras menos 9.

Sea t = la cifra de las decenas, u = la cifra de las unidades.

El número pedido = 10t + u; invirtiendo las cifras, el nuevo número = lOu + t. Luego

(1) 4u = 2t - 6 (2) IOt + u = 3(lOu + t) - 9y

Resolviendo el sistema formado por (1) y (2), t = 7, u = 2, el número pedido es 72.

13. Cinco cuadernos y 8 lapiceros cuestan 115 pts; 3 cuadernos y 5 lapiceros cuestan 70 pts. Hallar el precio de cadacuaderno y de cada lapicero.

Sea x = precio de un cuaderno, y = precio de un lapicero. Tendremos

(1) 5x+8y=115pts y (2) 3x + 5y = 70 pts

Resolviendo el sistema formado por (1) y (2) se obtiene x = 15 pts, y = 5 pts.

14. Un comerciante liquida sus existencias de lapiceros y gomas por 1 000 pts; los primeros los vende a razón de10 pts el conjunto de 3 lapiceros, y las segundas, a 2 pts cada una. Sabiendo que vendió solamente la mitadde los lapiceros y las 2 terceras partes de las gomas recaudando en total 600 pts, hallar las unidades que vendióde cada uno de los artículos citados.

Sea s = número de lapiceros vendidos, t = número de gomas vendidas. Tendremos

10(1) 3s + 2t = 1 000 y

Resolviendo el sistema formado por (1) Y (2), s = 120, t = 300.

15. Un inversionista ha colocado un cierto capital al 4 % una parte y al 5 % la otra recibiendo, anualmente, un in-terés de 1 100 pts. Si las hubiera invertido al revés, recibiría al año 50 pts más en concepto de intereses. Hallarla cantidad de dinero que ha invertido.

Sea x = cantidad invertida al 4 %, y = cantidad al 5 %. Tendremos

(1) O,04x + O,05y = 1 100 y (2) O,05x + O,04y = 1 150

Resolviendo el sistema formado por (1) y (2), x = 15000 pts, y = 10 000 pts, y su suma es 25000 pts.

16. Un depósito A contiene 10 litros de agua y 5 litros de alcohol puro. Otro depósito B contiene 12 litros de aguay 3 litros de alcohol. Hallar el número de litros que se deben extraer de cada depósito para conseguir una solu-ción de 8 litros que contenga un 25 %, en volumen, de alcohol.

En 8 litros de la solución que se quiere obtener habrá 0,25(8) = 2 litros de alcohol.Sean x, y = volúmenes extraídos de los depósitos A y B, respectivamente; en estas condiciones tendremosuna primera ecuación de la forma (1) x + y = 8.

5 1 3Fracción de alcohol en el depósito A = 10 + 5 = "3' y en el B = 12 + 3 = "5 . Por tanto, en x litros de

A habrá x/3 litros de alcohol, y en y litros de B habrá y/5 litros de alcohol; luego (2) x/3 + y/5 = 2.Resolviendo el sistema formado por (1) y (2) resulta x = 3 litros, y = 5 litros.

Otro método utilizando solo una incógnita.

Sea x = volumen extraído del depósito A; 8 - x = volumen extraído del B.

1 1 . liEntonces, -x + -(8 - x) = 2, de donde x = 3 htros, 8 - x = 5 itros,3 5

17. Cierta aleación contiene un 20 % de cobre y un 5 % de estaño. Hallar el número de kilogramos de cobre y estañoque se deben mezclar con 100 kg de la aleación dada para obtener una aleación que contenga un 30 % de cobrey un 10 % de estaño. Los tantos por ciento son en masa.

Sean x, y = número de kilogramos de cobre y de estaño que se han de alear, respectivamente.

18. Hall2 h I

19. Dosde 1,cruzsegu

20. Lasieny eltem

a)

b)

SISTEM

Resolver

21.

de

http://carlos2524.jimdo.com/

Page 113: Algebra Superior Murray R Spiegel

unida-que se

e cada

ón demitad

vendió

,un in-. Hallar

pts.

de aguana solu-

dremos

litros de

y estañoe cobre

SISTEMAS DE ECUACIONES LINEALES 105

En 100 kp de la aleación dada hay 20 kp de cobre y 5 kp de estaño. Luego, en la nueva aleación,

kilos de cobre 20 + xfracción de cobre o sea (1) 0,30 = -:-::-::-----

kilos de aleación 100 + x + y

fracción de estaño = kilos de estaño o sea (2) 0,10 = 5 + Ykilos de aleación 100 + x + y

La solución del sistema formado por (1) y (2) es x = 17,5 kp de cobre, y = 7,5 kp de estaño.

18. Hallar la velocidad de una barca, en aguas en reposo, y la velocidad de la corriente del río, sabiendo que emplea2 h en navegar 9 km a favor de la corriente y 6 h en recorrer dicha distancia en sentido contrario.

Sea x = velocidad en agua en reposo, y = velocidad de la corriente.

A favor de la corriente:En contra de la corriente:

2 h x (x + y) krn/h = 9 km6 h x (x - y) krn/h = 9 km

o seao sea

(1) 2x + 2y = 9.(2) 6x - 6y = 9.

Resolviendo el sistema formado por (1) y (2), x ~. 3 krn/h, y = 3/2 km/h.

19. Dos partículas se mueven a diferentes velocidades, pero constantes, alrededor de una circunferencia de 276 mde longitud. Hallar sus velocidades sabiendo que si parten del mismo punto e instante en sentido contrario secruzan cada 6 segundos, y si lo hacen en las mismas condiciones pero en el mismo sentido, se cruzan cada 23segundos.

Sean x, y = sus velocidades respectivas en mis.

6 s x (x + y) mis = 27623 s x (x - y) mis = 276

o seao sea

(1) 6x + 6y = 276.(2) 23x - 23y = 276 .:

17 mis.

Sentido opuesto:Mismo sentido:

Resolviendo el sistema formado por (1) y (2), x = 29 mis, y =

20. La temperatura en la escala fahrenheit = m(temperatura en la escala centígrada) + n, es decir, F = me + n,siendo m y n constantes. A la presión de 1 atm, la temperatura de ebullición del agua es 212° F, o bien 100° e,y el punto de congelación del agua es 3ZC F, o bien 0° C. a) Deducir los valores de m y n. b) Hallar latemperatura de la escala fahrenheit que corresponde a-273° C (la menor temperatura que se puede conseguir).

a) (1) 212 = m(100) + n y (2) 32 = m(O) + n. Resolviendo, m = 915, n = 32.

9 2 90' .. d °b) F = =C + 32 = -(-273) + 32 = -491,4 + 32 = -45 F, con aproxrmacion el.5 5

SISTEMAS DE TRES ECUACIONES LINEALES CON TRES INCOGNITAS

Resolver los sistemas siguientes:

21. (1) 2x- y+ z= 3(2) x + 3y - 2z = II(3) 3x - 2y + 4z = I

Para eliminar .r entre (1) Y (2) se multiplica (1) por 3 y se suma con (2) obteniendo

(1.) 7.'( + z = 20

Para eliminar}' entre (2) y (3) se multiplica (2) por 2, (3) por 3, y se suman los resultados

(2.) Ilx + 8z = 25

Resolviendo el sistema formado por (1, ) Y (2, ) se obtiene x = 3, z = - l. Sustituyendo estos valores en unade las ccuaciones dadas se obtiene y = 2.

Así. pues. la solución es x = 3, Y = 2. z = -l ..

SISTEMAS DE ECUACIONES LINEALES

En 100 kp de la aleación dada hay 20 kp de cobre y 5 kp de estaño. Luego, en la nueva aleación,

fracción de cobre kilos de cobre

kilos de aleación

fracción de estaño = kilos de estaño kilos de aleación

o sea

o sea

(1 ) O 30 = 20 + x , 100 + x + y

(2) 0,10 = -:-::-:::_5_+_ y __ 100 + x + y

La solución del sistema formado por (1) y (2) es x = 17,5 kp de cobre, y = 7,5 kp de estaño.

105

18. Hallar la velocidad de una barca, en aguas en reposo, y la velocidad de la corriente del río, sabiendo que emplea 2 h en navegar 9 km a favor de la corriente y 6 h en recorrer dicha distancia en sentido contrario.

Sea x = velocidad en agua en reposo, y = velocidad de la corriente.

A favor de la corriente: 2 h x (x + y) km/h = 9 km En contra de la corriente : 6 h x (x - y) km/h = 9 km

o sea o sea

(1) 2x + 2y = 9. (2) 6x - 6y = 9.

Resolviendo el sistema formado por (1) y (2), x ~. 3 km/h, y = 3/2 km/h.

19. Dos partículas se mueven a diferentes velocidades, pero constantes, alrededor de una circunferencia de 276 m de longitud. Hallar sus velocidades sabiendo que si parten del mismo punto e instante en sentido contrario se cruzan cada 6 segundos, y si lo hacen en las mismas condiciones pero en el mismo sentido, se cruzan cada 23 segundos.

Sean x, y = sus velocidades respectivas en mis.

Sentido opuesto : Mismo sentido:

6 s x (x + y) mis = 276 23 s x (x - y) mis = 276

o sea (1) 6x + 6y = 276. o sea (2) 23x - 23y = 276 ..

Resolviendo el sistema formado por (1) y (2), x = 29 mis, y = 17 mis.

20. La temperatura en la escala fahrenheit = m(temperatura en la escala centígrada) + n, es decir, F = mC + n, siendo m y n constantes. A la presión de 1 atm, la temperatura de ebullición del agua es 212° F, o bien 100° C, y el punto de congelación del agua es 3ZC F, o bien OC C. a) Deducir los valores de m y n. b) Hallar la temperatura de la escala fahrenheit que corresponde a-273° C (la menor temperatura que se puede conseguir).

a) (1) 212 = m(100) + n y (2) 32 = m(O) + n. Resolviendo, m = 9/ 5, n = 32.

9 2 b) F = - C + 32 = -( - 273) + 32 = -491,4 + 32 = -459° F, con aproximación de 1°.

5 5

SISTEMAS DE TRES ECUACIONES LINEALES CON TRES INCOGNITAS

Resolver los sistemas siguientes:

21. (1) 2x- y+ z= 3 (2) x + 3y - 2z = II (3) 3x - 2y + 4z =

Para eliminar .r entre (1) Y (2) se multiplica (1) por 3 y se suma con (2) obteniendo

(1.J 7x + z = 20

Para eliminar}' entre (2) y (3) se multiplica (2) por 2, (3) por 3, y se suman los resultados

(2,) \\x+8z=25

Resolviendo el sistema formado por (\,) Y (2,) se obtiene x = 3, Z = -1. Sustituyendo estos valores en una de las ecuaciones dadas se obtiene y = 2.

Así. pues, la solución es x = 3, Y = 2. Z = -l.

http://carlos2524.jimdo.com/

Page 114: Algebra Superior Murray R Spiegel

106 SISTEMAS DE ECUAClONES LINEALES

22.x y z

(1 ) "3 + "2 - 4 = 2.

Para quitar denominadores. multipliquemos las ecuaciones por 12, obteniendo el sistema

(1 tl 4x + 6y - 3z = 24(2tl 3x + 4y - 6z = 2(3tl 6x - 3y + 4z = 46

Para eliminar x entre (ltl y (2tl multiplicamos (1,) por 3, (2tl por -4, y sumamos los resultados ob-teniendo

Para eliminar x entre (2 tl y (3 tl multiplicamos (2 tl por 2 y restamos (3,) obteniendo

(22) Ily - 16z = -42

La solución del sistema formado por (12) Y (22) es y = 2, z = 4. Sustituyendo estos valores de y y z en unade las ecuaciones dadas, se obtiene x = 6.

Así, pues, la solución del sistema formado por las tres ecuaciones dadas es x = 6, Y = 2, z = 4.

23.122

(1) -----=0x y z '

313(3) - - - - - = 3.

x y z231

(2) - + - + - = 1,x y 2

Haciendo1 I I- = u. - = v, - = wx y 2

las ecuaciones dadas se transforman 'en

( 1 tl u - 2v - 2w = O(2tl 2u + 3v + w = I(3,) 3u- v -3w=3 del que se obtiene u = -2,v=3.w= -4.

I I 1Por tanto. - = -2 o x = -1/2. - = 3 o y = 1/3. - = -4 o 2 = -1/4.

x y

Comprobación:

24. (1) 3x + y - 2 = 4, (2) x + y + 42 = 3, (3) 9x + 5y + 10z = 8.

Restando (2) de (1). se obtiene (1 tl 2x - 52 = 1.Multiplicando (2) por 5 y restando (3) se obtiene (2tl -4x + 10z = 7.

Ahora bien. (1,) y (2tlson incompatibles, ya que al multiplicar (1,) por -2 resulta -4 x + 10z = -2 quees incompatible con (2,). Ello significa que el sistema dado es incompatible y que, por consiguiente, no tiene so-lución.

25. Los obreros A y B trabajando juntos pueden realizar una tarea en 4 días; B y e juntos pueden hacerlo en 3 días.y A Y e en 2.4 días. Hallar el tiempo que tardaría cada obrero en realizar dicha tarea actuando independiente-mente.

Sean {l. h. e = los días que precisan cada uno para efectuar solos el trabajo. respectivamente.

Tendremos ~. ~. ~ = fracción del trabajo completo que cada uno realiza en I día. respectivamente. Luego(I h e

I I l(2) b + -;:= "3'

I I I(3) - + - =-

a e 2.4\

Resolviendo el sistema formado por (1). (2) y (3). se obtiene II = 6. h = 12. e = 4 días.

SISTEMP

26. Reso

a)

h) ,

e)

27. Reso

a)

h)

e)

d) •

28. Indic

Ia) e

b)

29. Probl

a)

b) ~2

e) F

el)

http://carlos2524.jimdo.com/

Page 115: Algebra Superior Murray R Spiegel

SISTEMAS DE ECUACIONES LINEALES 107

PROBLEMAS PROPUESTOS

SISTEMAS DE ECUACIONES LINEALES CON DOS INCOGNITAS

26. Resolver los siguientes sistemas de ecuaciones por los métodos que se indican.

a) {2X -3y = 73x+ .1'=5

b) {3X - y = -62x + 3y = 7

e) {4X + 2y = 5Sx - 3.1' = -2

Resolver (1) por el método de reducción. (2) por el de sustitución.

Resolver (1) gráficamente. (2) por el método de reducción.

Resolver (1) gráficamente. (2) por el método de reducción. (3) por el de sustitución.

27. Resolver los siguientes sistemas de ecuaciones por uno de los métodos.

a) { 2x - Sy = 10 e) { 2x - 3y = 914x + 3y = 7 4x - y = 81

b) {2Y - x = I f) {2X + y + 1=02x+y=8 3x - 2y + 5 = Or y-, g) {2U - v = -Ss3 +"5- 3u + 21> = 7,. - 4s

e)x y h) {s/x - 3/y = I"6 - "2 = -4 2/x + l/y = 7

{"'-= 1 + Lk.-'~ 4 i) {ax - by = al + b1

3 4 Zbx - ay = 2h1 + 3ab - ald)

x+3_x-y=32 3

Hallar u y r en función de r y s.

Hallar x e y en función de ti y b.

28. Indicar cuáles de los sistemas siguientes son (1) compatibles. (2) indeterminados. (3) incompatibles.

{3X = 2y + 3

x - 2y/3 = I

{(x + 3 )/4 = (2y - 1)/6

3x - 4y = 2

a) e) el {2X - y = I

2y - .v = I

{(x + 2)/4 -.(y - 2)/12 = Sl4

y = 3x - 7

{

X + 3y = 42x- y= I

{2X - y = 5

2y = 7 + 4x .f)b) d)

29. Problemas dé números.

a) Hallar dos números sabiendo que si uno de ellos se suma con el doble del otro se obtiene 21. y que si esteúltimo se suma con el doble del primero resulta 18.

b) Hallar una fracción sabiendo que si se aumentan el numerador y el denominador en 3 unidades se obtiene2/3. y que si ambos se disminuyen en 2 unidades resulta 1/2.

e) Hallar dos números sabiendo que el doble de su suma es igual al triple de su diferencia más 8. y que susemisuma es igual a su diferencia más l.

d) Hallar dos números sabiendo que si se divide el mayor por el menor da un cociente 6 y un resto también6. '1 que si se divide el quintuplo del me-vor por el mayor. el cociente es 2 y el resto 3.

http://carlos2524.jimdo.com/

Page 116: Algebra Superior Murray R Spiegel

108 SISTEMAS DE ECUACIONES LINEALES

.30. Problemas de edades.

a) Hace 6 años. Agustín era 4 veces mayor que Pablo. Hallar sus edades actuales sabiendo que dentro de 4 añossolo será dos veces mayor que Pablo.

b) A es 11 veces mayor que B. Dentro de cierto número de años. A será 5 veces mayor que B y, 5 añosmás tarde será 3 veces mayor que B. Hallar sus edades actuales.

31. Problemas de dígitos.

a) Hallar un número de 2 cifras sabiendo que el triple de la cifra de las decenas es igual al cuádruplo dela correspondiente a las unidades más 2, y que la diferencia entre el número dado y el obtenido al invertirsus cifras es igual al doble de la suma de éstas menos 2.

b) Hallar un número de 2- cifras sabiendo que si se divide por el número obtenido al invertir sus cifras el co-ciente es 2 y el resto 7, y si se divide por la suma de sus cifras el cociente es 7 y el resto 6.

32. Problemas comerciales.

a) Dos kilogramos de café y 3 kg de mantequilla cuestan 420 pts. Al cabo de 1 mes, el precio del café ha su-bido un lO % y el de la mantequilla un 20 % de forma que la adquisición de los productos anteriores cuestaahora 486 pts. Hallar el precio primitivo de cada uno de los productos.

b) Si se mezclan 3 litros de aceite del tipo A con 7 litros del tipo B el precio de la mezcla es de 43 pts el litro.Sin embargo. si se mezclan 3 litros del aceite A con 2 litros de B el precio de la mezcla es de 46 pts ellitro. Hallar el precio del litro de cada uno de los tipos de aceite.

e) Un inversionista tiene colocado parte de su capital al 3 ~;,;y el resto al 5 ~<,.de interés simple. percibiendoanualmente 11 600 pts de intereses. Si aumenta en un 25 % el dinero que tiene al 3 %. y en un 40 /:" el quetiene al 5 %. sus intereses anuales aumentan en 4 100 pts. Hallar el dinero que tiene invertido a cada uno delos tipos de interés.

33. Problemas de mezclas.

a) Un depósito A contiene 32 litros de una solución de alcohol al 25 % en volumen. Otro depósito B contiene50 litros de solución de alcohol al 40 '1~en volumen. Hallar el volumen que se extrae de cada uno de ellospara formar 40 litros de solución de alcohol al 30 ~~ en volumen.

b) Un depósito A contiene 40 litros de una solución salina con una cantidad de sal de 80 kg. Otro depósito Btiene 120 litros de UDa solución con 60 kg de sal disuelta. Hallar el volumen que se debe extraer de cadauno de ellos para formar 30 litros de solución cuya concentración sea de 1.5 kg/litro.

e) Una aleación contiene un 10 '~~de cinc y un 20 ~~ de cobre. Hallar el número de kilogramos de cinc y cobreque se deben alear con 100 kg de la aleación dada. para obtener otra aleación con un 20 '."0 de cinc y un24 % de cobre. Los tantos por ciento son en masa.

d) Una aleación. cuya masa es de 600 kg. está compuesta por 100 kg de cobre y 50 kg de estaño. Otraaleación. de 1 000 kg. está compuesta de 300 kg de cobre y 150 kg de estaño. Hallar las masas de cobrey de estaño que se deben mezclar con las dos aleaciones dadas para obtener una tercera aleación con un32 % de cobre y un 28 ~';';de estaño. Los tantos por ciento son en masa.

34. Problemas de móviles.

a) Hallar la velocidad de una motora. en aguas en reposo. y la velocidad de la corriente de un río. sabiendo quetarda 3 h en recorrer una distancia de 45 km aguas arriba. y 2 h en recorrer 50 km aguas abajo.

b¡ Hallar las velocidades. en kilómetros por hora. de 2 automóviles sabiendo que se mueven. partiendo en elmismo instante y del mismo lugar. alrededor de una pista circular de 1 km de longitud. y que cuando se mue-ven en direcciones opuestas se cruzan cada 18 s. mientras que cuando lo hacen en la misma dirección se cruzancada 90 s.

e) Un pasajero. situado en la cabeza de un tren A. observa que otro tren B de 110 m de longitud tarda lIs

SIS1

35.

36.

37.

38.

SOL

26.

27.

28.29.

30.

32.

33.

34.

35.

36.

37.

38.

108 SISTEMAS DE ECUACIONES LINEALES

.30. Problemas de edades.

a) Hace 6 años, Agustín era 4 veces mayor que Pablo. Hallar sus edades actuales sabiendo que dentro de 4 años solo será dos veces mayor que Pablo.

b) A es 11 veces mayor que B. Dentro de cierto número de años. A será 5 veces mayor que B y, 5 años más tarde será 3 veces mayor que B. Hallar sus edades actuales.

31. Problemas de dígitos .

a) Hallar un número de 2 cifras sabiendo que el triple de la cifra de las decenas es igual al cuádruplo de la correspondiente a las unidades más 2, y que la diferencia entre el número dado y el obtenido al invertir sus cifras es igual al doble de la suma de éstas menos 2.

b) Hallar un número de 2" cifras sabiendo que si se divide por el número obtenido al invertir sus cifras el co­ciente es 2 y el resto 7, y si se divide por la suma de sus cifras el cociente es 7 y el resto 6.

32. Problemas comerciales.

a) Dos kilogramos de café y 3 kg de mantequilla cuestan 420 pts. Al cabo de 1 mes, el precio del café ha su­bido un 10 % y el de la mantequilla un 20 % de forma que la adquisición de los productos anteriores cuesta ahora 486 pts. Hallar el precio primitivo de cada uno de los productos.

b) Si se mezclan 3 litros de aceite del tipo A con 7 litros del tipo B el precio de la mezcla es de 43 pts el litro. Sin embargo. si se mezclan 3 litros del aceite A con 2 litros de B el precio de la mezcla es de 46 pts el litro. Hallar el precio del litro de cada uno de los tipos de aceite.

e) Un inversionista tiene colocado parte de su capital al 3 ~..;; y el resto al 5 ~I" . de interés simple, percibiendo anualmente 11 600 pts de intereses. Si aumenta en un 25 % el dinero que tiene al 3 %. y en un 40 /:" el que tiene al 5 %, sus intereses anuales aumentan en 4 100 pts. Hallar el dinero que tiene invertido a cada uno de los tipos de interés.

33. Problemas de mezclas.

a) Un depósito A contiene 32 litros de una solución de alcohol al 25 /:" en volumen. Otro depósito B contiene 50 litros de solución de alcohol al 40 '1~ en volumen . Hallar el volumen que se extrae de cada uno de ellos para formar 40 litros de solución de alcohol al 30 ~"~ en volumen .

b) Un depósito A contiene 40 litros de una solución salina con una cantidad de sal de 80 kg. Otro depósito B tiene 120 litros de UDa solución con 60 kg de sal disuelta. Hallar el volumen que se debe extraer de cada uno de ellos para formar 30 litros de solución cuya concentración sea de 1,5 kg/litro.

e) Una aleación contiene un 10 o/~ de cinc y un 20 ~~ de cobre. Hallar el número de kilogramos de cinc y cobre que se deben alear con 100 kg de la aleación dada, para obtener otra aleación con un 20 ' .... , de cinc y un 24 % de cobre. Los tantos por ciento son en masa.

d) Una aleación. cuya masa es de 600 kg. está compuesta por 100 kg de cobre y 50 kg de estaño. Otra aleación, de 1 000 kg, está compuesta de 300 kg de cobre y 150 kg de estaño. Hallar las masas de cobre y de estaño que se deben mezclar con las dos aleaciones dadas para obtener una tercera aleación con un 32 % de cobre y un 28 ~''';; de estaño. Los tantos por ciento son en masa.

34. Problemas de móviles.

a) Hallar la velocidad de una motora. en aguas en reposo. y la velocidad de la corriente de un río. sabiendo que tarda 3 h en recorrer una distancia de 45 km aguas arriba. y 2 h en recorrer 50 km aguas abajo.

h) Hallar las velocidades. en kilómetros por hora. de 2 automóviles sabiendo que se mueven. partiendo en el mismo instante y del mismo lugar. alrededor de una pista circular de 1 km de longitud. y que cuando se mue­ven en direcciones opuestas se cruzan cada 18 s. mientras que cuando lo hacen en la misma dirección se cruzan cada 90 s.

e) Un pasajero. situado en la cabeza de un tren A. observa que otro tren B de 110 m de longitud tarda 11 s

http://carlos2524.jimdo.com/

Page 117: Algebra Superior Murray R Spiegel

26. a)

27. a)

ontiene b)

e ellos e)

28. a)

'sito B 29. a)e cada

30. a)

y cobre 32. a)

e y un e)

33. a)

. Otracobre 34. a)

con un35. a)

b)

4 años

5 años

plo denvertir

el co-

ha su-cuesta

I litro.pts el

biendoel queuno de

do que

o en ele mue-cruzan

SISTEMAS DE ECUACIONES LINEALES 109

en pasar por delante de él cuando ambos trenes marchan en la misma dirección, mientras que cuando lo ha-cen en direcciones contrarias tarda solamente 1 s. Calcular las velocidades de ambos trenes.

SISTEMAS DE ECUACIONES LINEALES CON TRES INCOGNITAS

35. Resolver los sistemas de ecuaciones siguientes:

a)

{

2X-Y+2Z= -8x + 2y - 3z = 93x - y - 4z = 3

x y .!+.!+.!=53+2-z=7 x y z

e)x 3y z -6 d) 2 3 4 -114-2"+2= - - - --

x y z

x y z 321 -6'6-4-3 -+---=x y z

{

X=Y-2Zb) 2y = x + 3z + 1

z = 2y - 2x - 3

36. Indicar cuáles de los sistemas siguientes son (1) compatibles, (2) indeterminados, (3) incompatibies.

{

X+Y-Z=2a) x - 3y + 2z = 1

3x - 5y + 3z = 4 {

2X - y + z = 1b) x + 2y - 3z = -2

3x - 4y + 5z = 1 {

x+ y+2z=3e) 3x '- y + z = 1

2x + 3y - 4z = 8

37. Hallar 3 números sabiendo que el primero es igual al segundo más la mitad del tercero, que la suma del segundoy el tercero es igual al primero más 1, Y que si se resta el segundo de la suma del primero con el tercero elresultado es 5.

38. Hallar un número de 3 cifras sabiendo que si se divide por el número que resulta al invertir sus cifras elcociente es igual a 2 y el resto 25, que la cifra de las decenas es igual a la suma de la cifra de las centenas yla correspondiente a las unidades menos 1, Y que si se resta la cifra de las unidades de la cifra de las decenasse obtiene el doble de la cifra de las centenas.

SOLUCIONES DE LOS PROBLEMAS PROPUESTOS

x = 2, y = -1

x = 5(2, Y = -1x = 3, Y = 2x = 6, y = 10

e) x = 1(2, Y = 3(2

g) u = r - 2s, v = 2r + sh) x = 1(2, Y = 1(3i) x = a + b, y = a - b si a2 +- 2b2

d) Incompatible, e) Compatible, f) Indeterminado

b) x = -1, Y = 3

d) x = 5, Y = 2e) x = 31(2, Y = -21f) x = -1, Y = 1

Compatible, b) Incompatible, e) Indeterminado,

5, 8 d) 16,7

b) A tiene 22 años, B tiene 2 años 31. a) 64 b) 83

b) 7(12 e) 7, 3

Pablo 11 años, Agustín 26 años

Café 90 pts/kg, mantequilla 80 pts(kg120000 pts al 3 %, 160000 pts al 5 %26 2(3 1 de A b) 20 1 de A13 1(3 1 de B 10 1 de B

b) Tipo A 50 pts(l, Tipo B 40 pts/l

e) 150 kg cinc100 kg cobre

d) 400 kg cobre500 kg estaño

Motora 20 km/h, corriente 5 km/h b) 120 krn/h, 80 km/h

x = -1, Y = 2, z = - 2 e) x = 6, y = 4, z = - 3x == O, y = 2, z = 1 d) x = 1/2, Y = -1(3, z = 1(6

36. a) Indeterminado b) Incompatible e) Compatible

37. 4, 2, 3

38. 371

e) 60 mis, 50 mIs

http://carlos2524.jimdo.com/

Page 118: Algebra Superior Murray R Spiegel

CAPITULO 13

Ecuaciones de segundo grado con una incógnita

UNA ECUACION DE SEGUNDO GRADO en x es de la forma ax? + bx + e = O, siendo a, b,y e constantes y a =F O.

Por ejemplo, x2 - 6x + 5 = O, 2X2 + X - 6 = O Y 3x2 - 5 = O, son ecuaciones de segundogrado con una incógnita. Las dos últimas ecuaciones se pueden dividir por 2 y 3, respectivamente,

obteniéndose x2 + ~x - 3 = O Y x2 - ~ = O, siendo en ambos casos el coeficiente de x2 igual a l.

Una ecuación cuadrática pura es aquella que carece de término en x; por ejemplo, 4x2 - 5 = O.

RESOLVER UNA ECUACION DE SEGUNDO GRADO ax? + bx + e = O es hallar los valo-res de x que la satisfagan. Estos valores reciben el nombre de soluciones o raíces de la ecuación dada.

Por ejemplo, x2 - 5x + 6 = O se satisface para x = 2 Y x = 3. Por tanto, x = 2 Y x = 3 sonsoluciones o raíces de la citada ecuación.

METODOS DE RESOLUCION DE LAS ECUACIONES DE SEGUNDO GRADO.

A) Ecuaciones cuadráticas puras.

Ejemplo 1. x2 - 4 = O. Tendremos x2 = 4, x = ±2, Y las raíces son x = 2, -2.

2. 2X2 - 21 = O. Tendremos x2 = 21/2 Y las raíces son x = ± J2lii = ± !fo.3. x2 + 9 = O. Tendremos x2 = - 9 Y las raíces son x = ± P = ± 3i.

B) Por descomposición en factores.

Ejemplo 4. x2 - 5x + 6 = O se puede escribir en la forma (x - 3) (x - 2) = O. El productode los dos factores será cero cuando lo sea uno cualquiera de ellos o ambos a lavez. Si x - 3 = O, x = 3; si x - 2 = O, x = 2. Por consiguiente, las solucionesson x = 3, x = 2.

5. 3x2 + 2x - 5 = O se puede escribir en la forma (3x + 5) (x - 1) = O. Por tanto,de 3x + 5 = O Y x, 1 = O se obtienen las soluciones x = -5/3 Y x = l.

6. x2 - 4x + 4 = O se puede escribir en la forma (x - 2) (x - 2) = O.Por tanto, la ecuación tiene la raíz doble x = 2.

C) Formando un cuadrado perfecto.

Ejemplo 7. Resolver x2 - 6x - 2 = O.

110

1)

E)

LA SU!

CAPITULO 13

Ecuaciones de segundo grado con una incógnita

UNA ECUACION DE SEGUNDO GRADO en x es de la forma ax2 + bx + c = O, siendo a, b, y c constantes y a 4= O.

Por ejemplo, X2 - 6x + 5 = O, 2X2 + X - 6 = O Y 3X2 - 5 = O, son ecuaciones de segundo grado con una incógnita. Las dos últimas ecuaciones se pueden dividir por 2 y 3, respectivamente,

obteniéndose X2 + ~x - 3 = O Y X2 - ~ = O, siendo en ambos casos el coeficiente de X2 igual a l.

Una ecuación cuadrática pura es aquella que carece de término en x; por ejemplo, 4X2 - 5 = O.

RESOLVER UNA ECUACION DE SEGUNDO GRADO ax2 + bx + c = O es hallar los valo­res de x que la satisfagan. Estos valores reciben el nombre de soluciones o raíces de la ecuación dada.

Por ejemplo, X2 - 5x + 6 = O se satisface para x = 2 Y x = 3. Por tanto, x = 2 Y x = 3 son soluciones o raíces de la citada ecuación.

METODOS DE RESOLUCION DE LAS ECUACIONES DE SEGUNDO GRADO.

A) Ecuaciones cuadráticas puras.

Ejemplo 1. X2 - 4 = O. Tendremos X2 = 4, x = ±2, Y las raíces son x = 2, -2.

2. 2X2 - 21 = O. Tendremos X2 = 21 /2 Y las raíces son x = ± J2lii = ± !fo.

3. X2 + 9 = O. Tendremos X2 = - 9 Y las raíces son x = ± P = ± 3i.

B) Por descomposición en factores.

Ejemplo 4. X2 - 5x + 6 = O se pue~e escribir en la forma (x - 3) (x - 2) = O. El producto de los dos factores será Qero cuando lo sea uno cualquiera de ellos o ambos a la vez. Si x - 3 = O, x = 3; si x - 2 = O, x = 2. Por consiguiente, las soluciones son x = 3, x = 2.

5. 3X2 + 2x - 5 = O se puede escribir en la forma (3x + 5) (x - 1) = O. Por tanto, de 3x + 5 = O Y x, 1 = O se obtienen las soluciones x = -5/3 Y x = l.

6. X2 - 4x + 4 = O se puede escribir en la forma (x - 2) (x - 2) = O. Por tanto, la ecuación tiene la raíz doble x = 2.

C) Formando un cuadrado perfecto.

Ejemplo 7. Resolver X2 - 6x - 2 = O.

110

http://carlos2524.jimdo.com/

Page 119: Algebra Superior Murray R Spiegel

a, b,

ndoente,

1a 1.

= o.

valo-ada.

son

42.

uctoa laones

nto,

ECUACIONES DE SEGUNDO GRADO CON UNA INCOGNITA 111

Se escribe en un miembro los términos con la incógnita y se pasa el término in-dependiente al otro miembro.

x2-6x=2

Sumando 9 a ambos miembros el primero se transforma en un cuadrado perfecto,es decir,

x2 - 6x + 9 = 2 + 9 o (x - 3)2 = 11De donde x - 3 = ±fo Y las raíces son x = 3 ±fo.

Nota. Para aplicar este método (1) el coeficiente de x2 debe ser 1 y (2) el número que hayque sumar a los dos miembros ha de ser el cuadrado de la mitad del coeficiente de x.

Ejemplo 8. Resolver 3x2 - 5x + 1 = O.

Dividiendo por 3, 2 5xx - - =3

Sumando [_1 ( __ 5)J2 __ 25 Id' b2 3 36 a os os rmem ros,

2 5 25 1 25 13x - 3'x + 36 = - 3' + 36 = 36 '

5 2 13(x - 6') = 36'

¡!}± 6 y

5x--= 6

.) Aplicando la fórmula general.

Las soluciones de la ecuación de segundo grado ax: + bx + c = O vienen dadas por la fórmula

x = - b ± Jb2- 4ac

2a

en la que b2 - 4ac recibe el nombre de discriminante de la ecuación cuadrática.

Para deducir esta fórmula, véase el Problema 5.

Ejemplo 9. Resolver 3x2 - 5x + 1 = O. En este caso a = 3, b = - 5, e = 1 por tanto

- (-5) + J( -W - 4(3)(1) _ 5 ± ¡!}2(3) - 6

como en el Ejemplo 8.x=

E) Gráficamente

Las. raíces, o soluciones, reales de ax: + bx + e = O son los valores de x que correspondena y = O en la gráfica de la parábola y = ax" + bx + c. Esto es, las soluciones son las abscisasde los puntos en los que la parábola corta al eje x. Si la curva no corta al eje x, las raíces sonimaginarias.

LA SUMA Y EL PRODUCTO DE LAS RAICES de la ecuación cuadrática ax2 + bx + e = O vienen

b edados por S = - - y p = -.a a

Por ejemplo, en 2X2 + 7x - 6 = Otenemos a = 2,b = 7,c = -6 con lo que S = -7/2yP = -6/2 = -3.

ECUACIONES DE SEGUNDO GRADO CON UN A INCOGNITA 111

Se escribe en un miembro los términos con la incógnita y se pasa el término in­dependiente al otro miembro .

x2 -6x = 2

Sumando 9 a ambos miembros el primero se transforma en un cuadrado perfecto, es decir,

X2 - 6x + 9 = 2 + 9 o (x - 3)2 = 11

De donde x - 3 = ±fo Y las raíces son x = 3 ±fo.

Nota. Para aplicar este método (1) el coeficiente de X2 debe ser 1 y (2) el número que hay que sumar a los dos miembros ha de ser el cuadrado de la mitad del coeficiente de x .

Ejemplo 8. Resolver 3X2 - 5x + 1 = O.

Dividiendo por 3, 2 5x x - - = 3 3 ·

Sumando [_1 ( _ _ 5)J2 __ 25 Id· b 2 3 36 a os os mlem ros,

2 5 25 1 25 13 x - "3 x + 36 = - "3 + 36 = 36 '

5 2 13 (x - 6) = 36'

5 x - - =

6 ~ ± 6 y

1) Aplicando la fórmula general.

Las soluciones de la ecuación de segundo grado ax 2 + bx + c = O vienen dadas por la fórmula

x = - b + J b2 - 4ac

2a

en la que b2 - 4ac recibe el nombre de discriminante de la ecuación cuadrática.

Para deducir esta fórmula, véase el Problema 5.

Ejemplo 9. Resolver 3X2 - 5x + 1 = O. En este caso a = 3, b = - 5, c = 1 por tanto

x = - ( - 5) ± J (- W - 4(3)(1) _ 5 ± fo

2(3) - 6 como en el Ejemplo 8.

E) Gráficamente

Las. raíces, o soluciones, reales de ax2 + bx + c = O son los valores de x que corresponden a y = O en la gráfica de la parábola y = ax2 + bx + c. Esto es, las soluciones son las abscisas de los puntos en los que la parábola corta al ej·e x. Si la curva no corta al eje x, las raíces son imaginarias.

LA SUMA Y EL PRODUCTO DE LAS RAICES de la ecuación cuadrática ax2 + bx + c = O vienen

b c dados por S = - - y p = - .

a a

Por ejemplo, en 2X2 + 7x - 6 = O tenemos a = 2,b = 7,c = -6 con lo que S = - 7/2y P = - 6/2 = -3.

http://carlos2524.jimdo.com/

Page 120: Algebra Superior Murray R Spiegel

112 ECUACIONES DE SEGUNDO GRADO CON UNA INCOGNITA

Se deduce, pues, que una ecuación de segundo grado cuyas raíces son r¡ Yr2 presenta la formax2 - Sx + p = O, siendo S = r¡ + r2 YP = r¡r2' Por tanto, la ecuación de segundo grado cuyasraíces son x = 2 y x = - 5 es x2 - (2 - 5)x + 2( - 5) = O, es decir, x2 + 3x - 10 = O.

EL CARACTER DE LAS RAICES de la ecuación de segundo grado ax" + bx + e = O viene deter-minado por su discriminante b2 - 4ac.

Suponiendo que a, b, e, son números reales, se tiene:

1) Si b2 - 4ac > O, las raíces son reales y distintas.2) Si b2 - 4ac = O, las raíces son reales e iguales.3) Si b2 - 4ac < O, las raíces son imaginarias conjugadas.

En el caso de que los coeficientes a, b, c, sean números racionales, se tiene:

1) Si b2 - 4ac es un cuadrado perfecto =f O, las raíces son reales, racionales y distintas.2) Si b2 - 4ac = O, las raíces son reales, racionales e iguales.3) Si b2 - 4ac > O, pero no es un cuadrado perfecto, las raíces son reales, irracionales y distintas.4) Si b2 - 4ac < O, las raíces son imaginarias conjugadas.

Por ejemplo, 2X2 + 7x - 6 = O, cuyo discriminante es b2 - 4ac = 72 - 4(2)( -6) = 97,tiene raíces reales, irracionales y distintas.

ECUACION IRRACIONAL. Es aquella que tiene una, o más incógnitas, bajo el signo de una raíz(radical).

Por ejemplo Fx+3" - Jx = 1 Y ;yy = JY-=-4 son ecuaciones irracional es.

Para resolver una ecuación irracional, se despeja uno de los radicales, aislándolo en un miem-bro de la ecuación, y se pasan todos los demás términos al otro miembro. Elevando ambos miembrosde la ecuación a una potencia igual al índice del radical, desaparecerá dicha raíz. Este proceso secontinúa hasta que se hayan eliminado todos los radicales presentes.

Ejemplo 10. Resolver Fx+3 - Jx = 1.

Transponiendo términos Fx+3" = Jx + 1.

Elevando al cuadrado, x + 3 = x + 2Jx + 1 o sea Jx = l.

Finalmente, elevando al cuadrado los dos miembros de Jx = 1se obtiene x = l.

Comprobación. J1+3 - Jl = 1, 2 - 1 = 1.

Es muy importante comprobar los valores obtenidos ya que al aplicar este método se introdu-cen, frecuentemente, soluciones extrañas a la ecuación que habrá que rechazar.

UNA ECUACION DE TIPO CUADRA TlCO es de la forma az2" + bz" + e = O, siendo a =f O, b, e,y n =f O, constantes y z una función de x. Haciendo el cambio de variable z" = u, la ecuación setransforma en au2 + bu + e = O, que es una ecuación de segundo grado en la variable u. Con losvalores obtenidos de u se pueden obtener los correspondientes de z y, de estos, hallar los de x.

ECUA(

l. Res

a)

b)

e)

d)

e)

x'

no esn

POR O

2. Res

a)

b)

e)

d)

e)

f)

FORMi

3. Hall

al

b)

el

d)

http://carlos2524.jimdo.com/

Page 121: Algebra Superior Murray R Spiegel

rmayas

ter-

taso

97,

raíz

iem-broso se

= 1.

odu-

b, e,n selos

ECUACIONES DE SEGUNDO GRADO CON UNA INCOGNITA 113

PROBLEMAS RESUELTOS

ECUACIONES CUADRATICAS PURAS

l. Resolver.

a) x' - 16 = O. Luego x' = 16, x = ±4.

b) 4t' - 9 = O. De donde 4t' = 9, t' = 9/4, t = ± 3/2.

1e) 3 - x' = 2x' + 1. Luego 3x' = 2, x' = 2/3, x = ± J2i3 = ±3 fi.

3Luego x' = -9/4, x = ±J -9/4 = ±id) 4x' + 9 = O.

2x'-1 17e) ~ = x + 3 + x _ 3 .

x' = 9, Y x = ± 3.

Comprobación: Si x = 3 se sustituye en la ecuación dada, tendremos que dividir por cero, operación queno está definida. Luego x = 3 no es solución.

Tendremos2x' - 1 = (x + 3)(x - 3) + 17, 2x' - 1 = x' - 9 + 17,

2(-W - 1Si x = - 3, _ 3 _ 3

17- 3 + 3 + --- o sea

-3 - 317

6176

y x = - 3 es solución.

POR DESCOMPOSICION EN FACTORES

2. Resolver aplicando el método de la descomposición en factores.

a) x' + 5x - 6 = 0, (x + 6)(x - 1) = 0, x = -6, 1.

b) t' = 4t, t' - 4t = 0, 1(1 - 4) = 0, 1 = 0,4.

c) x' + 3x = 28, x' + 3x - 28 = 0, (x + 7)(x - 4) = 0, .v = -7, 4.

d) 5x - 2x' = 2, 2x' - 5x + 2 = 0, (2x-1)(x-2)=0, x= 1/2, 2.

l l 5el 1=1 + 1 _ 4 = 4' Multiplicando por 4(1 - 1)(1 - 4).

4(1 - 4) + 4(1 ~ 1) = 5(1 - 1)(1 - 4). 51' - 331 + 40 = O. (t - 5)(5t - 8) = 0,

Il y 3p 6.1" - 5py - 6p' = O. (~.r+ 2p)(2y - 3p) = 0, .r = - 2p/3. 3p/2.2p 6y -s¡,'

1 = 5. 8/5.

FORMANDO UN CUADRADO PERFECTO

3. Hallar el término que se debe sumar a las siguientes expresiones para transformarlas en un cuadrado perfecto.

a) x' - 2x. Sumar [t(coeficiente de x)]' = [j.( - 2)]' = l. Comprobocion : x' - 2x + 1 = (x _ 1)2

b) x' + 4x.

c) ,,' + 2".4

Sumar [t(coeficiente de x)]' = [1(4)]'

1 5, 25Sumar [-(-)] = -- .

24 64Comprobación:

Comprobacion . x' + 4x + 4 = (x + 2)'.

5 25 5,,' + -u + - = (u + _)24648

= 4.

d¡ x' + px". Sumar [!(p)]' = p'l4. Comprobacion : x' + px' + p'/4 = (x' + p/2)2

ECUACIONES DE SEGUNDO GRADO CON UNA INCOGNITA 113

PROBLEMAS RESUELTOS

ECUACIONES CUADRATICAS PURAS

l. Resolver.

a) x' - 16 = O. Luego x' = 16, x = ±4.

b) 41' - 9 = O. De donde 41' = 9, l' = 9/4, 1 = ± 3/2.

1 e) 3 - x' = 2x' + 1. Luego 3x' = 2, x' = 2/ 3, x = ±J2i3 = ±"3 fi.

3 d) 4x' + 9 = O. Luego x' = -9/4, x = ±J -9/4 = ±i

2x' - 1 17 e) ~ = x + 3 + x _ 3 . Tendremos2x' - 1 = (x + 3)(x - 3) + 17, 2x' - 1 = x' - 9 + 17,

x' = 9, Y x = ± 3.

Comprobación : Si x = 3 se sustituye en la ecuación dada, tendremos que dividir por cero, operación que no está definida. Luego x = 3 no es solución.

17 2(-W - 1 Si x = -3,

-3 - 3 - 3 + 3 + _ 3 _ 3 o sea 17

6

17

6 y x = - 3 es solución .

POR DESCOMPOSICION EN FACTORES

2. Resolver aplicando el método de la descomposición en factores.

a) x' + 5x - 6 = O. (x + 6)(x - 1) = O. x = -6, 1.

b) l' = 41. l' - 41 = O, 1(1 - 4) = O. 1 = 0,4.

c) x' + 3x = 28, x' + 3x - 28 = O. (x + 7)(x - 4) = O. x = -7. 4.

d) 5x - 2x' = 2, 2X2 - 5x + 2 = O, (2x-l)(x-2)=0. x= 1/2.2 .

1 1 5 el 1=1+ 1 - 4 =4' Multiplicando por 4(1 - 1)(1 - 4).

4(1 - 4) + 4(1 ~ 1) = 5(1 - 1)(1 - 41.

n y 2p

3p

6y -Si" 61" - 5py - 6p2 = O.

FORMANDO UN CUA DRADO PERFECTO

512 - 331 + 40 = O. (1 - 5)(51 - 8) = O,

(~r + 2p)(2y - 3p) = O. y = - 2p/3. 3p/2.

1 = 5. 8/5.

3. Hallar el término que se debe sumar a las siguientes expresiones para transformarlas en un cuadrado perfecto.

al x' - 2x. Sumar U(coeficiente de xl]2 = [1.( - 2)]2 = l. Comprohación: X2 - 2x + 1 = (x - 1)2

b) X2 + 4x.

5 cl u2 + 4u.

Sumar [!(coeficiente de x)]' = [1(41]2 = 4. Comprohación: X2 + 4x + 4 = (x + 2)2.

1 5 2 25 Sumar [-(- )] = -- .

24 64

d) x4 + px2 Sumar [!(p)]' = p2 /4.

Comprobación: 5 25 5

u 2 + - u + _ = (u + _ )2 4648

Comprohación: x' + px' + p2 !4 = (x 2 + p :" 212

http://carlos2524.jimdo.com/

Page 122: Algebra Superior Murray R Spiegel

114 ECUACIONES DE SEGUNDO GRADO CON UNA INCOGNITA

4. Resolver formando un cuadrado.

a) x2 _ 6x + 8 = O. Tenemos x2 - 6x = -8. x2- 6x + 9 = -8 + 9. (x - 3)2 = 1.

Luego .\' - 3 = ± l. x = 3 ± 1, las raíces son x = 4 Y x = 2.

Comprobacion : Para x = 4. 42 - 6(4) + 8 = O. 0=0. Para x = 2, 22 - 6(2) + 8 = O, 0=0.

h) 12 = 4 - 31.2 2 3 2 3 3 2 25

Tenemos 1 + 31 = 4. 1 + 31 + (z) = 4 + (Z)2, (r + Z) = 4'

3 5 3 5Luego 1 + Z = ±z' 1 = -z ± z· las raíces son 1 = 1. -4.

e) 3x2 + 8x + 5 = O.8 5 8 4 5 42 4 I

Tenemos x2 + -x = -- x2 + -x + (_)2 = --3 + (-3)' (x + -3)2= -9'.3' 3" 3 3

4 I 4 ILuego w + 3 = ±3' .v = -3 ± 3' las raíces son x = -1, -5/3.

d) x2 + 4.\' + I = O. Tenemos x2 + 4x = -1, x2 + 4x + 4 = 3, (x + 2)2 = 3.

Luegc x + 2 = ±)3. las raíces son x = -2 ± )3.

Comprohación: Para x = -2 + )3. (-2 + )3)2 + 4(-2 +)3) + I = (4 - 4)3 + 3) - 8 + 4)3 + 1 = O.

Para x = -2 -)3, (-2 - )3)2 + 4(-2 -)3)+ 1 = (4 + 4)3 + 3) - 8 - 4)3 + 1 = O.

2 2 6x 3 3 2 3 2 16e) 5x -6x+5=0. Tenemos5x -6x=-5, x2-'5+(51'=-I+(5)' (x-S) =-25

Luego x - 3/5 = ±) -16/25,3 4

las raíces son x = 5 ± Si.

S. Resolver la ecuación ax2 + bx + c = O, a -+ O, por el método de formar un cuadrado.

. . . . 2 b eDIvidiendo los dos miembros por a, x + -x + - = O o seaa a

2 bx + -x =a a

e

1 b b2 b b2

Sumando [-(-)y = -, a los dos miembros, x2 + -x + -, =2a 4a a 4a

b b2 - 4ac b ) b2- 4ac

Luego (x + _)2 = X + - = + 2a ,y x =2a 4a2 2a

e b2 b2 - 4ac--; + 4a2 = --¡r .-b ± )b2

- 4ac2a

APLICANDO LA FORMULA GENERAL

6. Resolver aplicando la fórmula general.

a) x2 - 3x + 2 = O. En este caso a = 1, h = - 3, e = 2. Luego

. -b + )b2 - 4ac -(-3) ± )(-31' - 4(1)(2) 3 ± 1x = = = -- o sea x = 1, 2.2a 2(1) 2

h) 412 + 121 + 9 = O. Tenemos a = 4, b = 12. e = 9. Luego

-12 :t )(12)2 - 4(4)(9) -12 ± O 3 31 = 2(4) = --8-- = -2 y 1 = -'2 es una raíz doble.

e) 9x2 + 18.\' - 17 = O. Tenemos a = 9, b = 18. c = -17. Luego

-18 ± )(18)2 - 4(9)(-17) -18 + j936 -18 ± 6j26x=--=--"--'---"-:-::--':"":':"'---"-:2(9) 18 18

-3 ± j263

GR,

7.

114 ECUACIONES DE SEGUNDO GRADO CON UNA INCOGN ITA

4. Resolver formando un cuadrado.

a) X2 - 6x + 8 = O. Tenemos X2 - 6x = -8, X2 - 6x + 9 = -8 + 9, (x - 3)2 = 1.

Luego .\: - 3 = ± 1, .\: = 3 ± 1, las raíces son .\: = 4 Y x = 2.

Comprohación: Para x = 4, 42 - 6(4) + 8 = O, 0=0. Para x = 2, 22 - 6(2) + 8 = O, 0=0.

h) / 2 = 4 - 31. Tenemos /2 + 3/ = 4, /2 + 3/ + (~)2 = 4 + (~)l, (1 + ~)2 = ~.

3 5 3 5 Luego / + 2 = ±2' / = -2 ± 2' las raíces son / = 1. -4.

e) 3X2 + 8x + 5 = O. 2 8 5 2 8 4 2 5 4 2 4 2 1

Tenemos x + 3x = - 3' x + 3x + (3) = -3 + (3)' (x + 3) ="9'

4 1 4 1 Luego.\: + 3 = ±3 ' .\: = -3 ± 3' las raíces son x = - 1, -5/3.

d) X2 + 4.\: + 1 = O. Tenemosx2 +4x= -1 , x 2 +4x+4=3, (X+2)2=3.

Luego .\: + 2 = ± fi, las raíces son x = - 2 ± fi ·

Comprohación: Para.\:= - 2+fi, (-2+fi)2+4(-2+fi)+I=(4-4fi+3)-8+4fi+1=0.

Para.\: = - 2 - fi, (-2 - fi)2 + 4(-2 - fi)+ 1 = (4 + 4fi + 3) - 8 - 4fi + 1 = O.

e) 5X2 - 6x + 5 = O. 2 6x 3 2 3 2 3

Tenemos 5x - 6x = -5, X2 - 5 + (s) = -1 + (S), (x - S)2 =

Luego .\: - 3/5 = ±J -16/25 , . 3 4

las ralces son x = S ± Si.

5. Resolver la ecuacíón ax2 + bx + c = O, a # O, por el método de formar un cuadrado.

Dividiendo los dos miembros por a, 2 b e

x + - x + - = O a a

o sea b

X2 + - x = a

e

a

1 h b2 b b2 e b2 b2 - 4ae Sumando [- (- ))' = ----, a los dos miembros, X2 + -x + - 2 = -- + - = - --

2 a 4a a 4a a 40 2 4a2 '

b b2 - 4ac Luego (x + _ )2 =

b ~ -b ± Jb 2 - 4ac

x+Zc;= + 2a ,y x= 2a 2a 4a 2

APLICANDO LA FORMULA GENERAL

6. Resolver aplicando la fórmula general.

a) X2 - 3x + 2 = O. En este caso a = 1, h = - 3, e = 2. Luego

-b ± J b2 - 4(1(' -(-3) ± J(- 3)2 - 4(1)(2) 3 ± 1 x = = = -- o sea x = 1, 2.

2a 2(\) 2

b) 4/2 + 12/ + 9 = O. Tenemos a = 4, b = 12, e = 9. Luego

- 12 :t J (12)2 - 4(4)(9) -12 ± O 3 3 . / = -2 es una ralz doble. / = 2(4) = --8- = - 2 y

e) 9X2 + 18.\: - 17 = O. Tenemos a = 9, h = 18. c = - 17. Luego

-18 ± J(18)2 - 4(9)( - 17) -18 ± j936 - 18 ± 6fi x=--=-..!.......----'---'.-----'

2(9) 18 18 -3 ±fi

3

16

25

http://carlos2524.jimdo.com/

Page 123: Algebra Superior Murray R Spiegel

= o.

=0.

=0.

EClJACIONES DE SEGUNDO GRADO CON UNA INCOGNITA 115

d) 611(2 - 11)= 7. Luego 611' - 1211+ 7 = O Y

-(-12) ± Ji-~2)2'=- 4(6)(7) 12 ± ¡-=--i;¡ 12 ± 2fii11 = --_. = = ----- =

2(6) 12 12

GRAFICAMENTE

+ fi i.- 6

h) 4x' - 12x + 9 = O.7. Resolver gráficamente: a) 2x' + 3x - 5 = O. e) 4x' - 4x + 5 = O.

La gráfica de)' = 2x' + 3x - 5 indica que cuando y = O.x = 1, Y - 2.5. Luego las raíces de 2x' + 3x - 5 = Oson x = 1, -2,5.

La gráfica de}' = 4x' - 12x + 9 es tangente al eje x en x = 1.5. es decir, cuando y = O, x = 1,5.

Luego 4x' - 12x + 9 = O tiene dos raíces iguales x = 1.5.

La gráfica de y = 4x' - 4x + 5 no corta al eje .v, es decir, no hay un valor real de x para el cual y = O.

Luego las raíces de 4x' - 4x + 5 = O son imaginarias.

1(Aplicando la fórmula general, las raíces son x = "2 ± i.)

y

-8

Raíces reales distintas(a)

a) y = 2x' + 3x - 5

h) Y = 4x' - 12x + 9

e) y = 4x' - 4x + 5

y y

-~+-+-+-~~~-x-~ -2 -1 1 2 1 2-~ -2 -1

Raíces reales iguales(h)

Raíces imaginarias(e)

x -3 -2 -1 O 1 12 Iy 4 -3 -6 -5 O 19 I

x -1 O 1 2 3 4

Y 25 9 1 1 9 25

x -2 -1 O 1 2 3-Y 29 13 5 5 13 29

http://carlos2524.jimdo.com/

Page 124: Algebra Superior Murray R Spiegel

116 ECUACIONES DE SEGUNDO GRADO CON UNA INCOGNITA

SUMA Y PRODUCTO DE RAICES

8. Demostrar que la suma S y el producto P de las raíces de la ecuación cuadrática ax? + bx + e = O sonb e

S = -- y P = -.a a

-b + Jh2- 4ac

Aplicando la fórmula general. las raíces son y2a

-b - Jh2 - 4ac2a

-2hLa suma de las raíces es S = -- =

2ab

El producto de las raíces esa

-h + Jh2 - 4ac -b - Jb2 - 4ac (_h)2 - (b2 - 4ae) eP=( )( )= =-.

~ ~ ~2 a

9. Sin resolver las ecuaciones. hallar la suma S y el producto P de las raíces.

a) x2 - 7x + 6 = O.b e

Aquí a = l. b = -7. e = 6; luego S = -- = 7 P = - = 6a ' a .6 -3

En este caso a = 2. b = 6, e = -3; luego S = -- = -3. P = -.2 21 5

Ordenando 3x2 + .v + 5 = O. Luego S = -"3' P = 3" .5

Tenemos a = 3, b = -5. e = O; luego S = 3"' P = O.

3Tenemos a = 2. b = O, e = 3; luego S = O. P = "2 .

m2 + n2 mnLuego S = ----. P = - = 1.

mn mn-0.01 I 4 40

Luego S = --- = -. P = - = -.0,3 30 0.3 3

b) 2X2 + 6x - 3 = O.

e) x + 3X2 + 5 = O.

d) 3.\'2 - 5x = O.

e) 2x2 + 3 = O.

f) mnx? + (m2 + ,,2).\ + mn = O.

g) 0.3x2 - O.Ol.v + 4 = O.

10. Hallar el discriminante b2 - 4ac de las ecuaciones siguientes y determinar el carácter de sus raíces.

a) x2 - 8x + 12 = O.

b) 3y2 + 2y - 4 = O.

e) 2X2 - x + 4 = O.

d) 4~2 - 12~ + 9 = O.

b2 - 4ac = (_8)2 - 4(1)(12) = 16; las raíces son reales. racionales. distintas.

h2 - 4ae = 52; las raíces son reales. irracionales. distintas.

h2 - 4ac = - 31; las raíces son imaginarias conjugadas.

h2 - 4ac = O; las raíces son reales. racionales, iguales.

e) 2x - 4x2 = 1 o 4x2 - 2x + 1 = O. h2 - 4ac = - 12; las raíces son imaginarias conjugadas.

f) )2 X2 - 4)3 x + 4)2 = O. Los coeficientes son reales pero no racionales.

h2 - 4ac = 16; las raíces son reales y distintas.

11. Hallar la ecuación cuadrática de coeficientes enteros cuyas raíces son las indicadas.(S = suma de raíces, P = producto de raíces.)

a) l. 2

Método l. S= 1 +2=3. P=2; luegox2-3x+2=0.

Método 2. (x - 1) Y (x - 2) deben ser factores de la expresión cuadrática.

Luego (x I)(x - 2) = O o sea x2 - 3x + 2 = O.

b) -3. 2

e)

d)

e)

n, Ha

al

b)

e)

d)

y,

el

de

13. H

a)

116 ECUACIONES DE SEGUNDO GRADO CON UNA INCOGNITA

SUMA Y PRODUCTO DE RAICES

8. Demostrar que la suma S y el producto P de las raíces de la ecuación cuadrática ax' + bx + c = O son b e

S = -- y P = -. a a

- b + J h' - 4ac - b - J h' - 4ae Aplicando la fórmula general. las raíces son y

2a 2a

-2b La suma de las raíces es S = -- =

2a

b

a El producto de las raíces es

-b + Jh' - 4ae -h - Jb' - 4ac (-h)' - (b' - 4ae) e P={ )( )= = - .

~ ~ ~'a

9. Sin resolver las ecuaciones. hallar la suma S y el producto P de las raíces.

a) x' - 7x + 6 = O.

b) 2x' + 6x - 3 = O.

e) x + 3x' + 5 = O.

d) 3x' - 5x = O.

e) 2x' + 3 = O.

b e Aquí a = 1. b = -7. e = 6; luego S = - - = 7 P = - = 6 a . a .

6 -3 En este caso a = 2. b = 6, e = -3 ; luego S = - - = -3. P = - .

2 2 I 5

Ordenando 3x' + x + 5 = O. Luego S = -"3' P = 3"' 5

Tenemos a = 3, b = -5. e = O; luego S = 3"' P = O.

3 Tenemos a = 2. b = O, e = 3; luego S = O. P = "2 .

f) mnx' + (m' + ,,')x + mn = O. m' + n' mn

Luego S = ----. P = - = 1. IJln mn

g) 0.3x' - O,Olx + 4 = O. -0.01 I 4 40

Luego S = - - - = -. p = - = - . 0,3 30 0.3 3

10. Hallar el discriminante b' - 4ac de las ecuaciones siguientes y determinar el carácter de sus raíces.

a) x' - 8x + 12 = O.

b) 3y' + 2y - 4 = O.

e) 2x' - x + 4 = O.

d) 4~' - 12~ + 9 = O.

b' - 4ac = (_ 8)' - 4(1)(12) = 16; las raíces son reales. racionales, distintas.

b' - 4ac = 52; las raíces son reales. irracionales. distintas.

h' - 4ac = - 31 ; las raíces son imaginarias conjugadas.

b' - 4ac = O; las raíces son reales. racionales. iguales.

e) 2x - 4x' = 1 o 4 .. ' - 2x + 1 = O. b' - 4ae = -12 ; las raíces son imaginarias conjugadas.

f) )2 x' - 4)3 x + 4)2 = O. Los coeficientes son reales pero no racionales.

b' - 4ae = 16; las raíces son reales y distintas.

11. Hallar la ecuación cuadrática de coeficientes ente ros cuyas raíces son las indicadas. (5 = suma de raíces, P = producto de raíces.)

a) 1, 2

Método l. S = 1 + 2 = 3. P = 2; luego x ' - 3x + 2 = O.

Método 2. (x - 1) Y (x - 2) deben ser factores de la expresión cuadrática.

Luego (x I)(x - 2) = O o sea x' - 3x + 2 = O.

b) - 3. 2

http://carlos2524.jimdo.com/

Page 125: Algebra Superior Murray R Spiegel

ECUACIONES DE SEGUNDO GRADO CON UNA INCOGNITA

Método l.

Método 2.

s = -1, P = -6; luego x2 + x - 6 = O.

[x - (-3)] Y (x - 2) son factores de la expresión cuadrática.

Luego (x + 3)(x - 2) = O o sea x2 + x - 6 = O.

S 11 P 4. luego X2_.!..!.X-~=0 o sea 15x2-11x-12=0.= 15' = -5' 15 54

e) 3"'35

d) 2+.j2,2-.j2

Método l. S = 4, P = (2 + .j2)(2 - .j2) = 2; luego x2 - 4x + 2 = O.

Método 2. [x - (2 + .j2)] y [x - (2 - .j2)] son factores de la expresión cuadrática.

Luego [x - (2 + .j2)][x - (2 - .j2)] = [(x - 2) - .j2][(x - 2) + .j2] = O,

(x - 2)2 - 2 = O o sea x2 - 4x + 2 = O.

Método 3. Como x = 2 ± .j2, x - 2 = ±.j2. Elevando al cuadrado (x - 2)2= 2 o x2 - 4x + 2 = O.

e) -3 + 2i, -3 - 2i

Método l. S = -6, P = (-3 + 2i)(-3 - 2i) = 13; luego x2 + 6x + 13= O.

Método 2. [x - (- 3 + 2i)] Y [x - (- 3 - 2i)] son factores de la expresión cuadrática.

Luego [(x + 3) - 2i][(x + 3) + 2i] = O, (x + W + 4 = O o bien x2 + 6x + 13~ O.

12. Hallar el valor de la constante p en las ecuaciones siguientes para que se satisfaga la condición que se indica.

a) 2X2 - px + 4 = O tenga una raíz igual a -3.

Como x = - 3 es una raíz, debe satisfacer a la ecuación dada.

Luego 2(-W - p(-3) + 4 = O Y P = -22/3.

b) (p + 2)x2 + 5x + 2p = O el producto de sus raíces sea igual a 2/3.2p 2p 2

El producto de las raíces es --2; luego --2 = -3 Y P = 1.p+ p+

e) 2px2 + px + 2x = x2 + 7p + 1 la suma de sus raíces sea igual a -4/3.

Escribiendo la ecuación en la forma (2p - l)x2 + (p + 2)x - (7p + 1) = O.p + 2 4

La suma de las raíces es --- = -- y p = 2.2p - 1 3

d) 3x2 + (p + l)x + 24 = O una de las raíces sea el doble de la otra. Sean las raíces r, 2r.

El producto de las raíces es r(2r) = 8; luego r2 = 4 Y r = ±2.p + 1 .

La suma de las raíces es 3r = - -3- .Sustituyendo r = 2 y r = - 2 en esta ecuación se obtiene p = - 19

Y P = 17, respectivamente.

e) 2X2 - 12x + p + 2 = O la diferencia entre sus raíces sea igual a 2.

Sean las raíces r, s; tendremos (1) r - s = 2. La suma de las raíces es 6; luego (2) r + s = 6. La solucióndel sistema formado por (1) Y (2) es r = 4, s = 2.

Sustituyendo x = 2 o x = 4 en la ecuación dada, se obtiene p = 1'+.

13. Hallar las raíces de las ecuaciones cuadráticas siguientes de forma que se cumpla la condición que se indica.

a) (2k + 2)x2+ (4 - 4k)x + k - 2 = O una de las raíces sea el recíproco de la otra.

117ECUACIONES DE SEGUNDO GRADO CON UNA INCOGNITA

Método 1. S = -1 , P = -6; luego X2 + x - 6 = O.

Método 2. [x - (- 3)] Y (x - 2) son factores de la expresión cuadrática.

Luego (x + 3)(x - 2) = O o sea X2 + x - 6 = O.

S 11 P 4 luego X2 _ .!..!.X _ ~ = O o sea 15x2 - 11x - 12 = O. = 15' = 5' 15 5 4

e) 3' 3

5

d) 2+.Ji,2-.Ji

Método 1. S = 4, P = (2 + .Ji)(2 - .Ji) = 2; luego X2 - 4x + 2 = O.

Método 2. [x - (2 + .Ji)] y [x - (2 - .Ji)] son factores de la expresión cuadrática.

Luego [x - (2 + .Ji)][x - (2 - .Ji)] = [(x - 2) - .Ji][(x - 2) + .Ji] = O,

(x - 2)2 - 2 = O o sea X2 - 4x + 2 = O.

117

Método 3. Como x = 2 ± .Ji, x - 2 = ± .Ji. Elevando al cuadrado (x - 2)2 = 2 o X2 - 4x + 2 = O.

e) -3 + 2i, -3 - 2i

Método 1. S = -6, P = (-3 + 2i)(-3 - 2i) = 13; luego X2 + 6x + 13 = O.

Método 2. [x - (- 3 + 2i)] Y [x - (- 3 - 2i)] son factores de la expresión cuadrática.

Luego [(x + 3) - 2i][(x + 3) + 2i] = O, (x + W + 4 = O o bien X2 + 6x + 13 ~ O.

12. Hallar el valor de la constante p en las ecuaciones siguientes para que se satisfaga la condición que se indica.

a) 2X2 - px + 4 = O tenga una raíz igual a -3 . .

Como x = - 3 es una raíz, debe satisfacer a la ecuación dada.

Luego 2(-W - p(-3) + 4 = O Y P = -22/3.

b) (p + 2)x2 + 5x + 2p = O el producto de sus raíces sea igual a 2/3. 2p 2p 2

El producto de las raíces es --2; luego - -2 = -3 Y P = 1. p+ p+

e) 2px2 + px + 2x = X2 + 7p + 1 la suma de sus raíces sea igual a -4/3.

Escribiendo la ecuación en la forma (2p - l)x2 + (p + 2)x - (7p + 1) = O. p + 2 4

La suma de las raíces es - -- = - - y p = 2. 2p - 1 3

d) 3X2 + (p + I)x + 24 = O una de las raíces sea el doble de la otra. Sean las raíces " 2,.

El producto de las raíces es ,(2,) = 8; luego ,2 = 4 Y , = ±2. p + 1 .

La suma de las raíces es 3, = - -3- . Susl1tuyendo, = 2 y, = - 2 en esta ecuación se obtiene p = - 19

Y P = 17, respectivamente.

e) 2X2 - 12x + p + 2 = O la diferencia entre sus raíces sea igual a 2.

Sean las raíces " s; tendremos (1) , - s = 2. La suma de las raíces es 6; luego (2) , + s = 6. La solución del sistema formado por (1) y (2) es , = 4, s = 2.

Sustituyendo x = 2 o x = 4 en la ecuación dada , se obtiene p = 1,+.

13. Hallar las raíces de las ecuaciones cuadráticas siguientes de forma que se cumpla la condición que se indica.

a) (2k + 2)xl + (4 - 4k)x + k - 2 = O una de las raíces sea el recíproco de la otra.

http://carlos2524.jimdo.com/

Page 126: Algebra Superior Murray R Spiegel

118 ECUACIONES DE SEGUNDO GRADO CON UNA INCOGNITA

Sean r y I/r las raíces, su producto será 1.k - 2

El producto de las raíces es --- = 1 de donde k = -4.2k + 2

Hacemos k = -4 en la ecuación dada; con 10 cual 3x2 - IOx + 3 = O Y las raíces son 1/3, 3.

b) kX2 - (1 + k)x + 3k + 2 = O la suma de sus raíces sea igual al doble de su producto.

1 + k 3k + 2 3Suma de raíces = 2(producto de raíces); tendremos -k- = 2(--k-) Y k = -5·

Sustituimos k = -3/5 en la ecuación dada; con 10 cual 3x2 + 2x - 1 = O Y las raíces son -1, 1/3.

e) (x + k)2 = 2 - 3k tenga raíces iguales.

Haciendo operaciones x2 + 2kx + (k2 + 3k - 2) = O siendo a = 1, b = 2k, e = k2 + 3k - 2. Las raí-ces son iguales si el discriminante (b2 - 4ae) = O.

Luego de b2 - 4ae = (2k)2 - 4(I)(k2 + 3k - 2) = O se obtiene k = 2/3.

Sustituyendo k = 2/'3 en la ecuación dada y resolviendo, se obtiene la raíz doble - 2/3.

ECUACIONES IRRACIONALES

14. Resolver.

a) ~ = 3. Elevando al cuadrado los dos miembros, 2x + 1 = 9 y x = 4.

Comprobación: J2(4l+l = 3, 3 = 3.

b) J5+2x = x + 1. Elevando al cuadrado los dos miembros, 5 + 2x = x2 + 2x + 1, x2 = 4 Y x = ±2.

Comprobación: Para x = 2, J5+2(2) = 2 + 1 o sea 3 = 3.

Para x = -2, J5 + 2(-2) = -2 + 1 o sea ji = -1 que no es cierto ya que ji = 1.

Así, pues, x = 2 es la única solución; x = - 2 es una raíz extraña.

e) ~ = x - 1. Elevando al cuadrado, 3x - 5 = x2 - 2x + 1, x2 - 5x + 6 = O y x = 3, 2.

Comprobación: Para x = 3, J3(3)=5 = 3 - 1 o sea 2 = 2. Para x = 2, J3(2) - 5 = 2 - 1 o sea 1 = 1.

Así, pues, x = 3 y x = 2 son soluciones de la ecuación dada.

d) 1X2 - X + 6 - 2 = O. Tendremos 1X2 - x + 6 = 2, x2 - X + 6 = 8, x2 - X - 2 = O y x = 2, -1.

Comprobación: Para x = 2, 122 - 2 + 6 - 2 = O o sea 2 - 2 = O.

Para x = -1, 1 (_1)2 - (-1) + 6 - 2 = O o sea 2 - 2 = O.

15. Resolver.

a) Jh+! - Jx = 1. Transponiendo términos, (1) ~ = Jx + 1.

Elevando al cuadrado los dos miembros de (1), 2x + 1 = x + 2Jx + 1 o sea (2) x = 2Jx.

Elevando al cuadrado (2), x2 = 4x; luego x(x - 4) = O y x = 0,4.

Comprobación: Para x = O, .J2(O)+1- JO-= 1, 1 = 1. Para x = 4, J2(4l+l - .J4 = 1, 1 = 1.

b) fo-=! + J2x + 3 = 1. Transponiendo términos, (1) Fx-=t = 1 - fo+3.

Elevando al cuadrado (1), 4x - 1 = 1 - 2f0+3 + 2x + 3 o sea (2) 2f0+3 = 5 - 2x.

Elevando al cuadrado (2), 4(2x + 3) = 25 - 20x + 4x2, 4x2 - 28x + 13 = O y x = 1/2, 13/2.

e) ,

16. Resc

a)

b)

17. Resc

mien

18, Reso

y x

ECUACII

19. Resc

a)

118 ECUACIONES DE SEGUNDO GRADO CON UNA INCOGNITA

Sean r y l l r las raíces, su producto será l. k - 2

El producto de las raíces es --- = 1, de donde k = -4. 2k + 2

Hacemos k = -4 en la ecuación dada ; con lo cual 3X2 - IOx + 3 = O Y las raíces son 1/3, 3.

b) kX2 - (1 + k)x + 3k + 2 = O la suma de sus raíces sea igual al doble de su producto.

1 + k 3k + 2 Suma de raíces = 2(producto de raíces); tendremos - k- = 2(--k-) Y k =

3

5

Sustituimos k = -3/5 en la ecuación dada; con lo cual 3X2 + 2x - 1 = O Y las raíces son -1, 1/3.

e) (x + k)2 = 2 - 3k tenga raíces iguales.

Haciendo operaciones X2 + 2kx + (k2 + 3k - 2) = O siendo a = 1, b = 2k, e = k 2 + 3k - 2. Las raí­ces son iguales si el discriminante (b 2 - 4ac) = O.

Luego de b2 - 4ac = (2k)2 - 4(I)(k2 + 3k - 2) = O se obtiene k = 2/3.

Sustituyendo k = 2/3 en la ecuación dada y resolviendo, se obtiene la raíz doble -2/3.

ECUACIONES IRRACIONALES

14. Resolver.

a) fo+l = 3. Elevando al cuadrado los dos miembros, 2x + 1 = 9 y x = 4.

Comprobación: J2(4f+I = 3, 3 = 3.

b) ~ = x + 1. Elevando al cuadrado los dos miembros, 5 + 2x = X2 + 2.x + 1, X2 = 4 y x = ±2.

Comprobación: Para x = 2, J5+2(2) = 2 + 1 o sea 3 = 3.

Para x = -2, J5 + 2( -2) = -2 + 1 o sea .JI = -1 quenoesciertoyaque.JI = 1.

Así, pues, x = 2 es la única solución; x = - 2 es una raíz extraña.

e) fo-=5 = x - 1. Elevando al cuadrado, 3x - 5 = X2 - 2x + 1, X2 - 5x + 6 = O y x = 3, 2.

Comprobación: Para x = 3, J3i3)=5 = 3 - 1 o sea 2 = 2. Para x = 2, J3(2)=5 = 2 - 1 o sea 1 = 1.

Así, pues, x = 3 y x = 2 son soluciones de la ecuación dada.

d) ';;X2 - X + 6 - 2 = O. Tendremos ';;X2 - x + 6 = 2, X2 - X + 6 = 8, X2 - X - 2 = O Y x = 2, -1.

Comprobación: Para x = 2, ';;22 - 2 + 6 - 2 = O o sea 2 - 2 = O.

Para x = - 1, .;; ( - 1 ¡z - (- 1) + 6 - 2 = O o sea 2 - 2 = O.

15. Resolver.

a) j27'+1 - Jx = l. Transponiendo términos, (1) fo+l = Jx + 1.

Elevando al cuadrado los dos miembros de (1), 2x + 1 = x + 2Jx + 1 o sea (2) x = 2.Jx. Elevando al cuadrado (2), X2 = 4x; luego x(x - 4) = O Y x = 0,4.

Comprobación: Para x = O, J2(Of+l - J O- = 1, 1 = 1. Para x = 4, J2(4)+1 - J4 = 1, 1.

b) ¡.¡:;-=I + J 2x + 3 = 1. Transponiendo términos, (1) Fx'=l = 1 - Ja + 3.

Elevando al cuadrado (1), 4x - 1 = 1 - 2~ + 2x + 3 o sea (2) 2~ = 5 - 2x.

Elevando al cuadrado (2), 4(2x + 3) = 25 - 20x + 4X2, 4X2 - 28x + 13 = O Y x = 1/2, 13/2.

http://carlos2524.jimdo.com/

Page 127: Algebra Superior Murray R Spiegel

raí-

±2.

= 1.

=1.

-1.

1.

ECUACIONES DE SEGUNDO GRADO CON UNA INCOGNITA 119

Comprobación: Para x = 1/2, J4(1/2) - 1 + J2(1/2) + 3 = 1 o sea 3 = 1 que no es cierto.

Para x = 13/2, J4(13/2) - 1 + J2(13/2) + 3 = 1 o sea 9 = 1 que no es cierto.

Luego x = 1/2 Y x = 13/2 son raíces extrañas; la ecuación no tiene solución.

e) .J Fx+16 - Jx = 2. Elevando al cuadrado, Fx+16 - Jx = 4 o sea (1) Fx+16 = fi + 4.

Elevando al cuadrado (1), x + 16 = x + 8fi + 16, 8Jx = O, Y x = O es una solución.

16. Resolver.

a) JX2 + 6x = x + fo.Elevando al cuadrado, x2 + 6x = x2 + 2xfo + 2x, 2xfo = 4x, x(fo - 2) = O.

Luego x = O; Y de fo - 2 = O, fo = 2, 2x = 4, x = 2.

Tanto x = O Y x = 2 satisfacen a las ecuaciones dadas.

2b) Jx - fi = 1. Multiplicando por Jx se obtiene (1) x - 2 = fi·

Elevando al cuadrado (1), x2 - 4x + 4 = x, x2 - 5x + 4 = O, (x - l)(x - 4) = O. y x = 1, 4.

Solamente x = 4 satisface a la ecuación dada; x = 1 es extraña.

17. Resolver la ecuación x2 - 6x - J x2 - 6x - 3 = 5.

Hacemos x2 - 6x = u; luego u - ~ = 5 o sea (1) ~ = u - 5.

Elevando al cuadrado (1), u - 3 = u2- 10u + 25, u2

- Ilu + 28 = O. y u = 7, 4.

Como solamente u = 7 satisface (1), sustituyendo u = 7 en x2 - 6x = u se obtiene

x2 - 6x - 7 = O, (x - 7)(x + 1) = O, Y x = 7, -1

Tanto x = 7 y x = - 1 satisfacen a la ecuación dada y, por tanto, son soluciones.

Nota. Si hubiésemos puesto Jx2 - 6x - 3 = x2 - 6x - 5 y hubiéramos elevado al cuadrado los dosmiembros. habríamos llegado a una ecuación de cuarto grado difícil de resolver.

4 - x 318. Resolver la ecuación ,=:=====

JX2 - 8x + 32 5

16 - 8x + x2 9Elevando al cuadrado. 2 • luego 25(16 - 8x + X2) = 9(x2 - 8x + 32). x2 - 8x + 7 = O.

x - 8x + 32 25y x = 7, 1. La única solución es x =' 1; se rechaza x = 7, solución extraña.

ECUACIONES DE TIPO CUADRATICO

19. Resolver.

a) x' - IOx2 + 9 = O. Hacemos x2 = u; tendremos u2- 10u + 9 = O y u = 1, 9.

Para u = 1, x2 = 1 Y x = ± 1; para u = 9. x2 = 9 y x = ± 3.

Las cuatro soluciones son x = ± l. ± 3; que satisfacen a la ecuación dada.

ECUACIONES DE SEGUNDO GRADO CON UNA INCOGNITA 119

Comprobación: Para x = 1/2, J 4(1 /2) - 1 + J 2(1 /2) + 3 = 1 o sea 3 = I que no es cierto.

Para x = 13/2, J 4(13/2) - 1 + J 2(13/2) + 3 = 1 o sea 9 = 1 que no es cierto.

Luego x = 1/2 Y x = 13/2 son raíces extrañas ; la ecuación no tiene solución.

e) .J Jx+16 - Jx = 2. Elevando al cuadrado, Jx+16 - Jx = 4 o sea (1) Jx+16 = fi + 4.

Elevando al cuadrado (1), x + 16 = x + 8fi + 16, 8Jx = O, Y x = O es una solución.

16. Resolver.

a) Jx2 +6x=x+fo.

Elevando al cuadrado, X2 + 6x = X2 + 2xfo + 2x, 2xfo = 4x, x(fo - 2) = O.

Luego x = O; Y de fo - 2 = O, fo = 2, 2x = 4, x = 2.

Tanto x = O Y x = 2 satisfacen a las ecuaciones dadas .

2 b) fi - Jx = 1. Multiplicando por fi se obtiene (1) x - 2 = p.

Elevando al cuadrado (1), X2 - 4x + 4 = x, X2 - 5x + 4 = O, (x - I)(x - 4) = O, Y x = 1, 4.

Solamente x = 4 satisface a la ecuación dada ; x = 1 es extraña.

17. Resolver la ecuación X2 - 6x - J X2 - 6x - 3 = 5.

Hacemos X2 - 6x = u ; luego u -~ = 5 o sea (1) ~ = u - 5.

Elevando al cuadrado (1), u - 3 = u2 - 10u + 25, u2

- Ilu + 28 = O. y u = 7, 4.

Como solamente u = 7 satisface (1), sustituyendo u = 7 en X2 - 6x = u se obtiene

X2 - 6x - 7 = O, (x - 7)(x + 1) = O, Y x = 7, -1

Tanto x = 7 Y x = - 1 satisfacen a la ecuación dada y, por tanto, son soluciones.

Nota. Si hubiésemos puesto J X2 - 6x - 3 = X2 - 6x - 5 y hubiéramos elevado al cuadrado los dos miembros, habríamos llegado a una ecuación de cuarto grado difícil de resolver.

4-x 18. Resolver la ecuación ,=7==:===:=

JX2 - 8x + 32

3

5

16 - 8x + X2 9 Elevando al cuad rado, 2 8 = -25; luego 25(16 - 8x + X2) = 9(x2

- 8x + 32), X2 - 8x + 7 = O. x - x + 32

y x = 7, 1. La única solución es x == 1; se rechaza x = 7, solución extraña.

ECUACIONES DE TIPO CUADRATICO

19. Resolver.

a) x 4 - IOx2 + 9 = O. Hacemos X2 = u ; tendremos u2 - 10u + 9 = O y u = 1, 9.

Para u = 1, X2 = 1 Y x = ± 1; para u '" 9, X2 = 9 y x '" ± 3.

Las cuatro soluciones son x = ± l. ± 3: que satisfacen a la ecuación dada.

http://carlos2524.jimdo.com/

Page 128: Algebra Superior Murray R Spiegel

120 ECUACIONES DE SEGUNDO GRADO CON UNA INCOGNITA

b) 2X4 + x2 - 1 = O. Hacemos x2 = u; tendremos 2u2 + u - 1 = O Y u = t, -1.

Si u = t, x2 = t y x = ±tj2; si u = -1, x2 = -1 Y x = ±i.Las cuatro soluciones son x = ±tj2, ±i.

e) Jx - fx - 2 = O. Hacemos fx = u; tendremos u2 - u - 2 = O Y u = 2, -1.

Si u = 2, fx = 2 Y x = 24 = 16. Como fx es positivo, no puede ser igual -1.

Luego x = 16 es la única solución de la ecuación dada.

1 1 1d) 2(x + _)2 - 7(x + -) + 5 = O. Hacemos x + - = u; tendremos 2u2 - 7u + 5 = O Y u = 5/2, 1.x x x

515Para u = -, x + - = -, 2X2 - 5x + 2 = O Y x = 2, t.2 x 2

1Para u = 1, x + - = 1, x2 - X + 1 = O Y x = t ± t.)3 i.

xLas cuatro soluciones son x = 2, t, !± t.)3 i.

e) 9(x + 2)-4 + 17(x + 2)-2 - 2 = O. Hacemos (x + 2)-2 = u; tendremos 9u2 + 17u - 2 = O Y u = 1/9, -2.

Si (x + 2)-2 = 1/9, (x + 2)2 = 9, (x + 2) = ±3 y x = 1, -5.

Si (x + 2)-2 = -2, (x + 2)2 = -t, (x + 2) = Hj2 i Y x = -2 ± tj2 t.

Las cuatro soluciones son x = 1, -5, -2 ± tj2 i,

20. Hallar los valores de x que satisfacen a las ecuaciones siguientes.

a) 16(_x_)4 - 25(_x_)2 + 9 = o. Hacemos (_X_)2 = u; tendremos 16u2 - 25u + 9 = O Y u = 1,9/16.x+1 x+1 x+1

Si u = 1, (_X_)2 = 1 o sea _x_ = + 1. La ecuación _x_ = 1 no tiene solución; la ecuación xx+1 x+1 - x+1 x+1

= -1 tiene solución x = - 1/2.

S. x 2 9 x 3 /1 U = 9/16, (--) = - o sea -- = ±-4 por tanto x = 3, -37.

x + 1 16 x + 1

Las soluciones son x = -1/2, - 3/7, 3.

b) (x2 + 3x + 2)2 - 8(x2 + 3x) = 4. Hacemos x2 + 3x = u; tendremos (u + 2)2 - 8u = 4 Y u = O, 4.

Si u = O, x2 + 3x = O Y x = O, -3; si u = 4, x2 + 3x = 4 Y x = -4, 1.

Las soluciones son x = -4, -3, O, 1.

PROBLEMAS DIVERSOS

21. Hallar 2 números positivos sabiendo que uno de ellos es igual al triple del otro más 5 y que el productode ambos es igual a 68.

Sea x = número menor; será 3x + 5 = número mayor.

Luego x(3x + 5) = 68, 3x2 + 5x - 68 = O, (3x + 17)(x - 4) = O, Y x = 4, -17/3.

Se rechaza - 17/3 ya que el enunciado establece que los números sean positivos.

Los números buscados son x = 4 Y 3x + 5 = 17.

22. H¡

23. H,

25. Lt<

a

b

120 ECUACIONES DE SEGUNDO GRADO CON UNA INCOGNITA

b) 2X4 + X2 - 1 = O. Hacemos X2 = u; tendremos 2u2 + u - 1 = O Y u = t, -1.

Si u = t, X2 = t y x = ±tJ2; si u = -1, X2 = -1 Y x = ±i. Las cuatro soluciones son x = ±tJ2, ±i.

e) Jx - .y-; - 2 = O. Hacemos .y-; = u; tendremos u2 - u - 2 = O Y u = 2, -1.

Si u = 2, .y-; = 2 Y x = 24 = 16. Como .y-; es positivo, no puede ser igual -1.

Luego x = 16 es la única solución de la ecuación dada.

1 1 1 d) 2(x +_)2 - 7(x + -) + 5 = O. Hacemos x + - = u; tendremos 2u2 - 7u + 5 = O Y u = 5/2,1. x x x

515 Para u = -, x + - = -, 2X2 - 5x + 2 = O Y x = 2, 1-

2 x 2 1

Para u = 1, x + - = 1, X2 - X + 1 = O Y x = t ± tJ3 i. x

Las cuatro soluciones son x = 2, l, l ± tJ3 i.

e) 9(x + 2)-4 + 17(x + 2)-2 - 2 = O. Hacemos (x + 2)-2 = u; tendremos 9u2 + 17u - 2 = O Y u = 1/9, -2.

Si (x + 2)-2 = 1/9, (x + 2)2 = 9, (x + 2) = ±3 y x = 1, -5.

Si (x + 2)-2 = -2, (x + 2)2 = -1, (x + 2) = ±tJ2i y x = - 2 ± tJ2i. Las cuatro soluciones son x = 1, -5, -2 ± tJ2 i.

20. Hallar los valores de x que satisfacen a las ecuaciones siguientes.

a) 16(_x_)4 - 25(_x_ )2 + 9 = O. Hacemos (_X_)2 = u; tendremos 16u2 - 25u + 9 = O Y u = 1,9/ 16. x+1 x+1 x+l

Si u = 1 (_X_)2 = 1 o sea _x_ = + l. La ecuación _x_ = l no tiene solución; la ecuación _x_ 'x +1 x+1 - x+1 x+1

= - 1 tiene solución x = - 1/2.

S· 6 x 2 9 x 3 / 1 U = 9/1 , (--) = - 6 o sea -- = ±-4 por tanto x = 3, - 3 7.

x + l l x + l

Las soluciones son x = -1/2, - 3/7, 3.

b) (x2 + 3x + 2)2 - 8(x2 + 3x) = 4. Hacemos X2 + 3x = u; tendremos (u + 2)2 - 8u = 4 Y u = O, 4.

Si u = O, X2 + 3x = O Y x = O, -3; si u = 4, X2 + 3x = 4 Y x = -4, 1.

Las soluciones son x = - 4, - 3, O, 1.

PROBLEMAS DIVERSOS

21. Hallar 2 números positivos sabiendo que uno de ellos es igual al triple del otro más 5 y que el producto de ambos es igual a 68.

Sea x = númerO menor; será 3x + 5 = número mayor.

Luego x(3x + 5) = 68, 3X2 + 5x - 68 = O, (3x + 17)(x - 4) = O, Y x = 4, -17/3.

Se rechaza - 17/3 ya que el enunciado establece que los números sean positivos.

Los números buscados son x = 4 Y 3x + 5 = 17.

http://carlos2524.jimdo.com/

Page 129: Algebra Superior Murray R Spiegel

-2.

116.

x

+ I

cto

ECUACIONES DE SEGUNDO GRADO CON UNA INCOGNITA

22. Hallar un número sabiendo que la suma del triplo del mismo con el doble de su recíproco es igual a 5.

Sea x = el número y l/x = su recíproco.

Tendremos 3x + 2(I/x) = 5, 3x2 - 5x + 2 = O, (3x - 2)(x - 1) = O, Y x = 1, 2/3.

Comprobación. Para x = 1,3(1) + 2(1/1) = 5; para x = 2/3,3(2/3) + 2(3/2) = 5.

23. Hallar las dimensiones de un rectángulo cuyo perímetro es 50 m y su área es 150 m".

Suma de los 4 lados = 50 m; luego suma de dos lados adyacentes = 25 m.

Sean x y 25 - x las longitudes de 2 lados adyacentes.

El área es x(25 - x) = 150; luego x2 - 25x + 150 = O, (x - lO)(x - 15) = O, y x = lO, 15.

Por tanto, 25 - x = 15, 10; y las dimensiones del rectángulo son 10 m por 15 m.

25-%15

%

D%!.

12

Problema 23 Problema 24 Problema 25

24. La hipotenusa de un triángulo rectángulo es igual a 34 cm. Hallar las longitudes de los catetos sabiendo que unode ellos es 14 cm mayor que el otro.

Sean x y x + 14 las longitudes de los catetos.

Tendremos x2 + (x + (4)' = (34)2, x2 + 14x - 480 = O. (x + 30)(x - 16) = O, y x = -30, 16.

Como x = - 30 no tiene sentido físico, tenemos x = 16 cm y x + 14 = 30 cm.

25. Las dimensiones exteriores de un marco de fotografía son 12 por 15 cm. Sabiendo que su ancho permanece cons-tante, hallar su valor a) cuando la superficie de la fotografía es de 88 cm", b¡ cuando dicha superficie vale 100 cm '.

Sea x = anchura del cuadro: las dimensiones del cuadro son (15 - 2x), (12 - 2x).

a) Area del cuadro = (15 - 2.':)(12 - 2x) = 88; luego 2X2 - 27x + 46 = O. (x - 2)(2x - 23) = O. y .v = 2.111. Es evidente que la anchura no puede ser II! cm. Luego la anchura del marco es 2 cm.

Comprohación. El área del cuadro es (15 - 4)( 12 - 4) = 88 cm".

h) En este caso (15 - 2x)(12 - 2x) = 100, 2.\"2 - 27x + 40 = O y. aplicando la fórmula general.

-b ± ~ 27 ± J4Ü9x = 2a = --4--- o sea x = l lB, 1,7 (aproximadamente).

Se rechaza x = 11,8 cm, que no puede ser la anchura. La anchura cs. pues. 1,7 cm.

121ECUACIONES DE SEGUNDO GRADO CON UNA INCOGN ITA

22. Hallar un número sabiendo que la suma del triplo del mismo con el doble de su recíproco es igual a 5.

Sea x = el número y l/x = su recíproco.

Tendremos 3x + 2(I /x) = 5, 3x' - 5x + 2 = O, (3x - 2)(x - 1) = O, Y x = 1, 2/ 3.

Comprobación. Para x = 1,3(1) + 2(1 / 1) = 5; para x = 2/ 3, 3(2/ 3) + 2(3/2) = 5.

23. Hallar las dimensiones de un rectángulo cuyo perímetro es 50 m y su área es 150 m'.

Suma de los 4 lados = 50 m; luego suma de dos lados adyacentes = 25 m.

Sean x y 25 - x las longi tudes de 2 lados adyacentes.

El área es x(25 - x) = 150; luego x' - 25x + 150 = O, (x - lO)(x - 15) = O, y x = lO, 15 .

Por tanto, 25 - x = 15, 10; y las dimensiones del rectángulo son 10 m por 15 m.

25-% • 15 %

D % ...

12

Problema 23 Problema 24 Problema 25

121

24. La hipotenusa de un triángulo rectángulo es igual a 34 cm. Hallar las longitudes de los catetos sabiendo que uno de ellos es 14 cm mayor que el otro.

Sean x y x + 14 las longitudes de los catetos.

Tendremos x' + (x + 14)' = (34)', x' + 14x - 480 = O, (x + 30)(x - (6) = O, y x = -30. 16.

Como x = -30 no tiene sentido físi co. tenemos x = 16 cm y x + 14 = 30 cm.

25. Las dimensiones exteriores de un marco de fotografía son 12 por 15 cm. Sabiendo que su ancho permanece cons­tante, hallar su valor a) cuando la superficie de la fotografía es de 88 cm', b ) cuando dicha superficie va le 100 cm'.

Sea x = anchura del cuadro: las dimensiones del cuadro son (15 - 2x), (12 - 2x).

a) Area del cuadro = (15 - 2x)( 12 - 2x) = 88 ; luego 2x' - 27x + 46 = O. (x - 2)(2.\" - 23) = O, y.\" = 2, li t. Es evidente que la anchura no puede ser li t cm. Luego la anchura del marco es 2 cm.

Comprohación. El área del cuadro es (15 - 4)(12 - 4) = 88 cm'.

b ) En este caso (15 - 2.\")(12 - 2x) = lOO, 2x' - 27.\" + 40 = O y. aplicando la fórmula general.

-b + l b' - 4ac 27 + J409 x = - v

2a = ~- o sea x = 11.8, 1,7 (aproximadamente).

Se rechaza .\" = 11 ,8 cm, que no puede ser la anchura. La anchura cs. pues. 1,7 cm.

http://carlos2524.jimdo.com/

Page 130: Algebra Superior Murray R Spiegel

122 ECUACIONES DE SEGUNDO GRADO CON UNA INCOGNITA

26. Un piloto realiza un vuelo de 600 km. Sabiendo que si aumenta la velocidad en 40 km/h podría recorrer dichadistancia empleando 30 minutos menos, hallar su velocidad.

distancia en kmTiempo en horas = velocidad en krn/h

Tiempo para volar 600 km a x krn/h - tiempo para volar 600 km a (x + 40) krn/h = t h.

Sea' x = velocidad media real en km/h.

600 600 1Luego - - --- = -. Resolviendo, se obtiene la velocidad x = 200 km/h.

x x+40 2

27. Un comerciante compra determinado número de lapiceros por 180 pts y los vende todos menos 6 con una ga-nancia de 2 pts en cada lapicero. Sabiendo que con el dinero recaudado en la venta podría haber comprado 30lapiceros más que antes, calcular el precio de cada lapicero.

Sea x = coste de un lapicero en pesetas; 180/x = número de lapiceros que compró.

180 180Luego (- - 6)(x + 2) = x(- + 30).

x xResolviendo x = 3 pts cada lapicero.

28. Dos operarios A y B, juntos, realizan una tarea en lO días. Trabajando por separado, A tardaría 5 días más que B.Hallar el número de días que tardarían en hacer la tarea trabajando cada uno por sí solo.

Sean n, n - 5 = número de días que tardarían A y B, respectivamente, en efectuar la tarea trabajando indi-vidualmente.

En 1 día, A realiza I/n y B hace I/(n - 5) de la tarea. Por tanto, en \O días trabajando juntos,

1 110(- + --) = 1 trabajo completo

n n - 5

25 + ,)625 - 200Luego 10(2n - 5) = n(n - 5), n2

- 25n + 50 = O, Y n = - 2 = 22,8. 2,2.

Despreciando n = 2,2. la solución es n = 22,8 días, n - 5 = 17,8 días.

29. Se lanza verticalmente hacia arriba un objeto con una velocidad inicial de Vo (metros por segundo) y su distancias (metros) al punto de lanzamiento viene dada, en función del tiempo / (segundos), por la fórmula s = Vol - 512

Suponiendo que el objeto se lanza con una velocidad inicial de 40 mis, hallar el tiempo que tardará en hallarsea una distancia de 35 m del punto de lanzamiento. (En la fórmula se ha tomado g = 10 mis en cada segundo.)

s = Vol - 5/2, 35 = 40/ - 5/2

, 512- 401 + 35 = O,

40 + foO1 = lO = 7 Y l

Para / = 1 s, s = 35 m y el objeto está ascendiendo; para / = 7 s, s = 35 m y el objeto está descendiendo.Se puede comprobar, además, gráficamente en la representación de la ley del movimiento s en función de 1.

80

El 64

...,48

32

16

4

/ seg

30. Res

a)

b)

e)

31. Res

a)

b)

e)

32. Res

a)

b)

33. Res

a)

b)

e)

34. Res

a)

b)

35. Cah

alb)

e)

36. Hal

a)

b)

37. Hal

a)

b)

e)

http://carlos2524.jimdo.com/

Page 131: Algebra Superior Murray R Spiegel

35. Calcular, sin resolver las ecuaciones, la suma S y el producto P de las raíces de las ecuaciones siguientes.

a) 2X2 + 3x + 1 = O d) 2X2 + 6x - 5 = O g) 2X2 + 5kx + 3k2 = O

b) x-x2=2 e) 3X2-4=0 h) 0,2x2 - O,lx + 0.03 = Oo.

e) 2x(x + 3) = 1 f) 4x2 + 3x = O i) Jix2 - fix + 1 = O

ha

a-30

B.

di-

cia¡'.rseo.)

ECUACIONES DE SEGUNDO GRADO CON UNA INCOGNITA 123

PROBLEMAS PROPUESTOS

30. Resolver las ecuaciones siguientes.

a) x2 - 40 = 9 x 4 1 - 2x x-2 2x 5d) f) h) x---=-- -b) 2X2 - 400 = O 16 x 3 - x 3x - 1 x+1 x + 1

y2 y2 1 1 1x2 + 36 = 9 - 2X2

e) -=-+2 g) -------=e) 3 6 2x - 1 2x + 1 4

31. Resolver las ecuaciones siguientes por el método de la descomposición en factores.

a) x2 - 7x = -12 d) 2X2 + 2 = 5x x 4ag)

2a=x+2a1 1

h) 4 _ x - 2 + x14

2x - 1 x + 2 10i) . x + 2 + 2x - 1 = "3j) 2e - 3y Y 2

~- 2y-e=")

b) x2 + X = 6

e) x2 = 5x + 24

e) 9x2 = 9x - 2

f) 4x - 5x2 = -12

32. Resolver las ecuaciones siguientes completando un cuadrado perfecto.

a) x2 + 4x - 5 = O

b) xix - 3) = 4

e) 2X2 = x + 1

d) 3x2 - 2 = 5x

e) 4x2 = 12x - 7

f) 6y2 = 19y - 15

g) 2X2 + 3a2 = 7ax

h) 12x - 9x2 = 5

33. Resolver las ecuaciones siguientes aplicando la fórmula general.

a) x2 - 5x = 6

b) x2- 6 = x

e) 3x2 - 2x = 8

d) 16x2 - 8x + 1 = O

e) x(5x - 4) = 2

f) 9x2 + 6x = -4

g) 5x2

- 2p2 = l?x 3

2x + 3 3x - 2h) 4x _ 1 = 3x + 2

34. Resolver gráficamente las ecuaciones siguientes.

a) 2X2 + x - 3 = O e) x2 - 2x = 2 e) 6x2 - 7x - 5 = O

f) 2x2+8x+3=0b) 4x2- 8x + 4 = O d) 2X2 + 2 = 3x

36. Hallar el discriminante b2 - 4ae y determinar el carácter de las raíces.

a) 2X2 - 7x + 4 = O

b) 3x2 = 5x - 2

e) 3x - x2 = 4

d) x(4x + 3) = 5

e) 2X2 = 5 + 3x

f) 4xfi = 4x2 + 3

/?) 1 + 2x + 2X2 = O

h) 3x + 25/3x = 10

37. Hallar una ecuación cuadrática de coeficientes enteros (si es posible) cuyas raíces sean las indicadas.

a) 2, -3 d) -2, -5 g) -1 + i, -1 - i j) fi - fi, fi + fih) -3, O e) -1/3, 1/2 h) -2 - fi, -2 + fi k) a + bi. a - bi a, b enteros

3 3 m+ fir-:

2 + fi, 2 - fi 1)m - ";11

e) 8, -4 f) i) 2 + "2i, 2 -"2i 2 2m, n enteros

http://carlos2524.jimdo.com/

Page 132: Algebra Superior Murray R Spiegel

124 ECUACIONES DE SEGUNDO GRADO CON UNA INCOGNITA

38. Hallar el valor de la constante p en las ecuaciones siguientes para que se satisfaga la condición que se indica .

. a) px" - x + 5 - 3p = O tenga una raíz igual a 2.

b) (2p + 1)x2 + px + p = 4(px + 2) la suma de sus raíces sea igual a su producto.

e) 3x2 + p(x - 2) + 1 = O una de sus raíces sea el recíproco de la otra.

d) 4x2 - 8x + 2p - I = O una de las raíces sea igual al triple de la otra.

e) 4x2 - 20x + p2 - 4 = O una raíz sea igual a la otra más 2.

f) x2 = 5x - 3p + 3 la diferencia entre sus raíces sea igual a 11.

39. Hallar las raíces de las ecuaciones siguientes de forma que se cumpla la condición que se indica.

a) 2px2 - 4px + 5p = 3x2 + X - 8 el producto de sus raíces sea igual al doble de su suma.

b¡ x2 - 3(x - p) - 2 = O una raíz sea igual al doble de la otra menos 3.

e) p(x2 + 3x - 9) = x - x2 las raíces sean iguales y de signo contrario.

d) (m + 3~X2 + 2m(x + 1) + 3 = O una raíz sea igual a la mitad del recíproco de la otra.

e) (2m + I )x2 - 4mx = 1 - 3m las raíces sean iguales.

40. Resolver las ecuaciones.

a) Jx2-x+2=2 e) J2x + 7 = Jx + 2 i) VX2 - ~ = 2 - x

b) fo=-2=x-1 f) J2X2 - 7 - x = 3 j) J2x - 10 + Jx-:;=9 = 2

e) ~=3-3x g) ~-4+~=0 k) $+s + .j2x+s = J8x + 25

2 - .JX2 + 2x = OI .J2x-1 =~d) h) 2Jx-~= 1)

J4x - 3

41. Resolver las ecuaciones.

a) x4- 13x2 + 36 = O

b) x4 - 3x2 - 10 = O

e) 4x-4 - 17x-2 + 4 = O

d) .'(-4/3 - 5x-2/3 + 4 = O

e) (x2 - 6X)2 - 2(x2 - 6x) = 35 h) Jx"+2-.yx+2=6

f) x2 + X = 7J.'(2 + X + 2 - 12 i) x3 - 7X3/2 - 8 = O

g) I 2 7 1 x2 + 2 8x(x + -) - -(x + -) = 2 j) --+--=6

x 2 x X x2 + 2

42. Problemas de números.

a) Hallar dos números sabiendo que la suma de sus cuadrados es 34y que uno de ellos es igual al doble del otromenos l.

b) Hallar tres números enteros consecutivos sabiendo que la suma de sus cuadrados es igual a 110.

e) Hallar dos números positivos sabiendo que su diferencia es igual a 3 y que la suma de sus recíprocoses 1/2.

d) Hallar un número sabiendo que es igual al doble de su raíz cuadrada más 3.

43. Problemas geométricos.

a) Hallar las dimensiones de un rectángulo sabiendo que su longitud es igual al triplo de su altura, y que si sedisminuye en I m la altura y se aumenta en 3 m la longitud, el área vale 72 m2.

b) El perímetro de un triángulo rectángulo es 60 cm y su hipotenusa vale 25 cm. Hallar las longitudes de losotros dos lados.

d

44.

45.

46.

47.

48.

http://carlos2524.jimdo.com/

Page 133: Algebra Superior Murray R Spiegel

ndica.

..

+ 2S

el otro

íprocos

ue si se

de los

ECUACIONES DE SEGUNDO GRADO CON UNA lNCOGNITA 125

e) Un cuadro de 8 por 12 cm se coloca en un marco de ancho constante. Hallar dicha anchura sabiendo queel área del cuadro es igual a la del marco.

d) Para formar una caja abierta de 60 cm cuadrados de base a partir de una placa rectangular de estaño de9 x 12 cm se cortan de sus esquinas unas piezas cuadradas y se doblan después las aristas. Hallar la longi-tud del lado del cuadrado que se corta en cada esquina.

44. Problemas de digitos.

a) Hallar un número de dos cifras sabiendo que la cifra de las decenas es igual al doble de la cifra de las uni-dades. y que si se multiplica dicho número por la suma de sus cifras se obtiene 63.

h) Hallar un número de dos cifras sabiendo que la cifra de las decenas excede en 3 a la cifra de las unidades. yque el número es igual a la suma de los cuadrados de sus cifras menos 4.

45_ Problemas de móviles.

a) Dos personas parten del mismo punto y al mismo tiempo dirigiéndose por dos caminos perpendiculares.Sabiendo que la velocidad de una de ellas es de 4 km/h más que la de la otra, y que al cabo de 2 hdistan 40 km, hallar sus velocidades .

h) Hallar la velocidad de un motorista sabiendo que si la aumenta en 10 krn/h recorrería 120 km en 36 minutos.

e) La velocidad de una canoa, en aguas en reposo. es de 12 km/h. Sabiendo que recorre 36 km aguas abajo yregresa al punto de partida en un tiempo de 8 h, hallar la velocidad de la corriente del río.

46. Problemas comerciales.

a) Un comerciante compra cierto número de unidades de un articulo por un total de 720 pts. Hallar el númerode unidades que compró sabiendo que al venderlas a 40 pts cada una obtiene una ganancia igual al dinero"que le costaron 8 de ellas.

h) Un comerciante compró cierto número de unidades de un artículo por 14,40 pts. Posteriormente, el preciode dicho articulo sufre un aumento de 2 céntimos cada unidad, con lo cual. por el mismo dinero le dan 24unidades menos que la vez anterior. Hallar las unidades que inicialmente compró y el precio de cada unade ellas.

47. Problemas de tiempos de trabajo.

a) El operario B tarda 6 h más que el A en efectuar un trabajo. Hallar cuánto tiempo tardarían en realizarlocada uno de ellos sabiendo que. juntos. invierten 4 h en terminarlo.

h) Por medio de un grifo A se llena un depósito en 4 h. Por medio de otro B se llena en 3 h más queempleando los dos grifos A y B simultáneamente. Hallar en cuánto tiempo se llena utilizando solo el grifo B.

48. Se lanza un objeto verticalmente hacia arriba. La distancia s (metros) del punto de partida en función del tiem-po, (segundos) viene dada por (tomando K = lO mis en cada segundo) s = 20, - 5,2

a) Hallar los instantes en los cuales el objeto está a una distancia de 15 m.h) Determinar si el objeto llega a alcanzar una altura de 25 m.el Hallar la máxima altura que alcanza.

",

ECUACIONES DE SEGUNDO GRADO CON UNA INCOGNITA 125

e) Un cuadro de 8 por 12 cm se coloca en un marco de ancho constante. Hallar dicha anchura sabiendo que el área del cuadro es igual a la del marco.

d) Para formar una caja abierta de 60 cm cuadrados de base a partir de una placa rectangular de estaño de 9 x 12 cm se cortan de sus esquinas unas piezas cuadradas y se doblan después las aristas. Hallar la longi­tud del lado del cuadrado que se corta en cada esquina.

44. Problemas de dígitos.

a) Hallar un número de dos cifras sabiendo que la cifra de las decenas es igual al doble de la cifra de las uni­dades, y que si se multiplica dicho número por la suma de sus cifras se obtiene 63.

h) Hallar un número de dos cifras sabiendo que la cifra de las decenas excede en 3 a la cifra de las unidades, y que el número es igual a la suma de los cuadrados de sus cifras menos 4.

45_ Problemas de móviles.

a) Dos personas parten del mismo punto y al mismo tiempo dirigiéndose por dos caminos perpendiculares. Sabiendo que la velocidad de una de ellas es de 4 km/h más que la de la otra, y que al cabo de 2 h distan 40 km , hallar sus velocidades.

h) Hallar la velocidad de un motorista sabiendo que si la aumenta en 10 km/h recorrería 120 km en 36 minutos.

e) La velocidad de una canoa, en aguas en reposo. es de 12 km/h. Sabiendo que recorre 36 km aguas abajo y regresa al punto de partida en un tiempo de 8 h, hallar la velocidad de la corriente del río.

46_ Problemas comerciales.

a) Un comerciante compra cierto número de unidades de un artículo por un total de 720 pts. Hallar el número de unidades que compró sabiendo que al venderlas a 40 pts cada una obtiene una ganancia igual al dinero > que le costaron 8 de ellas.

h) Un comerciante compró cierto número de unidades de un artículo por 14.40 pts. Posteriormente, el precio de dicho articulo sufre un aumento de 2 céntimos cada unidad, con lo cual , por el mismo dinero le dan 24 unidades menos que la vez anterior. Hallar las unidades que inicialmente compró y el precio de cada una de ellas.

47_ Problemas de tiempos de trabajo.

a) El operario B tarda 6 h más que el A en efectuar un trabajo. Hallar cuánto tiempo tardarían en realizarlo cada uno de ellos sabiendo que, juntos. invierten 4 h en terminarlo.

h) Por medio de un grifo A se llena un depósito en 4 h. Por medio de otro B se llena en 3 h más que empleando los dos grifos A y B simultáneamente. Hallar en cuánto tiempo se llena utilizando solo el grifo B.

48_ Se lanza un objeto verticalmente hacia arriba. La distancia s (metros) del punto de partida en función del tiem­po, (segundos) viene dada por (tomando!{ = 10 mis en cada segundo) s = 20, - 5,2

a) Hallar los instantes en los cuales el objeto está a una distancia de 15 m. h) Determinar si el objeto llega a alcanzar una altura de 25 m. el Ha llar la máxima altura que alcanza.

".

http://carlos2524.jimdo.com/

Page 134: Algebra Superior Murray R Spiegel

35. a) S= -3/2, P= 1/2b) S = 1, P = 2e) S= -3, P= -1/2

d) S = - 3, P = - 5/2e) S = O, P = -4/3

f) S = - 3/4, P = O

g) S = -5k/2, P = 3k2/2h) S = 0,5, P = 0,15

i) S = t)6, P = tfi

SOL

126 ECUACIONES DE SEGUNDO GRADO CON UNA INCOGNITA

SOLUCIONES DE LOS PROBLEMAS PROPUESTOS

30. a) x = +7 e) x = ±3i e) y = ±2.)3 g) x = ±3/2b) x=±lOfi d) x = ±8 f) x = ±I h) x = ±2

31. a) 3, 4 e) 8, -3 e) 1/3, 2/3 g) 2a, -4a i) 1, -7b) 2, -3 d) 2, 1/2 f) 2. -6/5 h) 2, -8 j) 2e/5, 4e/5

32. a) 1, -5 el 1, -1/2 3 ± fi f) 3/2, 5/3 2 ie) h) -+-

b) 4, -1 d) 2, -1/3 2 g) 3a, a/2 3 - 3

33. a) 6, -1 e) 2, -4/3 2±fof)

-1 ± i.)3g)

2p 3ph)

6±j42

b) 3, -2 d) 1/4, 1/4e)

5 3 3' 5 3 LA

36. a) 17; reales, irracional es, distintasb) 1; reales, racionales, distintase) - 7; imaginariasd) 89; reales, irracional es, distintas

e) 49; reales, racionales, distintas.f) O; reales, igualesg) - 4; imaginariash) O; reales, racionales, iguales

37. a) x2 + x - 6 = O

b) x2 + 3x = O

e) x2 - 4x - 32 = O

d) x2 + 7x + 10 = O

e) 6x2 - x - l = O

f) x2- 4x + 1 = O

g) x2 + 2x + 2 = O

h) x2 + 4x - 2 = O

i) 4x2- 16x + 25 = O

j) no es posible

k) x2 - 2ax + a2 + b2 = O

/) 4x2 - 4mx + m2 - n = O

38. a) p = -3 b) p = -4 d) P = 2 e)p=±5 f)p=-7e) p = -1

39. a) 3, 6 b) 1,,2 e) ±3f2 d) 1/2 ± tne) Si m = -1, las raíces son 2, 2; si m = 1/2, las raíces son 1/2, 1/2

40. a) 2, -1 e) 4/9 e) 9, I g) ±2 i) 3/2 k) -2b) 1, 3 d) -4,2 f) 8, -2 h) I j) no tiene solución /) 5/4

41. a) ±2, ±3 d) ± 1, ± 1/8 g) 2 ±.)3, -1/4 ± ij!5/4b) ±J5, ±ifi e) 7, 5, ± l hl 79e) ±2, ± 1/2 f) 1, -2. (-1 ± fo)/2 i) 4 j) I ± i, 2 ± fi

42. a) 5, 3 o -27/5, -11/5 b) 5,6.70 -7, -6. -5 e) 3. 6 dl 9

43. a) 5, 15 m b) 15, 20 cm e) 2 cm d) 1.3 cm

44. al 21 b) 85

45. a) 12, 16 kmfh b) 40 krn/h e) 6 krn/h

46. a) 24 h) 144, 10 céntimos

47. a) A, 6 h : B, 12 h h) 5,3 h aproximadamente

48. a) 1 y 3 segundos después de lanzado b) No e) 20 m

SOL

http://carlos2524.jimdo.com/

Page 135: Algebra Superior Murray R Spiegel

fi

CAPITULO 14

Ecuaciones de segundo grado con dos incógnitas

LA FORMA GENERAL DE UNA ECUACION DE SEGUNDO GRADO con dos incógnitas (o va-riables), x e y, es

ax? + bxy + cy2 + dx + ey + f = O (1)

siendo a, b, e, d, e, f constante y a, b, e distintas de cero.

Por ejemplo, 3x2 + 5xy = 2, x2 - xy + y2 + 2x + 3y = O, y2 = 4x, xy = 4 son ecuacio-nes cuadráticas en x e y.

LA GRAFICA DE LA ECUACION (1), siendo a, b, c, d, e y freales, depende del valor del discrimi-nante b2 - 4ac.

1) Si b2 - 4ac < O, la gráfica es, en general, una elipse. Sin embargo, si b = O Y a = e la gráficapuede ser una circunferencia, un punto, o no existir.

2) Si b2 - 4ac = O, la gráfica es una parábola, dos rectas paralelas coincidentes, o no existe.

3) Si b2 - 4ac > O, la gráfica es una hipérbola, o dos rectas que se cortan.

Estas figuras resultan al seccionar un cono recto circular por un plano, razón por la cual reci-ben el nombre de secciones cónicas.

SISTEMA DE ECUACIONES CUADRATICAS

SOLUCION GRAFICA. Las soluciones reales de un sistema de dos ecuaciones de segundo grado enx e y son los valores de x e y correspondientes a los puntos de intersección de las curvas o gráficasde ambas ecuaciones. Si no se cortan, las soluciones del sistema son imaginarias.

SOLUCION ALGEBRAICA

A) Una ecuación lineal y una cuadrática

Se despeja una de las incógnitas de la ecuación lineal y se sustituye en la de segundo grado.

Ejemplo Resolver el sistema 1) x + y = 72) x2 + y2 = 25

Despejando y en 1), y = 7 - x. Sustituyendo en 2) se obtiene

x2 + (7 - x)1 = 25, x2 - 7x + 12 = O, (x - 3) (x - 4) = O, y x = 3,4.

Para x = 3, y = 7 - x = 4; para x = 4, y = 7 - x = 3. Las soluciones del sis-tema son x = 3, y = 4 Y x = 4, y = 3.

127

CAPITULO 14

Ecuaciones de segundo grado con dos incógnitas

LA FORMA GENERAL DE UNA ECUACION DE SEGUNDO GRADO con dos incógnitas (o va­riables). x e y, es

ax2 + bxy + cy2 + dx + ey + f = O (1)

siendo a, b, c, d, e, f constante y a, b, c distintas de cero.

Por ejemplo, 3X2 + 5xy = 2, X2 - xy + y2 + 2x + 3y = O, y2 = 4x, xy = 4 son ecuacio­nes cuadráticas en x e y.

LA GRAFICA DE LA ECUACION (1), siendo a, b, c, d, e y freales, depende del valor del discrimi­nante b2

- 4ac.

1) Si b1 - 4ac < O, la gráfica es, en general, una elipse. Sin embargo, si b = O Y a = c la gráfica

puede ser una circunferencia, un punto, o no existir.

2) Si b2 - 4ac = O, la gráfica es una parábola, dos rectas paralelas coincidentes, o no existe.

3) Si b2 - 4ac > O, la gráfica es una hipérbola , o dos rectas que se cortan.

Estas figuras resultan al seccionar un cono recto circular por un plano, razón por la cual reci­ben el nombre de secciones cónicas.

SISTEMA DE ECUACIONES CUADRATICAS

SOLUCION GRAFICA. Las soluciones reales de un sistema de dos ecuaciones de segundo grado en x e y son los valores de x e y correspondientes a los puntos de intersección de las curvas o gráficas de ambas ecuaciones. Si no se cortan, las soluciones del sistema son imaginarias.

SOLUCION ALGEBRAICA

A) Una ecuación lineal y una cuadrática

Se despeja una de las incógnitas de la ecuación lineal y se sustituye en la de segundo grado.

Ejemplo Resolver el sistema 1) x + y = 7 2) X2 + y2 = 25

Despejando y en 1), Y = 7 - x. Sustituyendo en 2) se obtiene

X2 + (7 - x¡Z = 25, X2 - 7x + 12 = O, (x - 3)(x - 4) = O, y x = 3,4.

Para x = 3, Y = 7 - x = 4; para x = 4, Y = 7 - x = 3. Las soluciones del sis­tema son x = 3, Y = 4 y x = 4, Y = 3.

127

http://carlos2524.jimdo.com/

Page 136: Algebra Superior Murray R Spiegel

128 ECUACIONES DE SEGUNDO GRADO CON DOS INCOGNITAS

B) Dos ecuaciones de la forma ax? + by2 = c.

Se aplica el método de reducción.

Ejemplo. Resolver el sistema 1) 2X2 - y2 = 72) 3x2 + 2y2 = 14

Para eliminar y, se multiplica 1) por 2 y se suma a 2); así, pues,

7x2 = 28, x2 = 4 Y x = ± 2

Haciendo x = 2 o x = -2 en 1) se obtiene y = ± 1

Las cuatro soluciones son:

x = 2, Y = 1; x = - 2, Y = 1; x = 2, Y = - 1; x = - 2, Y = - 1

e) Dos ecuaciones de la forma ax' + bxy + cy2 = d.

Ejemplo. Resolver el sistema 1) x2 + xy = 62) x2 + Sxy - 4y2 = lO

Método l. Eliminar el término independiente entre las dos ecuaciones.

Multiplicando 1) por S, 2) por 3, y restando, tendremos

x2 - Sxy + 6y2 = O, (x - 2y) (x - 3y) = O, x = 2y Y x = 3y

Haciendo x = 2y en 1) o 2) se obtiene y2 = 1, Y = ± 1

Para y = 1, x = 2y = 2; para y = -1, x = 2y = -2.Luego dos soluciones son: x = 2, Y = 1; x = -2, Y = -1

. 2 1 liHaciendo x = 3y en 1) o 2) se obtiene y = 2' y = ± 2

0. 30. 0. 30.Para y = T' x = 3y = T; para y = - T' x = - TLuego las cuatro soluciones son: x = 2, Y = 1; x = - 2, Y = - 1;

_~ -li. __~ __lix- 2 ,y- 2'x- 2'y- 2

Método 2. Hacer y = mx en ambas ecuaciones.6

De 1): x2 + mx2 = 6, x2 = ---1 + m

lODe 2): x2 + Smx2 - 4m2x2 = lO, x2 = -+-ccSm---4n12

6Luego 1 + m

10 1 1S 4 2 de donde m = -2 ' -3; por tanto,+ m - m

y = x/2, y = xf). A partir de aquí se procede como en el Método 1.

D) Otros métodos

1) Algunos sistemas de ecuaciones se pueden resolver mediante otros equivalentes más sen-cillos (véase Problemas 10-12).

2) Una ecuación es simétrica con respecto a x e y cuando al permutar x por y no se altera.Por ejemplo, x2 + .1'2- 3xy + 4x + 4y = 8, es simétrica con respecto a x e y. Los siste-mas de ecuaciones simétricas se pueden resolver, en general, efectuando los cambios devariable x = u + v, )' = u - u. (Véase Problemas 13-14.)

1. Traza

al 4.'

-t-

a) 4.1

olos va

b) 4,x

01

e) 4,)

01

L,

128 ECUACIONES DE SEGUNDO GRADO CON DOS INCOGNITAS

B) Dos ecuaciones de la forma ax2 + by2 = c.

Se aplica el método de reducción.

Ejemplo. Resolver el sistema 1) 2X2 - y2 = 7 2) 3X2 + 2y 2 = 14

Para eliminar y, se multiplica 1) por 2 y se suma a 2); así, pues,

7X2 = 28, X2 = 4 Y x = ± 2

Haciendo x = 2 o x = -2 en 1) se obtiene y = ± 1

Las cuatro soluciones son:

x = 2, Y = 1; x = - 2, Y = 1; x = 2, Y = - 1; x = - 2, Y = - 1

e) Dos ecuaciones de la forma ax2 + bxy + cy2 = d.

Ejemplo. Resolver el sistema 1) X2 + xy = 6 2) X2 + Sxy - 4y2 = 10

Método l. Eliminar el término independiente entre las dos ecuaciones.

Multiplicando 1) por S, 2) por 3, y restando, tendremos

X2 - Sxy + 6y2 = O, (x - 2y) (x - 3y) = O, x = 2y Y x = 3y

Haciendo x = 2y en 1) o 2) se obtiene y2 = 1, Y = ± 1

Para y = 1, x = 2y = 2; para y = -1 , x = 2y = -2. Luego dos soluciones son: x = 2, Y = 1; x = -2, Y = -1

. 2 1 .Ji Haciendo x = 3y en 1) o 2) se obtIene y = 2' y = ± 2

0. 30. 0. 30. Para y = T' x = 3y = T; para y = - T' x = - T Luego las cuatro soluciones son: x = 2, Y = 1; x = - 2, Y = - 1 ;

_ ~ _ .Ji. __ ~ - _ .Ji x- 2 ,y- 2'x- 2' y - 2

Método 2. Hacer y = mx en ambas ecuaciones.

6 De 1): X2 + mx2 = 6, X2 = ---

1 + m 10

De 2): X2 + Smx2 - 4m2x 2 = 10, X2 = -+- S-m---4;;?"

Luego 6

1 + m

10 1 1 S 4 2 de donde m = -2 ' -3; por tanto, + m - m

y = x /2, y = x / 3. A partir de aquí se procede como en el Método 1.

D) Otros métodos

1) Algunos sistemas de ecuaciones se pueden resolver mediante otros equivalentes más sen­cillos (véase Problemas 10-12).

2) Una ecuación es simétrica con respecto a x e y cuando al permutar x por y no se altera. Por ejemplo, X2 + .1'2 - 3xy + 4x + 4y = 8, es simétrica con respecto a x e y. Los siste­mas de ecuaciones simétricas se pueden resolver, en general , efectuando los cambios de variable x = u + v, )' = u - v. (Véase Problemas 13-14.)

http://carlos2524.jimdo.com/

Page 137: Algebra Superior Murray R Spiegel

1.

ECUACIONES DE SEGUNDO GRADO CON DOS INCOGNITAS 129

PROBLEMAS RESUELTOS

l. Trazar la gráfica de las ecuaciones siguientes:

a) 4x2 + 9y2 = 36, bl 4x2 - 9y2 = 36, el 4x + 9y2 = 36.

y y y

(a) Elipse (b) Hipérbola (e) Parábola

al 4x2 + 9)'2 = 36,2

y=±3~

Obsérvese que)' es real cuando 9 - x2 ;¡; O, es decir, cuando - 3 ~ x ~ 3. Por tanto, hay que prescindir delos valores de x mayores que 3 y menores que - 3.

x -3 -2 -1 O 1 2 3

Y O ±1,49 ±1,89 ±2 ±1,89 ±1,49 O

La gráfica es una elipse con centro en el origen.4 2

bl 4,\'2 - 9y2 = 36, y2 = 9'(x2 - 9), Y = ± 3p-=9

Obsérvese que para que y sea real, x no puede tomar valores entre - 3 y 3.

x 6 5 4 3 -3 -4 -5 -6

Y ±3,46 ±2,67 ±1,76 O O ±1,76 ±2,67 ±3,46

La gráfica consta de dos ramas y se denomina hipérbola.

el 4x + 9,,2 = 36,4

,,2 - -(9 - xl.' - 9 "2

v=+- ~. - 3""" - .\

Obsérvese que si x es mayor que 9, )' es imaginario.

x -1 O 1 5 8 9

Y ±2,11 ±2 ±1,89 ±1,33 ±O,67 O

La gráfica es una parábola.

http://carlos2524.jimdo.com/

Page 138: Algebra Superior Murray R Spiegel

••l' :1

130 ECUACIONES DE SEGUNDO GRADO CON DOS INCOGNITAS

2. Trazar la gráfica de las ecuaciones siguientes:

a) xy = 8, b) 2X2 - 3xy + y2 + X - 2y - 3 = O, e) x2 + y2 - 4x + 8y + 25 = O,

y

(a) Hipérbola (b) Dos rectas que se cortan

a) xy = 8, Y = 8/x, Obsérvese que si x es un número real cualquiera excepto cero, y es real. La gráfica es unahipérbola,

x 4 2 1 ! -! -1 -2 -4Y 2 4 8 16 -16 -8 -4 -2

b) 2X2 - 3xy + y2 + X - 2y - 3 = O, Ordenando la ecuación y2 - (3x + 2)y + (2x2 + X - 3) = O Y resol-

3x + 2 + JX2 + 8x + 16 (3x + 2) + (x + 4)viéndola por la fórmula general se obtiene y = - 2 = 2 O sea y =

2x + 3, Y = x - 1, La ecuación dada es equivalente a dos ecuaciones lineales, como se puede ver expresando laecuación dada por (2x - y + 3) (x - y - 1) = O. La gráfica está formada por dos rectas que se cortan.

-4 + J-4(x2 - 4x + 9)e) Ordenando, y2 + 8y + (x2 - 4x + 25) = O; resolviendo, y = - 2

Como x2 - 4x + 9 = x2 - 4x + 4 + 5 = (x - 2)2 + 5 es siempre positivo, la cantidad subradical es ne-gativa. Por consiguiente, y es imaginario para todos los valores reales de x y no se puede trazar la gráfica corres-pondiente.

3. Resolver gráficamente los sistemas siguientes: x2 + y2 = 25 x2 + 4y2 = 16 x2 + 2y = 9a) x + 2y = 10' b) xy = 4 ,e) 2X2 _ 3y2 =

y

(a) x' + y' = 25 circunferencia (b) x' + 4y' = 16 elipse (e) x2 + 2y = 9 parábola

2X2 - 3y2 = 1 hipérbolax + 2y = 10 recta xy = 4 hipérbola

4. I

5. 1

6. 1

7. l

8. F

http://carlos2524.jimdo.com/

Page 139: Algebra Superior Murray R Spiegel

na

1,

la

neo

ECUACIONES DE SEGUNDO GRADO CON DOS INCOGNITAS 131

4 R l l· .. x + 2v = 4, eso ver os sistemas siguientes: a) y2 _ :"(.\'= -7 b) 3x - I + 2y = O

3x2 _),2 + 4 = O

a) Despejando x en la ecuación lineal, x = 4 - 2y. Sustituyendo en la ecuación cuadrática.

y2 _ y(4 - 2y) = 7, 3y2 - 4y - 7 = O, lv + 1) (3y - 7) = O e )' = -1, 7/3

Si Y = -1, x = 4 - 2y = 6; si y = 7/3, x = 4 - 2y = -2/3.

Las soluciones son x = 6, y = -1 y x = -2/3, Y = 7/3.

b) Despejando y en la ecuación lineal, y = 'W - 3x). sustituyendo en la ecuación cuadrática,

3x2 - [W - 3X)]2 + 4 = O,-2 ± J22 - 4(1) (5)

x = = -1 ± 2;2(1)

x2 + 2x + 5 = O y

Si x = -1 + 2;, y = Í(l - 3x) = t[l - 3( - I + 2;)] = í(4 - 6;) = 2 - 3;.

Si x = - I - 2;, y = t(l - 3x) = t[l - 3( - I - 2;)] = t(4 + 6;) = 2 + 3;.

Las soluciones son x = - I + 2;, y = 2 - 3; y x = - I - 2;, y = 2 + 3;.

S, Resolver el sistema: (1) 2X2 - 3y2 = 6, (2) 3x2 + 2y2 = 35.

Para eliminar y, se multiplica (1) por 2, (2) por 3 y se suman; luego I3x2 = 1l7, x2 = 9, x = ±3.

Sustituyendo x = 3 o x = -3 en (1) obtenemos y = ±2.

Las soluciones son: x = 3, Y = 2; x = '-3, Y = 2; x = 3, Y = -2; x = -3, Y = -2.

6. Resolver el sistema:8 3

(1)---=5x2 y2 '5 2

(2)"2 +"2 = 38.x y

I 1. 1 1Las ecuaciones son cuadráticas en - y -. Sustituyendo u = - y v = -, se obtiene

x y x y

8u2 - 3v2 = 5 y 5.u2 + 2v2 = 38

La solución de este sistema es, u2 = 4, v2 = 9 o x2 = 1/4, y2 = 1/9; luego x = ± 1/2, y = ± 1/3.

Las soluciones son: x = 1/2, Y = 1/3; x = -1/2, Y = 1/3; x = 1/2, Y = -1/3; x = -1/2, Y = -1/3.

7 R l l si (1) 5x2 + 4y2 = 48 l" do Ios té . . d di. eso ver e sistema (2) X2 + 2xy = 16 e munan o os termmos m epen lentes.

Multiplicamos (2) por 3 y se resta de (1) con lo cual

2X2 - 6xy + 4y2 = O, x2 - 3xy + 2y2 = O, (x - y) (x - 2y) = O y x = y, x = 2)'

16 4Sustituyendo x = y en (1) o (2), -tenemos y2 = "3 e y = ± 3)3.

Sustituyendo x = 2y en (1) o (2), tenemos y2 = 2 e y = ±)2.

Las cuatro soluciones son:

y =-)2

8. Resolver el sistema (1) 3x2

- 4xy = 4(2) x2 - 2y2 = 2 aplicando la sustitución y = mx.

ECUACIONES DE SEGUNDO GRADO CON DOS INCOGNITAS

4 R I l · .. x + 2v = 4 . eso ver os sistemas siguientes : a), . 7' Y - xy =

b) 3x - I + 2y = O 3x' - )" + 4 = O

a) Despejando x en la ecuación lineal, x = 4 - 2y . Sustituyendo en la ecuación cuadrática.

y' - y(4 - 2y ) = 7. 3y' - 4y - 7 = O, Lv + 1) (3y - 7) = O e y = -1 , 7/3

Si Y = -1 , x = 4 _. 2y = 6; si y = 7/3, x = 4 - 2y = -2/3.

Las soluciones son x = 6, y = -1 y x = -2/3, Y = 7/3.

b) Despejando y en la ecuación lineal , y = i(1 - 3x). sustituyendo en la ecuación cuadrática,

131

3x' - U(1 - 3x)]' + 4 = O, x' + 2x + 5 = O y x = -2 ± J2' - 4(1)(5) = - 1 ± 2; 2(1)

Si x = -1 + 2;, y = t(1 - 3x) = t[t - 3( - I + 2i)] = t(4 - 6i) = 2 - 3i.

Si x = - 1 - 2i, Y = i (1 - 3x) = t[t - 3( -1 - 2i)] = Í(4 + 6i) = 2 + 3i.

Las soluciones son x = - I + 2i, Y = 2 - 3i Y x = - I - 2i, Y = 2 + 3i.

5. Resolver el sistema: (1) 2x ' - 3y' = 6, (2) 3x' + 2y' = 35.

Para eliminar y, se multiplica (1) por 2, (2) por 3 y se suman; luego 13x' = 117, x' = 9, x = ±3.

Sustituyendo x = 3 o x = - 3 en (1) obtenemos y = ± 2.

Las soluciones son: x = 3, y = 2; x = '- 3, y = 2; x = 3, Y = -2 ; x = - 3, y = -2.

6. Resolver el sistema: 8 3

(1) - - - = 5 x' y' '

5 2 (2) - + - = 38.

x' y '

1 1 1 1 Las ecuaciones son cuadráticas en - y - . Sustituyendo u = - y v = -, se obtiene

x y x Y

8u' - 3v' = 5 y 5)1' + 2v' = 38

La solución de este sistema es, u' = 4, v' = 9 o x' = 1/4, y' = 1/9 ; luego x = ± 1/2, y = ± 1/3.

Las soluciones son : x = 1/2 , Y = 1/3; x = -1 /2, y = 1/3 ; x = 1/2, Y = -1 /3; x = -1 /2, y = -1 /3.

7 R l l · (1) 5x' + 4y' = 48 l" di " . d d ' . eso ver e sistema (2) x' + 2xy = 16 e Imman o os termmos m epen lentes.

Multiplicamos (2) por 3 y se resta de (1) con lo cual

2x' - 6xy + 4y' = O, x' - 3xy + 2y' = O, (x - y ) (x - 2y ) = O y x = y , x = 2y

S . ,16 4 ¡;;

ustItuyendo x = y en (1) o (2), ·tenemos y = 3' e y = ± 3'; 3.

Sustituyendo x = 2y en (1) o (2), tenemos y' = 2 e y = ±fi.

Las cuatro soluciones son :

x = ifi, y = ifi; x = - ifi, y = - ifi; x = 2fi, y = fi; x = -2fi , Y = - fi

(1) 3x' - 4xy = 4 8. Resolver el sistema (2) x' _ 2y' = 2 aplicando la sustitución y = mx.

http://carlos2524.jimdo.com/

Page 140: Algebra Superior Murray R Spiegel

132 ECUACIONES DE SEGUNDO GRADO CON DOS INCOGNITAS

3x' - 41>1x' x24

Sustituimos r = »r.v en (1): luego =4 Y =3 - 4m

x2 2n:!.x2 .\"2 2Sustituimos v = ni.\" en (2): luego - = 2 Y =

2m'I -

Por tanto4 2

3-=-4-;;; = .--=-W . 4m' - 4/11 + I = O. (2m - 1)' = ° y ni = j'. t·

Sustituyendo ahora .1' = mx = j-x en (1) o (2) se obtiene x' = 4. x = ± 2.

Las soluciones son x = 2. .l· = 1 Y .v = -2 • .1' = -l.

9. Resolver el sistema: (1) x' + y' = 40. (2) Xl' = 12.

De (2) • .r = 12/x; sustituyendo en (1). tenemos

144 -,x' + -Z = 40. x' - 40.\' + 144 = O. (x' - 36) (x' - 4) = ° y x = ± 6. ± 2

x

Para x = ±6 . .l' = 12/x = ±2: para x = ±2 . .r = ±6.

Las cuatro soluciones son: x = 6. )' = 2: .v = -6 .. 1' = -2; x = 2. )' = 6; x = -2 . .r = -6.

No/a. La ecuación (2) indica que aquellas soluciones en las que el producto Xl' sea negativo (por ejemplo ..\"= 2. Y = - 6) son extrañas.

10. Resolver el sistema: (1) x' + y' + 2x - y = 14. (2) x' + y' + x - 21' = 9.

Restando (2) de (I): .v + y = 5 o .l' = 5 - x.

Sustituyendo y = 5 - x en (1) o (2): 2x' - 7x + 6 = O. (2x - 3) (x - 2) = ° y x = 3/2. 2.

Las soluciones son x = 3/2. .r = 7/2 y x = 2. r = 3.

11. Resolver el sistema: (l) x3 + y3 = 35. (2) x + .1' = 5.

Dividiendo (1) por (2).x3 + y3

x+y35- y (3) x' - xy + y' = 7.5

De (2). y = 5 - .v ; sustituyendo en (3). tenemos

x' - x(5 - .r] + (5 - x)' = 7. x' - 5x + 6 = O. (x - 3l (x - 2) = ° Y x = 3. 2.

Las soluciones son .v = 3 . .1' = 2 Y .v = 2 . .1' = 3.

12. Resolver el sistema: (1) x' + 3x\' + 2.1" = 3. (2) x' + 5xy + 6\" = 15.

Dividiendo (l) por (2).x' + 3xy + 21"x' + 5x)" + 6y'

(x + y) (x + 2\') = X + V = ~

(x + 3r) (x + 2\') X + 3y 5

De x + y 1 .\' = -2x. Sustituyendo .1' = -2x en (1) o (2). x' = 1 Y .v.v + 3)" "5'

Las soluciones son x = 1, Y = -2 y x = -1, Y = 2.

±l.

13. Resolver el sistema: (1) x' + y' + 2x + 2y = 32. (2) x + y + 2xr = 22,."

Las ecuaciones son simétricas en x e y, ya que cambiando x por y se obtiene la misma ecuación. Sustituyendox = u + 1'• .\' = !I -.l' en (1) y (2). se obtiene

son

x=

14. Res,

11 -

ti =

x=

15. Hall

pedi

16. Hall

(pos

17. Hall

18. La

dos

http://carlos2524.jimdo.com/

Page 141: Algebra Superior Murray R Spiegel

plo.

ndo

ECUACIONES DE SEGUNDO GRADO CON DOS INCOGNITAS

(3) u' + u2 + 2u = 16 Y (4) u' - 11' + U = II

Sumando (3) Y (4). se obtiene 2u2 + 3u - 27 = O. (u - 3)(2u + 9) = O Y u = 3. -9/2.

Para u = 3.1" = I Y v = ± 1; para u = -9/2. ¡-' = 19'4 Y l' = ±fo,'2. Luego las soluciones de (3) y (4)son: u = 3. 11 = 1; u = 3. v = -1; u = -9/2. v = fo12; u = -9/2. ,. = -foI2.

Luego. como .v = u + r, l' = U-l'. las cuatro soluciones de (1) Y (2) son:x = 4. .1' = 2;x = 2. .1' = 4;x = -9/2 + fo/2.y = -9/2 -fo/2;x = -912 -)19/2 . .1' = -9,12+ ./19/2.

14. Resolver el sistema: (1) x' + y' = 180.I I I

(2) - + - = -.x y 4

De (2) se obtiene (3) 4x + 4.1' - xy = O. Como (1) Y (3) son simétricas en .v e .1'. sustituimos x = u + v. y =

11 - ,. en (1) Y (3) obteniendo

(4) 11' + v2 = 90 Y (5) 811 - u2 + ,.2 = O

Restando (5) de (4). tenemos u' - 4u - 45 = O. (u - 9) (u + 5) = O Y u = 9. -5.

Para u = 9. v = ±3; para u = -5. l' = ±.)65. Luego las soluciones de (4) y (5) son:ti = 9. v = 3; u = 9. l' = -3; u = -5. l' =.)65; U = -5. l' = -.)65.

Luego las cuatro soluciones de (1) Y (2) son:x= 12.y=6; x=6.y= 12; x= -5+.)65 . .1'= -5-.)65; x= -5-.)65. l' -5 + .)65.

15. Hallar dos números sabiendo que su suma es 25 y su producto 144.

Sean los números x, y. Tendremos (1) x + y = 25 Y (2) xr = 144.

Las soluciones del sistema formado por (1) Y (2) son x = 9. .1'= 16 Y x = 16. Y = 9. Luego los números

pedidos son 9. 16.

16. Hallar dos números positivos sabiendo que su diferencia es 3 y que la suma de sus cuadrados es 65.

Sean los números p. q. Tendremos (1) p - q = 3 Y (2) p2 + q' = 65.

Las soluciones del sistema formado por (1) Y (2) son p = 7. q = 4 Y P = -4. q = -7. Luego los números

(positivos) pedidos son 7. 4.

17. Hallar las dimensiones de un rectángulo cuyo perímetro es 60 cm y su área 216 cm?

Sean las longitudes de los lados .v, 1'. Tendremos (1) 2x + 2y = 60 Y (2) XI' = 216.

Resolviendo el sistema formado por (1) Y (2) obtenemos los lados de 12 y 18 cm.

18. La hipotenusa de un triángulo rectángulo mide 41 cm y su área es 180 cm". Hallar las longitudes de los

dos catetos.Sean las longitudes de los catetos x. y. Tendremos (1) x' + y' = (41)2 Y (2) í(xY) = 180.

Resolviendo el sistema formado por (1) y (2) obtenemos los catetos de 9 y 40 cm.

133ECUACIONES DE SEGUNDO GRADO CON DOS INCOGN ITAS 133

(3) u' + u' + 2u = 16 Y (4) u' - ¡¡' + 1I = II

Sumando (3) Y (4), se obtiene 2u' + 3u - 27 = O, (u - 3)(2u + 9) = O Y u = 3, -9/2.

Para 1I = 3,1" = I Y v = ± 1; para u = -9/2, r' = 19 '4 Y r = ±fo:2. Luego las soluciones de (3) y (4)

son : u = 3, ¡¡ = 1; u = 3, v = -1; 1I = -9/2. v = fo/2; u = -9/2. r = -fo/2.

Luego, como x = u + r, .l ' = U - 1'. las cuatro soluciones de (1) y (2) son:

x = 4,.1' = 2;x = 2 . .1' = 4 ;x = -9/2 + fo/2.y = -9/2 -fo/2;x = -912 -)19/2 . .1' -9/2 + j19/2.

I I I 14. Resolver el sistema: (1) x' + y' = 180, (2) - + - = - .

x y 4

De (2) se obt iene (3) 4x + 4.1' - xy = O. Como (1) y (3) son simétricas en x e .\' , sustituimos x = u + v. y =

u - r en (1) Y (3) obteniendo

(4)u'+v'=90 y (S)811-u'+r'=0

Restando (5) de (4). tenemos u' - 4u - 45 = O. (u - 9) (u + 5) = O Y u = 9. -5.

Para u = 9, v = ±3; para u = -5. r = ±.)65. Luego las soluciones de (4) y (5) son:

u = 9. v = 3; u = 9. r = -3; u = -5. r =.)65; u = -5. r = -.)65.

Luego las cuatro so luciones de (1) Y (2) son:

x = 12. Y = 6; x = 6 . .1'= 12 ; x = -5 + .)65 . .1' = -5 -.)65 ; x = -5 - .)65,.l' -5 + .)65.

15. Hallar dos números sabiendo que su suma es 25 y su producto 144.

Sean los números x, y. Tendremos (1) x + .r = 25 Y (2) X.l· = 144.

Las soluciones del sistema formado por (1) Y (2) son x = 9. y = 16 Y x = 16. Y = 9. Luego los números

pedidos son 9. 16.

16. Hallar dos números positivos sabiendo que su diferencia es 3 y que la suma de sus cuadrados es 65.

Sean los números p. q. Tendremos (1) P - q = 3 Y (2) p' + q' = 65.

Las soluciones del sistema formado por (1) Y (2 ) son p = 7. q = 4 Y P = - 4. q = -7. Luego los números

(positivos) pedidos son 7. 4.

17. Hallar las dimensiones de un rectángulo cuyo perímetro es 60 cm y su área 216 cm'

Sean las longitudes de los lados x. 1'. Tendremos (1) 2x + 2y = 60 Y (2) .Y\ · = 216.

Resolviendo el sistema formado por (1) Y (2) obtenemos los lados de 12 y 18 cm.

18. La hipotenusa de un triángulo rectángulo mide 41 cm y su área es 180 cm'. Hallar las longi tudes de los

dos catetos. Sean las longitudes de los catetos x. y . Tendremos (1) x' + y' = (41)' Y (2) j-(xy ) = 180.

Resolviendo el sistema formado por (1) y (2) obtenemos los catetos de 9 y 40 cm.

http://carlos2524.jimdo.com/

Page 142: Algebra Superior Murray R Spiegel

134 ECl.:¡\CIONFS DE SEGUNDO GRADO CON DOS INCOGNITAS

PROBLEMAS PROPUESTOS

19. Representar gráficumcnte las ecuaciones siguientes:

al x' + y' = 9 <') y' = 4x

b) xy = -4 f) x' + 3y' - I =0 i) x' + y' - 2x + 2y + 2 = O

e) 4X' + y' = 16 g) x' + 3xy + y' = 16 j) 2X' - xy - y' - 7x - 2y + 3 = O

ti) x' - 4y' = 36 h) x' + 4y = 4

20. Resolver gráficamente los sistemas siguientes:

a) x' + y' = 20. 3..- - Y = 2

b) x' + 4y' = 25. x' - y' = 5

e) y' = x. x' + 21" = 24

d) x2 + 1 = 4y. 3x - 2y = 2

21. Resolver algebraicamente los sistemas siguientes:

a) 2X2 - v' = 14. x.!.. Y = 1

b) xy + x' = 24. Y - 3x + 4 = O

e) 3xy - 10x = y. 2 - Y + x = O

ti) 4x + 5y = 6. xy = -2

e) 2X2 - y2 = 5, 3x2 + 4/ = 57

f) 9/X2 + 16/y2 = 5, 18/x2 - 12/y' = -1

.Ir) x2 - xy = 12. xy - y' = 3

h) x' + 3xy = 18, x' - 5.1'2 = 4

i) x2 + 2xy = 16, 3x2 - 4xy + 2}'2 = 6

j) x2 _ xy + y' = 7, x2 + y' = 10

k) x2 - 3y2 + 10)' = 19, x' - 3y' + 5x = 9

1) x3 - y3 = 9, x - y = 3

m) x3 - y3 = 19.' x2y - xy2 = 6

n) l/x3 + l/y3 = 35, l/x2 - I/xy + l/y2 = 7

22. Hallar dos números sabiendo que el cuadrado de uno de ellos excede en 16 al doble del otro, y que la suma de suscuadrados es 208.

23. La diagonal de un rectángulo mide 85 centímetros. Sabiendo que si el lado menor se aumenta en 11 y el mayorse disminuye en 7. centímetros, la longitud de la diagonal no varía, hallar las dimensiones del rectángulo original.

SOLUCIONES DE LOS PROBLEMAS PROPUESTOS

19. a) circunferenciab) hipérbola

e) parábolaf) elipse

i) un solo punto (1, -1)j) dos rectas que se cortan

g) hipérbolah) parábola

e) elipsed) hipérbola

20. a) (2.4). (-0.8. -4.4) e) (4,2), (4, -2)b) (3.2), (-3.2). (3. -2), (-3, -2) d) (1, 0,5), (5, 6.5)

21. a) (3,2), (-5. -6) i) (2,3). (-2, -3)b) (3.5). (-2. -10) j) (1,3). (-1, -3), (3. 1), (-3. -1)e) (2.4), (-1/3. 5/3) k) (-12. -5), (4,3)d) (-1. 2). (5í2. -4/5) 1) (1, -2), (2, -1)

e) (j? 3). (j? -3). (-,/1.3). (-j? -3) m) (-2, -3). (3. 2)f) (3,2), (3. -2). (-3.2). 1-3. -2) n) (1/2, 1/3), (1/3, 1/2).Ir) (4.1), (-4. -1)

.. r-: -7ifi .Íc 7ifih) (3,1). (-3. -1). (31y'5.~-5-)' (-3iv5. 5-)

22. 12.8; -12. -8; 12. -8; -12.8

23. 40 cm. 75 cm

LA RA:

Así,

PROPOI

térna, b

cion

LEYES

1)

2)

PROPO

1)

2)

3)

4)

5)

6)

http://carlos2524.jimdo.com/

Page 143: Algebra Superior Murray R Spiegel

= o

de sus

ayoriginal.

1)ortan

CAPITULO 15

Razón, proporción y proporcionalidad

LA RAZON de dos números a y b se escribe a : b, y es el cociente o fracción ~ con b +- O.

Así, pues, a : b = ~, b =F O. Si a = b =F O, la razón es 1 : 1 o + = l.

Ejemplos. 1) La razón de 4 a 6 = 4 : 6 = ~ = ~ .

2 4 2/3 52) "3'"5 4/5 6

3y 5x 20x3) 5x:-4 = 3y/4 = 3Y

PROPORCION. Es una igualdad entre dos razones. Por ejemplo, a : b = e : d. ~ bien, ~ = ~. Los

términos a y d reciben el nombre de extremos y b Y e el de medios; d es la cuarta proporcional entrea, b y c.

En la proporción a : b = b : e, e es la tercera proporcional entre a y b, Y b es la media propor-cional entre a y c.

LEYES DE UNA PROPORCIONo Sia e verificab d' se

b 5) a - b c-dd b =r:1) ad = be 3) a

e

b2) ad a + b

4) bc+d

da + b

6) a _ bc+dc-de

PROPORCIONALIDAD

1) Si x varía directamente con y, o bien, si .ves proporcional a y (se escribe .vco y), se puede escribir

x = k y; o lo que es igual, ~ = k, siendo k la constante de proporcionalidad.y

2) Si .v varía directamente con )"2, se tiene XCI:: y2 Y x = ky",

3 S· ,. . k) I X vana mversamente con 1', se nene x = - .. y

4 S· ,. 2' k) I X vana mversamente con y , se tiene x = 2 .

Y5) Si x varía simultáneamente con y y z, se tiene x = kvz.

. k y!6) Si x varía directamente con \,2 e inccrsamcntc con z. se tiene x = - .

. Z

135

http://carlos2524.jimdo.com/

Page 144: Algebra Superior Murray R Spiegel

\.)6 RAZON. PROPORCION y PROPORCIONALIDAD

La constante k está determinada si se conoce un conjunto de valores correspondientes de lasvariables.

PROBLEMAS RESUELTOSRAZON y PROPORCION

1. Expresar las razones siguientes por medio de una fracción simplificada:

96 3a) 96: 128 = 12s = 4

2 3h) 3' 4

2/3 83/4 9

, xv" ye) xy" X"y = - = -

. x"y x

xy' - x'y xy(y - x) xyd) (xy' - x'y) : (x _ y)' - (x - y)' - (x - y)' y - x

2. Hallar la razón entre las cantidades siguientes:

a) 6 kilogramos a 30 gramos.Se suelen expresar las cantidades en las mismas unidades.La razón entre 6 000 gramos y 30 gramos es 6 000 : 30 = 200 : l.

b) 3 centimetros cúbicos a 2 litros.La razón entre 3 centímetros cúbicos y 2 000 centímetros cúbicos es 3 : 2000.

e) 3 metros cuadrados a 6 centímetros cuadrados.La razón entre 30000 centimetros cuadrados y 6 centímetros cuadrados es 30000 cm" : 6 cm' = 5000:

3. Hallar el valor de .Y en las proporciones siguientes:

a) (3 - x) : (x + 1) = 2: 1.3 - x 2 1

3x +y

b) (x + 3): 10 = (3x - 2): 8.x + 3 3x - 2-10-=-8- y x = 2.

x-Ie) (.c - 1): (x + 1) = (2x -4): (x + 4). x + 1

2x -4

x + 4x' - 5x = O. xIx - 5) = O Y x = 0.5.

4. Hallar la cuarta proporcional de los conjuntos de números siguientes:

Sea. en cada caso. x la cuarta proporcional.

a) 2. 3. 6. Tendremos 2: 3 = 6: .v,2 6

Y .v = 9.3 x

25y .v = 2h) 4. -5. lO. Tendremos 4: -5 = 10: x

Tendremos a': ab = 2: x . a'x = Lab2h.\"= _....a

y

5. Hallar la tercera proporcional de los pares de números siguientes:

Sea. en cada caso. x la tercera proporcional.

a) 2. 3. Tendremos 2: 3 = 3: x y .v = 92.

8 8 8 32h) -2. Tendremos -2 -:x y x= 93 3 3

6. ¡.

7. Un

8. L

9. D

10. Si

PROP'

11. E

a

ba

ddite

fs,

ga

h]

pl

http://carlos2524.jimdo.com/

Page 145: Algebra Superior Murray R Spiegel

e las

5.

RAZON. PROPORCION y PROPORCIONALIDAD 137

6. Hallar la media proporcional entre 2 y 8.

Sea x la medida proporcional. Tendremos 2: x = x : 8. x2 16 Y x = ±4.

7. Un segmento de 30 centímetros se divide en dos partes cuyas longitudes están en la razón 2 : 3. Hallar las longi-tudes de ambas partes.

Sean las longitudes pedidas x y 30 - .r. Tendremos.v 2

330 - x = 18 cm.y .r = 12 cm,30 - .Y

8. Las edades actuales de dos hermanos son S y 8 años respectivamente. ¿Al cabo de cuántos años (x) sus edadesestarán en la razón 3 : 4?

Las edades al cabo de x años son S + x y 8 + x .

Por tanto. ¡'S + x): (8 + x) = 3: 4; 4(S + x) = 3(8 + x); de donde x = 4.

9. Dividir 2S3 en cuatro partes proporcionales a 2. S. 7. 9.

Sean las cuatro partes Lk ; Sk, 7k, 9k.

Tendremos 2k + Sk + 7k + 9k = 2S3 y k = 11. Luego las cuatro partes son 22, SS. 77. 99.

10. Si .v: y : z = 2: - S : 4 y .v - 3y + z = 63, hallar x, y, z .

Sean x = 2k. Y = - Sk ; z = 4k.

Sustituyendo estos valores en x - 3y + z = 63 se obtiene 2k - 3( - Sk) + 4k = 63 de donde k = 3.

Luego x = 2k = 6, Y = -5k = -IS, z = 4k = 12.

PROPORCIONALIDAD

11. Expresar mediante una ecuación en la que intervenga una constante de proporcionalidad k los enunciados siguientes:

al La longitud de una circunferencia e es directamente proporcional a su diámetro. Sol. e = kd.

h) El periodo T de la oscilación de un péndulo simple en un lugar determinado es directamente proporcionala la raíz cuadrada de su longitud. Sol. T = kJl

e) La energía radiante E em' da en la unidad de tiempo y por unidad de área por un radiador perfecto es direc-tamente proporcional a l' cuarta potencia de su temperatura absoluta T. Sol. E = kT'

d) El calor Q (calorí ) que se genera en un conductor de resistencia R (ohmios) por el que circula una corrientede intensidad 1 (arn rios) es directamente proporcional al cuadrado de la intensidad, a la resistencia del conduc-tor y al tiempo ( durante el cual pasa la corriente. Sol. Q = kR12{

e) La intensidad 1de una onda sonora es directamente proporcional al cuadrado de la frecuencia n, al cuadradode su amplitud r, a la velocidad del sonido v y a la densidad del medio en el que se propaga. Sol. 1 = kn2,2vd

f) La fuerza de atracción F entre dos masas m, Ym2 es directamente proporcional al producto de ambas e inver-. . . m,m2samente proporcional al cuadrado de la distancia r entre ellas. Sol. F = k ~

g) A temperatura constante, el volumen V de una masa dada de un gas perfecto es inversamente proporcionala la presión p a la cual está sometida. Sol. V = klp o bien p V = k

h) La resultante F de un sistema de fuerzas aplicadas a un sólido le comunica una aceleración a directamenteF

proporcional a dicha resultante e inversamente proporcional a la masa m del sólido en cuestión. Sol. a = k-m

RAZON. PROPORCION y PROPORCIONALIDAD 137

6. Hallar la media proporcional entre 2 y 8.

Sea x la medida proporcional. Tendremos 2: x = x : 8, X2 16 Y x = ±4.

7. Un segmento de 30 centímetros se divide en dos partes cuyas longitudes están en la razón 2 : 3. Hallar las longi­tudes de ambas partes.

Sean las longitudes pedidas x y 30 - x . Tendremos x

30 - x

2

3 y x = 12 cm, 30 - x = 18 cm.

8. Las edades actuales de dos hermanos son S y 8 años respectivamente. ¿Al cabo de cuántos años (x) sus edades estarán en la razón 3 : 4?

Las edades al cabo de x años son S + x y 8 + x .

Por tanto., f-S + x): (8 + x) = 3: 4; 4(S + x) = 3(8 + x); de donde x = 4.

9. Dividir 2S3 en cuatro partes proporcionales a 2, S, 7, 9.

Sean las cuatro partes 2k, Sk, 7k, 9k.

Tendremos 2k + Sk + 7k + 9k = 2S3 y k = 11. Luego las cuatro partes son 22, SS, 77. 99.

10. Si x: y : ~ = 2 : - S : 4 y x - 3y + = = 63, hallar x, y, z .

Sean x = 2k. Y = - Sk, z = 4k .

Sustituyendo estos valores en x - 3y + z = 63 se obtiene 2k - 3( - Sk) + 4k = 63 de donde k = 3.

Luego x = 2k = 6, Y = -5k = -IS, z = 4k = 12.

PROPORCIONALIDAD

11. Expresar mediante una ecuación en la que intervenga una constante de proporcionalidad k los enunciados siguientes:

a) La longitud de una circunferencia e es directamente proporcional a su diámetro. Sol. e = kd.

h) El periodo T de la oscilación de un péndulo simple en un lugar determinado es directamente proporcional

a la raíz cuadrada de su longitud. Sol. T = kJl

e) La energía radiante E em' da en la unidad de tiempo y por unidad de área por un radiador perfecto es direc-tamente proporcional a l' cuarta potencia de su temperatura absoluta T. Sol. E = k'T'

d) El calor Q (calorí ) que se genera en un conductor de resistencia R (ohmios) por el que circula una corriente de intensidad 1 (amplrios) es directamente proporcional al cuadrado de la intensidad, a la resistencia del conduc-tor y al tiempo j durante el cual pasa la corriente. Sol. Q = kR12¡

e) La intensidad 1 de una onda sonora es directamente proporcional al cuadrado de la frecuencia n, al cuadrado de su amplitud r, a la velocidad del sonido v y a la densidad del medio en el que se propaga. Sol. 1 = kn2,2vd

f) La fuerza de atracción F entre dos masas m, Y m2 es directamente proporcional al producto de ambas e inver­m,m2 samente proporcional al cuadrado de la distancia , entre ellas. Sol. F = k ~

g) A temperatura constante, el volumen V de una masa dada de un gas perfecto es inversamente proporcional a la presión p a la cual está sometida. Sol. V = k/p o bien pV = k

h) La resultante F de un sistema de fuerzas aplicadas a un sólido le comunica una aceleración a directamente F

proporcional a dicha resultante e inversamente proporcional a la masa m del sólido en cuestión . Sol. a = k-m

http://carlos2524.jimdo.com/

Page 146: Algebra Superior Murray R Spiegel

138 RAZON. PROPORCION y PROPORCIONALIDAD

12. La energía cinética E de un cuerpo es directamente proporcional a su masa m y al cuadrado de su velocidad v.Un cuerpo de 8 kilogramos animado de una velocidad de 4 metros por segundo posee una energía cinética de 2 ju-lios. Hallar IfIenergía cinética de un móvil de 6 000 kilogramos de masa a una velocidad de 88 metros por segundo.

Para hallar k: E = k Wv2 o k = ~ = _2_ = ~Wv2 8(42) 64 .

Luego la energía cinética del móvil esWv2 6ooo(8W

E = 64 = 64 = 726 000 rn-kp.

13. La presión p de una masa dada de un gas perfecto es inversamente proporcional al volumen V que ocupa y direc-tamente proporcional a la temperatura absoluta T. ¿A qué presión se deben someter 100 metros cúbicos de helioa 1 atmósfera de presión y 253 grados absolutos de temperatura para que se reduzcan a 50 metros cúbicos a unatemperatura de 313 grados absolutos?

T k __pV __ 1(100) __ 100.Para hallar k: p = k V o T 253 253

100 T 100 313Luego la presión pedida es p = - - = -(-) = 2.47 atmósferas.

253 V 253 50

Otro método. Designemos por 1 y 2 las condiciones inicial y final del gas respectivamente.

14. Sabiendo que 8 hombres tardan 12 días en poner a punto 16 máquinas. hallar el número de días que emplearán15 hombres en poner a punto 50 máquinas.

El número de días (x) varía directamente con el número de máquinas (y) e inversamente con el número dehombres (z).

Luegoky

x = --'-z

k = ~ = 12(8) = 6.Y 16

siendo

El número de días que se precisan es x = ~ = 6(50) = 20 días.z 15

PROBLEMAS PROPUESTOS15. Expresar las razones siguientes por medio de una fracción simplificada:

a) 40: 64 b) 4/5: 8/3

16. Hallar la razón entre las siguientes cantidades.

a) 20 metros a 40 centímetrosb) 8 litros a 4 centímetros cúbicos

e) 2 metros cua rados a 50 centímetros cuadrados.d) 6 litros yo centímetros cúbicos

17. Hallar el valor de x en las proporciones siguientes:

a). (x + 3) : (x - 2) = 3: 2b) (x + 4): 1 = (2 - x): 2

e) (x + 1): 4 = (x + 6) : 2x

d) (2x + 1): (x + 1) = 5x : (x + 4)

18. Hallar la cuarta proporcional de los siguientes conjuntos de números:

a) 3. 4, 12b) -2, 5, 6

e) a, b, ed) m + 2, m - 2, 3

19. Halla

a) 3

20. Halla

al 3

21. Si (x

22. Halla

23. Un senúme

24. Si x:

25. a) Sb) Se) S

26. La dicuadrcenso

27. La fUidradocidadfuera

28. Sabieque s

SOLUCIO

15. a) 5,

16. a) 51

17. a) 1:

18. a) l'

19. a) 2.

20. al :!

21. 7/3

22. 12, 11

23. 30, 4(

24. -8, I

25. a) 1:

26. 1600

27. 180 k

28. 3 han

http://carlos2524.jimdo.com/

Page 147: Algebra Superior Murray R Spiegel

d v.ju-do.

rec-eliouna

rán

de

RAZON, PROPORCION y PROPORCIONALIDAD 139

19. Hallar la tercera proporcional de los' pares' de números siguientes:

a) 3, 5 d) ab,.¡;;bb) -2,4 e) a, b

20. Hallar la media proporcional de los pares de números siguientes:

a) 3, 27 e) 3.fi Y 6.fi d) m + 2 Y m + Ib) -4,-8

21. Si (x + y) : (x - y) = 5: 2, hallar x: y.

22. Hallar dos números sabiendo que están en la razón 3 : 4 y que sumándoles 4 unidades su razón se convierte en 4 : 5.

23. Un Segmento de 120 centímetros se divide en tres partes cuyas longitudes son directamente proporcionales a losnúmeros 3, 4, 5. Hallar las longitudes de cada una de ellas.

24. Si x: y : z = 4 : - 3 : 2 y 2x + 4y - 3z = 20, hallar x, y, z.

25. a) Si x es directamente proporcional a y y para x = 8, Y = 5, hallar y cuando x = 20.b) Si x es directamente proporcional a y2 y para x = 4, Y = 3, hallar x cuando y = 6.e) Si x es directamente proporcional a y y para x = 8, Y = 3 hallar y cuando x = 2.

26. La distancia recorrida por un cuerpo que cae libremente, partiendo del reposo, es directamente proporcional alcuadrado del tiempo de descenso. Sabiendo que un cuerpo que cae desde 144 metros emplea 3 segundos en el des-censo, hallar el espacio que recorrerá en 10 segundos.

27. La fuerza ejercida por el viento sobre la vela de un barco es directamente proporcional al área de la vela y al cua-drado de la velocidad del viento. Sabiendo que la fuerza ejercida sobre 1 metro cuadrado de vela cuando la velo-cidad del viento es de 15 kilómetros por hora vale un kilopondio, hallar la que se ejercerá cuando la velocidadfuera de 45 kilómetros por hora y el área de la vela de 20 metros cuadrados.

28, Sabiendo que 2 hombres pueden transportar 6 metros cúbicos de tierra en 4 horas, hallar el número de hombresque se necesitan para transportar 18 metros cúbicos en 8 horas.

SOLUCIONES DE LOS PROBLEMAS PROPUESTOS

15. a) 5/8 b) 3/10 e) x/3y d) I/ab

16. a) 50: 1 b)

27e) 400: 1 d) 200: 1

17. al 12 b) -2 el 4, -3 d) 2, -2/3

18. a) 16 b) -1 e) befa d) 3(m - 2)/(m + 2l

19. a) 25/3 b) 8 el b2/a d) 1

20. al ±9 b) ± 4.fi e) ±6 d) ± Jm2 + 3m + 2

21. 7/3

22, 12, 16

23. 30,40, 50 cm

24. -8,6, -4

25. a) 12! b) 16 el 12

26. 1600 m

27. 180 kp

28. 3 hombres

http://carlos2524.jimdo.com/

Page 148: Algebra Superior Murray R Spiegel

CAPITULO 16

Progresiones

SUCESION. Es un conjunto ordenado de números que se deducen unos de otros mediante una regladefinida. Los números de la sucesión reciben el nombre de términos.

PROGRESIONES ARITMETICAS

A) Una progresión aritmética (p. a.) es una sucesión en la cual todos los términos, posteriores alprimero, se deducen del anterior añadiendo un número constante que se llama razón de la progresión.

Por ejemplo, 3, 7, 11, 15, 19, ... , es una progresión aritmética, ya que cada término se obtienesumando 4 unidades al anterior. En la progresión aritmética, 50, 45, 40, ... , la razón es 45 - 50= 40 - 45 = -5.

B) Fórmulas de las progresiones aritméticas.

1) El término enésimo, o el último: 1 = a + (n - I)d

2) La suma de los n primeros términos:n n

S = i(a + 1) = i[2a + (n - 1)dJ

siendo a = primer término; d = razón;n = número de términos; 1 = término enésimo o último término;S = suma de los n primeros términos.

Ejemplo. Consideremos la progresión aritmética 3, 7, 11, ... siendo a = 3 y d = 7 - 311 - 7 = 4. El sexto término es 1 = a + (n - l)d = 3 + (6 - 1)4 = 23.

La suma de los seis primeros términos es In 6 n 6

S = i(a + 1) = i(3 + 23) = 78 o S = i72Cl + (n - l)dJ = i2(3) + (6 - I)4J = 78.

PROGRESIONES GEOMETRICAS

A) Una progresión geométrica (p. g.) es una sucesión en la cual todos los términos, posteriores alprimero, se deducen del anterior multiplicándolo por una constante que se llama razón de la pro-gresión.

Por ejemplo, 5, 10, 20, 40, 80, ... , es una progresión geornétrica, ya que cada término se ob-1 1

tiene multiplicando por 2 el anterior. En la progresión geométrica, 9, -3, 1, -3' "9' ... , la ra-

. - 3 l - 1/3 1/9 1zon es 9 = _ 3 = -) - = - 1/3 = - 3.

140

B]

2010

PROG

ab

Cl'

PROC

ar

un

M E DI.die

es

mi

CAPITULO 16

Progresiones

SUCESION. Es un conjunto ordenado de números que se deducen unos de otros mediante una regla definida. Los números de la sucesión reciben el nombre de términos.

PROGRESIONES ARITMETICAS

A) Una progresión aritmética (p. a.) es una sucesión en la cual todos los términos, posteriores al primero, se deducen del anterior añadiendo un número constante que se llama razón de la progresión.

Por ejemplo, 3, 7, 11, 15, 19, ... , es una progresión aritmética, ya que cada término se obtiene sumando 4 unidades al anterior. En la progresión aritmética, 50, 45, 40, ... , la razón es 45 - 50 = 40 - 45 = -5.

B) Fórmulas de las progresiones aritméticas.

1) El término enésimo, o el último : 1 = a + (n - l)d

2) La suma de los n primeros términos: n n

S = i(a + 1) = i2a + (n - 1 )dJ

siendo a = primer término; d = razón; n = número de términos; 1 = término enésimo o último término ; S = suma de los n primeros términos.

Ejemplo. Consideremos la progresión aritmética 3, 7, 11, ... siendo el = 3 y d = 7 - 3 11 - 7 = 4. El sexto término es t = a + (n - l)d = 3 + (6 - 1)4 = 23.

La suma de los seis primeros términos es I n 6 n 6

S = "2(a + t) = "2(3 + 23) = 78 o S = i 7a + (n - l)dJ = i2(3) + (6 - I)4J = 78.

PROGRESIONES GEOMETRICAS

A) Una progresión geométrica (p. g.) es una sucesión en la cual todos los términos, posteriores al primero, se deducen del anterior multiplicándolo por una constante que se llama razón de la pro­gresión.

Por ejemplo, 5, 10, 20, 40, 80 .... , es una progresión geométrica, ya que cada término se ob-1 1

tiene multiplicando por 2 el anterior. En la progresión geométrica, 9. -3, 1, - 3 ' "9 ' . .. , la ra-

. - 3 1 - 1/3 1/9 1 zoo es 9 = _ 3 = - ) - = -) /3 = - 3 .

140

http://carlos2524.jimdo.com/

Page 149: Algebra Superior Murray R Spiegel

a regla

iores algresión.

obtiene5 - 50

7 - 3

iores alla pro-

o se ob-

. , la ra-

PROGRESIONES 141

B) Fórmulas de las progresiones geométricas

1) El término enésimo o último término: 1= ar :'

2) La suma de los n primeros términos: s = a(r" - 1) rl - ar - 1 -~'

,. =F 1

siendo a = primer término; r = razón; n = número de términos;I = término enésimo o último término; S = suma de los n primeros términos.

Ejemplo.; Consideremos la progresión geométrica 5, 10, 20, ... siendo a = 5 Y r = 150=

~~ = 2. El séptimo término es I = ar" - I = 5(27- 1) = 5(26) = 320.

L d l . . .. S a(r" - 1) 5(27 - 1) -a suma e os siete pnmeros termmos es = = 2 = 63,.r - 1 - 1

PROGRESIONES GEOMETRICAS INDEFINIDAS

La suma (S",) de los términos de una progresión geométrica indefinida de razón r, en valorabsoluto menor que la unidad, viene dada por

aS",= -1--'- r siendo Irl < 1.

E· l Consid l . . ... d finid 1 1 1 1 . dJemp o. onsi eremos a progresion geometnca In e ni a - 2 + "4 - "8 + . . . sien o

1a = 1 Y r = -2' Su suma es S",

1 1 21 - (-1/2) 3/2 T

a1 - r

PROGRESIONES ARMONICAS

Una progresión armónica es una sucesión de números cuyos recíprocos forman una progresiónaritmética.

P' 111111 ., .. 246810or ejemp o, 2' "4' "6' "8' 10' ... es una progresion armomca, ya que , , , , , ... es

una progresión aritmética

MEDIAS. os términos de una progresión comprendidos entre dos dados reciben el nombre de me-dias de aquéllos.

Por ejemplo, en la progresión aritmética, 3, 5, 7, 9, 11, ... , la media aritmética entre 3 y 7es 5 y cuatro medias entre 3 y 13 son 5, 7, 9, 11.

En la progresión geométrica 2, -4, 8, -16, ... , dos medias entre 2 y -16 son -4,8.

E l .. ,. 1 1 l di .. 1 1 1n a progresión armornca 2 ' "3 ' "4 ' "5 ' "6' ... , a me la armomca entre 2 y "4 es "3 ' y tres

di 1 1 1 1 1me las entre 2 y "6 son "3 ' "4 ' "5 .

PROG RESION ES 141

B) Fórmulas de las progresiones geométricas

1) El término enésimo o último término: 1= ar"-l

2) La suma de los n primeros términos: s = a(r" - 1) rl - a r - 1 = -;--=-¡ , ,. =F 1

siendo a = primer término; r = razón; n = número de términos; I = término enésimo o último término; S = suma de los n primeros términos.

Ejemplo. ~ Consideremos la progresión geométrica 5, 10, 20, ... siendo a = 5 Y r = 150 =

~~ = 2. El séptimo término es I = ar" - 1 = 5(27- 1) = 5(26

) = 320.

L d l . . .. S a(r" - 1) 5(27 - 1) -

a suma e os siete pnmeros termlnos es = = 2 = 63,. r - 1 - 1

PROGRESIONES GEOMETRICAS INDEFINIDAS

La suma (S",) de los términos de una progresión geométrica indefinida de razón r, en valor absoluto menor que la unidad, viene dada por

a S"'=-I--' - r siendo Irl < 1.

E· 1 C'd l . . ., . d fi'd 1 1 1 1 . d Jemp o. onsl eremos a progreslOn geometnca In e ni a - 2 + "4 - "8 + . . . sien o

1 a = 1 Y r = -2' Su suma es S",

1 1 2 1 - (-1/2) 3/2 T

PROGRESIONES ARMONICAS

Una progresión armónica es una sucesión de números cuyos recíprocos forman una progresión aritmética.

P . l 1 1 1 1 1 . . . . 2 4 6 8 10 or eJemp o, 2' "4' "6' "8' 10' ... es una progreslOn armOnlca, ya que , , , , , .. . es

una progresión aritmética

MEDIAS. os términos de una progresión comprendidos entre dos dados reciben el nombre de me-dias de aquéllos.

Por ejemplo, en la progresión aritmética, 3, 5, 7, 9, 11, . . . , la media aritmética entre 3 y 7 es 5 y cuatro medias entre 3 y 13 son 5, 7, 9, 11.

En la progresión geométrica 2, -4, 8, -16, ... , dos medias entre 2 y -16 son -4,8.

E l . . . . 1 1 l d' . . 1 1 1 n a progreslOn armonlca 2 ' "3 ' ¡ , 5 ' "6' ... , a me la armonlca entre 2 y "4 es "3 ' y tres

d' 1 1 1 1 me las entre 2 y "6 son "3 ' ¡ , 5 .

http://carlos2524.jimdo.com/

Page 150: Algebra Superior Murray R Spiegel

142

PROGRESIONES ARITMETlCAS

PROGRESIONES

PROBLEMAS RESUELTOS

l. Determinar cuáles de las siguientes ecuaciones son progresiones aritméticas:

a) 1, 6, 11, 16....

I 5 7b) 3' 1, 3' 3' ...e) 4, -1, -6, -11,

d) 9, 12, 16,

I I 1e) 2' 3' 4'f) 7. 9 + 3p, 11 + 6p,

es una p. a. ya que 6 - 1 = 11 - 6 = 16 - 1I = 5. (d = 5)

1 5 7 5 2 21 - 3 = 3 - 1 = 3 - 3 = 3' (d = 3)-1-4= -6- (-1)= -11- (-6)= -5. (d = -5)

12 - 9 + 16 - 12.

I 1 I 13-2+4-3'

es una p. a. ya que

es una p. a. ya que

no es una p. a. ya que

no es una p. a. ya que

es una p. a. con d = 2 + 3p.

2. Deducir la fórmula S = ~(a + 1) de la suma de los n primeros términos de una progresión aritmética.

La suma de los n primeros términos de una progresión aritmética se puede escribir:

o

S = a + (a + d) + (a + 2d) + + 1

S= 1+(I-d)+(I-2d)+ +a

(n términos)

(n términos)

Sumando,

en la que se ha escrito la suma en orden inverso.

2S = (a + 1) + (a + 1) + (a + 1) + ... + (a + 1) para n términos

2S = n(a + 1)n

S = 2(a + 1)y

3. Hallar el dieciseisavo término de la p. a.: 4. 7, 10. . ..

Tenemos a = 4. n = 16. d = 7 -4 = 10 - 7 = 3. y 1 = a + (n - l)d = 4 + (16 - 1)3 = 49.

4. Hallar la suma de los 12 primeros términos de la p. a.: 3. 8. 13,

Tenemos (J = 3, d = 8 - 3 = 13 - 8 = 5. n = 12. Y

Otro método:

n 12S = -[20 + (n - I)d] = -[2(3) + (12 - 1)5] = 366.2 2

I=a+ (n-l(d=3+ (12-1)5=~

n 12S = 2(a + 1) = 2(3 + 58) = 366.

5. Hallar el cuarentavo término y la suma de los 40 primeros términos de la p. a.: lO, 8. 6, ...

Tenemos d = 8 - 10 = 6 - 8 = -2. a = 10. n = 40.

Luego 1 = a + (n - l)d = 10 + (40 - 1)(-2) = -68n 40

S = -(a + 1) = -(10 - 68) = - 1 160.2 2

y

6. ¿(

7. H

8. H

9. Hel

n'

10. ¿(

11. e

12. e

13. Hp

14. H

142 PROGRESIONES

PROBLEMAS RESUELTOS

PROGRESIONES ARITMETlCAS

1. Detenninar cuáles de las siguientes ecuaciones son progresiones aritméticas:

a) 1, 6, 11 , 16,

b) 1 5 7 3' 1, 3' 3'

e) 4, -1 , - 6, -11,

d) 9, 12, 16,

1 e) 2'3'4 ' J) 7, 9 + 3p, 11 + 6p,

n

es una p. a. ya que

es una p. a. ya que

es una p. a. ya que

no es una p. a. ya que

6 - 1 = 11 - 6 = 16 - 11 = 5. (d = 5)

1 5 7 5 2 2 1- 3 =3- 1 =3-3=3' W=~

-1 - 4 = - 6 - (-1)= -11 - (-6)= -5. (d = -5 )

12 - 9 j 16 - 12.

1 1 1 1 no es una p. a. ya que 3 - 2 j 4 - 3 .

es una p. a. con d = 2 + 3p.

2. Deducir la fórmula S = 2(a + 1) de la suma de los n primeros términos de una progresión aritmética.

La suma de los n primeros términos de una progresión aritmética se puede escribir:

S = a + (a + d) + (a + 2d) + .. . + 1

o S= 1+(I-d)+(I-2d)+ .. . +a

en la que se ha escrito la suma en orden inverso.

(n términos)

(n términos)

Sumando, 2S = (a + 1) + (a + 1) + (a + 1) + .. . + (a + 1)

2S = n(a + 1) y n

S = 2(a + 1)

3. Hallar el dieciseisavo término de la p . a.: 4. 7. 10, . . .

para n términos

Tenemos a = 4, n = 16, d = 7 - 4 = 10 - 7 = 3, y 1 = a + (n - l)d = 4 + (16 - 1)3 = 49 .

4. Ha llar la suma de los 12 primeros términos de la p. a.: 3. 8, 13,

Tenemos {/ = 3, d = 8 - 3 = 13 - 8 = 5. n = 12, Y

aIro mélodo:

n 12 S = -[2a + (n - I)a'] = - [2(3) + (12 - 1 )5J = 366.

2 2

I=a+ (n-I(d= 3 + (12-1)5=5~ n 12

S = 2(a + 1) = "2(3 + 58) = 366.

S. Hallar el cuarentavo término y la suma de los 40 primeros términos de la p. a.: 10. 8. 6, ...

Tenemos d = 8 - 10 = 6 - 8 = -2, a = lO. n = 40.

Luego 1 = a + (n - 1)d = 10 + (40 - 1)( - 2) = - 68 y n 40

S = 2«/ + /) = "2( 10 - 68) = - 1 160.

http://carlos2524.jimdo.com/

Page 151: Algebra Superior Murray R Spiegel

-5)

PROGRESIONES 143

6. ¿Qué término de la progresión 5. 14, 23, es 239'!

I = a + (n - l sd, 239 = 5 + (/1 - 1)9. 9/1 = 243 es el término /1 = 27.

7. Hallar la suma de los primeros enteros positivos múltiplos de 7.

La sucesión es 7. 14. 21. ... una p. a. en la cual a = 7. d = 7. 11= 100.

Luego11 100

S = 2[2a + (/1 - I)d] = 2[2(7) + (100 - 1)7] = 35350.

8. Hallar cuántos enteros consecutivos a partir de 10 se deben tomar para que su suma valga 2035.

La sucesión es 10, 11. 12. ... una p. a. en la cual a = lü, d = 1, S = 2035.

11Aplicando S = 2[2a + (n - I}d],

n2035 = 2[20 + (n - 1)1],

n2035 = 2(n + 19),se obtiene

n2 + 19n - 4070 = O, (n - 55) (n + 74) = O, n = 55. -74. Luego hay que tomar 55 enteros.

9, Hallar el tiempo que se empleará en saldar una deuda de 880 pesetas pagando 25 pesetas el primer mes, 27 pesetasel segundo. 29 pesetas el tercero, etc.

Den

S = 2[2a + (n - I)d],11

880 = 2[2(25) + (n - 1)2], 880 = 24n + n2,se obtiene

n2 + 24/1 - 880 = O, (n - 20) (n + 44) = O, n = 20, -44. La deuda se salda en 20 meses.

10. ¿Cuántos términos de la p. a. 24, 22, 20, ... se necesitan para que su suma sea 150? Escribir los términos.

n150 = 2[48 + (n - 1)(-2)], n2 - 25n + 150 = O, (n - 10)(n - 15) = O, n = 10,15.

Para n = lO:Para /1 = 15:

24, 22, 20, 18. 16, 14, 12, lO, 8, 6.24, 22, 20. 18, 16, 14, 12, lO, 8, 6, 4, 2, O, -:-2, -4.

11. Determinar la p. a. en lo que la suma de los n primeros términos es n2 + 2n.

Término enésimo = suma de n términos - suma de n - 1 términos

= n2 + 2n - [In - I f + 2(n - 1)] = 2n + 1. Luego la p. a. es 3, 5, 7, 9, ...

12. Demostrar que la suma de n enteros impares consecutivos a partir del I es igual a n2

Tenemos que hallar la suma de los n primeros términos de la p. a. 1, 3, 5,

n nTenemos a = 1, d = 2. /1 = n y S = '2[2a + (n - 1}d] = '2[2(1) + (n - 1)2] = n2•

13. Hallar tres números en p. a. sabiendo que la suma del primero y el tercero es 12, y que el producto del primeropor el segundo es 24.

Sean los números en p. a. (a - d), a, (a + di. Luego (a - d) + (a + d) = 12 de donde a = 6.

Por tanto, (a - dja = 24. (6~~d¡;-~ = 2. Luego los números son 4, 6, 8.

14. Hallar tres números en p. a. cuya suma es 21 y cuyo producto es 280.

Sean los números (a - di, a, (a + d). Tendremos (a - d) + a + (a + d) = 21 o sea a = 7.

Por otra parte (a - d) (a) (a + d) = 280. a(a2 - d2) = 7(49 - d') = 280 y d = ± 3.

Los números son 4, 7, 10 o 10. 7, 11.

''1I~..

PROGRESIONES 143

6. ¿Qué término de la progresión 5. 14, 23, es 239'!

I = {/ + (n - 1 Id, 239 = 5 + (11 - 1 )9. 911 = 243 es el término 11 = 27.

7. Hallar la suma de los primeros enteros positivos múltiplos de 7.

La sucesión es 7. 14. 21 . . . . una p. a. en la cual {/ = 7. d = 7. 11 = 100.

Luego 11 100

S = 2[2{/ + (11 - I)d) = 2[2(7) + (100 - 1)7] = 35350.

8. Ha llar cuántos enteros consecutivos a partir de 10 se deben tomar para que su suma valga 2035.

La sucesión es 10, 11, 12. ... una p. a. en la cual a = 10, d = 1, S = 2035.

11 Aplicando S = 2[2a + (n - I)d), se obtiene

n 2035 = 2[20 + (n - 1)1],

n 2035 = 2(n + 19),

n 2 + 19n - 4070 = O, (n - 55) (n + 74) = O, n = 55, -74. Luego hay que tomar 55 enteros.

9, Hallar el tiempo que se empleará en saldar una deuda de 880 pesetas pagando 25 pesetas el primer mes, 27 pesetas el segundo. 29 pesetas el tercero, etc.

De 11

S = z[2a + (11 - I)d), se obtiene 11

880 = 2[2(25) + (n - 1)2], 880 = 24n + n2,

n 2 + 2411 - 880 = O, (n - 20) (n + 44) = O, n = 20, -44. La deuda se salda en 20 meses.

10. ¿Cuántos términos de la p. a. 24, 22, 20, ... se necesitan para que su suma sea 150? Escribir los términos.

n 150 = 2[48 + (n - 1)(-2)], n2

- 25n + 150 = O, (n - 10) (n - 15) = O, n = 10,15.

Para n = 10: 24, 22, 20, 18. 16, 14, 12, lO, 8, 6. Para 11 = 15: 24.22.20. 18, 16. 14, 12, 10,8, 6,4, 2, O. -:-2. -4.

11. Determinar la p. a. en lo que la suma de los n primeros términos es n2 + 2n.

Término enésimo = suma de n términos - suma de n - 1 términos

= n 2 + 2n - [In - 1 f + 2(n - 1)] = 2n + 1. Luego la p. a . es 3, 5, 7, 9, ...

12. Demostrar que la suma de n enteros impares consecutivos a partir del 1 es igual a n2

Tenemos que hallar la suma de los n primeros términos de la p. a . 1, 3, 5,

n n Tenemos a = 1, d = 2, 11 = n y S = 2[2a + (n - I)d) = 2[2(1) + (n - 1 )2] = n 2 •

13. Hallar tres números en p. a. sabiendo que la suma del primero y el tercero es 12. y que el producto del primero por el segundo es 24.

Sean los números en p. a . (a - dI, a, (a + dI. L~ (a - d) + (a + d) = 12 de donde a = 6.

Por tanto. (a - dja = 24. (6~¡;-ctOnde d = 2. Luego los números son 4, 6, 8.

14. Hallar tres números en p. a. cuya suma es 21 y cuyo producto es 280.

Sean los números (a - dI, a , (a + dI. Tendremos (a - d) + a + (a + d) = 21 o sea a = 7.

Por otra parte (a - d) (a) (a + d) = 280. a(a2 - d2

) = 7(49 - J2) = 280 y d = ±3.

Los números son 4, 7, 10 o 10, 7, 11.

http://carlos2524.jimdo.com/

Page 152: Algebra Superior Murray R Spiegel

144 PROGRESIONES

15. Tres números están en la relación 2: 5 : 7. Hallar dichos números, sabiendo que si se resta 7 del segundo los nú-meros forman una p. a.

Sean los números 2x, 5x, 7x. Los números formando una p. a. son 2x, (5x - 7), 7x.

Por tanto (5x - 7) - 2x = 7x - (5x - 7) de donde x = 14. Luego los números son 28. 70. 98.

16. Hallar la suma de todos los enteros comprendidos entre 100 y 800 que sean múltiples de 3.

La p. a. es 102, 105, 108, ... , 798. Luego 1 = a + (n - l)d, 798 = 102 + (n - 1)3. n = 233,n 233

y S = 2(a + 1) = 2(102 + 798) = 104 850.

17. Sobre una superficie horizontal se levanta una rampa de pendiente uniforme por medio de 10 soportes igualmenteespaciados. Las alturas de los soportes mayor y menor son 42.5 y 2 metros respectivamente. Hallar la altura decada uno de los soportes.

De I=a+(n-I)d tenemos 42t=2+(I0-1)d y d=4tm.

Luego las alturas son 2, 61, 11, 15t. 20. 24t. 29. 3J!, 38, 42t metros respectivamente.

18. Un cuerpo cae libremente. partiendo del reposo, y recorre 16 metros durante el primer segundo, 48 metros en elsegundo, 80 metros en el tercero. etc. Hallar la distancia que recorre durante el quinceavo segundo y la distanciatotal que recorre en 15 segundos, partiendo del reposo.

Tenemos d = 48 - 16 = 80 - 48 = 32.

Durante el quinceavo segundo recorre una distancia 1 = a + (n - l)d = 16 + (15 - 1)32 = 464 m.... n 15

La distancia total recornda en 15 segundos es S = 2(a + 1) = 2(16 + 464) = 3600 m.

19. Se colocan 8 bolas en línea recta separadas entre sí una distancia de 6 metros. A 6 metros de la primera, al otrolado de las bolas, está situada una persona con una cesta que va andando por la fila recogiéndolas de una en una eintroduciéndolas en la misma. Sabiendo que empieza a recogerlas partiendo de la posición en que inicialmente seencuentra, hallar la distancia total que recorrerá hasta que termine la operación

11 8Tenemos a = 2· 6 = 12 m y 1 = 2(6' 8) = 96 m. Luego S = 2(a + /) = 2(12 + 96) = 432 m.

20. Demostrar que si los lados de un triángulo rectángulo están en p. a. su relación es 3 : 4 : 5.

Sean los lados (a - di. a, (a + di, siendo la hipotenusa (a + di.

Tendremos (a + d)2 = a2 + (a - d)2 o sea a = 4d. Luego (a - d) : a : (a + d) = 3d: 4d: Sd = 3 : 4 : 5.

MEDIAS ARITMETlCAS

21. Deducir la fórmula de la media aritmética (x) entre dos números p y q.

Como p, x, q están en p. a., tenemos x - p = q - x o seá x = t(P + q).

22. Hallar la media aritmética de los pares de números siguientes:

a) 4 y 56.. ,. 4 + 56

Media antmetica = --2- = 30.

b) 3fi y -6fi.

---------------. .. 3fi + (-6fi) 3fi

Media antmenca = = ---2 2

e) II + 5d y a - 3d.(a + 5d) + (a - 3d)

Media aritmética = 2 = a + d.

23. Situ,

24. Situtant

PROGRI

25. Det(

a)

b)

e)

d)

e)

f)

26. Ded

1)

2)

27. Hall

28. Hal

144 PROGRESIONES

IS. Tres números están en la relación 2: 5 : 7. Hallar dichos números, sabiendo que si se resta 7 del segundo los nú· meros forman una p. a .

Sean los números 2x, 5x, 7x. Los números formando una p. a. son 2x, (5x - 7), 7x.

Por tanto (5x - 7) - 2x = 7x - (5x - 7) de donde x = 14. Luego los números son 28. 70. 98.

16. Hallar la suma de todos los enteros comprendidos entre 100 y 800 que sean múltiplos de 3.

La p. a. es 102, 105, 108, . .. , 798. Luego 1 = a + (n - 1)d, 798 = 102 + (n - 1)3. n = 233 , n 233

y S = 2(a + 1) = 2(\02 + 798) = 104 850.

17. Sobre una superficie horizontal se levanta una rampa de pendiente uniforme por medio de 10 soportes igualmente espaciados. Las alturas de los soportes mayor y menor son 42.5 y 2 metros respectivamente. Hallar la altura de cada uno de los soportes.

De I=a+(n - I)d tenemos 421=2+(\0-1)d y d=41m.

Luego las alturas son 2, 61, 11 , 151. 20. 241. 29. 3Jj-, 38, 421 metros respectivamente.

18. Un cuerpo cae libremente. partiendo del reposo, y recorre 16 metros durante el primer segundo, 48 metros en el segundo, 80 metros en el tercero. etc. Hallar la distancia que recorre durante el quinceavo segundo y la distancia total que recorre en 15 segundos, partiendo del reposo.

Tenemos d = 48 - 16 = 80 - 48 = 32.

Durante el quinceavo segundo recorre una distancia 1 = a + (n - l)d = 16 + (\5 - 1 )32 = 464 m. ... n 15

La distanCia total recornda en 15 segundos es S = ita + 1) = 2(16 + 464) = 3600 m.

19. Se colocan 8 bolas en línea recta separadas entre sí una distancia de 6 metros. A 6 metros de la primera, al otro lado de las bolas, está situada una persona con una cesta que va andando por la fila recogiéndolas de una en una e introduciéndolas en la misma. Sabiendo que empieza a recogerlas partiendo de la posición en que inicialmente se encuentra, hallar la distancia total que recorrerá hasta que termine la operación

Il 8 Tenemos a = 2· 6 = 12 m y 1 = 2(6' 8) = 96 m. Luego S = 2(a + 1) = 2(12 + 96) = 432 m.

20. Demostrar que si los lados de un triángulo rectángulo están en p. a. su relación es 3 : 4 : 5.

Sean los lados (a - dI. a, (a + d), siendo la hipotenusa (a + dI.

Tendremos (a + d)2 = a2 + (a - d)2 o sea a = 4d. Luego (a - d) : a : (a + d) = 3d : 4d : Sd = 3 : 4 : 5.

MEDIAS ARITMETICAS

21. Deducir la fórmula de la media aritmética (x) entre dos números p y q.

Como p. x, q están en p. a ., tenemos x - p = q - x o seá x = 1(P + q).

22. Hallar la media aritmética de los pares de números siguientes:

a) 4 y 56. . .. 4 + 56

Media antmellca = - -2- = 30. -------------. .. 3fi + (-6fi) 3fi Media antmetlca = = - - -

2 2 b) 3fi y - 6fi .

e) a + 5d y a - 3d. (a + 5d) + (a - 3d)

Media aritmética = 2 = a + d.

http://carlos2524.jimdo.com/

Page 153: Algebra Superior Murray R Spiegel

nú-

98.

233,

entea de

n el, ncia

otrona ete se

4: S.

PROGRESIONES 145

23. Situar 5 medias aritméticas entre 8 y 26.

Tenemos que hallar una p. a. de la forma 8, -, -, -, -, -,26; por tanto, a = 8, 1=26 Y n = 7.

Luego I = a + (n - l)d, 26 = 8 + (7 - l)d, d = 3.

Las cinco medias aritméticas son 11, 14, 17, 20, 23.

24. Situar entre 1 y 36 un número de medias aritméticas de tal forma que la suma de la progresión aritmética resul-tante sea 148

s = in(a + /), 148 = in(1 + 36), 37n = 296 Y n = 8.

I = a + (n - 1Id, 36 = 1 + (8 - 1Id, 7d = 35 Y d = 5.

La progresión aritmética completa es 1, 6, 11, 16, 21, 26, 31, 36.

PROGRESIONES GEOMETRICAS

25. Determinar cuáles de las sucesiones siguientes son progresiones aritméticas:

6 12a) 3, 6. 12, ... es una p. g. ya que 3 ="6 = 2. (1' = 2)

12 9 3 3b) 16, 12, 9, ... es una p. g. ya que

16 12 4(1' =;¡-)

-l. 3. -9, 3 -9e) ... es una p. g. ya que -=-= -3. (1' = -3)-1 3

d) 1. 4. 9. 4 9... no es una p. g. ya que - =fo-.

1 4

1 1 2 1/3 2/9 2 2e) 2' 3' 9' es una p. g. ya que

1/2 1/3 3 (1' = 3)

1 1 I/h 1/2/¡3 1 1f) 2/¡. ¡;. 2/¡3' es una p. g. ya que2h I//¡ 2/¡2 . (1' = 2h2)

26. Deducir la fórmulaa(r" - 1) .. .

S = --;:--=-1" de la suma de los n pnmeros termmos de una progresión geométrica.

La suma de los 1/ primeros términos de una progresión geométrica se puede escribir:

1) S = a + al' + ar2 + ar' + + ar :' (n términos). Multiplicando 1) por 1', se obtiene

2) rS = al' + ar' + ar3 + + ar">' + ar" (n términos).

Restando 1) de 2), rS - S = ar" - a.air" - 1)

S=---.r - 1

(1' - l)S = a(,ft - 1) Y

27. Hallar el octavo término y la suma de los ocho primeros términos de la progresión 4. 8, 16.

Tenemos a = 4. r = 8/4 = 16/8 = 2. n = 8.

El octavo término es 1= ar"-' = 4(2)8-' = 4(27) = 4(128) = 512.

a(r" - 1) 4(28 - 1) 4(256 - 1)La suma de los ocho primeros términos es S = -;=-I = 2 _ 1 = --1 -- = 1 020.

28. Hallar el séptimo término y la suma de los siete primeros términos de la-progresién . - ~.

Tenemos a = 9-6 4 2

r=~-=~·=9 -6 3

2 64luego el séptimo término es 1= {/Ift-' = 9(- __)7-'

3 = 81'

t :

,.~' ..http://carlos2524.jimdo.com/

Page 154: Algebra Superior Murray R Spiegel

146 PROGRESIONES

a(r" - 1) a(1 - r") 9[1 - (-2/3)7] 9[1 - (-128/2187)] 463S=---=---= = . =-

r-l I-r 1-(-2/3) 5/3 81

29. El segundo término de una p. g. es 3 y el quinto es 81/8. Hallar el octavo.

81Quinto término = ar" = 8 '

ar

81/8 3 27 3-3-' r =8 y r=2'Segundo término = ar = 3. Luego

81 27 2187Por tanto, el octavo término = ar? = (ar4)r3 = 8(8)= M'

30. Hallar tres números en p. g. cuya suma es 26 y cuyo producto es 216.

Sean los números en p. g. alr, a, aro Se tiene (a/r) (a) (ar) = 216, a3 = 216 y a = 6.

Por otra parte alr + a + ar = 26, 6/r + 6 + 6r = 26, 6r2 - 20r + 6 = O de donde r = 1/3, 3.

Para r = 1/3, los números son 18, 6, 2; para r = 3, los números son 2, 6, 18.

31. El primer término de una p. g. es 375 y el cuarto es 192. Hallar la razón y la suma de los cuatro primeros términos.

Primer término = a = 375, cuarto término = ar3 = 192. Luego 375r3 = 192, r3 = 64/125 de donde r = 4/5.

S = a(l - r") = 375[1 - (4/5)4] = 1 107.1 - r 1 - 4/5

La suma de los cuatro primeros términos es

32. El primer término de una p. g. es 160 y la razón es 3/2. Hallar los términos consecutivos que se deben tomar paraque su suma sea 2 110.

S = a(r" - 1) 2110 = 160[(3/2' - 1] (~, _ 1 = ~ (~ = 243 = (~)S n = 5.r - 1 ' 3/2 - 1 '2 32' 2' 32 2'

Los cinco términos consecutivos son 160, 240, 360, 540, 810.

33. Una progresión geométrica de razón positiva consta de cuatro términos. Sabiendo que la suma de los dos primeroses 8 y que la correspondiente de los dos últimos es 72, determinar dicha progresión.

Los cuatro términos son a, ar, ar", ar". Se tiene a + ar = 8 Y ar2 + ar3 = 72.

ar2 + ar3 ar2 (1 + r) 2 72Luego = = r = - = 9 de donde r = 3.

a + ar a(1 + r) 8'

Como a + ar = 8, a = 2 y la progresión es 2, 6, 18, 54.

34. Demostrar que x, x + 3. x + 6 no pueden formar una progresión geométrica.

x+3 x+6Si x, x + 3. x + 6 forman p. g. se tiene r = -- = --, x2 + 6x + 9 = x2 + 6x o sea 9 = O.

x x + 3

Como esta igualdad nunca puede ser cierta, x, x + 3, x + 6 no pueden estar en p. g.

35. Un muchacho gana una peseta el primer día, dos pesetas el segundo, cuatro el tercero, ocho el cuarto, etc. Hallarel número de pesetas que ganará al cabo de 12 días.

Tenemos a = 1, r = 2, n = 12.

a(r" - 1)S = --- = 212 - 1 = 4096 - 1 = 4095 cént = 40,95 pts.

r - 1

36. Se (tanl

de t

37. Delse e:en 1

con

38. Se itteré

a)

b)

e)

39. Hallfom

por

MEDIAS

40. Ded

http://carlos2524.jimdo.com/

Page 155: Algebra Superior Murray R Spiegel

,3.

inos.

4/5.

r para

meros

9 = o.

aliar

PROGRESIONES 147

36. Se estima que la población de una cierta ciudad se incrementará en un 10% anual durante cuatro años. ¿En quétanto por ciento aumentará la población después de los cuatro años?

Sea p la población inicial. Después de un año la población es 1,IOp; después de dos años, (l,10)2p; despuésde tres años, (I,1O)3p; y después de cuatro años, (I,1O)4p = 1,46p. Por tanto, la población aumentará en un 46 %.

37. De un depósito, que contiene 240 litros de alcohol, se extraen 60 litros y se sustituyen por agua. A continuaciónse extraen 60 litros de la mezcla y se les remplazan por agua, etc. Hallar el número de litros de alcohol que habráen el depósito después de haber efectuado 5 extracciones de 60 litros.

Después de la primera extracción quedan en el depósito 240 - 60 = 180 litros de alcohol.

240 - 60 3Después de la segunda quedan 180( 240 ) = 180(4) litros de alcohol. etc.

El número de litros de alcohol que quedan en el depósito después de cada extracción forman una p. g., 180,3 32, 3

180(4)' 180(4)"" SIendo a = 180 y r = 4 .

1 34 .Después de la quinta extracción (n = 5): 1= ar"" = 180(4) = 57 htros de alcohol, que son los que

contiene el depósito.

38, Se invierten 400 000 pesetas a un 6% anual. Calcular el capital que se habrá formado al cabo de cinco años si el in-terés es compuesto a) anual, b) semestral. e) trimestral.

Sea P = capital inicial, i = rédito, en tantos por ciento, por periodo de tiempo.

S = Capital acumulado al cabo de n periodos.

Al final del primer periodo: Interés =. Pi, capital = P + Pi = P(I + i).

Al final del segundo periodo: Interés = P(I + i)i, capital = P(I + i) + P(I + i)i = P(I + i)2.

El capital acumulado al cabo de n periodos será, S = P(I + ir.

a) Como se cobran los intereses una vez por año, n = 5 e i = 0,06.

S = P(I + ir = 400 000(1 + 0,06)5 = 400 000(1,3382) = 535280 pts.

b) Como se cobran los intereses dos veces por año, n = 2(5) = lO e i = í(O,06) = 0,03.

S = P(I + ir = 400 000(1 + 0,03)10 = 400 000(1,3439) = 537560 pts.

e) Como se cobran los intereses cuatro veces por año, n = 4(5) = 20 e i = *(0,06) = 0,015.

S = P(I + ir = 400 000(1 + 0,015)20 = 400 000(1,3469) = 538760 pts.

39. Hallar el capital (P) que se debe invertir al 4% de interés compuesto semestral para que al cabo de 3,5 años se trans-forme en un capital (S) de 500 000 pesetas.

Como se cobran intereses dos veces por año, n = 2(3,5) = 7 (periodos) y el rédito, en tantos por ciento ypor periodo, es i = í(O,04) = 0,02.

Por tanto, S = P(I + i)n, de donde P = S(I + ¡¡-n = 500 000(1 + 0,02)-7 = 500 000(0,87056) = 435 280 pts.

MEDIAS GEOMETRICAS

40. Deducir la fórmula de la media geométrica, G, entre dos números p y q.

G qComo p, G, q están en progresión geométrica, se tiene p = G' G2 = pq

Se suele tomar G = ¡pq si p y q son positivos

y G = - ¡pq si p y q son negativos.

PROGRESIONES 147

36. Se estima que la población de una cierta ciudad se incrementará en un 10% anual durante cuatro años. ¿En qué tanto por ciento aumentará la población después de los cuatro años ?

Sea p la población inicial. Después de un año la población es 1,lOp ; después de dos años, (l,1O)2p ; después de tres años, (1 ,1O)3p ; y después de cuatro años, (l ,1O)4p = 1,46p. Por tanto, la población aumentará en un 46 %.

37. De un depósito, que contiene 240 litros de alcohol. se extraen 60 litros y se sustituyen por agua. A continuación se extraen 60 litros de la mezcla y se les remplazan por agua, etc. Hallar el número de litros de alcohol que habrá en el depósito después de haber efectuado 5 extracciones de 60 litros.

Después de la primera extracción quedan en el depósito 240 - 60 = 180 litros de alcohol.

240 - 60 3. Después de la segunda quedan 180( 240 ) = 180(4) litros de alcohol , etc.

El número de litros de alcohol que quedan en el depósito después de cada extracción forman una p. g., 180, 3 3. 3

180(4)' 180(4)2, . .. SIendo a = 180 Y r = 4 '

Después de la quinta extracción (n = 5): 1= ar" - 1 = 180(~)4 = 57 litros de alcohol, que son los que

contiene el depósito.

38. Se invierten 400 000 pesetas a un 6% anual. Calcular el capital que se habrá formado al cabo de cinco años si el in­terés es compuesto a) anual, b) semestral, e) trimestral.

Sea P = capital inicial, i = rédito, en tantos por ciento, por periodo de tiempo.

S = Capital acumulado al cabo de n periodos.

Al final del primer periodo : Interés =. Pi, capital = P + Pi = P(I + i).

Al final del segundo periodo : Interés = P(1 + ili, capital = P(I + i) + P(I + i)i = P(I + i)I.

El capital acumulado al cabo de n periodos será, S = P(1 + ir.

a) Como se cobran los intereses una vez por año, n = 5 e i = 0,06.

S = P(I + ir = 400 000(1 + 0,06)5 = 400 000(1 ,3382) = 535280 pts.

b) Como se cobran los intereses dos veces por año, n = 2(5) = lO e i = !(0,06) = 0,03.

S = P(I + ir = 400 000(1 + 0,03)10 = 400 000(1,3439) = 537560 pts.

e) Como se cobran los intereses cuatro veces por año, n = 4(5) = 20 e i = *(0,06) = 0,015.

S = P(1 + ir = 400 000(1 + 0,015)20 = 400 000(1,3469) = 538760 pts.

39. Hallar el capital (P) que se debe invertir al 4% de interés compuesto semestral para que al cabo de 3,5 años se trans­forme en un capital (S) de 500 000 pesetas.

Como se cobran intereses dos veces por año, n = 2(3,5) = 7 (periodos) y el rédito, en tantos por ciento y por periodo, es i = !(0,04) = 0,02.

Por tanto, S = P(1 + i)", de donde P = S(I + ¡¡-o = 500 000(1 + 0,02)- 7 = 500 000(0,87056) = 435 280 pts.

MEDIAS GEOMETRICAS

40. Deducir la fórmula de la media geométrica, G, entre dos números p y q.

G q Como p, G, q están en progresión geométrica, se tiene p = G' G2 = pq YG=~ Se suele tomar G = .¡¡;q si p y q son positivos

y G = -.¡¡;q si p y q son negativos.

http://carlos2524.jimdo.com/

Page 156: Algebra Superior Murray R Spiegel

148 PROGRESIONES

47.

43. Situar dos medias geométricas entre 686 y 2. 48.

Tenemos que completar la p. g. 686, -, -, 2 siendo a = 686, 1 = 2, n = 4.

Como l=a,..-I, 2=686r3, r3=1/343 y r=I/7.

Luego la p. g. es 686, 98, 14, 2 Y las medias son 98, 14.

Nota. Realmentev r ' = 1/343 se satisface para tres valores diferentes de r, uno de ellos real y los otros doscomplejos. Aquí prescindimos de las p. g. con términos complejos.

41. Hallar la media geométrica de los pares de números siguientes:

a) 4 Y. 9.

b) -2 Y -8.

e) j7 + j3 y j7 - j3.

G=J4(9)=6G= -)(-2)(-8)= -4

G =J(j7 + j3)(j7 - j3) = ~ = 2

42. Demostrar que la media aritmética A de los números positivos p y q es mayor que o igual a su media geométrica G.

La media aritmética de p y q es A = t(P + q). La media geométrica de p y q es G = .¡¡;q.Luego A - G = t(P + q) - .¡¡;q = t(P - 2.¡¡;q + q) = t(JP - Jq)2.

Ahora bien ttJP - Jq)2 es siempre positivo o cero; luego A ?; G. (A = G si Y solo si p = q.)

44. Situar cinco medias geométricas entre 9 y 576.

Tenemos que formar la p. g. 9, -, -, -, -, 576 siendo a = 9, 1 = 576, n = 7.

Como 1 = a,..-I, 576 = 9,6, ,6 = 64, r3 = ±8 y r = ±2.

Luego las progresiones son 9, 18, 36, 72, 144, 288, 576 Y 9, -18, 36, -72, 144, -288, 576; Y lasmedias correspondientes son 18.36,72, 144,288 Y -18,36, -72, 144, -288.

PROGRESIONES GEOMETRICAS INDEFINIDAS

45. Hallar la suma de las series geométricas siguientes:

1 1a) 2 + 1 + 2: + 4 + ...

1 2 4 8b) "3 - 9' + 27 - 8í + ...

1 1c) 1 + 1,04 + (1,04)' + ...

S¿ = _a_ = __ 2_ = 41 - r 1 - 1/2

S = _a_ = 1/3 = ~00 1 - r 1 - (- 2/3 ) 5

S = _a _ = 1 ~ = 104 = 2600 1 _ r 1 - 1/1,04 1,04 - 1 4

46. Expresar los números periódicos siguientes por medio de una fracción racional.

(1) 0,444. b) 0.4:!72727... e) 6,305305... d) 0,78367836 ..

a) 0,444 ... = 0.4 + 0,04 + 0.004 + ... , siendo a = 0.4, r = 0,'1.

S = _a_ = _~ 0.4 400 1 _ r 1 - 0,1 = 0,9 = 9'

b) 0,4272727 = 0.4 + 0.0272727 ...0.0272727 = 0.027 + 0.00027 + 0,0000027 + . . .. siendo a = 0,027. r = 0.01.

a 0,027 27 4 3 47S~ = 0.4 + T-=-; = 0,4 + ¡--=-O,Q¡ = 0.4 + 990 = 10 + TiO = TiO

PRO<

49.

SI.

148 PROGRESIONES

41. Hallar la media geométrica de los pares de números siguientes:

a) 4 Y. 9. G=j4(9)=6

b) -2 Y -8. G = - )( -2)(-8) = -4

e) J7 + j3 y J7 - j3. G =.J(J7 + j3)(J7 - j3) = ~ = 2

42. Demostrar que la media aritmética A de los números positivos p y q es mayor que o igual a su media geométrica G.

La media aritmética de p y q es A = t(P + q). La media geométrica de p y q es G = ¡¡;q. Luego A - G = t(P + q) - ¡¡;q = t(P - 2jPq + q) = t(Jp - Jq)2.

Ahora bien t(Jp - Jq)2 es siempre positivo o cero; luego A ~ G. (A = G si Y solo si p = q.)

43. Situar dos medias geométricas entre 686 y 2.

Tenemos que completar la p. g. 686, -, -, 2 siendo a = 686, 1 = 2, n = 4.

Como l=a,.. - I, 2=686,3, ,3=1/343 y,=1/7.

Luego la p. g. es 686, 98, 14, 2 Y las medias son 98, 14.

Nota. Realmente , ,3 = 1/343 se satisface para tres valores diferentes de " uno de ellos real y los otros dos complejos. Aquí prescindimos de las p. g. con términos complejos.

44. Situar cinco medías geométricas entre 9 y 576.

Tenemos que formar la p. g. 9, - , -, - , -, 576 siendo a = 9, 1 = 576, n = 7.

Como 1 = a,..-I, 576 = 9,6, ,6 = 64, ,3 = ±8 y , = ±2.

Luego las progresiones son 9, 18, 36, 72, 144, 288, 576 Y 9, -18, 36, -72, 144, -288, 576 ; Y las medias correspondientes son 18, 36,72, 144, 288 Y -18, 36, -72, 144, -288 .

PROGRESIONES GEOMETRICAS INDEFINIDAS

45. Hallar la suma de las series geométricas siguientes:

1 1 a) 2 + 1 + "2 + 4 + ...

a 2 S"" = -- = --- = 4

1 -, 1 - 1/2

1 2 4 8 b) "3 - 9" + 27 - 81 + . . .

a 1/3 1 S = --= =-

"" 1-,1-(-2/3) 5

1 1 el 1 + 1,04 + (1,04)2 +

S = _a_ = 1 = ~ = 104 = 26 "" I -r 1-1/ 1,04 ' 1,04-1 4

46. Ellpresar los números periódicos siguientes por medio de una fracción racíonal.

'1) 0,444 ... b) 0,4272727 ... e) 6,305305 ... d) 0,78367836 . ..

a) 0,444 . . . = 0,4 + 0,04 + 0.004 + .. . , siendo a = 0,4. r = 0:1.

a 0,4 0,4 4 S = -- = --- = - = -

oc, 1 - r 1 - 0,1 0.9 9

b) 0,4272727 .. . = 0,4 + 0.0272727 . .. 0.0272727 . .. = 0.027 + 0.00027 + 0,0000027 + . . .. siendo a = 0,027. r = 0.01.

a 0,027 27 4 3 47 S~ = 0,4 + "1-=-;: = 0,4 + ¡-=-0,01 = 0,4 + 990 = 10 + liO = liO

http://carlos2524.jimdo.com/

Page 157: Algebra Superior Murray R Spiegel

e) 6,305305... = 6 + 0,305305 ...0,305305. . . = 0,305 + 0,000305 + ... , siendo a = 0,305, ,= 0,001.

a 0,305 305 305S =6+--=6+ =6+-=6--

co 1 - r 1 - 0,001 999 999

d) 0,78367836 ... = 0,7836 + 0,00007836 + ... , siendo a = 0,7836, r = 0.0001.

S = _a_ = 0,7836 7836 2612ica C. ec 1 -, 1 - 0,0001 9999 3333

S = _a_ = _1_6_ = ~ = 64 centímetrosq.) eo 1 -, 1 - 3/4 1/4

os dos

y las

PROGRESIONES 149

47. Las amplitudes de las sucesivas oscilaciones de un periodo forman la progresión geométrica. 16, 12, 9, ... , cen-tímetros. Hallar la distancia total recorrida por la esferilla del péndulo hasta alcanzar el reposo.

48.. 1 1 1

Hallar el menor número de términos que se deben tomar de la sene 3 + 6 + 12 + ... para que su suma

1difiera de la suma correspondiente a los infinitos términos en menos de 1 000 .

Sea S", = suma de la progresión, S" = suma de n términos. Luego

a a(1 - r") ar"S -S=-----=--

co " 1-, 1-, 1-,

ar" 1Se desea que -- < -- siendo a = 1/3, r = 1/2.

1-,1000'

Luego .:..(1",--/3-'..)..:..{I.:..-/2..:...)"< _1_1 - 1/2 1000'

1 1--<--3(2") 2000'

22">666-.

33(2") > 2 000,

2 2Para n = 9, 2" < 6663; para n = 10, 2" > 6663, Luego se deben tomar por lo menos 10 términos.

PROGRESIONES ARMONICAS

49. Determinar cuáles de las sucesiones siguientes son progresiones armónicas:

1 1 1progresión armónica 3, 5, 7,a) 3' s' 7' es una ya que es una p. a.

1 1 1b) 2, 4, 6, no es una progresión armónica ya que 2'4'6' no es una p. a.

1 2 1 15e) 12' 15' 3' es una progresión armónica ya que 12, '2 3. ... es una p. a.

1 1 1SO, Calcular el término número 15 de la progresión armónica 4' 7' 10'

La p. a. correspondiente es 4, 7, 10, ... ; su término 15 es , = a + (n - 1ld = 4 + (15 - 1)3 = 46.1

Luego el término número 15 de la progresión armónica es 46'

51. Deducir la fórmula de la media armónica, H, entre dos números P y q.

Como p, H, q forman una progresión armónica. -p

1- es una progresión aritmética.

H q

1 1 1 1Luego H - P = q - H '

2 1 1 P + q-=-+-=--H p q pq

2p'lH =----.fi + q

y

http://carlos2524.jimdo.com/

Page 158: Algebra Superior Murray R Spiegel

150 PROGRESIONES

Otro método:

l lMedia armónica entre P y q = recíproco de la media aritmética entre - y -- .

p ql l 11 l p+q

Media aritmética entre - y - = _(_o + _.)= o .n '1 2 P '1 2p'l

. . LpqLuego la media armoruca entre" y q = --_.- .

P + q

52. Hallar la media armónica entre 3/8 y 4:

Media armónica entre8 l l 8 l 35 33 y 4 = 2(3 + 4) = 24' Luego la media armónica entre 8 y 4 = 24/35.

... . 2pq 2(3/8) (4) 24También aplicando la formula = ._- = --- = -

p + q 3/8 + 4 35

53. Situar cuatro medias armónicas entre 1/4 y 1/64.Para hallar cuatro medias en la progresión armónica entre 4 y 64 tenemos : I = a + (n - 1)d, 64 = 4 + (6 - l)d,

d = 12. Así. pues. las cuatro medias en la progresión aritmética entre 4 y 64 son, 16, 28, 40, 52.

LId' l .. . . l 1 l 1 1 luego as cuatro me las en a progresión arrnomca entre 4 y 64 son 16' 2s' 40' 52'

54. Hallar las medias armónicas entre 10 y 20.

1 lPara hallar tres medias armónicas entre 10 y 20 :

l 1 lI = a + (n - 1)d, 20 = 10 + (5 - 1)d, d = - 80'

1 l 7 6 5Las tres medias aritméticas entre 10 y 20 son 80' 80' 80'

80 40Luego las tres medias armónicas entre 10 y 20 son '7' 3' 16.

55. Determinar si la sucesión - 1, - 4, 2 es una progresión aritmética, geométrica o armónica.

Como -4 - (-1) j 2 - (-4). no es una progresión aritmética.

-4 2=I +- -4'Como no es una progresión geométrica.

l l 1Como =I' -4' 2 es una progresión aritmética. ya que

1 1 1- - (-1) = - - (--).-4 2-4

la sucesión

dada es una progresión armónica.

PROGRES

56. Hallarel vale

a) 1,h) 2,

57. Hallara) l.

58. El prinhallar

59. El últirhallar

60. Hallar

61. Hallar

62. Hallar

63. Hallar

64. Una pemetroscentirm

65. Un mucibirá ¡

66. Hallarmeros I

67. Hallar

68. Hallar

69. a) Hah) Hae) Had) En

PROGRESI

70. Hallarque se

a) 2,h) 6.e) 1.

http://carlos2524.jimdo.com/

Page 159: Algebra Superior Murray R Spiegel

24/35.

(6 - l)d,

180

sucesión

PROGRESIONES 151

PROBLEMAS PROPUESTOS

PROGRESIONES ARITMETICAS

56. Hallar el término enésimo y la suma de los n primeros términos de las progresiones aritrncticas siguientes parael valor de n que se indica:

a) 1. 7. 13.h) 2. 5t. 9.

n = 100n = 23

r) -26. -24. -22.ti) 2.6. 10....

e) 3. 4!. 6./) .\' - y. x, x + y.

11 = 40n = 16

11 = 3711 = 30

57. Hallar la suma de los n primeros términos de las progresiones aritméticas siguientes:a) 1, 2. 3. .. h) 2, 8, 14, .. e) 11. 5. 8i,

58. El primer término de una progresión aritmética es 4 y el último 34. Sabiendo que la suma de sus términos es 247.hallar el número de términos y la razón.

59. El último término de una progresión aritmética, que consta de 49 términos. es 28. Sabiendo que la razón es 1/2.hallar el primer término y la suma de todos ellos.

60. Hallar la suma de todos los enteros pares comprendidos entre 17 y 99.

61. Hallar la suma de todos los enteros comprendidos entre 84 y 719 que sean múltiplos de 5.

62. Hallar el número de términos que se deben tomar de la p. a .. 3. 7. 11..... para que su suma sea 1275.

63. Hallar tres números en p. a. cuya suma sea 4g y la correspondiente a sus cuadrados 800.

64. Una pelota rueda por un plano inclinado. partiendo del reposo. de forma que en el primer segundo recorre 3 centí-metros. en el segundo 5 centímetros. en el tercero 7 centímetros. etc. Hallar el tiempo que tardará en recorrer 120centímetros.

65. Un muchacho cobra 1 peseta el primer día. 2 pesetas el segundo. 3 pesetas el tercero. etc. Hallar el dinero que per-cibirá al cabo de 365 días.

66. Hallar el término enésirno de una p. a. sabiendo que la suma de los 40 primeros es 430 y que la suma de los 60 pri-meros es 945.

67. Hallar una p. a. sabiendo que la suma de sus n primeros términos es igual a 2n1 + 3n.

68. Hallar la media aritmética entre al 15 y 41. h) -16 Y 23. e) 2 - J3-Y 4 + :.J'1.. d sv - 3y Y 5.\' + 2y.

69. a) Hallar cuatro medias en una p. a. entre 9 y 24.h) Hallar dos medias en una p. a. entre - 1 Y 11.e) Hallar 3 medias en una p. a. entre x + 2y Y x + 10y.d) Entre los términos 5 y 26 de una p. a .. hallar un número de medias tal que la suma de la p. a. resultante sea 124.

PROGRESIONES GEOMETRICAS

70. Hallar el término enésimo y la suma de los 11 primeros términos de las sucesiones siguientes y para el valor de 11

que se indica.

a) 2. 3. 9¡2. 11 = 5 d) 1. :.. 9. n = Rb) 6. -12. 24. .. 11 = 9 e) 8. •• 2. n = 12e) l. 1/2. Ij4. .. 11 = 10 f) /3. 3. 3Ji n = S

http://carlos2524.jimdo.com/

Page 160: Algebra Superior Murray R Spiegel

152 PROGRESIONES

71. Hallar la suma de los 11 primeros términos de las progresiones geométricas siguientes.

a) 1. 1/3. 1/9. hl 4/3. 2. 3.... el 1. -2.4.

72. El primer término de una progresión geométrica es 3 y el último 48. Sabiendo que cada término es el doble deianterior. hallar el número de términos y la suma de todos ellos.

73. Demostrar que la suma S de los términos de una p. g. cuyo primer término es a. el último es / y la razón r, viene1'/ - a

dada por S = -;:-=-¡- .

74. En una p. g. el segundo término excede al primero en 4 unidades y la suma del segundo y el tercero es 24. Demos-trar que es posible encontrar dos p. p. g. g. que satisfagan estas condiciones y hallar la suma de los cinco primerostérminos de cada una de ellas.

75. Determinar una p. g. de cuatro términos sabiendo que la razón es positiva, que la suma de los dos primeros tér-minos es 10 y que la suma de los dos últimos es 22~.

76. Los dos primeros términos de una p. g. son b/( I + e) y b/(I + C)2. Demostrar que la suma de los n primeros

. . d .. . d di" S -_ b[1 - (1 + e)-"] .termmos e esta progresión viene a a por a expresione

77. Hallar la s~ma de los n primeros términos de la p. g., a - Zb. ah2 - 2b3, ab" - 2hs ....

78. El tercer término de una p. g. es 6 y el quinto es 81 veces mayor que el primero. Escribir los cinco primeros términosde la progresión suponiendo que los términos son positivos.

79. Hallar tres números en p. g. sabiendo que su suma es 42 y su producto 512.

80. El tercer término de una p. g. es 144 y el sexto 486. Hallar I<!suma de los cinco primeros términos de la progresión.

81. Un depósito contiene una solución de sal en agua siendo la masa de sal disuelta igual 972 kilogramos. Se extraeun tercio de la solución y se remplaza por agua pura. Una vez agitada la mezcla hasta conseguir su uniformidad.se extrae un tercio de la solución y se remplaza de nuevo por agua. Hallar la cantidad de sal que queda en la solu-ción después de la cuarta extracción.

82. La suma de los tres primeros términos de una p. g. es 26 y la suma de los seis primeros términos 728. Hallar eltérmino enésimo de dicha progresión.

83. La suma de tres términos en p. g. es 14. Sabiendo que si se incrementan los dos primeros términos en una unidady se disminuye en la misma cantidad el tercero. los números que resultan forman una p. a .. establecer la p. g.

84. Determinar la media geornétrica entre:

a) 2 Y 18. b) 4 Y 6. e) -4 Y -16. d) a + h Y 4a + 4h.

85. a) Hallar dos medias. en una p. g.. entre 3 y 192.h) Hallar cuatro medias, en una p. g., entre fi y 8.<:) La media geornétrica de dos números es 8. Si uno de los números es 6. hallar el otro.

PROBLEMAS DIVERSOS SOBRE PROGRESIONES A. 'Y G.

86. El primer término de una p. a. es 2. y el primero, tercero y séptimo forman una p. g. Hallar la suma de los sieteprimeros términos de la p. a.

87. Hallar el número de términos que se deben sumar de la p. a., 9, 11, 13, ... , para que la suma sea igual a la delos nueve primeros términos de la p. g., 3. -6, 12. -24, ...

88. Hall;sienc

89. Hall

PROGRE

90. Hall

a)

b)

91. La SI

nos

92. La ~tres

93. Lasque

94. Expla) I

b) I

PROGRE

95. a)b)e)

96. Halla)

97. a)b)

98. Un

la v

arm

SOLVel

56. a)bl

57. a)

58. n=

63. 12.

67. 5. ~

http://carlos2524.jimdo.com/

Page 161: Algebra Superior Murray R Spiegel

ble dei

r, viene

Demos-rimeros

ros tér-

rirneros

érminos

gresión.

e extraermidad,la solu-

aliar el

unidad. g.

los siete

Ia la de

PROGRESIONES 153

88_ Hallar cuatro números sabiendo que los tres primeros están en p. g. y los tres últimos en p. a. de razón 6,siendo el primer número igual al cuarto.

89. Hallar dos números cuya diferencia es 32 y cuya media aritmética excede a la geométrica en 4.

PROGRESIONES GEOMETRICAS INDEFINIDAS

90. Hallar la suma de las series geométricas indefinidas siguientes:

e) 1 + 1/22 + 1/24 + ...d) 6 - 2 + 2/3 - ...

a) 3 + 1 + 1/3 + .b) 4 + 2 + 1 + .

e) 4 - 8/3 + 16/9 - .

f) 1 + 0,1 + 0,01 + .

91. La suma de los dos primeros términos de una p. g. decreciente es 5/4, y la correspondiente a sus infinitos térmi-nos es 9/4. Escribir los tres primeros términos de la progresión.

92_ La suma de los infinitos términos de una p. g. decreciente es 3 y la de sus cuadrados es también 3. Escribir lostres primeros términos de la progresión.

93. Las amplitudes sucesivas (en centímetros) de la oscilación de un péndulo son 36, 24, 16, ... Hallar la distanciaque recorrerá la esferilla hasta alcanzar el reposo.

94. Expresar los números periódicos siguientes mediante una fracción racional.a) 0,121212... e) 0,270270... e) 0,1363636 ...b) 0,090909... d) 1,424242... f) 0,428571428571428 ...

PROGRESIONES ARMONICAS

95. a) Hallar el octavo término de la progresión armónica 2/3, 1/2, 2/5, .b) Hallar el décimo término de la progresión armónica 5, 30/7, 15/4, .e) Hallar el término enésimo de la progresión armónica 10/3, 2, 10/7, .

96. Hallar la media armónica entre los pares de números siguientes:

a) 3 y 6 b) 1/2 Y 1/3 e) fi y fi d) a + b y a - b

97. a) Hallar dos medias, en una progresión armónica, entre 5 y 10.b) Hallar cuatro medias, en una progresión armónica, entre 3/2 y 3/7 .

98. Un móvil se desplaza a velocidad constante a entre los puntos A y B y, acto seguido, va desde B hasta A a2ab

la velocidad constante b. Demostrar que la velocidad media del recorrido total viene dada por a + b' media

armónica entre a y b. Calcular la velocidad media en el supuesto de que a = 30 y b = 60 metros por segundo.

SOLUCIONES DE LOS PROBLEMAS PROPUESTOS

56. a) 1 = 595, S = 29 800 e) 1 = 52, S = 520 e) 1= 57, S= 1110b) 1 = 79, S = 931~ d) 1 = 62, S = 512 f) 1 = x + 28y, S = 30x + 405y

57,n(n + 1)

b) n(3n - 1) e)n(7n - 1)

a)2 4

58. n = 13, d", 5/2 59. a = 4, S = 784 60. 2378 61. 50800 62. 25

66.n+

63. 12, 16, 20 64. 10 s 65, ·667,95 pts2

67. 5, 9. 13. 17, término 11 = 4n + 1 68. a) 28, b) 7/2. e) 3 + fi, d) 3x - y/2

http://carlos2524.jimdo.com/

Page 162: Algebra Superior Murray R Spiegel

154

69. a) 12, 15, 18, 21b) 3, 7

PROGRESIONES

e) x + 4y, x + 6y, x + 8yd) La p. a. es 5, 8, 11, 14, 17, 20, 23, 26

70. a) 1 =81/8, S = 211/8b) 1 = 1 536, S = 1 062

e) 1 = 1/512, S = 1 023/512

d) 1 = 2 187, S = 3280

e) 1 = 1/256, S = 4095/256f) 1 = 81, S = 120 + 4Oj3

3 171. a) "2 [1 - (3f]

72. n = 5, S = 93

75. 4, 6, 9, 27/2

80. 844

8 3b) - [(-f - 1]

3 21 - (-2f

e) 3

74. 2, 6, 18, ... y S = 242; 4, 8, 16, ... y S = 124

(a - 2b )(b20 - 1)77. b2 _ 1 78. 2/3, 2, 6, 18, 54

81. 192 kg 82. 2' 3"-1 83. 2, 4, 8

84. a) 6 b) 2J6 e) -8 d) 2a + 2b

85. a) 12, 48 b) 2, 2.fi, 4, 4.Ji e) 32/3

86. 187 Ó 22 87. 19 88. 8, -4, 2, 8 89. 18, 50

91. 3/4, 1/2, 1/3

90. a) 9/2 b) 8 e) 4/3 d) 9/2 e) 12/5 f) 10/9

92. 3/2, 3/4, 3/8 93. 108 cm

94. a) 4/33 b) 1/11 e) 10/37 d) 47/33 e) 3/22 f) 3/7

95. a) 1/5 b) 2

96. a) 4 b) 2/5

10e) 2n + 1

e) 6.Ji - 4j3

97. a) 6, 15/2 b) 1, 3/4, 3/5, 1/2 98. 40 mis

79. 2, 8, 32

e)(m·

NOTACICescritc

2

5

DESARR(

A

B

si]\

1. Calcula,

7'a)

4!

154

69. a) 12, 15, 18, 21 b) 3, 7

PROGRESIONES

e) x + 4y, x + 6y, x + 8y d) La p. a. es 5, 8, 11, 14, 17, 20, 23, 26

70. a) 1 = ·81/8, S = 211/8

b) 1 = 1 536, S = 1 062

d) 1 = 2 187, S = 3280

e) 1 = 1/256, S = 4095/256

J) 1 = 81, S = 120 + 40J3 e) 1 = 1/512, S = 1023/512

3 1 71. a) "2 [1 - ("3f]

8 3 b) - [(-f - 1]

3 2

1 - (-2f e) 3

72. n = 5, S = 93 74. 2, 6, 18, . .. y S = 242; 4, 8, 16, . .. y S = 124

7S. 4, 6, 9, 27/2 (a - 2b)(b2• - 1)

77.

so. 844 81. 192 kg 82. 2.3.- 1

84. a) 6 b) 2.j6 e) -8 d) 2a + 2b

85. a) 12, 48 b) 2, 2j2, 4, 4)2 e) 32/3

86. 187 Ó 22 87. 19 88. 8, -4, 2, 8

90. a) 9/2 b) 8 e) 4/3 d) 9/2 e) 12/5

91. 3/4, 1/2, 1/ 3 92. 3/2, 3/4, 3/8 93.

94. a) 4/33 b) 1/ 11 e) 10/37 d) 47/33 e)

95. a) 1/5 b) 2 e) 10

2n + 1

6)2 - 4J3 d) a2 _ b2

96. a) 4 b) 2/5 e) a

78. 2/3, 2, 6, 18, 54

83. 2, 4, 8

89. 18, 50

f) 10/9

108 cm

3/22 J) 3/7

97. a) 6, 15/2 b) 1, 3/4, 3/5, 1/2 98. 40 mis

79. 2, 8, 32

http://carlos2524.jimdo.com/

Page 163: Algebra Superior Murray R Spiegel

NOTACION FACTORIAL. Las identidades siguientes indican el significado de «factorial de ni>escrito n!, o bien ~.

CAPITULO 17

Teorema del binomio de Newton

2! = 1 . 2 = 2, 3! = 1 . 2 . 3 = 6, 4! = 1 . 2 . 3 . 4 = 24

5!=1'2'3'4'5=120, n!=1·2·3 ... n, (r-I)!=1·2·3 ... (r-l)

•'"

DESARROLLO DE (a + xl"

A) Para valores de n enteros y positivos.

n(n - 1) n(n - I)(n - 2)(a + x)" = a" + nan-I x + «:> x2 + an-3 x32! 3!-

n(n - 1)(n - 2) ... (n - r + 2) n-r+1 •.r-1 -"+ ... + (r _ I)! a A + ... + A

Este desarrollo constituye el teorema del binomio, o fórmula del binomio de Newton.

El término r del desarrollo de (a + x)" es

.' _n(n-I)(n-2) ... (n-r+2) n-r+1 •..•.-1termmo r - (r _ I)! a A

E) Para valores de n negativos y fraccionarios.

El teorema del binomio es válido también para valores de n negativos y fraccionariossiempre que el binomio sea de la forma (a + x)", siendo el valor absoluto de x menor que a.No obstante, cuando n es un número negativo o fraccionario el desarrollo es ilimitado.

PROBLEMAS RESUELTOSl. Calcular:

7! 1'2·3'4'5'6'7a) =5'6'7=210

4! l' 2' 3' 4

b ) ~ = (1 . 2 . 3 . 4 . 5 . 6)(1 . 2 . 3) = 36

5' 1·2·3'4'5

m ! 1 . 2 . 3 ... (m - 2)(m - I)me) ---- = - m

(m - 1)! l . 2 . 3 ... (m - 2)(m - 1)- -15'

Por ejemplo. _: = 1514!

155

CAPITULO 17

Teorema del binomio de Newton

NOTACION FACTORIAL. Las identidades siguientes indican el significado de «factorial de n» escrito n!, o bien ~ .

2! = 1 . 2 = 2, 3! = 1 . 2 . 3 = 6, 4! = 1 . 2 . 3 . 4 = 24

5! = 1 ' 2'3'4'5 = 120, n! = 1·2·3 .. . n, (r - 1)! = 1 ' 2'3 . . . (r - 1)

DESARROLLO DE (a + xl"

A) Para valores de n enteros y positivos.

n(n - 1) n(n - 1)(n - 2) (a + x l" = an + nan

-I x + an

-2

X2 + an-

3 x: 3

2! 3! -

n(n - 1)(n - 2) ... (n - r + 2) n - r+1 • .,-1 JI

+ ... + (r _ 1)! a Á + . . . + A

Este desarrollo constituye el teorema del binomio, o fórmula del binomio de Newton.

El término r del desarrollo de (a + xl" es

. . _ n(n - 1)(n - 2) . . . (n - r + 2) n-r+1 • ...-1 termmo r - (r _ 1)! a A

E) Para valores de n negativos y fraccionarios.

El teorema del binomio es válido también para valores de n negativos y fraccionarios siempre que el binomio sea de la forma (a + xl", siendo el valor absoluto de x menor que a. No obstante, cuando n es un número negativo o fraccionario el desarrollo es ilimitado.

PROBLEMAS RESUELTOS l. Calcular:

7! 1'2·3'4'5 ' 6 ' 7 a) =5 ' 6 '7 =210

4! l' 2 ' 3 ' 4

m! I . 2 . 3 . . . (m - 2)(m - I)m e) ---- = - m

(m - l) ! 1·2 · 3 ... (m - 2j(m - I)·-

b ) ~ = (1 . 2 . 3 . 4 . 5 . 6)(1 . 2 . 3) = 36

5' 1 · 2 · 3'4'5

15' Por ejemplo. _: = 15

14!

155

http://carlos2524.jimdo.com/

Page 164: Algebra Superior Murray R Spiegel

•."

156 TEOREMA DEL BINOMIO DE NEWTON

n! 1 . 2 . 3 ... (n - 3 )(n - 2 )(n - l)n . 18 !d) --- = = n(n - 1). Por ejemplo, -16! = 18' 17 = 306

(n - 2)! 1 . 2 . 3 ... (n - 3)(n - 2)

(p - 2)! _ 1 ·2' 3 ... (p - 4)(P - 3)(P - 2) _ _ 3 _ 2e) (p _ 4)! - 1 . 2 . 3 ... (p - 4) - (p )(p )

alta + b - 2)! [ata - l)(a - 2)!](a + b - 2)' ata - 1)f) (a _ 2)!(a + b)! (a - 2)![(a + b)(a + b - l)(a + b - 2)!] (a + b)(a + b - 1)

DESARROLLO DEL BINOMIO CON EXPONENTE ENTERO Y POSITIVO

Desarrollar por la fórmula del binomio:

Obsérvese que en el desarrollo de (a + xf:

1) El exponente de a + el exponente de x = n (es decir, el grado de cada término es n).

2) ,El número de términos es n + 1, cuando n es un número entero y positivo.

3) Hay dos términos medios cuando n es un número entero, impar y positivo .

4) Hay un término medio cuando n es un número entero, par y positivo .

5) Los coeficientes de los términos que equidistan de los extremos son iguales. Dichos coeficientes se pueden dis-poner de la forma siguiente:

(a + x)o

(a + x)'

(a + X)2

(a + X)3

(a + X)4 .

(a + x)s

(a + x)~

etc ...

2

,1 3 31 4 6 4 1

1 5 10 10 5 1

6 15 20 15 6

Esta disposición de números recibe el nombre de triángulo de Paseal. El primero y el último término son igua-les a 1 y los demás se obtienen sumando los dos números a su derecha e izquierda de la fila anterior.

6.

7. (

8. (

9. (

10. (

u. (

12. ,

13. I

156

d) n!

(n - 2)!

e) (p - 2)! =

(p - 4)!

TEOREMA DEL BINOMIO DE NEWTON

1 . 2 . 3 ... (n - 3)(n - 2)(n - l)n 18 1 -----:-----=-----=-'----:-~_:_:_:---'..:_::.,_____'- = n(n - 1). Por ejemplo, --.: = 18· 17 = 306

1 · 2 · 3 . . . (n - 3)(n - 2) 16!

1·2·3 . . . (p-4)(p-3)(p-2)_ -3 -2 1 . 2 . 3 . . . (p - 4) - (p )(p )

alfa + b - 2)! f) (a - 2)!(a + b)!

[ata - I)(a - 2)!](a + b - 2)! ata - 1)

(a - 2)![(a + b)(a + b - I)(a + b - 2)!] (a + b)(a + b - 1)

DESARROLLO DEL BINOMIO CON EXPONENTE ENTERO Y POSITIVO

Desarrollar por la fórmula del binomio:

Obsérvese que en el desarrollo de (a + x r:

1) El exponente de a + el exponente de x = n (es decir, el grado de cada término es n).

2) .El número de términos es n + 1, cuando n es un número entero y positivo.

3) Hay dos términos medios cuando n es un número entero, impar y positivo.

4) Hay un término medio cuando n es un número entero, par y positivo.

5) Los coeficientes de los términos que equidistan de los extremos son iguales. Dichos coeficientes se pueden dis­poner de la forma siguiente:

(a + x)o

(a + x)'

(a + x¡Z 2

(a + X)3 , } 3 3

(a + X)4 4 6 4

(a + x)~ 1 5 10 10 5 1

(a + X)6 6 15 20 15 6

etc ...

Esta disposición de números re,cibe el nombre de triángulo de Pascal. El primero y el último término son igua­les a 1 y los demás se obtienen sumando los dos números a su derecha e izquierda de la fila anterior.

http://carlos2524.jimdo.com/

Page 165: Algebra Superior Murray R Spiegel

en dis-

n igua-oro

TEOREMA DEL BINOMIO DE NEWTON 157

En el desarrollo de un binomio de la forma (a - b)". siendo n un número entero y positivo, los términos sonalternativamente positivos y negativos.

4·3 4·3·26. (3a3 - 2b)4 = (30')4 + 4(3a3)3(-2b) + - (3a3)2(-2W + -- (3a3)(-2b)3 + (-2b)4

I . 2 I ·2·3

= 81a'2 - 216a9b + 216a6b2 - 96a3b3 + 16b4

7·6 7·6·5 7·6·5·47. (x - 1)7 = x7 + 7x6(-I) + _x5(_1)2 + __ x4(_1)3 + x3(_1)41·2 1·2·3 1·2·3·4

7·6·5·4·3 7'6'5'4·3·2+ x2(_1)5 + x(_1)6 + (_1)71·2·3'4'5 1·2'3'4'5'6

= x7- 7x6 + 21x5 - 35x4 + 35x3 - 21x2 + Tx - I

5'4 5'4·3 5'4'3'28. (1 - 2X)5 = I + 5(-2x) + - (-2X)2 + -- (-2X)3 + (-2X)4 + (-2X)5

1·2 1·2·3 1'2'3'4

= I - IOx + 40x2 - 80x3 + 80x4 - 32x5

x 2 x x 2 4'3x 2 4'3'2x 2 29. (_ + _)4 = (-)4 + 4(_)3(-) + _(-)2(-)2 + __ ._(-)(-)3 + (-)43 Y 3 3 Y 1'23 Y 1'2'33 Y Y

x4 8x3 8x2 32x 16=-+-+-+-+-81 27y 3y2 3v3 y4

n. (fi - ~)4 = (x'/2 _ x-1/2)4..¡X

4'3= (X'/2)4 + 4(XI/2)3( _X-1/2) + _ (x'/2)'( _x-1/2)2I • 2

7'6 7'6'513. (eX - e-X)7 = (eX)7 + 7(ex)6( _e-X) + _ (e')5( _e-x)2 + __ (e')4( _e-x)3l' 2 I . 2·3

7'6'5'4 7'6'5'4'3+ (e')3( _e-X)4 + (ex)2( _e-x)51·2'3'4 1\2'3'4'5

7'6'5'4'3'2+ (eX)(_e-x)6 + (-e-Xf1'2·3'4'5'6

TEOREMA DEL BINOMIO DE NEWTON 157

En el desarrollo de un binomio de la forma (a - bj" , siendo n un número entero y positivo, los términos son alternativamente positivos y negativos .

4 ,3 4,3,2 6, (3a' - 2b)4 = (3a')4 + 4(3a')'(-2b) + - (3a'f(-2b)2 + -- (3a')(-2b)' + (-2b)4

I . 2 I . 2·3

7'6 7'6'5 7 ' 6'5'4 7. (x - 1)7 = x7 + 7x6(-I) + _x5(_1)2 + _ _ x4 (_ I )' + x'(_1)4

1' 2 1·2'3 1·2·3'4

7'6'5'4'3 7'6'5'4'3'2 + x2(_1)5 + x(_1)6 + (_1)7 1·2·3'4'5 1·2'3'4'5·6

= x 7 - 7x6 + 21x 5 - 35x4 + 35x' - 21x2 + 7x - I

5'4 5'4 ' 3 5'4 ·3'2 8. (1 - 2X)5 = I + 5(-2x) + - (-2X)2 + - - ( - 2x)' + ( -2X)4 + (-2X)5

1'2 1·2·3 1·2 ' 3 ' 4

= I - 10x + 40X2 - 80x' + 80x4 - 32x5

x 2 x x 2 4 '3x 2 4·3·2x 2 2 9. (_ + _ )4 = (-)4 + 4(- )'(-) + _(-)2(-)2 + _-o _(-)(-)' + (-)4

3 Y 3 3 Y 1 '23 Y 1·2·33 Y Y

x· 8x' 8X2 32x 16 = - + - +-+-+-

81 27y 3y2 3y' .1'4

I 11. (Jc - r:-t = (X 1/2 - x- 1/2)4

..¡X

4·3 = (X 1/2)4 + 4(X1 /2)'(_X-1/2) + _ (X 1/2)2(_X- 1/2)2

I ·2

4·3·2 " + _ _ (X 1/2)( _X- 1/2)' + (_X-1/2)4 = x 2 _ 4x + 6 _ 4x - 1 + x - 2 1 · 2 ·3

7·6 7'6 ' 5 13. (e' - e- x )7 = (eX)7 + 7(e' )6( _ e-X ) + _ (e')5 ( _e- X)2 + __ (e')4( _e- X)' 1 ·2 1 . 2' 3

7'6'5'4 7'6·5'4·3 + (e' )'( _e - X)4 + (eX)2( _ e- x )5 1'2'3'4 1 ¡ 2'3'4'5

7'6'5'4·3·2 + (e')( - e- x )6 + (_e- x )7 1'2 ·3 ·4'5'6

http://carlos2524.jimdo.com/

Page 166: Algebra Superior Murray R Spiegel

I.

158 TEOREMA DEL BINOMIO DE NEWTON

28.

3'214. (a + b - e)3 = [(a + b) - e]3 = (a + W + 3(a + W(-e) + N(a + b)(-e)2 + (_e)3

= a3 + 3a2b + 3ab2 + b3 - 3a2e - 6abe - 3b2e + 3ae2 + 3be2 - e3

3·215. (x2 + x - W = [X2 + (x - 3)]3 = (X2)3 + 3(X2)2(X - 3) + N (x2)(x - 3f + (x - 3)3

= x6 + (3x5 - 9x4) + (3x4 - 18x3 + 27x2) + (x3 - 9x2 + 27x - 27)= x6 + 3x5 - 6x4 - 17x3 + 18x2 + 27x - 27

24

En los Problemas 16·21 escribir el término indicado de cada desarrollo aplicando la fórmula

n(n - l)(n - 2) ... (n - r + 2)término r de (a + x)" = d'-dlx,-I

(r - 1)

16. Sexto término de (x + y)15. n = 15, r = 6, n - r + 2 = 11, r - 1 = 5, n - r + 1 = 10

15' 14' 13 . 12' 116.0 término = xlOy5 = 3003xlOys

1 ·2·3·4· 5

17. Quinto término de (a - fi)9. n = 9, r = 5, n - r + 2 = 6, r - 1 = 4, n - r + 1 = 59·8·7·6

5.0 término = a5(-fi)4 = 126a5b21·2' 3·4

25.

18. Cuarto término de (x2 - y2)11. n = 11, r = 4, n - r + 2 = 9, r - 1 = 3, n - r + 1 = 8

11' 10·94.0 término = (X2)8(_y2)3 = -165x16y6

1 ·2· 3

x 1 1219. Nuevo término de (- + -) . n = 12, r = 9, n - r + 2 = 5, r - 1 = 8, n - r + 1 = 42 x 26.12 . 11 . 10' 9 . 8 . 7 . 6 . 5 x 1 495

9.° término = l' 2 . 3 . 4 . 5 . 6 . 7 . 8 . ("2f(';l = 16x4

120. Decimoctavo término de (1 - _)20 n = 20, r = 18, n - r + 2 = 4, r - 1 = 17, n - r + 1 = 3

x

20 . 1·9. 18 . 17 ... 4 1 1718.0 término = (--) =

1·2'3·4 ... 17 x

20' 19· 18l : 2' 3Xl7

1140-~

121. Término medio (4.0) de (X1/3 - - X-2)6. n = 6, r = 4, n - r + 2 = 4, r - 1 = 3, n - r + 1 = 3

2

DE

En

22. Hallar el término en x2 del desarrollo de {x3 + ~)IOX

27.

De (x3)IO-dl{x-Irl = x2 se obtiene 3{1O - r + 1) - I{r - 1) = 2 de donde r = 8

Para el 8.0 término: n = 10, r = 8, n - r + 2 = 4, r - 1 = 7, n - r + 1 = 3

123. Hallar el término independiente de x en el desarrollo de {x2 __ )9

X

158 TEOREMA DEL BINOMIO DE NEWTON

3'2 14. (a + b - e)3 = [(a + b) - e]3 = (a + W + 3(a + b)'(-e) + M(a + b)(-e)2 + ( _ e)3

= a3 + 3a2b + 3ab2 + b3 - 3a2e - 6abe - 3b2e + 3ae2 + 3be2 - e3

3 ' 2 15. (x2 + x - W = [X2 + (x - 3)]3 = (X2)3 + 3(X2)2(X - 3) + M (x2)(x - 3)2 + (x - 3)3

= x6 + (3xS - 9x4) + (3x4 - 18x3 + 27x2) + (x3 - 9X2 + 27x - 27) = x6 + 3xS - 6x4 - 17x3 + 18x2 + 27x - 27

En los Problemas 16-21 escribir el término indicado de cada desarrollo aplicando la fórmula

n(n - l)(n-2) . .. (n - ,+2) . +1 '1 término, de (a + xf = d" x'

(, - 1)

16. Sexto término de (x + y)IS. n = 15, , = 6, n - , + 2 = 11 , , - 1 = 5, n - , + 1 = 10

15 ' 14 ' 13 • 12' 11 6.0 término = xl0yS = 3003x 10y s

1 · 2·3·4·5

17. Quinto término de (a - Jb)9 . n = 9, , = 5, n - , + 2 = 6, , - 1 = 4, n - , + 1 = 5 9' 8' 7' 6

5.0 término = a>(-Jb)4 = 126asb2 l' 2' 3' 4

18. Cuarto término de (x2 - y2)11 n = 11 , , = 4, n - , + 2 = 9, , - 1 = 3, n - , + 1 = 8

4.0 término = 11 . 10· 9 (X2)8( _ y2)3 = _165x16y6 1·2·3

x 1 12 19. Nuevo término de (- + - ) . n = 12, , = 9, n - , + 2 = 5, , - 1 = 8, n - , + 1 = 4 2 x

12' 11 . 10' 9 . 8 . 7 . 6 . 5 x 1 495 9.0 término = . (_ )4(_ )8 = __

1 . 2 . 3 . 4 ' 5 . 6 . 7 . 8 2 X 16x4

1 20. Decimoctavo término de (1 - _)20. n = 20, , = 18, n - , + 2 = 4, , - 1 = 17, n - , + 1 = 3

x

20 . ~9 . 18 . 17 ... 4 1 17 20 . 19 . 18 1 140 18 ,o término= 1'2'3'4 ... 17 (-:;) = 1'2 '3x17 -'71

1 21. Término medio (4.°) de (X 1/3 - - X' 2)6. n = 6, , = 4, n - , + 2 = 4, , - 1 = 3, n - , + 1 = 3

2

22. Hallar el término en X2 del desarrollo de (x 3 + ~)10 X

De (X3)10'<+I(X' I )"1 = x2 se obtiene 3(10 -, + 1) - 1(, - 1) = 2 de donde, = 8

Para el 8.0 término: n = 10, , = 8, n - , + 2 = 4, , - 1 = 7, n - , + 1 = 3

10'9'8 ·7'6'5'4 a 8.0 término = (X 3)3(_)7 = 120a7x2

1'2'3'4 · 5·6'7 x

1 23. Hallar el término independiente de x en el desarrollo de (x 2

__ )9 X

http://carlos2524.jimdo.com/

Page 167: Algebra Superior Murray R Spiegel

TEOREMA DEL BINOMIO DE NEWTON 159

Ixl < lal

De (X2)9-dl(X-Iy-1 = x", se obtiene 2(9 - r + 1) - I(r - 1) = O; de donde r = 7

Para el 7.° término: n = 9, r = 7, n - r + 2 = 4, r - 1 = 6. n - r + 1 = 3

24. Calcular (1,03)1 ° con cinco cifras significativas.

10·9 10'9·8 10'9'8'7(1,03)10 = (1 + 0,03)10 = 1 + 10(0,03) + -- (0,03)2 + --- (0,03)3 + (0,03)4 + ...1'2 1'2'3 )'2·3'4

= 1 + 0,3 + 0,0405 + 0,00324 + 0,00017 + ... = 1,3439

Obsérvese que el desarrollo de (1 + 0,03)1 ° consta de 11 términos.

25. Calcular (0,99)15 con cuatro cifras decimales.

15'14 15'14·13(0,99)15 = (1 - 0,01)15 = 1 + 15(-0,01) + -- (-0.01)2 + (-0,01)31 . 2 1·2' 3

15' 14' 13 . 12+ 2 4 (-0,01)4+ ...l' . 3 .

= 1 - 0,15 + 0,0105 - 0,000455 + 0,000014 - ... = 0,8601

26. Hallar la suma de los coeficientes del desarrollo de a) (1 + X)IO, b) (1 _ X)lo.

a) Si 1, C" c2' •.• , cIO son los coeficientes, tenemos la identidad

Entonces (1 + 1)10 = 1 + el + e2 + ... + cIO = suma de los coeficientes = 21o = 1024

b) Hagamos x = 1. Entonces (1 - X)IO = (1 - 1)10 = ° = suma de los coeficientes.

DESARROLLO CON EXPONENTE FRACCIONARIO Y NEGATIVO

En los Problemas 27-28 escribir los cuatro primeros términos del desarrollo.

27. (a - x)1/3 = al/3 + ~a-2/3(_x) + (1/3)(-2/3) a-S/3(-xf + (1/3)(-2/3)(-5/3) a-S/3(_x)3 + ...3 r- 2 1·2·3

3/2 3 (3/2)(1/2) 2 (3/2)(1/2)( -1/2) 328. (1 + x) = 1 + '2 x + -1-'-2- x + 1 .2. 3 x + .. Ixl < 1

TEOREMA DEL BINOMIO DE NEWTON 159

De (X2)9-dl(X- I)' - 1 = x o, se obtiene 2(9 - r + 1) - I(r - 1) = O; de donde r = 7

Para el 7.° término : n = 9, r = 7, n - r + 2 = 4, r - 1 = 6, n - r + 1 = 3

9· 8 '7· 6' 5'4 7.0 término = (X2 )3 ( _ X - I )6 = 84

1'2·3'4·5 · 6

24. Calcular (1 ,03)10 con cinco cifras significativas.

10 · 9 10'9·8 10·9'8'7 (1,03)10 = (\ + 0,03)'° = 1 + 10(0,03) + -- (0,03)2 + - - - (0,03)3 + (0,03)4 +

\ '2 1 ' 2 ' 3 1 ' 2·3'4

= 1 + 0,3 + 0,0405 + 0,00324 + 0,00017 + .. . = 1,3439

Obsérvese que el desarrollo de (1 + 0,03)1 ° consta de \\ términos.

25. Calcular (0,99)1 5 con cuatro cifras decimales.

15' 14 15 ' 14 ' 13 (0,99)15 = (1 - 0,01)1 5 = 1 + 15( - 0,01) + - - (-0,01)2 + (-0,01)3

1 . 2 1 · 2 ' 3

15 . 14' 13 . 12 + 1'2 . 3'4 (-0,0\)4+ . . .

= 1 - 0,15 + 0,0105 - 0,000455 + 0,000014 - . . . = 0,8601

26. Hallar la suma de los coeficientes del desarrollo de a ) (1 + X )I O, b ) (1 - X) I O.

a) Si \ , c I , c2 ' • . • , ClO son los coeficientes, tenemos la identidad

Entonces (1 + \)1 0 = 1 + el + C2 + ... + cIO = suma de los coeficientes = 21o = 1024

b) Hagamos x = 1. Entonces (1 - X)IO = (1 - 1)10 = O = suma de los coeficientes.

DESARROLLO CON EXPONENTE FRACCIONARIO Y NEGATIVO

En los Problemas 27-28 escribir los cuatro primeros términos del desarrollo.

27 (a _ ~)1 /3 = al /3 + ~a - 2 /3( _ ) + (1 /3)( - 2/3) - 5/3(_ )2 + (1 /3)(-2/3)(-5/3) -8/3(_ )3 + " 3 x 1.2 a x 1.2 . 3 a x . . .

28. (1 + .X)3/2 = 1 3 (3/2)(1 /2) 2 (3/2)(1 /2)(-1/2) 3 +

+ '2 x + - 1- '-2- x + 1 . 2 . 3 x ... Ixl < 1

http://carlos2524.jimdo.com/

Page 168: Algebra Superior Murray R Spiegel

160 TEOREMA DEL BINOMIO DE NEWTON

29. Calcular j26 con seis ciíras significativas.

j26 = (52 + 1)1/2 = (52)1/2 + -21(52)-1/2 + (1/2)(-1/2) (52)-3/2l ·2

(1/2)(-1/2)(-3/2) 52 -5/2+ 1·2· 3 () + ...

l1 1 1 l 1 l 1 1= 5 + 2(5) - 8(5"3) + 16("5') - = 5 + 10 - 1000 + 50000 - ...

= 5 + 0,1 - 0,001 + 0,00002 - = 5,09902

No/a. Es incorrecto calcular j26 desarrollando (1 + 52)1/2. ¿Por qué?

30. Calcular.y998 con seis cifras significativas.

fo8 = (W _ 2)1/3 = (103)1/3 + ~ (W)-l/3(_2) + (1/3)(-2/3) (103)-5/3(_2)2 +3 1 . 2

2 1 4 1= lO - iw) - 9(W) - ... = lO - 0,006667 - 0,000004 - ... = 9,99333

31. Escribir el sexto término del desarrollo de (1 - X2)1/2.

n = 1/2. r = 6, n - r + 2 = -7/2, r - 1 = 5, n - r + 1 = -9/2

6.0 término = (1/2)(-1/2)(-3/2)(-5/2)(-7/2) (_X2)5 = _~XIO

1 . 2 . 3 . 4 . 5 256

En los Problemas 32-36 escribir los cuatro primeros términos del desarrollo.

32. (a + X)-I _ -1 + ( 1)0-2 + (-1)(-2) -3.2 + (-1)(-2)(-3) -43+-a - x 1.2 a x 1'2'3 a x ...

válido para Ixl < lal

33. (1 _ X)-2 = 1 (-2)(-) (-2)(-3) (_ .)2 (-2)(-3)(-4) (_ )3 ,+ x+ 1.2 .\ + 1.2.3 x, ...

= 1 + 2x + 3x2 + 4x3 + ... válido para Ixl < 1

34. (1 _ X)-4 = 1 + (-4)(-x) + (-4){-5) (_X)2 + (-4)(-5)(-6) (_X)3 +1 ·2 1 . 2' 3

= l + 4x + IOx2 + 20--.-3+ ...

35 (2 )-3 = 2-3 + (-3)(2-4)X + (-3)(-4)(2-5)X2 + (-3)(-4)(-5) (2-6)X3 + .. +x 1.2 1.2'3

1 3 3 2 5= - - -x + -x - _x3 + . válido para Ixl < 2

8 16 16 32

(-1'2)(-]'7)36. (9 + X)-1/2 = 9-1/2 + (-1/2)(9-312)X + . /- (9-';2).\"2 válido para Ixl < 9

1 . 2

(-1/2)(-3/2){-5j2) -7/2 3 I X x2 5x3

+ I .2 . 3 (9)x +... = '3 - 54 + 648 - 34 992 +

37. Ese

38. Ese

39. Call

a)

40. Des

a)

b)

41. Esel

a)

b)

e)

d)

42. Hal

43. Hal

44. Cal.

45. Cal,

46. Ese

a)

b)

http://carlos2524.jimdo.com/

Page 169: Algebra Superior Murray R Spiegel

TEOREMA DEL BINOMIO DE NEWTON 161

37. Escribir el quinto término del desarrollo de (.j;¡y - $x)-4

n = -4, r = 5, n - r + 2 = -7, r - I = 4, n - r + I = -8

5 o té . _ (-4)(-5)(-6)(-7) (12)_0 ( y1/2)4 _ 35 -6 6. ermmo - I . 2 . 3 . 4 y'/2 - x'/2 - X Y

i",

I38. Escribir los cuatro primeros términos del desarrollo de (1 + -r.

n

I n(n - 1) I n(n - I)(n - 2)= 1 + 1 + -2,--2- + -3' 3 + .... n . n

PROBLEMAS PROPUESTOS

39. Calcular los cocientes siguientes:

6!a) 3! b) ~

5!lO!

e) 6! 4!12!

d) 7' 3! 2!(2n + 1)!

e)(2n - 1)!

n!f) (n - 3)! 3!

(n - 1)! 4!g)

(n + 2)!

40. Desarrollar por la fórmula del binomio.

Ia) (x +, 2)6

b) (x - 2)5

g) (~+ ~)42 Y

h) (y1l2 + y-1/2)6

e) (y + 3)4

1d) (x + X)5 f) (a - 2W

41. Escribir el término indicado en los siguientes desarrollos.

a) Quinto término de (a - b)7 Ie) Séptimo término de (a _ .;;/0

Ib) Séptimo término de (x2 __ )9

Xf) Decimosexto término de (2 - I/X)'B

Ie) Término medio de (y __ )B

Yx2

d) Octavo término de (- - 2y)'62

g) Sexto término de (x2 - 2y)"

Ih) Onceavo término de (x + ;;/4

I42. Hallar el término independiente del desarrollo de (Jx + -3 2)'0

x

I43. Hallar el término en x3 del desarrollo de (x2 + _)12.

X

44. Calcular (0,98)6 con cinco cifras decimales.

45. Calcular (1,1)' o con una aproximación de una centésima.

46. Escribir los cuatro primeros términos de los desarrollos siguientes:

al (1 + 2X)-1

b) (1 _ y)-3

e) (a + x¡-"2 el (1 + Xl"3 g) (4 + X)'/2

Ih) (1 + _)-2/3

Sd) (1 + r)-6 f) (1 - 3y)-1/3

TEOREMA DEL BINOMIO DE NEWTON

37. Escribir el quinto término del desarrollo de (..¡;¡y _ .jYf;:¡-4.

n = -4, r = 5, n - r + 2 = -7, r - 1 = 4, n - r + 1 = -8

5 0 t' . _ (-4)( - 5)( -6)( -7) ,x1/2)_0 ( y I /2)4 35 -6 6 . ermmo - \-- -- = x y

1 . 2 • 3 . 4 y' /2 X' /2

1 38. Escribir los cuatro primeros términos del desarrollo de (1 + -ro

n

(1 1)"_1 (1) n(n-I)(1)2 n(n-l)(n-2)1 3 + - - +n- +---- + H + ... n n 1·2 n 1 ,2 ,3 n

1 1 n(n - 1) 1 n(n - I)(n - 2) = + 1 + -2,--2- + -31 3 + .. .

. n . n

PROBLEMAS PROPUESTOS

39. Calcular los cocientes siguientes:

6! a) 3!

8! b) 5!

lO! e) 6! 4!

12! d) 7! 3! 2!

(2n + 1)! e)

(2n - I)!

40. Desarrollar por la fórmula del binomio.

1 a) (x +, 2)6

b) (x - 2)'

e) (y + 3)4

1 d) (x + x)' f) (a - 2W

41. Escribir el término indicado en los siguientes desarrollos.

n! f) (n - 3)! 3!

g) (~+ ~)4 2 Y

h) (y' /2 + y-1 /2)6

a) Quinto término de (a - b)7 e) Séptimo término de (a _ ~)IO

b) 1

Séptimo término de (x2 __ )9 f) Decimosexto término de (2 - l /x) '8 x

I Sexto término de (x2 - 2y)'1 e) Término medio de (y _ _ )8 g)

Y X2

Onceavo término de (x + ~)14 d) Octavo término de (- - 2y)'6 h) 2

1 42. Hallar el término independiente del desarrollo de (Jx + -3 2)'0

x

1 43. Hallar el término en x 3 del desarrollo de (x 2 + _ )'2

X

44. Calcular (0 ,98)6 con cinco cifras decimales.

45. Calcular (1,1)'0 con una aproximación de una centésima.

46. Escribir los cuatro primeros términos de los desarrollos siguientes:

161

(n - 1)! 4! g)

(n + 2)!

a) (1 + 2X) - 1 e) (a + X) - 1/2 e) (1 + X)' / 3 g) (4 + X)' /2

b) (1 _ y) - 3 d) (1 + r)-6 f) (1 - 3y) - I/3 h) (1 + ~)-2/l s

http://carlos2524.jimdo.com/

Page 170: Algebra Superior Murray R Spiegel

47. Calcular con la aproximación indicada:

TEOREMA DEL BINOMIO DE NEWTON162

b) ,y2s; con tres cifras decimales

e) ,y34 ; con aproximación de una centésimaa) fo; con cuatro cifras significativas

d) 1l.Oi; con cinco cifras decimales

248. Escribir el decimosexto término del desarrollo de (x + _)-s.x

SOLUCIONES DE LOS PROBLEMAS PROPUESTOS

39. a) 120 b) 336 e) 210 d) 7920 e) 2n(2n + 1)

15 5 15 3 140. a) x6 + 3x' + - x4 + - x3 + - x2 + - X + -

4 2 16 16 64

b) XS - IOx4 + 4Ox3- 80x2 + 80x - 32

e) y4 + 12y3 + 54y2 + 108y + 81

S 3 lO 5 1d) x + 5x + IOx + - + - + -

X x3 XS

f)' n(n - I)(n - 2)5

24g)-----

n(n + I)(n + 2)

e) 70 e) 2100

b) 84 d) _2860XI8y7.6528

f) -7>

42. 5 43. 792x3 44. 0,88584 45. 2,59

46. a) 1 - 2x + 4x2 - 8x3 + ... Ixl < ~

b) 1 + 3y + 6y2 + lOy3 + . . . Iyl < 1

1 2 5e) a-1/2 __ a-3/2x + _a-S/2x2 __ a-7/2x3 +2 8 16 ...

d) 1 - 6r + 21,2 - 56,3 + ...

47. a) 4,123

403248. -7>

b) 3,037 e) 2,02 d) 1,00199

g) -14784x12yS

1001h)

x

X x2 5x3

e) 1 + "3- 9 + 81 - ...2 14 3

f) 1 + Y + 2y + 3y + ...X x2 x3

g) 2 + - - - + --4 64 512 ...

2 5 40h) 1 - 3s + 9s2 - 81s3 + . . . Isl > 1

EL Ppd1112t;

l. DI

Pr

Se.

qu

n .ZÓI

2. DI

mi

Pr

162 TEOREMA DEL BINOMIO DE NEWTON

47. Calcular con la aproximación indicada:

a) fo; con cuatro cifras significativas e) ~ ; con aproximación de una centésima

b) 128; con tres cifras decimales d) ,yi,Oi; con cinco cifras decimales

2 48. Escribir el decimosexto término del desarrollo de (x + -)-'.

x

SOLUCIONES DE LOS PROBLEMAS PROPUESTOS

39. a) 120 b) 336 e) 210 d) 7920 e) 2n(2n + 1)

15 5 15 3 1 40. a) x 6 + 3x' + - x4 + - x) + - X2 + - X + -

4 2 16 16 64

b) x' - 10x4 + 4Ox) - 80X2 + 80x - 32

e) y4 + 12y) + 54y2 + 108y + 81

,) 10 5 1 d) x + 5x + 10x + -; + ? + :;s

f)' n(n - 1)(n - 2)

5

24 g) ----­

n(n + 1)(n + 2)

h) y) + 6y2 + 15y + 20 + 15y- 1 + 6y-2 + y-)

e) 70

b) 84

42. 5 43. 792x) 44. 0,88584

46. a) 1 - 2x + 4X2 - 8x) + ...

b) 1 + 3y + 6y2 + 10y) + ... Iyl < 1

d) 1 - 6r + 21,2 - 56,) + .. .

47. a) 4,123 b) 3,037 e) 2,02

4032 48. -7"'

e) 2100

6528 f) -7"'

45. 2,59

d) 1,00199

g) -14784x12y S

1001 h)

x

X X2 5x) e) 1 + "3 - 9" + 81 - ...

2 14) f) 1 + Y + 2y + 3" y + ...

X X2 x) g) 2 + ¡ - 64 + 512 - . ..

2 5 40 h) 1 - -3 + -9 2 - -8 ) + ... Isl> 1

s s Is

http://carlos2524.jimdo.com/

Page 171: Algebra Superior Murray R Spiegel

241)(n+ 2)

-)

\s\ > 1

CAPITULO 18

Principio matemático de inducción completa

EL PRINCIPIO MATEMATICO DE INDUCCION COMPLETA es un procedimiento que sirvepara demostrar un teorema general, o una fórmula, a partir de casos particulares. Para hacer unademostración por este método se procede de la forma siguiente:1) Se comprueba, por simple sustitución, que el teorema propuesto, o fórmula, se verifica paralos primeros valores de n, enteros y positivos, por ejemplo, n = 1, n = 2, etc.2) Se supone que el teorema, o fórmula, es cierto para n = k y, a continuación, se demuestra quetambién se verifica para el siguiente n = k + l.

PROBLEMAS RESUELTOS

l. Demostrar, por el principio delinducción completa, que para todos los valores de n, entero y positivo, se verifica:n(n + 1)

1+2+3+ ... +n=--2-

,. 1(1 + 1)La formula se venfica para n = 1, ya que 1 = --2-- = 1.Primero.

Segundo. Supongamos que la fórmula es cierta para n = k. Entonces, sumando (k + 1) a los dos miembros,

k(k+l) (k+l)(k+2)1 + 2 + 3 + ... + k + (k + 1)= --2- + (k + 1) = 2

n(n + 1)que es el valor de --4-- cuando se sustituye n por (k + 1).

Por tanto, si la fórmula es cierta para n = k, hemos demostrado que también se verifica para el siguiente,n = k + 1. Como la fórmula se verifica para n = 1, también se verificará para n = 1 + 1 = 2 y, por la misma ra-zón, para n = 2 + 1 = 3, y así sucesivamente. Es decir, se verifica para todos los valores de n, entero y positivo.

2. Demostrar, por el principio de inducción completa, que la suma de los n primeros términos de una progresión arit-n

mética, a, a + d, a + 2d, ... , es i2a + (n - I)d], es decir,

na + (a + d) + (a + 2d) + ... + [a + (n - l)d] = Z{2a + (n - l)d]

1Primero. La fórmula se verifica para n = 1, ya que a = 2"[20 + (1 - !)d] = a.

Segundo. Supongamos que es cierta para n = k. Entonces,

a + (a + d) + (a + 2d) + ... + [a + (k - !)d] = ~[2a + (k - !)ti]

163

CAPITULO 18

Principio matemático de inducción completa

EL PRINCIPIO MATEMATICO DE INDUCCION COMPLETA es un procedimiento que sirve para demostrar un teorema general, o una fórmula, a partir de casos particulares. Para hacer una demostración por este método se procede de la forma siguiente : 1) Se comprueba, por simple sustitución, que el teorema propuesto, o fórmula, se verifica para los primeros valores de n, enteros y positivos, por ejemplo, n = 1, n = 2, etc. 2) Se supone que el teorema, o fórmula, es cierto para n = k y, a continuación, se demuestra que también se verifica para el siguiente n = k + 1.

PROBLEMAS RESUELTOS

l. Demostrar, por el principio del inducción completa, que para todos los valores de n, entero y positivo, se verifica: n(n + 1)

1+2+3+ ... +n=--2-

Primero. , . 1(1 + 1)

La formula se venfica para n = 1, ya que 1 = --2-- = 1.

Segundo. Supongamos que la fórmula es cierta para n = k. Entonces, sumando (k + 1) a los dos miembros,

k(k+l) (k+l)(k+2) 1 + 2 + 3 + ... + k + (k + 1) = --2- + (k + 1) = 2

n(n + 1) que es el valor de --4-- cuando se sustituye n por (k + 1).

Por tanto, si la fórmula es cierta para n = k, hemos demostrado que también se verifica para el siguiente, n = k + 1. Como la fórmula se verifica para n = 1, también se verificará para n = 1 + 1 = 2 y, por la misma ra­zón, para n = 2 + 1 = 3, y así sucesivamente. Es decir, se verifica para todos los valores de n, entero y positivo.

2. Demostrar, por el principio de inducción completa, que la suma de los n primeros términos de una progresión arit­n

mética, a, a + d, a + 2d, . .. , es i2a + (n - I)d] , es decir,

n a + (a + d) + (a + 2d) + ... + [a + (n - Ild] = 2{2a + (n - Ild]

1 Primero. La fórmula se verifica para n = 1, ya que a = i2a + (1 - 1 ld] = a.

Segundo. Supongamos que es cierta para n = k . Entonces,

a + (a + d) + (a + 2d) + . . . + [a + (k - I)d] = ~[2a + (k - l)d]

163

http://carlos2524.jimdo.com/

Page 172: Algebra Superior Murray R Spiegel

"

164 PRINCIPIO MATEMATICO DE INDUCCION COMPLETA

Sumando a los dos miembros de la última ecuación el término que ocupa el lugar (k + 1), que es igual a (a + kd),se tiene

k .a + (a + d) + (a + 2d) + ... + [a + (k - 1)d] + (a + kd) = i2a + (k - 1)d] + (a + kd)

k2d kd k2d + kd + 2ka + 2aEl segundo miembro de esta ecuación es = ka + 2 -2 + a + kd = 2

kd(k + 1) + 2a(k + 1) k + I= 2 = -2-(2a + kd)

que es el valor de i[2a + (n - 1)d] cuando se sustituye n por (k + 1).

Por tanto, si la fórmula es cierta para n = k, hemos demostrado que también se verifica para n = k + l.Como la fórmula se verifica para n = 1, también se verificará para n = 1 + 1 = 2 y, por-la misma razón, paran = 2 + 1 = 3, y así sucesivamente. Es decir, se verifica para todos los valores de n, entero y positivo.

3. Demostrar, por el principio de inducción completa, que para todos los valores de n, entero y positivo, se verifica

Primero.

2 2 2 2 n(n + 1)(2n + 1)1+2+3+ ... +n= 6

1(1 + 1)(2 + 1)La fórmula se verifica para n = 1, ya que 12 = 6 = 1.

Segundo. Supongamos que es cierta para n = k. Entonces,

2 2 2 2 k(k + 1)(2k + 1)1+2+3+ ... +k= 6

Sumando a ambos miembros de esta ecuación el término que ocupa el lugar (k + 1), que es igual a (k + 1)2,

k(k + 1)(2k + 1)12 + 22 + 32 + ... + k2 + (k + 1j2 = + (k + 1)2

6

El segundo miembro de esta ecuación k(k + 1)(2k + 1) + 6(k + 1)2I es = 6

(k + 1)[(2k2 + k) + (6k + 6)]6

(k + 1)(k + 2)(2k + 3)6

n(n + 1)(2n + 1)que es el valor de 6 cuando se sustituye n por (k + 1).

Por tanto, si la fórmula es cierta para n = k, hemos demostrado que también se verifica para n = k + 1.Como la fórmula se verifica para n = 1, también se verificará para n = 1 + 1 = 2 y, por la misma razón, paran = 2 + 1 = 3, y así sucesivamente. Es decir, se verifica para todos los valores de n; entero y positivo.

4. Demostrar, por el principio de inducción completa, que para todos los valores de n, entero y positivo, se verifica

1 1 1 1 n--+--+--+ ... + =---1 . 3 3 . 5 5 . 7 (2n - 1)(2n + 1) 2n + 1

Primero. La fórmula se verifica para n = 1, ya que 1 = 2 +1 1 = -31.

(2 - 1)(2 + 1)

Segundo. Supongamos que es cierta para n = k. Entonces

1 1 1 1 k--+--+--+ ... + =---1 . 3 3 . 5 5 . 7 (2k - 1)(2k + 1) 2k + 1

Surnai

(2k +

El segi

sustitu

p(lafóm= 3, Y

s, Demoquiera

Prime

Segun,

se d

Pla fórny así:

6. Demo

(a

para t

Prime

Segun,

Multi¡

http://carlos2524.jimdo.com/

Page 173: Algebra Superior Murray R Spiegel

kel),

1.

ara

ifica

1.

ara

fica

PRINCIPIO MATEMATICO DE INDUCCION COMPLETA 165

Sumando a los dos miembros de la ecuación anterior el término que ocupa el lugar (k + 1), que es igual a

(2k + 1)(2k + 3)

1 1 1 1 l k l-1 '-3 + -3 '-5 + -5 '-7 + ... + (2k - 1)(2k + 1) + (2k + l )(2k + 3) = -2k-+-1 + (2k-+--:-:1)-::(2C:-k-+~3)

k(2k + 3) + 1 k + 1 nEl segundo miembro de esta ecuación es = = --- , que es el valor de ._-- cuando se

(2k + 1)(2k + 3) 2k + 3 2n + lsustituye n por (k + 1).

Por tanto, si la fórmula es cierta para n = k, hemos demostrado que también se verifica para n = k + l. Comola fórmula se verifica para n = 1, también se verificará para n = 1 + 1 = 2 y, por la misma razón, para n = 2 += 3, Y así sucesivamente. Es decir, se verifica para todos los valores de n, entero y positivo.

5. Demostrar, por el principio de inducción completa, que a2" - b2" es divisible por a + b, siendo n un número cual-quiera entero y positivo.

Primero. El teorema se cumple para n = 1, ya que a2 - b2 = (a + b)(a - b).

Segundo. Supongamos que es cierto para n = k. Entonces,

a2" - b2

" es divisible por a + b

Tenemos que demostrar que a2k+2 - b2k+2 es divisible por a + b. De la identidad

se deduce que a2k+2 - b2k+ 2 es divisible por a + b si lo es a2k _ b":

Por tanto, si la fórmula es cierta para n = k, hemos demostrado que también se verifica para n = k + l. Comola fórmula se verifica paran = 1, también se verificará para n = 1 + 1 = 2y, por la misma razón, paran = 2 + 1 = 3,Y así sucesivamente. Es decir, 'Se verifica para todos los valores de n, entero y positivo.

6. Demostrar la fórmula del binomio.

n(n-I) n(n-I) (n-r+2)(a + x)" = a" + na"-Ix + - __ a"-2x2 + ... + ... a"-'+lx'-1 + ... + x"2! (r - 1)!

para todos los valores de n entero y positivo.

Primero. La fórmula se verifica para n = l.

Segundo. Supongamos que es cierta para n = k. Entonces,

" " .1<-1 k(k - 1) "-2 2 k(k - 1) ... (k - r + 2) ,,-,+1 ,-1 + ... + x"(a + x) = a + ka x + --2-!-a x + ... + (1 _ 1)1 a x

Multiplicando ambos miembros por a + x, el segundo miembro se hace igual a

ak+l + ka:» + k(k - l}a"-lx2 + .. + k(k - 1) ... (k - r + 2)a"-d2x-l + ... + ax"2! (r - 1 )1

k(k - 1) ... (k - r + 3)n"-'+2x-l +. + x"+1+a'x+ka<-lx2+ ... + (r-2)! u

k(k - 1) ... (k - r + 2)a<-d2X-l + k(k - 1) ... (k - r + 3)a"-d2x-lya que (r _ I)! (r - 2)!

PRINCIPIO MATEMATICO DE INDUCCION COMPLETA 165

Sumando a los dos miembros de la ecuación anterior el término que ocupa el lugar (k + 1), que es igual a

(2k + 1)(2k + 3)

1 1 1 1 1 k 1 - + - + - + ... + + = -- + - ___ c:-:--.,-1 . 3 3 . 5 5 . 7 (2k - 1 )(2k + 1) (2k + 1 )(2k + 3) 2k + 1 (2k + 1 )(2k + 3)

. ., k(2k + 3) + 1 k + 1 n El segundo miembro de esta ecuaClOn es = = --- , que es el valor de ._-- cuando se

(2k + 1 )(2k + 3) 2k + 3 2n + 1 sustituye n por (k + 1),

Por tanto, si la fórmula es cierta para n = k , hemos demostrado que también se verifica para n = k + 1, Como la fórmula se verifica para n = 1, también se verificará para n = 1 + 1 = 2 y, por la misma razón , para n = 2 + 1 = 3, Y así sucesivamente. Es decir, se verifica para todos los valores de n, entero y positivo.

S. Demostrar, por el principio de inducción completa, que a2n - b2n es divisible por a + b, siendo n un número cual­quiera entero y positivo.

Primero, El teorema se cumple para n = 1, ya que a2 - b2 = (a + b)(a - b),

Segundo, Supongamos que es cierto para n = k, Entonces,

a2k - b2k es divisible por a + b

Tenemos que demostrar que a2k+2 - b2k+2 es divisible por a + b. De la identidad

se deduce que a2k+ 2 - b2k+ 2 es divisible por a + b si lo es a2k _ b2k.

Por tanto, si la fórmula es cierta para n = k , hemos demostrado que también se verifica para n = k + 1, Como la fórmula se verifica paran = 1, también se verificará para n = 1 + 1 = 2y, por lamisma razón , paran = 2 + 1 = 3, Y así sucesivamente. Es decir, 'Se verifica para todos los valores de n, entero y positivo,

6. Demostrar la fórmula del binomio.

n(n - 1) (a + x)n = an + nan-lx + ___ an- 2x2 +

2! + n(n - 1) . . . (n - r + 2)an - ,+lr- 1 + . .. + x"

(r - 1)'

para todos los valores de n entero y positivo.

Primero, La fórmula se verifica para n = 1,

Segundo. Supongamos que es cierta para n = k. Entonces,

k k -'< - ' k(k-l)k_22 k(k - I)."(k-r +2) ak- ' +lx' - I +,,.+xk (a+x) =a +ku x+--

2-!-a x +".+ (1-1)'

Multiplicando ambos miembros por a + x, el segundo miembro se hace igual a

ak.+ 1 k(k - 1) 2 k(k - 1) ... (k - r + 2) '-,+2r - ' . .k

+ ka'x+ --2-,- ak- lx+". + (r-1)' a +.,,+ ax

+ akx + kd - IX2 + + k(k - 1) . . . (k - r + 3) k-.+2r - ' + + ~+l

. . . (r _ 2)! -a . , ,

k(k - 1) ... (k - r + 2)d - , +2r -1 + k(k - 1), . . (k - r + 3)ak- ' +2,'(' - 1 ya que (r _ I)! (r - 2)!

http://carlos2524.jimdo.com/

Page 174: Algebra Superior Murray R Spiegel

166 PRINCIPIO MATEMATICO DE INDUCCION COMPLETA

= k(k - 1) ... (k - r + 3)tf-'+2X'-1 {k - r + 2 + 1} = (k + I)k(k - 1) ... (k - r + 3)tf-,+2x'-1

(r - 2)! r - 1 (r - I)! '

luego

(a +x¡HI = tf+1 + (k + l)akX +... + (k + I)k(k - 1) ... (k - r + 3)tf-d2x'-1 + ... + x"+1(r - I)!

que es la fórmula del binomio cuando se sustituye n por k + 1.

Por tanto, si la fórmula es cierta para n = k, también se verifica para n = k + 1. Como la fórmula se veri-fica para n = 1, también se verificará para n = 1 + 1 = 2, y así sucesivamente. Es decir, se verifica para todos losvalores de n, entero y positivo.

PROBLEMAS PROPUESTOS

Demostrar, por el principio de inducción completa, que se verifican las siguientes expresiones, siendo n un númeroentero y positivo.

7. 1 + 3 + 5 + ... + (2n - 1) = n2

3" - 18. 1 + 3 + 32 + ... + 3"-1 =--

2

. 2 _ 1 a(r" - 1)10. a +ar +ar + ... + ar" =--;:--=-¡, r i= 1

1 1 1 1 n11. -+-+-+ ... +---=--

l : 2 2·3 3' 4 n(n + 1) n + 1

(2n - 1)3"+1 + 312. l' 3 +2 . 32 + 3 . 33 +... +n • 3" =---'-.,.---

4

1 1 1 1 n13. -+-+--+ ... + =--

2' 5 5·8 8' 11 (3n - 1)(3n + 2) 6n + 4

1 1 1 1 n(n + 3)14. --+--+--+ ... + =

1'2·32'3'43'4'5 n(n+I)(n+2) 4(n+l)(n+2)

15. a" - b" es divisible por a - b, para n = entero y positivo.

16. a2"-1 + b2"-1 es divisible por a + b, para n = entero y positivo.

DEFINICnor q

A

1) a2) a3) a4) a5) O

6)

T..

que iry b, Y

T..

letras.

TEOREM

1) Esus dedad n

F

2) Ereal s

3) Enegat

4) S

166 PRINCIPIO MATEMATICO DE INDUCCION COMPLETA

= k(k - 1) ... (k - r + 3)tf-'+2X'-1 {k - r + 2 + 1} = (k + I)k(k - 1) ... (k - r + 3)tf-H2x'-1

(r - 2)! r - 1 (r - I)! '

luego

(a + X¡>+1 = aH1 + (k + l)akX + .. . + (k + I)k(k - 1) ... (k - r + 3)tf-H2x'-1 + ... + x"+1 (r - 1)!

que es la fórmula del binomio cuando se sustituye n por k + 1.

Por tanto, si la fórmula es cierta para n = k, también se verifica para n = k + 1. Como la fórmula se veri­fica para n = 1, también se verificará para n = 1 + 1 = 2, y así sucesivamente. Es decir, se verifica para todos los valores de n, entero y positivo.

PROBLEMAS PROPUESTOS

Demostrar, por el prillcipio de inducción completa, que se verifican las siguientes expresiones, siendo n un número entero y positivo.

7. 1 + 3 + 5 + ... + (2n - 1) = n2

3" - 1 8. 1 + 3 + 32 + ... + 3"-1 = -2-

. 2 _ 1 a(r" - 1) 10. a + ar + ar + ... + ar" = --;:-=¡-, r i= 1

1 1 1 1 n 11. -+-+-+ . . . +---=--

l' 2 2·3 3· 4 n(n + 1) n + 1

(2n - 1)3"+1 + 3 12. l' 3 + 2 . 32 + 3 . 33 + ... + n • 3" = -'---'--- -

4

1 1 1 1 n 13. -+-+--+ . .. + =--

2'55·88'11 (3n-I)(3n+2) 6n+4

1 1 1 1 n(n + 3) 14. --+--+--+ ... + =

1'2·32'3'43'4 ' 5 n(n+I)(n+2) 4(n+l)(n+2)

15. a" - b" es divisible por a - b, para n = entero y positivo.

16. a2"-1 + b2

"-1 es divisible por a + b, para n = entero y positivo.

http://carlos2524.jimdo.com/

Page 175: Algebra Superior Murray R Spiegel

-1

+1

se veri-todos los

número

CAPITULO 19

Desigualdades

DEFINICIONES.nor que otra.

Una desigualdad expresa que una cantidad real, o una expresión, es mayor o me-

A continuación se indica el significado de los signos de desigualdad.

1) a> b significa que «a es mayor que b» (o bien que a - b es un número positivo).

2) a < b significa que «a es menor que b» (o bien que a - b es un número negativo).

3) a ~ b significa que «a es mayor o igual que b»,

4) a ~ b significa que «a es menor o igual que b».

5) O < a < 2 significa que «a es mayor que cero, pero menor que 2».

6) -2 ~ x < 2 significa que «x es mayor o igual que -2, pero menor que 2».

Una desigualdad absoluta es aquella que se verifica para todos los valores reales de las letrasque intervienen en ella. Por ejemplo, (a - b)2 > -1 es cierta para todos los valores reales de ay b, ya que el cuadrado de todo número real es un número positivo o cero.

Una desigualdad condicional es aquella que solo es cierta para determinados valores de lasletras. Por ejemplo, x - 5 > 3 solo es verdad para x mayor que 8.

Las desigualdades a > b y e > d son del mismo sentido. Las desigualdades a > b y x < yson de sentido contrario.

TEOREMAS DE LAS DESIGUALDADES

1) El sentido de una desigualdad no se modifica si se suma, o se resta, un mismo número real asus dos miembros. Por consiguiente, para pasar un término de un miembro a otro de una desigual-dad no hay más que cambiarle de signo.

Por ejemplo, si a > b, se tiene a + e > b + e, y a - e > b - c, y a - b > O.

2) El sentido de una desigualdad no se altera si se multiplica, o divide, por un mismo númeroreal sus dos miembros.

Por ejemplo, si a > b y k > O, se tiene ka > kb y I > ~ .

3) El sentido de una desigualdad se invierte cuando se multiplica, o divide, por un mismo númeronegativo sus dos miembros.

Por ejemplo, si a > b y k < O, se tiene ka < kb y I < ~ .

4) Si a > b y a, b, n son positivos, se tiene a" > b", pero a-n < h-n.

167

CAPITULO 19

Desigualdades

DEFINICIONES. Una desigualdad expresa que una cantidad real, o una expresión, es mayor o me­nor que otra.

A continuación se indica el significado de los signos de desigualdad.

1) a > b significa que «a es mayor que b» (o bien que a - b es un número positivo).

2) a < b significa que «a es menor que b» (o bien que a - b es un número negativo).

3) a ~ b significa que «a es mayor o igual que b».

4) a:::;; b significa que «a es menor o igual que b».

5) O < a < 2 significa que «a es mayor que cero, pero menor que 2».

6) -2:::;; x < 2 significa que «x es mayor o igual que -2, pero menor que 2».

Una desigualdad absoluta es aquella que se verifica para todos los valores reales de las letras que intervienen en ella. Por ejemplo, (a - b)2 > -1 es cierta para todos los valores reales de a y b, ya que el cuadrado de todo número real es un número positivo o cero.

Una desigualdad condicional es aquella que solo es cierta para determinados valores de las letras. Por ejemplo, x - 5 > 3 solo es verdad para x mayor que 8.

Las desigualdades a > b y c > d son del mismo sentido. Las desigualdades a > b y x < y son de sentido contrario.

TEOREMAS DE LAS DESIGUALDADES

1) El sentido de una desigualdad no se modifica si se suma, o se resta, un mismo número real a sus dos miembros. Por consiguiente, para pasar un término de un miembro a otro de una desigual­dad no hay más que cambiarle de signo.

Por ejemplo, si a > b, se tiene a + c > b + c, y a - c > b - c, y a - b > O.

2) El sentido de una desigualdad no se altera si se multiplica, o divide, por un mismo número real sus dos miembros.

Por ejemplo, si a > b y k > O, se tiene ka > kb y I > ~ .

3) El sentido de una desigualdad se invierte cuando se multiplica, o divide, por un mismo número negativo sus dos miembros.

Por ejemplo, si a > b y k < O, se tiene ka < kb y I < ~ .

4) Si a > b y a, b, n son positivos, se tiene a" > b", pero a-o < h-".

167

http://carlos2524.jimdo.com/

Page 176: Algebra Superior Murray R Spiegel

168 DESIGUALDADES

Ejemplos.1 1

5 > 4; se tiene 53 > 43 o 125 > 64, pero 5-3 < 4-3 o 125 < 64 .

1 116> 9,' se tiene 161/2> 91/2 04> 3, pero 16-1/2 < 9-1/2 o <4 }.

5) Si a > b Y e > d, se tiene (a + c) > (b + di·

6) Si a > b > ° y C > d > 0, se tiene ac > bd.

PROBLEMAS RESUELTOS

1. Si a > b Y e > d, demostrar que a + e > b + d.

Como (a - b) Y (e - d) son ambos positivos, (a - b) + (e - d) es positivo.

Luego (a - b) + (e - d) > O, (a + e) - (b + d) > O Y (a + e) > (b + d).

2. Encontrar el error en el siguiente razonamiento:a) Sean a = 3, b = 5, es decir,b) Multiplicando por a,e) Restando b2

,

d) Descomponiendo en factores,e) Dividiendo por a - b,1) .Sustituyendo a = 3, b = 5,

a < ba2 < ab

a2 _ b2 <ab _ b2

(a + b)(a - b) < b(a - b)a+b<b

8 < 5

Los pasos a), b), e), d) son correctos. El error se comete en e), ya que la desigualdad se divide por a - b quees un número negativo y, por tanto, hay que invertir el sentido de la desigualdad.

3. Hallar los valores de x para los cuales se verifican las desigualdades siguientes:

dr

se

qi

S. D

6. D

7. Si

8. DI

sit

a) 4x + 5 > 2x + 9. Tenemos 4x - 2x > 9 - 5, 2x> 4 y x> 2.se

x 1 2x 1b) 2-3<3+2' Multiplicando por 6 se obtiene

3x - 2 < 4x + 3, 3x - 4x < 2 + 3, -x < 5, x> -5M

e) x2 < 16.9. O

Método J. x2 - 16 < O, (x - 4)(x + 4) < O. El producto de los factores (x - 4) por (x + 4) es negati-vo. Son posibles dos casos:

1) x - 4 > O Y x + 4 < O simultáneamente. Es decir, x> 4 y x < -4. Esto es imposible, ya que x no puedeser, al mismo tiempo, mayor que 4 y menor que -4.

2) x - 4 < O y x + 4 > O simultáneamente. Es decir, x < 4 y x > -4. Esto es posible únicamente cuando- 4 < x < 4. Luego

Método 2. (X2)1/2 < (16)1/2 Ahora bien (X2)1/2 = x si x;;; O, Y (X2)1/2 = -x si x ~ O.

Si x ;;; O, (X2)1/2 < (16)1/2 se puede escribir x < 4. Luego O :;;; x < 4.

Si x ~ O, (X2)1/2 < (16)1/2 se puede escribir -x < 4 o bien x> -4. Luego -4 < x ~ O.

Así, pues, O:;;; x < 4 Y -4 < x ~ O, o sea -4 < x < 4.

4. Demostrar que a2 + b2 > 2ab si a y b son números reales.

E:

10. DI

Sidu

http://carlos2524.jimdo.com/

Page 177: Algebra Superior Murray R Spiegel

- b que

negati-

o puede

cuando

DESIGUALDADES 169

Si a2 + b2 > 2ab, se tiene, a2 + b2 - 2ab > O, o sea (a - b)2 > O. Esto siempre es cierto, ya que el cua-drado de un número real cualquiera, distinto de cero, es positivo.

El razonamiento anterior sugiere un método de hacer una demostración. Partiendo de (a - b)2 > O, quese verifica siempre que a + b, se obtiene a2 - 2ab + b2 > O, o sea a2 + b2 > 2ab.

Obsérvese que esta demostración es, esencialmente, la misma que la del primer párrafo, pues no hay másque invertir el orden de los sucesivos pasos de que consta.

S. Demostrar que la suma de un número positivo cualquiera con su recíproco nunca es menor que 2.

Tendremos que demostrar que (a + l/a) ;::;2 si a > O.

Si (a + l/a) ;::;2, se tiene a2 + 1 ;::; 2a, a2 - 20 + 1 ;::; O Y (a - 1)2 ;::; O, lo cual es cierto.

Para demostrar el teorema, partimos de (a - 1)2 ;::; O, que sabemos es verdad.

Por tanto, a2 - 2a + 1 ;::; O, a2 + 1 ;::; 20, y dividiendo por a, a + l/a;::; 2.

6. Demostrar que a2 + b2 + e2 > ab + be + ea para todos los valores reales de a, b, e salvo para a = b = e.

Como a2 + b2> 2ab, b2 + e2 > 2be, e2 + a2 > 2ea (véase Problema 4), sumando tendremos

2(a2 + b2 + e2) > 2(ab + be + ea) o a2 + b2 + e2 > ab + be + ea

(Si a = b = e, será a2 + b2 + e2 = ab + be + ea).

7. Si a2 + b2 = 1 Y e2 + d2 = 1, demostrar que ae + bd < 1.

a2 + e2 > 2ae y b2 + d2 > 2bd; luego, sumando

(a2 + b2) + (e2 + d2) > 2ae + Zbd , o 2> 2ae + 2bd, es decir, 1 > ae + bd

8. Demostrar que x3 + y3 > x2y + y2x, si x e y son números reales positivos y distintos.

Si x3 + y3 > x2y + y2x se tiene (x + y)(x2 - xy + y2) > xy(x + y). Dividiendo por x + y, que es po-sitivo,

x2 - xy + y2 > xy o x2 - 2xy + y2 > O, es decir, (x - y)2 > O que es cierto si x + y

La demostración se puede llevar a cabo razonando en sentido inverso. Partiendo de (x - y)2 > O, X + y,se obtiene

x2 _ xy + y2 > xy

Multiplicando ambos miembros por x + y, tenemos (x + y)(x - xy + y2) > xy(x + y) o x3 + y3 > x2y + y2x.

9. Demostrar que ti' + b" > a"-Ib + ab":", siempre que a y b sean positivos y distintos, y n > 1.

Si ti' + b" > ti'-Ib + ab">, será (ti' - ti'-Ib) - (ab":! -bol > O o

a"-I(a - b) - b"-I(a - b) > O, es decir, (a"-I - b"-I)(a - b) > O

Esto es cierto, ya que ambos factores son positivos o negativos.

También se puede demostrar razonando en orden inverso al expuesto.

1 110. Demostrar que a3 + 3" > a2 + 2" si a > O Y a + 1.

a a

Multiplicando ambos miembros de la desigualdad por a3 (que es positivo, ya que a > O), tenemos

a6 + 1 > aS + a, a6 - aS - a + 1 > O y (aS - 1 )(a - 1) > O

Si a > 1 ambos factores son positivos, mientras que si O < a < 1 ambos son negativos. En cualquier caso el pro-ducto es positivo. (Si a = 1, el producto es cero.)

También se puede. demostrar razonando en orden inverso.

DESIGUALDADES 169

Si a2 + b2 > 2ab, se tiene, a2 + b2 - 2ab > O, o sea (a - b)2 > O. Esto siempre es cierto, ya que el cua­drado de un número real cualquiera, distinto de cero, es positivo.

El razonamiento anterior sugiere un método de hacer una demostración. Partiendo de (a - b)2 > O, que se verifica siempre que a + b, se obtiene a2 - 2ab + b2 > O, o sea a2 + b2 > 2ab.

Obsérvese que esta demostración es, esencialmente, la misma que la del primer párrafo, pues no hay más que invertir el orden de los sucesiwos pasos de que consta.

S. Demostrar que la suma de un número positivo cualquiera con su recíproco nunca es menor que 2.

Tendremos que demostrar que (a + l /a) ;:::; 2 si a > O.

Si (a + l /a) ;:::; 2, se tiene a2 + 1 ;:::; 2a, a2 - 20 + 1 ;:::; O Y (a - 1)2 ;:::; O, lo cual es cierto.

Para demostrar el teorema, partimos de (a - 1)2 ;:::; O, que sabemos es verdad.

Por tanto, a2 - 2a + 1 ;:::; O, a2 + 1 ;:::; lo, y dividiendo por a, a + l /a;:::; 2.

6. Demostrar que a2 + b2 + e2 > ab + be + ea para todos los valores reales de a, b, e salvo para a = b = e.

Como a2 + b2 > 2ab, b2 + e2 > 2be, e2 + a2 > 2ea (véase Problema 4), sumando tendremos

2(a2 + b2 + e2) > 2(ab + be + ea) o a2 + b2 + e2 > ab + be + ea

(Si a = b = e, será a2 + b2 + e2 = ab + be + ca).

7. Si a2 + b2 = l Y e2 + d2 = 1, demostrar que ae + bd < 1.

a2 + e2 > 2ae y b2 + d2 > 2bd; luego, sumando

(a2 + b2) + (e2 + d2) > 2ae + 2bd. o 2> loe + 2bd, es decir, 1 > ae + bd

8. Demostrar que x 3 + y3 > x2y + y2x, si x e y son números reales positivos y distintos.

Si x 3 + y3 > x2y + y2x se tiene (x + y)(x2 - xy + y2) > xy(x + y). Dividiendo por x + y, que es po_ sitivo,

X2 - xy + y2 > xy o X2 - 2xy + y2 > O, es decir, (x - y)2 > O que es cierto si x + y

La demostración se puede llevar a cabo razonando en sentido inverso. Partiendo de (x - y)2 > O, X + y, se obtiene

X2 _ xy + y2 > xy

Multiplicando ambos miembros por x + y , tenemos (x + y)(x - xy + y2) > xy(x + y) o x3 + y3 > x2y + y2x.

9. Demostrar que e/' + b' > an-Ib + ab,-I , siempre que a y b sean positivos y distintos, y n > 1.

Si e/' + b' > e/' - Ib + ab,-I , será (e/' - e/'-Ib) - (ab,-I - b') > O o

a,-I(a - b) - b, - I(a - b) > O, es decir, (a,-I - bn-I)(a - b) > O

Esto es cierto, ya que ambos factores son positivos o negativos.

También se puede demostrar razonando en orden inverso al expuesto.

1 1 10. Demostrar que a3 + 3" > a2 + 2" si a > O Y a + 1.

a a

Multiplicando ambos miembros de la desigualdad por a3 (que es positivo, ya que a > O), tenemos

a6 + 1 > aS + a, a6 - aS - a + 1 > O y (aS - 1 )(a - 1) > O

Si a > 1 ambos factores son positivos, mientras que si O < a < 1 ambos son negativos. En cualquier caso el pro­ducto es positivo. (Si a = 1, el producto es cero.)

También se puede. demostrar razonando en orden inverso.

http://carlos2524.jimdo.com/

Page 178: Algebra Superior Murray R Spiegel

170 DESIGUALDADES

11 S' b d . . . a e d a + e e. I a, ,e, ,son numeros POSItiVOSy b > d' emostrar que b + d > d'

S· a + e el"Método l. I b + »: d' mu tiplicando por d(b + d), se obtiene

a e(a + e)d> e(b + d), ad + cd > be + cd, ad > be; dividiendo por bd resulta, b > d' que se verifica por hipó-

tesis. También se demuestra razonando en orden inverso.

a e . a e e e a + e e(b + d)Método 2. Como b> d' se tiene b + b > d + 'i: -b- > bd

12. Demostrar que a) x2 - y2 > X - Y si x + y > y x > y

b) x2 - y2 < X - Y si x + y > y x < y

a) Como x > y, x - y > O. Multiplicando ambos miembros de x + y > por el número positivo x - y,

(x + y)(x - y) > (x - y) o x2 - y2 > X - Y

b) Como x < y, x - y < O. Multiplicando ambos miembros de x + y > 1 por el número negativo x - y,se invierte el sentido de la desigualdad; luego

(x + y)(x - y) < (x - y) o x2 - / < x - y

13 L d' . .. d dos ni b a + bId' .. Cb l di . . 2ab. a me la antmetica e os numeros a y es -2-' a me la geometnca es y ao, y a me la armoruca es a + b .

a v b r: 2abDemostrar que -- > y ab > -- si a y b son positivos y distintos.

2 a + b

a) Si a; b > fo, se tiene (a + b)2 > (2fo)2, a2 + 2ab + b2 > 4ab, a2 - 2ab + b2 > O Y

a+b eL.(a - b)2 > Oque es cierto si a + b. Invirtiendo el razonamiento, tenemos -2-- > yab.

eL. 2ab 4a2b2b) Si yab > --b' se tiene ab > -(---2'

a + a + b)

eL. 2abInvirtiendo el razonamiento, tenemos yab > --b'

a+

(a + b)2 > 4ab y (a - bl > O queesciertosia + b.

a + b eL. 2abDe a) y b), -- > yab > --b'

2 a +

14. Hallar los valores de x para los cuales a) x2 - 7x + 12 = O, b) x2 - 7x + 12 > O, e) x2 - 7x + 12 < O.

a) x2 - 7x + 12 = (x - 3)(x - 4) = O para x = 3 ó 4.

b) x2 - 7x + 12> O o (x - 3)(x - 4) > O para (x - 3) > O Y (x - 4) > O simultáneamente, o para(x - 3) < O Y (x - 4) < O simultáneamente.

(x - 3) > O Y (x - 4) > O simultáneamente cuando x > 3 Y x > 4, es decir, cuando x > 4

(x - 3) < O Y (x - 4) < O simultáneamente cuando x < 3 Y x < 4, es decir, cuando x < 3,

Luego x2 - 7x + 12 > O se verifica cuando x > 4 o x < 3.

e) x2 - 7x + 12 < O o (x - 3)(x - 4) < O cuando (x - 3) > O Y (x - 4) < O simultáneamente, ocuando (x - 3) < O Y (x - 4) > O simultáneamente.

(x - 3) > O Y

(x - 3) < O Y

(x - 4) < O simultáneamente cuando x > 3 Y x < 4, es decir, cuando 3 < x < 4.

(x - 4) > O simultáneamente cuando x < 3 Y x> 4, que es absurdo.

Luego x2 - 7x + 12 < O se verifica cuando 3 < x < 4.

15. Deter

1Y =;

16. Si a

17. Si a

18. Halla

al 2

19. ¿par¡

20. Demcuam

21. Dem

22. Dem

23. Dem

24. Si a

25. Dem

26. Dete

a) ;

27. Dete

SOLUC](

18. a)

19. a>

26. a)

27. a)

http://carlos2524.jimdo.com/

Page 179: Algebra Superior Murray R Spiegel

y

b.

DESIGUALDADES 171

15. Determinar gráficamente el campo de variación de x definido por

a) x2 + 2x - 3 ; Ob) x2 + 2x - 3 > Oe) x2 + 2x - 3 < O

La figura representa la gráfica de la función definida pory = x2 + 2x - 3. De ella se deduce que

a) y = O para x = 1, x = -3b) y> O para x> 1 o x < -3e) y < O para --=3 < x < 1

PROBLEMAS PROPUESTOS16. Si a > b, demostrar que a - e > b - e siendo e un número real cualquiera.

17. Si a > b Y k > O demostrar que ka > kb.

18. Hallar los valores de x para los cuales se verifican las desigualdades siguientes:

a) 2(x + 3) > 3(x - 1) + 6x 2 2x 1

b) - + - < - --4 3 3 6

1 3 7e) :; + 4x > 8 d) x2 > 9

19. ¿Para qué valores de a será (a + 3) < 2(2a + 1)?

20. Demostrar que -Ha2 + b2) !1; ab para todos los valores reales de a y b, Y que la igualdad solo se verificacuando a = b.

1 1 2 _ _ .21. Demostrar que - + - > -- SI X e y son pOSitiVOS y x =F y.

x y x+y

x2 + y2Demostrar que --- < x + y

x+y22. si x> O, y> O.

23. Demostrar que xy + 1 !1; x + y si x !1; 1 e y !1; 1 o si x ~ 1 e y ~ 1.

24.1 1

Si a > O, a =F 1 Y n es un entero positivo, demostrar que e: 1 + an+ 1 > a" + ;n -

25. Demostrar que j2 + j6 < J3 + .¡s.26. Determinar los valores de x para los cuales se verifican las desigualdades siguientes:

l 7d) 3x + - > -

x 2

27. Determinar gráficamente el campo de variación de x definido por a) x2 - 3x - 4 > O, b) 2X2 - 5x + 2 < O.

a) x2 + 2x - 24 > O b) x2 - 6 < x e) 3x2 - 2x < 1

SOLUCIONES DE LOS PROBLEMAS PROPUESTOS

18. a) x < 3

119. a> '3

b) x> 2 e) O < x < 2 d) x < -3 o x> 3

26. a) x> 4 o x < -6 b)1-- < x <3

2 1d) .r > '3 o O < x < 2-2 < x < 3 e)

27. a) x > 4 o x < -11

b) - < x < 22

DESIGUALDADES

15. Determinar gráficamente el campo de variación de x definido por

a) X2 + 2x - 3 ,O b) X2 + 2x - 3 > O e) X2 + 2x - 3 < O

La figura representa la gráfica de la función definida por y = X2 + 2x - 3. De ella se deduce que

a) y = O para x = 1, x = -3

b) y> O para x> 1 o x < -3 e) y < O para -=3 < x < 1

PROBLEMAS PROPUESTOS

16. Si a > b, demostrar que a - e > b - e siendo e un número real cualquiera.

17. Si a > b Y k > O demostrar que ka > kb.

18. Hallar los valores de x para los cuales se verifican las desigualdades siguientes:

a) 2(x + 3) > 3(x - 1) + 6 x 2 2x 1

b) - + - < - --4 3 3 6

1 3 7 e) - + - >-

x 4x 8

19. ¿Para qué valores de a será (a + 3) < 2(2a + I)?

171

d) X2 > 9

20. Demostrar que i(a2 + b2) ~ ab para todos los valores reales de a y b, Y que la igualdad solo se verifica

cuando a = b.

1 1 2 . . Demostrar que - + - > -- SI X e y son positivos y x *" y.

x y x+y 21.

X2 + y2 22. Demostrar que --- < x + y si x > O, Y > O.

x+y

23. Demostrar que xy + 1 ~ x + y si x ~ 1 e y ~ 1 o si x ~ 1 e y ~ 1.

24. 1 1

Si a > O, a *" 1 Y n es un entero positivo, demostrar que an+ 1 + --... > a" + - .

a" a"

25. Demostrar que j2 + j6 < j3 + fi· 26. Determinar los valores de x para los cuales se verifican las desigualdades siguientes:

a) Xl + 2x - 24 > O b) X2 - 6 < x e) 3X2 - 2x < 1 1 7

d) 3x + - > -x 2

27. Determinar gráficamente el campo de variación de x definido por a) X2 - 3x - 4 > O, b) 2Xl - 5x + 2 < O.

SOLUCIONES DE LOS PROBLEMAS PROPUESTOS

18. a) x < 3 b) x>2 e) 0 <x<2 d) x<-30x>3

1 19. a> -

3

1 26. a) x>4 o x < -6 b) -2 < x < 3 e) -3 < x <

2 1 d) x > 3 o O < x < 2

27. a) x>4 o x < -1 b) 1 2<x<2

http://carlos2524.jimdo.com/

Page 180: Algebra Superior Murray R Spiegel

CAPITULO 20

Forma polar de los números complejos

UN NUMERO COMPLEJO es una expresión de la forma a + bi en la que a y b son números realese i = p. La suma, resta, multiplicación y división de números complejos se ha expuesto enel Capítulo 8.

REPRESENTACION GRAFICA DE LOSNUMEROS COMPLEJOS

Empleando un sistema de coordenadas rec-tangular, el número complejo x + yi se representapor, o se corresponde con, el punto cuyas coorde-nadas son (x, y). Por ejemplo:

Para representar el número complejo 3 + 4i,se llevan 3 unidades sobre el eje X' X hacia la dere-cha de O y, acto seguido, 4 unidades hacia arriba.

Para representar el número - 2 + 3i, se llevan2 unidades sobre el eje X' X hacia la izquierda de Oy, luego, 3 unidades hacia arriba.

Para representar el número - 1 - 4i, se llevaunidad sobre el eje X' X hacia la izquierda de O

y, a continuación, 4 unidades hacia abajo.

y

x

3 + 4i

-2 + 3i

2i

I-3 4

O

-2i

-1-4i 2-4i

yl

Para representar el número 2 - 4i, se llevan 2 unidades sobre el eje X' X hacia la derecha deO y, luego, 4 unidades hacia abajo.

Los números imaginarios puros (como son 2i, - 2i) vienen representados por los puntos deleje Y' Y. Los números reales (como son 4, - 3) son los puntos del eje X' X.

FORMA POLAR DE LOS NUMEROS COMPLEJOS

En la figura, x = r cos e e y = r sen e. Por tanto,

x + yi = r(cos O + i sen O)

La expresión r(cos e + i sen e) es la forma polar,y x + yi es la forma binómica del mismo número com-plejo.

La longitud r = Jx2 + y2 es siempre positiva y sellama módulo o valor absoluto, del número complejo.El ángulo O se denomina amplitud o argumento.

172

y

% + yi

y

--~~~---%-----L-----X

x

Lasak

MULT

me

DIVISI

gur

FORMI

Estplo

RAICE:

Lm

http://carlos2524.jimdo.com/

Page 181: Algebra Superior Murray R Spiegel

realeslo en

x

yi

x

FORMA POLAR DE LOS NUMEROS COMPLEJOS 173

MUL TIPLICACION DE NUMEROS COMPLEJOS ESCRITOS EN FORMA POLAR

El módulo del producto de dos números complejos es el producto de sus módulos y el argu-mento es la suma de sus argumentos.

Por ejemplo, 4(cos 45° + i sen 45°) 7(cos 30° + i sen 30°) = 28(cos 75° + i sen 75°).

DIVISION DE NUMEROS COMPLEJOS ESCRITOS EN FORMA POLAR

El módulo del cociente de los números complejos es igual al cociente de los módulos y el ar-gumento es igual a la diferencia de los argumentos del dividendo y divisor.

6(cos 82° + i sen 82°) o. °Por ejemplo, 2( 500 . 500) = 3(cos 32 + 1 sen 32 ).cos + 1 sen

FORMULA DE MOIVRE. La potencia enésima r(cos (J + i sen (J) es

[r(cos (J + i sen (J)J" = r"(cos n(J + i sen n(J)

Esta relación es la fórmula de Moivre y se verifica para todo valor real del exponente. Por ejem-plo, si el exponente es una fracción lln,

[r(cos (J + i sen (J)Jl/" = r1/"(cos ~ + i sen ~)n n

RAICES DE UN NUMERO COMPLEJO EN FORMA POLAR. Si k es un entero cualquiera,

cos (J = cos «(J + k 360°) Y sen (J = sen «(J + k 360°)

Luego (x + yi)l/" = [r(cos (J + i sen (J)Jl/"

= {r[cos «(J + k 360°) + i sen «(J + k 3600)W1"

= r1/"[ cos (J + k 360° + i sen _(J _+_k_3_60_o]n n

Un número cualquiera (real o complejo), excepto el cero, tiene n raíces enésimas distintas.Las n raíces enésimas de un número complejo x + yi, o bien r (cos (J + i sen (J), se obtienen dandoa k los valores sucesivos O, 1,2, 3, ... , n - 1, en la fórmula anterior.

FORMA POLAR DE LOS NUMEROS COMPLEJOS 173

MUL TIPLICACION DE NUMEROS COMPLEJOS ESCRITOS EN FORMA POLAR

El módulo del producto de dos números complejos es el producto de sus módulos y el argu­mento es la suma de sus argumentos.

Por ejemplo, 4(cos 45° + ¡sen 45°) 7(cos 30° + ¡sen 30°) = 28(cos 75° + i sen 75°).

DIVISION DE NUMEROS COMPLEJOS ESCRITOS EN FORMA POLAR

El módulo del cociente de los números complejos es igual al cociente de los módulos y el ar­gumento es igual a la diferencia de los argumentos del dividendo y divisor.

. 6(cos 82° + i sen 82°) o . . ° Por ejemplo, 2( 500 . 500) = 3(cos 32 + I sen 32 ). cos + 1 sen

FORMULA DE MOIVRE. La potencia enésima r(cos {} + i sen (}) es

[r(cos {} + i sen {})J" = r"(cos n{} + ¡sen n{})

Esta relación es la fórmula de Moivre y se verifica para todo valor real del exponente. Por ejem­plo, si el exponente es una fracción Iln,

[r(cos {} + i sen (})Jl /" = r1 /"(cos f!.. + ¡sen f!..) n n

RAICES DE UN NUMERO COMPLEJO EN FORMA POLAR. Si k es un entero cualquiera,

cos {} = cos ({} + k 360°) Y sen {} = sen ({} + k 360° )

Luego (x + yi) l /" = [r(cos {} + i sen {})J 1/"

= { r[cos ({} + k 360°) + i sen ({} + k 3600 )W1"

= r1/"[ cos {} + k 360° + i sen _{}_+_k_3_60_o ] n n

Un número cualquiera (real o complejo), excepto el cero, tiene n raíces enésimas distintas. Las n raíces enésimas de un número complejo x + yi, o bien r (cos {} + i sen (}), se obtienen dando a k los valores sucesivos O, 1,2, 3, . .. , n - 1, en la fórmula anterior.

http://carlos2524.jimdo.com/

Page 182: Algebra Superior Murray R Spiegel

174 FORMA POLAR DE LOS NUMEROS COMPLEJOS

PROBLEMAS RESUELTOS

SUMA Y RESTA GRAFICAS DE NUMEROS COMPLEJOS

1. Efectuar algebraica y gráficamente las operaciones indicadas:a) (2 + 6i) + (5 + 3i), b) (-4 + 2i) - (3 + 5i).

y P .7+9t y

--~---------------------x

---------T--------~~---x

(a)

a) Algebraicarnente, (2 + 6i) + (5 + 3i) = 7 + 9i.

Gráficamente. Representemos los dos números complejos por los puntos p¡ y P2, respectivamente,como indica la Figura (a). Uniendo P, y P2 con el origen O y completando el paralelogramo de lados adyacen-tes OP¡ y OP2, el vértice P (punto 7 + 9i) representa la suma de los números complejos dados.

b) Algebraicamente. ( - 4 + 2i) - (3 + 5i) = - 7 - 3i.

Gráficamente. ( - 4 + 2i) - (3 + 5i) = (- 4 + 2i) + (- 3 + 5i). Sumemos ahora ( - 4 + 2i) con ( - 3 - 5i).

Representemos los dos números complejos (-4 + 2i) Y(-3 - 5i) por los puntos P, y P2, respectivamente,como indica la Figura (b). Uniendo P; y P2 con el origen O y completando el paralelogramo de lados adyacen-tes OP¡ Y OP2, el vértice P (punto -7 - 3i) representa la diferencia (-4 + 2i) - (3 + 5i). .

FORMA POLAR DE LOS NUMEROS COMPLEJOSy

Hallar la forma polar de los números complejos siguientes:

2. 2 + 2i 2+ 2i

2Amplitud o argumento, f) = tg-¡ 2: = tg " ¡) = 45°.

Módulo o valor absoluto, r = .J22+22 = J8 = 2fi·

Luego 2 + 2i = ricos O + i sen O)

= 2fi (cos 450 + i sen 45")

~~--~~----~---x

3. )-

4. -3

S. -)

6. J:

7. Ha

al

bl

el

ifl

174 FORMA POLAR DE LOS NUMEROS COMPLEJOS

PROBLEMAS RESUELTOS

SUMA Y RESTA GRAFICAS DE NUMEROS COMPLEJOS

l. Efectuar algebraica y gráficamente las operaciones indicadas: a) (2 + 6i) + (5 + 3i), b) ( - 4 + 2i) - (3 + 5i).

y P . 7+9t y

----------+---------~~---x

--~---------------------x

(a)

a) Algebraicamente, (2 + 6i) + (5 + 3i) = 7 + 9i.

Gráficamente. Representemos los dos números complejos por los puntos PI y P2 , respectivamente, como indica la Figura (a). Uniendo PI y P 2 con el origen O y completando el paralelogramo de lados adyacen­tes OPI y OP2 , el vértice P (punto 7 + 90 representa la suma de los números complejos dados.

b) Algebraicamente. ( - 4 + 2i) - (3 + 5i) = - 7 - 3i.

Gráficamente. ( - 4 + 2i) - (3 + 5i) = ( - 4 + 2i) + (- 3 + 5i). Sumemos ahora ( - 4 + 2i) con ( - 3 - 5i).

Representemos los dos números complejos (-4 + 2i) Y (-3 - 5i) por los puntos PI y P2 , respectivamente, como indica la Figura (b). Uniendo PI y P 2 con el origen O y completando el paralelogramo de lados adyacen-tes OP I y OP2 , el vértice P (punto -7 - 3i) representa la diferencia (-4 + 2i) - (3 + Si). .

FORMA POLAR DE LOS NUMEROS COMPLEJOS y

Hallar la forma polar de los números complejos siguientes:

2. 2 + 2i 2+ 2i

2 Amplitud o argumento, e = tg- I 2: = tg - 11 = 45°.

Módulo o valor absoluto, r = .J22+22 = J8 = 2fi·

Luego 2 + 2i = ricos O + i sen O) --~--~--~----~----x

= 2fi (cos 45 0 + i sen 45")

http://carlos2524.jimdo.com/

Page 183: Algebra Superior Murray R Spiegel

x

FORMA POLAR DE LOS NUMEROS COMPLEJOS 175

3. 1 + J3i 1+ /3" iO _,160°Amplitud o argumento, = cos 2"= .

Módulo o valor absoluto, r== J3+I = 2.

Luego 1 + fii = ,(cos O + i sen O)= 2(cos 60° + i sen 60°) --~--L---~---------X

4. -3 + 3i

0= 180° - 45° = 135°, ,= J9+9 = 3)2; luego -3 + 3i = 3)2 (cos 135° + i sen 135°)

5. -1 - J3i0= 180° + 60° = 240°, r = J3+I = 2; luego -1 - J3i = 2(cos 240° + i sen 240°) ..

6. J3 - i0= 360° - 30° = 330°, r = J3+I = 2; luego J3 - i = 2(cos 330° + i sen 330°)

y y

Problema 4 Problema 5 Problema 6

7. Hallar la forma polar de los números complejos siguientes: a) 5, b) 2i, e) -4, d) -4i.

a) 5. 0=0°, r = 5; luego 5 = 5(cos 0° + i sen 0°).

b) 2i. O = 90°, r = 2; luego 2i = 2(cos 90° + i sen 90°).

e) -4. 0= 180°, r = 4; luego -4 = 4(cos 180° + i sen 1800).

d) -4i. O = 2700, r = 4; luego -4i = 4(cos 2700 + i sen 270°).

y y y y

2iX°v'0'"

2 -4 490° O X4

X -4iO

(b) (e) (d)

5-O:;:-l--~5-~-X

(a)

I..http://carlos2524.jimdo.com/

Page 184: Algebra Superior Murray R Spiegel

176 FORMA POLAR DE LOS NUMEROS COMPLEJOS

-2

8_ - "Escribir los números complejos siguientes en la forma rectangular (a + bi). 2b)

a) 2(cos 30° + i sen 30°) = 2(1J3 + ti) = J3 + i

b) 6(cos 60c + i sen 60°) = 6(1-+ tJ3i) = 3 + 3J3;

e) 10(cos45° + ; sen 45°) = 10(í12 + íJ2i) = 512 + 5J2;

d) 3(cos 90° + i sen 90°) = 3(0 + i) = 3i

e) 2(cos 150°+ i sen 150°)= 2(-tJ3 + ti) = -J3 + i

f) 8(cos 240° + i sen 240°) = 8(-t - íJ3i) = -4 - 4J3;

g) 6(cos 315° + i sen 315°) = 6(íJ2 - tJ2i) = 312 - 312;h) 4(cos 720° + i sen 720°) = 4(cos 0° + i sen 0°) = 4(1 + O)= 4

A

G

PRODUcrOS y COCIENTES EN FORMA POLAR POTENCli

9. Efectuar las operaciones indicadas expresando los resultados en forma rectangular.ll. Hallar

tangula) [4(cos 20° + i sen 200)][3(cos 25° + i cos 25°)] = 12(cos45° + i sen 45°)

= 12(í12 + íJ2;) = 612 + 6J2;

b) [2(cos 18° + i sen 18°)][5(cos 42° + i sen 42°)] = 10(cos 60° + ; sen 60°)

= 10(1+ tJ3i) = 5 + 5J3;

a) [4

b) [3e) [2

e) [3(cos 80° + i sen 800)][6(cos 130°+ ; sen 130°)] = 18(cos 210° + i sen 210°)

= 18(-íJ3 - ti) = -9J3 - 9;

12(cos 54° + i sen 54°) lo lod) = 4(cos 30° + i sen 30°) = 4(1", 3 + ti) = 2", 3 + 2;

3(cos 24° + ; sen 24°)d) (el

e) (1

4J2(cos 45° + ; sen 45°) ¡;;e) 2 3 50 - 3 = 2",2(cos -270° + ; sen _270°)

(cos I + 1 sen 15°)= 2J2(cos 90° + i sen 90°) = 2J2(0 + ;) = 2J2;

f) (1

10. Determinar analítica y gráficamente los productos y cocientes indicados.g) (t.

a) (J3 + ;)(-J3 + 3i)

Analíticamente: (J3 + i)(-J3 + 3;) = -3 + 3J3i - J3i - 3 = -6 + 2J3; RAICES [

12. HallarGráficamente: P, = J3 + i = 2(cos 30° + i sen 30°),

P2 = -J3 + 3; = 2J3(cos 120°+ ; sen 120°), y

P = P, . P2 = 4J3lcos 150°+ i sen 150°) a) .J

P-6 + 211'3 i

y

______________________~~~---------------------x

176 FORMA POLAR DE LOS NUMEROS COMPLEJOS

8 . . "Escribir los números complejos siguientes en la forma rectangular (a + bi).

a) 2(cos 30° + i sen 30°) = 2(1J3 + íi) = J3 + i

b) 6(cos 60° + i sen 60°) = 6(í + tJ3i) = 3 + 3J3i

e) 10(cos 45° + i sen 45°) = 10(1.)2 + í.)2i) = 5.)2 + 5.)2i

d) 3(cos 90° + i sen 90°) = 3(0 -+- i) = 3i

e) 2(cos 150° + i sen 150°) = 2( -íJ3 + Íil = -J3 + i

f) 8(cos 240° + i sen 240°) = 8( -í - ífti) = - 4 - 4fti

g) 6(cos 315° + i sen 315°) = 6(1.)2 - íJ2i) = 3.)2 - 3.)2i

h) 4(cos 720° + i sen 720°) = 4(cos 0° + i sen 0°) = 4(1 + O) = 4

PRODUCTOS Y COCIENTES EN FORMA POLAR

9. Efectuar las operaciones indicadas expresando los resultados en forma rectangular.

a) [4(cos 20° + i sen 200)][3(cos 25° + i cos 25°)] = 12(cos 45° + i sen 45°)

= 12(1.)2 + íJ2i) = 6.)2 + 6J2i

b) [2(cos 18° + i sen 18°)][5(cos 42° + i sen 42°)] = 10(cos 60° + i sen 60°)

= 10(1 + ífti) = 5 + 5fti

e) [3(cos 80° + i sen 800)][6(cos 130° + i sen 130°)] = 18(cos 210° + i sen 210°)

= 18(-íft - íi) = -9J3 - 9i

12(cos 54° + i sen 54°) ¡;; ¡;; d) = 4(cos 30° + i sen 30°) = 4(í", 3 + íi) = 2", 3 + 2i

3(cos 24° + i sen 24°)

4J2(cOS 45° + i sen 45°) ¡;; e) 2( 3150 . 3 ° = 2",2(cos -270° + i sen -270°) cos + 1 sen 15)

= 2J2(cos 90° + i sen 90°) = 2J2(0 + i) = 2J2i

10. Determinar analítica y gráficamente los productos y cocientes indicados.

a) (J3 + i)(-J3 + 3i)

Analíticamente : (J3 + i)(-J3 + 3i) = -3 + 3fti - fti - 3 = -6 + 2fti

Gráficamente : P, = J3 + i = 2(cos 30° + i sen 30°),

P2 = -J3 + 3i = 2J3(cos 120° + i sen 120°), y

P = P, . P 2 = 4J3lcos 150° + i sen 150°)

P -6 + 2/3 i y

----------------------~~~---------------------x

http://carlos2524.jimdo.com/

Page 185: Algebra Superior Murray R Spiegel

FORMA POLAR DE LOS NUMEROS COMPLEJOS 177

2 - 2j3ib) 1+ i

y

2 - 2j3i1+ i

I - i 2 - 2i - 2j3i - 2.}3

1- i 2

= (I - j3) - (I + j3)i

Analíticamente:

Gráficamente: p¡ = 2 - 2j3i = 4(cos 300° + i sen 300°),

P2 = fi(cos 45° + i sen 45°), y

r, r:P = - = 2...;2 (cos 255° + i sen 255°)P2

POTENCIAS DE LOS NUMEROS COMPLEJOS

ll. Hallar las potencias indicadas de los números complejos siguientes, expresando los resultados en forma rec-tangular:

a) [4(CbS 15° + i sen 15°)]2 = 42(cos 30° + i sen 30°) = 16H.}3 + ti) = 8.}3 + 8i

b) [3(cos 45° + i sen 45°)]4 = 34(cos 180° + i sen 180°) = 81(-1 + Oi) = -81

e) [2(cos 80° + i sen 80°)]6 = 26(cos 480° + i sen 480°)

= 64(cos 120° + i sen 120°)

= 64(-t + tj3i) = -32 + 32j3id) (cos 30° + i sen 30°)3 = 13(cos 90° + i sen 90°) = 1(0 + i) = i

e) (I + i)3 = [fi(cos 45° + i sen 45°)]3 = 2fi(cos 135° + i sen 135°)

= 2fi( -tfi + tfii) = -2 + 2i

f) (I - i)4 = [fi(cos 315° + i sen 315°)]4 = 4(cos 1260° + i sen 1260°)

= 4(cos 180° + i sen 180°) = 4(-1 + Oi) = -4

g) (t.}3 + ti)IOO = [I(cos 30° + i sen 30°)]'00 = cos 3000° + i sen 3000°

= cos 120° + i sen 120° = -t + tj3i

RAICES DE LOS NUMEROS COMPLEJOS

12. Hallar todas las raíces indicadas (~I' Z2' etc.) y representarlas gráficamente:

60° + k 360' 60° + k 360a) Jl6(cos 60° + i sen 60°) = 4(cos 2 + i sen 2

y_1 (k = O) = 4(cos 30° + i sen 30°)

= 4(!-.}3 + ti) = 2.}3 + 2iz,

~2 (k = 1) = 4(cos 210° + i sen 210°)

= 4( -!-.}3 - ti)

= -2.}3 - 2i

________-+__~~~,-~-----x

2 - 2fii b)

1 + i

Analíticamente:

Gráficamente:

FORMA POLAR DE LOS NUMEROS COMPLEJOS

2 - 2fi; 1 +;

1 - i

1 - i

2 - 2; - 2fii - 2fi 2

= (I - fi) - (I + fi)i

PI = 2 - 2fii = 4(cos 300° + i sen 300°),

P2 = fi(cos 45° + i sen 45°), y

PI M P = - = 2...; 2 (cos 255° + i sen 255°) P2

POTENCIAS DE LOS NUMEROS COMPLEJOS

177

y

11. Hallar las potencias indicadas de los números complejos siguientes, expresando los resultados en forma rec­tangular:

a) [4(CbS 15° + i sen 15°)]2 = 42(cos 30° + i sen 30°) = 16(í}3 + tn = 8}3 + 8i

b) [3(cos 45° + ; sen 45°)]4 = 34(cos 180° + i sen 180°) = 81(-1 + Oi) = -81

e) [2(cos 80° + i sen 80°)]6 = 26(cos 480° + i sen 480°)

= 64(cos 120° + ; sen 120°)

= 64(-t + tfin = -32 + 32fi; d) (cos 30° + i sen 30°)3 = 13(cos 90° + i sen 90°) = 1(0 + i) = i

e) (I + ;)3 = [j2(cos 45° + ; sen 45°)]3 = 2j2(cos 135° + ; sen 135°)

= 2j2( -tj2 + tfin = -2 + 2i

f) (I - ;)4 = [fi(cos 315° + i sen 315°)]4 = 4(cos 1260° + i sen 1260°)

= 4(cos 180° + i sen 180°) = 4( -1 + Oí) = -4

g) (t}3 + tn lOO = [I(cos 30° + i sen 30°)]'00 = cos 3000° + i sen 3000°

= cos 120° + ; sen 120° = -t + tfii

RAICES DE LOS NUMEROS COMPLEJOS

12. Hallar todas las raíces indicadas (~I ' ' 2' etc.) y representarlas gráficamente:

60° + k 360 60° + k 360 a) Jl6(cos 60° + i sen 60°) = 4(cos 2 + i sen 2

- 1 (k = O) = 4(cos 30° + i sen 30°)

= 4(!-}3 + ti) = 2}3 + 2i

y z,

~2 (k = 1) = 4(cos 210° + i sen 210°) ________ -+ __ ~~~,_~-----x

= 4( -!-}3 - ti)

= -2}3 - 2i

http://carlos2524.jimdo.com/

Page 186: Algebra Superior Murray R Spiegel

178 FORMA POLAR DE LOS NUMEROS COMPLEJOS

50° + k 360n 50" + k 360"h) ~h2(cos 50n + i sen 50°) = 2(cos 5 + i sen 5 )

;:, (k = O) = 2(cos 10° + i sen 10°)

;:2 (k = 1) = 2(cos 82° + i sen 82°)

;:) (k = 2) = 2(cos 154° + i sen 154°)

_. (k = 3) = 2(cos 226° + i sen 226°)

Z5 (k = 4) = 2(cos 298° + i sen 298°)

Obsérvese que las raíces z" Z2' Z), Z., z" es-tán situadas sobre una circunferencia cuyo radio (2)es el módulo de cada una de ellas, y que el ángulo for-

360°mado por dos raíces consecutivas es -5- = Tl",

y

.>"/

/I

IZ3 I

III~' ++__~~~~L-~~X

0° + k 360° 0° + k 360°e) ys = 8'/) = [8(cos 0° + í sen 0°)]'/) = 2(cos + í sen )3 3

"' (k = O) = 2(cos 0° + í sen 0°) = 2(1 + Oí)= 2

;:2 (k = 1) = 2(cos 120° + í sen 120°) = 2(-,-í + íflí) = -1 + flí

;:) (k = 2) = 2(cos 240° + i sen 240°) = 2( -í - ífli) = -1 - flí

y

_____ +-_.Jo:,:~...L.......:2:..._ .•Z;...1- X

(e)

y

(d)

=, (k =' O) = cos 60° + í sen 60° = í + íflí

;:2 (k = 1) = cos 180° + i sen 180° = -1

=) (k = 2) = cos 300° + í sen 300° = í - íflí

13. Hallar las raíces siguientes:a) Raíces cuartas de 1.b) Raíces cúbicas de 1 - íe) Raíces cúbicas de - 8íd) Ralces cuartas de 2 + 2ftí

178 FORMA POLAR DE LOS NUMEROS COMPLEJOS

50° + k 3600 50" + k 360' h) 1 32(cos 500 + ¡ sen 50°) = 2(cos 5 + ¡ sen 5 )

: , (k = O) = 2(cos 10° + ¡ sen 10°)

=2 (k = 1) = 2(cos 82° + ¡ sen 82°)

:3 (k = 2) = 2(cos 154° + ¡ sen 154°)

=4 (k = 3) = 2(cos 226° + ¡ sen 226°)

z, (k = 4) = 2(cos 298° + ¡ sen 298°)

Obsérvese que las raíces z,. Z2. Z). Z4' Zs. es­tán situadas sobre una circunferencia cuyo radio (2) es el módulo de cada una de ellas. y que el ángulo for-

360° mado por dos raíces consecutivas es -5- = 72°,

y

/ "' ..... - -... ..... .... , / ,

/ , / ,

23 I \ I \

: \ Z1

~' ----~--~~~~L,~~X

\ \

0° + k 360° 0° + k 360° e) ,ys = 8"3 = [8(cos 0° + ¡ sen 0°))"3 = 2(cos + ¡ sen )

3 3

z , (k = O) = 2(cos 0° + ¡ sen 0°) = 2(1 + O¡) = 2

=2 (k = 1) = 2(cos 120° + ¡ sen 120°) = 2(-,-í + íJ3'¡) = -1 + J3'¡

: 3 (k = 2) = 2(cos 240° + ¡ sen 240°) = 2( -í - íJ3'i) = -1 - J3'¡

y y

______ +-_~~~~2~ __ .Z1~X

(e) (d)

180° + k 360° 180° + k 360° d) P = (_1)' /3 = [I(cos 180° + ¡ sen 180°)]' /3 = cos 3 + ¡ sen 3

=, (k == O) = cos 60° + ¡ sen 60° = í + íJ3'¡

:2 (k = 1) = cos 180° + ¡ sen 180° = -1

=3 (k = 2) = cos 300° + ¡ sen 300° = í - íJ3'¡

13. Hallar las raíces siguientes : a) Raíces cuartas de 1. b) Raíces cúbicas de 1 - ¡ e) Raíees cúbicas de -8¡ d) Ralees cuartas de 2 + 2J3'¡

http://carlos2524.jimdo.com/

Page 187: Algebra Superior Murray R Spiegel

FORMA POLAR DE LOS NUMEROS COMPLEJOS 179

O" + k 360" O" + k 360°a) 11/4 = [I(cos 0° + ; sen 0°)]'/4 = COS + ; sen --,---4 4

=, (k = O) = cos 0° + ; sen 0° = 1 + O; = 1

=2 (k = 1) = cos 90° + ; sen 90° = O + ; = ;

:3 (k = 2) = cos 180" +; sen 1800 = -1

Z4 (k = 3) = cos 270" + ; sen 270° = =i,\\ b)\

\\ 2,ie'' I

XI

II

II

/

e)

23 2,X XX O

23

2. 22

(a) (b)

y 22 Y2,

2

X X

22 2.

2.

(e) (d)

315° + k 360° 315° + k 360°(1 - ;)'/3 = [J2(cos 315° + ; sen 315°)]'/3 = ,i2(cos 3 + ; sen 3 )

z, (k = O) = ,i2(cos 105° + ; sen 105°)

Z2 (k = 1) = ,i2(cos 225° + ; sen 225°)

=3 (k = 2) = ,i2(cos 345° + ; sen 345°)

270° + k 360° 270° + k 360°(- 8i)'!3 = [8(cos 270° + ; sen 270°)) 1/3 = 2(cos 3 + ; sen 3 )

:, (k = O) = 2(cos 90° + ; sen 90°) = 2(0 + ;) = 2;

"2 (k = 1) = 2(cos 210° + ; sen 210°) = 2(-!)3 - !;) = -)3 - ;

Z3 (k = 2) = 2(cos 330° + ; sen 330°) = 2(!)3 - !i) = )3 - ;./

60° + k 360° 60° + k 360°d) (2 + 2J3i)1/4 = [4(cos 60° + ; sen 60°)]'/4 = J2(cos 4 + ; sen 4 )

z, (k = O) = J2(cos 15° + ; sen 15°) Z3 (k = 2) = J2(cos 195° + ; sen 195°)

=2 (k = 1) = J2(cos 105° + ; sen 105°) Z4 (k = 3) = J2(cos 285° + ; sen 285°)

y 2, Y

http://carlos2524.jimdo.com/

Page 188: Algebra Superior Murray R Spiegel

1,

180 FORMA POLAR DE LOS NUMEROS COMPLEJOS

PROBLEMAS PROPUESTOS14. Efectuar analítica y gráficamente las operaciones indicadas:

a) (3 + 4i) + (4 + 3i)b) (2-i)+(-4+5i)

e) (4 - 3i) - (-2 + i)d) ( - 2 + 2i) - (- 2 - i)

15. Escribir los siguientes números complejos en forma polar:

a) 3 - 3i

b) -J3 + ie) 4 - 4J3id) -5

e) 6J3 + 6if) -4 - 4i

g) 2h) iJ3

i)-2i

j) -1 + i

16. Escribir los números complejos siguientes en la forma rectangular (a + biv.

a) 4(cos 45° + i sen 45°) f) 10J2(cos 225° + i ser 225°)

b) 12(cos 30° + i sen 30°) g) 2(cos 300° + i sen 300°)

e) 6(cos 120° + i sen 120°) h) 5(cos 360° + i sen 360°)

d) 8(cos 180° + i sen 180°) i) 8(cos 90° + i sen 90°)

e) 3(cos 270° + i sen 270°) j) 16(cos 210° + i sen 210°)

17. Efectuar las operaciones indicadas. expresando los resultados en forma rectangular:

a) [3(cos 15° + i sen 15°)J[2(cos 75° + i sen 75°)J

b) [4(cos 40° + i sen 400)J[5(cos 20° + i sen 200)J

e) [2(cos 100° + i sen 1000)J[4(cos 50° + i sen 500)J

d) [6(cos 25° + i sen 25°)J[3(cos 290° + i sen 2900)J

20(cos 83° + i sen 83°) 6J3(cos 40° + i sen 40°)e) 5(cos 230 + i sen 23°) f) 3(cos 190° + i sen 190°)

g) [2(cos 12° + i sen 12°)J[3(cos 84° + i sen 84°)J[5(cos 24' + i sen 24°)J

12(cos 16° + i sen 16°)h)

[3(cos 44° + i sen 44°)J[2(cos 62° + i sen 62°)J

. 18. Hallar analítica y gráficamente el producto y cociente indicados:

a) (-1 + J3i)(2J3 -2i)4 - 4i

b)J3 - i

19. Hallar las potencias indicadas de los números complejos siguientes, expresando los resultados en forma rectangular.

b) [5(cos 30° + i sen 30°)]3 d) [4(cos 200 + i sen 20°)]3(tJ3 - ti)'0

b) [5(cos 300 + i sen 300)J3 e) (_1 + i)6 g)

h) (tJ2 + tJ2i)30

e) [J2(COS 36° + i sen 36°)J5 j) (1 + J3i)7

20. Hallar todas las raíces indicadas y representarlas gráficamente:

ft+i 1J3-ia) J4(cos 120° + i sen 120°) e) i)

b) .j'81(cos 1800 + i sen 1800) f) N2

YI ~2 - 2J3ie) ,y8(cos 60° + i sen 60°) g) j)

d) ,y'32(cos 200° + i sen 200°) h) .yí6i

SOLVe

14. a)

15. a)

b)

e)

d)

e)

, 16. a)

b)

17. a)

b)

18. a)

19. a)

b)

20. a)

b)

e)

d)

e)

f)

g)

h)

i)

j)

http://carlos2524.jimdo.com/

Page 189: Algebra Superior Murray R Spiegel

ngular.

FORMA POLAR DE LOS NUMERaS COMPLEJOS 181

SOLUCIONES DE LOS PROBLEMAS PROPUESTOS

. 14. a) 7 + 7i b) -2 + 4i e) 6 - 4i d) 3i

15. a) 3)2(cos 3W + i sen 315°) f) 4)2(cos 225° + i sen 225°)

b) 2(cas 150° + i sen 150°) g) 2(cos 0° + i sen 0°)

e) S(cos 300° + i sen 300°) h) J3(cos 90° + i sen 90°)

d) 5(cos ISO° + i sen ISOO) i) 2(cos 270° + i sen 270°)

e) 12(cos 30° + i sen 30°) j) )2(cos 135° + i sen 135°)

, 16. a) 2)2 + 2)2i e) -3 + 3fii e) -3i g) 1- fii i) Si

b) 6J3 + 6i d) -S f) -10 - 10i h) 5 j) -sJ3 - Si

17. a) 6i e) -4J3 + 4i e) 2 + 2fii g) -15 + 15J3i

b) 10 + lOJ3i d) 9)2 - 9)2i f) -3 - fii h) -2i

18. a) Si b) J3 + 1 + (1 - J3)i

19. a) -2J3 + 2i e) -4)2 e) Si g) t + tfii

b) 125i d) 32 + 32fii f) 64 + 64fii h) -i

20. a) 2(cos 60° + i sen 60°), 2(cos 240° + i sen 240°)

b) 3(cos 45° + i sen 45°),

3(cos 3150 + i sen 315°)

3(cos 135° + i sen 135°), 3(cos 225° + i sen 225°),

e) 2(cos 20° + i sen 20°), 2(cos 140° + i sen 140°), 2(cos 260° + i sen 260°)

d) 2(cos 40° + i sen 40°), 2(cos 112° + i sen 112°), 2(cos IS4° + i sen IS4°),

2(cos 256° + i sen 256°), 2(cos 32So + i sen 32S0)

e) fi(cos 15° + i sen 15°), fi(cos 135° + i sen 135°), fi(cos 255° + i sen 255°)

f) 2(cos 36° + i sen 36°), 2(cos 10So + i sen 10SO), 2(cos ISO° + i sen ISOO),

2(cos 252° + i sen 252°), 2(cos 3240 + i sen 324°)

g) cos 0° + i sen 0°, cos 120° + i sen 120°, cos 240° + i sen 240",

h) 2(cos 22,5° + i sen 22,5°), 2(cos 112,5° + i sen 112,5°), 2(cos 202,5° + i sen 202,5°),

2(cos 292,5° + i sen 292Y)

i) fi(cos 110° + i sen 110°), fi(cos 2300 + i sen 230°), fi(cos 350° + i sen 350°)

j) .,y4( cos 60° + i sen 60°), .,y4(COs 132" + i sen 132"), .,y4( cos 204° + i sen 204°),

.,y4(cos 276° + i sen 276°), .,y4(COS 34So + i sen 34S0)

http://carlos2524.jimdo.com/

Page 190: Algebra Superior Murray R Spiegel

CAPITULO 21

Teoría de ecuaciones

UNA ECUACION ENTERA RACIONAL de grado n en la variable x es de la forma

siendo n un número entero y positivo y ao, al' a2, ... , a._¡, a. constantes. La ecuación anteriortambién se puede escribir así:

x" + p¡x"-¡ + P2x"-2 + ... + P._¡x + P. = O

al dividir por ao '" O, con lo que la mayor potencia de x siempre es 1.

Las ecuaciones 4x3 - 2X2 + 3x - 5 = O, x2 - J2x + .x: = O Yx4 + px - 8 = O sonracionales enteras en x de grados 3, 2 y 4, respectivamente. Obsérvese que en cada una de estasecuaciones los exponentes de x son números enteros y positivos y los coeficientes de la variableson constantes (números reales o complejos).

En este capítulo trataremos únicamente de las ecuaciones racionales enteras.

UN POLINOMIO de grado n en la variable x es una función de x de la forma

ao '" Osiendo n un número entero y positivo, y ao, al' a2, ... , a._¡, a., constantes. Entonces,f(x) = Oes una ecuación racional entera de grado n en x.

Sif(x) = 3x3 + x2 + 5x - 6, se tienef(-2) = 3(-2)3 + (_2)2 + 5(-2) - 6 = -36.

Sif(x) = x2 + 2x - 8, se tienef(fi) = 5 + 2j5 - 8 = 2)5 - 3.

Todo valor de x que anule af(x) recibe el nombre de raíz de la ecuaciónf(x) = O. Por ejem-plo, una raíz de la ecuación f(x) = 3x3 - 2X2 - 5x - 6 = O es 2, ya que f(2) = 24 - 8 -lO - 6 = O.

rTEOREMA DEL RESTO. Si r es una constante y se divide el polinomio f(x) por (x - r), el resto

que se obtiene esf(r).

Por ejemplo, en la división de f(x) = 2X3 - 3x2 - X + 8 por x + 1, se tiene r = -1 yresto = f( - 1) = - 2 - 3 + 1 + 8 = 4. Por tanto,

2X3 - 3x

2- X + 8 = P(x) + _4_ siendo P(x) un polinomio en x.

x+l x+l'

182

TEORE(x -raíz

ya I

de;

REGLAsienla d

TEOREial n:

Con

TEORBal rr

(x -las ~

IDENTImisrx so

si A

RAICES

mite

mal

CAPITULO 21

Teoría de ecuaciones

UNA ECUACION ENTERA RACIONAL de grado n en la variable x es de la forma

siendo n un número entero y positivo y ao, al ' a2 , ••. , an - I , an constantes. La ecuación anterior también se puede escribir así:

x" + Plx"-I + P2x"-2 + . .. + Pn-IX + Pn = O

al dividir por ao =F O, con lo que la mayor potencia de x siempre es 1.

Las ecuaciones 4x3 - 2X2 + 3x - 5 = O, X2 - J2x + ~ = O Y x4 + px - 8 = O son

racionales enteras en x de grados 3, 2 y 4, respectivamente. Obsérvese que en cada una de estas ecuaciones los exponentes de x son números enteros y positivos y los coeficientes de la variable son constantes (números reales o complejos).

En este capítulo trataremos únicamente de las ecuaciones racionales enteras.

UN POLINOMIO de grado n en la variable x es una función de x de la forma

ao =F O

siendo n un número entero y positivo, y ao, al' a2 , .. • , an - I , an, constantes. Entonces,f(x) = O es una ecuación racional entera de grado n en x .

Sif(x) = 3x3 + X2 + 5x - 6, se tienef(-2) = 3(-2)3 + (_2)2 + 5(-2) - 6 = -36.

Sif(x) = X2 + 2x - 8, se tienef(fi) = 5 + 2j5 - 8 = 2)5 - 3.

Todo valor de x que anule af(x ) recibe el nombre de raíz de la ecuaciónf(x) = O. Por ejem­plo, una raíz de la ecuación f(x) = 3x3 - 2X2 - 5x - 6 = O es 2, ya que f(2) = 24 - 8 -lO - 6 = O.

, TEOREMA DEL RESTO. Si r es una constante y se divide el polinomio f(x) por (x - r), el resto

que se obtiene esf(r).

Por ejemplo, en la división de f(x) = 2X3 - 3X2 - X + 8 por x + 1, se tiene r = -1 y resto = f( - 1) = - 2 - 3 + 1 + 8 = 4. Por tanto,

2X3 - 3X2 - X + 8 = P(x) + _4_ siendo P(x) un polinomio en x .

x+l x+l'

182

http://carlos2524.jimdo.com/

Page 191: Algebra Superior Murray R Spiegel

erior

sonestasiable

=0

jem-8-

resto

1 Y

TEORIA DE ECUACIONES 183

TEOREMA DEL DIVISOR. Si r es una raíz de la ecuación f(x) = O, es decir, f(r) = O, el binomio(x - r) es un divisor def(x). Recíprocamente, si (x - r) es un divisor def(x), el número r es unaraíz de la ecuaciónf(x) = O, esto es,f(r) = O.

Por ejemplo, 1, - 2, - 3 son las tres raíces de la ecuación f(x) = x3 + 4x2 + X - 6 = O,ya que f(l) =f(-2) =f(-3) = O. Por consiguiente, (x - 1), (x + 2) y (x + 3) son divisoresde x3 + 4x2 + X - 6.

REGLA DE RUFINI. Proporciona un método sencillo para dividir un polinomio f(x) por (x ± r),siendo r un número dado. Por este método se determinan los coeficientes del cociente y el resto dela división.

TEOREMA FUNDAMENTAL DEL ALGEBRA. Toda ecuación racional entera f(x) = O admiteal menos una raíz real o compleja.

Por ejemplo, x' - 3x5 + 2 tiene por lo menos una raíz.

Sin embargo,f(x) = Jx + 3 no tiene raíces, ya que no existe un número r tal quej{r) = O.Como esta ecuación no es racional, no cumple el teorema fundamental del álgebra.

TEOREMA FUNDAMENTAL DEL ALGEBRA. Toda ecuación racional entera f(x) = O admiteal menos una raíz real o compleja.

Por ejemplo, 2X3 + 5x2 - 14x - 8 = O tiene 3 raíces que son 2, - ~ y -4.

Algunas de las raíces pueden ser iguales. Por ejemplo, la ecuación de sexto grado(x - 2)3 (x - 5)2 (x + 4) = O tiene la raíz triple 2, la raíz doble 5 y la raíz simple -4; es decir,las seis raíces son 2, 2, 2, 5, 5, -4.

IDENTIDAD DE POLINOMIOS. Si dos polinomios de grado n en la misma variable x toman losmismos valores numéricos para más de n valores de x, los coeficientes de las mismas potencias dex son iguales, y ambos polinomios son idénticos.

Por ejemplo, la ecuación 5x2 - 2x + 3 = AX2 + (B + Cjx + 7(B - e) es una identidadsi A = 5, B + e = - 2 Y 7(B - e) = 3.

RAICES COMPLEJAS E IRRACIONALES

1) Si un número complejo, a + bi, es una raíz de una ecuación racional entera, f(x) = O,de coeficientes reales, el complejo conjugado, a - bi, es también raíz de dicha ecuación.,

Se deduce, pues, que toda ecuación racional entera de grado impar con coeficientes reales ad-mite por lo menos una raíz real.

2) Si la ecuación racional entera j{x) = O de coeficientes racionales tiene una raíz de la for-ma a + fi, siendo a y b racionales y Jh irracional, (/ - Jh es otra raíz de la ecuación.

TEORIA DE ECUACIONES 183

TEOREMA DEL DIVISOR. Si r es una raíz de la ecuación f(x) = O, es decir, f(r) = O, el binomio (x - r) es un divisor def(x). Recíprocamente, si (x - r) es un divisor def(x), el número r es una raíz de la ecuaciónf(x) = O, esto es,!(r) = O.

Por ejemplo, 1, - 2, - 3 son las tres raíces de la ecuación f(x) = x 3 + 4X2 + X - 6 = O, ya que f(l) =f(-2) =f(-3) = O. Por consiguiente, (x - 1), (x + 2) y (x + 3) son divisores de x 3 + 4X2 + X - 6.

REGLA DE RUFINI. Proporciona un método sencillo para dividir un polinomio f(x) por (x ± r), siendo r un número dado. Por este método se determinan los coeficientes del cociente y el resto de la división.

TEOREMA FUNDAMENTAL DEL ALGEBRA. Toda ecuación racional entera f(x) = O admite al menos una raíz real o compleja.

Por ejemplo, x 7 - 3x5 + 2 tiene por lo menos una raíz.

Sin embargo, f(x) = Jx + 3 no tiene raíces, ya que no existe un número r tal que j{r) = O. Como esta ecuación no es racional, no cumple el teorema fundamental del álgebra.

TEOREMA FUNDAMENTAL DEL ALGEBRA. Toda ecuación racional entera f(x) = O admite al menos una raíz real o compleja.

Por ejemplo, 2X 3 + 5X2 - 14x - 8 = O tiene 3 raíces que son 2, - ~ y -4.

Algunas de las raíces pueden ser iguales. Por ejemplo, la ecuación de sexto grado (x - 2)3 (x - 5)2 (x + 4) = O tiene la raíz triple 2, la raíz doble 5 y la raíz simple -4 ; es decir, las seis raíces son 2, 2, 2, 5, 5, -4.

IDENTIDAD DE POLINOMIOS. Si dos polinomios de grado n en la misma variable x toman los mismos valores numéricos para más de n valores de x, los coeficientes de las mismas potencias de x son iguales, y ambos polinomios son idénticos.

Por ejemplo, la ecuación 5X2 - 2x + 3 = AX2 + (B + e)x + 7(B - e) es una identidad si A = 5, B + e = - 2 Y 7(B - e) = 3.

RAICES COMPLEJAS E IRRACIONALES

1) Si un número complejo, a + bi, es una raíz de una ecuación racional entera , f(x) = O, de coeficientes reales, el complejo conjugado, a - bi, es también raíz de dicha ecuación .,

Se deduce, pues, que toda ecuación racional entera de grado impar con coeficientes reales ad­mite por lo menos una raíz real.

2) Si la ecuación racional entera j{x) = O de coeficientes racionales tiene una raíz de la for­

ma a + jb, siendo a y b racionales y Jh irracional , (f - Jb es otra raíz de la ecuación.

http://carlos2524.jimdo.com/

Page 192: Algebra Superior Murray R Spiegel

184 TEORIA DE ECUACIONES

RAICES RACIONALES

1) Sea bic una fracción racional irreducible que sea raíz dela ecuación de coeficientes enteros

Entonces, b es un divisor de Un Y e lo es de ao.

Por ejemplo, si blc es una raíz racional de 6x~ + 5x2 - 3x - 2 = O, los valores posiblesde b son los divisores de 2, que son ± 1, ± 2, y los de e son los divisores de 6, que son ± 1, ± 2, ± 3,± 6. Por tanto, las posibles raíces racionales son ± 1, ± 2, ± 1/2, ± 1/6, ± 2/3.

2) En estas condiciones. si una ecuación f(x) = O de coeficientes enteros se escribe con elcoeficiente de la mayor potencia de .v igual a 1,

.v" + r.s" 1 + P2Xn-2 + ... + Pn-IX + P« = O

las raíces racionales de f(x) = O son divisores de Pn'

Por ejemplo, si la ecuación x3 + 2X2 - Ilx - 12 = O tiene raíces racionales, éstas debenser divisores de 12; por consiguiente, las únicas posibles son ± 1, ±2, ±3, ±4, ±6, ± 12.

METODO GRAFICO PARA HALLAR LAS RAICES REALES. Si f(x) = O es una ecuación ra-cional entera de coeficientes reales, se pueden obtener unos valores aproximados de sus raíces realestrazando la curva y = f(x) y hallando los valores de x que corresponden a los puntos de interseccióncon el eje x (y = O). Un hecho fundamental en este método es que sif(a) y f(b) son de signo con-trario,f(x) = O tiene, por lo menos, una raíz comprendida entre x = a y x = b. Se justifica por lacontinuidad de la función y = f(x) cuando f(x) es un polinomio de coeficientes reales.

ACOTACION DE RAICES. Un número a se llama cota superior de las raíces reales de la ecuaciónf(x) = O, si ninguna de las raíces es mayor que a. Se dice que un número b es cota inferior de lasraíces reales de f(x) = O si ninguna de sus raíces es menor que b. Para determinar las cotas supe-rior e inferior de las raíces, se aplica el teorema siguiente:

Sea el polinomio f(x) = aoxn + alxn-I + ... + pn = O, en el que ao, al' ... , an son reales

yao > O. Luego:

1) Si al dividir f(x) por (x - a), siendo a ~ O, mediante la regla de Rufini, todos los coeficientesdel polinomio cociente (números obtenidos en la tercera fila) son positivos o cero, entonces a esuna cota superior de las raíces reales de f(x) = O.

2) Si al dividir fix¡ por (x - b), siendo b 2 O, mediante la regla de Rufini, todos los coeficientesdel polinomio cociente son alternativamente positivos y negativos (o cero), entonces b es una cotainferior de las raíces reales de j{x) = O.

REGLA DE LOS SIGNOS DE DESCARTES

Ordenando los términos de un polinomio f(x) de coeficientes reales según las potencias de-crecientes de x, diremos que se produce una variación de signo cuando dos términos consecutivosson de signo contrario. Por ejemplo, x3 - 2X2 + 3x - 12 tiene 3 variaciones de signo, y2x 7 - 6x5 - 4x4 + x2 - 2x + 4 tiene 4 variaciones de signo.

ci<

de

trepaprlara

TRAN

vepe

es

noúlti

son

dedeesde

METOirr,

RELA(gu

exi

1)

2)3)

4)

tiei

184 TEORIA DE ECUACION ES

RAICES RACIONALES

1) Sea h/c una fracción racional irreducible que sea raíz dela ecuación de coeficientes enteros

Entonces. b es un divisor de (In Y e lo es de ao.

Por ejemplo. si b/ c es una raíz racional de 6x~ + 5X2 - 3x - 2 = O. los valores posibles de b son los divisores de 2. que son ± 1, ± 2. y los de e son los divisores de 6. que son ± l. ± 2, ± 3, ± 6. Por tanto. las posibles raíces racionales son ± 1, ± 2, ± 1/2, ± 1/6. ± 2/3.

2) En estas condiciones. si una ecuación I(x) = O de coeficientes enter ·)s se escribe con el coeficiente de la mayor potencia de x igual a l.

+ Pn - lX + Pn = O

las raíces racionales de I(x) = O son divisores de Pn '

Por ejemplo, si la ecuación x 3 + 2X2 - Ilx - 12 = O tiene raíces racionales, éstas deben ser divisores de 12; por consiguiente. las únicas posibles son ±I, ±2, ±3, ±4, ±6, ±12.

METODO GRAFICO PARA HALLAR LAS RAICES REALES. Si f(x) = O es una ecuación ra­cional entera de coeficientes reales, se pueden obtener unos valores aproximados de sus raíces reales trazando la curva y = f(x) y hallando los valores de x que corresponden a los puntos de intersección con el eje x (y = O). Un hecho fundamental en este método es que sif(a) y f(b) son de signo con­trario,f(x) = O tiene, por lo menos. una raíz comprendida entre x = a y x = b. Se justifica por la continu idad de la función y = f(x) cuando f(x) es un polinomio de coeficientes reales.

ACOTACION DE RAICES. Un número a se llama cota superior de las raíces reales de la ecuación f(x) = O, si ninguna de las raíces es mayor que a. Se dice que un número b es cota inferior de las raíces reales de f(x) = O si ninguna de sus raíces es menor que b. Para determinar las cotas supe­rior e inferior de las raíces. se aplica el teorema siguiente:

Sea el polinomio f(x) = aoxn + alXn- 1 + ... + (In = O, en el que ao• al' .. . , an son reales yao > O. Luego: '

1) Si al dividir f(x) por (x - a), siendo a ~ O, mediante la regla de Rufini , todos los coeficientes del polinomio cociente (números obtenidos en la tercera fila) son positivos o cero , entonces a es una cota superior de las raíces reales de f(x) = O.

2) Si al dividir f(x) por (x - b), siendo b ;2; O, mediante la regla de Rufini, todos los coeficientes del polinomio cociente son alternativamente positivos y negativos (o cero), entonces b es una cota inferior de las raíces reales de j{x) = O.

REGLA DE LOS SIGNOS DE DESCARTES

Ordenando los términos de un polinomio f(x) de coeficientes reales según las potencias de­crecientes de x, diremos que se produce una variación de signo cuando dos términos consecutivos son de signo contrario. Por ejemplo. x 3

- 2X2 + 3x - 12 tiene 3 variaciones de signo, y 2x 7

- 6x 5 - 4x4 + X2 - - 2x + 4 tiene 4 variaciones de signo.

http://carlos2524.jimdo.com/

Page 193: Algebra Superior Murray R Spiegel

ros

bles±3,

el

ben

ra-alesiónon-r la

iónlaspe-

ales

de-vos, y

TEORIA DE ECUACIONES IS5

La regla de Descartes de los signos establece que: El número de raíces positivas de la ecua-ción f(x) = ° es igual al número de variaciones de signo del polinomio f(x), o bien a este númeromenos un entero par. El número de raíces negativas de f(x) = ° es igual al número de variacionesde signos de f( - .x), o bien a este número menos un entero par.

Por ejemplo, en la ecuación fix¡ = x9 - 2X5 + 2X2 - 3x + 12 = 0, como el polinomiof'(x)tiene 4 variaciones de signo, el' número de raíces de ftx¡ = ° son 4, (4 - 2) o (4 - 4). Por otraparte,f(-x) = (_X)9 - 2(_X)5 + 2(_X)2 - 3(-x) + 12 = _x9 + 2x5 + 2x2 + 3x + 12 = °presenta una variación de signo, con lo que f(x) = ° posee una sola raíz negativa. En resumen,la ecuación dada podrá tener 4, 2 ó ° raíces positivas, 1 raíz negativa y, al menos, 9 - (4 + 1) = 4raíces complejas. (Hay 4,6 u 8 raíces complejas. ¿Por qué)

TRANSFORMACION DE ECUACIONES

Multiplicando cada rai: pOI' una constante. Para obtener una ecuación cuyas raíces sean kveces las correspondientes a otra dada, se multiplica el segundo término de ésta por k, el terceropor k2, el cuarto por k.", etc., teniendo en cuenta los términos de coeficiente nulo si los hay.

Por ejemplo, la ecuación cuyas raíces sean dobles de las raíces de x3 - 3x2 - 10x + 24 = °es x3 - 3x2(2) - 10x(22) + 24(23) = 0, es decir, x3 - 6x2 - 40x + 192 = O.

Cambiando el signo de cada raiz, Para obtener una ecuación cuyas raíces sean iguales y de sig-no contrario a las de otra dada, se cambia el signo de los términos de grado impar de estaúltima.

Por ejemplo, las raíces de 2X3 + 3x2 - 3x - 2 = ° son 1, - ~ y - 2; entonces, - 1, ~ y 2son raíces de la ecuación -2x3 + 3x2 + 3x - 2 = 0, es decir, 2X3 - 3x2 - 3x + 2 = O.

Disminuyendo cada rai: en una constante. Para obtener una ecuación cuyas raíces sean lasde otra dada, fix¡ = 0, menos un número h, se divide ftx¡ por (x - Iz), y el resto es el coeficientedel último término de la ecuación buscada. Dividiendo el cociente obtenido por (x - Iz), el restoes el penúltimo término de la ecuación, etc. Esta operación se realiza fácilmente aplicando la reglade Rufini. (Véase Problema 48.)

METODO DE HORNER. Permite obtener, con una aproximación prefijada de antemano, las raícesirracionales de una ecuación racional entera. (Véanse Problemas 57-59.)

RELACIONES ENTRE LAS RAICES y LOS COEFICIENTES. En una ecuación, escrita de formaque el coeficiente de la mayor potencia de x sea 1,

existen las siguientes relaciones entre los coeficientes y las raíces:

1) - P 1 = suma de las raíces;

2) P2 = suma de los productos de las raíces tomadas dos a dos;

3) - P3 = suma de los productos de las raíces tomadas tres a tres; etc.;

4) (- 1t», = producto de todas las raíces.

Por ejemplo, si llamamos XI' X 2 Y x 3 a las raíces de la ecuación x3 - 6x2 - 'lx - 8 = 0, setiene Xl + X2 + X3 = -(-6) = 6, X1X2 + X,;¡X3 + X3X1 = -7, X1X2X3 = -(-8) = 8.

TEORIA DE ECUACIONES ¡SS

La regla de Descartes de los signos establece que: El número de raíces positivas de la ecua­ción f(x) = O es igual al número de variaciones de signo del polinomio f(x), o bien a este número menos un entero par. El número de raíces negativas de f(x) = O es igual al número de variaciones de signos de f( - x), o bien a este número menos un entero par.

Por ejemplo, en la ecuaciónf(x) = x 9 - 2X5 + 2X2 - 3x + 12 = O, como el polinomio/L,)

tiene 4 variaciones de signo, el' número de raíces de f(x) = O son 4, (4 - 2) o (4 - 4). Por otra parte, f( -x) = (_X)9 - 2( _X)5 + 2( _X)2 - 3( -x) + 12 = _x9 + 2X5 + 2X2 + 3x + 12 = O presenta una variación de signo, con lo que f(x) = O posee una sola raíz negativa . En resumen , la ecuación dada podrá tener 4, 2 ó O raíces positivas, 1 raíz negativa y, al menos, 9 - (4 + 1) = 4 raíces complejas . (Hay 4,6 u 8 raíces complejas. ¿Por qué)

TRANSFORMACION DE ECUACIONES

Multiplicando cada raí:: pOI' una constante. Para obtener una ecuación cuyas raíces sean k veces las correspondientes a otra dada, se multiplica el segundo término de ésta por k , el tercero por k 2

, el cuarto por k 3, etc. , teniendo en cuenta los términos de coeficiente nulo si los hay.

Por ejemplo, la ecuación cuyas raíces sean dobles de las raíces de x 3 - 3X2 - 10x + 24 = O

es x 3 - 3x2 (2) - IOx(2 2

) + 24(2 3) = O, es decir , x 3 - 6X2 - 40x + 192 = O.

Cambiando el signo de cada raí::. Para obtener una ecuación cuyas raíces sean iguales y de sig­no contrario a las de otra dada, se cambia el signo de los términos de grado impar de esta última.

Por ejemplo, las raíces de 2X3 + 3X2 - 3x - 2 = O son 1, - ~ y - 2 ; entonces, - 1, ~ y 2 son raíces de la ecuación -2x3 + 3X2 + 3x - 2 = O, es decir, 2X3 - 3X2 - 3x + 2 = O.

Disminuyendo cada raí:: en una constante. Para obtener una ecuación cuyas raíces sean las de otra dada, f(x) = O, menos un número 11, se divide f(x) por (x - 11), y el resto es el coeficiente del último término de la ecuación buscada. Dividiendo el cociente obtenido por (x - 11), el resto es el penúltimo término de la ecuación, etc . Esta operación se realiza fácilmente aplicando la regla de Rufini. (Véase Problema 48.)

METODO DE HORNER. Permite obtener, con una aproximación prefijada de antemano, las raíces irracionales de una ecuación racional entera . (Véanse Problemas 57-59.)

RELACIONES ENTRE LAS RAICES y LOS COEFICIENTES. En una ecuación, escrita de forma que el coeficiente de la mayor potencia de x sea 1,

existen las siguientes relaciones entre los coeficientes y las raíces:

1) - PI = suma de las raíces;

2) P2 = suma de los productos de las rakes tomadas dos a dos;

3) - P3 = suma de los productos de las raíces tomadas tres a tres; etc.;

4) (- 1 )"Pn = producto de todas las raíces.

Por ejemplo, si llamamos XI' X 2 Y x 3 a las raíces de la ecuación x 3 - 6X2 - 7x - 8 = O, se tiene XI + X2 + X 3 = -(-6) = 6, X I X2 + -"lX3 + -"3XI = -7, XIX2X3 = -(-8) = 8.

http://carlos2524.jimdo.com/

Page 194: Algebra Superior Murray R Spiegel

186

PROBLEMAS RESUELTOSTEORIA DE ECUACIONES

TEOREMA DEL RESTO Y DEL DIVISOR

l. Demostrar el teorema del resto: Si un polinomio f(x) se divide por (x - r), el resto es f(r).

En la división de [ix¡ por (x - r), sea Q(x) el cociente y R una constante, el resto

Por definición, f(x) = (x - r)Q(x) + R es una identidad para todos los valores de x. En particular. parax = r, f(r) = R.

2. Hallar el resto de las divisiones siguientes:

a) (2x' + 3x' - 18x - 4) -7 (x - 2).

b) (x' - 3x' + 5x + 8) -7 (x + 1).

, z . 1e) (4x +5x -1)~(x+"2)'

d) (x' - 2x' + x - 4) -7 X.

8, 4, 3e) (27x - 9x + x - "2) -7 (2x - 3).

f) (x8 - x' - x' + 1) -7 (x + F)·

R = f(2) = 2(2') + 3(2') - 18(2) - 4 = -12R = f( - 1) = (- 1)4 - 3( - 1)' + 5( - 1) + 8 = 1 + 3 - 5 + 8 = 7

1 1 , 1 z 1R =f(--) = 4(--) + 5(--) - 1 = --

2 2 2 4R =f(O) = -4

3 83, 43, 3 3R =f(-) = -(-) - -(-) + - - - = O2 272 92 2 2

R =f(-il = (_i)8 - (-i)' - (-i)' + 1 = i8 + ¡' + ¡' + 1=1+i-i+I=2

3. Demostrar el teorema del divisor: Si r es una raíz de la ecuación jtx) = O, se verifica que (x - r) es un divisor def(x); recíprocamente, si (x - r) es un divisor de f(xl, r es una raíz de f(x) = O.

En la división de f(x) por (x - r}, sea Q(x) el cociente y R, una constante, el resto. En estas condiciones,[ix¡ = (x - r)Q(x) + R, o sea f(x) = (x - r)Q(x) + f(r), según el teorema del resto.

Sea r una raíz de f(x) = O, esto es, f(r) = O. Entonces, f(x) = (x - r)Q(x), con lo cual (x - r) es un divi-sor de f(x).

Recíprocamente, si (x - r) es un divisor de f(x), el resto de la división de f(x) por (x - r) es cero. Por tanto,f(r) = O, es decir, r es una raíz de f(x) = o.

4. Demostrar que (x - 3) es un divisor del polinomio f(x) = x' - 4x' - Tx? + 22x + 24.

f(3) = 81 - 108 - 63 + 66 + 24 = O. Como (x - 3) es un divisor de f(x), 3 es un cero del polinomiof(x). o bien 3 es una raíz de la ecuación [ix¡ = O.

S. a) ¿Es -1 una raíz de la ecuación j{x) = x' - Tx - 6 = O?

b) ¿Es 2 una r~íz de la ecuación f(x) = y' - 2y' - Y + 7 = O'>

e) ¿Es 2i una raíz de la ecuación f(~) = 2~' + 3z' + 8: + 12 = O'>

a) f( - 1) = -1 + 7 - 6 = O. Por tanto, -1 es una raíz de la ecuación f\x) = O, Y [x - (-1)] = x + 1es un divisor del polinomio f(x).

h) f(2) = 16 _ 8 - 2 + 7 = 13. Por tanto. 2 no es una raíz de f(y) = O, e (y - 2) no es un divisor de

y' - 2.1" - Y + 7.

el fl2i) = 2(2i)' + 3(2i)' + 8(2i) + 12 = -16i - 12 + 16i + 12 = O. Por tanto. 2i es una raíz de [iz¡ = o.y (: - 2i) es un divisor del polinomio f\:)·

6. Dernc

7. a)h)

a) f(

h) f

8. Demrcuam

cuan,

(

sible

9. Hall,

a)

h)

REGLA 1

Hall,

10. (3x'

en 1,de s

el di2. q:4 semen

http://carlos2524.jimdo.com/

Page 195: Algebra Superior Murray R Spiegel

ar. para

+8=7

ivisorde

diciones,

un divi-

or tanto,

linomio

]=x+1

divisor de

1(z) = O.

TEORIA DE ECUACIONES 187

6. Demostrar que x - a es un divisor de .v" - u". si 1/ es un número entero y positivo.

ftx¡ = x" - o": luego [ia¡ = a" - a" = O. Como /Ia) = O. x - a es un divisor de x" - a"

7. a) Demostrar que x' + ,,5 es divisible por x + a.h) ¿Cuál es el resto de la división de y" + o" por y + a'

a) .f(x) = x5 + a5; luego .f(-a) = (_a)' + a5 = _a' + a' = O.Como /(-a) = O, x5 + a5 es divisible por x + a.

h) /(1') =)'6 + a6• Resto =/(-,,) = (_a)6 + ,,0 = ar. + o" = 2a"-

8. Demostrar que x + a es un divisor de .v" - a" siempre que 11 sea un número entero. positivo y par. pero no lo escuando 11 sea un entero positivo e impar. Se supone que a = O.

.fh) = x" - a".

Cuando n es par, /( -a) = (-al" - a" = a" - a" = O. Luego /( -a) = O. x + a es un factor de ,1(" - u"cuando n es par.

Cuando n es impar, .j"(-a) = (-a)" - o" = -a" - a" = +La". Como j(-a) = O, .v" - u" no es divi-sible por .v + u cuando n es impar (el resto es - 2a").

9. Hallar los valores de l' para los cuales a) 2x.1 - px' + 6x - 31' es divisible por x + 2.h) (x· - p'x + 3 - 1') -i- (x - 3) tiene de resto 4.

a) El resto es 2(-2)-' - 1'(_2)' + 6(-2) - 31' = -16 - 41' - 12 - 31' = -28 - 71' = O. Luego l' = -4.

h) El resto es 34- 1"(3) + 3 - P = 84 - 31" - l' = 4.

Luego 31" + P - 80 = O, (p - 5)(31' + 16) = O Y P = 5. - J6i3.

REGLA DE RUFINI

Hallar el cociente y el resto de las divisiones siguientes. aplicando .Ia regla de Rufini.

10. (3x5 - 4x' - 5x3 - 8x + 25) -i- (x - 2)

3 - 4 - 5 + O - 8 + 25~I 6 + 4 - 2 - 4 - 24

3 + 2 - I - 2 - 12 + 1

Cociente: 3x· + 2X3 - x' - 2x - 12

Resto:

Los números que aparecen en la primera fila son los coeficientes del dividendo. habiéndose puesto un ceroen la potencia de x que falta (Ox'). El número 2 de la segunda fila es el segundo término del divisor cambiandode signo (ya que el coeficiente de x en el divisor es 1).

El primer coeficiente de la primera fila se escribe en el primer lugar de la tercera y se le multiplica porel divisor 2. El producto 6 se coloca en el primer lugar de la segunda fila y se suma con - 4 obteniéndose2, que se pone en el segundo lugar de la tercera fila. Este 2 se multiplica por el 2 del divisor y el producto4 se coloca en la segunda fila y se suma con - 5, dando lugar a - I en la tercera fila, etc. El último nú-mero de la tercera fila es el resto y todos los números a su izquierda son los coeficientes del cociente.

Como el dividendo es de 5.° grado y el divisor de 1.0. el cociente es de 4.° grado.

La solución se escribe: 3x' + 2X3 - x' - 2x - 12 + x - 2 .

TEORIA DE ECUACIONES 187

6. Demostrar que x - a es un divisor de x" - a" . si 1/ es un número entero y positivo .

.I(x) = x" - a" ; luego I(a) = a" - a" = O. Como .I(a ) = O. x - a es un divisor de x" - a"

7. a) Demostrar que x 5 + aS es divisible por x + a .

h) ¿Cuál es el resto de la división de y O + aO po r y + a')

a) .f(x) = x 5 + a 5; luego .n-a) = ( _a )5 + a5 = - a' + a' = O. Como /( - a) = O. x 5 + aS es divisible por x + a.

b) /ll') =)'6 + a6• Resto =/(-a) = ( _ a)" + ah = af> + ah = 2a6

8. Demostrar que x + a es un di visor de x" - a" siempre que 1/ sea un número entero. posi tivo y par. pero no lo es cuand o 11 sea un entero positivo e impar. Se supone que a = O.

/(x) = x" - a".

Cuando 11 es par. f( -a) = ( - al" - an = an

- a" = O. Luego .n -a) = O. x + a es un factor de ;e" _ a" cuando 11 es par.

C uando n es impar . .f(-a) = ( -a)" - a" = - a" - a" = -2a". Como .I( - a) = O. x" - (J" no es divi­sible por x + (J cuando n es impar (el resto es - 2a").

9. Hallar los valores de l' para los cuales al 2x.1 - px2 + 6x - 31' es divi sible por.\' + 2. b) (x· - p 2X + 3 - ,,) -7- (x - 3) tiene de resto 4.

a) El resto es 2(-2)' - 1'(_2)2 + 6(-2) - 3" = - 16 - 4" - 12 - 3" = -28 - 71' = O. Luego l' = -4.

b) El resto es 3· - 1'2(3) + 3 - p = 84 - 31" - P = 4. Luego 31'2 + l' - 80 = O. (p - 5)(3" + 16) = O Y l' = 5. -1 6/3.

REGLA DE RUFINI

Hallar el cocien te y el resto de las divi siones siguientes. aplicando . Ia regla ' de Rufini .

10. (3x 5 - 4x' - 5x 3 - 8x + 25) -7- (x - 2)

3 - 4 - 5 + O - 8 + 25 ~I 6 + 4 - 2 - 4 - 24

Cociente: 3x· + 2X 3 - X2 - 2x - 12

3 + 2 - 1 - 2 - 12 + 1 Resto:

Los números que aparecen en la primera fila son los coeficientes del d ividendo. habiéndose puesto un cero en la potencia de x que falta (OX2) . El número 2 de la segunda fila es el segundo término del di visor cambiando de signo (ya que el coeficiente de x en el divisor es 1).

El primer coeficiente de la primera fi la se escr ibe en el primer lugar de la tercera y se le multiplica por el divisor 2. El prod ucto 6 se coloca en el primer lugar de la segunda fila y se suma con - 4 obteniéndose 2. que se pone en el segundo lugar de la tercera fila. Este 2 se multiplica por el 2 del divisor y el producto 4 se coloca en la segunda fi la y se suma con - 5. dando lugar a - I en la tercera fil a. etc. El último nú­mero de la tercera fila es el resto y todos los números a su izquierda son los coeficientes del cociente.

Como el dividendo es de 5.0 grado y el divisor de 1.0. el cociente es de 4.° grado.

La solución se escribe : 3x' + 2X 3 - x' - 2x - 12 + x - 2

http://carlos2524.jimdo.com/

Page 196: Algebra Superior Murray R Spiegel

188 TEORfA DE ECUACfONES

11. (x4 - 2X3 - 24x2 + 15x + 50) -7 (x + 4)

I - 2 - 24 + 15 + 50~I - 4 + 24 - O - 60

1-6+ 0+ 15-10

10Solución: x3 - 6x2 + f 5 - --

x+4

12. (24 - I7x2 - 4) -7 (x + 3)

2 + O - 17 + O - 4~ - 6 + 18 - 3 + 9

2-6+ 1-3+5

Solución:5

2X3 - 6x2 + X - 3 + --x + 3

13. (4x3 - IOx2 + X - 1) -7 (x - 1/2)

4 - 10 + f - 1~ + 2 - 4 - 3/2

4 - 8 - 3 - 5/2

Solución: 4x2 _ 8x _ 3 __ 5_2x - 1

14. Dado f(x) = x3 - 6x2 - 2x + 40. calcular a) f( - 5) Y b) f(4) aplicando la regla de Rufini.

a) 1 - 6 - 2 + 40~ - 5 + 55 - 265

1 - 11 + 53 - 225

f(-5) = ~225

b) 1 - 6 - 2 + 40~ +4- 8-40

1 - 2 - 10 + O

f(4) = O

SOLUCION DE ECUACIONES CONOCIENDO ALGUNAS DE SUS RAICES

15. Sabiendo que una raíz de x3 + 2,,2 - 23x - 60 = O es 5. resolver la ecuación.

1 + 2 - 23 - 60~I + 5 + 35 + 60

1 + 7 + 12 + O

. Dividiendo x3 + 2X2 - 23x - 60 por x - 5.

.Se obtiene la ecuación x3 + 7x + 12 = O cuyas raíces son - 3.-v

Las tres raíces son 5. - 3. - 4.

16. Dos raíces de x· - 2X2 - 3x - 2 = O son - 1 Y 2. Resolver la ecuación.

1+0-2-3-2-=-U -1+1+1+2

1-1-1-2+0

Dividiendo x4 - 2X2 - 3x - 2 por x + l.

Se obtiene la ecuación x3 - x2 - X - 2 = O.

1-1 - I - 2~ +2+2+2

1+ I + 1+ O

Dividiendo x3 - x2 - X - 2 por x - 2.

Se obtiene la ecuación x2 + x + I = O cuyas raíces sonI I-+-il31-2 v=

Las cuatro raíces son -1, 2, -~ ± ~iJ3../

ESCRII

17. Ha

a)

b)

e)

d)

e)

J)

18. Ese

a)

a)

blde

el

dl

19. Ese

al

alb)

el

d)

POLlNC

lO, Ha

21. Haes

osi,

http://carlos2524.jimdo.com/

Page 197: Algebra Superior Murray R Spiegel

TEORJA DE ECUACJONES 189

ESCRIBIR UNA ECUACION CONOCIENDO SUS RAICES

17. Hallar las raíces de las ecuaciones siguientes:

a) (x - 1)2(x + 2)(x + 4) = O. Sol.

b) (2x + 1)(3x - 2)3(2x - 5) = O.

e) X3(X2 - 2x - 15) = O.

d) (x + I + )3)(x + I - )3)(x - 6) = O.

e) [(x - O(x + o]l(x + I¡z = o.J) 3(x + m)4(5x - n)2 = O.

J como raíz doble, -2, -4

- 1/2, 2/3 como raíz triple, 5/2

O como raíz triple, 5, - 3

( - I - )3), (- I + )3), 6

± i como raíces triples, - I como raíz doble

-m como raíz cuádruples, n/5 como raíz doble

18. Escribir las ecuaciones cuyas raíces son las que se indican:

a) 5, 1, -3; b) 2, -1/4, -1/2;

a) (x - 5)(x - I)(x + 3) = O o

I Ib) (x - 2)(x + ¡)(x + 2) = O o

de coeficientes enteros.

e) ±2, 2 ±)3; d) O, I ± 5i.

x3 - 3x2 - 13x + 15 = O.

x3 _ 5x2

_ ~ _ ~ = O484

que es8x3 - IOx2 - Ilx - 2 = Oo

e) (x - 2)(x + 2)[x - (2 - )3)][x - (2 + )3)] = (x2 - 4)[(x - 2) + )3][(x - 2) - )3]= (x2 - 4)[(x - 2f - 3] = (x2 - 4)(x2 - 4x + 1) = O, o x4 - 4x3 - 3x2 + 16x - 4 = O

d) x[x - (1 + 5i)][x - (1 - 50] = x[(x - 1) - 5i][(x - 1) + 5i] = x[(x - 1)2 + 25]= x(x2 - 2x + 26) = O, o x3 - 2X2 + 26x = O

19. Escribir la ecuación de coeficientes enteros cuyas raíces son las que se indican:

I 1a) 1, 2' - 3; d) 2 como raíz triple, - 1.

3 2b) 0'3'3,-1;

a) (x - 1)(2x - 1)(3x + 1) = O o 6x3 - 7x2 + I = O

b) x(4x - 3)(3x - 2)(x + 1) = O o 12x4 - 5x3 - lIX2 + 6x = O

I I Ie) (x - 3i)(x + 3i)(x - -)2)(x + -)2) = (x2 + 9)(x2 - -) = O, (x2 + 9)(2x2 - 1) = O,

2. 2 2

o 2X4 + 17x2 - 9 = O

d) (x - 2)3(X + 1) = O o x4 - 5x3 + 6x2 + 4x - 8 = O

POLINOMIOS IDENTICOS

20. Hallar los valores de A y B para los cuales la ecuación A(2x + 3) + B(x - 4) = 3x + 10 es una identidad.

Ordenando la ecuación (2A + B)x + 3A - 4B = 3x + 10.Esto es una identídad si, y solo si, 2A + B = 3, 3A - 4B = 10. Resolviendo, A = 2 y B = - 1.

21. Hallar los valores de A, B, C para los cuales A(x - 3)(x - 1) + B(x + I)(x - 1) + C(x + I)(x - 3) = 6x - 10es una identidad.

Ordenando la ecuación A(x2 - 4x + 3) + B(x2 - 1) + C(x2 - 2x - 3) = 6x - 10

o (A + B + C)x2 + (-4A - 2C)x + 3A - B - 3C = 6x - 10. Es una identidad

si, y solo si, A + B + C = O, -4A - 2C = 6, 3A - B - 3C = -10. Resolviendo, A = -2, B = 1, C = 1.\

TEORIA DE ECUACIONES

ESCRIBIR UNA ECUACION CONOCIENDO SUS RAICES

17. Hallar las raíces de las ecuaciones siguientes:

a) (x -I)2(x+2)(x+4)=0.

b) (2x + 1)(3x - 2)3(2x - 5) = O.

e) X 3(X 2 - 2x - 15) = O.

Sol. I corno raíz doble, -2, -4

- 1/2, 2/3 corno raíz triple, 5/2

O corno raíz triple, 5, - 3

d) (x + 1 + J3)(x + 1 - J3)(x - 6) = O. ( - I - J3), (- 1 + J3), 6

e) [(x - il(x + il]l(x + 1)2 = O.

J) 3(x + m)4(5x - n)2 = O.

± i corno raíces triples, - 1 corno raíz doble

-m corno raíz cuádruples, n/5 corno raíz doble

18. Escribir las ecuaciones cuyas raíces son las que se indican:

a) 5, 1, -3; b) 2, - 1/4, - 1/2 ; e) ±2, 2 ± J3; d) O, I ± 5i.

a) (x - 5)(x - I)(x + 3) = O o x 3 - 3X2 - 13x + 15 = O.

I I

189

b) (x - 2)(x + ¡)(x + 2") = O o x3 _ 5X2 _ ~ _ ~ = O 484

o 8x3 - 10x2 - lIx - 2 = O que es

de coeficientes enteros.

e) (x - 2)(x + 2)[x - (2 - J3)][x - (2 + J3)] = (x2 - 4)[(x - 2) + J3][(x - 2) - J3] = (x2 _ 4)[(x - 2)2 - 3] = (x2 - 4)(x2 - 4x + 1) = O, o x 4 - 4x3

- 3X2 + 16x - 4 = O

d) x[x - (1 + 5i)][x - (J - 5il] = x[(x - 1) - 5i][(x - 1) + 5i] = xCIx - 1)2 + 25] = x(x2 - 2x + 26) = O, o x 3 - 2X2 + 26x = O

19. Escribir la ecuación de coeficientes enteros cuyas raíces son las que se indican:

1 I 3 2 a) 1, 2 ' - 3; b) O, 3' 3 ' -1; d) 2 corno raíz triple, - 1.

a) (x - I )(2x - I )(3x + 1) = O o 6x3 - 7 X2 + I = O

b) x(4x - 3){3x - 2)(x + 1) = O o 12x 4 - 5x3 - IIx2 + 6x = O

I I I e) (x - 3i)(x + 3i)(x - 2)2){X + 2)2) = (x2 + 9)(x2 - 2) = O, (x2 + 9)(2x2 - 1) = O,

o 2X 4 + 17x2 - 9 = O

d) (x - 2)3(X + 1) = O o x 4 - 5x3 + 6X2 + 4x - 8 = O

POLINOMIOS IDENTICOS

20. Hallar los valores de A y B para los cuales la ecuación A(2x + 3) + B(x - 4) = 3x + 10 es una identidad.

Ordenando la ecuación (2A + B)x + 3A - 4B = 3x + 10. Esto es una identidad si, y solo si, 2A + B = 3, 3A - 4B = 10. Resolviendo, A = 2 y B = - 1.

21. Hallar los valores de A, B, C para los cuales A(x - 3)(x - 1) + B(x + I)(x - 1) + C(x + I)(x - 3) = 6x - 10 es una identidad.

Ordenando la ecuación A(x2 - 4x + 3) + B(x2 - 1) + C(x2 - 2x - 3) = 6x - 10

o (A + B + C)x2 + (-4A - 2C)x + 3A - B - 3C = 6x - 10. Es una identidad

si, y solo si, A + B + C = O, -4A - 2C = 6, 3A - B - 3C = -10. Resolviendo,· A = -2, B = 1, C = 1. \

http://carlos2524.jimdo.com/

Page 198: Algebra Superior Murray R Spiegel

190 TEORIA DE ECUAqPNES

~

do

30. E

ti

a

b

31. ti:

y

b;e

a;bl

RAlCE

32. H

al

b)

28. Hallar las cuatro raíces de x· + 2X2 + I = O. e)

RAICES COMPLEJAS E IRRACIONALES

22. Sabiendo que los números indicados son raíces de una ecuación de coeficientes reales, hallar otra raíz de dichaecuación. a) Zi, b) - 3 + 2i. e) - 3 - ifi.

a) -2i, h) -3 - 2i, e) -3 + ifi

23. Sabiendo que los números indicados son raíces de una ecuación de coeficientes racionales, hallar otra raíz de di-

cha ecuación. a) -j7, b) -4 + 2)3, e) 5 - ~fi.2

1a) j7, b) -4 - 2)3. e) 5 + 2fi

24. Estudiar los razonamientos siguientes:

a) x3 + 'lx - 6i = O tiene una raíz .v = i ; por tanto. .v = - i es otra raíz.b) x3 + (1 - 2)3)x2 + (5 - 2)3)x + 5 = O tiene una raíz igual a )3 - ifi; por tanto . .j3 + ifi es

otra raíz.

e) x4 + (1 - 2fi)x3 + (4 - 2fi)x2 + (3 - 4fi)x + 1 = O tiene una raíz x = -1 + fi; por tanto.x ;= - 1 - fi es otra raíz.

a) x = - i no es necesariamente una raíz, porque no son reales todos los coeficientes de la ecuación dada.

b) La conclusión es válida porque los coeficientes de la ecuación dada son reales.

e) x = - 1 - fi no es necesariamente una raíz. porque no son racionales todos los coeficientes de la ecua-ción dada. Se puede comprobar. por sustitución. que x = - 1 - fi no es una raíz.

25. Escribir la ecuación de menor grado. de coeficientes reales. que tenga dos raíces iguales a 2 y 1 - 3i.

(x - 2)[x - (1 - 3i)][x - (1 + 3i)] = (v - 2)(x2 - 2x + lO) = O o bien x3 - 4x2 + 14x - 20 = O

26. Escribir la ecuación de menor grado, de co~ficientes racionales. que tenga dos raíces iguales a - 1 + J5 y - 6.

[x - (-1 + J5)][x - (-1 - J5)](x + 6) = (x2 + 2x - 4)(x + 6) = O o bien x3 + 8x2 + 8x - 24 = O

27. Escribir la ecuación de cuarto grado, con coeficientes racionales. que tenga dos raíces iguales a a) - 5i Y

}6, b) 2 + i Y 1 - )3.

a) (x + 5i)(x - 5i)(x - }6)(x + }6) = (x2 + 25)(x2- 6) = O o bien x· + 19x2 - 150 = O

h) [x - (2 + i)][x - (2 - i)][x - (1 - .j3)][x - (1 + )3)] = (x2 - 4x + 5)(x2 - 2x - 2) = O o bien

x· - 6x3 + 11 x2 - 2x - 10 = O

x· + 2X2 + 1 = (x2 + 1)2 = [(x + i)(x - iJ]2 = O. Las raíces son i. i. -i. -i

29. Resolver la ecuación x4- 3x3 + 5x2

- 27x - 36 = O sabiendo que una de sus raíces es un número imagina-no puro de la forma bi, siendo 6 un número real.

Sustituyendo bi por x. 64 + 3b3i - 5h2 - 27hi - 36 = O.

Igualando a cero las partes reales e imaginarias:

64- 5h2 - 36 = O. (h2 - 9 )(h2 + 4) = O Y b = ± 3 ya que b es real;

3h3-276=O, 3h(62-9)=0 y 6=0. ±3.

±

d)

ra

190 TEORIA DE ECUACI'pNES

~

RAICES COMPLEJAS E IRRACIONALES

22. Sabiendo que los números indicados son raíces de una ecúación de coeficientes reales, hallar otra raíz de dicha

ecuación. o) 2i, b) - 3 + 2i. e) - 3 - ifi.

a) -2i, h) -3 - 2i, e) -3 + ifi

23. Sabiendo que los números indicados son raíces de una ecuación de coeficientes racionales, hallar otra raíz de di­

cha ecuación. a) - ,Ji. b) -4 + 2fi, e) 5 - ~fi. 2

I o),Ji, b) -4 - 2fi. e) 5 + 2fi

24. Estudiar los razonamientos siguientes:

a) Xl + 7x - 6i = O tiene una raíz x = i; por tanto. x = - i es otra raíz.

b) Xl + (1 - 2fi )x2 + (5 - 2fi)x + 5 = O tiene una raíz igual a fi - ifi; por tanto . .)3 + ifi es otra raíz.

e) x 4 + (1 - 2fi)xl + (4 - 2fi)x2 + (3 - 4fi)x + I = O tiene una raíz x = - 1 + fi; por tanto.

x ~ - I - fi es otra raíz.

a) x = - i no es necesariamente una raíz, porque no son reales todos los coeficientes de la ecuación dada.

b) La conclusión es válida porque los coeficientes de la ecuación dada son reales.

e) x = - I - fi no es necesariamente una raíz. porque no son raciono les todos los coeficientes de la ecua­ción dada. Se puede comprobar. por sustitución. que x = - I - fi no es una raíz .

25. Escribir la ecuación de menor grado. de coeficientes reales. que tenga dos raíces iguales a 2 y I - 3i.

(x - 2)[x - (1 - 3i)][x - (1 + 3i)] = (x - 2)(x2 - 2x + 10) = O o bien Xl - 4X2 + 14x - 20 = O

26. Escribir la ecuación de menor gr;:¡do. de coeficientes racionales. que tenga dos raíces iguales a - I + J5 y - 6.

[x - (-1 + J5 )][x - (-1 - J5)](x + 6) = (x2 + 2x - 4)(x + 6) = O o bien Xl + 8X2 + 8x - 24 = O

27. Escribir la ecuación de cuarto grado, con coeficientes racionales. que tenga dos raíces iguales a a) - 5i Y

)6, b) 2 + i Y I - fi·

a) (x + 5i)(x - 5i)(x - )6)(x + )6) = (x2 + 25)(x2 - 6) = O o bien ... + 19x2 - 150 = O

h) [x - (2 + i)][x - (2 - i)][x - (1 - .)3 )][x - (1 + .)3)] = (x2 - 4x + 5)(x2 - 2x - 2) = O o bien

x 4 - 6Xl + II X2 - 2x - 10 = O

28. Hallar las cuatro raíces de ... + 2X2 + I = O.

x 4 + 2X2 + I = (x 2 + 1)2 = [(x + i)(x - iJ]2 = O. Las raíces son i . i. -i. - i

29. Resolver la ecuación x' - 3Xl + 5X2 - 27x - 36 = O sabiendo que una de sus raíces es un número imagina­no puro de la forma hi. siendo h un número real.

Sustituyendo hi por x. h4 + 3bl i - 5h2 - 27hi - 36 = O.

Igualando a cero las partes reales e imaginarias:

h4 - 5h 2

- 36 = O. (h 2 - 9)W + 4) = O Y h = ± 3 ya que h es real;

3h l -27h=O. 3h(h2 -9)=O y h=O. ±3.

http://carlos2524.jimdo.com/

Page 199: Algebra Superior Murray R Spiegel

e dicha

íz de di-

ifi es

tanto.

da.

a ecua-

=0

5 Y -6.

-s. Y

o bien

TEORIA DE ECUACIONES 191

La solución común es b =. ± 3; por tanto. dos raíces son ± 3i. y (x - 3i)(x + 3i) = ;2 + 9 es un divisorde x4

- 3x3 + 5x2- 27x ::.. J"f El otro factor. obtenido por simple división. es x2 - 3x - 4 = (x - 4)(x + 1).

con lo que las otras dos raíces son 4 y - l.

Las cuatro raíces son ± 3i. 4, - l.

30, Escribir la ecuación de menor grado, de coeficientes racionales, que tenga una raíz igual a:a) J3 -)2, h))2 + R·a) Hacemos x = J3 - )2.

Elevando al cuadrado los dos miembros, x2 = 3 - 2../6 + 2 = 5 - 2,)6 Y x2 - 5 = - 2,)6.Elevando al cuadrado, de nuevo, x4 - IOx1 + 25 = 24 Y x4 - IOx2 + I = O.

h) Hacemos x = )2 + R.Elevando al cuadrado los dos miembros, x2 = 2 + 2,)=2 - I = 1 + 2F-2 Y x2 - I = 2,)=2.Elevando al cuadrado de nuevo, x4 - 2X2 + I = - 8 Y x4 - 2X2 + 9 = O.

31. a) Escribir la ecuación de menor grado, de coeficientes constantes (reales o complejos), que tenga las raíces 2y I - 3i. Compárese con el Problema 25.

b) . Escribir la ecuación de menor grado, de coeficientes reales. que tenga las raíces -6 y -1 + .)5.Compárese con el Problema 26.

a) (x - 2)[x - (1 - 3i)] = O o x2 - 3(1 - i)x + 2 - 6i = O

b) (x + 6)[x - (-1 + .)5)] = O o x2 + (7 - .)5)x - 6(.)5 - 1) = O

RAICES RACIONALES

32. Hallar las raíces reales, si existen de las ecuaciones siguientes:

a) x4 - 2X2 - 3x - 2 = O

Las raíces racionales posibles son los divisores enteros de 2, que son ± l. ± 2.

Ensayando estos valores, en el orden + l. - l. + 2, - 2, mediante la regla de Rufini, o por sustitución. sededuce que las únicas raíces racionales son - 1 Y 2.

b) x3 - X - 6 = O

Las raíces racionales posibles son los divisores enteros de 6, que son ± l. ± 2, ± 3. ± 6.

Ensayando estos valores, en el orden + 1, - l. + 2, - 2. + 3, - 3, + 6. - 6, se deduce que la única raíz ra-cional es 2.

e) 2x' + x2 - 7x - 6 = O

Si blc (en forma irreducible) es una raíz racional, los valores posibles de b son ± 1, ± 2, ± 3, ± 6, y los de e,± 1, ± 2. Por tanto, las raíces racionales posibles son ± 1, ± 2, ± 3, ± 6, ± 1;'2, ± 3/2.

Ensayando todos estos valores se deduce que las raíces racionales - 1, 2 y - 3/2.

d) 2X4 + x2 + 2x - 4 = O

Si btc es una raíz racional. los valores posibles de b son ± 1, ±2, ±4, y los de e,. ± l. ±2. Por tanto lasraíces racionales posibles son ± l. ±2, ±4, ± 1/2.

Ensayando todos estos valores se deduce. que no existen raíces racionales.

TEORIA DE ECUACIONES 191

La solución común es ~ = . ± 3; por tanto. dos raíces son ± 3i. y Ix 3i)(x + 3i) = ;2 + 9 es un divisor de x· - 3x3 + 5X2 - 27x ::.. 3 . El otro factor. obtenido por simple división. es X2 - 3x - 4 = (x - 4)(x + 1). con lo que las otras dos raíces son 4 y - l.

Las cuatro raíces son ± 3i. 4. - l.

JO. Escribir la ecuación de menor grado. de co(j/ril'l1les racionoles. que tenga una raíz igual a:

o) J3 -.fi. h) .fi + ,,0.

ti) Hacemos x = J3 - .fi. Elevando al cuadrado los dos miembros. X2 = 3 - 2,/ 6 + 2 = 5 - 2fi Y X2 - 5 = - 2fi.

Elevando al cuadrado. de nuevo. x' - 10x' + 25 = 24 Y x 4 - IOx 2 + 1 = O.

h) Hacemos x = .fi + p. Elevando a l cuadrado los dos miembros. X2 = 2 + 2P - 1 = l + 2P Y X2 - l = 2P. Elevando al cuadrado de nuevo. x· - 2X2 + 1 = - 8 y x· - 2X2 + 9 = O.

31. a) Escribir la ecuación de menor grado, de coeficientes cons/anles (reales o complejos). que tenga las raíces 2 y 1 - 3i. Compárese con el Problema 25 .

b) . Escribir la ecuación de menor grado. de coeficientes reales. que tenga las raíces -6 y -1 + fi. Compárese con el Problema 26.

a) (x - 2)[x - (1 - 3i)] = O o x 2 - 3( l - i)x + 2 - 6i = O

b) (x + 6)[x - (-1 + fi J] = O o X2 + (7 - Js)x - 6(fi - 1) = O

RAICES RACIONALES

32. Hallar las raíces reales. si existen de las ecuaciones siguientes:

a) x· - 2X2 - 3x - 2 = O

Las raíces racionales posibles son los divisores enteros de 2. que son ± 1, ± 2.

Ensayando estos valores, en el orden + 1, - 1, + 2. - 2, mediante la regla de Ruf'ini. o por sustitución, se deduce que las únicas raíces racionales son - 1 y 2.

h) x 3 - X - 6 = O

Las raíces racionales posibles son los divisores enteros de 6. que son ± 1, ± 2. ± 3. ± 6.

Ensayando estos valores. en el orden + 1, - 1, + 2, - 2, + 3. - 3. + 6, - 6. se deduce que la única raíz ra­cional es 2.

e) 2xJ + X2 - 7x - 6 = O

Si b/c (en forma irreducible) es una raíz racional. los valores posibles de h son ± 1, ± 2. ± 3. ± 6. y los de e. ± l. ± 2. Por tanto. las raíces racionales posibles son ± 1. ± 2. ± 3. ± 6. ± 1/2. ± 3/2.

Ensayando todos estos valores se deduce que las raíces racionales - 1, 2 y - 3/2.

d) 2x· + X2 + 2x - 4 = O

Si b/c es una raíz racional. los valores posibles de b son ± 1. ±2. ±4. y los de e, . ± 1, ±2. Por tanto las raíces racionales posibles son ± 1, ±2. ±4. ± 1/2.

Ensayando todos estos valores se deduce. que no existen raíces rac ionales.

http://carlos2524.jimdo.com/

Page 200: Algebra Superior Murray R Spiegel

192 TEORIA DE ECUACIONES

33. Resolver la ecuación x3 - 2X2 - 31x + 20 = O.

Las raíces racionales posibles son los divisores enteros de ,~(). que son. ± 1, ± 2. ± 4. ± 5. ± 10. ± 20.

Ensayando todos estos valores. mediante la regla de Ruflni. se ob-tiene la raíz racional - 5.

La ecuación que resulta, x' - 7x + 4 = O. tiene las raíces irracio-

1 - 2 - 31 + 20~ - 5 + 35 - 20

nales 7/2 ± J33/2.Por tanto. las tres raíces de la ecuación dada son - 5. 7/2 ± j33/2.

1-7+ 4+ O

34. Resolver la ecuación 2x' - 3x3 - 7X' - 8x + 6 = O.

Si b/c es una raíz racional. los valores posibles de b son ± l. ± 2. ± 3. ± 6. y los de c. ± 1, ± 2. Por tanto.las raíces racionales posibles son ±1. ±2. ±3, ±6. ±1/2. ±3/2.

Ensayando todos estos valores, mediante la regla de Rufini, se ob-tiene la raíz 3.La ecuación que resulta, 2X3 + 3X' + 2x - 2 = O. tiene laraíz 1/2.

La ecuación 2X2 + 4x + 4 = O. o bien x2 + 2x + 2 = O. tiene lasraíces complejas - 1 + i.

Las cuatro raíces son 3, 1/2, -1 ± i.

2-3-7-8+6~ +6+9+6-6

2+3+2-2+0

2+3+2-2!EJ +1+2+2

2+4+4+0

35. Demostrar que J3 + fi es un número irracional.

Sea x = J3 + fi; entonces x' = (J3 + fi)' = 3 + 2}6 + 2 = 5 + 2}6 Y x' - 5 = 2}6.

Elevando de nuevo al cuadrado. x' - 10x' + 25 = 24, o bien x' - IOx' + 1 = O. Las únicas raíces ra-cionales posibles de esta ecuación son ± 1. Ensayando ambas se deduce que no existen raíces racionales. Por tanto.

x = J3 + fi es un número irracional.

METODO GRAFlCO PARA HALLAR RAICES REALES

36. Representar la función f(x) = x3 + X - 3. De la gráfica deducir:a) El número de raíces positivas. negativas o complejas de x3 + x - 3 = O.b) Una raíz real de x3 + x - 3 = O aproximada con dos cifras decimales.

I x I -3 1 -2 -1 O 1 2 3 4

I f(x) I -331-13 -5 -3 -1 7 27 65

a) De la curva se deduce que hay una raíz real posi-tiva.

Por tanto. las otras dos raíces-son complejas conjugadas.

h) El valor aproximado de la raíz real es 1+. Obsérveseque f(x) cambia de signo entre x = 1 Y x = 2.

-+-+--I--t-~L+---j-I---X

Se puede apreciar que la raíz se aproxima más a 1,0quea 2.0. Para lograr mayor .aproximación, se dibuja la curvacon más detalle para los valores de x comprendidos entre 1 y 2.

raíz'de 1,puntsu v,se d,

o se

be e

o se,

COTAS:

37. Hall

a)

COII

supe

Col'

b)

COI,

http://carlos2524.jimdo.com/

Page 201: Algebra Superior Murray R Spiegel

±20.

tanto,

es ra-tanto,

TEORIA DE ECUACIONES 193

B

Como f(x) cambia de signo entre x = 1,2 Y x = 1,3, laraíz estará comprendida entre estos dos valores. La recta ABde la figura es. una aproximación de la curva real entre lospuntos A y B. La raíz está situada, aproximadamente, en R ysu valor es 1,2 + k,. De los triángulos semejantes ABC y ARDse deduce,

k,0,1

0,072 0,0720,072 + 0,497 = 0,569 = 0,1 +

o sea, k, = 0,01+. Luego la raíz es 1,21+.

Este procedimiento de localización de la raíz suponiendo que A y B están unidos por una línea recta reci-be el nombre de interpolación lineal.

x 1,21 1,22 If(x) -0,0184 +0,03581

Laraíz es aproximadamente 1,21 + k2. Por interpolación,

k2 0,0184 = 0,0184 = 0,3 +0,01 0,0184 + 0,0358 0,0542

o sea.x, = 0,003+. Luego la raíz es 1,21 +0,OO3,osea 1,213+.

Así, pues, la raíz con dos cifras decimales es 1,21.

COTAS SUPERIOR E INFERIOR DE RAICES REALES

37. Hallar las cotas superior e inferior de las raíces reales de a) x3 - 3x2 + 5x + 4 = O, b) x3 + x2 - 6 = O.

a) Las raíces racionales posibles son ± 1, ±2, ±4.

Cota superior.

1-3+5+4.!J +1-2+3

1-2+3+7

1-3+5+ 4~ +2-2+ 6

1-1+3+10

1-3+5+ 4~ + 3 + 0+15

1 + O + 5 + 19

Como todos los números de la tercera fila de la división de f(x) por x - 3 son positivos (o cero), una cotasuperior de las raíces es 3, es decir, no hay raíces mayores que 3.

Cola inferior.

1 - 3 + 5 + 4~I -1+4-9

1-4+9-5

Como los números de la tercera fila son alternativamente positivos y ne-gativos, -1 es una cota inferior de las raíces, es decir, no hay raíces menoresque -1.

b) Las raíces racionales posibles son ± 1, ±2, ±3, ±6.

COla superior.

1+1+0-6.!J +1+2+2

1+2+2-4

1+1+0- 6~ + 2 + 6 + 12

1+3+6+.6

Luego 2 es una cota superior de las raíces.

http://carlos2524.jimdo.com/

Page 202: Algebra Superior Murray R Spiegel

194 TEORIA DE ECUACIONES

Cola inferior. .~.

1+1+0-61-0+0--------

1+0+0-6

Como todos los números de la primera 'fila son alternativamente positivosy negativos (o cero). una cota inferior de las raíces es - l.

38. Hallar las raíces racionales de 4x' + 15.\ - 36 = O y. a continuación. resolver completamente la ecuación.

Las raíces racionales posibles son ± l. ±2. ±3. ±4. ±6. ±9. ± 12. ± 18. ±36. ± 1/2. ±3/2. ±9/2.± 1/4. ±3f4. ±9!4. Para evitamos el ensayo de todas estas posibilidades, hallamos las cotas superior e inferiorde las raíces.

Cola superior.

4 + O + 15 - 36-=-!J + 4 + 4 + 19

4+4+ 19-17

4 + O + 15 - 36~ + 8 + 16 + 62

4 + 8 + 31 + 26

Luego no hay raíces (reales) mayores que. o igua-les a. 2.

Cola inferior.

4 + O + 15 - 36-=-!J - 4 + 4 - 19

4 - 4 + 19 - 55

Luego no hay raíces reales menores que. o iguales a. - l.

Las únicas raíces racionales posibles mayores que -1 y menores que 2 son + 1. ± 1/2. ± 3/2. ± 1/4. ± 3/4.Ensayando estos valores se obtiene que 3/2 es la única raíz racional.

4 + O + 15 - 36+ 6 + 9 + 36-------

4 + 6 + 24 + O

Las otras raíces son soluciones de 4.\·2+ 6x + 24 = O, o bien 2.\2 + 3'\ + 12 = O.

3 j87.es decir, x = - '4 ± -4- /.

REGLA DE LOS SIGNOS DE DESCARTES

39. Estudiar el número de raíces positivas, negativas y complejas de las ecuaciones siguientes. aplicando la regla delos signos de Descartes.

al 2.\' + 3.\2 _ 13.\ + 6 = O d) 2X4 + 7x2 + 6 = O g) x. + x' - 1 =0

b) ,\4 _ 2.\2 - 3.\ - 2 = O e) x4- 3x2 - 4 = O !t) x6 - 3X2 - 4x + 1 =0

el x2-2.\+7=0 f) x' + 3x - 14 = O

al Hay 2 variaciones de signo enf(x) = 2x' + 3x2 - I3x + 6. Hay 1 variación de signo enf(-x) = -2.\'+ 3x2 + 13.\ + 6. Por tanto, existen. como máximo, 2 raíces positivas y 1 negativa.

Las raíces pueden ser: (1) 2 positivas, 1 negativa, O complejas; (2) O positivas, 1 negativa. 2 complejas (lasraíces complejas son conjugadas dos a dos).

b) Hay una variación de signo en f(x) = x4 - 2X2 - 3x - 2 y 3 variaciones de signo en f( -x) = .\4 - 2.\2+ 3x - 2. Por tanto, existen. como máximo, 1 raíz positiva y 3 raíces negativas.

Las raíces pueden ser: (1)(2)

positiva, 3 negativas. O complejaspositiva. 1 negativa. 2 complejas

e) Hay 2 variaciones de signo enf(x) = x2 - 2x + 7 y ninguna variación de signo enf(-x) = x2 + 2.\ + 7.

Por tanto. las raíces pueden ser: (1) 2 positivas, O negativas, O complejas(2) O positivas. O negativas. 2 complejas

d)

son

e)

f)

g)

!tI+ 4

40. Detb) /

a)

h)

41. Hall

al

a)

h)

e)raci:Las

d)V3m

http://carlos2524.jimdo.com/

Page 203: Algebra Superior Murray R Spiegel

positivos

ción.

2. ±9/2.e inferior

. o igua-

/4. ±3/4

+ 12 = O.

a regla de

o

) = -2.\"

lejas (las

2.\' + 7.

TEORIA DE ECLJACIONES 195

d) Como ni ¡(x) = 2x' + Ix? + 6 ni ti -x) = 2x· + 7x' + ó presentan variaciones de signos. las 4 raícesson complejas, ya que .1(0) ofo O''" .

e) Hay 1 variacíón de signo en .n.~')= x4 - 3x' - 4 = O Y 1 variación de signo en I( -xl = x" - 3x' - 4.

Por tanto, las raíces son: 1 positiva. 1 negativa. 2 complejas.

.fl Hay 1 variación de signo en f(x) = x' + h - 14 Y ninguna en .fi -xl = -X" - h - 14.

Por tanto, las raíces se 1: 1 positiva. 2 complejas.

g) Hay 1 variación de signo en fix¡ = .\.6 + X' - I Y variación cnfl -xl = .v'' - X" - 1.

Por tanto, las raíces son: I positiva. I negativa. 4 complejas .

11) Hay 2 variaciones de signo en [ix¡ = x" - 3x' - 4x + 1 Y 2 variaciones de signo en f1-xl = .v'' - 3x'+ 4x + l.

Por tanto, las raíces pueden ser:(1) 2 positivas, 2 negativas, 2 complejas(2) 2 positivas, O negativas. 4 complejas

(3) O positivas. 2 negativas. 4 complejas(4) O positivas. O negativas. 6 complejas

40. Determinar la naturaleza de las raíces de la ecuación .v" - I = O. siendo n entero y positivo y a) n par.b) n impar.

a) .I(x) = x" - 1 tiene I variación de signo y .I( -x) = x" - I también presenta variación de signo.

Por tanto. las raíces son: 1 positiva. 1 negativa. (n - 2) complejas.

h) ((x) = .v" - I tiene I variación de signo, y f( -x) = -x" - I no presenta variaciones de signo.

Por tanto. las raíces son: 1 positiva. O negativas. (n - 1) complejas.

41. Hallar las raíces racionales. si existen. de las ecuaciones siguientes. aplicando la regla de los signos de Descartes.

a) x3 - x' + 3x -- 27 = O. hl x3 + 2x + 12 = O, e) 2X5 + x - 66 = O. d) 3x' + 7x' + 6 = O

a) Aplicando la regla de los signos de Descartes, la ecuación tiene 3 ó 1 raíces positivas y ninguna negativa. Portanto, las posibles raíces racionales son los divisores positivos de 27. es decir. 1, 3.9, 27.

Ensayando todos estos valores de x, la única raíz racional es 3.

h) Aplicando la regla de los signos de Descartes. la ecuación carece de raíces positivas y tiene I negativa. Portanto. las posibles raíces racionales son los divisores enteros y negativos de 12. es decir. - 1, - 2. - 3.-4. -6. -12.

Ensayando todos estos valores de .v, la única raíz racional es - 2.

e) Aplicando la regla de los signos. la ecuación tiene I raíz positiva y ninguna negativa. Por tanto. las raícesracionales posibles son números de la forma bic, en donce h es un divisor entero de 66 y l' uno de 2.Las raíces posibles son. l. 2. 3. 6. 11. 21. 33. 66. I '2. 3:2. 11 '2. 33 '2.

Ensayando todos estos valores de .v se obtiene la raíz racional 2.

d) La ecuación no tiene raíces reales. ya que ni .flx 1 = 3x" + 7x' + 6 ni f( - x l = 3.\.4 + 7x' + 6 presentanvariaciones de signo y .flO l + O.

Por tanto. todas las raíces son complejas.

TEORIA DE ECUACIONES 195

J) Como ni ¡(x) = 2x' + 7X2 + 6 ni ¡I - x) = 2x· + 7x' + ó presentan va riaciones de signos. las 4 raices son compleja~. ya que .1(0) ~ O' .

e) Hay I variación de signo en . ((.~. ) = .\4 - 3x' - 4 = O Y I variación de signo en .n - xl = x' - 3x' - 4.

Por tanto , las raíces son: I positiva, I negativa, 2 complejas .

. fl Hay I variación de signo en ¡(x) = x 3 + h - 14 Y ninguna en .fl-xl = -X" - h - 14.

Por tanto , las raíces se 1: I positiva, 2 complejas.

g) Hay I variación de signo en .I(x) = x" + x 3 - I Y variación en .11 -xl = x" - x·, - 1.

Por tanto , las raíces son: I positiva. I negati va, 4 complejas.

h) Hay 2 variaciones de signo en ¡(x) = x" - 3X2 - 4x + 1 Y 2 variaciones de signo en I( -x) = XO - 3x'

+ 4.\ + l .

Por tanto, las raíces pueden ser: (1) 2 positivas, 2 negativas, 2 complejas (2) 2 positivas, O negativas, 4 complejas

(3) O positivas. 2 negativas. 4 complejas (4) O positivas, O negativas. 6 complejas

40. Determinar la naturaleza de las raíces de la ecuación .\" - 1 = O, siendo n entero y positivo y a) n par. b) n impar.

a) .1(.\) = x" - I tiene I variación de signo y .l1-x) = x" - 1 también presenta variación de signo.

Por tanto. las raíces son: I positiva, I negativa. (n - 2) complejas.

h) Frx) = .\JO - 1 tiene I variación de signo. y I( -x) = - x" - I no presen ta variaciones de signo.

Por tanto. las raíces son: I positiva. O negativas. (n - 1) complejas.

41. Hallar las raíces racionales. si existen. de las ecuaciones siguientes. aplicando la regla de los signos de Descartes.

a) x 3 - X2 + 3x -- 27 = O. h) x 3 + 2.\ + 12 = O. e) 2X5 + x - 66 = O, d) 3x4 + 7X2 + 6 = O

a) Aplicando la regla de los signos de Descartes, la ecuación tiene 3 ó I raíces positivas y ninguna negativa. Por tanto, las posibles raíces racionales son los divisores positivos de 27. es decir. 1, 3. 9. 27.

Ensayando todos estos valores de x. la única raíz racional es 3.

h) Aplicando la regla de los signos de Descartes, la ecuación carece de raíces positivas y tiene I negativa. Por tanto, las posibles raíces racionales son los divisores enteros y negativos de 12. es decir. - 1, - 2. - 3, - 4, -6, -12.

Ensayando todos estos valores de x . la única raíz racional es - 2.

e) Aplicando la regla de los signos. la ecuación tiene I raíz positiva y ninguna negativa. Por tanto. las raices racionales posibles son números de la forma b/l'. en donce h es un divisor entero de 66 y e uno de 2. Las raíces posibles son. l. 2. 3, 6, 11, 22, 33. 66. I '2. 3:2. II '2. 332.

Ensayando todos estos valores de x se obtiene la raíz racional 2.

d) La ecuación no tiene raíces reales. ya que ni .flx I = 3x' + 7 X2 + 6 ni I( - x) = 3.\.4 + 7 X2 + 6 presentan variaciones de signo y .110 I +- O.

Por tanto , todas las raíces son complejas.

http://carlos2524.jimdo.com/

Page 204: Algebra Superior Murray R Spiegel

196 TEORIA DE ECUACIONES

ESCRIBIR UNA ECUACION CUY AS RAICES SEAN LAS DE OTRA DADA MULTIPLICADAS POR UNACONSTANTE

42. Escribir una ecuación, en la variable o incógnita y, cuyas raíces sean el doble de las raíces de x3 - 6x2 + Ilx - 6 = o..,Toda raíz y de la ecuación pedida debe ser el doble de la correspondiente x de la ecuación dada. Por tanto,

y = 2x, o bien x = y/2, y la ecuación pedida es

(~)3 _ 6(~)2 + II(~) _ 6 = O o y3 - 2(6y)2) + 22(1ly) - 23(6) = O o y3 - 12y2 + 44y - 48 = O.222

Comprobación: Las raíces de x3 - 6x2 + 1Ix - 6 = O son 1,2,3.Las raíces de y3 - 12y2 + 44y - 48 = O son 2,4,6.

43. Escribir las ecuaciones en y cuyas raíces sean iguales a las raíces de las ecuaciones dadas multiplicadas por losnúmeros que figuran entre paréntesis.

a) x3 - x2 - 7x + 3 = O (2)

b) x3 - 19x - 30 = O (3)

d) 32x4 - 2x - I = O

e) 2x' + 3x2 + 1 = O

(4)

(-3)

3x2 4e) x4 + 5 -25 = O (5) f) x3 - 12x2 - 16x + 192 = O

a) y3 - 2(y2) - 22(7y) + 23(3) = O o y3 - 2y2 - 28y + 24 = O

b) y3 + 3(Oy2) - 32(19y) - 33(30) = O o y3 - I71y - 810 = O

3y2 4e) y4 + 5(Oy3) + 52(5) + 53(Oy) - 54(25) = O o y4 + 15y2 - 100 = O

d) 32y4 - 43(2y) - 44 = O o 32y4 - 128y - 256 = O o y4 - 4y - 8 = O

e) 2y'+ (-W(3y2) + (-3)'(1)=0 o 2y'-8Iy2-243=0

3 12y2 16y 192f) Y - 4 - 42 + 41 = O o y3 - 3y2 - Y + 3 = O

(1/4)

44. Hallar las ecuaciones en y cuyas raíces sean iguales a las raíces de las ecuaciones dadas multiplicadas por elmenor número entero y positivo necesario para que las ecuaciones pedidas tengan todos sus coeficientes enterosy el coeficiente de la mayor potencia sea igual a la unidad.

3 2 3x 9a) x - x - 2 + 8= O,

4 3 X 1b) x - 3x - 27 + 9" = O, e) 8x3 + 18x2 + X - 6 = O, d) 5x3 + 3 = O

a) Las raices de la ecuación en y deben ser el doble de las raíces de la dada.

3 2 2 3y 39 3 2Y - 2(y ) - 2 (-) +2 H = O o sea y - 2y - 6y + 9 = O2 8

b) Las raíces de la ecuación en y deben ser el triple de las raíces de la dada.

y4 _ 3(3y3) _ 33(1'..) + 34(~) = O o sea y4 - 9y3 - Y + 9 = O27 9

3 9x2X 3

e) Dividiendo los términos de la ecuación dada por 8: x + 4 + 8- ¡= O.

Las raíces de la ecuación en y deben ser el cuádruplo de las raíces de la dada.

9 2 3y3 + 4(~) + 42(~) _ 43(_) = O o sea y3 + 9)/ + 2y - 48 = O

4 8 4

d) Dividiendo los términos de la ecuación dada por 5: x3 + ~ = O.5

45. R,

±re

ra

46. R

eres

se

di

ESCRI

47. E

Q

a

b

196 TEORIA DE ECUACIONES

ESCRIBIR UNA ECUACION CUY AS RAICES SEAN LAS DE OTRA DADA MULTIPLICADAS POR UNA CONSTANTE

42. Escribir una ecuación, en la variable o incógnita y, cuyas raí«es sea_n el doble de las raíces de x 3 - 6X2 + Ilx - 6 = o . . ,

Toda raíz y de la ecuación pedida debe ser el doble de la correspondiente x de la ecuación dada. Por tanto, y = 2x, o bien x = y/2, y la ecuación pedida es

(~)3 _ 6(~)2 + II(~) _ 6 = O o y3 - 2(6y)2) + 22(1ly) - 23(6) = O o y3 - 12y2 + 44y - 48 = O. 222

Comprobación: Las raíces de x 3 - 6X2 + I Ix - 6 = O son 1,2,3. Las raíces de y3 - 12y 2 + 44y - 48 = O son 2,4,6.

43. Escribir las ecuaciones en y cuyas raíces sean iguales a las raíces de las ecuaciones dadas multiplicadas por los números que figuran entre paréntesis.

a)

b)

e)

a)

x 3 - X2 - 7 x + 3 = O (2)

x 3 - 19x - 30 = O (3)

3X2 4 X4 + 5 -2"5=0 (5)

y3 _ 2(y2) _ 22(7y) + 23 (3) = O

d) 32x4 - 2x - 1 = O

e) 2x' + 3X2 + I = O

f) x 3 - 12x2 - 16x +

o y3 _ 2y2 - 28y + 24 = O

b) y3 + 3(Oy2) - 32(19y) - 33 (30) = O o y3 - I7ly - 810 = O

3y2 4 e) y4 + S(Oy3) + 52(5) + S3(Oy) - 54 (2"5) = O o y4 + ISy2 - 100 = O

192 = O

d) 32y4 - 43(2y) - 44 = O o 32y4 - 128y - 256 = O o y4 - 4y - 8 = O

e) 2y'+ (-W(3y2) + (-3)'(1)=0 o 2y'-8Iy2-243=0

3 12y2 16y 192 f) Y - -4- - 42 + 41 = O o y3 - 3y2 - Y + 3 = O

(4)

(-3)

(1/4)

44. HaBar las ecuaciones en y cuyas raíces sean iguales a las raíces de las ecuaciones dadas multiplicadas por el menor número entero y positivo necesario para que las ecuaciones pedidas tengan todos sus coeficientes enteros y el coeficiente de la mayor potencia sea igual a la unidad.

3 2 3x 9 a) x - x -"2 + 8 = O,

4 3 X 1 b) x - 3x - 27 + 9" = O, e) 8x3 + 18x2 + X - 6 = O, d)

a) Las raices de la ecuación en y deben ser el doble de las raíces de la dada.

3 2 2 3y 3 9 3 2 Y -2(y)-2(-)+·2(-)=0 osea y -2y -6y+9=0 2 8

b) Las raíces de la ecuación en y deben ser el triple de las raíces de la dada.

y4 _ 3(3y 3) _ 33 (1:'.) + 34(~) = O o sea y4 - 9y3 - Y + 9 = O 27 9

3 9x2 X 3 e) Dividiendo los términos de la ecuación dada por 8: x + 4 + 8 - ¡ = o.

Las raíces de la ecuación en y deben ser el cuádruplo de las raíces de la dada.

9 2 3 y3 + 4(~) + 42(~) _ 43 (-) = O o sea y 3 + 9y 2 + 2y - 48 = O

4 8 4

d) Dividiendo los términos de la ecuación dada por 5: x 3 + ~ = O. 5

5x3 + 3 = O

http://carlos2524.jimdo.com/

Page 205: Algebra Superior Murray R Spiegel

NA

=0.

nto,

= O.

r los

r elteros

=0

TEORIA DE ECUACIONES 197

Las raíces de la ecuación en y deben ser el quíntuplo de las raíces de la dada.

y3 + 53(3/5) = O o sea )'3 + 75 = O

45. Resolver la ecuación 54x3 - 9x' - 12x - 4 = O.

di b 3 X' 2x 2Divi iendo por 54, se o tiene la ecuación x· - ti - 9 - n = o.

Para obtener una ecuación en y de coeficientes enteros, multiplicamos por 6 las raíces.

3 y' 6,2y 3 2Y - 6(-) - (-) - 6 (-) = O6 9 27o sea y3 - y' - 8y - 16 = O

Las posibles raíces racionales de la ecuación en y son los divisores±2, ±4, ±8, ± 16. Ensayando todos estos valores de y mediante laregla de Rufini se deduce la raíz 4. La ecuación que resulta de la división,

z O' I . 3 i.j'iY + 3y + 4 = ,tiene as raices - 2" ± -2- .

Las raíces de la ecuación en y divididas por 6 son iguales a lasraíces de la ecuación en x, esto es, x = y/6.

2 1 i.j'iLuego las raíces pedidas son - - - + -- .

3' 4 - 12

enteros de 16, es decir. ± l.

1 - 1 - 8 - 16

~ + 4 + 12 + 161+3+ 4+ O

46. Resolver la ecuación 64x· - 32x' + 4x' - 8x - 3 = O .. , • x3 x' x 3

Dividiendo pur 64, se obtiene la ecuacion x - '2 + 16 - 8' - 64 = o.

Para obtener una ecuación en y de coeficientes enteros, multiplicamos por 4 las raíces.

• y3 z y' , Y • 3y - 4(-) + 4 (-) - 4 (-) - 4 (-) = O o sea y. - 2y' + y' - 8y - 12 = O2 16 8 64

Las posibles raíces racionales de la ecuación en y son los divisoresenteros de 12, es decir, ±I, ±2, ±3. ±4, ±6, ±12. Ensayando todosestos valores de x mediante la regla de Rufini se deduce la raíz -1.

1 - 2 + 1 - 8 - 12.=:..!J - 1 + 3 - 4 + 12

1 - 3 + 4 - 12 + O

Las raíces de la primera ecuación obtenida al efectuar el cociente,según la regla de los signos, son todas positivas, y las racionales son losdivisores enteros de 12, es decir. 1, 2, 3, 4, 6, 12. Ensayando todosestos valores se deduce la raíz 3. Las raíces de la segunda ecuación son ± 2i.

1 - 3 + 4 - 12

3 I + 3 + O + 12

1+0+4+ OLas raíces de la ecuación en y, transformada, son - 1, 3. ± 2i. Por tanto. las raíces de la ecuación en x son

-1/4, 3/4. ± i/2, ya que x = y/4.

ESCRIBIR UNA ECUACION CUYAS RAICES SEAN OPUESTAS A LAS DE OTRA DADA

47. Escribir las ecuaciones en y cuyas raíces sean opuestas a las raíces de las ecuaciones siguientes.

a) x' + Tx? + 11x + 5 = O. b).'(· + 3x' - x - 27 = O. e) 2X5 - ,IOx· - 3x + 15 = O

Las ecuaciones en y se obtienen cambiando el signo de los términos de grado impar de las ecuaciones dadasy sustituyendo x por y, o bien efectuando la sustitución x = -y.

a) -y' + 7y' - Ily + 5 = O o )" - 7)" + 11)' - 5 = O

Las raíces de x' + 7x' + Ilx + 5 = O son -5. -1, -1; de y' - 7)" + 11)' - 5 = O son 5. 1. 1.

b) y. + 3y' + .\'- 27 = O

e) _2y5 - IOy· + 3y + 15 = O, o bien 2y5 + IOy· - 3r - 15 = O

TEORIA DE ECUACIONES 197

Las raíces de la ecuación en y deben ser el quíntuplo de las raíces de la dada.

y3 + 53 (3/5) = O o sea )'3 + 75 = O

45. Resolver la ecuación 54x3 - 9X2 - 12x - 4 = O.

D' 'd' d 3 X2 2x 2 IVI len o por 54, se obtiene la ecuación x - 6 - 9 - 27 = O.

Para obtener una ecuación en y de coeficientes enteros, múltiplicamos por 6 las raíces.

3 y2 622y 3 2 Y - 6(6) - (9) - 6 (27) = O o sea y3 - y2 - 8.1' - 16 = O

Las posibles raíces racionales de la ecuación en y son los divisores ± 2, ± 4. ± 8, ± 16. Ensayando todos estos valores de y mediante la regla de Rufini se deduce la raíz 4. La ecuación que resulta de la división,

enteros de 16, es decir. ± 1,

3 i.Ji y2 + 3y + 4 = O, tiene las raíces - 2 ± -2- '

Las raíces de la ecuación en y divididas por 6 son iguales a las raíces de la ecuación en x , esto es, x = y/6.

2 1 i.Ji Luego las raíces pedidas son :3 ' - 4 ± U .

46. Resolver la ecuación 64x· - 32x3 + 4 X2 - 8x - 3 = O. .• 4 x 3

X2 X 3 Dividiendo pur 64, se obtiene la ecuaclOn x - '2 + 16 - g - 64 = o.

1 - 1 - 8 - 16

~ + 4 + 12 + 16 1+3 + 4+ O

Para obtener una ecuación en )' de coeficientes enteros, multiplicamos por 4 las raíces.

• )'3 2 y2 J Y • 3 • 3 2 Y - 4('2) + 4 (16) - 4 (g) - 4 (64) = O o sea y - 2y + Y - 8)' - 12 = O

Las posibles raíces racionales de la ecuación en y son los divisores enteros de 12, es decir, ±I. ±2, ±3, ±4, ±6, ±12. Ensayando todos

estos valores de x mediante la regla de Rufini se deduce la raíz - l .

Las raíces de la primera ecuación obtenida al efectuar el cociente, según la regla de los signos, son todas positivas, y las racionales son los divisores enteros de 12, es decir. 1, 2. 3, 4, 6, 12 . Ensayando todos estos valores se deduce la raíz 3. Las raíces de la segunda ecuación son ± 2i.

1 - 2 + 1 - 8 - 12 -=-U - 1 + 3 - 4 + 12

1 - 3 + 4 - 12 + O

- 3 + 4 - 12

~ + 3 + O + 12

1 +0+4+ O

Las raíces de la ecuación en y, transformada, son - 1, 3. ± 2i. Por tan to. las raíces de la ecuación en x son - 1/4, 3/4. ± i/2, ya que x = y/4.

ESCRIBIR UNA ECUACION CUYAS RAICES SEAN OPUESTAS A LAS DE OTRA DADA

47. Escribir las ecuaciones en y cuyas raíces sean opuestas a las raíces de las ecuaciones siguientes.

a) x 3 + 7X2 + 11x + 5 = O. b)."(· + 3X2 - X - 27 = O. e) 2x s - .10x· - 3,\ + 15 = O

Las ecuaciones en y se obtienen cambiando el signo de los términos de grado impar de las ecuaciones dadas y sustituyendo x por y, o bien efectuando la sustitución x = - y.

a) _ ),3 + 7y2 _ 11)' + 5 = O o )'3 - 7)" + 11)' - 5 = O

Las raíces de x 3 + 7x' + Ilx + 5 = O son -5. -1, -1 ; de )'3 - 7)" + 11 )' - 5 = O son 5. 1. 1.

b) y . + 3.1'2 + r - 27 = O

e) _2y s - 10y· + 3y + 15 = O, o bien 2y s + 10y· - 31' - 15 = O

http://carlos2524.jimdo.com/

Page 206: Algebra Superior Murray R Spiegel

198 TEORIA DE ECUACIONES

ESCR!BIR UNA ECUACION CUYAS RAICES SEAN IGUALES A LAS DE OTRA DADA DISMINUIDASEN UNA CONSTANTE

48. Escribir las ecuaciones en y cuyas raíces sean iguales a las raíces de las dadas disminuidas en los números que figu-ran entre paréntesis.

a) 2.\"' - 17.\"2+ 26x + 45 = O (3)h) .\"'-x2-17x-15=0 (-3)

a) 2 - 17 + 26 + 45~ + 6 - 33 - 21

1 - 11 - 7 + 24+ 6 - 15

2 - 5 - 22+ 6

2 + I

La ecuación pedida es

2.1" + .1'2 - 22.1' + 24 = O

Las raíces de la ecuación en x son 5, 9/2, - 1.Las raíces de la ecuación en y son 2, 3/2, - 4.

Otra forma: Haciendo y = x - 3 o x = y + 3se obtiene el mismo resultado

e) 1 + O + O - 12 - 5~ +2+4+8-8

1 + 2 + 4 - 4 - 13+ 2 + 8 + 24

1 + 4 + 12 + 20+ 2 + 12

1 + 6 + 24+2

1+8La ecuación pedida es

y' + 8.1" + 24y2 + 20y - 13 = O

e) . .\". - 12x - 5 = O (2)d) .~'+8.\"2_2=0 (0,4)

b) 1-1-17-15-=2J - 3 + 12 + 15

1-4- 5+ O- 3 + 21

1- 7 + 16- 3

1-10

La ecuación pedida es

)" - 10.1'2 + 16y = O

Las raíces de la ecuación en x son -3, -1,5.Las raíces de la ecuación en y son O, 2, 8.

Obsérvese que disminuir las raíces en - 3 es lo mismoque incrementarlas en 3.

d) 1 + 8 + O - 2~ + 0,4 + 3,36 + 1,344

1 + 8,4 + 3,36 - 0,656+ 0,4 + 3,52

1 + 8,8 + 6,88+ 0,4

I + 9,2La ecuación pedida es

y' + 9,2y2 + 6,88y - 0,656 = O

RELACIONES ENTRE LAS RAICES Y LOS COEFICIENTES

49. Escribir las ecuaciones cuyas raíces sean 1, 3 - 2, - 4.

La ecuación será de la forma x' + p,x' + P2X2 + p,x + P. = O.p, = -(suma de raíces) = -(1 + 3 - 2 - 4) = 2.

P2 = + (suma de productos de las raíces tomadas dos a dos)

= (1)(3) + (1)(-2) + (1)(-4) + (3)(-2) + (3)(-4) + (-2)(-4) = -13.

Ps = - (suma de productos de las raíces tomadas tres a tres)

= -[(1)(3)(-2) + (1)(3)(-4) + (1)(-2)(-4) + (3)(-2)(-4)J = -14.

P. = +(producto de las raíces) = (1)(3)(-2)(-4) = 24.

La ecuación pedida es x' + 2.\"' - I3x2 - 14x + 24 = O.

SO. Dad,de o

51. Trancer ~

Ipoterraíce

didamásdada.

L

52. Hall:

53. Dosecuar

OtroLa e

54. Dad,a)b)e)d)

e)

a)

h)

http://carlos2524.jimdo.com/

Page 207: Algebra Superior Murray R Spiegel

ros que figu-

o mismo

TEORIA DE ECUACIONES 199

so. Dada la ecuación x3 - 8x2 + 9x + k = O, hallar el valor entero de k para que una de las raices sea el doblede otra.

Sean las raíces a. 2a, b.

Sumaderaíces=-(-8)=8=a+2a+b o (l)b=8-3a.Suma de productos de las raíces tomadas dos a dos = 9 = a(2a) + a(b) + 2a(b) o (2) 2a2 + 3ab = 9.Producto de las raíces = -.k = a(2a)(b) o (3) k = -2a2b.La solución de (1) y (2~ es a = 3, b = -1. Sustituyendo en (3), k = 18.

51. Transformar la ecuación 2x' + 8x3 + 5x2- 3x + 6 = O en una ecuación en y que carezca de término de ter-

cer grado.

Escribamos la ecuaciÓn, en primer lugar, con el coeficiente de la mayorpotencia igual a la unidad y apliquemos, a continuación, las relaciones entre lasraíces y los coeficientes.

Suma de raíces = - 8/2 = - 4. La suma de las raíces de la ecuación pe-dida en y debe ser cero, esto es, la suma de las raíces de la ecuación dadamás 4. Ello se consigue aumentando en 1 las cuatro raíces de la ecuacióndada, es decir, y = x + 1.

La ecuación pedida es 2y' - 7y2 + 3y + 8 = O.

2 + 8 + 5 - 3 + 6-=-!J -2-6+1+2

2+6-1-2+8-2-4+5

2+4-5+3- 2 - 2

2 + 2 - 7- 2

2 + O

52. Hallar la suma de los cuadrados de las raíces de x3 - 2X2 - 23x + k = O.

Sean las raíces a, b, e. Como (a + b + e)2 = a2 + b2 + e2 + 2(ab + be + ea), tendremos a2 + b2 + e2

= (a + b + e)2 - 2(ab + be + ea) = 22- 2( - 23) = 50.

53. Dos raíces de la ecuación incompleta 3x3- 17x2 + ... = O son 2,4. Hallar la tercera raíz y completar la

ecuación.

Sea la ecuación 3x3 - 17x2 + hx + k = O y sus raíces 2.4."

Suma de raíces = 17/3 = 2 + 4 + r ; de donde, = -1/3.

Suma de productos de las raíces tomadas dos a dos = h/3 = 2(4) + 2( - 1/3) + 4( -1/3) = 18/3; de don-de h = 18.

Producto de raíces = -k/3 = 2(4)(-1/3) = -8/3; de donde k = 8.

Así. pues. r = -1/3. Y la ecuación completa es 3x3 - 17x2 + 18x + 8 = O.

Otro método. Suma de raíces = 17/3 = 2 + 4 + r, de donde, = -1/3. Por tanto. las raíces son 2.4, -1/3.La ecuación cuyas raices son estas tres es (x - 2)(x - 4)(3x + 1) = O. o bien, 3:'(3 - 17x2 + 18x + 8 = O.

54. Dada la ecuación x3 - 9x + k = O. hallar el valor de la constante k para que:a) Dos raíces sean opuestas.b) Exista una raíz doble.e) Las tres raíces estén en progresión geométrica.d) Las tres raíces estén en progresión aritmética.e) Una raíz sea 19 + ,y3.a) Sean las raíces a. - a. b.

Tendremos. a + (-a) + b = O. ya que no hay término de segundo grado. Por tanto. b = O.Producto de raíces = a(-a)(b) = O = -k. Luego k = O.

b) Sean las raices a. a. b.Suma de raices = a + a + b = O (1).Producto de raices tomadas dos a dos = 2ab + a2

Producto de las tres raíces = ,,2b = - k (3 l.-9 (2).

TEORIA DE ECUACIONES 199

SO. Dada la ecuación x 3 - 8X2 + 9x + k = O, hallar el valor entero de k para que una de las raíces sea el doble

de otra.

Sean las raíces a. 2a, b.

Sumaderaíces=-(-8)=8=a+2a+b o (l)b=8-3a . Suma de productos de las raíces tomadas dos a dos = 9 = a(2a) + a(b) + 2a(b) o (2) 2a2 + 3ab = 9. Producto de las raíces = - .k = a(2a)(b) o (3) k = -2a2b. La solución de (1) y (2~ es a =:' 3, b = -1. Sustituyendo en (3), k = 18.

51. Transformar la ecuación 2x' + 8x3 + 5X2 - 3x + 6 = O en una ecuación en y que carezca de término de ter­cer grado.

Escribamos la ecuaciÓn, en primer lugar, con el coeficiente de la mayor potencia igual a la unidad y apliquemos, a continuación, las relaciones entre las raíces y los coeficientes.

Suma de raíces = - 8/2 = - 4. La suma de las raíces de la ecuación pe­dida en y debe ser cero, esto es, la suma de las raíces de la ecuación dada más 4. Ello se consigue aumentando en 1 las cuatro raíces de la ecuación dada , es decir, y = x + 1.

La ecuación pedida es 2y' - 7y2 + 3y + 8 = O.

52. Hallar la suma de los cuadrados de las raíces de x 3 - 2X2 - 23x + k = O.

2 + 8 + 5 - 3 + 6 ~ - 2 - 6 + l + 2

2+6-1-2+8 -2-4+5

2+4-5+3 - 2 - 2

2 + 2 - 7 - 2

2 + O

Sean las raíces a, b, e. Como (a + b + e)2 = a2 + b2 + e2 + 2(ab + be + ca), tendremos a2 + b2 + e2

= (a + b + e)2 - 2(ab + be + ca) = 22 - 2( - 23) = 50.

53. Dos raíces de la ecuación incompleta 3x 3 - 17x2 + ... = O son 2,4. Hallar la tercera raíz y completar la

ecuación.

Sea la ecuación 3x3 - 17x2 + hx + k = O y sus raíces 2,4,r.

Suma de raíces = 17/3 = 2 + 4 + r; de donde r = - 1/3.

Suma de productos de las raíces tomadas dos a dos = h/3 = 2(4) + 2( - 1/3) + 4( -1 /3) = 18/3; de don­de h = 18.

Producto de raíces = -k/3 = 2(4)(-1/3) = -8/3 ; de donde k = 8.

Así. pues, r = -1 /3, Y la ecuación completa es 3x 3 - 17x2 + 18x + 8 = O.

Otro método. Suma de raíces = 17/3 = 2 + 4 + r, de donde r = -1 /3. Por tanto, las raíces son 2, 4, - 1/3. La ecuación cuyas raíces son estas tres es (x - 2)(x - 4)(3x + 1) = O, o bien, 3,,3 - 17x2 + 18x + 8 = O.

54. Dada la ecuación x 3 - 9x + k = O, hallar el valor de la constante k para que:

a) Dos raíces sean opuestas. b) Exista una raíz doble . e) Las tres raíces estén en progresión geométrica. d) Las tres raíces estén en progresión aritmética.

e) U na raíz sea 19 + .,j3. a) Sean las raíces a, - a, b.

Tendremos, a + ( -a ) + b = O. ya que no hay término de segundo grado. Por tanto. b = O. Producto de raíces = a( - a)(b) = O = - k. Luego k = O.

b) Sean las raíces a, a. b. Suma de raíces = a + a + b = O (1). Producto de raíces tomadas dos a dos = 2ab + l/2

Proelucto ele las tres raíces = l/2h = - k (3).

-9 (2).

http://carlos2524.jimdo.com/

Page 208: Algebra Superior Murray R Spiegel

200 TEORIA DE ECUACIONES

Resolviendo el sistema formado por (1) Y (2) se deduce a = ±.)3, b = +2.)3. Sustituyendo en (3), k = ±6.)3.

el Sean las raíces alr, a, aroTendremos. alr + a + ar = a(l/r + I + r) = O, a2/r + a2 + a2r = a2(I/r + I + r) = -9, a3 = -k.Ahora bien. ni a ni (I/r + 1 + r) puede ser cero. ¿Por qué? En consecuencia, no existe valor alguno de k parael cúal las raíces están en progresión geométrica.

d) Sean las raíces a - d, a, a + d.Suma de raíces = a - d + a + a + d = 3a = O, es decir, a = O. Luego k = - (a - d)(a)(a + d) = O.

e) Sustituyendo en la ecuación dada, (19 + j3)3 - 9(.y9.. + j3) + k = O

Desarrollando, 9 + 919 + 9j3 + 3 - 9(19 + .j3i + k = o, de donde k = -12.

55. Si a, b, e, son las raíces de la ecuación 2X3 - 5x2 + 6x - 1 f, O, escribir la ecuación cuyas raíces sean 1.. , 1.. , 1.. ., a b e

Sustituyendo y = l/x se tiene, 2(1/y)3 - 5(1/y)2 + 6(1/y) - 1 = O, o bien y3 - 6y2 + 5y - 2 = O, cuyasraíces son los recíprocos de las correspondientes de la ecuación dada. (Comparar los coeficientes de la ecuacióndada con los de la pedida.)

56. Las raíces de x3 - 2X2 + 3x - 4 = O son a, b, e. Hallar los valores de las relaciones siguientes.

al a + b + e e) abe e) a3 + b3 + e3

1 1 1f) -+-+-a b eb) ab + be + ea d) a2 + b2 + e2

a) 2 (suma de raíces)b) 3 (suma de productos de raíces tomadas dos a dos)e) 4 (prod ucto de raíces)d) a2 + b2 + e2 = (a + b + e)2 - 2(ab + be + ea) = 22 - 2(3) = -2e) a3 - 2a2 + 3a - 4 = O, b3

- 2b2 + 3b - 4 = O Y e3 - 2e2 + 3e - 4 = OSumando, a3 + b3 + e3 - 2(a2 + b2 + e2) + 3(a + b + e) - 12 = OLuego a3 + b3 + e3 = 2(-2) - 3(2) + 12 = 2

1 1 1 1 1 1 1 1 1 3f) abe(- + - + -) = be + ae + ab. Se tiene 4(- + - + -) = 3 y - + - + - = -.abe abe abe4

Otro método. La ecuación cuyas raíces son dos recíprocas de las raíces de la ecuación dada es4y3 - 3y2 + 2y - 1 = O.

3 . 1 1 1 3La suma de las raíces de esta ecuación es -4' Luego - + - + - = - .

a b e 4

METODO DE HORNER PARA HALLAR RAICESIRRACIONALES

57. Hallar las raíces positivas de x3 - 2X2 - 2x - 7 = Ocon tres cifras decimales exactas.

f(%)

Como f(x) cambia de signo entre x = 3 y x = 4,existe una raíz entre estos dos valores y su primera cifraes 3, como se puede comprobar gráficamente.

% f(%)

O -71 -102 -113 -4

4 17

-----d-------------l-----x

a) Aplicando la regla de los signos de Descartes sededuce que esta ecuación tiene una raíz positiva.

lb

12

b) Se representa la funciónf(x) = x3 - 2X2 - 2x - 7desde x = O hasta x = 4.

nui

rab

d)gati

ecu

e)

ciórdonde

f)

y s

16,1

Es"es :

g)

Y SI

16,1

exa

http://carlos2524.jimdo.com/

Page 209: Algebra Superior Murray R Spiegel

r

TEORIA DE ECUACIONES 201

Se escribe la ecuación cuyas raíces sean las de la anterior dismi-nuidas en 3, es decir, se hace X, = x - 3.

1-2- 2-7~ +3+ 3+3

1 + 1 + 1 - 4+ 3 + 12

1+4+13+ 3

1 + 7

= -k.e k para Se obtiene la ecuación x~ + 7x~ + 13x, - 4 = tl que tiene una

raíz comprendida entre O y 1.

= O.

d) Por tanteos se deduce que I,(xd = x~ + 7x~ + 13x, - 4 es ne-gativo para X, = 0,2 (-1,112), y positivo para X, = 0,3 (0,557).

1+7 +13 -4~ + 0,2 + 1,44 + 2,888

1 + 7,2 + 14,44 - 1,112

l l,¡;,~.O, cuyasecuación

Por tanto, la raíz positiva de It!xd.= O es 0,2+ (y la raíz de laecuación dada es 3,2+ ).

1+7 +13 -40,3 I + 0,3 + 2,19 + 4,557------------------~

1 + 7,3 + 15,19 + 0,557

e) Se disminuyen en 0,2 las raíces de I,(xd, (x2 = X, - 0,2).

La segunda ecuación transformada es

1+7 +13 -40,2 I + 0,2 + 1,44 + 2,888

1 + 7,2 + 14,44 + 1,112+ 0,2 + 1,48

1 + 7,4 + 15,92+ 0,2

1 + 7,6

12(x2) = x~ + 7,6x~ + 15,92x2 - 1,112 = O

y su raíz positiva está comprendida entre O y 0,1.

Para aproximar esta raíz hasta la centésima, resolvemos la ecua-ción que resulta al prescindir de los términos anteriores al de primer grado. Es decir, 15,92x2 - 1,112 = O, dedonde X2 = 0,06+, 10cual indica que la raíz positiva de 12(x2) = O está comprendida entre 0,06 y 0,07 (y la raízde la ecuación dada es 3,26+).

La tercera ecuación transformada es

1 + 7,6 + 15,92 - 1,1120,06 I + 0,06 + 0,4596 + 0,982776

1 + 7,66 + 16,3796 - 0,129224+ 0,06 + 0,4632

1 + 7,72 + 16,8428+ 0,06

1 + 7,78

1) Se disminuye en 0,06 las raíces de 12(x2) = o.

13(x3) = x~ + 7,78x~ + 16,8428x3 - 0,129224 = O

y su raíz positiva está comprendida entre O y 0,01.

dada es Resolviendo la parte lineal,

16,8428x3 - 0,129224 = O, se deduce X3 = 0,007+.

Esto indica que la raíz positiva de/3(x3) = Oestá comprendida entre 0,007 y 0,008 (y la raíz de la ecuación dadaes 3,267+).

14(x4) = x¡ + 7,801x¡ + 16,951867x4

- 0,010942837 = O

1 + 7,78 + 16,8428 - 0,1292240,007 I + 0,007 + 0,054509 + 0,118281163

1 + 7,787 + 16,897309 - 0,010942837+ 0,007 + 0,054558

1 + 7,794 + 16,951867+ 0,007

1 + 7,801

g) Se disminuye en 0,007 las raíces de 13(x3) = o.

La cuarta ecuación transformada es

y su raíz positiva está comprendida entre O y 0,001.

Resolvíendo la parte lineal,

16,951867x4 - 0,010942837 = O,se deduce X4 = 0,0006+.

Por tanto, la raíz pedida de la ecuación 'dada es J,2676 +", o bien, tomando tres cifras decimalesexactas, 3,268.

TEORIA DE ECUACIONES

Se escribe la ecuación cuyas raíces sean las de la anterior dismi­nuidas en 3, es decir, se hace X, = x - 3.

Se obtiene la ecuación x~ + 7x f + \3x , - 4 = () que tiene una raíz comprendida entre ° y 1.

d) Por tanteos se deduce que f, (x d = x~ + 7 xf + \3x I - 4 es ne­gativo para x, = 0,2 (-1,112), y positivo para x, = 0,3 (0,557).

Por tanto, la raíz positiva de fdxd .= ° es 0,2+ (y la raíz de la ecuación dada es 3,2 + ).

1-2- 2-7 ~ +3+ 3+3

1 + 1 + 1 - 4 + 3 + 12

1+4+\3 + 3

1 + 7

1+7 +\3 -4

201

~ + 0,2 + 1,44 + 2,888 1 + 7,2 + 14,44 - 1,112

1+7 +\3 -4 0,3 I + 0,3 + 2,19 + 4,557

------------------~

e) Se disminuyen en 0,2 las raíces de f,(xd , (x2 = X, - 0,2).

La segunda ecuación transformada es

f2(X2) = x~ + 7 ,6x~ + 15,92x2 - 1,112 = ° y su raíz positiva está comprendida entre ° y 0,1.

1 + 7,3 + 15,19 + 0,557

1+7 +\3 -4 ~ + 0,2 + 1,44 + 2,888

I + 7,2 + 14,44 + 1,112 + 0,2 + 1,48

1 + 7,4 + 15,92 + 0,2

1 + 7,6

Para aproximar esta raíz hasta la centésima, resolvemos la ecua-ción que resulta al prescindir de los términos anteriores al de primer grado. Es decir, 15,92x2 - 1,112 = 0, de donde X2 = 0,06+, 10 cual indica que la raíz positiva de f2(X2) = ° está comprendida entre 0,06 y 0,07 (y la raíz de la ecuación dada es 3,26+).

f) Se disminuye en 0,06 las raíces de f2(X2) = O.

La tercera ecuación transformada es

f3(X3) = x~ + 7,78x~ + 16,8428x3 - 0,129224 = ° y su raíz positiva está comprendida entre ° y 0,01.

Resolviendo la parte lineal,

16,8428x3 - 0,129224 = 0, se deduce X3 = 0,007 +.

1 + 7,6 + 15,92 - 1,112 0,06 I + 0,06 + 0,4596 + 0,982776

1 + 7,66 + 16,3796 - 0,129224 + 0,06 + 0,4632

1 + 7,72 + 16,8428 + 0,06

1 + 7,78

Esto indica que la raíz positiva def3(x3) = ° está comprendida entre 0,007 y 0,008 (y la raíz de la ecuación dada es 3,267 +).

g) Se disminuye en 0,007 las raíces de f3(X3) = O.

La cuarta ecuación transformada es

f4(X4) = x¡ + 7,801x¡ + 16,951867x4

- 0,010942837 = ° y su raíz positiva está comprendida entre ° y 0,001.

Resolviendo la parte lineal,

16,951867x4 - 0,010942837 = 0, se deduce X4 = 0,0006+.

1 + 7,78 + 16,8428 - 0,129224 0,007 I + 0,007 + 0,054509 + 0,118281163

1 + 7,787 + 16,897309 - 0,010942837 + 0,007 + 0,054558

1 + 7,794 + 16,951867 + 0,007

1 + 7,801

Por tanto, la raíz pedida de la ecuación 'dada es :1,2676 +, o bien, tomando tres cifras decimales exactas, 3,268.

http://carlos2524.jimdo.com/

Page 210: Algebra Superior Murray R Spiegel

202 TEORIA DE ECUACIONES

58. Hallar una raíz real de la ecuación x3 + 3x + 8 = O con dos cifras decimales exactas.

Sea f(x) = x3 + 3x + 8; se tiene f(-x) = _x3- 3x + 8

a) Aplicando la regla de los signos de Descartes, f(x) = O tiene1 raíz real que es negativa, y f( - x) = O tiene 1 raíz real que espositiva.

----;;I--"!""""+-~-~- r

b) La raíz positiva de -f(-x) = x3 + 3x - 8 = O es de signocontrario a la raíz negativa de [tx¡ = O.

x O 1 12g(x) -8 -4\6

e) Sea g(x) = x3 + 3x - 8. La raíz positiva de f(-x) = O esigual a la raíz positiva de g(x) = O.

Como g(x) cambia de signo entre x = 1 Y x = 2, la primera cifra de la raíz de g(x) = O es 1.

d) Se disminuye en 1 las raíces de g(x).1 + O + 3 - 8

.!J +1+1+41 + 1 + 4 - 4

+1+2---1+2+6

+ 1

T+3

Se obtiene la ecuación x~ + 3x~ + 6x 1 - 4 = O que tiene una raízcomprendida entre O y l.

e) Calculando gtlxtl = xl + 3xf + 6x1 - 4 para las sucesivas dece-nas, se obtiene que es negativa para XI = 0,5 y positiva para XI = 0,6.

Por tanto, la raíz positiva de gtlxtl = O es 0,5+ (y la raíz de g(x) = O es 1,5+).

f) Se disminuye en 0,5 las raíces de g 1(x 1)'1+3 +6 -4

~ + 0,5 + 1,75 + 3,8751 + 3,5 + 7,75 - 0,125

+ 0,5 + 21 + 4 + 9,75

+ 0,51 + 4,5

La segunda ecuación transformada es

g2(X2) = x~ + 4,5xi + 9,75x2 - 0,125 = O

y su raíz positiva está comprendida entre O y O, l.

Para aproximar esta raíz hasta la centésima. resolvemos la partelineal, 9,75x2 - 0,125 = O, con lo cual X2 = 0,01 +. indica que la raízpositiva de g2(X2) = O está comprendida entre 0,01 y 0.02 (y la raíz de g(x) = O es 1,51 +).

g) Se disminuye en 0.01 las raíces de g2(X2) = O.

La tercera ecuación transformada es

l + 4,5 + 9.75 - 0,125O,Olj + 0,01 + 0,0451 + 0,097951

1 + 4,51 + 9,7951 - 0,027049+ 0,01 + 0,0452

1+4.52 + 9,8403+ 0.01

1 + 4,.53

g3(X3) = x~ + 4,53x~ + 9.8403x3 - 0.027049 = Oy su raíz positiva está comprendida entre O y 0.01.

Resolviendo la parte lineal, 9,8403.\3 - 0.027049 = O. sededuce X3 = 0.002+. Esto indica que la raíz de g3(X3) = O es0,002 + y la raíz de g(x) = O es 1,5 12 + .

Por tanto. la raíz de x3 + 3x + 8 = O con dos cifras decimales exactas es - 1,51.

59. Hallar

SI

SI

e

0,4

0.04

Luego

http://carlos2524.jimdo.com/

Page 211: Algebra Superior Murray R Spiegel

%

844

- 0,125+ 0,09795 I- 0,027049

TEORIA DE ECUACIONES 203

59. Hallar13 con tres cifras decimales exactas.

Sea x = 13. Tendremos x) = 3 Y [ix¡ = x) - 3 = O.

Según la regla de los signos de Descartes . .lix) = O tiene una raíz positiva (13) y dos raíces imaginarias.

Como .lil) es - y 1(2) es +", la primera cifra de la raíz es l.

1+0+0-3.!J ~.2:_L.:~:-..!.

1 + 1 + 1 - 2+1+2

1 + 2 + 3+1

1 + 3

Disminuimos las raíces de fix) = x) - 3 = O en \.

Entonces. f.<x tl = x~ + 3xf + 3x, - 2 = O.

1+3 +3 -2~ + 0,4 + 1,36 + 1,744

1 + 3,4 + 4,36 - 0.256+ 0,4 + 1,52

1 + 3.8 + 5,88+ 0.4

1 + 4.2

Como 1.<0,4) es - y f, (0.5) es +. disminuimos las raí-ces en 0.4.

1 + 4,2 + 5.88 - 0.2560.04 I + 0.04 + 0.1696 + 0,2420

1 + 4,24 + 6.0496 - 0.0140+ 0.04 + 0.1712

1 + 4.28 + 6.2208+ 0.04

1 + 4.32

5.88x2 - 0.256 = O. aproximadamente.Luego X2 = 0.04. aproximadamente.

1 + 4.32 + 6.2208 - 0.01400.002 I + 0.002 + 0.0086 + 0.0125

1 + 4.322 + 6.2294 - 0.0015+ 0.002 + 0.0086

1+-4,324 "+ 6.2380+ 0.002

----I + 4.326

6.2208.\') - 0.0140 = O. aproximadamente.Luego x) = 0.002. aproximadamente.

0.0015 .(-- = 0.0002. aproximadamente).6.2380

Luego 13 = 1,442. con tres cifras decimales exactas.

TEORIA DE ECUACIONES 203

59. Hallar13 con tres cifras decimales exactas.

Sea x = 13. Tendremos x 3 = 3 Y I(x) = x 3 - 3 = O.

Según la regla de los signos de Descartes. f(x) = O tiene una raíz positiva (13) y dos raíces imaginarias.

Como f(l) es - y j12) es +. la primera cifra de la raíz es 1.

1+0+0 - 3

!J~~ 1+1+1-2

+1+2

1 + 2 + 3 +1

1 + 3

1+3 +3 -2 ~ + 0,4 + 1,36 + 1,744

1 + 3,4 + 4,36 - 0.256 + 0,4 + 1,52

1 + 3.8 + 5,88 + 0,4

1 + 4,2

1 + 4,2 + 5.88 - 0.256 0.04 I + 0,04 + 0,1696 + 0,2420

1 + 4.24 + 6,0496 - 0.0140 + 0.04 + 0,1712

1 + 4.28 + 6.2208 + 0,04

1 + 4.32

1 + 4.32 + 6,2208 - 0,0140 0,002 I + 0.002 + 0,0086 + 0.0125

1 + 4.322 + 6.2294 - 0,0015 + 0.002 + 0.0086

¡-:¡:--4.324 -:; 6.2380 + 0.002

1 + 4.326

Disminuimos las raíces de /lx) = x3 - 3 = O en l .

Entonces. f, (x,) = xt + 3xr + 3x, - 2 = O.

Como 1, (0,4) es - y f, (0,5) es +. disminuimos las raí­ces en 0,4.

5,88x2 - 0.256 = O. aproximadamente. Luego x 2 = 0.04, aproximadamente.

6.2208x3 - 0,0140 = O. aproximadamente. Luego X3 = 0.002. aproximadamente.

0.0015 . (- - = 0.0002. aprox,madamente). 6.2380

Luego 13 = 1,442. con tres cifras decimales exactas.

http://carlos2524.jimdo.com/

Page 212: Algebra Superior Murray R Spiegel

204 TEORIA DE, ECUACIONES

PROBLEMAS PROPUESTOS

60. Si f(x) = 2X3 - x2 - X + 2. hallar a) f(O). h) f(2). e) J( - 1). d) /l/i). e) f(.}2).

61. Hallar el resto de las divisiones siguientes:

a) (2x' - 7) -i- (x + 1)

b) (x3 + 3x2 - 4x + 2) -i- (x - 2)

e) (3x3 + 4x - 4) -i- (x - t)

d) (4y3 + Y + 27) -i- (2y + 3)

e) (xl2+x6+1)+(x-.J=1)

/) (2xJJ + 35) -i- (x + 1)

62. Demostrar que x + 3 es un divisor de x3 + 7x2 + 10x - 6 Y que x = - 3 es una raíz de la ecuaciónx3 + 7x2 + 10x - 6 = O.

63. Determinar cuáles de los números siguientes son raíces de la ecuación y' + 3y3 + 12)' - 16 = O: a) 2.b) -4. e) 3, d) 1, e) 2i.

64. Hallar los valores de k para los cuales a) 4x3 + 3x2 - kx + 6k es divisible por x + 3.h) x' + 4kx - 4k2 = O tiene la raíz .r = 2.

6S. Aplicando la regla de Rufini, hallar el

a) (2x3 + 3x2 ~ 4x - 2) -i- (x + 1)b) (3x' + x3 - 4) -i- (x - 2)

cociente y el resto de las divisiones siguientes:

e) (y6 - 3y' + 4y - 5) -i- (y + 2)d) (4x3 + 6x2 - 2x + 3) -i- (2x + 1)

66. Si fix¡ = 2x' - 3x3 + 4x - 4. calcular /(2) y /(-3) aplicando la regla de Rufini.

67. Sabiendo que una raíz de x3 - 7x - 6 = O es -1, hallar las otras dos.

68. Demostrar que 3 y - /i son raíces de 2x' - x3- 3x2 - 31x - 15 = O. Hallar la otra raíz.

69. Hallar las raíces de las ecuaciones siguientes:

a) (x + 3)2(X - 2)3(X + 1) = Ob) '4x'(x + 2)4(X - 1) = O

e) (x2 + 3x + 2)(x2 - 4x + 5) = Od) (y2 + 4)2lv + 1)2 = O

70. Hallar las ecuaciones de coeficientes enteros cuyas raíces sean:

a) 2, -3, -/i e) ±3i. raíz doble 2 d) - 1 ± 2i, 2 ± ib) O, -4, 2/3, 1

71. Hallar una ecuación cuyas raíces sean ±.}2. - 1 ± ifi·

72. Hallar los valores de A y B para los cuales la ecuación A(2x - 3) + Bix - 2) = x es una identidad.

73. Hallar los valores de A. B y e para los cuales la ecuación siguiente es una identidad:A(x - l)(x - 2) + B(x + 2)(x - 2) + e(x + 2)(x - 1) = x2

- 5x - 2.

74. Escribir la ecuación de menor grado posible con coeficientes enteros. cuyas raíces sean las dadas.a) l. O. i b) 2 + i e) -1 + fi. 1/3 d) -2. iJ3 e).}2. i /) tn. 6/5

7S. En la ecuación x3 + ax? + bx + a = O. a y b son números reales. Si x = 2 + i es una raiz de la ecuación. ha-llar a y b.

76. Escribir la ecuación de menor grado posible con coeficientes enteros que tenga .}2 - 1 como raíz doble.

77. Escribir la ecuación de menor grado posible con coeficientes enteros que tenga una raíz igual a J3 + 2i.

78. Re

a)

b)

79. Ha

a)

b)

SO. Re

a)

b)

e)

81. Del

82. Re¡ro (te i

83. Sinla 1

84. Hala)

8S. Haltod,

86. Aplde 1

a)

b)

87. Dadnúrr

88. Dad

89. Dellplej:

90. a)

b)

e)

d)

91. Iraloper

a)

b)

http://carlos2524.jimdo.com/

Page 213: Algebra Superior Murray R Spiegel

uación

al 2.

ión, ha-

doble.

2i.

TEORIA DE ECUACIONES 205

78. Resolver las ecuaciones siguientes, una de cuyas raíces es la indicada:

al x4 + x3- 12x2 + 32x - 40 = O; 1 - ij3

bl 6:.:4 - 11x3 + x2 + 33x - 45 = O; 1 + ifiel x3

- 5x2 + 6 = O; 3 - j3di x4

- 4:.:3 + 6x2 - 16x + 8 = O; 2i

79. Hallar las raíces racionales, si existen, de las ecuaciones siguientes:

al x4 + 2X3 - 4x2 - 5x - 6 = O e) 2X4 - x3 + 2X2 - 2x - 4 = O

b) 4x3 - 3x + 1 = O d) 3x3 + x2 - 12x - 4 = O

SO. Resolver las ecuaciones siguientes:

a) x3 - x2 - 9x + 9 = O d) 4x4 + 8x3 - 5x2 - 2x + 1 = O

b) 2X3 - 3x2 - Ilx + 6 = O e) 5x4 + 3x3 + 8x2 + 6x - 4 = O

e) 3x3 + 2X2 + 2x - 1 = O f) 3x5 + 2X4 - 15x3 - IOx2 + 12x + 8 = O

81. Demostrar que a) fi - fi y b) .y2 son números irracionales. MI

82. Representar gráficamente la función f(x) = 2X3 - 3x2 + 12x - 16. A partir de ella, determinar a) el núme-ro de raíces positivas, negativas y complejas de 2X3 - 3x2 + 12x - 16 = O, b) un valor aproximado, median-te interpolación, de algún cero real de f(x) con dos cifras decimales exactas.

83. Situar gráficamente entre dos enteros consecutivos las raíces reales de la ecuación x4 - 3x2 - 6x - 2 = O. Hallarla menor raíz positiva de la ecuación, aproximada con dos cifras decimales exactas.

84. Hallar las cotas superior e inferior de las raíces reales de las ecuaciones siguientes.a) x3

- 3x2 + 2x - 4 = O b) 2X4 + 5x2 - 6x - 14 = O

85. Hallar las raíces racionales de 2X3 - 5x2 + 4x + 24 = O y, a continuación, resolver la ecuación para calculartodas sus soluciones.

86. Aplicando la regla de los signos de Descartes, determinar el número de raíces positivas, negativas y complejasde las ecuaciones siguientes:

a) 2X3 + 3x2 + 7 = O

b) 3x3- x2 + 2x - 1 = O

e) x5 + 4x3- 3x2 - X + 12 = O

d) x5 - 3x - 2 = O

87. Dada la ecuación 3x4- x3 + x2 - 5 = O, determinar a) el máximo número de raíces positivas, b) el mínimo

número de raíces positivas, e) el número exacto de raíces negativas, d) el máximo número de raíces complejas.

88. Dada la ecuación 5x3 + 2x - 4 = O, hallar el número de raíces a) negativas, b) reales.

89. Determinar si la ecuación x6 + 4x4 + 3x2 + 16 = O tiene a) 4 raíces complejas y 2 reales, b) 4 reales y 2 com-plejas, e) 6 complejas, d) 6 raíces reales.

90. a) Determinar el número de raíces positivas de la ecuación x6 - 7x2 - 11 = O.

'b) Determinar el número de raíces complejas de la ecuación x7 + x4 - x2 - 3 = O.

e) Demostrar que x6 + 2X3 + 3x - 4 = O tiene, exactamente, 4 raíces complejas.

d) Demostrar que x4 + x3 - x2 - 1 = O solo tiene una raíz negativa.

91. Transformar las ecuaciones siguientes en otras cuyas raíces sean las raíces de las dadas, teniendo en cuenta laoperación que se indica.

a) 2X3 - x2 + 6x - 3 = O; multiplicadas por 2

b) 2X4 - 5 = O; divididas por 3

http://carlos2524.jimdo.com/

Page 214: Algebra Superior Murray R Spiegel

206 TEORIA DE ECUACIONES

el x3 - 2X2 + 5x + 7 = O; multiplicadas por -2

dl x3 - 20x2 + 500x - 4000= O; divididas por lO

el 3x' + 2X3 '- 5x2 + 4x - 2 = O; con signo contrario

fl x3 + 3x2- 2x + 5 = O; disminuidas en 2

gl x3 + 3x2 - 2x + 1 = O; aumentadas en 2

h) x3 + 3x2 + 3x + 2 = O; aumentadas en 1

92. Demostrar que la ecuación ax3 + bx? + ex + d = O se puede transformar en otra que carezca de términoen x2 aumentando las raíces en btsa. Como aplicación, transformar x3 + 6x2 + 3x + 8 = O en otra ecuaciónque carezca de término en x2

.

93. Transformar 'Ias ecuaciones siguientes en otras de coeficientes enteros en las cuales el coeficiente del términode mayor grado sea igual a la unidad.

al 2X3 - 3x" - 1 = O b) 5x3 + 2x - 4 = O e) 2x' - 1 = O

94. Resolver las ecuaciones siguientes:

a) 8x3 - 20x2 + 14x - 3 = O

b) 8x' - 14x3 - 9x2 + llx - 2 = O

e) 4x3 + 5x2 + 2x - 6 = O

d) 2x' - x3 - 23x2 + 18x + 18 = O

95. Dada la ecuación 5x3 + 2x - 4 = O, hallar a) la suma de sus raíces, b) el producto de las mismas, e) la sumade los productos de sus raíces tomadas dos a dos.

96. Dada la ecuación 3x6 - x2 - 6 = O, hallar a) la suma de los productos de sus raíces tomadas tres a tres, b) lasuma de los productos de sus raíces tomadas cuatro a cuatro, e) el producto de las raíces.

97. Hallar la suma y el producto de las raíces de la ecuación 2X3 - 3x2 + bx - 6 = O.

98. Hallar las constantes a y b en la ecuación ax3 - 6x2 + 2ax - 3b = O sabiendo que la suma de sus raíces es3 y su producto 6.

99. Dada la ecuación x3 + 3x2 - 16x + k = O. hallar el valor de k sabiendo que una de las raíces es doble de laotra y que las tres son enteras.

100. Hallar el valor de k en la ecuación x3 + kx + 16 = O sabiendo que tiene dos raíces iguales.

101. Hallar las tres raíces de la ecuación 2X3 - x2 + ex + 4 = O sabiendo que dos de ellas son opuestas.

102. Hallar el valor de k en la ecuación x3 - 3X2 - 6x + k = O para que sus raíces estén en progresión a-rit-mética.

103. Hallar el valor de k en la ecuación x3 - 3x + k = O para que dos de sus raíces sean iguales.

104. Hallar las raíces de la ecuación 2X3 + (k + 2lx2 + (2k - 2lx + 1 - k = O sabiendo que su suma es 1/2.

lOS. Trañsforrnar la ecuación 3:e' - 12x3 + Tx + 5 = O en otra. en la incógnita y, que carezca de término de ter-cer grado.

106. Las raíces de la ecuación 2X3 + 3x2 + 4x + 2 = O son a. b. e. Hallar el valor de:

al a + b + e e) ahe(/3 + h3 + e3 1 1 1 1 1 1

el f) -+-+- gl -+-+-.b) ab + be + ea d) a2 + b2 + e2 (/ b e ab be ea

107. H

al

ble)

dl

e)

fl

108. El+

109. uSa

lIO. H¡

SOLVO

60. al

61. a)

63.

64. al

65. al

b)

66. 12,

67. 3,

68.

69. a)b)

70. a)b)

71. :e'

72. 'A,

73. A,

74. a)ble)

15. a =

76. x,

78. al

http://carlos2524.jimdo.com/

Page 215: Algebra Superior Murray R Spiegel

on

no

la

es

la

r-

TEOR1A DE ECUAC10NES 207

107. Hallar, por el método de Horner, la raíz que se indica con la aproximación especificada:

a) 2X3 + 3X2 - 9x - 7 = O; raíz positiva, con una cifra decimal exacta

b) x3 + 9x2 + 27x - 50 = O; raíz positiva, con dos cifras decimales exactas

e) x3 - 3x2 - 3x + 18 = O; raíz negativa, con una cifra decimal exacta

d) x3 + 6x2 + 9x + 17 = O; raíz negativa, con una cifra decimal exacta

e) XS + x4 - 27x3 - 83x2 + 50x + 162 = O; raíz comprendida entre 5 y 6, con dos cifras decimales exactas

f) x4 - 3x3 + Xl - 7x + 12 = O; raíz comprendida entre 1 y 2 con dos cifras decimales exactas.

108. En el cálculo de la flecha de una viga cargada de cierta longitud es necesario resolver la ecuación 4x3 - 150x2+ 1 500x - 2 871 = O. Hallar la raíz de dicha ecuación comprendida entre 2 y 3 con una cifra decimal exacta.

109. La longitud de una caja paralelepipédica es doble de su espesor y su altura es un metro mayor que su anchura.Sabiendo que el volumen de la caja en cuestión vale 64 m", hallar su ancho con una cifra decimal exacta.

110. Hallar,y20 con dos cifras decimales exactas.

SOLUCIONES DE LOS PROBLEMAS PROPUESTOS

60. a) 2

61. a) -9

e) O d) 3/2 e) 3fi

e) -13/8 d) 12 e) f) 33

b) 12

b) 14

63. -4, 1 y u son raíces

64. a) k = 9 b) k = 4, -2

2 365. al 2x + x - 5 + x _ l

100b) 3x4 + 6x3 + 13x2 + 26x + 52 + x _ 2

66. 12, 227

147e) yS - 5y4 + 10.1'3 - 20y2 + 40y - 76 + Y + 2

2 5d) 2x + 2x - 2 + 2x + 1

67. 3,-2

68. -1 ± u

69. a) raíz doble -3, raíz triple 2, -1b) raíz cuádruple O, raíz cuádruple -2, 1

e) -1, -2,2 ± id) raíces dobles ± 2i, raíz doble - 1

70. a) 2X3 + 3x2 - 11x - 6 = Ob) 3x4 + 7x3 - 18x2 + 8x = O

71. x4 - x2 - 10x - 4 = O

e) x4 - 4x3 + 13x2 - 36x + 36 = Od) x4 - 2X3 + 2X2 - 10x + 25 = O

72. A";' 2, B = - 3

73. A = 1, B = 2, C = - 2

74. a) x4- x3 + x2 - X = O

b l x2 - 4x + 5 = Oe) 3x3 + 5x2 - 8x + 2 = O

d) x3 + 2X2 + 3x + 6 = Oe) x4 - x2 - 2 = Of) 20x3 - 24x2 + 5x - 6 = O

15. a = -5, b = 9

76. x4 + 4x3 + 2X2 - 4x + 1 = O 77. x4 + 2X2 + 49 = O

78. a) 1 ± i.j3, -5, 2 b) 1 ± i.j2, - 5/3, 3/2 e) 3 ±.j3, -1 d) ±2i. 2 ± .j2

TEORIA DE ECUACIONES 207

107. Hallar, por el método de Horner, la raíz que se indica con la aproximación especificada:

a) 2X3 + 3X2 - 9x - 7 = O; raíz positiva, con una cifra decimal exacta

b) x 3 + 9X2 + 27x - 50 = O; raíz positiva, con dos cifras decimales exactas

e) x 3 - 3X2 - 3x + 18 = O; raíz negativa, con una cifra decimal exacta

d) x 3 + 6X2 + 9x + 17 = O; raíz negativa, con una cifra decimal exacta

e) XS + x4 - 27x3 - 83x2 + 50x + 162 = O; raíz comprendida entre 5 y 6, con dos cifras decimales exactas

f) x4 - 3x3 + x' - 7x + 12 = O; raíz comprendida entre 1 y 2 con dos cifras decimales exactas.

108. En el cálculo de la flecha de una viga cargada de cierta longitud es necesario resolver la ecuación 4x 3 - 150x2

+ 1 500x - 2871 = O. Hallar la raíz de dicha ecuación comprendida entre 2 y 3 con una cifra decimal exacta.

109. La longitud de una caja paralelepipédica es doble de su espesor y su altura es un metro mayor que su anchura. Sabiendo que el volumen de la caja en cuestión vale 64 m3

, hallar su ancho con una cifra decimal exacta.

110. Hallar fiO con dos cifras decimales exactas.

SOLUCIONES DE LOS PROBLEMAS PROPUESTOS

60. a) 2 b) 12 e) O d) 3/2 e) 3fi

61. a) -9 b) 14 e) -13/8 d) 12 e)

63. -4, 1 y 2i son raíces

64. a) k = 9 b) k = 4, -2

2 3 65. al 2x + x - 5 + x _ 1

100 b) 3x4 + 6x3 + 13x2 + 26x + 52 + x _ 2

66. 12, 227

67. 3, -2

68. -1 ± 2i

69. a) raíz doble -3, raíz triple 2, -1 b) raíz cuádruple O, raíz cuádruple -2, 1

70. a) 2X 3 + 3X2 - 11x - 6 = O b) 3x4 + 7x3

- 18x2 + 8x = O

71. x4 - X2 - 10x - 4 = O

72. A";' 2, B = - 3

73. A = 1, B = 2, C = - 2

74. a) x4 - x 3 + X2 - X = O b l X2 - 4x + 5 = O e) 3x3 + 5X2 - 8x + 2 = O

75. a = -5, b = 9

76. x4 + 4x3 + 2X2 - 4x + I = O

78. a) 1 ± ¡fi, -5, 2 b) I ± ifi, -5/3, 3/2

f) 33

147 e) y' - 5y4 + 10)'3 - 20y2 + 40y - 76 + --

. y + 2 2 5

d) 2x + 2x - 2 + --2x + 1

e) -1, -2,2 ± i d) raíces dobles ± 2i, raíz doble - 1

e) x4 - 4x3 + 13x2 - 36x + 36 = O d) x4 - 2X 3 + 2X2 - 10x + 25 = O

d) x 3 + 2X2 + 3x + 6 = O e) x 4

- X2 - 2 = O f) 20x3 - 24x2 + 5x - 6 = O

77. x4 + 2X2 + 49 = O

el 3 ±.j3, -1 d) ±2i. 2 ± .Ji

http://carlos2524.jimdo.com/

Page 216: Algebra Superior Murray R Spiegel

208 TEORIA DE ECUACIONES

79. a) -3, 2 b) 1/2, 1/2, -1 e) no tiene raíces racionales d) -1/3, ±2

SO. a)· 1, ±3

b) 3, -2, 1/2

e) 1/3, -t ± tftd) H, -1 ± fl

e) -1,2/5, ±flif) ±1, ±2, -2/3

82. 1,3783. Raíz positiva comprendida entre 2 y 3; raíz negativa comprendida entre -1 y O; raíz positiva = 2,41 aprox.

b) Cota superior 2, cota inferior - 284. a) Cota superior 3, cota inferior -1

85.

86.

-3/2, 2 ± 2i

a) 1 negativa, 2 complejasb) 3 positivas o 1 positiva, 2 complejase) 1 negativa, 2 positivas, 2 complejas od) 1 positiva, 2 negativas, 2 complejas o

1 negativa, 4 complejas1 positiva, 4 complejas

88. a) ninguna b) una 89. e)87. a) 3 b) 1 e) 1 d) 2

90. a) una b) cuatro o seis

91. a) x3 - x2 + 12x - 12 = Ob) 162x4

- 5 = Oe) 33 + 4x2 + 20x - 56 = O

92. x3 - 9x + 18 = O

93.

94.

95.

97.

100.105.

106.

107.

108.

109. 2,9 m

110. 2,71

d) x3 - 2X2 + 5x - 4 = Oe) 3x4 - 2X3 - 5x2 - 4x - 2 = Of).. x3 + 9x2 + 22x + 21 = O

g) x3 - 3x2 - 2x + 9 = Oh) x3 + 1 = O

a) x3 - 3x2 - 4 = O b) x3 + 10x - 100= O e) x2-2=O

a) 1/2, 1/2, 3/2 b) 2, -1, 1/4, 1/2 e) 3/4, -1 ± i d) 3, 3/2, -2 ± fl

a) O b) 4/5 e) 2/5 96. a) O b) -1/3 e) -2

a) Suma = 3/2, producto = 3 98. a = 2, b = 4 99. 12

-12 101. ±2, 1/2 102. 8 103. ±2 104. ±2, 1/2

3y4 - 18y2 - 17y + 3 = O

a) -3/2 b) 2 e) -1 d) -7/4 e) 21/8 f) -2 g) 3/2

a) 1,9 b) 1,25 e) -2,2 d) -4,9 e) 5,77 f) 1,38

2,5

DEFINICdistirEs di

1x = 1rresp

PROPIEr

1.

11.

IlI.

http://carlos2524.jimdo.com/

Page 217: Algebra Superior Murray R Spiegel

prox.CAPITULO 22

Logaritmos

DEFINICION DEL LOGARITMO. El logaritmo de un número positivo N en base b, POSItiVOYdistinto de la unidad, es el exponente x a que hay que elevar la base para obtener dicho número.Es decir b" = N, o bien x = log, N. ..

=0 Ejemplo 1. Como 32 = 9, el logaritmo de 9 en base 3 es 2, es decir, 2 = log, 9.

2. log, 8 es un número x al que se debe elevar la base 2 para obtener 8, es decir,2X = 8, x = 3. Por tanto, log, 8 = 3.

Las relaciones b" = N Y x = log, N son equivalentes; b" = N es la forma exponencial, yx = log, N la forma logaritmica. Como consecuencia, a cada propiedad de la potenciacion, le co-rresponde una propiedad de la logaritmación.

PROPIEDADES DE LA LOGARITMACION

1. El logaritmo del producto de dos números positivos M y N es igual a la suma de los logarit-mos de ambos, es decir,

11. El logaritmo del cociente de dos números positivos M y N es igual a la diferencia de los 10-garitmos de ambos, es decir,

III. El logaritmo de la potencia p de un número positivo M es igual al producto del exponente ppor el logaritmo de la base, es decir,

Ejemplo 1. log, 3(5) = log, 3 + log , 5

172. loglo 24 = log.¿ 17 - loglo 24

3h 1/3 110glO -J 2 = log.¿ 2 = 3" log.¿ 2

209

CAPITULO 22

Logaritmos

DEFINICION DEL LOGARITMO. El logaritmo de un número positivo N en base b, positivo y distinto de la unidad , es el exponente x a que hay que elevar la base para obtener dicho número. Es decir b" = N, o bien x = 10gb N.

Ejemplo 1. Como 32 = 9, el logaritmo de 9 en base 3 es 2, es decir, 2 = log3 9.

2. log2 8 es un número x al que se debe elevar la base 2 para obtener 8, es decir, 2" = 8, x = 3. Por tanto, log2 8 = 3.

Las relaciones b" = N Y x = 10gb N son equivalentes; b" = N es la forma exponencial, y x = 10gb N la forma logarítmica. Como consecuencia, a cada propiedad de la potenciación, le co­rresponde una propiedad de la logaritmacióll.

PROPIEDADES DE LA LOGARITMACION

1. El logaritmo del producto de dos números positivos M y N es igual a la suma de los logarit­mos de ambos, es decir,

11. El logaritmo del cociente de dos números positivos M y N es igual a la diferencia de los lo­garitmos de ambos, es decir,

III . El logaritmo de la potencia p de un número positivo M es igual al producto del exponente p por el logaritmo de la base, es decir,

Ejemplo 1. log2 3(5) = log2 3 + log2 5

17 2. 10g1o 24 = 10g1o 17 - 10g1o 24

3. log7 53 = 3 log7 5

10glo.y2 = 10g1o 21/3 = ~ 10g1o 2

209

http://carlos2524.jimdo.com/

Page 218: Algebra Superior Murray R Spiegel

210 LO(j¡\RITMOS

LOGARITMOS DECIMALES O VULGARES

LOGARITMOS DECIMALES O VULGARES. El sistema de logaritmos cuya base es 10 recibeel nombre de sistema decimal, vulgar o de Briggs. Cuando no se escriba la base, se sobrentiendeque ésta es igual a 10. Por ejemplo, log 25 = logia 25.

Consideremos la tabla siguiente.

Número N 0,0001 0,001 0,01 0,1 10 100 1000 10 000

Forma exponen- 10-4 10-3 10-2 10-1 100 101 102 103 104

cial de N

log N -4 -3 -2 -1 O 2 3 4

Es evidente que 101.5377 será un número mayor que 10 (que es 101), pero menor que 100 (quees 102). Realmente, 101.5377 = 34,49; por tanto, log 34,49 = 1,5377.

La parte entera de un logaritmo se denomina característica del mismo y la parte decimal man-tisa. En el ejemplo anterior la característica del logaritmo es 1 y la mantisa 0,5377.

La mantisa del logaritmo de un número se encuentra en tablas en donde aparece sin la comadecimal. Ha de entenderse, sin embargo, que dicha mantisa es la parte decimal, siempre positiva,de un número cuya parte entera (característica) no figura en las tablas, por deducirse de forma in-mediata como vamos a ver.

LA CARACTERISTICA de un logaritmo se determina en función del número de que se trate,de acuerdo con las reglas siguientes:

1) Si el número es mayor que 1, la característica es positiva y es igual al número de cifras enterasdisminuido en una unidad. Por ejemplo:

Número 5297

Característica 3

348

2

900

2

34,8

1

60 5,764

O

3

O

2) Si el número es menor que 1, la característica es negativa y es igual al número de ceros que hayainmediatamente después de la coma aumentado en una unidad. El signo negativo de la carac-terística se puede escribir de dos maneras: a) encima de la característica, como, por ejemplo,I, 2, etc., b) en la forma 9 -10,8 -lO, etc. Más concretamente, la característica dellogaritmodel número 0,3485 es 1, o bien 9 - 10; la correspondiente a 0,0513 es 2, o bien 8 - 10; la de0,0024 es l o bien 7 - 10.

HALLAR EL LOGARITMO DECIMAL DE UN NUMERO POSITIVO. Emplearemos la tablade logaritmos que figura en uno de los Apéndices.

Supongamos que se necesita conocer el logaritmo del número 728. Se busca en la tabla de 10-garitmos el número 72 en la columna N y siguiendo la horizontal, debajo de la columna 8, apareceel número 8621. que es la mantisa del logaritmo en cuestión. Como la característica es 2, podre-mos escribir, log 728 = 2,8621. (Quiere decir que 102.8621 = 28.)

La mantisa de log 72,8, log 7,28, log 0,728, log 0,0728, etc., es la misma e igual a 0,8621, perosus características son diferentes. Por ejemplo:

pr

re

ANTliqtre

http://carlos2524.jimdo.com/

Page 219: Algebra Superior Murray R Spiegel

ibende

(que

an-

omativa,a in-

rate,

teras

hayaarac-plo,

ritmola de

tabla

de 10-areceodre-

, pero

LOGARITMOS 211

lag 728 = 2,8621lag 72,8 = 1,8621lag 7,28 = 0,8621

lag 0,728 = 1,8621lag 0,0728 = 2,8621lag 0,00728 = 3.8621

o 9,8621-10u 8,8621-10o 7,8621-10

Si el número tiene cuatro cifras, la mantisa se obtiene interpolando por el método de las partesproporcionales.

Ejemplo. Hallar lag 4,638.

La característica es O. La mantisa se halla como sigue.

Mantisa de lag 4640 = 0,6665Mantisa de lag 4630 = 0,6656

Diferencia tabular = 0.0009

0,8 x diferencia tabular = 0,00072, o bien 0,0007 con cuatro cifras decimales.Mantisa de lag 4638 = 0,6656 + 0,0007 = 0,6663.

Luego lag 4,638 = 0,6663.

La mantisa de lag 4638, lag 463,8, lag 46,38, etc., es 6663, pero las características son dife-rentes. Por ejemplo, .

,.,

lag 4638 = 3,6663lag 463,8 = 2,6663lag 46,38 = 1,6663

lag 4,638 = 0,6663

lag 0,4638 = 1,6663

lag 0,04638 = 2,6663

lag 0,004638 = ~,6663

lag 0,0004638 = 4,6663

o bien

o bien

o bien

o bien

9,6663-10

8,6663-107,6663-10

6,6663-10

ANTILOGARITMO. Es el número correspondiente a un logaritmo dado. «El antilogaritmo de 3»quiere decir «el número cuyo logaritmo es 3»; en este caso, es fácil deducir que se trata del núme-ro 1 000.

Ejemplo 1. Dado lag N = 1,9058, hallar N.

En la tabla, la mantisa 9058 corresponde al número 805. Como la característicade lag N es 1, el número tendrá dos cifras enteras; por tanto, N = 80,5. (Escri-biremos este resultado así: anti¡og 1,9058 = 80,5 )

2. Dado lag N = 7,8657-10, hallar N.

En la tabla, la mantisa 8657 corresponde al número 734. Como la característicaes 7-10, el número tendrá dos ceros inmediatamente después de la coma; portanto, N = 0,00734 (es decir, antilog 7,8657-10 = 0,00734).

3. Dado lag N = 9,3842-10, hallar N.

Como la mantisa 3842 no aparece en las tablas, tendremos que hacer la inter-polación correspondiente.

Mantisa de lag 2430 = 0,3856Mantisa de lag 2420 = 0,3838

Diferencia tabular = 0,0018

Mantisa dada = 0,3842Mantisa más próxima menor = 0,3838

Diferencia = 0,0004

Luego 2420 + I~ (2430 - 2420) = 2422 con cuatro cifras, y N = 0,2422.

lag 728 = 2,8621

lag 72,8 = 1,8621

lag 7,28 = 0,8621

LOGARITMOS

lag 0,728 = 1,8621 o 9,8621-10

lag 0,0728 = 2,8621 u 8,8621-10

lag 0,00728 = 3.8621 o 7,8621 -10

211

Si el número tiene cuatro cifras, la mantisa se obtiene interpolando por el método de las partes proporcionales.

Ejemplo. Hallar lag 4,638.

La característica es O. La mantisa se halla como sigue .

Mantisa de lag 4640 = 0,6665 Mantisa de lag 4630 = 0,6656

Diferencia tabular = 0.0009

0.8 x diferencia tabular = 0.00072. o bien 0,0007 con cuatro cifras decimales. Mantisa de lag 4638 = 0,6656 + 0,0007 = 0,6663.

Luego lag 4,638 = 0,6663.

La mantisa de lag 4638, lag 463.8, lag 46,38. etc., es 6663, pero las características son dife­rentes. Por ejemplo ,

lag 4638 = 3.6663 lag 0,4638 = 1,6663 o bien 9,6663-10

lag 463,8 = 2,6663 lag 0,04638 = 2,6663 o bien 8,6663-10

lag 46.38 = 1,6663 lag 0,004638 = ~ ,6663 o bien 7,6663-10

lag 4,638 = 0,6663 lag 0.0004638 = 4,6663 o bien 6,6663-10

ANTlLOGARITMO. Es el número correspondiente a un logaritmo dado . «El antilogaritmo de 3» quiere decir «el número cuyo logaritmo es 3»; en este caso, es fácil deducir que se trata del núme­ro 1 000.

Ejemplo 1. Dado lag N = 1,9058, hallar N.

En la tabla , la mantisa 9058 corresponde al número 805. Como la característica de lag N es 1, el número tendrá dos cifras enteras; por tanto, N = 80,5. (Escri­biremos este resultado así : anti)og 1,9058 = 80,5 )

2. Dado lag N = 7,8657-10, hallar N.

En la tabla, la mantisa 8657 corresponde al número 734. Como la característica es 7-10, el número tendrá dos ceros inmediatamente después de la coma ; por tanto, N = 0,00734 (es decir. antilog 7,8657-10 = 0,00734).

3. Dado lag N = 9,3842-10, hallar N.

Como la mantisa 3842 no aparece en las tablas, tendremos que hacer la inter­polación correspondiente .

Mantisa de lag 2430 = 0,3856 Mantisa de lag 2420 = 0,3838

Diferencia tabular = 0.0018

Mantisa dada = 0.3842 Mantisa más próxima menor = 0,3838

Diferencia = 0,0004

Luego 2420 + 1~ (2430 - 2420) = 2422 con cuatro cifras, y N = 0.2422 .

http://carlos2524.jimdo.com/

Page 220: Algebra Superior Murray R Spiegel

212 LOGARITMOS

EL COLOGARITMO de un número positivo es el logaritmo de su recíproco.

1Así, pues, colog N = log IV = log 1 - log N = - log N, ya que log 1 = O.

Los cologaritmos se utilizan con mucha frecuencia en todos los cálculos en los que intervienendivisiones, o cocientes, ya que en lugar de restar el logaritmo del divisor, se puede sumar su co-logaritrno.

56Por ejemplo, log 73 = log 56 - log 73 = log 56 + colog 73.

PROBLEMAS RESUELTOS

FORMAS EXPONENCIAL y LOGARITMICA

1. Expresar las siguientes formas exponenciales en forma logarítmica:

a) p' = r; b) 23 = 8, d) r2 = ~9 '

e) 8-2/3 = ~4

e) 42 = 16,

a) q = log, r, b) 3 = log, 8,1

d) - 2 = log, 9"e) 2 = lo~ 16,

2. Expresar las siguientes formas logarítmicas en forma exponencial:

a) log, 25 = 2,I

e) log'l. 16 = 2, e) log, I = Od) log., a3 = 3,b) log, 64 = 6,

a) 52 = 25, I 1 Ie) (4) = 16' e) ,-0 = Ib) 26 = 64, d) a3 = a3,

3. Hallar ~I valor de los logaritmos siguientes:

a) log, 64.

h) log, 81.

el log'12 8.

d) log jiO = .v,

Sea log; 64 = x ; tendremos 4' = 64 = 43 Y X = 3.

Sea log, 81 = x; tendremos 3' = 81 = 34 Y x = 4.

Sea log'12 8 = x; tendremos (1)' = 8, (2-'r = 23, 2-' = 23 Y X = -3.

10' = jiO = 10'13, x = 1/3

e) log; 125.)5 = x, 5' = 125.)5 = 53. 5'12 = 57/2, x = 7/2

4. Resolver las ecuaciones siguientes:

a) log, x = 2, 32 = x, x=9

1y=-

8

e) log, 25 = 2, X' = 25, x = ± 5. Como la base es positiva, la solución es x = 5

9 2d) log, 4 = - '3 . _ 2'3 9

y , = 4' 1'213 = ~. 9'

4 8r = (_)3/2 = - es la solución. 9 27

e) log (3x' + 2x - 4) = O, 10° = 3X' + 2x - 4, x = 1, -5/33X' + 2x - 5 = O.

PROPIEI

S. Demc

s

6. Expre

a) h

b) I

e) h

d) 1,

e) 1,

f) l.

g) 1,

h) 1,

7. Sabietro el

a) l.

b) l.

e)

http://carlos2524.jimdo.com/

Page 221: Algebra Superior Murray R Spiegel

nenco-

213LOGARITMOS

PROPIEDADES DE LOS LOGARITMOS

S. Demostrar las propiedades de los logaritmos.

Sea M = bX y N = bY; tendremos x = log, M e y = log, N ..

1. Como MN = bí : bY = ~+Y, será log, MN = x + y = log, M + log, N.

M ~ M11. Como N = bY = b":", será log, N = x - y = log, M - log, N.

III. Como MP = (~)" = bPx, será log, MP = px = P log, M.

6. Expresar los logaritmos siguientes como una suma algebraica de logaritmos, aplicando las propiedades 1, 11, 111:

a) log, UVW = log, (UV)W = Iog, UV + log, W = log, U + log, V + log, W

UVb) log, IV = log, UV - log¡, W = log, U + log, V - log, W

XYZe) log -- = log XYZ - log PQ = log X + log Y + log Z - (Iog P + log Q)

PQ = log X + log Y + log Z - log P - log Q

U2d) log V3 = log U2 - log V3 = 2 log U - 3 log V

U2V3e) log W4 = log U2V3 - log ~ = log U2 + log V3 - log W4

= 2 log U + 3 log V - 4 log W

UI/2 1 2f) log -- = log U1/2 - log V2/3 = - log U - - log V

V2/3 2 3

P X3/2 3 3g) log, -- = log. - = log, X3/2 - log, y3/4 = - log, x - - log, YV y3/4 2 4

1 3 1h) log .ya2b-3/4el/3 = -{2 log a - - log b + - log e}

4 4 3

I 3 I= "2 log a - 16 log b + 12 log e

7. Sabiendo que log 2 = 0,3010, log 3 = 0,4771, log 5 = 0,6990, log 7 = 0,8451 (todos en base 10), hallar con cua-tro cifras decimales los logaritmos siguientes:

a) log 105 = log (3' 5· 7) = log 3 + log 5 + log 7 = 0,4771 + 0,6990 + 0,8451 = 2,0212

b) log 108 = log (22• 33) = 2 log 2 + 3 log 3 = 2(0,3010) + 3(0,4771) = 2,0333

2e) log yn. = log .;}32 • 23 = log (32/3• 2) = 3" log 3 + log 2 = 0,6191

LOGARITMOS 213

PROPIEDADES DE LOS LOGARITMOS

S. Demostrar las propiedades de los logaritmos.

Sea M = Ir Y N = b"; tendremos x = lo&, M e y = lo&, N . .

1. Como MN = bX' b" = Ir+Y, será lo&, MN = x + y = lo&, M + lo&, N.

M Ir M 11. Como N = bY = Ir-Y, será lo&, N = x - y = lo&, M - lo&, N.

III. Como MP = (Ir)" = bP\ será lo&, MP = px = P 10gb M.

6. Expresar los logaritmos siguientes como una suma algebraica de logaritmos, aplicando las propiedades 1, Il, IlI:

a) lo&, VVW = 10gb (VV)W = 10gb VV + 10gb W = lo&, V + 10gb V + lo&, W

VV b) lo&, IV = lo&, VV - 10gb W = 10gb V + lo&, V - lo&, W

XYZ e) log -- = log XYZ - log PQ = log X + log Y + log Z - (Iog P + log Q)

PQ = log X + log Y + log Z - log P - log Q

V2

d) log V 3 = log V 2 - log V 3 = 2 log V - 3 log V

V 2 V3

e) log W 4 = log V 2 V 3 - log ~ = log V 2 + log V 3

- log W 4

= 2 log V + 3 log V - 4 log W

Vl~ 1 2 f) log -- = log V 1/2 - log V 2/3 = - log V - - log V

V2/3 2 3

P x3/2 3 3

g) log. -- = log. - = log. x 3/2 - log. y3/4 = - log. x - - log. Y V y3/4 2 4

1 3 1 h) log ~a2b-3/4el/3 = -{2 log a - - log b + - log e}

4 4 3

I 3 I = - log a - - log b + - log e

2 16 12

7. Sabiendo que log 2 = 0,3010, log 3 = 0,4771, log 5 = 0,6990, log 7 = 0,8451 (todos en base 10), hallar con cua­tro cifras decimales los logaritmos siguientes:

a) log 105 = log (3' 5· 7) = log 3 + log 5 + log 7 = 0,4771 + 0,6990 + 0,8451 = 2,0212

b) log 108 = log (22 • 33 ) = 2 log 2 + 3 log 3 = 2(0,3010) + 3(0,4771) = 2,0333

2 e) log yn. = log ':;3 2 • 23 = log p2/1 . 2) = 3" log 3 + log 2 = 0,6191

http://carlos2524.jimdo.com/

Page 222: Algebra Superior Murray R Spiegel

214 LOGARITMOS

24 3' 23

d) log 2.4 = log lo = log 10 = log 3 + 3 log 2 - log 10

= 0,4771 + 3(0,3010) - 1 = 0,3801

81e) log 0.0081 = log 104 = log 81 - log 104 = log 34

- log 104

= 4 log 3 - 4 log lO = 4(0,4771) - 4 = -2,0916, o bien 7,9084-10

Nota. En forma exponencial será 10-2.0916 = 0,0081.

8. Expresar las siguientes relaciones por un solo logaritmo (mientras no se diga lo contrario, la base es 10):

2 2 10a) log 2 - log 3 + log 5 = log "3 + log 5 = log 3(5) = log 3

23 8b) 3 log 2 - 4 log 3 = log 23

- log 34 = log ? = log 8t

1 1 2e) - log 25 - - log 64 + - log 27 = log 251/2 - log 64113 + log 272/3

2335 5 45

= log 5 - log 4 + log 9 = log - + log 9 = log -(9) = log -444

5 1d) log 5 - 1 = log 5 - log lO = log lo = log 2"

e) 2 log 3 + 4 log 2 - 3 = log 32 + log 24 - 3 log lO = log 9 + log 16 - log 103

9' 16= log (9' 16) - log J03 = log ----¡Q3 = log 0,144

9. En las ecuaciones siguientes, despejar la incógnita que se indica:

a) log2 x = y + e x.

b) log a = 2 log b a.

el log, 1 = log, lo - I : l. log. 1 = log, lo - t log, e = log, lo + log, e:'

d) 2 log x + 3 log Y = 4 log z - 2 : y.

Despejando log y. 3 log Y = 4 log z - 2 - 2 log x y

4 2 2log Y = - log z - - - - log x = log Z4/3 + log 10-2/3 + log x-2/3 = log z4t310-2/3x-2t3.3 3 3

e) log (x + 3) = log x + log 3 log (x + 3) = log 3x, x + 3 = 3x, x = 3/2x.

LOGAR!"

10. Hall,

a) ,

b)

a)b)

11. Hall,

a)b)e)d) ,e)f)g)

12. Hall.

alb)e)d)

elj)g)

13. Expi

a)

b)

Efec

14. P =

http://carlos2524.jimdo.com/

Page 223: Algebra Superior Murray R Spiegel

LOGARITMOS 215

LOGARITMOS VULGARES

10. Hallar la característica de los logaritmos vulgares de los números siguientes:

a) 57 e) 5,63 e) 982,5 g) 186000 i) 0,7314 k) 0,0071b) 57,4 d) 35,63 f) 7824 11) 0,71 j) 0,0325 1) 0,0003

a) e) ° e) 2 g) 5 il 9 - 10 k) 7 - 10b) d) I f) 3 h) 9 - 10 j) 8 - 10 1) 6 - 10

11. Hallar los logaritmos siguientes:

a) log 87,2 = 1,9405b) log 37300 = 4,5717e) log 753 = 2,8768d) log 9,21 = 0,9643e) log 0,382 = 9,5821 - 10f) log 0,00159 = 7,2014 - 10g) log 0,0256 = 8,4082 - 10

11) log 6,753 = 0,8295i) log 183,2 = 2,2630j) log 43,15 = 1,6350k) log 876400 = 5,94271) log 0,2548 = 9,4062 - 10

m) log 0,04372 = 8.6407 - 10n) log 0,009848 = 7,9933 - 10

(8293 + 2)(2625 + 5)(6345 + 5)(9425 + 2)(4048 + 14)(6405 + 2)(9930 + 3)

12. Hallar los antilogaritmos siguientes:

a) Antilog 3,8531 = 7130b) Antilog 1,4997 = 31,6e) Antilog 9,8267 - 10 = 0,671d) Antilog 7,7443 - 10 = 0,00555e) Antilog 0,1875 = 1,54I) Antilog 2,3927 = 0,0247g) Antilog 4,9360 = 86300

11) Antilog 2,6715 = 469,3i) Antilog 4,1853 = 15320j) Antilog 0,9245 = 8,404k) Antilog f,6089 = 0,40641) Antilog 8,8907 - 10 = 0,07775

m) Antilog 1.2000 = 15.85n) Antilog 7,2409 - 10 = 0.001742

(3/9 x 10 = 3 aprox.)(6/28 x 10 = 2 aprox.)(2/5 x 10 = 41(4/11 x 10 = 4 aprox.)(3/6 x 10 = 51(13/27 x 10 = 5 aprox.)(4/25 x 10 = 2 aprox.)

13. Expresar los números siguientes cor-o potencias de 10: a) 893, b) 0.358.

a) Hallamos x de forma que 10" = 893. Luego x = log 893 = 2,9509 Y 893 = 102.9509

b) Hallamos x de forma que 10" = 0,358.

Luego x = log 0,358 = 9,5539 - 10 = -0,4461 Y 0,358 = 10-0.446'.

Efectuar con logaritmos las operaciones siguientes:

14. P = 3,81 x 43,4

log P = log 3,81 + log 43,4

log 3,81 = 0.5809(+) log 43,4 = 1.6375

log P = 2.2184 Luego P = antilog 2,2184 = 165.3

Obsérvese el significado exponencial del cálculo. Es decir,

3.81 x 43,4 = 10°.5800 x 101.6)75

100.'800+ 1.6)7S = 102.2184 = 165.3

http://carlos2524.jimdo.com/

Page 224: Algebra Superior Murray R Spiegel

216

b) col

LOGARITMOS

<

15. P = 73,4~ x 0.00462 x 0.5143 .• Iog P = log 73.42 + log 0,00462 + log 0,5143 20. P<

log 73,42 1,8658(+) log 0,00462 = 7,6646 - 10(+) log 0.5143 = 9,7112 - 10

log P = 19,2416 - 20 = 9,2416 - 10.

lag

Luego P = 0.1744 r21. P = -

784,6 x 0,043116. P = 28,23 log P = log 784,6 + log 0,0431 - log 28,23

(+

(+)log 784,6 = 2,8947log 0,0431 = 8,6345 - \O

11,5292 - 10

log 28,23 = 1,4507log P = \0,0785 - 10 = 0,0785

P = 1,198

(-)

0,4932 x 653,717. P = -::-'::-=-:-::--::-:-=-::

0,072 \3 x 8456

22. El persiendocada s

Numerador N Denominador D

log 0,4932 = 9,6930 - 10(+) log 653,7 = 2,8154

log N = 12,5084 - 10(-) log D = 2,7853

log P = 9,7231 - \O

P = 0,5286

log 0,07213 = 8,8581 - \O(+) log 8456 = 3,9272

.,.-::-c=-::-=---:-:-log D = 12,7853 - 10

= 2,7853

18. P = (7,284)'

log P = 5 log 7,284 = 5(0,8623) =e 4,3115 y P = 20490

COLOGAI<

23. Hallar(63,28)3 (0,00843 )2(0,4623)

19. P = -'-------:-:-:~c::-:-:-:-:-;---(412,3)(2,184)'

a) col

10g P = 3 log 63,28 + 2 10g 0,00843 + log 0,4623 - (Iog 412,3 + 5 log 2,184)

Numerador N Denominador D

(+)(+)

3 log 63,28 = 3(1,8013) = 5,40392 log 0,00843 = 2(7,9258 - 10) = 15,8516 - 20

log 0,4623 = 9,6649 - 10log N = 30,9204 - 30

(-) log D = 4,3117

log P = 26,6087 - 30 = 6,6087 - \OP = 0,0004062 (o bien, 4,062 x \0-4)

log D = 4,3117

log 412,3 = 2,6152(+) 5 log 2,184 = 1,6965

216 LOGARITMOS

. 15. P = 73,4;1 x 0.00462 x 0.5143 .. Iog P = log 73.42 + log 0,00462 + log 0,5143

log 73,42 1,8658 (+) log 0,00462 = 7,6646 - 10 (+) log 0.5143 = 9,7112 - 10

log P = 19,2416 - 20 = 9,2416 - 10. Luego P = 0.1744

784,6 x 0,0431 16. P= ----c:-c:--::-c:---

28,23 log P = log 784,6 + log 0,0431 - log 28,23

log 784,6 = 2,8947 (+) log 0,0431 = 8,6345 - 10

11 ,5292 - 10

(-) log 28,23 = 1,4507

log P = 10,0785 - 10 = 0,0785

P = 1,198

0,4932 x 653,7 17. P = --'-------,---'--

0,07213 x 8456

18. P = (7,284)'

Numerador N

log 0,4932 = 9,6930 - 10 (+) log 653,7 = 2,8154

log N = 12,5084 - 10 (-) log D = 2,7853

log P = 9,7231 - 10

P = 0,5286

Denominador D

log 0,07213 = 8,8581 - 10 ( +) log 8456 = 3,9272

~=-:c:---:-::­log D = 12,7853 - 10

= 2,7853

log P = 5 log 7,284 = 5(0,8623) '" 4,3115 Y P = 20490

(63,28)3 (0,00843 )2(0,4623) 19. P = -----:-:-:-:-=-:::-=--:-7'--

(412,3)(2,184)'

log P = 3 log 63,28 + 2 log 0,00843 + log 0,4623 - (log 412,3 + 5 log 2,184)

Numerador N

3 log 63,28 = 3(1,8013) = 5,4039 (+) 2 log 0,00843 = 2(7,9258 - 10) = 15,8516 - 20 (+) log 0,4623 = 9,6649 - 10

log N = 30,9204 - 30 (-) log D = 4,3117

Denominador D

log 412,3 = 2,6152 (+) 5 log 2,184 = 1,6965

log D = 4,3117

log P = 26,6087 - 30 = 6,6087 - 10 P = 0,0004062 (o bien, 4,062 x 10- 4

)

http://carlos2524.jimdo.com/

Page 225: Algebra Superior Murray R Spiegel

LOGARITMOS 217

20. P = 10,8532

I I Ilog P = 5 log 0,8532 = 5(9,9310 - 10) = 5(49,9310 - 50) = 9,9862 - 10 Y P = 0,9688

1 1log P = 3 log 78,41 + - log 142,3 - - log 0,1562

2 4Numerador N Denominador D

3 log 78,41 = 3(1,8944) = 5,6832(+) t log 142,3 = -!-(2,1532) = 1,0766

log N = 6,7598 = 16,7598 - 10

(-) log.D = 9,7984 - 10

log P = 6,9614

P = 9 150 000 o bien

* log 0,1562 = *(9,1937 - 10)

= :t(39,1937 - 40)log D = 9,7984 - 10

9,15 X 106

22. El periodo T de la oscilación de un péndulo simple de longitud 1 viene dado por la fórmula T = 2nJ/¡gsiendo g la aceleración de la gravedad. Hallar T (en segundos) si 1 = 281,3 cm y g = 981,0 cm por segundo encada segundo. El valor 2n = 6,283.

T = 2n j!g = 6,283281,3981,0

log T = log 6,283 + t(log 281,3 - log 981,0)

log 6,283 = = 0,7982(+) t log 281,3 = t(2,4492) = 1,2246

2,0228

(-) t log 981,0 = t(2,9917) = 1,4959log T = 0,5269

T = 3,365 segundos

COLOGARITMOS

23. Hallar a) colog 42,36, b) colog 0,8536.

1a) colog 42,36 = log 42,36 = log 1 - log 42,36

log 1 = 10,0000 - 10(-) log 42,36 = 1,6270

colog 42,36 = 8,3730 - 10

1b) colog 0,8536 = log 0,8536 = log 1 - log 0,8536

log 1 = 10,0000 - 10(-) log 0,8536 = 9,9313 - 10

colog 0,8536 = 0,0687

LOGARITMOS 217

20. P = 10,8532

1 1 1 log P = 5 log 0,8532 = 5(9,9310 - 10) = 5(49,9310 - 50) = 9,9862 - 10 Y P = 0,9688

(78 ,41)3 J 142,3 21. P = --=====--

10,1562

. 1 1 log P = 3 log 78,41 + - log 142,3 - - log 0,1562

Numerador N

3 log 78,41 = 3(1 ,8944) = 5,6832 (+) t log 142.3 = W,1532) = 1,0766

log N = 6,7598 = 16,7598 - 10

(-) 10g. D = 9,7984 - 10

log P = 6,9614

2 4

Denominador D

i log 0,1562 = *(9,1937 - 10)

= i{39,1937 - 40) log D = 9,7984 - 10

P = 9 150000 o bien 9,15 x 106

22. El periodo T de la oscilación de un péndulo simple de longitud I viene dado por la fórmula T = 2nJlii siendo g la aceleración de la gravedad. Hallar T (en segundos) si 1= 281,3 cm y g = 981,0 cm por segundo en cada segundo. El valor 2n = 6,283.

COLOGARITMOS

A ~81,3 T = 2n - = 6,283 --

g 981,0

log T = log 6,283 + t(log 281,3 - log 981,0)

log 6,283 = = 0,7982 (+) t log 281,3 = t(2,4492) = 1,2246

2,0228

(-) t log 981 ,0 = t(2,9917) = 1,4959

log T = 0,5269

T = 3,365 segundos

23. Hallar a) colog 42,36, b) colog 0,8536.

1 a) colog 42,36 = log 42,36 = log 1 - log 42,36

log 1 = 10,0000 - 10 (-) log 42,36 = 1,6270

colog 42,36 = 8,3730 - 10

1 b) colog 0,8536 = log 0,8536 = log 1 - log 0,8536

log 1 = 10,0000 - 10 (-) log 0,8536 = 9,9313 - 10

colog 0,8536 = 0,0687

http://carlos2524.jimdo.com/

Page 226: Algebra Superior Murray R Spiegel

218 LOGARITMOS

24. Demostrar que a) colog MN = colog M + colog N y b) colog MP = p colog M, siendo M,N> O.

l 1 1a) colog MN = log MN = log M + log N = colog M + colog N

1 1b) colog MP = log MP = P log M = P colog M

(372,1)(0,0862)25. Calcular, empleando cologaritmos, P = (4,315)(0,7460)'

log P = log 372,1 + log 0,0862 - log 4,315 - log 0,7460

log 372,1 = 2,5706(+ ) log 0,0862 = 8,9355 - 10(+) colog 4,315 = 9,3650 - 10 (log 4,315 = 0,6350)(+) colog 0,7460 = 0,1273 (log 0,7460 = 9,8727 - 10)

log P = 20,9984 - 20 = 0,9984

P = 9,963

26. Calcular, empleando cologaritmos, P = 4~ •~ 37,31 (4,863)3

!- log 0,8730 = !-(9,9410 - 10) = 1(19,9410 - 20) = 9,9705 - 10(+) * colog 37,31 = *(8,4282 - 10) = *<38,4282 - 40) = 9,6071 - 10(+) 3 colog 4,863 = 3(9,3\31 - 10) = 27,9393 - 30 7,9393 - 10

log P = 27,5169 - 30 = 7,5169 - 10

P = 0,003288 o 3,288 x 10- 3

ECUACIONES EXPONENCIALES

27. Calcular x en: 52%+2 = 35.- i

Tomando logaritmos,

Luego

(2x + 2) log 5 = (5x - 1) log 3

2x log 5 - 5x log 3 = -Iog 3- 2 log 5,

x(2 log 5 - 5 log 3) = -Iog 3 - 2 log 5,

ylog 3 + 2 log 5 0,4771 + 2(0,6990) 1,8751

x= = =--.5 log 3 - 2 log 5 5(0,4771) - 2(0,6990) 0,9875

log 1,875 = 10,2730 - 10(-) log 0,9875 = 9,9946 - 10

log x = 0,2784

x = 1,898

28. H

29. R,

a)

b)

30. El

a)

31. D

a)

b)

32. o,

33. D

34. H

a:b)

35. H

alb)

36. H

a)b)

37. H

a'jh:

38. H

ab

39. E

40. ea

b

d

http://carlos2524.jimdo.com/

Page 227: Algebra Superior Murray R Spiegel

o.

LOGARITMOS 219

PROBLEMAS PROPUESTOS

28. Hallar: a) log, 32, b) log 110, e) log, 1/9, d) logl/4 16. e) log, e". f) 10gB 4.

29. Resolver las ecuaciones siguientes:

a) log, x = 3

b) log Y = -2

e) log, x3 = 3/2

f) log¡,_l} (4x - 4) = 2

e) log, 8 = -3

d) log, (2x + 1) = 1

30. Expresar como suma algebraica de logaritmos:

e) log e .JXI/2y-1/2

31. Despejar la incógnita que se indica en las ecuaciones siguientes:

a) 2 log x = log 16; x

b) 3 log Y + 2 log 2 = log 32; Y

e) log, F = log, 4 - 2 log, x; F

d) log, (30 - U) = log, 30 - 21; U

32. Demostrar que si a y b son positivos y j 1, (log, b)(log¡, a) = 1.

33. Demostrar que 10108 N = N siendo N > O.

34. Hallar la característica de los logaritmos vulgares de los números siguientes:

a) 248b) 2,48

e) 0,024d) 0,162

e) 0,0006f) 18,36

k) 237,631) 146,203

m) 7000000n) 0,000007

g) 1,06h) 6000

i) 4j) 40,60

35. Hallar el logaritmo vulgar de los números siguientes:

a) 237b) 28,7

e) 1,26d) 0,263

e) 0,086f) 0,007

i) 0,000000728j) 6000000

m) 1n) 1000

k) 23,701) 6,03

g) 10400h) 0,00607

36. Hallar el antilogaritmo de los números siguientes:

a) 2,8802 e) 0.6946 e) 8,3160 - lO ,g) 4,6618 i) r,9484b) 1,6590 d) 2,9042 f) 7,8549 - lO h) 0,4216 j) 9,8344 - lO

37. Hallar. interpolando, el logaritmo de los números siguientes:

a) 1463 e) 86,27 e) 0,6041 g) 1,006 i) 460,3h) 810,6 d) 8,106 f) 0,04622 h) 300,6 j) 0,003001

38. Hallar, interpolando, el antilogaritmo de los números siguientes:

a) 2.9060 e) 1,6600 e) 3,7045 g) 2,2500 i) r.ooob) r,4860 d) r,9840 f) 8,9266 - 10 h) 0,8003 j) 1,2925

39. Expresar los números siguientes como potencias de 10: a) 45,4, b) 0,005278.

40. Calcular:

a) (42,8 )(3,26 )(8,10) 5608 h)906

(0,148 )(47,6)e)

(0,4536)(11000) (3.142)(14.6)b)

284 (3,92)3(72,16)y(l600)(310M

(1,86 )(86, 7) f) i)e) j6s4 7290

(2,87)(1,88)

2453 g) 3,14) 11,65/32j)

(5,52)(2610)d) ---

(67,2)(8,55) (7,36)(3.142)

http://carlos2524.jimdo.com/

Page 228: Algebra Superior Murray R Spiegel

220 LOGARITMOS

20.0 0.0613 1 3241. Resolver la ecuación: - = (--) ..

14.7 x

42. La fórmula D = :; W se utiliza para calcular el diámetro que debe tener un globo esférico para0.5236(A - G)

elevar un peso W. Hallar D si A = 0,0807, G = 0.0056 Y W = 1250.

43. Dada la fórmula T = 27[JIfg. hallar 1 si T = 2.75. 1[ = 3.142 Y 11= 9,81.

44. Hallar x en las ecuaciones:

a) 3' = 243 e) 2,+2 = 64 e) X-3/4 = 8 g) t«: 1/2 = 4 i) 5'- 2 = 1

b) 5' = 1/125 d) x-2 = 16 f) x-2/3 = 1/9 11) 3' = 1 j) 22,+3 = l

45. Resolver las ecuaciones: a) 42'-1 = 5'+2, b) 3'-1 = 4' 51-h

46. Hallar a) colog 58,3, b) colog 0,07312

47. Calcular. empleando cologaritmos,(28,3)(471,5)

b)(4,36)(2,143)3

a) (55,21 )(3,142)' (0,0258)2

SOLUCIONES J>E LOS PROBLEMAS PROPUESTOS

28. a) 5 b) 1/4 e) -2 d) -2 e) x f) 2/3

29. a) 8 h) 0,01 e) 1/2 d) e) 2 f) 5

I 130. a) 3 log U + 2 log V - 5 log W e) (; log, x - (; log, Y

1 3 1 7 3b) - log 2 + - log x + - log J' - - log ~ d) log x - 2 log Y + 3 log z - 2 log a + 4 log b

2 2 2' 2

31. a) 4 h) 2 e) F = 4/x2 d) U = 30(1 - e-2/)

34. a) 2 e) 2 e) 4 11) O i) O k) 2 m) 6b) O d) f f) 1 h) 3 j) 1 1) 2 n) tí

35. a) 2,3747 e) 0,1004 e) 2,9345 11) 4,0170 i) 7,8621 k) 1,3747 m) 0.0000b) 1,4579 d) T,4200 f) 7,8451-10 11) 5,7832 j) 6,7782 /) 0,7803 n) 3,0000

36. a) 759 e) 4,95 e) 0,0207 11) 45900 i) 0,888b) 45,6 d) 0,0802 f) 0,00716 11) 2,64 j) 0.683

37. a) 3,1653 e) 1,9359 e) f,7811 g) 0.0026 i) 2.6631b) 2,9088 d) 0,9088 f) 8,6648-10 11) 2,4780 j) 7,4773-10

38. a) 805,4 e) 45,71 e] 5064 11) 177.8 i) 0,2951b) 0,3062 d) 0,9638 fJ 0,08445 11) 6,314 j) 19,61

39. a) 101.6571 b) \0- 2,2776

40. al 1130 e) 29,9 e) 1,124 11) 1,90 i) 145,5 41. 0,0486 42. 31,7 43. 1,88b) 0,0248 d) 4,27 f) 860 11) 4,44 j) 8,54

44. a) 5 e) 4 e) 1/16 11) 49/16 i) 2b) -3 d) ± 1/4 f) ±27 11) O j) -3/2

45. a) 3,958 b) 0,6907

46. a) 8,2343 - \O b) 1,1360,47. a) 76,93 b) 254,0

INTEIdeP(la

REOI'dedE%

2C

INTEIfnur

y

alde

INTEIprmfaaeselinigal

lo

http://carlos2524.jimdo.com/

Page 229: Algebra Superior Murray R Spiegel

para

1,88

CAPITULO 23

Intereses y anualidades

INTERES es el dinero pagado por un individuo u organización por el uso y disfrute de una cantidadde dinero llamada capital. El interés se paga, normalmente, al final de periodos de tiempo iguales,por ejemplo, al año, al semestre o al trimestre. La suma del capital inicial y su interés es el capi-tal final.

REDITO O TIPO DE INTERES. Es la relación entre el interés de un capital en una unidadde tiempo y dicho capital. La unidad de tiempo, mientras no se advierta otra cosa, se considerade un año. El rédito se expresa, generalmente, en tantos por ciento (%) o en tantos por uno igual al% dividido por 100.

Por ejemplo, si el capital es de 10000 pesetas y el interés es de 200 pesetas al año, el rédito es200/10000 = 0,02 = 2 %. El tanto por ciento es 2 y el tanto por uno 0,02.

INTERES SIMPLE. Es el interés calculado sobre el capital inicial durante el tiempo que se dis-fruta. El interés simple, 1, de un capital, e, colocado durante t años a un rédito anual, r (tanto poruno = %/100), viene dado por

1= Crt

y el capital final A (capital inicial e más el interés 1) viene dado por

A = e(l + rt)

Por ejemplo, si se solicita un préstamo de 80 000 pesetas para devolver en 2,5 años, el interésa pagar es l = 80 000(0,04 )2,5 = 8 000 pesetas, con lo que el dinero que se debe devolver al cabode esos 2,5 años es A = 88 000 pesetas.

INTERES COMPUESTO. Supongamos que el interés correspondiente a un capital al final delprimer periodo de tiempo (número dado de intervalos iguales) se suma al capital inicial y que estanueva cantidad se considera como capital inicial para el segundo periodo, y así sucesivamente du-rante un tiempo determinado. En este caso, el interés se convierte en capital, esto es, se acumulaa él. La suma en que se incrementa el capital inicial en un cierto número de intervalos de tiempoes el interés compuesto en ese tiempo. La suma del interés compuesto y del capital inicial recibeel nombre de capital compuesto. Los sucesivos intervalos de tiempo iguales durante los cuales elinterés se va acumulando el capital recibe el nombre de periodo de concersion y, normalmente, esigual a tres meses, seis meses o un año; el interés se acumula trimestral, semestral. o anualmente,al capital.

Los réditos se refieren, normalmente, a un periodo de un año aun cuando el interés compues-to se calcule para cada periodo de conversión. El rédito anual se llama rédito nominal.

Por ejemplo, si el rédito nominal es el 4 % y se acumulan los intereses cada tres meses, el periodode conversión es de 3 meses y el rédito es 1/4 (4 %) = 1 % para cada periodo de conversión.

221

CAPITULO 23

Intereses y anualidades

INTERES es el dinero pagado por un individuo u organización por el uso y disfrute de una cantidad de dinero llamada capital. El interés se paga, normalmente, al final de periodos de tiempo iguales, por ejemplo, al año, al semestre o al trimestre. La suma del capital inicial y su interés es el capi­tal final .

REDITO O TIPO DE INTERES. Es la relación entre el interés de un capital en una unidad de tiempo y dicho capital. La unidad de tiempo, mientras no se advierta otra cosa, se considera de un año. El rédito se expresa, generalmente, en tantos por ciento (%) o en tantos por uno igual al % dividido por 100.

Por ejemplo, si el capital es de 10000 pesetas y el interés es de 200 pesetas al año, el rédito es 200/ 10000 = 0,02 = 2 %. El tanto por ciento es 2 y el tanto por uno 0,02.

INTERES SIMPLE. Es el interés calculado sobre el capital inicial durante el tiempo que se dis­fruta. El interés simple, 1, de un capital, C, colocado durante / años a un rédito anual, r (tanto por uno = %/ 100), viene dado por

1= Crt

y el capital final A (capital inicial C más el interés 1) viene dado por

A = C(I + rt)

Por ejemplo, si se solicita un préstamo de 80 000 pesetas para devolver en 2,5 años, el interés a pagar es I = 80 000(0,04 )2,5 = 8 000 pesetas, con lo que el dinero que se debe devolver al cabo de esos 2,5 años es A = 88 000 pesetas.

INTERES COMPUESTO. Supongamos que el interés correspondiente a un capital al final del primer periodo de tiempo (número dado de intervalos iguales) se suma al capital inicial y que esta nueva cantidad se considera como capital inicial para el segundo periodo, y así sucesivamente du­rante un tiempo determinado. En este caso, el interés se convierte en capital, esto es, se acumula a él. La suma en que se incrementa el capital inicial en un cierto número de intervalos de tiempo es el interés compuesto en ese tiempo. La suma del interés compuesto y del capital inicial recibe el nombre de capital compuesto. Los sucesivos intervalos de tiempo iguales durante los cuales el interés se va acumulando el capital recibe el nombre de periodo de conrersión y, normalmente, es igual a tres meses, seis meses o un año; el interés se acumula trimestral, semestraL o anualmente, al capital.

Los réditos se refieren, normalmente, a un periodo de un año aun cuando el interés compues­to se calcule para cada periodo de conversión. El rédito anual se llama rédito nominal.

Por ejemplo, si el rédito nominal es el 4 % y se acumulan los intereses cada tres meses, el periodo de conversión es de 3 meses y el rédito es 1/4 (4 %) = 1 % para cada periodo de conversión.

221

http://carlos2524.jimdo.com/

Page 230: Algebra Superior Murray R Spiegel

222 INTERESES Y ANUALIDADES

Sea e el capital inicial. i el rédito por periodo de conversión (tanto por uno) y n el número deperiodos de conversión; el capital final A al cabo de los n periodos de conversión viene dado por

A = C(l + i)"

El interés compuesto es 1 = A - C.

Ejemplo. Un individuo invierte 100000 pesetas al 6 %, acumulándose los intereses cadaseis meses. Hallar el capital final A y el interés 1 al cabo de 2 años.

e = 100000 pts, i = 1/2 (6 %) = 3 % = 0,03, n = 4 (ya que cada periodo de conversiónes medio año y hay 4 periodos en 2 años).

Por tanto,

e = 100 000(\ + 0,03)4 = 100 000(1,03)4 = 1 125,50 pts1 = A - e = I 125,50 pts - 100000 pts = 125,50 pts

VALOR ACTUAL de una determinada cantidad al cabo de un periodo de tiempo dado es elcapital que, invertido a cierto rédito, durante dicho intervalo se transforma en la citada cantidad.Si la suma de dinero que se desea obtener, al cabo de n periodos de conversión, es A y el rédito porperiodo es i (tanto por uno), el valor actual e viene dado por

e = (1 : i)" = A(1 + ¡¡-n

Ejemplo. Hallar el valor actual de una deuda de 100000 pts que se debe saldar en 3 añosa interés compuesto del 6 %, acumulándose los intereses al capital cada trimestre.

A = lO 000 pts, i = 1/4(6 ~/(;)= 1.5 0,;. = 0.015.11 = 3(4) = 12. Por tanto,

100000 -12e = (\ + 0,015)12 = 100000(1.015) = 83639 pts,

Descontar de una cantidad de dinero A de una deuda que se debe saldar al cabo de n periodosde conversión a un rédito i por periodo es determinar el capital inicial e de A un tiempo igual an periodos antes de saldarse la deuda. La diferencia A - e recibe el nombre de descuento de A.

En el ejemplo anterior el descuento es A - e = 100000 - 83639 = 16361 pesetas.

ANUALIDADES. Es una sucesión de pagos iguales que se realizan periódicamente. El tiempo quetranscurre entre dos pagos sucesivos se denomina periodo de pago. El tiempo transcurrido entre elprincipio del primer periodo y el final del último recibe el nombre de término de la anualidad. Lacantidad de dinero pagada en un año es la renta anual. Supondremos que los pagos se efectúanal final de cada periodo de pago.

Por ejemplo, en una anualidad de 20000 pts que se paga semestralmente durante diez años,el intervalo o periodo de pago es de seis meses. el término es diez años y la renta anual de 40 000pesetas.

EL CAPITAL DE LA ANUALIDAD es la suma que se obtendría si se colocasen cada uno de los pagosa interés compuesto desde el momento en que se imponen hasta el final del tiempo especificado.

Una anualidad en la cual se pagase una peseta al final de cada uno de los 11 periodos igualesy cuyo rédito por periodo sea i, da lugar a un capital

VALORpagoseta

INTERES

1. Una pedebe p

Interi

2. Hallar

a) 1=b) 1=

3. Calcula

A

El120 ()()(

4. Hallar

A,

5. Una pe5 ~;.;.peCalcula

El

http://carlos2524.jimdo.com/

Page 231: Algebra Superior Murray R Spiegel

INTERESES Y ANU.'\LIDADES 223(1 + i)" - l

s"""ñ]. = ---i--

Si la cantidad que se paga periódicamente es R (pese.as). el capital de la anualidad es

VALOR ACTUAL DE UNA ANUALIDAD. Es la suma de los valores actuales de todos lospagos. Una anualidad en la cual los pagos periódicos al final de 11 periodos iguales es de una pe-seta y cuyo rédito por periodo es i, tiene un valor actual

I _ (1 + ¡¡-n(//il i =

El valor actual de una anualidad en la cual el pago periódico es R (pesetas). viene dado por

AIil i = Ra'n¡ i

PROBLEMAS RESUELTOS

INTERES SIMPLE

1. Una persona pide un préstamo de 40 000 pts para pagar en 2 años a interés simple del 3 ~;.;.Hallar la cantidad quedebe pagar al cabo de los 2 años.

Interés 1 = e r 1 = 40000(0.03)(2) = 2400 pts. Capital final A = capital inicial e + interés 1 = 42400 pts

2. Hallar el interés I y el capital final A en los casos siguientes: a) 60000 pts durante 8 meses (2/3 año) al 4 ~/~.h) 156 260 pts durante 3 años. 4 meses (10/3 años)

al 3,5 %.a) 1 = e r 1 = 60000(0,04)(2/3) = 1600 pts.h) 1 = e r 1 = 156260(0,035)(10/3) = 18230 pts.

A = e + 1 = 61 600 pts.A = e + 1 = 174 490 pts.

3. Calcular el capital que se debe imponer al 4~;'; para formar, al cabo de 5 años. una suma de 120 GOOpts.

A = e(1 + rr) o seaA 120000e = -- = .,-----:-::-::-:-=_:_

1 + rt + (0,04)(5)120000

1,2100 000 pts

El capital inicial de 100000 pts se denomina valor actual de las 120000 pts. Dicho de otra manera. devolver120 000 pts dentro de 5 años al 4 ~~ de interés simple equivale a pagar 100 000 pts hoy.

4. Hallar el rédito a que se deben invertir 80000 pts para tener, al cabo de 5 años. 100 000 pts.

A = e(1 + rl). o seaA-e

r=--=Ct

100 000 - 80 00080000(5) = 0,05, es decir, 5 ~;.;.

5. Una persona solicita un préstamo de 20000 pts. Para ello se dirige al banco y allí le dicen que el rédito es del5 :%;, pero que los intereses los tiene que pagar por anticipado y al cabo de un año ha de devolver las 20 000 pts.Calcular el rédito al que en realidad efectuó la operación.

El interés simple de 20000 pts en l año al 5 ~,~ es 1 = 20000(0,05)(1) = 1000 pts. Entonces recibe

INTERESES Y ANU.'\L1DADES 223

(1 + i)" - l s""ñl, = ---i--

Si la cantidad que se paga periódicamente es R (peselas). el capital de la anualidad es

VALOR ACTUAL DE UNA ANUALIDAD. Es la suma de los valores actuales de todos los pagos. Una anualidad en la cual los pagos periódicos al final de 11 periodos iguales es de una pe­seta y cuyo rédito por periodo es i. tiene un valor actual

(//il ; = I _ (1 + ¡¡-n

El valor actual de una anualidad en la cual el pago periódico es R (pesetas), viene dado por

PROBLEMAS RESUELTOS

INTERES SIMPLE

1. Una persona pide un préstamo de 40 000 pts para pagar en 2 años a interés simple del 3 ~;.; . Hallar la cantidad que debe pagar al cabo de los 2 años.

Interés 1 = e rl = 40000(0.03)(2) = 2400 pts. Capital final A = capital inicial e + interés 1 = 42400 pts

2. Hallar el interés 1 y el capital final A en los casos siguientes: a) 60000 pts durante 8 meses (2/3 año) al 4 ~/~. h) 156260 pts durante 3 años. 4 meses (10/3 años)

al 3,5 %. a) 1 = e rl = 60000(0,04)(2/3) = 1600 pts. A = e + 1 = 61 600 pts.

A = e + 1 = 174 490 pts. h) 1 = e r 1 = 156260(0,035)(10/3) = 18230 pts.

3. Calcular el capital que se debe imponer al 4 ~;'; para formar , al cabo de 5 años. una suma de 120 (j00 pts.

A = e(1 + rr) o sea A 120000 120000

e = 1 + rl = 1 + (0,04)(5) = - 1,-2- = 100000 pts

El capital inicial de 100000 pts se denomina valor actual de las 120000 pts. Dicho de otra manera. devolver 120000 pts denlro de 5 años al 4 ~~ de interés simple equivale a pagar 100 000 pts hoy.

4. Hallar el rédito a que se deben invertir 80000 pts para tener, al cabo de 5 años, 100 000 pts.

A = e(1 + rl), o sea A-e

r= - - = Ct

100 000 - 80 000 80000(5) = 0,05, es decir,

5. Una persona solicita un préstamo de 20000 pts. Para ello se dirige al banco y allí le dicen que el rédito es del 5 :%;, pero que los intereses los tiene que pagar por anticipado y al cabo de un año ha de devolver las 20 000 pts. Calcular el rédito al que en realidad efectuó la operación.

El interés simple de 20000 pts en 1 añ'O al 5 ~,~ es 1 = 20000(0,05)(1) = 1000 pts. Entonces recibe

http://carlos2524.jimdo.com/

Page 232: Algebra Superior Murray R Spiegel

224 INTERESES Y ANUALIDADES

20 000 - 1 000 = 19 000 pts. Como debe devolver 20 000 pts dentro de un año, e = 19 000 ptas, A = 20 000 pts,A - e 20000 - 19000

I = 1 año. Luego r ~ --- = = 0,0526, es decir, el rédito efectivo es del 5,26 %.Ct 19000(1)

6. Un comerciante pide un préstamo de 400 000 pts a pagar 20000 pts al final de cada periodo de 3 meses con el in-terés simple al 6 % del capital que adeuda en cada momento. Hallar la cantidad total que ha de devolver.

Como tiene que devolver 400 000 pts (sin tener en cuenta los intereses) a razón de 20 000 pts cada 3 meses,400000

tardará --- = 5 años, es decir, ha de efectuar 20 pagos.20000(4)

Interés que paga en la 1a entrega (por los 3 primeros meses) = 400 000(0.06)(./:;) = 6 000 pts.Interés que paga en la 2.a entrega = 380 000(0,06)(./:;) = 5700 pts.Interés que paga en la ]a entrega = 360 000(0.06)(./:;) = 5400 pts.

Interés que paga en la 20" entrega = 20000(0,06 )(./:;) = 300 pts.

El interés total es 6000 + 5 700 + 5400 + ... + 900 + 600 + 300, que representa la suma de los térmi-n

nos de una progresión aritmética cuyo valor es S = 2(a + 1), siendo a = primer término, 1 = último término,

n = número de términos.

20Por tanto. S = 2(6000 + 300) = 6300 pts, con lo que la cantidad total que ha de devolver es de

463000 pts.

7. Una persona necesita inmediatamente 80000 pts para devolverlas al cabo de I año. El banco le cobra. por anti-cipado, los intereses de la cantidad que le presta a un 6 % de interés simple. Deducir el dinero que debe pedir pres-tado al banco.

Sean x pesetas = cantidad que pide prestada.Interés de x pesetas al- 6 % durante 1 año = x(O,06)(1) = 0,06x.El banco le entrega x - 0,06x = 0,94x = 80000, de donde x = 80000/0,94 = 85 106 pts.

8. Una deuda de D pesetas se salda efectuando k pagos al año en cada uno de los cuales se entregan p pesetas másel interés simple, a un rédito r, del capital que adeuda en cada momento. Deducir la fórmula de la cantidad totalde dinero que debe pagar.

Como efectúa k pagos al año.

interés que paga en la 1" entrega = D(r)(l/k) = Drlkinterés que paga en la 2." entrega = (D - p)(r)(l/k) = (D - p)r/kinterés que paga en la 3." entrega = (D - 2p)(r)(l/k) = (D - 2p)r/k

interés que paga en la n.a (última) entrega = [D - (11 - I)p](r)(l/k) = [D - (11 - I)p]r/k.

Número de pagos x cantidad pagada en cada uno = total prestado.

DPor tanto. "1' = D, o sea n = - (se supone que este cociente es entero).

l'

Interés total = Dr + (D - p)r + (D - 2p)r +k k k

gresión aritmética cuyo valor es

[D - (11 - I)p]r+ . suma de los términos de una pro-k

Dya que 11 = -.

p11 IIDr [D-(n-I)p]r Dr(D+I')2(a+I)="2{T+ k }= 2pk'

Dr(D +1')La cantidad total a pagar. en pesetas. es = D '1- --'----'-'-

2pk

El Problema 6 es un caso particular en el que D = 400000. k = 4. l' = 20000 y r = 0.06.

INTERES

9. Hallalándo

Méto.

Méto.

11. Hallapital

12. Calculeal n

(

capits

nominen el,tralrm

13. Dedu

(

= e(

I

224 INTERESES Y ANUALIDADES

20000- 1000= 19000 pts. Como debe devolver 20 000 pts dentro de un año, e = 19000 ptas, A = 20000 pts, A - e 20000 - 19000

I = 1 año. Luego r == = 0.0526, es decir. el rédito erectivo es del 5.26 %. el 19000(1)

6. Un comerciante pide un préstamo de 400 000 pts a pagar 20000 pts al final de cada periodo de 3 meses con el in­terés simple al 6 % del capital que adeuda en cada momento. Hallar la cantidad total que ha de devolver.

Como tiene que devolver 400 000 pts (sin tener en cuenta los intereses) a razón de 20 000 pts cada 3 meses, 400000 .

tardará --- = 5 años, es decir. ha de erectuar 20 pagos. 20000(4)

Interés que paga en la l. a entrega (por los 3 primeros meses) = 400 000(0.06 H,Y. ) = 6 000 pts. Interés que paga en la 2.a entrega = 380 OOO(0,06H,Y.) = 5700 pts. Interés que paga en la 3' entrega = 360 000(0.06H,Y.) = 5400 pts.

Interés que paga en la 20a entrega = 20000(0,06H,Y.) = 300 pts.

El interés total es 6000 + 5 700 + 5400 + ... + 900 + 600 + 300, que representa la suma de los térmi­n

nos de una progresión aritmética cuyo valor es S = 2(a + 1). siendo a = primer término. 1 = últimu término.

n = número de términos.

20 Por tanto. S = 2(6000 + 3(0) = 6300 pts. con lo que la cantidad total que ha de devolver es de

463000 pts.

7. Una persona necesita inmediatamente 80000 pts para devolverlas al cabo de 1 año . El banco le cobra. por anti­cipado, los intereses de la cantidad que le presta a un 6 % de interés simple. Deducir el dinero que debe pedir pres­tado al banco.

Sean x pesetas = cantidad que pide prestada. Interés de x pesetas al - 6 % durante 1 año = .\"(0.06HI) = 0,06x. El banco le entrega x - 0.06x = 0,94x = 80000. de donde x = 80000/0.94 = 85 106 pts.

8. Una deuda de D pesetas se salda erectuando k pagos al año en cada uno de los cuales se entregan p pesetas más el interés simple. a un rédito r, del capital que adeuda en cada momento. Deducir la rórmu la de la cantidad total de dinero que debe pagar.

Como erectúa k pagos al año.

interés que paga en la 1." entrega = D(r)( l/k) = Drj k interés que paga en la 2." entrega = (D - p)(r)(l/k) = (D - p)r/k interés que paga en la 3." entrega = (D - 2p)(rHI /k) = (D - 2p)r/k

interés que paga en la na (última) entrega = [D - (11 - l)pJ(r)(l /k) = [D - (11 - l)pJr/k .

Número de pagos x cant idad pagada en cada uno = total prestado.

D Por tanto, IIp = D. o sea n = - (se supone que este cociente es entero).

p

. Dr (D - p)r (D - 2p)r [D - (11 - l)pJr Interes total = k + --k- - + k + ... + k . suma de los términos de una pro-

gresión aritmética cuyo valor es

11 IIDr [D - (n-l)pJr Dr(D+I') D - (a+I)=-{-+ }= . ya quclI= - . 2 2 k k 2pk p

. Dr(D + p) La cantidad tota l a pagar. en pesetas. es = D + .

2pk

El Probletna 6 es un caso particular en el que D = 400000. k = 4. " = 20000 Y r = 0.06.

http://carlos2524.jimdo.com/

Page 233: Algebra Superior Murray R Spiegel

INTERESES Y ANUALIDADES 225

r

INTERES COMPUESTO

9. Hallar el capital que se formará al cabo de 2 años imponiendo 50000 pts a interés compuesto del 2 % acumu-lándose los intereses al capital cada seis meses.

Método l. Sin aplicar la fórmula.

Interés al final del 1.0 medio año = 50 000(0,02)(Yz) 500 ptsInterés al final del 2.° medio año = 50 500(0,02)(Yz) = 505 ptsInterés al final del 3.° medio año = 51 005(0,02)(Yz) = 510 ptsInterés al final del 4.° medio año = 51 515(0,02)(Yz) = 515 pts

Interés total = 2 030 pts Capital final = 52030 pts

1-

Método 2. Aplicando la fórmula.

C = 50000 pts, i = rédito por periodo = 0,02/2 = 0,01, n = número de periodos = 4.

A = C(I + i)" = 50 OOO( 1,01)4 = 50 OOO( 1,0406) = 52 030 pts.

Nota. El cálculo de la potencia (1,01)4 se puede hacer por la fórmula del binornio, por logaritmos o pormedio de tablas.

10. Hallar el capital final y el interés que resultan si se imponen 280000 pts durante 8 años a interés compuesto del5 'X. acumulándose los intereses al capital cada tres meses.

1-

s-

A = C(I + i)" = 280000(1 + 0,05/4)32 = 280000(1,0125)32 = 280000(1,4881) = 416668 pts.

Interés = A - C = 416668 - 280000 = 136668 pts.

11. Hallar la cantidad de dinero que se debe imponer a interés compuesto al 6 %, acumulándose los intereses al ca-pital cada tres meses, para obtener 200000 pts dentro de 10 años. ¿Cuál es el descuento?

Tendremos que calcular el capital C que al cabo de 10 años se transforme en A = 200 000 pts.

A C(I + i)", o seaA 200000

C = --- = --- = 110252 pts.(1 + i)" (1,015)40

ás

alEl descuento es 200000 - 110252 = 89748 pts.

12. Calcular el rédito en interés compuesto, en el que se acumulan los Intereses al capital anualmente, que equiva-le al rédito en interés compuesto del6 ~%;en el que los intereses, sin embargo, se acumulan al capital cada seis meses.

Cantidad formada a partir de un capital C en 1 año a un rédito r = C(I + r).

Cantidad formada a partir de un capital C en 1 año al 6 % acumulándose los intereses cada seis meses alcapital = C(I + 0,03)2.

Igualando ambas cantidades, C(I + r) = C(I,03)2, I + r = (1,03)2, r = 0,0609, o sea 6,09 ~~.

El rédito por año cuando los intereses se acumulan un número de veces por año recibe el nombre de réditonominal. El rédito r cuando los intereses se acumulan una vez al año y que dé lugar a los mismos intereses queen el caso anterior, recibe el nombre de rédito efectivo, En este ejemplo, el 6 ~~ acumulándose los intereses semes-tralmente es el rédito nominal y el 6,09 o,~ es el rédito efectivo.

0-13. Deducir la fórmula que expresa el valor del rédito efectivo en función del rédito nominal.

Sea r = rédito efectivo.i = rédito anual. cuando los intereses se acumulan k veces al año, es decir, el rédito nominal.

Cantidad formada a partir de un capital C en I año a un rédito r = C( I + r).

Cantidad formada a partir de un capital C en I año a un rédito i, acumulándose los intereses k veces al añoC(I + i!k)'.

Igualando ambas cantidades, C(I + r) = e(l + ik)'.

Por tanto, ,. = (1 + ilk)' - l.

INTERESES Y ANUALIDADES 225

INTERES COMPUESTO

9. Hallar el capital que se formará al cabo de 2 años imponiendo 50000 pts a interés compuesto del 2 % acumu­lándose los intereses al capital cada seis meses.

Método l. Sin aplicar la fórmula.

Interés al final del 10 medio año = 50 000(0,02)(y,:) 500 pts Interés al final del 2.° medio año = 50 500(0,02)(y,:) = 505 pts Interés al final del 3.° medio año = 51 005(0,02)(y,:l = 510 pts Interés al final del 4.° medio año = 51 515(0,02)(y,:) = 515 pts

Interés total = 2 030 pts

Método 2. Aplicando la fórmula. Capital final = 52030 pts

C = 50000 pts, i = rédito por periodo = 0,02/2 = 0,01 , n = número de periodos = 4.

A = C(I + i)" = 50 OOO( 1,01)4 = 50 OOO( 1 ,0406) = 52 030 pts.

Nota. El cálculo de la potencia (1 ,01)4 se puede hacer por la fórmula del binomio, por logaritmos o por medio de tablas.

10, Hallar el capital final y el interés que resultan si se imponen 280000 pts durante 8 años a interés compuesto del 5 X. acumulándose los intereses al capital cada tres meses.

A = C(1 + il" = 280000(1 + 0,05/4)32 = 280000(1,0125)32 = 280000(1,4881) = 416668 pts.

Interés = A - C = 416668 - 280000 = 136668 pts.

11. Hallar la cantidad de dinero que se debe imponer a interés compuesto al 6 %, acumulándose los intereses al ca­pital cada tres meses, para obtener 200000 pts dentro de 10 años. ¿Cuál es el descuento?

Tendremos que calcular el capital C que al cabo de 10 años se transforme en A = 200 000 pts.

A C(I + il" , o sea A 200000

C = --- = --- = 110252 pts. (1 + i)" (1,015)40

El descuento es 200000 - 110252 = 89748 pts.

12, Calcular el rédito en interés compuesto, en el que se acumulan los Intereses al capital anualmente , que equiva­le al rédito en interés compuesto del6 ~%; en el que los intereses, sin embargo, se acumulan al capital cada seis meses.

Cantidad formada a partir de un capital C en 1 año a un rédito r = C(1 + r).

Cantidad formada a partir de un capital C en 1 año al 6 % acumulándose los intereses cada seis meses al capital = C(1 + 0,03)2.

Igualando ambas cantidades, C(1 + r) = C(I,03l2, 1 + r = (1,03)2, r = 0,0609, o sea 6,09 ~~ .

El rédito por año cuando los intereses se acumulan un número de veces por año recibe el nombre de rédito nominal. El rédito r cuando los intereses se acumulan una vez al año y que dé lugar a los mismos intereses que en el caso anterior, recibe el nombre de rédito electil'O. En este ejemplo, el 6 ~,~ acumulándose los intereses semes­tralmente es el rédito nominal y el 6,09 '\ es el rédito efectivo.

)3, Deducir la fórmula que expresa el valor del rédito efectivo en función del rédito nominal.

Sea r = rédito efectivo. i = rédito anual, cuando los intereses se acumulan k veces al año, es decir, el rédito nominal.

Cantidad formada a partir de un capital C en I año a un rédito r = C( I + rl.

Cantidad formada a partir de un capital C en I año a un rédito i, acumulúndose los intereses k veces al año C (l + i/kl'.

Igualando ambas cantidades, C(I + r) = C(I + ik)'.

Por tanto, r = (1 + i/k)' - l.

http://carlos2524.jimdo.com/

Page 234: Algebra Superior Murray R Spiegel

226 INTERESES Y ANUALIDADES

ANUALIDADES

14. Hallar el capital que se forma al cabo de 5 años imponiendo al final de cada año una anualidad de 10000

pesetas al 3 ~:'~.

Método l. Sin aplicar la fórmula.

Cantidad formada a partir de la La anualidad al cabo de 4 años = 10000(1.03)4 = II 255 ptsCantidad formada a partir de la 2.a anualidad al cabo de 3 años = 10 OOO(l ,03)' = 10 927 ptsCantidad formada a partir de la 3.a anualidad al cabo de 2 años = 10 OOO(1.03)2 = 10 609 ptsCantidad formada a partir de la 4a anualidad al cabo de I año = 10000(1.03)' = 10300 ptsCantidad formada a partir de la 5.a anualidad al cabo de O años = 10000 = 10000 pts

Capital final = 53091 pts

Método 2. Aplicando la fórmula

,(I+if-I (1+0,03)'-1S"ñ]i= S'5]0.03 = R\ i } = lO 000: 0,03 } = 10000(5.3091) = 53091 pts (tablas).

15. Hallar el capital actual de una anualidad de lO 000 pts por año pagadas al final de cada año durante 5 años a in-terés compuesto del 3 %.

Método l. Sin aplicar la fórmula

Valor actual del 1.0 pago = lO 000/(1.03)' 10000(1.03)'1 = 9709 ptsValor actual del 2.0 pago = 10000/(1,03)2 = 10000(1,03)'2 = 9426 ptsValor actual del 3.° pago = 10000/(1,03)3 = 10000(1.03)" = 9 151 ptsValor actual del 4.° pago = 10 000/(1,03)4 = lO 000(1.03)'4 = 8885 ptsValor actual del 5.° pago = 10000/(I,OW = 10000(1.03)" = 8626 pts

Valor actual total = 45 797 pts

Método 2. Aplicando la fórmula

1 - (1 + i)"n 1 - (1 + 0,03)'5A"ñ]i= A'5]0.03 = R{ i } = 10 000 { 0.03 } 10000(4.5797) = 45797 pts

16. Hallar el capital final y el valor actual de una anualidad de 12000 pts pagadas al final de cada 3 meses durante12 años, al 6/~. acumulándose los intereses trimestralmente.

R = 12000 pts, n = 4(12) = 48. i = 1/4(6 ~%:)= 1.5 % = 0.015

1 fi 000 {(1,015¡48 - 1,

Capita nal S"1o.o" = 12 S"10.015 = 12000 0.015 J12000(69.5652) = 834782 pts

1 - (1015)'48Valor actual AnnOOl5 = 12 ooOtlnno 01' = \2 000 { . -} = 12000(34.0426) = 408511 pts~o, . ~u,. . 0,015 -

17. Deducir la cantidad que ha de imponer una persona al final de cada año para constituir un capital de 2000000pesetas en 20 años a un interés del 3 Y".

Sea R (pts) = cantidad que se impone al final de cada año. Los pagos anuales de R pesetas constituyen unaanualidad, en la cual S-nli = 2000000 pts, n = 20. i = 3 % = 0.03.

(1 + ir - 1S"ñ]i= R{----.--} = RS"ñ]i'

I

R = S"ñ]i= 2000000 = 2000 000 = 74431 ptsS"ñ]i Sl1ll0.03 26,8704

de donde

IS. H¡mII ~

19. Urañ,

en

80

80

20. Seeado

COI

21. Hapri

do

R(l

el

22. Appa¡

http://carlos2524.jimdo.com/

Page 235: Algebra Superior Murray R Spiegel

0000

a in-

urante

ts

000

en una

INTERESES Y ANUALIDADES 227

18_ Hallar la cantidad que ha de imponer una empresa cada 6 meses a un interés del 6 /~, acumulándose semestral-mente los intereses al capital, con objeto de poder disponer al final de un periodo de 15 años de un capital deI 200000 pts para renovar parte de su equipo.

Sea R (pts) = cantidad depositada al final de cada periodo de 6 meses. El capital constituido al cabo de 15años es el correspondiente a una anualidad de 30 imposiciones.

R = Sii]; = 1200000Sii]; = 1200 000 pts, n = 30, i = )-i(0,04) = 0,02, Y

sii]; S:nll 0.02

1200 00040,5681 = 29 580 pts

19. Una deuda de 800000 pts se debe amortizar en 6 años mediante imposiciones iguales realizadas al final de cadaaño a un interés compuesto del 5 %. Hallar la cantidad que se debe imponer anualmente y la cantidad pagadaen concepto de intereses.

Sea R (pts) = cantidad impuesta al año. Los pagos constituyen una anualidad cuyo valor actual es Añl; =800000 pts, n = 6, i = 5 % = 0,05.

,1-(1+¡¡-",Aii]; = R\ i i = R aii];, es decir, R = Aii]; = 800000 = 800000 = 157614 pts

aii]; a~o,os 5,0757

La cantidad total pagada es de 800000 pts más 6(157614) = 945 684. Los intereses importan 945 684 -800000 = 145684 pts.

20, Se compra una finca pagando al contado 100 000 pts y 30000 pts al final de cada trimestre durante 10 años.Calcular el precio a que se deberia vender la casa al contado, suponiendo que en caso contrario el dinero paga-do periódicamente se habia colocado a un interés compuesto del 4 % acumulándose los intereses por trimestres.

El valor actual de una anualidad da 30000 pts al final de cada trimestre durante 10 años al 4 % de interéscompuesto acumulable trimestralmente es

A41J]o.ol = 30000 a41J]o.ol = 30000(32,8347) = 985041 pts

Por tanto, el precio pedido será 100000 + 985041 = 1 085041 pts.

21. Hallar el capital final y el valor actual de una anualidad en la que se efectúan n pagos de R pesetas cada uno alprincipio de cada periodo de pago a un rédito i por periodo.

El capital final y el valor actual de una anualidad en la que se efectúan n pagos al principio de cada perio-do son, respectivamente, iguales al capital final y valor actual de una anualidad en la que se realizan n pagos de

. .. . (1 + i)" - 1\,R( 1 + i) pesetas al final de cada periodo. Por tanto, el capital de la anuahdad es = R(I + I){ }, y

el valor actual = R(I + i){ 1 - (1 + ¡¡-"}.I

22. Aplicar las fórmulas del Problema 21 para hallar el capital final y el valor actual de una anualidad en la que sepagan 20 000 pts al principio de cada semestre durante 20 años acumulándose los intereses cada 6 meses.

R=20000, n=2(20)=40, i=)-i(2%)=I%=0,01.

Capital final = 20000(1 + 0,01 )S41J]O.Ol = 20200(48,8864) = 987505 pts.

Valor actual = 20000(1 + 0,01 )a41J]o.ol =: 20200(32,8347) = 663261 pts.

INTERESES Y ANUALIDADES 227

18. Hallar la cantidad que ha de imponer una empresa cada 6 meses a un interés del 6 /~ , acumulándose semestral ­mente los intereses al capital , con objeto de poder disponer al final de un periodo de 15 años de un capital de l 200000 pts para renovar parte de su equipo .

Sea R (pts) = cantidad depositada al final de cada periodo de 6 meses. El capital constituido al cabo de 15 años es el correspondiente a una anualidad de 30 imposiciones.

R = S¡,,; = l 200 000

S¡,,; = 1200 000 pts, n = 30, i = ji(O,04) = 0,02, Y S:nll 0 .02

1200 000 40,5681 = 29 580 pts

19. Una deuda de 800000 pts se debe amortizar en 6 años mediante imposiciones iguales realizadas al final de cada año a un interés compuesto del 5 %. Hallar la cantidad que se debe imponer anualmente y la cantidad pagada en concepto de intereses.

Sea R (pts) = cantidad impuesta al año. Los pagos constituyen una anualidad cuyo valor actual es A¡;¡; =

800000 pts, n = 6, i = 5 % = 0,05 .

Añ!; 800000 800000 R = - = - -- = --- = 157614 pts

añ! ; a~O, 05 5,0757

La cantidad total pagada es de 800000 pts más 6(157614) = 945 684. Los intereses importan 945 684 -800000 = 145684 pts.

20, Se compra una finca pagando al contado 100 000 pts y 30000 pts al final de cada trimestre durante 10 años. Calcular el precio a que se debería vender la casa al contado, suponiendo que en caso contrario el dinero paga­do periódicamente se había colocado a un interés compuesto del 4 % acumulándose los intereses por trimestres.

El valor actual de una anualidad da 30 000 pts al final de cada trimestre durante 10 años al 4 % de interés compuesto acumulable trimestralmente es

A¡¡¡¡¡o,OI = 30000 a¡¡¡¡¡O,OI = 30000(32,8347) = 985041 pts

Por tanto, el precio pedido será 100000 + 985041 = I 085041 pts.

21. Hallar el capital final y el valor actual de una anualidad en la que se efectúan n pagos de R pesetas cada uno al principio de cada periodo de pago a un rédito i por periodo.

El capital final y el valor actual de una anualidad en la que se efectúan n pagos al principio de cada perio­do son, respectivamente, iguales al capital final y valor actual de una anualidad en la que se realizan n pagos de

. .. . (1 + i)" - I \, R(I + i) pesetas al final de cada penodo. Por tanto, el capital de la anuahdad es = R(I + ¡)( 1-. y

el valor actua l = R(I + i){ 1 - (1 + ¡¡-"}. ¡

22. Apl icar las fórmulas del Problema 21 para hallar el capital final y el valor actual de una anualidad en la que se pagan 20 000 pts al principio de cada semestre durante 20 años acumulándose los intereses cada 6 meses.

R = 20000, n = 2(20) = 40, i = ji(2 %) = 1 % = 0,01.

Capital final = 20000(1 + 0,01 )S¡¡¡¡¡O,OI = 20200(48,8864) = 987505 pts.

Valor actual = 20000(1 + 0,01 )a¡¡¡¡¡o.ol =:' 20200(32,8347) = 663261 pts.

http://carlos2524.jimdo.com/

Page 236: Algebra Superior Murray R Spiegel

228 INTERESES Y ANUALIDADES

PROBLEMAS PROPUESTOS23. Hallar el interés simple y el capital en los casos siguientes: a) 120000 pts durante 4 meses al 3 %, b) 180000 pts

durante 16 meses al 2,5 %, e) 61 230 pts durante 2 años y 8 meses al 4 %.24. Una persona solicita un préstamo de D pesetas. Para ello, se dirige al banco y allí le dicen que el rédito es r

(tanto por uno), pero que los intereses los tiene que pagar por adelantado y que al cabo de un año ha de devol-lOOr

ver las D pesetas. Demostrar que el rédito efectivo es --- (%) y hallar su valor en el caso de que r = 6.100 - r

·25. Una deuda de 300000 pts se ha de saldar efectuando dos pagos al año en cada uno de los cuales se entregan50000 pts más el interés simple, con un rédito del 5,5 % del capital que se adeuda en cada momento. Hallar lacantidad de dinero que hay que devolver.

26. Una persona necesita D pesetas inmediatamente, para devolverlas al cabo de 1 año. El banco cobra, por antici-pado, los intereses de la cantidad que le presta a un rédito r a interés simple. Demostrar que la cantidad que

100Ddebe pedir prestada al banco es 100 _ r pesetas.

27. Hallar el interés compuesto y el capital final en los casos siguientes: a) 250 000 pts durante 3 años al4 %' acumu-lándose los intereses al capital trimestralmente, b) 480 000 pts durante 6 años al 5 %, acumulándose los interesesanualmente, e) 735000 pts durante 10 años al 3 %, acumulándose los intereses semestralmente.

28. Hallar el dinero que se debe depositar en un banco para constituir un capital de 200 000 pts al cabo de 6 años,sabiendo que dicho banco paga el6 % de interés compuesto acumulándose los intereses cada a) tres meses, h) seismeses.

29. Hallar el valor actual de una deuda de 500000 pts que se debe saldar en 8 años si el rédito es del 4 % acumulán-dose los intereses cada, a) tres meses, b) seis meses, e) doce meses. Hallar los descuentos correspondientes.

30. Hallar el valor actual de a) una deuda de 150000 pts a saldar en 3 años si el rédito es del 6 % acumulándose losintereses cada tres meses, b) 545 020 pts a saldar en 10 años con un rédito del 5 % acumulándose los intereses cadaseis meses.

31. Hallar el rédito efectivo que corresponde a un rédito nominal del 4 % a) acumulándose los intereses cada seis me-ses, b) acumulándose los intereses cada tres meses.

32. Hallar el capital final y el valor actual de las anualidades siguientes:a) 40000 pts al final de cada 6 meses durante 8 años a un rédito del4 %, acumulándose los intereses cada seis meses.b) 100000 pts al final de cada año durante 20 años al 5 %, acumulándose los intereses cada año.e) 25000 pts al final de cada 3 meses durante 12 años al 6 %, acumulándose los intereses cada tres meses.

33. Una persona desea imponer cierta cantidad de dinero al final de cada trimestre para constituir al cabo de 10 añosun capital de 500 000 pts. Hallar la cantidad que debe pagar periódicamente a un interés compuesto del 4 % acumu-lándose los intereses cada tres meses.

34. Una persona debe 100 000 pts y desea saldar esta deuda en 10 años mediante entregas de cantidades de dineroiguales, efectuadas al final de cada semestre con un rédito del 6 %, acumulándose los intereses cada seis meses.Hallar, a) los pagos semestrales, b) la cantidad de dinero total que paga al cabo de los 10 años.

35. Se adquiere una máquina pagando al contado 150000 pts y 50000 pts al final de cada semestre durante 5 años.Hallar el precio a que se debería vender dicha máquina, al contado, suponiendo que, en caso contrario, el dineropagado periódicamente se habría colocado a un interés compuesto del 5 %, acumulándose los intereses cada seismeses.

SOLUCIONES DE LOS PROBLEMAS PROPUESTOS

23. a) 1200 pts, 121200 pts b) 6000 pts, 186000 pts27. a) 31 700 pts, 281 700 pts h) 163248 pts. 643248 pts28. a) 139908 pts, b) 140276 pts29. a) 363650 pts, 136350 pts b) 364 225 pts, 135775 pts30. a) 125 459 pts h) 332 609 pts31, a) 4,04 % b) 4,06 %32, a) 745 572 pts, 543 108 pts33. 10 228 pts34. a) 67216 pts35. 587605' pts

e) 6531 pts 67761 pts 25, 3288,75 ptse) 254 972 pts, 989 972 pts

e) 365 345 pis. 134655 pts

el 1 739 130 pts, 851 065 ptsh) 3 306 600 pts, 1 246220 pts

h) 1 344 320 pts.

TEOREMcuandambo:

p

3' 5 '

UNA VAun or

phe, cc

UNA PEIellos I

pCualq

(

los er

UNA eadiendi

pab, be

(

econsti

NOTACICescritc

251\

EL SIMBpartic

/les el

VARIACI

228 INTERESES Y ANUALIDADES

PROBLEMAS PROPUESTOS 23. Hallar el interés simple y el capital en los casos siguientes: a) 120000 pts durante 4 meses al 3 %, b) 180000 pts

durante 16 meses al 2,5 %, e) 61 230 pts durante 2 años y 8 meses al 4 %.

24. Una persona solicita un préstamo de D pesetas. Para ello, se dirige al banco y allí le dicen que el rédito es r

(tanto por uno), pero que los intereses los tiene que pagar por adelantado y que al cabo de un año ha de devol­lOOr

ver las D pesetas. Demostrar que el rédito efectivo es - - - (%) y hallar su valor en el caso de que r = 6. 100 - r

·25. Una deuda de 300000 pts se ha de saldar efectuando dos pagos al año en cada uno de los cuales se entregan 50000 pts más el interés simple, con un rédito del 5,5 % del capital que se adeuda en cada momento. Hallar la cantidad de dinero que hay que devolver.

26. Una persona necesita D pesetas inmediatamente, para devolverlas al cabo de I año. El banco cobra, por antici­pado, los intereses de la cantidad que le presta a un rédito r a interés simple. Demostrar que la cantidad que

100D debe pedir prestada al banco es 100 _ r pesetas.

27. Hallar el interés compuesto y el capital final en los casos siguientes: a) 250 000 pts durante 3 años al4 %' acumu­lándose los intereses al capital trimestralmente, b) 480 000 pts durante 6 años al 5 %, acumulándose los intereses anualmente, e) 735000 pts durante 10 años al 3 %, acumulándose los intereses semestralmente.

28. Hallar el dinero que se debe depositar en un banco para constituir un capital de 200000 pts al cabo de 6 años, sabiendo que dicho banco paga el 6 % de interés compuesto acumulándose los intereses cada a) tres meses, b) seis meses.

29. Hallar el valor actual de una deuda de 500000 pts que se debe saldar en 8 años si el rédito es del 4 % acumulán­dose los intereses cada, a) tres meses, b) seis meses, e) doce meses. Hallar los descuentos correspondientes.

30. Hallar el valor actual de a) una deuda de 150000 pts a saldar en 3 años si el rédito es del 6 % acumulándose los intereses cada tres meses, b) 545 020 pts a saldar en 10 años con un rédito del 5 % acumulándose los intereses cada seis meses.

31. Hallar el rédito efectivo que corresponde a un rédito nominal del 4 % a) acumulándose los intereses cada seis me­ses, b) acumulándose los intereses cada tres meses.

32. Hallar el capital final y el valor actual de las anualidades siguientes: a) 40000 pts al final de cada 6 meses durante 8 años a un rédito del4 %, acumulándose los intereses cada seis meses. b) 100000 pts al final de cada año durante 20 años al 5 %, acumulándose los intereses cada año . e) 25000 pts al final de cada 3 meses durante 12 años al 6 %, acumulándose los intereses cada tres meses.

33. Una persona desea imponer cierta cantidad de dinero al final de cada trimestre para constituir al cabo de 10 años un capital de 500 000 pts. Hallar la cantidad que debe pagar periódicamente a un interés compuesto del4 % acumu­lándose los intereses cada tres meses.

34. Una persona debe 100 000 pts y desea saldar esta deuda en 10 años mediante entregas de cantidades de dinero iguales, efectuadas al final de cada semestre con un rédito del 6 %, acumulándose los intereses cada seis meses. Hallar, a) los pagos semestrales, b) la cantidad de dinero total que paga al cabo de los 10 años.

35. Se adquiere una máquina pagando al contado 150000 pts y 50000 pts al final de cada semestre durante 5 años. Hallar el precio a que se debería vender dicha máquina, al contado, suponiendo que, en caso contrario, el dinero pagado periódicamente se habría colocado a un interés compuesto del 5 %, acumulándose los intereses cada seis meses.

SOLUCIONES DE LOS PROBLEMAS PROPUESTOS

23. a) 1200 pts, 121200 pts b) 6000 pts, 186000 pts e) 6531 pIs 67761 pIS 25, 3288,75 pIS 27. a) 31700 pts, 281700 pts h) 163248 pIS, 643248 pts e) 254 972 pIS, 989 972 pIS 28. a) 139908 pts, b) 140276 pts 29. a) 363650 pts, 136350 pts b) 364225 pIS, 135775 pts e) 365 345 pIS. 134655 pIS 30. a) 125 459 pts h) 332 609 pIS 31, a) 4,04 % b) 4,06 % 32. a) 745 572 pIS, 543 108 pIS h) 3 306 600 pts, 1 246220 pIS e ) 1 739 130 pIS, 85 1 065 pIS 33. 10 228 pIS 34. a) 67216 pIS h) 1 344 320 pIS. 35. 587605 ' pIS

http://carlos2524.jimdo.com/

Page 237: Algebra Superior Murray R Spiegel

229

CAPITULO 24

ts CombinatoriaTEOREMA FUNDAMENTAL. Si un suceso puede tener lugar de m maneras distintas y

cuando ocurre una de ellas se puede realizar otro suceso independiente de m formas diferentes,ambos sucesos, sucesivamente, pueden tener lugar de mn maneras distintas.

Por ejemplo, si existen 3 candidatos para la presidencia y 5 para la vicepresidencia, existen3 . 5 = 15 parejas distintas de presidente y vicepresidente.

UNA VARIACION de un cierto número de entes es una disposición de una parte de ellos enun orden determinado.

Por ejemplo, las variaciones de las tres letras, a, b y e, tomadas de dos en dos, son ab, ac, ha,be, ea y cb. Cualquiera de estas disposiciones es una variación.

u-es

s,IS

UNA PERMUTACION de un cierto número de entes es una disposición en la que entran todosellos en un orden determinado.

Por ejemplo, las permutaciones de las tres letras a, b y e son abc, acb, bca, bac, cba y caboCualquiera de estas disposiciones es una permutación.

Obsérvese que las permutaciones son un caso particular de las variaciones en que entran todoslos entes en lugar de una parte de ellos.n-

s.

UNA COMBINACION de un número de entes es una disposición de una parte de ellos prescin-diendo del orden, a diferencia de una variación.

Por ejemplo, las combinaciones de las tres letras, a, b y e, tomadas de dos en dos, sonab, be y ae.

Cualquiera de estas disposiciones es una combinación.

Obsérvese que ab y ba son una misma combinación (se prescinde del orden), mientras queconstituyen dos variaciones distintas (interesa el orden), de las letras a y b.

osda

e-

osu-

NOT ACION FACTORIAL. Las identidades siguientes expresan el significado de factorial de nescrito Il!

21 1 . 2 = 2, 3! = 1 . 2 . 3 = 6. 41 = 1 . 2 . 3 . 4 = 24

51= 1'2'3'4'5= 120. 11!= 1'2'3 ... 11, (r-I)I= 1·2·3 ... (r-l)

Nora. 01 = 1 por definición.

EL SIMBOLO "V, representa el número de variaciones de n·elementos tomados de r en r. Como casoparticular, "V"' es el número de permutaciones de n elementos y se escribe, normalmente. P".

Así, pues. 8 V 3 representa el número de variaciones de 8 elementos tomados de 3 en 3. y Pses el número de permutaciones de 5 elementos.

Nora. Los símbolos V(n, r), V"., Y V;' tienen el mismo significado que" V,.

VARIACIONES DE n ELEMENTOS TOMADOS DE r EN r111

"V, = 11(11 - 1)(11 - 2) ... (11 - r + 1) = 1(11 - r).

Para r = 11, "V, = "V" = P" = 11(11 - 1)(/1 - 2) ... 1 = 111

CAPITULO 24

Combinatoria

TEOREMA FUNDAMENTAL. Si un suceso puede tener lugar de m maneras distintas y cuando ocurre una de ellas se puede realizar otro suceso independiente de m formas diferentes, ambos sucesos, sucesivamente, pueden tener lugar de mn maneras distintas.

Por ejemplo, si existen 3 candidatos para la presidencia y 5 para la vicepresidencia, existen 3 . 5 = 15 parejas distintas de presidente y vicepresidente.

UNA VARIACION de un cierto número de entes es una disposición de una parte de ellos en un orden determinado.

Por ejemplo, las variaciones de las tres letras, a. b y e, tomadas de dos en dos, son ah. ae, ha, be, ca y eb. Cualquiera de estas disposiciones es una variación .

UNA PERMUT ACION de un cierto número de entes es una disposición en la que entran todos ellos en un orden determinado.

Por ejemplo , las permutaciones de las tres letras a, b y e son abe, aeb, hea, bae, eba y cah. Cualquiera de estas disposiciones es una permutación.

Obsérvese que las permutaciones son un caso particular de las variaciones en que entran todos los entes en lugar de una parte de ellos .

UNA COMBINACION de un número de entes es una disposición de una parte de ellos prescin­diendo del orden, a diferencia de una variación.

Por ejemplo. las combinaciones de las tres letras, a, b y e, tomadas de dos en dos, son ab, he y ae.

Cualquiera de estas disposiciones es una combinación.

Obsérvese que ab y ba son una misma combinación (se prescinde del orden), mientras que constituyen dos variaciones distintas (interesa el orden), de las letras a y b.

NOT ACION FACTORIAL. Las identidades siguientes expresan el significado de facroria! de n escrito 11!

2' = 1 . 2 = 2, 3! = 1 . 2 . 3 = 6. 4' = 1 . 2 . 3 . 4 = 24

5'= 1'2'3 '4 '5 = 120. 11!= 1 '2'3 ... 11 , (r-I)'= 1·2·3 ... (r-l)

Nota. O' = 1 por definición.

EL SIMBOLO "V, representa el número de variaciones de 11 elementos tomados de r en r. Como caso particular, "V"' es el número de permutaciones de 11 elementos y se escribe, normalmente, p".

Así , pues. sV3 representa el número de variaciones de 8 elementos tomados de 3 en 3. y Ps es el número de permutaciones de 5 elementos.

Nafa. Los símbolos V(I1. r). V"., Y V;' tienen el mismo significado que" V,.

VARIACIONES DE n ELEMENTOS TOMADOS DE r EN r 1/'

" V, = 11(1/ - 1)(1/ - 2) ... (11 - r + 1) = ,

(1/ - rJ.

Para r = 11 , "V, = "V" = p" = 1/(1/ - 1)(/1 - 2) .. . 1 = 11'

229

http://carlos2524.jimdo.com/

Page 238: Algebra Superior Murray R Spiegel

230 COM 111NATORI¡\

Por ejemplo, s VI = 5, 5 V2 = 5' 4 = 20, 5 V3 = 5' 4 . 3 = 60, 5 V4 = 5' 4 . 3 . 2 = 120,sV5 = P5 = 5' = 5'4'3'2'1 = 120, 101'7 = 10'9'8'7'6'5'4 = 604800.

Por ejemplo, el número de maneras distintas en que 4 personas se pueden sentar en un cochede seis plazas es 6 V4 = 6 . 5 . 4 . 3 = 360.

PERMUTACIONES CON REPETICION DE n ELEMENTOS. El número P de permutacionesde n elementos repitiéndose uno de ellos nI veces, otro n2 veces, etc., viene dado por

siendo nI + 112 + 113 + ... = n

Por ejemplo, el número de maneras en que se pueden distribuir 3 monedas de veinticinco pe-setas y 7 monedas de cinco entre 10 muchachos de forma que a cada uno de ellos le corresponda una

10' 10'9'8sola moneda es 3! 7! = ~-:-} = 120.

PERMUTACIONES CIRCULARES. El número de maneras en que se pueden colocar n elernen-tos diferentes a lo largo de una circunferencia es igual a (/1 - I)!

Por ejemplo, 10 personas se pueden sentar alrededor de una mesa redonda de (10 - 1)' = 9!maneras distintas.

EL SIMBOLO ner representa el número de combinaciones de n elementos tomados de r en r,

Así. pues, 9C4 representa el número de combinaciones de 9 elementos tomados de 4 en 4.

Nota. Los símbolos e(n, r), en. r Y e~tienen el mismo significado que nCr'

COMBINACIONES DE n ELEMENTOS TOMADOS DE r EN r.

nVr n!.c. =-1r. r!(n - r)'n(/1 - 1)(/1 - 2) ... (n - r + 1)

r'Por ejemplo, el número de saludos que se pueden intercambiar entre sí 12 estudiantes, si cada

uno solo saluda una vez a los otros. es12' 12! 12· 11

12C2 = 2!(12 _ 2)! = 2!10! = ~ = 66.

Una propiedad muy útil que simplifica los cálculos en que intervienen combinaciones es,

La fórmula indica que el número de combinaciones de n elementos tomados de r en r es igualal de combinaciones de n elementos tomados de /1 - r en n-r.

Ejemplos.

9'8~ 36, C , e = 25 . 24 . 23 = 2 300

25 22 = _5 J 1 . 2 . 3

En cada uno de los casos, el numerador y el denominador tienen el mismo número de factores.

NUMEIbins

dntida

PERMU

1. Calcl

2. Halla

a)

e

b)

e

Lueg.

3. Un e:fonm

4. ¿De eden e

a) Etir de

p

h) Econce

F

5. ¿De ,

http://carlos2524.jimdo.com/

Page 239: Algebra Superior Murray R Spiegel

COMBINATORIA 231

20,OO.

NUMERO TOTAL DE COMBINACIONES DE n ELEMENTOS. El número total e de com-binaciones de n elementos distintos tomados de 1, 2, 3, ... , n, viene dado por

hee = 2" - I

nes

Por ejemplo, una persona tiene en su bolsillo una moneda de una peseta, otra de cinco, otrade veinticinco y otra de cincuenta. El número total de formas en que puede sacar de su bolsillo can-tidades de dinero diferentes es 24 - 1 = 15.

pe-una PROBLEMAS RESUELTOS

PERMUTACIONES y VARIACIONES

1. Calcular 20V2, sV" 7V" P7

20V2 = 20'19 = 380 7 V, = 7 . 6 . 5 . 4 . 3 = 2520

s Vs = 8· 7 . 6' 5 . 4 = 6720 P7 = 7! = 7' 6 . 5 . 4 . 3 ·2· 1 = 5040

a) 7n(n - I)(n - 2) = 6(n + I)(n)(n - 1).

Como n + 0,1 se puede dividir por n(n - 1), con lo cual 7(n - 2) = 6(n + 1), n = 20.

b) 3n(n - I)(n - 2)(n - 3) = (n - I)(n - 2)(n - 3)(n - 4)(n - 5).

Como n + 1, 2, 3, se puede dividir por (n - I)(n - 2)(n - 3) con lo cual

3n = (n - 4)(n - 5), n2 - 12n -<- 20 = O, (n - 1O)(n - 2) = O

Luego n = 10.

cada 3. Un estudiante tiene que elegir un idioma y una asignatura entre 5 idiomas y 4 asignaturas. Hallar el número deformas distintas en que puede hacerlo.

Puede elegir el idioma de 5 maneras y, por cada una de ellas, hay 4 formas de elegir la asignatura.

Por tanto, puede hacerlo de 5 . 4 = 20 maneras.

igual

4. ¿De cuántas formas se pueden repartir dos premios entre 10 personas sabiendo que ambos premios, a) no se pue-den conceder a una misma persona, b) se pueden conceder a la misma persona?

a) El primer premio se puede repartir de 10 formas diferentes y, una vez concedido, el segundo se puede repar-tir de 9 formas, ya que ambos no se pueden conceder a la misma persona.

Por tanto, se puede hacer de 10· 9 = 90 formas distintas.

h) El primer premio se puede repartir de 10 formas diferentes y el segundo de otras 10, ya que ambos se puedenconceder a la misma persona.

Por tanto, se puede hacer de 10· 10 = 100 formas distintas.

tores.

5. ¿De cuántas maneras se pueden introducir 5 cartas en 3 buzones?

Cada una de las 5 cartas se pueden introducir en cualquiera de los tres buzones.En consecuencia, se puede efectuar de 3· 3 . 3 . 3 . 3 = 35 = 243 maneras.

COMBINATORI A 231

NUMERO TOTAL DE COMBINACIONES DE n ELEMENTOS . El número total e de com­binaciones de n elementos distintos tomados de 1, 2, 3, ... , n, viene dado por

e = 2n - 1

Por ejemplo, una persona tiene en su bolsillo una moneda de una peseta, otra de cinco, otra de veinticinco y otra de cincuenta. El número total de formas en que puede sacar de su bolsillo can­tidades de dinero diferentes es 24

- 1 = 15.

PROBLEMAS RESUELTOS

PERMUTACIONES Y VARIACIONES

1. Calcular 20V2' sV" 7V" P7

20 V2 = 20, 19 = 380 7 V, = 7, 6 . 5 . 4' 3 = 2520

S V5 = 8 · 7 . 6' 5 . 4 = 6720 P 7 = 7! = 7' 6 . 5 . 4 . 3 ·2· 1 = 5040

2. Hallar n si a) 7' n V 3 = 6 · .+ I V3 , b) 3·.V. = .-IV"

a) 7n(n - l)(n - 2) = 6(n + l)(n)(n - 1).

Como n + 0,1 se puede dividir por n(n - 1), con lo cual 7(n - 2) = 6(n + 1), n = 20.

b) 3n(n - I)(n - 2)(n - 3) = (n - I)(n - 2)(n - 3)(n - 4)(n - 5).

Como n + 1, 2, 3, se puede dividir por (n - I)(n - 2)(n - 3) con lo cual

3n = (n - 4)(n - 5), n2 - 12n + 20 = O, (n - lO)(n - 2) = O

Luego n = lO.

3. Un estudiante tiene que elegir un idioma y una asignatura entre 5 idiomas y 4 asignaturas. Hallar el número de formas distintas en que puede hacerlo.

Puede elegir el idioma de 5 maneras y, por cada una de ellas, hay 4 formas de elegir la asignatura.

Por tanto , puede hacerlo de 5 . 4 = 20 maneras.

4. ¿De cuántas formas se pueden repartir dos premios entre lO personas sabiendo que ambos premios. a) no se pue­den conceder a una misma persona, b) se pueden conceder a la misma persona?

a) El primer premio se puede repartir de 10 formas diferentes y, una vez concedido, el segundo se puede repar­tir de 9 formas, ya que ambos no se pueden conceder a la misma persona.

Por tanto, se puede hacer de 10· 9 = 90 formas distintas.

h) El primer premio se puede repartir de lO formas diferentes yel segundo de otras lO, ya que ambos se pueden conceder a la misma persona.

Por tanto, se puede hacer de 10· 10 = 100 formas distintas.

5. ¿De cuántas maneras se pueden introducir 5 cartas en 3 buzones?

Cada una de las 5 cartas se pueden introducir en cualquiera de los tres buzones. En consecuencia, se puede efectuar de 3 . 3 . 3 . 3 . 3 = 35 = 243 maneras .

http://carlos2524.jimdo.com/

Page 240: Algebra Superior Murray R Spiegel

232 COMBINATORIA

6. Hay 4 candidatos para presidente de un club, 6 para vicepresidente y 2 para secretario. ¿De cuántas maneras sepueden ocupar estos tres puestos?

Un presidente se puede elegir de 4, un vicepresidente de 6 y un secretario de 2 formas distintas.En consecuencia, se podrán ocupar de 4· 6 . 2 = 48 formas distintas.

7. ¿De cuántas maneras distintas se pueden ordenar 5 personas en una fila?

La primera persona puede ocupar uno de los 5 puestos y, una vez que se ha situado en uno de ellos, la se-gunda puede ocupar uno de los 4 restantes, etc. Por tanto. se podrán colocar de 5' 4 . 3 . 2' I = 120 manerasdistintas.

Otro método. Número de formas = número de permutaciones de 5 personas= 1'5 = 5! = 5'4'3'2'1 = 120.

8. ¿De cuántas maneras se pueden colocar 7 libros sobre una estantería?

Número de formas = número de permutaciones de 7 libros= 1'7 = 7! = 7' 6' 5' 4' 3' 2· 1 = 5040.

9. Hallar el número de formas en que se pueden colocar en fila 4 cuadros de una colección que se compone de12 cuadros.

El primer lugar lo puede ocupar uno cualquiera de los 12 cuadros. el segundo uno cualquiera de los 11, eltercero uno cualquiera de los 10 y el cuarto uno cualquiera de los 9 restantes.

Por tanto, el número de formas es 12' 11 . 10 . 9 = 11 880.

Otro método. Número de formas = número de variaciones de 12 elementos tomados de 4 en 4= 12V4 = 12'11'10·9 = 11880

10. ¿De cuántas maneras se pueden colocar en una fila 5 hombres y 4 mujeres de forma que éstas ocupen los lugarespares?

Los hombres se pueden situar de 1'5 maneras y las mujeres de 1'4 formas. Cada una de las colocaciones delos hombres se puede asociar con una de las mujeres.

Luego se podrá efectuar de 1'5 • 1'4 = 5! 4! = 120· 24 = 2 880 maneras.

11. ¿De cuántas maneras se pueden colocar 7 cuadros diferentes en una fila sabiendo que uno de ellos debe de estar.a) en el centro, b) en uno de los extremos?

a) Como el cuadro en cuestíón debe situarse en el centro. solo quedan 6 cuadros para colocarlos en la fila. Portanto. se puede hacer de 1'6 = 6! = 720 maneras.

b) Una vez colocado el cuadro en uno de los dos extremos. los otros 6 se pueden disponer de 1'6 maneras.En consecuencia. se puede hacer de 2' P6 = 1 440 maneras.

12. ¿De cuántas maneras se pueden colocar 9 libros diferentes sobre una estantería de forma que, a) 3 de ellos esténsiempre juntos. b) 3 de ellos no estén nunca todos juntos.

a) Los 3 libros en cuestión se pueden colocar. entre ellos. de 1'3 formas. Como estos libros han de estar siem-pre juntos. se pueden considerar como lino solo. Así. pues. es como si tuviéramos 7 libros. el anterior más los 6restantes. y éstos se pueden colocar de 1'7 formas.

Por tanto. se puede hacer de 1'3 . 1'7 = 3! 7! = 6' 5 040 = 30240 formas.

b 1 El número de maneras en que se pueden colocar 9 libros sobre una estantería, sin poner condición alguna.es 9' = 362880.

El número de formas en que se pueden colocar 9 libros de modo que 3 de ellos. determinados. estén todosjuntos es. según (/l. 3' 7' = 30 240.

1

ellos.

13. ¿De I

no o

!2 de

l2 del

14. Sobnlos d

1los d

15. Hallaa) si I

signií

a)

b) r

16. Hall¡cadaformó

a) ~b) ~e) (

d) (

dos I1

17. Hallaplea

a) Lquien

rb) Lpuede8 díg

1

18. a)'Hapetirs

http://carlos2524.jimdo.com/

Page 241: Algebra Superior Murray R Spiegel

COMBINATORIA

Por tanto, el número de maneras en que se pueden colocar 9 libros sobre una estantería de forma que 3 deellos, determinados, no estén todos juntos será 362 880 - 30 240 = 332 640.

13. ¿De cuántas maneras se pueden disponer en una fila n hombres con la condición de que 2 determinados de ellosno ocupen posiciones contiguas?

El número de maneras en que se puede colocar n hombres en una fila, sin poner condición alguna, es p•. Si2 de los n hombres deben ocupar siempre posiciones contiguas, el número de formas será = 2! (P.- d.

Por tanto, el número de maneras en que se pueden colocar n hombres en una fila, con la condición de que2 determinados de ellos no estén juntos es p. - 2!P.-1 = n(n - I)! - 2(n - I)! = (n - 2)' (n - I)!

14. Sobre una estantería se tienen que colocar 6 libros distintos de biología, 5 de química y 2 de física, de forma quelos de cada materia estén juntos. Hallar el número de formas en que se puede hacer.

Los libros de biología se pueden disponer entre sí de 6! maneras, los de química de 5!, los de física de 2! Ylos de tres grupos de 3! maneras.

Por tanto, se pueden colocar de 6! 5! 2! 3! = 1 036 800 maneras.

el

15. Hallar el número de palabras diferentes de 5 letras que se pueden formar con las letras de la palabra empujado,a) si cada letra no se emplea más de una vez, b) si cada letra se puede repetir. (Estas palabras no necesitan tenersignificado. )

a) Número de palabras = variaciones de 8 elementos tomados de 5 en 5= B Vs = 8 . 7 . 6 ' 5 . 4 = 6720 palabras

h) Número de palabras = 8, 8 . 8 . 8 . 8 = 8s = 32768 palabras.

r.

16. Hallar los números que se pueden formar con 4 de los 5 dígitos 1,2, 3, 4, 5, a) si éstos no se pueden repetir encada número, b) sí se pueden repetir. Si los dígitos no se pueden repetir, ¿cuántos números de 4 cifras se puedenformar, e) empezando por 2, terminando en 25?

a) Números formados = s V4 = 5' 4' 3 . 2 = 120 números.

b) Números formados = 5' 5 . 5 . 5 = 54 = 625 números.

e) Como la primera cifra de cada número es una determinada, quedan 4 dígitos para colocar en 3 lugares.

Números formados = 4V3 = 4·3·2 = 24 números.

d) Como las dos últimas cifras de cada número son dos determinadas, quedan 3 dígitos para colocar endos lugares.

Números formados = 3 V2 = 3 . 2 = 6 números.or

17. Hallar cuántos números de 4 cifras se pueden formar con los 10 dígitos, O, 1,2,3, ... , 9, si cada uno solo se em-plea una vez. ¿Cuántos de estos números son impares?

a) La primera cifra puede ser ocupada por uno cualquiera de los lO dígitos excepto el O, es decir, por uno cual-quiera de 9 dígitos. Los 9 dígitos restantes, se pueden colocar en los otros 3 lugares de 9 V3 maneras.

Números formados = 9· 9 V3 = 9(9' 8 . 7) = 4536 números.

b) La última cifra puede ser ocupada por uno cualquiera de los 5 dígitos impares, 1, 3, 5, 7, 9. La primera cifrapuede ser uno cualquiera de los 8 digitos, es decir, los 4 dígitos impares restantes y los dígitos pares 2, 4, 6, 8. Los8 dígitos restantes se pueden colocar en las 2 posiciones centrales de B V2 maneras.

Números formados = 5'.8' BV2 = 5·8' 8· 7 = 2240 números impares.

én

ffi-S 6

a.

os 18. a) Hallar los números de 5 cifras que se pueden formar con los lO dígitos, O, 1, 2, 3, .... 9, pudiendo éstos re-petirse. ¿Cuántos de estos números b) empiezan por 40, e) son pares, d) son divisibles por 5?

233

http://carlos2524.jimdo.com/

Page 242: Algebra Superior Murray R Spiegel

234 COMBINATORIA

a) La primera cifra puede ser uno cualquiera de 9 dígitos (todos, excepto O). Cada una de las otras cifras puedenser uno cualquiera de los lO dígitos.

Números formados = 9· 10' 10' 10' lO = 9· 104 = 90000 números.

b) Las dos primeras cifras están formadas por el número 40. Las otras tres pueden ser cualquiera de los 10 di-gitos.

Números formados = 1 . 10 . 10' 10 = lO' = 1 000 números.

e) La primera cifra puede ser uno cualquiera de 9 dígitos y la última uno de los 5 números, O, 2, 4, 6, 8. Cadauna de las otras tres cifras pueden ser cualquiera de los 10 dígitos.

Números pares = 9' 10' 10· 10· 5 = 45000 números.

d) La primera cifra puede ser uno cualquiera de 9 dígitos, y la última pueden ser 2 números, el O y el 5, ylas otras 3 cifras uno cualquiera de los 10 dígitos.

Números divisibles por 5 = 9· 10·10· 10·2 = 18000 números.

19. ¿Cuántos números comprendidos entre 3 000 Y 5 000 se pueden formar con los 7 digitos, O, 1,2,3,4,5,6, si cadauno no se puede repetir en cada número?

Como los números están comprendidos entre 3 000 Y 5 000, constarán de 4 cifras. La primera puede ser el3 o el 4. Los seis dígitos restantes se pueden colocar en los otros tres lugares de 6 V, maneras.

Números formados = 2· 6 V, = 2(6' 5 . 4) = 240 números.

20. Entre 11 novelas y 3 diccionarios se seleccionan 4 novelas y 1 diccionario y se colocan en una estantería de formaque el diccionario esté en el medio. Hallar el número de formas en que esto se puede llevar a cabo.Las posibilidades de seleccionar un diccionario son 3 y el número de variaciones de 11 novelas tomadas de 4 en4es11V4·

Por tanto. se puede hacer de 3· 11 V;' = 3(11 . 10' 9· 8) = 23760 formas.

21. ¿Cuántas señales se pueden hacer con 5 banderolas diferentes sacando un número cualquiera de ellas a la vez?

Las señales se pueden hacer sacando las banderolas 1,2,3,4 Y 5 al mismo tiempo. Luego el número totalde señales es

sV1 + sV2 + sV, + sV4 + sVs = 5 + 20 + 60 + 120 + 120 = 325 señales

22. Hallar la suma de los números de 4 cifras que se pueden formar con los cuatro dígitos 2, 5, 3 Y 8. sabiendo quecada dígito no puede figurar más de una vez en cada número.

Los números que se pueden formar son V4 = 4! = 4' 3' 2' 1 = 24.

La suma de los dígitos = 2 + 5 + 3 + 8 = 18. Ycada uno de ellos estará 24/4 = 6 veces ocupando el lugarde las unidades. decenas, centenas y millares. En consecuencia, la suma de los números formados es

1(6' 18) + 10(6' 18) + 100(6' 18) + 1000(6' 18) = 119998

23. a) Hallar el número de palabras que se pueden formar con las letras de la palabra cooperador tomadas todas a lavez. ¿Cuántas de estas palabras. b) tienen juntas' las tres «o». e) empiezan por las dos «r»? (Las palabras no nece-sitan tener significado.)

a) La palabra cooperador consta de 10 letras: 3 «o», 2 «r» y 5 letras diferentes.

10' 10·9·8'7'6'5'4'3'2'1Número de palabras = -- = = 302400.

3'2! (I'2'3){1'2)

b) Considerando las tres «o» como una sola letra. tendremos 8 letras. de las cuales dos son «r».

8'Número de palabras = 21 = 20 160.

e)

24. Se di

25. a) Ib) ¿estén

a) ~4! fo

b) (

sonar= 2·

26. ¿De,cada

y las

27. ¿Cuá

1tad s

COMBIN,

28. Halla

a) .'

b) .'

e)

29. Siend

http://carlos2524.jimdo.com/

Page 243: Algebra Superior Murray R Spiegel

en

í·

da

, y

da

el

en

ez?

tal

que

gar

a laece-

COMBINATORIA 235

e) El número de palabras que se pueden formar con las 8 letras restantes, de las cuales hay tres «o», es8!j3! = 6 720.

24. Se dispone de 3 ejemplares de 4 libros diferentes. ¿De cuántas maneras se pueden colocar en una estantería?

Hay 3 . 4 = 12 libros, de los cuales cada uno está repetido 3 veces.

Número de formas = (3' 4)! = ~ = 369600.3! 3! 3! 3! (3!)4

25. a) De cuántas maneras se pueden sentar 5 personas alrededor de una mesa redonda?b) ¿De cuántas maneras se pueden sentar 8 personas alrededor de una mesa redonda de forma que dos de ellasestén siempre juntas?

a) Supongamos que una de ellas se sienta en un lugar cualquiera. Las 4 personas restantes se pueden sentar de4! formas. Por tanto, hay 4! = 24 maneras de disponer a 5 personas alrededor de una mesa circular.

b) Consideremos a las dos personas determinadas como una sola. Como hay 2! maneras de disponer a 2 perosonas entre sí y 6! formas de colocar a 7 personas alrededor de una mesa circular, el número pedido será = 2! 6!= 2 . 720 = 1 440.

26. ¿De cuántas maneras se pueden colocar 4 hombres y 4 mujeres alrededor de una mesa redonda de manera quecada mujer esté entre dos hombres?

Supongamos, en primer lugar, que se sientan los hombres. Estos se pueden colocar de 3! maneras distintasy las mujeres de 4! formas.

Por tanto, el número pedido es = 3! 4! = 144.

27. ¿Cuántas pulseras se pueden hacer ensartando en un hilo 9 cuentas de colores diferentes?

El número de formas en que se pueden disponer las cuentas en la pulsera es igual a 8!; sin embargo, la mi-tad se deduce de la otra mitad girando la pulsera.

Por tanto, se pueden formar i(8!) = 20160 pulseras diferentes.

COMBINACIONES

28. Hallar n en: a) nen-2 = 10, b) ne15 = nel!> e) nV4 = 30·nes.

n(n - 1) n2- n

a) nen-2 = ne2 = --2-'- = --2- = 10 n2 - n - 20 = O, n = 5

15 = n - 11, n = 26b) ne, = nen-,'

30' e = 30(nVs) = 30' nV4 • (n - 4)

e) n S 5! 5!

30(n - 4)

120n = 8.Luego

29. Siendo nV, = 3024 Y ne, = 126, hallar r.

r' = n V, = 3 024 = 24. ne, 126 '

r = 4nV, = r!(ne,),

COMBINATORIA 235

e) El número de palabras que se pueden formar con las 8 letras restantes, de las cuales hay tres «o», es 8!j3' = 6 720.

24. Se dispone de 3 ejemplares de 4 libros diferentes . ¿De cuántas maneras se pueden colocar en una estantería?

Hay 3 . 4 = 12 libros, de los cuales cada uno está repetido 3 veces.

Número de formas = (3' 4)! = ~ = 369600. 3! 3! 3! 3! (3!)4

25. a) De cuántas maneras se pueden sentar 5 personas alrededor de una mesa redonda? b) ¿De cuántas maneras se pueden sentar 8 personas alrededor de una mesa redonda de forma que dos de ellas estén siempre juntas?

a) Supongamos que una de ellas se sienta en un lugar cualquiera. Las 4 personas restantes se pueden sentar de 4! formas. Por tanto, hay 4! = 24 maneras de disponer a 5 personas alrededor de una mesa circular.

b) Consideremos a las dos personas determinadas como una sola. Como hay 2! maneras de disponer a 2 per­sonas entre sí y 6! formas de colocar a 7 personas alrededor de una mesa circular, el número pedido será = 2! 6! = 2 . 720 = 1 440.

26. ¿De cuántas maneras se pueden colocar 4 hombres y 4 mujeres alrededor de una mesa redonda de manera que cada mujer esté entre dos hombres?

Supongamos, en primer lugar, que se sientan los hombres. Estos se pueden colocar de 3! maneras distintas y las mujeres de 4! formas.

Por tanto, el número pedido es = 3! 4! = 144.

27. ¿Cuántas pulseras se pueden hacer ensartando en un hilo 9 cuentas de colores diferentes?

El número de formas en que se pueden disponer las cuentas en la pulsera es igual a 8!; sin embargo, la mi­tad se deduce de la otra mitad girando la pulsera.

Por tanto, se pueden formar 1(8!) = 20160 pulseras diferentes.

COMBINACIONES

28. Hallar n en : a) .C._2 = 10, b) .C15 = .Cll , e) .V4 = 30 ' .C5 •

n(n - 1) n 2 - n

a) .C.- 2 = .C2 = - -2-'- = - -2- = 10 n2 - n - 20 = O, n = 5

b) .C, = .C. _" 15 = n - 11, n = 26

30' C = 30(·V5 ) = 30' .V4 • (n - 4) e) • 55! 5!

Luego 30(n - 4)

120 n = 8.

29. Siendo .V, = 3024 Y .C, = 126, hallar r .

.V, = r!(.C,), r' = • V, = 3 024 = 24 . .C, 126 '

r = 4

http://carlos2524.jimdo.com/

Page 244: Algebra Superior Murray R Spiegel

236 COMBINATORIA

30. ¿Cuántos grupos de 4 alumnos se pueden formar con 17 alumnos aventajados para representar a un colegio enun concurso de preguntas de matemáticas?

Número de grupos = número de combinaciones de 17 alumnos tomados de 4 en 4

17' 16' 15' 14= 17C4 = 1.2 . 3 . 4 = 2 380 grupos de 4 alumnos.

31. ¿De cuántas maneras se pueden elegir 5 idiomas de entre 8?

Número de formas = número de combinaciones de 8 idiomas tomados de 5 en 5

8' 7' 6= sCs = sC3 = --- = 56 formas.

I . 2' 3

32. ¿De cuántas formas se pueden repartir 12 libros entre dos personas, A y B, de manera que a uno le toquen 9 yal otro 3?

En cada una de las divisiones de los 12 libros en 9 y 3, A recibe 9 y B recibe 3, o bien A recibe 3y B recibe 9.

12· 11 . lOPor tanto, el número de formas es = 2' 12C9 = 2· 12C3 = 2( ) = 440 formas.

l : 2' 3

33. Determinar el número de triángulos diferentes que se pueden formar uniendo los seis vértices de un exágono.Número de triángulos = número de combinaciones de 6 puntos tomados de 3 en 3

6'5'4= 6C3 = --- = 20 triángulos.

1·2·3

34. ¿Cuántos ángulos menores de 1800 forman 12 semirrectas que se cortan en un punto sabiendo que ninguna deellas puede estar en prolongación de cualquiera de las otras?

Número de ángulos = número de combinaciones de 12 elementos tomados de 2 en 2

12' II12C2 = ~ = 66 ángulos.

35. ¿Cuántas diagonales tiene un octógono?

8·7Número de rectas = número de combinaciones de 8 puntos tomados de 2 en 2 = sC2 = N = 28.

Como 8 de estas 28 rectas son los lados del octógono, el número de diagonales = 20.

36. ¿Cuántos paralelogramos se pueden formar al cortar un sistema de 7 rectas paralelas por otro sistema de 4 rec-tas paralelas?

Cada una de las combinaciones de 4 rectas tomadas de 2 en 2 forman un paralelogramo al cortar a cada unade las combinaciones de 7 rectas tomadas de 2 en 2.

Número de paralelogramos = 4C2 • 7C2 = 6· 21 = 126 paralelogramos.

37. En un plano están situados 10 puntos de forma que 4 de ellos están sobre una recta y entre los restantes no haytres en prolongación. Hallar el número de rectas que se pueden formar uniendo los lO puntos.

10' 9Número de rectas suponiendo que de los lO puntos no hay tres colineales = lOC2 = -2- = 45.

4'3Número de rectas formadas por 4 puntos de los que no hay 3 colineales = 4C2 = -2- = 6.

38. ¿D

a)

b)e)

al

b)

e)

39. Unbro

rner

40. ¿Cuencr

41. ¿Cu:de,

tinta

42. ¿Cu.cadarenn

a)h)e)

Cad:

. I

http://carlos2524.jimdo.com/

Page 245: Algebra Superior Murray R Spiegel

COMBINATORIA 237

n Como los 4 puntos son colineales. forman una recta en lugar de 6.

El número de rectas pedido es 45 - 6 + I = 40.

38. ¿De cuántas maneras se pueden elegir 3 hombres de entre un grupo de' 15. de forma que

a) uno de ellos debe figurar en cada grupo seleccionado.b) dos de ellos no deben figurar en cada grupo seleccionado.e) uno de ellos debe, y otros 2 no deben, figurar en cada grupo?

a) Como uno debe figurar siempre, tendremos que elegir 2 de entre 14.14·13

El número de formas en que se puede hacer es = 14e2 = -2- = 91.

h) Como hay 2 que no deben figurar. tendremos que elegir 3 de entre 13.13' 12' 11

El número de formas es = l3e3 = 3' = 286.

12' 11e) Número de formas = IS-I-2e3-1 = 12e2 = -2- = 66.

y

o.

39. Un equipo científico consta de 25 miembros, de los cuales 4 son doctores. Hallar el número de grupos de 3 miem-bros que se pueden formar. de manera que en cada grupo haya por lo menos un doctor.

Número total de grupos de 3 que se pueden formar COII 25 miembros = 2Se3'

Número de grupos de 3 que se pueden formar de manera que no figure en ellos un doctor = 2S-4e3 = 21e3•

Por tanto. el número de grupos de 3 miembros que se pueden formar de manera que en ellos exista por lo25 . 24 . 23 21 . 20 . 19

menos un doctor = 2Se3 - 21e 3 = 3! 3! = 970 grupos.

de 40. ¿Cuántos grupos de 7 miembros se pueden formar con 6 químicos y 5 biólogos de manera que en cada uno seencuentren 4 químicos?

Cada grupo de 4 químicos de los 6 se puede asociar con cada uno de 3 biólogos de los 5.

Por tanto. el número de grupos es = 6e4 • se3 = 15' 10 = 150.

41. ¿Cuántas palabras de 5 letras se pueden formar con 8 consonantes y 4 vocales, de manera que cada una constede 3 consonantes diferentes y 2 vocales distintas?

Las 3 consonantes distintas se pueden elegir de se3 maneras, las 2 vocales de 4e2 formas y las 5 letras dis-tintas (3 consonantes y 2 vocales) se pueden disponer entre ellas de Ps = 5! formas.

Por tanto. el número de palabras es = se3• 4e2' 5! = 56· 6' 120 = 40320.

42. ¿Cuántas palabras de 4 letras se pueden formar con 7 mayúsculas, 3 vocales y 5 consonantes, de manera quecada una empiece por una mayúscula y tenga al menos una vocal. siendo todas las letras de cada palabra dife-rentes?

hay

La primera letra. mayúscula, se puede elegir de 7 formas.

Las 3 letras restantes, pueden ser:

a) 1 vocal y 2 consonantes. que se pueden tomar de 3el . se2 maneras.h) 2 vocales y I consonante. que se pueden tomar de 3e2' se, maneras. Ye) 3 vocales, que se pueden tomar de 3e, = I forma.

Cada una de las 3 letras de estos grupos se pueden disponer entre sí de P, = 3! maneras.

Por tanto, el número de palabras = 7' 3 !(3el . se2 + 3e2 • se, + 1)= 7 . 6(3 . 10 + 3 . 5 + 1) = I 932.

COMBINATORIA

Como los 4 puntos son colineales. forman una recta en lugar de 6.

El número de rectas pedido es 45 - 6 + I = 40.

38. ¿De cuántas maneras se pueden elegir 3 hombres de entre un grupo de' 15. de forma que

a) uno de ellos debe figurar en cada grupo seleccionado. b) dos de ellos no deben figurar en cada grupo seleccionado. e) uno de ellos debe, y otros 2 no deben, figurar en cada grupo?

a) Como uno debe figurar siempre. tendremos que elegir 2 de entre 14. 14·13

El número de formas en que se puede hacer es = '4C2 = - 2- = 91.

h) Como hay 2 que no deben figurar. tendremos que elegir 3 de entre 13. 13' 12' 11

El número de formas es = "C) = 3! = 286.

e ) Número de formas = 'S-'-2C) - , = 12· 11

'2 C2 = -2- = 66.

237

39. Un equipo científico consta de 25 miembros, de los cuales 4 son doctores. Hallar el número de grupos de 3 miem-bros que se pueden formar. de manera que en cada grupo haya por lo menos un doctor.

Número total de grupos de 3 que se pueden formar COIl 25 miembros = 2SC),

Número de grupos de 3 que se pueden formar de manera que no figure en ellos un doctor = 2S-4C) = 2'C),

Por tanto. el número de grupos de 3 miembros que se pueden formar de manera que en ellos exista por lo 25 . 24 . 23 21 . 20 . 19

menos un doctor = 2SC) - 21 C) = 3! 3! = 970 grupos.

40. ¿Cuántos grupos de 7 miembros se pueden formar con 6 químicos y 5 biólogos de manera que en cada uno se encuentren 4 químicos?

Cada grupo de 4 químicos de los 6 se puede asociar con cada uno de 3 biólogos de los 5.

Por tanto. el número de grupos es = 6C4 • sC) = 15' 10 = 150.

41. ¿Cuántas palabras de 5 letras se pueden formar con 8 consonantes y 4 vocales, de manera que cada una conste de 3 consonantes diferentes y 2 vocales distintas?

Las 3 consonantes distintas se pueden elegir de sC) maneras, las 2 vocales de 4C2 formas y las 5 letras dis­tintas (3 consonantes y 2 vocales) se pueden disponer entre ellas de Ps = 5! formas.

Por tanto. el número de palabras es = sC) . 4C2 • 5! = 56· 6' 120 = 40320.

42. ¿ Cuántas palabras de 4 letras se pueden formar con 7 mayúsculas, 3 vocales y 5 consonantes, de manera que cada una empiece por una mayúscula y tenga al menos una vocal. siendo todas las letras de cada palabra dife­rentes?

La primera letra. mayúscula , se puede elegir de 7 formas.

Las 3 letras restantes, pueden ser:

a) 1 vocal y 2 consonantes. que se pueden tomar de )C, . sC2 maneras. h) 2 vocales y I consonante. que se pueden tomar de )C2 • sC, maneras. y e) 3 vocales, que se pueden tomar de )C) = I forma.

Cada una de las 3 letras de estos grupos se pueden disponer entre sí de p) = 3! maneras.

Por tanto, el número de palabras = 7' 3 !()C, . sC2 + 3C2 . sC, + 1) = 7 . 6(3 . 10 + 3 . 5 + 1) = I 932.

http://carlos2524.jimdo.com/

Page 246: Algebra Superior Murray R Spiegel

238 COMBINATORIA

43. Un niño A tiene 3 cromos y otro B tiene 9. ¿De cuántas maneras se los pueden intercambiar si cada uno tiene siem-pre el número inicial de cromos?

A puede cambiar I cromo con B de ,e, . ge,' = 3 . 9 = 27 maneras.

A puede cambiar 2 cromos con B de ,e2 . ge2 = 3 . 36 = 108 maneras.

A puede cambiar 3 cromos con B de ,e, . ge, = I .84 = 84 maneras.

N úmero total = 27 + 108 + 84 = 219 maneras.

Otro método. Supongamos que A y B juntan sus cromos. El problema se reduce a hallar de cuántas ma-neras A puede elegir 3 cromos, de entre los 12, excluyendo sus tres cromos originales. Este número viene

12' II . 10dado por '2e, - I = - I = 219 maneras.

I ·2·3

44. a) ¿De cuántas maneras se pueden repartir 12 libros entre 3 alumnos de forma que cada uno reciba 4 libros?b) ¿De cuántas maneras se pueden dividir 12 libros en 3 grupos de 4 libros cada uno?

a) El primer alumno puede elegir 4 libros. de entre los 12, de '2e4 maneras.'EI segundo puede elegir 4 libros, de entre los 8 restantes, de se4 maneras.El tercer alumno puede elegir 4 libros, de entre los 4 restantes, de 1 forma.

Número pedido = '2e4' se •• 1 = 495·70· I = 34650 maneras.

h) Los 3 grupos se pueden distribuir entre los alumnos de 3' = 6 maneras.34650

Por tanto, el número pedido = -3-'- = 5 775 grupos.

45. Se dispone de 4 objetos diferentes. ¿De cuántas maneras se puede escoger uno o más de dichos objetos?

Cada objeto se puede considerar de dos formas, que se elija o que no se elija. Ahora bien, cada una de estasdos formas se puede asociar con las dos correspondientes a cada uno de los otros objetos; por tanto, el númerode formas relativo a los 4 objetos es = 2 . 2 . 2 . 2 = 24. Pero en 2· está incluido el caso en que no se elija nin-guno de los objetos.

En consecuencia. el número pedido es = 24- 1 = 16 - 1 = 15.

Otro método. Los objetos que se pueden elegir son uno. dos. etc. Luego el número pedido es = 4e, + 4e2+ 4e, + .e. = 4 + 6 + 4 + 1 = 15 maneras.

46. ¿Cuántas sumas de dinero distintas se pueden sacar de una caja que contiene cinco monedas de 1, 5, 25, 50 y 100pesetas, una de cada clase?

Número de sumas = 26 - I = 63.

47. ¿De cuántas maneras se pueden elegir dos o más corbatas de entre una colección de 8?

Una o más corbatas se pueden elegir de 12" - 1) formas. Pero como hay que elegir dos o más, el número pe-dido es = 2s - 1 - 8 = 247.

O,}'O método, Elegir 2. 3. 4. 5. 6. 7. 8. corbatas se pueden hacer de

se2 + se, + se. + se, + seo + se, + ses = "e2 + se, + se. + se, + se2 + se, + I

= 28 + 56 + 70 + 56 + 28 + 8 + I = 247 maneras.

48. Se dispone de telas de 5 lonas diferentes de color gris. 4 tonos diferentes de color azul y 3 tonos diferentes de colorrojo. Hallar el número de selecciones de tonos que se puede efectuar con la condición de tomar siempre un tonogris y un tono azu 1.

Los tonos grises se pueden elegir de 12' - 1) formas. los azules de (2· - 1) formas y los rojos de2' formas.

Número de selecciones = 12' - 1)(24 - 1)12') = 31 . 15' 8 = 3720.

VARIA,

49. Cal

SO. Ha

51. ¿DI

52. Hado

53. Haltres

54. ¿D¡rrue

55. ¿Detos

56. Enen I

57. Ha\'

58. Hall

59. Hall

60. Hall

61. Hall6, 7

62. Hall.2, 3,

63. Hall

64. Hall:

65. Hall:

66. Hall:mar

67. Hall<dígin

68. Hall<ra qi

69. ¿Cuánen I

70. En cisignado qi

71. ¿De eniños

http://carlos2524.jimdo.com/

Page 247: Algebra Superior Murray R Spiegel

COM BI NA TOR lA 239

PROBLEMAS PROPUESTOS

71. ¿De cuántas maneras se pueden colocar en una fila 3 niñas y 3 niños de manera que no haya ni dos niñas ni dosniños ocupando lugares contiguos? .

VARIACIONES Y PERMUTACIONES

a-ne

49. Calcular 16V3' 7V4' ,V" .12 V"

SO. Hallar n en las ecuaciones a) 10· .V2 = .+ IV", h) 3' 2.+4V3 = 2' .+4V",

51. ¿De cuántas maneras se pueden sentar seis personas en un banco?

52. Hallar el número de señales distintas que se pueden hacer con cuatro banderas de colores diferentes desplegan-do dos banderas una encima de la otra.

s?53. Hallar el número de señales distintas que se pueden realizar con seis banderas de colores diferentes desplegando

tres banderas una encima de la otra.

54. ¿De cuántas maneras se puede elegir un presidente, un secretario y un tesorero en un club formado por 12miembros?

55. ¿De cuántas maneras se pueden colocar 2 libros distintos encuadernados en rojo, 3 distintos en gris y 4 distin-tos en azul sobre una estantería de manera que todos los libros de un mismo color estén juntos?

56. En una pared están clavadas 4 perchas. ¿De cuántas maneras distintas se pueden colgar de ellas 3 chaquetas, unaen cada percha?

s7

57. Hallar cuántos números de dos cifras distintas se pueden formar con los dígitos O, 3, 5, 7.

58. Hallar cuántos números pares de dos cifras distintas se pueden formar con los dígitos 3, 4, 5, 6, 8.

59. Hallar cuántos números de tres cifras distintas se pueden formar con los dígitos 1, 2, 3, 4, 5.tasroin- 60. Hallar cuántos números de tres cifras se pueden formar con los dígitos 1, 2, ... , 9.

61. Hallar cuántos números de tres cifras, iguales o distintas, se pueden formar con los dígitos 3, 4, '5,6, 7.

62. Hallar cuántos números impares de tres cifras, dos iguales y otra distinta, se pueden formar con los dígitos a) 1,2, 3, 4, b) 1, 2, 4, 6, 8.

100 63. Hallar cuántos números de cuatro cifras distintas se pueden formar con los dígitos 3, 5, 6, 7, 9.

64. Hallar cuántos números de 5 cifras distintas se pueden formar con los dígitos 2, 3, 5, 7, 9.

65. Hallar cuántos números enteros comprendidos 100 y 1 000 tienen todas sus cifras distintas.

66. Hallar cuántos números enteros mayores de 300 y menores de 1 000, con todas sus cifras distintas, se pueden for-mar con los digitos 1, 2, 3, 4, 5.

pe-

67. Hallar cuántos números comprendidos entre 100 y 1 000, con todas sus cifras distintas, se pueden formar con losdígitos O, 1, 2, 3, 4.

68. Hallar cuántosnúmeros de cuatro cifras mayores que 2000 se pueden formar con los dígitos 1,2,3,4 de mane-ra que las cifras a) no se puedan repetir, b) se puedan repetir.

69. ¿Cuántas palabras se pueden formar con las letras de problemas de manera que empiecen por una vocal y termi-nen por una consonante? (Las palabras no necesitan tener significado.)

larano

de

70. En cierto sistema telefónico se utilizan cuatro letras diferentes P, R, S, T Y los cuatro dígitos 3, 5, 7, 8 para de-signar a los abonados. Hallar el máximo de «números de teléfono» de que puede constar dicho sistema, sabien-do que cada uno está formado por una letra seguida de un número de cuatro cifras distintas.

COMBINATORIA 239

PROBLEMAS PROPUESTOS

VARIACIONES Y PERMUTACIONES

49. Calcular 16V3' 7V4' ,V" .12 V"

SO. Hallar n en las ecuaciones a) 10· .V2 = n + I V 4 , h) 3' 2n+4V3 = 2' n + 4V4'

51. ¿De cuántas maneras se pueden sentar seis personas en un banco ?

52. Hallar el número de señales distintas que se pueden hacer con cuatro banderas de colores diferentes desplegan­do dos banderas una encima de la otra.

53. Hallar el número de señales distintas que se pueden realizar con seis banderas de colores diferen tes desplegando tres banderas una encima de la otra.

54. ¿De cuántas maneras se puede elegir un presidente, un secretario y un tesorero en un club formado por 12 miembros ?

55. ¿De cuántas maneras se pueden colocar 2 libros distintos encuadernados en rojo, 3 distintos en gris y 4 distin­tos en azul sobre una estantería de manera que todos los libros de un mismo color estén juntos?

56. En una pared están clavadas 4 perchas. ¿De cuántas maneras distintas se pueden colgar de ellas 3 chaquetas, una en cada percha?

57. Hallar cuántos números de dos cifras distintas se pueden formar con los dígitos O, 3, 5, 7.

58. Hallar cuántos números pares de dos cifras distintas se pueden formar con los dígitos 3, 4, 5, 6, 8.

59. Hallar cuántos números de tres cifras distintas se pueden formar con los dígitos 1, 2, 3, 4, 5.

60. Hallar cuántos números de tres cifras se pueden formar con los dígitos 1, 2, ... , 9.

61. Hallar cuántos números de tres cifras, iguales o distintas, se pueden formar con los dígitos 3, 4, '5 , 6, 7.

62. Hallar cuántos números impares de tres cifras, dos iguales y otra distinta, se pueden formar con los dígitos a) 1, 2, 3, 4, b) 1, 2, 4, 6, 8.

63. Hallar cuántos números de cuatro cifras distintas se pueden formar con los dígitos 3, 5, 6, 7, 9.

64. Hallar cuántos números de 5 cifras distintas se pueden formar con los dígitos 2, 3, 5, 7, 9.

65. Hallar cuántos números enteros comprendidos 100 y 1 000 tienen todas sus cifras distintas.

66. Hallar cuántos números enteros mayores de 300 y menores de 1 000, con todas sus cifras distintas, se pueden for­mar con los dígitos 1, 2, 3, 4, 5.

67. Hallar cuántos números comprendidos entre 100 y 1 000, con todas sus cifras distintas, se pueden formar con los dígitos O, 1, 2, 3, 4.

68. Hallar cuántos 'números de cuatro cifras mayores que 2000 se pueden formar con los dígitos 1, 2, 3, 4 de mane­ra que las cifras a) no se puedan repetir, b) se puedan repetir.

69. ¿Cuántas palabras se pueden formar con las letras de problemas de manera que empiecen por una vocal y termi­nen por una consonante? (Las palabras no necesitan tener significado.)

70. En cierto sistema telefónico se utilizan cuatro letras diferentes P, R, S, T y los cuatro dígitos 3, 5, 7, 8 para de­signar a los abonados. Hallar el máximo de <<números de teléfono» de que puede constar dicho sistema, sabien­do que cada uno está formado por una letra seguida de un número de cuatro cifras distintas.

71. ¿De cuántas maneras se pueden colocar en una fila 3 niñas y 3 niños de manera que no haya ni dos niñas ni dos niños ocupando lugares contiguos?

http://carlos2524.jimdo.com/

Page 248: Algebra Superior Murray R Spiegel

240 COMBINATORIA

72. ¿Cuántos caracteres telegráficos se pueden formar con 3 puntos y 2 rayas en cada uno de ellos?

73. ¿Cuántas jugadas distintas se pueden presentar al lanzar tres dados?

74. ¿Cuántas palabras de tres letras distintas se pueden formar con las letras del alfabeto griego?

75. ¿Cuántas señales se pueden realizar con 8 banderas de las cuales 2 son rojas, 3 blancas y 3 azules, sabiendo quese izan de una vez sobre un asta vertical?

76. ¿De cuántas maneras se pueden sentar 4 hombres y 4 mujeres alrededor de una mesa redonda de madera que nohaya dos hombres juntos?

77. ¿De cuántas maneras distintas se pueden disponer los factores del producto a a b b b b e e c?

78. ¿De cuántas maneras se pueden repartir 9 premios diferentes entre 2 estudiantes de manera que uno reciba 3 yel otro 6?

79. ¿Cuántas estaciones de radio se pueden denominar con 3 letras diferentes del alfabeto? ¿ Cuántas se pueden deno-minar con 4 letras diferentes del alfabeto ocupando la w el primer lugar?

COMBINACIONES

SO. Hallar n en: a) 4' .C2 = .+2C3, b) .+2C. = 45, e) .C12 = .Ca.

81. Si 5' .P3 = 24' .C4, hallar n.

82. Calcular a) 7C7, b) sC3, e) 7C2, d) 7CS' e) 7C6, 1) sC7, g) sCs, h) 100C9S'

83. ¿Cuántas rectas determinan, a) 6 puntos, b) n puntos, sabiendo que no hay tres colineales?

84. ¿Cuántas cuerdas determinan siete puntos de una circunferencia?

85. Un alumno tiene que escoger 5 preguntas de entre 9. ¿De cuántas maneras puede hacerlo?

86. ¿Cuántas sumas diferentes de dinero se pueden formar tomando dos monedas de entre las siguientes: I pta, 5 pts,25 pts, 50 pts y 100 pts?

87. ¿Cuántas sumas diferentes de dinero se pueden formar con las monedas del Problema 86?

88. En una competición de tenis intervienen 6 parejas. Hallar el número de partidos que se han de jugar sabiendoque cada pareja tiene que enfrentarse con cada una de las otras a) dos veces, b) tres veces.

89. ¿Cuántos grupos diferentes de dos hombres y una mujer se pueden formar con a) 7 hombres y 4 mujeres, b) 5 hom-bres y 3 mujeres?

90. ¿ De cuántas maneras se pueden elegir 5 colores de entre 8 diferentes, 3 de los cuales son el rojo, el azul y el verde,sabiendo quea) el azul y el verde se elijan siempre,b) no se elija el rojoe) el rojo y el azul se elijan siempre y no se elija el verde?

91. ¿Cuántos grupos de investigación de 6 miembros se pueden formar con 5 flsicos, 4 químicos y 3 matemáticos,de manera que en cada grupo haya 3 fisicos, 2 químicos y 1 matemático?

92. Con los datos del Problema 91, hallar el número de grupos de 6 miembros que se pueden elegir de forma quea) 2 miembros sean matemáticos,b) por lo menos 3 miembros sean fisicos.

93. ¿Cuántas palabras de 2 vocales y 3 consonantes se pueden formar con las letras de a) stenographic .b) facetious't

94. ¿De cuántas maneras se puede colorear un cuadro con 7 colores diferentes?

95. ¿Cuántos grupos se pueden formar con 8 mujeres sabiendo que en cada uno de ellos debe haber por lo me-nos 3'1

96. Unfon

97. Encua

98. ¿Cl

99. Enrea'

100. H¡

de

saLve

49. 33

SO. a)

51. 72(

52. 12

53. 12(

54. 13

55. 1 7

56. 24

57. 9

58. 12

59. 60

SO. a)

81. 8

82. a)

d)g)

83. a)

84. 21

85. 12t

86. 10

http://carlos2524.jimdo.com/

Page 249: Algebra Superior Murray R Spiegel

rde,

82. a)

d) 21g) 56

83. al 1584. 21

85. 126

86. 10

b) 10 e) 21

e) 7 f) 8h) 4950

n(n - 1)b)-2-

87. 31

88. a) 30, b) 45

89. a) 84, b) 30

90. a) 20, b) 21, e) 1091. 180

92. a) 378, b) 462

93. a) 40320, b) 4800

94. 127

95. 219

96. a) 35,

97. 216

98. 336099. 10

100. a) 17,

b) 120

COMBINATORIA 241

ue

96. Una caja contiene 7 tarjetas rojas, 6 blancas y 4 azules. ¿De cuántas maneras se pueden elegir tres tarjetas deforma que a) todas sean rojas, b) ninguna sea roja?

. 97. En un campeonato de tenis, ¿cuántos partidos de dobles se pueden organizar con 13 jugadores sabiendo quecuatro de ellos, A, B, C y D, solo pueden formar parejas entre sí?

98. ¿Cuántos grupos de 3 demócratas y 2 republicanos se pueden formar con 8 republicanos y 10 demócratas?

99. En una reunión, después de que cada uno de los asistentes saludó una sola vez a cada uno de los restantes, serealizaron 45 salutaciones. Hallar el número de las personas que componían la reunión.

100. Hallar el número de a) combinaciones y b) variaciones de 4 en 4 que se pueden formar con las letrasde la palabra TENNESSEE.

no

y

0- SOLUCIONES DE LOS PROBLEMAS PROPUESTOS

pIS.

49. 3360, 840, 120, 12

SO. a) 4, b) 6

51. 720

52. 12

53. 120

54. 1320

55. 1728

56. 24

57. 9

58. 12

59. 60

60. 504

61. 125

62. a) 12, b) 12

63. 24

64. 120

65. 648

66. 36

67. 48

68. a) 18, b) 192

69. 90, 720

70. 1024

71. 72

72. 10

73. 216

74. 12144

75. 560

76. 144

77. 1260

78. 168

79. 15600; 13 800

do80. a) 2, 7 b) 8 e) 20

81. 8

om-

b) 163

icos,

que

lúe,

me-

COMBINATORIA 241

96. Una caja contiene 7 tarjetas rojas, 6 blancas y 4 azules. ¿De cuántas maneras se pueden elegir tres tarjetas de forma que a) todas sean rojas, b) ninguna sea roja?

. 97. En un campeonato de tenis, ¿cuántos partidos de dobles se pueden organizar con 13 jugadores sabiendo que cuatro de ellos, A, B, C y D, solo pueden fonnar parejas entre sí?

98. ¿Cuántos grupos de 3 demócratas y 2 republicanos se pueden fonnar con 8 republicanos y 10 demócratas?

99. En una reunión, después de que cada uno de los asistentes saludó una sola vez a cada uno de los restantes, se realizaron 45 salutaciones. Hallar el número de las personas que componían la reunión.

100. Hallar el número de a) combinaciones y b) variaciones de 4 en 4 que se pueden fonnar con las letras de la palabra T ENNESSEE.

SOLUCIONES DE LOS PROBLEMAS PROPUESTOS

49. 3360, 840, 120, 12 60. 504 71. 72

SO. a) 4, b) 6 61. 125 72. 10

51. 720 62. a) 12, b) 12 73. 216

52. 12 63. 24 74. 12144

53. 120 64. 120 75. 560

54. 1320 65. 648 76. 144

55. 1728 66. 36 77. 1260

56. 24 67. 48 78. 168

57. 9 68. a) 18, b) 192 79. 15600; 13 800

58. 12 69. 90, 720

59. 60 70. 1024

SO. a) 2, 7 b) 8 e) 20 87. 31 95. 219

81. 8 88. a) 30, b) 45 96. a) 35 , b) 120

82. a ) b) 10 e) 21 89. a) 84, b) 30 97. 216

d) 21 e) 7 f)8 90. a) 20, b) 21 , e) 10 98. 3360 g ) 56 h) 4950 91. 180 99. 10

83. a) 15 n(n - 1)

92. a ) 378 , b) 462 100. a) 17, b) 163 b) -2-84. 21 93. a) 40320, b) 4 800

85. 126 94. 127

86. 10

http://carlos2524.jimdo.com/

Page 250: Algebra Superior Murray R Spiegel

CAPITULO 25

Probabilidades

DEFINICION. Supongamos que un suceso se puede realizar de h maneras (casos favorables) y queno ocurre de/formas (casos no favorables), teniendo el conjunto de las h + /maneras (casos po-sibles) la misma posibilidad de producirse. La probabilidad de que este suceso ocurra es

p d h : / = ~, y la de que no ocurra es q = h ~ / = ~, siendo n = h + f

Se deduce, pues, p + q = 1, p = 1 - q y q = 1 - p.

Las posibilidades a favor de la ocurrencia del suceso son hff, y las posibilidades en conue ftt«.

Llamando p a la probabilidad de que se produzca un suceso, las posibilidades en favor de queocurra son rt«. o bien, p/(l - p), y las posibilidades en contra, qlp, o bien, 1 - pjq.

SUCESOS INDEPENDIENTES. Dos o más sucesos son independientes si la realización, o norealización, de uno cualquiera de ellos no afecta a la probabilidad de que ocurran, o no, cualquierade los restantes.

Por ejemplo, si se lanza una moneda al aire cuatro veces y se obtiene cara en todas ellas, allanzarla la quinta vez puede salir cara o cruz, con independencia del resultado de las pruebas an-teriores.

La probabilidad de que se produzcan dos o más sucesos independientes es igual al productode las probabilidades que tienen cada uno de ellos.

Por ejemplo, la probabilidad de obtener cara al lanzar una moneda la quinta y sexta vez, en1 1 1

las dos tiradas, es 2 (2) = 4 .

SUCESOS DEPENDIENTES. Dos o más sucesos son dependientes cuando la realización, o norealización, de uno de ellos afecta a la probabilidad de que se produzca uno cualquiera delos restantes.

Consideremos dos o más sucesos dependientes. Sea p¡ la probabilidad del primer suceso, P2la probabilidad del segundo después de ocurrir el primero, P3 la probabilidad de que se produzcael tercero después de haberse presentado el primero y el segundo, etc.; la probabilidad de que seproduzcan todos los sucesos, en el orden citado, es igual al producto p¡ . P2 . P3 ...

Por ejemplo, una caja contiene 3 bolas blancas y 2 negras. Si se extrae una bola al azar, la pro-

babilidad de que sea negra es 3!2 = ~. Si esta bola no se vuelve a introducir en la caja y se ex-

242

tr

te

SUCEsi

te

ESPEIdt

te

INTElqte:

l. Dede

al

bl

2. Un:de I

blat

al

el

http://carlos2524.jimdo.com/

Page 251: Algebra Superior Murray R Spiegel

PROBABILIDADES 243

trae una segunda, la probabilidad de que esta última sea también negra es 3 ~ l = ~. Por tan-

2 l lto, la probabilidad de que ambas sean negras es 5(4) = \O.

SUCESOS QUE SE EXCLUYEN MUTUAMENTE. Dos o más sucesos se excluyen mutuamentesi la realización de uno de ellos implica la no realización de los otros.

La probabilidad de que se produzca uno de entre dos o más sucesos que se excluyen mutuamen-te es la suma de las probabilidades de los mismos.

ESPERANZA MA TEMATICA. Sea p la probabilidad de que una persona reciba una cantidadde dinero m; el valor de su esperanza es p . m.

Por ejemplo, si la probabilidad de conseguir un premio de 1 000 pts es 1/5, la esperanza ma-

temática es i(1 000 pts) = 200 pesetas.

INTENTOS REPETIDOS. Sea p la probabilidad de que se produzca un suceso en un intento yq = 1 - p la probabilidad contraria; la probabilidad de que suceda exactamente r veces en n in-tentos es nC,p'qn-,. (Véanse Problemas 22-23.)

La probabilidad de que un suceso se produzca por /0 menos r veces en n intentos es

que

noiera

, alan-

eto 1.

, en

Esta expresión es la suma de los n - r + 1 primeros términos del desarrollo del binomio (p + q)".(Véanse Problemas 24-26.)

PROBLEMAS RESUELTOS

De una caja que contiene 3 bolas rojas, 2 blancas y 4 azules se extrae una bola al azar. Hallar la probabilidad pde que a) sea roja, b) no sea roja, e) sea blanca, d) sea roja o azul.

casos favorables (3 bolas rojas) 3 3 Ia) p = = = - =-

casos posibles (3 + 2 + 4 bolas) 3 + 2 + 4 9 3

I 2b) p = 1-3" = 3" 2

e) p = 9"3 + 4 7

d) p = -9- = 9"

node 2. Una bolsa contiene 4 bolas blancas y 2 negras; otra bolsa contiene 3 bolas blancas y 5 negras. Se extrae una bola

de cada bolsa. Determinar la probabilidad p de que a) las dos sean blancas, b) las dos sean negras, e) una seablanca y otra negra.

45 5e) Probabilidad de que la primera bola sea blanca y la segunda negra = -(-) = -.

68 12

. .. 2 3 1Probabilidad de que la pnmera bola sea negra y la segunda blanca = 6(g) = g'

http://carlos2524.jimdo.com/

Page 252: Algebra Superior Murray R Spiegel

244 PROBABILIDADES

5 1 13Estos sucesos se excluyen mutuamente; luego la probabilidad pedida p = 12 + "8 = 24 .

Otro método. p = 1I 5 13----=-4 24 24

3. Hallar la probabilidad de obtener 8 puntos tirando 2 dados al aire una sola vez sabiendo que las caras de éstos vannumerados del I al 6.

Cada una de las caras de un dado se puede asociar con una cualquiera del otro; luego el total de casos po-sibles es 6' 6 = 36.

Hay 5 posibilidades de obtener 8 puntos: 2,6; 3,5; 4,4; 5,3; 6,2.

casos favorablesPor tanto, la probabilidad es = 'bl

casos post es5

36

4. Hallar la probabilidad de obtener por lo menos 1 tirando dos veces un dado al aire.

La probabilidad de no obtener un en una tirada es I - 1/6 = 5/6.

La probabilidad de no obtener un en dos tiradas es (5/6)(5/6) = 25/36.

Luego la probabilidad de sacar por lo menos un 1 en dos tiradas = 1 - 25/36 = 11/36.

S. La probabilidad que tiene A de ganar a B una partida de ajedrez es igual a 1/3. ¿Cuál es la probabilidad que tieneA de ganar por lo menos una de tres partidas?

La probabilidad de A de perder una partida es 1 - 1/3 = 2/3, Y la probabilidad de que pierda las tres par-tidas es (2/W = 8/27.

Luego la probabilidad de que por lo menos gane una partida es I - 8/27 = 19/27.

6. De una baraja de 52 cartas se sacan tres naipes de uno en uno y se vuelven a introducir en el mazo después de cadaextracción. Hallar la probabilidad p de que todos sean a) tréboles, b) ases, e) corazones o tréboles.

En una baraja de 52 cartas hay 13 tréboles, 4 ases y 26 entre corazones y tréboles.

4 3 1b) p = <52) = 2197

7. Las posibilidades que tiene una persona de que le toque un premio de 50000 pts son de 23 contra 2. Hallar su es-peranza matemática.

2Esperanza = probabilidad de que le toque x valor del premio = (---)(50000) = 4000 pts.

23 + 2

8. En una caja hay 9 bolas numeradas del I al 9. Si se extraen dos al azar, ¿cuál es la probabilidad p de ob-tener a) dos números impares, b) dos números pares, e) un número par y otro impar, d) los números 2 y 5?

Hay 5 números impares y 4 números pares.

casos favorables ,e2 5a) p = casos posibles = ge2 = 18 .

4e2 lb) p = -- =-

ge2 62e2 l

d) p = ge2 = 36 .5'4 536 9

9. Una bolsa contiene 6 bolas rojas, 4 blancas y 8 azules. Si se extraen 3 bolas al azar. hallar la probabilidad de sa-car, a) tres rojas, b) tres azules. e) dos blancas y una roja. d) por lo menos una roja, e) una de cada color, f) unaroja, una blanca y una azul, en este orden.

a)

b)

d)

e)

f)

10. Deel a:e) r

a)

b)case

e)

d)

e)

11. ¿Cucuei

rey¡

244 PROBABILIDADES

5 1 13 Estos sucesos se excluyen mutuamente; luego la probabilidad pedida p = 12 + "8 = 24 .

Otro método. p = 1 5 13

4 24 24

3. Hallar la probabilidad de obtener 8 puntos tirando 2 dados al aire una sola vez sabiendo que las caras de éstos van numerados del 1 al 6.

Cada una de las caras de un dado se puede asociar con una cualquiera del otro ; luego el total de casos po­sibles es 6' 6 = 36.

Hay 5 posibilidades de obtener 8 puntos: 2,6; 3,5; 4,4 ; 5,3; 6,2.

casos favorables Por tanto, la probabilidad es = 'b

casos pOSI les

5

36

4. Hallar la probabilidad de obtener por lo menos 1 tirando dos veces un dado al aire .

La probabilidad de no obtener un en una tirada es 1 - 1/6 = 5/6.

La probabilidad de no obtener un en dos tiradas es (5/6)(5/6) = 25/36.

Luego la probabilidad de sacar por lo menos un I en dos tiradas = I - 25/36 = 11 /36.

S. La probabilidad que tiene A de ganar a B una partida de ajedrez es igual a 1/3. ¿Cuál es la probabilidad que tiene A de ganar por lo menos una de tres partidas?

La probabilidad de A de perder una partida es 1 - 1/3 = 2/3, Y la probabilidad de que pierda las tres par­tidas es (2/W = 8/27.

Luego la probabilidad de que por lo menos gane una partida es 1 - 8/27 = 19/27.

6. De una baraja de 52 cartas se sacan tres naipes de uno en uno y se vuelven a introducir en el mazo después de cada extracción. Hallar la probabilidad p de que todos sean a) tréboles, b) ases, e) corazones o tréboles.

En una baraja de 52 cartas hay 13 tréboles, 4 ases y 26 entre corazones y tréboles.

13 3 1 a) p = ("52) = 64

4 3 1 b) p =!:s2) = 2 197

7. Las posibilidades que tiene una persona de que le toque un premio de 50000 pts son de 23 contra 2. Hallar su es­peranza matemática.

2 Esperanza = probabilidad de que le toque x valor del premio = (---)(50000) = 4000 pts.

23 + 2

8. En una caja hay 9 bolas numeradas del I al 9. Si se extraen dos al azar, ¿cuál es la probabilidad p de ob­tener a) dos números impares, b) dos números pares, e) un número par y otro impar, d) los números 2 y 5?

Hay 5 números impares y 4 números pares.

casos favorables sC 2 5 a) p = casos posibles = 9 C2 = 18

4C2 1 b) p = -- = -

9C2 6

5'4 5

36 9 2C2 1

d) p = 9 C2 = 36 .

9. Una bolsa contiene 6 bolas rojas, 4 blancas y 8 azules. Si se extraen 3 bolas al azar, hallar la probabilidad de sa­car, a) tres rojas, b) tres azules. e) dos blancas y una roja. d) por lo menos una roja, e) una de cada color. f) una roja, una blanca y una azul, en este orden.

http://carlos2524.jimdo.com/

Page 253: Algebra Superior Murray R Spiegel

PROBABILIDADES 245

Luego la probabilidad de que por lo menos una sea roja = 155

204149204

casos favorables 6e3 5a) p=

casos posibles lSe3 204 .

se3 7 4e2 • 6el 3b) p=-=- e) p=---=-

lSe3 102 lSe3 68

. (4+S)e3 12e3 55d) Probabilidad de que ninguna sea roja = --- = -- = -

lSe3 lSe3 204

6·4'8 6'4·8 4e) p = -- = -----,----

lSe3 18' 17' 16/6 17

o6·4·8 6·4·8 2

p=--= =-lSP3 18'17'1651

lO. De una baraja de 52 cartas se sacan tres naipes. Determinar la probabilidad p de que a) sean todos ases, b) seanel as de tréboles, el de corazones y el de picas, en este orden, e) sean todos tréboles, d) sean todos del mismo palo,e) no haya dos del mismo palo.

a) Hay 52e3 formas de sacar 3 cartas del mazo de 52, y 4e3 de sacar 3 ases de entre los 4.

a

b) Hay 52 V3 formas de sacar 3 naipes del mazo de 52, teniendo en cuenta el orden establecido, y solo existe uncaso favorable.

1 1Luego p = -- = c:c:--:c-:--:-::-

52V3 52'51'50

7132600

13e3 11e) Hay [3e3 formas de sacar 3 tréboles de entre 13. Luego p = -- = - .

52e3 850

d) Hay 4 palos cada uno formado por 13 naipes. Por tanto, hay 4 formas de que el naipe sea de uno de ellos,y 13e3 maneras de obtener 3 naipes de un palo dado.

4·13e3 22Luego p = --- = -

52e3 425

e) Hay 4e3 = 4el = 4 formas de sacar 3 de los cuatro palos y 13 . 13 . 13 maneras de seleccionar un naipede cada uno de los 3 palos dados.

4'13·13·13Luego p = ----::----

52e3

169425

11. ¿Cuál es la probabilidad de que dos naipes, distintos y cualesquiera, de una baraja de 52 estén juntos sin tener encuenta el palo?

a Consideremos, por ejemplo, la probabilidad de que un as y un rey estén juntos. En la baraja hay 4 ases y 4reyes. Por tanto, un as se puede escoger de 4 formas y, una vez realizado, se puede elegir un rey de otras 4 ma-

http://carlos2524.jimdo.com/

Page 254: Algebra Superior Murray R Spiegel

246 PROBABILIDADES

neras. Luego, un as y después un rey se puede obtener de 4·4 = 16 maneras. Análogamente, un rey y luego unas se puede obtener de 16 maneras. Un as y un rey pueden estar juntos de 2' 16 = 32 maneras.

Por cada una de las combinaciones (as, rey), el resto de los 50 naipes y la propia combinación se pueden per-mutar de 51' formas. El número de casos posibles es, pues, 32(51 !). Como el número total de posiciones de todos

los naipes en la baraja es 52!, la probabilidad pedida es 32(51 !) = ~ = ~ .52! 52 13

12. El número total de papeletas de una rifa es 20. Sabiendo que hay 2 premios, hallar la probabilidad que tiene unindividuo que adquiere 2 papeletas de que le toque a) los dos premios, b) ninguno de ellos, e) uno de los dos.

a) El número de casos posibles es lOCl·

. 1 1Luego la probabilidad de que le toquen los dos premIOs es = -- = - .

lOCl 190

Otro método. La probabilidad de que le toque el primer premio es 2/20 = l/lO. Después de haber salidoel primer premio (él tiene una papeleta y hay 19 papeletas entre las que debe salir el segundo) la probabilidad deque le toque el segundo premio es 1/9.

Luego la probabilidad de que le toquen los dos premios es ~(~) = ~.10 19 190

b) Hay 20 papeletas de las cuales 18 no tienen premio.

. ¡sCl 153Luego la probabilidad de que no le toque premio es -- = -

lOCl 190

Otro método. La probabilidad de que no le toque el primer premio es 1 - 2/20 = 9/10. Si no le toca el pri-mero (él tiene 2 papeletas) la probabilidad de que no le toque el segundo premio es 1 - 2/19 = 17/19.

9 17 153Luego la probabilidad de que no le toque premio es 10(19) = 190

e) Probabilidad de que .le toque uno de los dos premios

- probabilidad de que no le toque premio - probabilidad de que le toquen los dos premios

153 1 36 18- 190 - 190 = 190 = 95

d P . id d de oue le touue el nri 2 18 9Otro méto o. robabili a e que le toque el primer premio, pero no el segundo, -(-) = -.20 19 95

b b· id d d .. 18 2 9Pro a 111 a e que no le toque el primer premio, pero sí el segundo, -(-) = - .20 19 95

. . 9 9 18Luego la probabilidad de que le toque uno de los dos es - + - = - .

95 95 95

13. Una caja contiene 7 papeletas numeradas del 1 al 7, ambos inclusive. Si se extraen, sucesivamente, 3 papeletas,determinar la probabilidad de que sean impar, par, impar o par, impar, par.

La probabilidad de que la primera sea impar es (4/7), de que la segunda sea par (3/6) y de que la tercera sea

. (3/5 . 1 b bi . . 4 3 3 6Impar ): en consecuencia, a pro a ilidad en el primer caso es -(-)(-) = -.76 5 35

La probabilidad de que la primera sea par es (3/7), de que la segunda sea impar (4/6) Y de que la tercera sea. . 34 2 4

par (2/5): luego la probabilidad en el segundo caso es -(-)(-) = -.76 5 35

Por tanto. la probabilidad para los dos casos es = ~ + ~ = ~ .35 35 7

14. Las 1inten

respe

tres

IS. La pmáse) vi

a) 1

b) 1

e)

16. Parade Bfavor

a) 1

b) 1

17. Unapesetcincc

pesetgundserá

http://carlos2524.jimdo.com/

Page 255: Algebra Superior Murray R Spiegel

b) La probabilidad de C es 1S6

1"6 . Luego las posibilidades de C son de 1 contra S.

PROBABILIDADES 247

r-os

Otro método, Variaciones de 7 elementos tomados de 3 en 3 = 7V3 = 7·6· S = 210.

Número de ellas de la forma impar. par. impar = 4· 3 . 3 = 36.

Número de ellas de la forma par. impar. par = 3 . 4 . 2 = 24.

36 + 24 60 2Probabilidad pedida = --- = - = - .

210 210 7

un

un14. Las probabilidades que tienen A, By C de resolver un mismo problema son 4/S. 2/3 y 3/7. respectivamente. Si

intentan hacerlo los tres, determinar la probabilidad de que se resuelva el problema.

Las probabilidades que tienen A, By C de no resolverlo son I - 4/S = l/S. I - 2/3 = 1/3 Y 1 - 3/7 = 417.respectivamente.

lidode

.. 1 1 4La probabilidad de no hacerlo los tres es -(-)(-). Luego la probabilidad de que lo resuelvan entre los

S 3 71 1 4 4 101

tres es 1 - S(3)(7) = 1 - lOS = lOS·

15. La probabilidad de que cierta persona viva 2S años más es 3/7 y la probabilidad de que viva su esposa 2S añosmás es 4/S. Hallar la probabilidad de que. dentro de 2S años. a) vivan los dos. h) viva por lo menos uno de ellos.e) viva solamente el marido.

. . . 34 12a) La probabilidad de que vivan los dos es -(-) = ~.

7 S 3S

pri-

3 4 4 I 4b) La probabilidad de que hayan muerto los dos es (1 - 7)(1 - s) = 7(S) = 35·

4 31Luego la probabilidad de que viva por lo menos uno de ellos es 1 - 35 = 35 .

e) La probabilidad de que viva el marido es 3/7 y la de que no viva su esposa es I - 4/S = l/S.

. . . . 3 I 3Luego la probabilidad de que viva solamente el mando es 7(S) = 35

mios

16. Para ocupar cierta vacante se presentan 3 candidatos, A, By C. Las posibilidades de A son de 7 contra S y lasde B de 1 contra 3. a) Hallar la probabilidad de que A o B ocupen el puesto, b) ¿cuáles son las posibilidades afavor de C?

7 7 I 1a) Probabilidad de A = 7 + S = u: y probabilidad de B = T+3 = 4"

7 1 SLuego la probabilidad de que A o B ocupen el puesto es u: + 4" = "6

a sea

17. Una bolsa contiene S monedas de una peseta y 2 de cinco pesetas, y una segunda bolsa contiene 1 moneda de unapeseta y 3 de cinco pesetas. Si se saca una moneda de una de ellas al azar, hallar la probabilidad de que sea decinco pesetas.

La probabilidad de sacar la moneda de la primera bolsa es (1/2) y la de sacar de ella una moneda de cincopesetas es (2/7); por tanto. la probabilidad será (1/2)(2/7) = 1/7. La probabilidad de sacar una moneda de la se-gunda bolsa es (1/2) y la de sacar de ella una moneda de cinco pesetas es (3;4); luego la probabilidadserá (1/2)(3/4) = 3/8.

1 3 29La probabilidad pedida es 7 + 8 = 56 .

PROBABILIDADES

Otro método. Variaciones de 7 elementos tomados de 3 en 3 = 7V3 = 7·6· 5 = 210.

Número de ellas de la forma impar, par, impar = 4, 3 . 3 = 36.

Número de ellas de la forma par, impar, par = 3 . 4 . 2 = 24.

36 + 24 60 2 Probabilidad pedida = --- = - = - .

210 210 7

247

14. Las probabilidades que tienen A, By C de resolver un mismo problema son 4/ 5. 2/3 Y 3/ 7, respectivamente. Si intentan hacerlo los tres, determinar la probabilidad de que se resuelva el problema.

Las probabilidades que tienen A, By C de no resolverlo son I - 4/5 = 1/5. I - 2/3 = 1/3 Y I - 317 = 417 . respectivamente.

.. 11 4 La probabIlIdad de no hacerlo los tres es -(-JI- l. Luego la probabilidad de que lo resuelvall entre los

5 3 7 1 1 4 4 101

tres es 1 - S(3)(7) = 1 - 105 = 105 .

15. La probabilidad de que cierta persona viva 25 años más es 3/7 y la probabilidad de que viva su esposa 25 años más es 4/5. Hallar la probabilidad de que, dentro de 25 años, a) vivan los dos, h) viva por lo menos uno de ellos. e) viva solamente el marido.

34 12 a) La probabilidad de que vivan los dos es - (- ) = -.

7 5 35

3 4 4 I 4 b) La probabilidad de que hayan muerto los dos es (1 - 7)(1 - S) = 7(S) = 35'

4 31 Luego la probabilidad de que viva por lo menos uno de ellos es 1 - 35 = 35 .

e) La probabilidad de que viva el marido es 3/7 y la de que no viva su esposa es I - 4/5 = 1/5.

. . . . 3 I 3 Luego la probabIlIdad de que vIva solamente el mando es 7(S) = 35

16. Para ocupar cierta vacante se presentan 3 candidatos, A, By C. Las posibilidades de A son de 7 contra 5 y las de B de 1 contra 3. a) Hallar la probabilidad de que A o B ocupen el puesto, b) ¿cuáles son las posibilidades a favor de C?

7 7 1 1 a) Probabilidad de A = 7 + 5 = u: y probabilidad de B = T+3 = 4'

7 1 5 Luego la probabilidad de que A o B ocupen el puesto es u: + 4' = (;

b) La probabilidad de Ces 1 5

6

1 (; . Luego las posibilidades de C son de 1 contra 5.

17. Una bolsa contiene 5 monedas de una peseta y 2 de cinco pesetas, y una segunda bolsa contiene 1 moneda de una peseta y 3 de cinco pesetas. Si se saca una moneda de una de ellas al azar, hallar la probabilidad de que sea de cÍnco pesetas.

La probabilidad de sacar la moneda de la primera bolsa es (1 /2) y la de sacar de ella una moneda de cinco pesetas es (2/7); por tanto, la probabilidad será (1 /2)(2/7) = 1/7. La probabilidad de sacar una moneda de la se­gunda bolsa es (1/2) y la de sacar de ella una moneda de cinco pesetas es (3;4); luego la probabilidad será (1 /2)(3/4) = 3/8.

I 3 29 La probabilidad pedida es 7 + 8 = 56 .

http://carlos2524.jimdo.com/

Page 256: Algebra Superior Murray R Spiegel

248 PROBARI LI DADES

18. Una bolsa contiene 2 bolas blancas y 3 negras. Cuatro personas. A, B, C, D, en este orden, sacan una sola bolay la dejan fuera de la bolsa. La que primero saque una bola blanca tiene un premio de 1 000 pts. Hallar lasesperanzas matemáticas de cada persona.

2 2La probabilidad que tiene A de ganar es "5 y su esperanza matemática 5(1 000 pts) = 400 pts,

Cálculo de la esperanza B: La probabilidad que tiene A de no acertar es 1 - 2/5 = 3/5. Si A no acierta,la bolsa contiene 2 bolas blancas y 2 negras. Entonces, la probabilidad que tiene B, si A falla, es 2/4 = 1/2. Luegola probabilidad de ganar B es = (3/5)(1/2) = 3/10 Y su esperanza matemática de 300 pts,

Cálculo de la esperanza C: La probabilidad que tiene A de fallar es 3/5 y la correspondiente de B es1 - 1/2 = 1/2. Si A Y B fallan. la bolsa contiene 2 bolas blancas y 1 bola negra. Entonces, la probabilidad de C,

• 3 1 2 1si A y B fallan. es 2/3. Luego la probabilidad de ganar C es 5(2)(:3) = 5 y su esperanza matemática

15(1 000 pts) = 200 pts.

Si A, By C fallan, solamente quedan en la bolsa bolas blancas y D acertará siempre. Luego la probabilidad

3 1 1 1 1 .. lde ganar D es = 5(2)(:3)(1) = 10 y su esperanza rnatematica 10(1000 pts) = 100 pts.

2 3 1 1Comprobación. 400 pts + 300 pts + 200 pts + 100 pts = 1000 pts y 5 + 10 + 5 + 10 = 1.

19. Once libros, de los cuales 5 son de ingeniería, 4 de matemáticas y 2 de química, se colocan al azar en una estan-tería. Hallar la probabilidad p de que los libros de cada materia estén todos juntos.

Cuando los libros de cada materia estén juntos los de ingeniería se pueden disponer de 5! maneras. los dematemáticas de 41, los de química de 2! Y los tres grupos de 3! maneras distintas.

casos favorablesp=

casos posibles5!41 2! 3!----

111

11 155

20. Cinco bolas rojas y 4 blancas se colocan al azar en una fila. Hallar la probabilidad de que las bolas de los extre-mos sean rojas.

(5 + 4) 91Número de formas en que se puedan colocar 5 bolas rojas y 4 blancas = 5T4! = 5! 4! = 126.

(9 - 2)! 7 35.Número de las anteriores en que los extremos están ocupados por bolas rojas = ---'--(5 - 2)!4! 3!41

35 5Por tanto. la probabilidad pedida es p = 126 = 18 .

21. Una bolsa contiene 6 monedas de cobre y 1 de plata; una segunda bolsa contiene 4 monedas de cobre. Se sacan5 monedas de la primera y se introducen en la segunda y, a continuación, se sacan 2 monedas de la segunda y seintroducen en la primera. Hallar la probabilidad de que la moneda de plata esté a) en la segunda bolsa, b) en laprimera bolsa.

Inicialmente, la primera bolsa contiene 7 monedas. Al sacar de ella 5 e introducirlas en la segunda. la pro-babilidad de que la moneda de plata sea una de las introducidas es 5/7 y la probabilidad de que continúe en laprimera bolsa es 2/7. .

La segunda bolsa contiene ahora 5 + 4 = 9 monedas. Al final. después de que 2 de estas monedas se hayan

pasado a la primera bolsa. la probabilidad de que la moneda de plata esté en la segunda es ~(~) = ~ ,y la pro-

.. .. 2 52 4. 5 4habilidad de que este en la prImera es - + -(-) = - (o bien I - - = -).

. 7 79 9 9 9

22. Ha

que

23. Hal

obu

dos

la ¡

24. Si 1;rrer

25. Se 1;posi

trestarru

248 PROBABILIDADES

18. Una bolsa contiene 2 bolas blancas y 3 negras. Cuatro personas. A, B, C, D, en este orden, sacan una sola bola y la dejan fuera de la bolsa. La que primero saque una bola blanca tiene un premio de 1 000 pts. Hallar las esperanzas matemáticas de cada persona.

2 2 La probabilidad que tiene A de ganar es :5 y su esperanza matemática :5(1 000 pts) = 400 pts.

Cálculo de la esperanza B: La probabilidad que tiene A de no acertar es I - 2/5 = 3/5. Si A no acierta , la bolsa contiene 2 bolas blancas y 2 negras. Entonces, la probabilidad que tiene B, si A falla , es 2/4 = 1/2. Luego la probabilidad de ganar B es = (3/5)(1 /2) = 3/ 10 Y su esperanza matemática de 300 pts.

Cálculo de la esperanza C: La probabilidad que tiene A de fallar es 3/ 5 y la correspondiente de B es I - 1/2 = 1/2. Si A Y B fallan . la bolsa contiene 2 bolas blancas y 1 bola negra . Entonces, la probabilidad de C,

_ 3 1 2 1 si A y B fallan . es 2/3. Luego la probabilidad de ganar C es :5(2)(:3) = 5 y su esperanza matemática

1 5(1 000 pts) = 200 pts .

Si A, By C fallan, solamente quedan en la bolsa bolas blancas y D acertará siempre. Luego la probabilidad

31 1 1 1 .. I de ganar D es = 5(2)(:3)(1) = 10 y su esperanza matematlca 10(1000 pts) = 100 pts.

2 3 1 1 Comprobación. 400 pts + 300 pts + 200 pts + 100 pts = 1000 pts y 5 + 10 + 5 + 10 = l.

19. Once libros, de los cuales 5 son de ingeniería, 4 de matemáticas y 2 de química, se colocan al azar en una estan­tería. Hallar la probabilidad p de que los libros de cada materia estén todos juntos.

Cuando los libros de cada materia estén juntos los de ingeniería se pueden disponer de 5! maneras, los de matemáticas de 4 '. los de química de 2! Y los tres grupos de 3! maneras distintas.

casos favorables p=

casos posibles

5!4' 2! 3' ----

11' 1 155

20. Cinco bolas rojas y 4 blancas se colocan al azar en una fila. Hallar la probabilidad de que las bolas de los extre­mos sean rojas.

(5 + 4) 9' Número de formas en que se puedan colocar 5 bolas rojas y 4 blancas = --:sT4! = 5! 4' = 126.

(9 - 2)' 7 Número de las anteriores en que los extremos están ocupados por bolas rojas = = - - = 35.

(5 - 2)!4! 3!4!

35 5 Por tanto. la probabilidad pedida es p = 126 = 18 .

21. Una bolsa contiene 6 monedas de cobre y 1 de plata ; una segunda bolsa contiene 4 monedas de cobre. Se sacan 5 monedas de la primera y se introducen en la segunda y, a continuación. se sacan 2 monedas de la segunda y se introducen en la primera. Hallar la probabilidad de que la moneda de plata esté a) en la segunda bolsa. b) en la primera bolsa.

Inicialmente. la primera bolsa contiene 7 monedas. Al sacar de ella 5 e introducirlas en la segunda. la pro­babilidad de que la moneda de plata sea una de las introducidas es 5;7 y la probabilidad de que continúe en la primera bolsa es 2/7. .

La segunda bolsa contiene ahora 5 + 4 = 9 monedas. Al final. después de que 2 de estas monedas se hayan . . . 57 5

pasado a la pnmera bolsa. la probab,lIdad de que la moneda de plata esté en la segunda es 7(9) = 9 ' y la pro-

.. .. 2 52 4 . 5 4 babllldad de que este en la pnmera es - + - (-) = - (o bIen I - - = - ).

. 7 79 9 9 9

http://carlos2524.jimdo.com/

Page 257: Algebra Superior Murray R Spiegel

PROBABILIDADES

22. Hallar la probabilidad de sacar dos «unos» al tirar simultáneamente 9 dados.

La probabilidad de que dos de los 9 dados den lugar a dos «unos» es ~(~) = (-61)2. La probabilidad de

661 5

que los otros 7 dados no den lugar a «unos» es (1 - _)7 = (_)7 Como se pueden formar 9C2 pares dife-6 61 2 5 7 78 125

remes con los 9 dados, la probabilidad de obtener exactamente 1 par de «unos» es 9C2(6) (6) = 279936'

Aplicando la fórmula: b b· . 1 2 5 7 78 125Pro a ilidad = nC,p'q"-' = 9C2(-) (-) = ---.6 6 279936

23. Hallar la probabilidad de obtener una sola vez 9 puntos con dos dados tirándolos simultáneamente 3 veces.

Un 9 se puede obtener de 4 maneras: 3,6; 4,5; 5,4; 6,3.

4 1En una tirada, la probabilidad de obtener 9 con los dos dados es H = 9 y la probabilidad de no

1 8obtener 9 es 1 - 9 = 9' La probabilidad de obtener un 9 en una tirada y de no conseguirlo en las otras

1 8dos es (9)(9)2 Como hay 3C¡ = 3 maneras distintas en las cuales una tirada es un 9 y las otras dos no,

1 8 2 64la probabilidad de conseguir una sola vez 9 puntos en tres tiradas es 3C¡(9)(9) = 243'

Aplicando la fórmula:

24. Si la probabilidad media que tiene un alumno que comienza sus estudios de no completar los cuatro años de ca-rrera es 1/3, hallar la probabilidad p de que de 4 alumnos que empiezan 3 de ellos adquieran el título.

Aplicando la fórmula: p = 2 (n - r + 1 = 4 - 3 +. 2 4 2 3 1 16 32 16

es decir, (3) + 4C¡(3) (3) = 81 + 81 = n'

2 1 41) primeros términos del desarrollo de (3 + 3) ,

nea

25. Se lanza una moneda al aire 6 veces. Hallar la probabilidad de obtener por lo menos tres caras. ¿Cuáles son lasposibilidades de obtener por lo menos 3 caras?

La probabilidad de obtener cara, en cada tirada, es igual a la probabilidad de obtener cruz, esto es, 1/2.

La probabilidad de obtener 3 caras en 6 tiradas es (1/2)3 La probabilidad de que no salga cara en las otrastres tiradas es (1/2)3. Como se pueden formar 6C3 grupos de 3 con las 6 tiradas, la probabilidad de obtener exac-tamente 3 caras esa

n

Análogamente, probabilidad de obtener 4 caras = 6C~(I/2)6 = 6C2(1/2)6,

probabilidad de obtener 5 caras = 6C5(1/2)6 = 6C¡ (1/2)6,

probabilidad de obtener 6 caras = (1/2)6

249PROBABILIDADES 249

22. Hallar la probabilidad él.: sacar dos «unos» al tirar simultáneamente 9 dados.

La probabilidad de que dos de los 9 dados den lugar a dos «unos» es ~(~) = (-61

)2. La probabilidatl de 66

1 5 que los otros 7 dados no den lugar a «unos» es (1 - _ )7 = (_ )7 Como se pueden formar 9C2 pares dife-

6 6 1 2 5 7 78 125

rentes con los 9 dados, la probabilidad de obtener exactamente 1 par de «unos» es 9C 2 (6) (6) = 279936'

Aplicando la fórmula: b b· . 1 2 5 7 78 125

Pro a ¡hdad = nC,p'q"-' = 9C2 (- ) (- ) = ---. 6 6 279936

23. Hallar la probabilidad de obtener una sola vez 9 puntos con dos dados tirándolos simultáneamente 3 veces.

Un 9 se puede obtener de 4 maneras: 3,6; 4,5; 5,4; 6,3.

4 1 En una tirada, la probabilidad de obtener 9 con los dos dados es H = 9 y la probabilidad de no

1 8 obtener 9 es I - 9 = 9 ' La probabilidad de obtener un 9 en una tirada y de no conseguirlo en las otras

1 8 dos es (9)(9)2 Como hay 3C¡ = 3 maneras distintas en las cuales una tirada es un 9 y las otras dos no,

1 8 2 64 la probabilidad de conseguir una sola vez 9 puntos en tres tiradas es 3C ¡(9)(9) = 243'

Aplicando la fórmula :

24. Si la probabilidad media que tiene un alumno que comienza sus estudios de no completar los cuatro años de ca­rrera es 1/3, hallar la probabilidad p de que de 4 alumnos que empiezan 3 de ellos adquieran el título.

Aplicando la fórmula: p = 2 (n - r + 1 = 4 - 3 + . 2 4 2 3 1 16 32 16

2 1 4 1) primeros términos del desarrollo de (3 + 3) ,

es deClf, (3) + 4 C ¡(3) (3) = 81 + 81 = 27'

25. Se lanza una moneda al aire 6 veces. Hallar la probabilidad de obtener por lo menos tres caras. ¿Cuáles son las posibilidades de obtener por lo menos 3 caras?

La probabilidad de obtener cara, en cada tirada, es igual a la probabilidad de obtener cruz, esto es, 1/2.

La probabilidad de obtener 3 caras en 6 tiradas es (1/2)3 La probabilidad de que no salga cara en las otras tres tiradas es (1/2)3 . Como se pueden formar 6C 3 grupos de 3 con las 6 tiradas, la probabilidad de obtener exac­tamente 3 caras es

Análogamente, probabilidad de obtener 4 caras = 6C~(I/2)6 = 6C2(l/2)6,

probabilidad de obtener 5 caras = 6C5(1/2)6 = 6C¡(1/2)6,

probabilidad de obtener 6 caras = (1/2)6

http://carlos2524.jimdo.com/

Page 258: Algebra Superior Murray R Spiegel

250 PROBABILIDADES

Las posibilidades a favor de obtener por lo menos 3 caras es de 21 contra 11.

Aplicando la fórmula: p = 4(n - r + 1 = 6 - 3 + 1) primeros términos del desarrollo (~ + ~)6, esto es,

1 6 1 6 1 6 1 6 21(2) + 6C¡(2J + 6C2(2) + 6C3(2) = 32

26. Hallar la probabilidad p de que de los 5 hijos de una familia haya por lo menos 2 niños y 1 niña. Se supone que laprobabilidad de nacer niño o niña es 1/2.

Los tres casos favorables son: 2 niños, 3 niñas; 3 niños, 2 niñas; 4 niños, 1 niña.

PROBLEMAS PROPUESTOS

27. Hallar la probabilidad de que al escoger al azar un dígito de entre 1.2, 3, ... , 9, sea a) impar, b) par, e) múlti-plo de 3.

28. Se lanza una moneda al aire 3 veces y sea H = cara y T = cruz. Hallar la probabilidad de que se obtenga a) HTH,b) THH, e) HHH, en el orden que se indica.

29. Se lanzan tres monedas al aire. Hallar la probabilidad de obtener a) tres caras, b) dos caras y una cruz.

30. Hallar la probabilidad de obtener 7 puntos lanzando, una sola vez, dos dados al aire.

31. Hallar la probabilidad de obtener 8 u 11 puntos lanzando, una sola vez, dos dados al aire.

32. Se lanza un dado al aire dos veces. Hallar la probabilidad de obtener un 4 o un 5 en la primera y un 2 oun 3 en la segunda. ¿Cuál es la probabilidad de no obtener un 1 en las dos tiradas?

33. Se lanza una moneda al aire seis veces. Hallar la probabilidad de obtener por lo menos una cara.

34. Una bolsa contiene cinco bolas numeradas con los dígitos 1,2,3,4,5. Hallar la probabilidad de que al sacar tresbolas al azar, su suma sea mayor que 10.

35. Una caja contiene 5 bolas rojas, 8 blancas y 4 negras. Se sacan tres bolas al azar y se pide, a) probabilidad de quelas tres sean blancas, b) probabilidad de que dos sean negras y una roja, e) probabilidad de que sean unade cada color.

36. De una baraja de 52 naipes se extraen 4. Hallar la probabilidad de que a) sean todos reyes, b) sean dos reyes ydos ases, e) todos sean del mismo palo, d) todos sean tréboles.

37. Un hombre ganará 320 pts si lanzando 5 veces una moneda al aire consigue sacar HTHTH, o bien THTHT, sien-do H = cara y T = cruz. Hallar su esperanza matemática.

38. En una discusión ocurrida en una reunión a la que asistían veinte personas, fueron injuriadas tres de ellas. Entrelos asistentes había tres periodistas. Hallar la probabilidad de que estos tres periodistas fueran las personasmaltratadas.

39. Entre 5 hombres y 4 mujeres se tiene que formar un grupo de tres miembros. Si la selección se realiza al azar,hallar la probabilidad de que a) las tres sean mujeres, b) dos sean hombres.

40. Seis personas se sientan alrededor de una mesa redonda. Hallar la probabilidad de que dos personas determina-das ocupen lugares contiguos.

41. SeIr¡

42. DIbiequerr

43. Sese,

44. Sem¡

45. Seen

46. ¿C

47. Seo

SOLVe

27. a)

29. a)

31. 71'33. 63,

35. a)

37. 20

39. a)

41. a)

43.715

45. a)

9947.

121

http://carlos2524.jimdo.com/

Page 259: Algebra Superior Murray R Spiegel

27. a) 5/9 b) 4/9 e) 1/3 28. a) 1/8 b) 1/8 e) 1/8

29. a) 1/8 bl 3/8 30. 1/6

31. 7/36 32. 1/9, 25/36

33. 63/64 34. 1/5

i- 1 7 4 1 36 44 1135. a) b) e) 36. a) b) e) d)

170 34 17 270725 270725 4165 4165

37. 20 pts 38. 1/1 140

39. a) 1/21 b) 10/21 40. 2/5

29 31 2141. a) b) 42. 32' 21 : 11330 66

7 35 247 219 743. - 44. a) b)

256el

256dl

32o 15 128

17 1 91945. al b) 46.

162 144 5832

es 9947.

128uea

y

n-

reas

r,

a-

a

PROBABILIDADES 251

41. Se escogen al azar cuatro letras de la palabra M1SS1SSIPPI. Determinar la probabilidad de que al tres de las le-tras sean «1», b) a lo sumo haya dos consonantes.

42. Dos personas, A y B, lanzan alternativamente una moneda al aire. El primero que saque cara es el que gana. Sa-biendo que cada uno no puede tirar más de cinco veces en cada juego, hallar la probabilidad de que la personaque lance en primer lugar sea la que gane la partida. ¿Cuáles son las posibilidades en contra de A si es él quienempieza a tirar?

43. Se colocan al azar en una fila seis bolas rojas y 4 blancas. Hallar la probabilidad de que las dos bolas centralessean del mismo color.

44. Se lanza 8 veces una moneda al aire. Hallar la probabilidad de que se obtengan al exactamente 4 caras, b) por lomenos 2 cruces, e) como máximo 5 caras, d) exactamente 3 cruces.

45. Se hacen dos tiradas con dos dados a la vez. Determinar la probabilidad de obtener a) exactamente 11 puntosen una sola vez, b) 10 puntos en dos veces.

46. ¿Cuál es la probabilidad de obtener por lo menos 11 puntos en tres tiradas con dos dados?

47. Se lanza una moneda al aire 10 veces. Hallar la probabilidad de que el número de caras que se obtengan sea igualo mayor que 3 e igual o menor que 6.

SOLUCIONES DE LOS PROBLEMAS PROPUESTOS

http://carlos2524.jimdo.com/

Page 260: Algebra Superior Murray R Spiegel

CAPITULO 26

Determinantes y sistemas de ecuaciones lineales

DETERMINANTES DE SEGUNDO ORDEN. El símbolo

formado por los cuatro números a¡, b¡, a2' b2, ordenados en una matriz de dos filas y dos colum-nas representa un determinante de segundo orden o determinante de orden dos. Los cuatro númerosanteriores se denominan elementos de la matriz o del determinante. Por definición, el determinantede una matriz de segundo orden es el polinomio

Por ejemplo, 1_ f _~1 = (2)(-2) - (3)(-1) = -4 + 3 = -1. Los elementos 2 y 3 cons-tituyen la primera fila y los -1 Y - 2 la segunda fila. Los elementos 2 y - 1forman la primera co-lumna y los elementos 3 y - 2 la segunda columna.

Un determinante de primer orden es un solo número.

LOS SISTEMAS DE DOS ECUACIONES LINEALES con dos incógnitas se pueden resolver em-pleando el concepto de determinante de una matriz de segundo orden. Dado el sistema de ecuaciones

{a¡x + b¡y = e¡

a2x + b2Y = e2(1)

aplicando uno de los métodos del Capítulo 12, se obtiene la solución

Estos valores de x e y se pueden expresar en función de determinantes de segundo orden comosigue:

I a¡ e¡

y = 1:: ~:a2 b2

(2)

252

DETi

~ap

q

g

1.

2.

CAPITULO 26

Determinantes y sistemas de ecuaciones lineales

DETERMINANTES DE SEGUNDO ORDEN. El símbolo

formado por los cuatro números al' bl , a2' b2, ordenados en una matriz de dos filas y dos colum­nas representa un determinante de segundo orden o determinante de orden dos. Los cuatro números anteriores se denominan elementos de la matriz o del determinante. Por definición, el determinante de una matriz de segundo orden es el polinomio

Por ejemplo, I-f _~ I = (2)(-2) - (3)(-1) = -4 + 3 = -1. Los elementos 2 y 3 cons­

tituyen la primera fila y los - 1 Y - 2 la segunda fila . Los elementos 2 y - 1 forman la primera co­lumna y los elementos 3 y - 2 la segunda columna.

Un determinante de primer orden es un solo número.

LOS SISTEMAS DE DOS ECUACIONES LINEALES con dos incógnitas se pueden resolver em­pleando el concepto de determinante de una matriz de segundo orden. Dado el sistema de ecuaciones

{

alx + bly = el

a2x + blJ = e2

aplicando uno de los métodos del Capítulo 12, se obtiene la solución

(1 )

Estos valores de x e y se pueden expresar en función de determinantes de segundo orden como sigue:

I al el

y = I:~ ~~ a2 b2

(2)

252

http://carlos2524.jimdo.com/

Page 261: Algebra Superior Murray R Spiegel

DETERMINANTES Y SISTEMAS DE ECUACIONES LINEALES 253

La regla de aplicación es:

1. Los denominadores de (2) son el determinantel:~ t~I en el que sus elementos son los

coeficientes de x e y dispuestos como en las ecuaciones dadas (l). Este determinante, que sesuele representar por la letra griega L\, recibe el nombre de determinante de los coeficientes.

2. El numerador correspondiente a cada una de las incógnitas se forma a partir del determinan-te de los coeficientes,L\, sustituyendo la columna de los coeficientesde la incógnita que se des-peja, por la columna de términos independientes, de las ecuaciones (1), pasados al segundomiembro.

Ejemplo. Resolver el sistema {2x + 3y = 8x - 2y = -3

El denominador de x e y es L\ = I i _~1= 2(-2) - 3(1) = -7.

se

Luego x = I-~ -~ 1= 8(-2) - 3(-3) = 1 e y = Ii -~1= 2(-3) - 8(1) = 2.-7 -7 -7-7

El método de resolución de sistemas de ecuaciones lineales mediante determinantes se llamaregla de Cramer.

DETERMINANTES DE TERCER ORDEN. El símbolo

s-al bl cl

a2 b2 C2

a3 b3 c3

formado por nueve números ordenados en una matriz de tres filas y tres columnas representa eldeterminante de una matriz de tercer orden. Por definición, el valor de este determinante viene dadopor el polinomio

es

1) que se llama desarrollo del determinante.

Con objeto de recordar fácilmente cómo se obtiene este desarrollo, se propone la norma si-guiente: Se escriben, al lado del determinante, las dos primeras columnas del mismo:

o (26-9)

+ + +

(2)1. Se multiplican los elementos de las tres diagonales, en el sentido de izquierda a derecha y de

arriba abajo, afectando a cada producto del signo más.

2. Se multiplican los elementos de las otras tres diagonales, en el sentido de derecha a izquier-da y de arriba abajo, afectando a cada producto del signo menos.

DETERMINANTES Y SISTEMAS DE ECUACIONES LINEALES 253

La regla de aplicación es:

1. Los denominadores de (2) son el determinante l :~ t~ I en el que sus elementos son los

coeficientes de x e y dispuestos como en las ecuaciones dadas (1). Este determinante, que se suele representar por la letra griega L\, recibe el nombre de determinante de los coeficientes.

2. El numerador correspondiente a cada una de las incógnitas se forma a partir del determinan­te de los coeficientes, L\, sustituyendo la columna de los coeficientes de la incógnita que se des­peja, por la columna de términos independientes, de las ecuaciones (1), pasados al segundo miembro.

Ejemplo. Resolver el sistema {2x + 3y = 8 x - 2y = - 3

El denominador de x e y es L\ = I i _ ~ 1= 2( - 2) - 3(1) = - 7.

Luego x = I-~ - ~ 1= 8( - 2) - 3( - 3) = 1 e y = I i - ~ 1= 2( - 3) - 8(1) = 2. -7 -7 -7 - 7

El método de resolución de sistemas de ecuaciones lineales mediante determinantes se llama regla de Cramer.

DETERMINANTES DE TERCER ORDEN. El símbolo

al b l c l

a2 b2 c2

a3 b3 c 3

formado por nueve números ordenados en una matriz de tres filas y tres columnas representa el determinante de una matriz de tercer orden . Por definición, el valor de este determinante viene dado por el polinomio

que se llama desarrollo del determinante.

Con objeto de recordar fácilmente cómo se obtiene este desarrollo, se propone la norma si­guiente : Se escriben, al lado del determinante, las dos primeras columnas del mismo :

(26-9)

1. Se multiplican los elementos de las tres diagonales, en el sentido de izquierda a derecha y de arriba abajo, afectando a cada producto del signo más.

2. Se multiplican los elementos de las otras tres diagonales, en el sentido de derecha a izquier­da y de arriba abajo, afectando a cada pmducto del signo menos.

http://carlos2524.jimdo.com/

Page 262: Algebra Superior Murray R Spiegel

¡alX+ bly + elz = dia2x + b2y + e2z = d2a3x + b3y + e3z = d3

(3)

2

254 DETERMINANTES Y SISTEMAS DE ECUACIONES LINEALES

3. La suma algebraica de los seis productos 1) Y 2) es el desarrollo del determinante.

3 -2 2 IEjemplo. Desarrollar 6 1 - 21 Se escribe, _ 6

-2 -3(26-10)

+ +

El valor del determinante es

(3)(1)(2) + (-2)(-1)(-2) + (2)(6)(-3) - (2)(1)(-2) - (3)(-1)(-3) - (-2)(6)(2) = -15

REGLA DE CRAMER. Se aplica también en la resolución de sistemas de tres ecuaciones linea-les con tres incógnitas x, y, z.

En realidad, es una generalización de la regla de Cramer para los sistemas de ecuaciones li-neales con dos incógnitas. Resolviendo el sistema de ecuaciones (3) por uno de los métodos delCapítulo 12, se obtiene

Esta solución se puede expresar por medio de determinantes como sigue: DETER

1. Calcdi bl el al d2 el al bl did2 b2 e2 a2 d2 e2 a2 b2 d2

x= d3 b3 e3 Y=a3 d3 e3 z=

a3 b3 d3 (4)~ ~ ~

I al bl el Isiendo ~ = I a2 b2 e2 I el determinante de los coeficientes de x, y, z en las ecuaciones (3)

I a3 b3 e3 I

a)

b)

e)

d)

suponiendo que sea distinto de cero.e)

La regla de aplicación práctica es la siguiente:

1. Los denominadores de (4) son el determinante ~ cuyos elementos son los coeficientes de lasincógnitas x, y, z dispuestos como en las ecuaciones dadas (3). 2. a)

http://carlos2524.jimdo.com/

Page 263: Algebra Superior Murray R Spiegel

DETERMINANTES Y SISTEMAS DE ECUACIONES LINEALES 2552. El numerador correspondiente a cada una de las incógnitas se forma a partir del determinan-

te de los coeficientes, A, sustituyendo la columna de los coeficientes de la incógnita que se des-peja por la columna de términos independientes, del sistema (3), pasados al segundo miembro.

Ix + 2y - z = -3

Ejemplo. Resolver el sistema 3x + y + z = 4x-y+2z=6

21

-1-11; = 2 + 2 + 3 + 1 + 1 - 12= -3

1-3 2 -114 1 1

x= 6 -1 2 =-6+12+4+6-3-16=-=2=1-3 -3 -3

-3 -11: ; = 8 - 3 - 18+ 4 - 6 + 18= ~ = -1

-3 -3 -3

2 -31 4

- 1 6 = 6 + 8 + 9 + 3 + 4 - 36= - 6 = 2-3 -3 -3

PROBLEMAS RESUELTOS

DETERMINANTES DE SEGUNDO ORDEN

l. Calcular las determinantes siguientes:

a) li ¡I = (3)(4) - (2)(1) = 12 - 2 = 10

b) I~ -11-2 = (3)(-2) - (-1)(6) = -6 + 6 = O

e) I~ _;I = (0)(-5) - (3)(2) = O - 6 = -6

I~x2

1d) y2 = xy2 - x2y

IX+2 2x+51e) 3x _ 1 x _ 3 = (x + 2)(x - 3) - (2x + 5)(3x - 1) = -5x2 - 14x - 1

2. a) Demostrar que si en un determinante de segundo orden se intercambian las filas por las colunsuas, novaría.

http://carlos2524.jimdo.com/

Page 264: Algebra Superior Murray R Spiegel

256 DETERMINANTES Y SISTEMAS DE ECUACIONES LINEALES

e)

b) Demostrar que si los elementos de una fila (o columna) son proporcionales a los elementos correspondien-tes de la otra, el determinante es nulo.

b.

a) Sea el determinante \:; !; I = a.b¿ - a2b i-

Si se intercambian las filas por las columnas. el nuevo determinante es

\~: ~: \= a.b¿ - a2b •.

b) El determinante cuyos elementos de una fila son proporcionales a los de la otra es

\ ~~. ~bl \ = a.kb, - b Jca, = O.

6. R

R l l ., \2X-I 2X+I\ O3. eso ver a ecuación x + 1 4x + 2 = .a)

\2X - 1 2x + 1\ = (2x _ 1)(4x + 2) _ (2x + I)(x + 1) = 6x2

- 3x - 3 = Ox + 1 4x + 2

Como 2X2 - x - 1 = (x - 1)(2x + 1) = O se obtiene x = 1, -1/2.

ECUACIONES LINEALES DE DOS INCOGNITAS

4. Resolver los sistemas siguientes: b)

a) {4X + 2y = 53x - 4y = 1

\i -;I -22x = \-4-2\= -22 = 1,

3 -4

u =\ '__:~__ -_i \= -42 =1_; -il 7

1 ~ i1 -11 1Y = 1-4--2-1 = -22 = 2"

3 -4

b) {3u + 2v = 18

-5u- v= 12-6, v =1'---_-_;_:~_I= 126 = 18

7 7

DETEe) {

5x - 2y - 14 = O2x + 3}' + 3 = O

Se escribe como {5X - 2y = 142x + 3y = -3 .

\ 14 -;\ 1; 14-3 36 -3 -43

x= -;1 19 ' y=19 19I 5

2 a)

7. ea

5. Despejar los sistemas:

t3X - ~ + 7Y + I = 10 (1 )

5 10a)

~_"!:._~_ 2)" - 5 = 3 (2)2 3

Multiplicando (\) por 10: 6x + Tv = 103.

Multiplicando (2) por 6: 3x - 4y = -1. b)

Luego \103 7\-1 -4 -405

x= -----=--=9. 1 6 7\ -45 .

3 -4

\6 103\3 -1 -315

y = = -- = 7.-45 -45

http://carlos2524.jimdo.com/

Page 265: Algebra Superior Murray R Spiegel

DETERMINANTES Y SISTEMAS DE ECUACIONES LINEALES 257

n- Multiplicando (1) por (x + I)(y + 1): 2x - 3y = 1.

Multiplicando (2) por (x - 7)(2y - 3): 3x + 4y = 27.

Luego

6. Resolver los sistemas de ecuaciones siguientes:

{

~-~=~a) 2 3 1

-+-=-x y 2

S . l' 1 1on ecuaciones ineales en - y -x y

11/6 -6l 1;

1/6Luego 1/2 3 _ 7/2 _ ~ 1 1/2 7/6 1

1-~ 1- 21 - 6'

- o x = 6, Y = 18.x 3 y 21 21 -182

!-'-+2~] r 81-(-) + -(-) = 3

b)2x 5y

se puede poner 2 x 5 y4 1 1 41---= 1 -(-)+-(-)= 13y x x 3y

13 8/51 1

3/2 i1= 9/2 = ~1 4/3 12/5 2 1 -1 3 4Luego = 18/5 ="3' - = o sea x=iY=sx

13/2 8/51 Y 18/5 18/5 4-1 4/3

DETERMINANTES DE TERCER ORDEN

7. Calcular los siguientes determinantes:

Repitiendo las dos primeras columnas: ~

(3)(4)(2)+ (-2)(5)(6) + (2)(1)(-1) - (2)(4)(6)- (3)(5)(-1) - (-2)(1)(2) = -67

I-~ 2-3/ la b elb) -3 2 = 29 d) e a b = a3 + b3 + e3 - 3abe

-1 -3 b e a

e) I ~ 3~ 1=-15

I (x - 2) (y + 3) (z - 2) I-2 e) -2 3 ~ = Ilx + 6y + z - 6-1 4 1 -2

http://carlos2524.jimdo.com/

Page 266: Algebra Superior Murray R Spiegel

258 DETERMINANTES Y SISTEMAS DE ECUACIONES LINEALES

a) Demostrar que si dos filas (6 dos columnas) de un determinante de tercer orden son proporcionales (elemen-tos correspondientes), el valor del determinante es cero.

b) Demostrar que si los elementos de una fila cualquiera (o columna) se multiplican por una constante y losproductos obtenidos se suman a los elementos correspondientes de otra fila (o columna), el valor del deter-minante no varía.

Ial bl ('1 I

a) Tenemos que demostrar que ka, kb, kc, = O, siendo proporcionales los elementos de las dos pri-a, b, e,

meras filas. Para ello, basta con desarrollar el determinante.

Tenemos que demostrar que si k es una constante

habiendo multiplicado cada elemento de la segunda fila del determinante dado por k y sumado a estos produc-tos los elementos de la tercera fila. El teorema se demuestra desarrollando ambos determinantes y viendoque son iguales.

ECUACIONES LINEALES DE TRES INCOGNITAS

9. Resolver los sistemas de ecuaciones siguientes:rX+Y-Z=56 = I ~

1-1\a) 3x - 2y + 2z= - 3 Se tiene -2 2 = 42 Y

x - 3y - 3z = -2 -3 -3

15 1 -;1 I~ 5 -; I I~ 1 5

I -42-3 -2 -3 -2 -3-2 -3 -3 42 -2 -3 84 -3 -2

x= =42= 1, y= = 42= 2, z= =-=-16 6 6 42

{

X+2Z=7b) 3x + y = 5

2.1' - 3= = -5 {

X + Oy + 2z = 7Tendremos 3x + y + Oz = 5

Ox + 2y - 3z = - 5y~ ~ 1= 9

2 -3Luego 6 = Ii

1,li j JI18

y = ----:-6----' = <) = 2,99

10. Las ecuaciones para hallar las intensidades de corriente ;1' ;2';' en un determinado circuito eléctrico de tres mallas

11. (

a

12.

13.

a

e,

14. e

a

15. H

16. DU1

17. R

a)

18. O

a)

SOLU(

U. a)

13. a)b)e)

14. a)

15. Te

17. a)

18. a)

http://carlos2524.jimdo.com/

Page 267: Algebra Superior Murray R Spiegel

DETERMINANTES Y SISTEMAS DE ECUACIONES LINEALES 259

n-

PROBLEMAS PROPUESTOSosr- 11. Calcular los determinantes siguientes:

ri- I4 - 3 I 1 - 2 41 12 -1 I d) 1 - 2x - 3y 1 e) 1 a + b a - b 1 /) 12x - l x + l 1

a) _ 1 2 b) - 3 7 e] 4 O 4x - y a - b x + 2 x - 2

12. Demostrar que si los eleme tos de una fila (o columna) de un determinante de segundo orden se multiplican porun mismo número, el determinante queda multiplicado por dicho número.

13. Resolver los sistemas siguientes:

e)

{5X + 2y = 42x - y = 7

IX; 3 + y; 4 = 7

x+2_y-~=_37 2

b) {3r - 5s = -64r+2s=5 e) {

28 + 4x + 5y = O- 3x + 4y + 10 = O d) {

5X - 4y = 162x + 3y = -10

a)

e-do

¡3x + 2y + 1 = 4x+y

/)5x + 6y - 7 = 2

x+y

14. Calcular los determinantes siguientes:

-231

1-1

3 JI ~ -~ -; I-4 2-1 I

3 -1e) -2 1

1 3-~ I-2

Ik + 3 1

15. Hallar el valor de k para que 3 -2-k -3

-21; = O?

16. Demostrar que si los elementos de una fila (o columna) de un determinante de tercer orden se multiplican porun mismo número, el valor del determinante queda multiplicado por dicho número.

17. Resolver los sistemas siguientes:

{

3X + y - 2z = 1a) 2x + 3y - z = 2

x - 2y + 2z = -10{

U + 2v - 3w = - 7b) 2u - v + w = 5

3u - v + 2w = 8 {

2X + 3y = -2e) 5y - 2z = 4

3z + 4x = -7

y 18. Despejar la incógnita indicada:

para i2 {

l/x + 2/y + I/z = 1/2b) 4/x + 2/y - 3/z = 2/3

3/x - 4fy + 4/z = 1/3para x

SOLUCIONES DE LOS PROBLEMAS PROPUESTOS

las11. a) 5 b) -2 e) 4

13. a) x = 2, Y = -3b) r = 1/2, s = 3/2e) x= -2, y = -4

14. a) 43 b) 19 e) O

15. Todos los valores de k.

17. a) x = -2, Y = 1, z = -3

18. a) i2 = 0,8 b) x=(

d) 14xy e) _a2 _ b2

d) X = 8/23, Y = - 28/23e) x = 12, Y = 16f) x = 5, Y = -2

/) x2- ax

g) x = 12, Y = 15h) u = 2/3, v = 3/5

d) 5x + 8y - 14z

b) u = 1, v = -1, w = 2 e) x = -4, Y = 2, z = 3

http://carlos2524.jimdo.com/

Page 268: Algebra Superior Murray R Spiegel

all a12 al3 al.a21 a22 a23 a2•a31 a32 a33 a3.

doCAPITULO 27

Determinantes de orden nPROPI

INVERSION. En una disposición cualquiera de números en una fila existe una inversión cuandouno de ellos está delante de otro más pequeño.

Por ejemplo, en la disposición 4, 3, 1, 5, 2, el número 4 está delante del 3, del 1, Ydel 2; el 3está delante del 1 y del 2, y el 5 está delante del 2; por tanto, hay 6 inversiones.

Análogamente, si se trata de letras del alfabeto en lugar de números, existe una inversión cuan-do una letra está delante de otra que es anterior a ella en la ordenación alfabética. Por ejemplo,en la disposición bdea, la letra b está delante de la a, la d está delante de las e y a y la e está delan-te de la a; por tanto, hay 4 inversiones.

DETERMINANTES DE ORDEN n. El símbolo

a. b; e. ... k.

II

al bl el kla2 b2 e2 k2a3 b3 e3 k3

formado por n2 números (llamados elementos) dispuestos en n filas y n columnas, representa eldeterminante de una matriz de orden n. Este símbolo es una forma abreviada de expresar el polino-mio constituido por la suma algebraica de todos los productos posibles, cada uno de n factores,de manera que:

1) En cada producto figura solamente un elemento de cada fila y uno solo de cada columna. Habrá,por tanto, n! productos.

2) El signo de cada producto es positivo o negativo, según que el número de inversiones de lossubíndices sea par o impar, después de haber colocado las letras en el orden en el que figuranen la primera fila. .

La suma algebraica así obtenida se denomina desarrollo o valor del determinante. Cada produc-to, con su signo, recibe el nombre de término del desarrollo del determinante.

1\

Un determinante de orden n se representa también por

v

Con esta notación, cada elemento se caracteriza por dos subíndices, el primero indica la fila y el

260

CAPITULO 27

Determinantes de orden n

INVERSION. En una disposición cualquiera de números en una fila existe una inversión cuando uno de ellos está delante de otro más pequeño.

Por ejemplo, en la disposición 4, 3, 1, 5, 2, el número 4 está delante del 3, del 1, Y del 2 ; el 3 está delante del 1 y del 2, y el 5 está delante del 2; por tanto, hay 6 inversiones.

Análogamente, si se trata de letras del alfabeto en lugar de números, existe una inversión cuan­do una letra está delante de otra que es anterior a ella en la ordenación alfabética. Por ejemplo, en la disposición bdea, la letra b está delante de la a, la d está delante de las e y a y la e está delan­te de la a ; por tanto, hay 4 inversiones.

DETERMINANTES DE ORDEN n. El símbolo

al b l el k l

a2 b2 e 2 k 2

a3 b3 e3 k3

a. b. e. .. . k.

formado por n2 números (llamados elementos) dispuestos en n filas y n columnas, representa el determinante de una matriz de orden n. Este símbolo es una forma abreviada de expresar el polino­mio constituido por la suma algebraica de todos los productos posibles, cada uno de n factores, de manera que :

1) En cada producto figura solamente un elemento de cada fila y uno solo de cada columna. Habrá, por tanto, n! productos.

2) El signo de cada producto es positivo o negativo, según que el número de inversiones de los subíndices sea par o impar, después de haber colocado las letras en el orden en el que figuran en la primera fila. .

La suma algebraica así obtenida se denomina desarrollo o valor del determinante. Cada produc­to, con su signo, recibe el nombre de término del desarrollo del determinante.

Un determinante de orden n se representa también por

a ll a12 a l3 al.

a21 a22 a23 a2•

a 31 a32 a33 a3.

Con esta notación, cada elemento se caracteriza por dos subíndices, el primero indica la fila y el

260

http://carlos2524.jimdo.com/

Page 269: Algebra Superior Murray R Spiegel

DETERMINANTES DE ORDEN n 261

segundo la columna a las que pertenece o en donde se encuentra. Así, pues, a23 es el elemento dela segunda fila y tercera columna; a32 es el correspondiente de la tercera fila y segunda columna.

La diagonal principal de un determinante está formada por los elementos de la matriz situa-dos sobre la recta que une el primer elemento de la primera fila con el último de la última fila.

PROPIEDADES DE LOS DETERMINANTES

1. Si se intercambian las filas por las columnas en un determinante, su valor no se modifica.Por consiguiente, todo teorema que se demuestre para las filas, se verificará también paralas columnas, y recíprocamente. De aquí en adelante, con la denominación línea significa-remos una fila o una columna.

Ejemplo.

11. Si todos los elementos de una línea (fila o columna) son nulos, el determinante vale cero.

Ejemplo.

IlI. Si se permutan dos líneas (filas o columnas), el determinante cambia de signo.

Ejemplo.

IV. Si un determinante tiene dos líneas (filas o columnas) iguales, su valor es cero.

Ejemplo.

V. Si todos los elementos de una línea (fila o columna) de un determinante se multiplican porun mismo número p, el valor del determinante queda multiplicado por p.

Ejemplo.

VI. Si todos los elementos de una línea (fila o columna) de un determinante son suma de dos(o más) términos, el determinante es igual a la suma de dos (o más) determinantes.

Ejemplo. Ial + al' b,al + al' h2

a , + a3' b ,

DETERMINANTES DE ORDEN n 261

segundo la columna a las que pertenece o en donde se encuentra. Así, pues, a23 es el elemento de la segunda fila y tercera columna; a32 es el correspondiente de la tercera fila y segunda columna.

La diagonal principal de un determinante está formada por los elementos de la matriz situa­dos sobre la recta que une el primer elemento de la primera fila con el último de la última fila.

PROPIEDADES DE LOS DETERMINANTES

1. Si se intercambian las filas por las columnas en un determinante, su valor no se modifica. Por consiguiente, todo teorema que se demuestre para las filas, se verificará también para las columnas, y recíprocamente. De aquí en adelante, con la denominación línea significa­remos una fila o una columna.

Ejemplo.

I

al = b l

c l

11. Si todos los elementos de una línea (fila o columna) son nulos, el determinante vale cero.

Ejemplo.

IlI . Si se permutan dos líneas (filas o columnas), el determinante cambia de signo.

Ejemplo.

IV. Si un determinante tiene dos líneas (filas o columnas) iguales, su valor es cero.

Ejemplo.

V. Si todos los elementos de una línea (fila o columna) de un determinante se multiplican por un mismo número p , el valor del determinante queda multiplicado por p .

Ejemplo.

VI. Si todos los elementos de una línea (fila o columna) de un determinante son suma de dos (o más) términos, el determinante es igual a la suma de dos (o más) determinantes.

Ejemplo. I al + al ' b l

al + a l' h 2

a 3 + a3 ' b3

http://carlos2524.jimdo.com/

Page 270: Algebra Superior Murray R Spiegel

262 DETERMINANTES DE ORDEN n

VII. Si todos los elementos de una línea (fila o columna) de un determinante se suman con loselementos correspondientes de otra multiplicados por un número m, el valor del determi-nante no varía.

Ejemplo.al + mb, b, el I I al b, el Ia2 + mb¿ b2 e2 = a2 b2 e2

a3 + mb ; b3 e3 a3 b3 e3

Estas propiedades se pueden demostrar en los casos particulares de los determinantes de se-gundo y tercer orden aplicando, simplemente, los desarrollos dados en el Capítulo 26. En los Pro-blemas, se pueden ver las demostraciones de los casos generales.

MENOR COMPLEMENTARIO de un elemento de un determinante de orden n es el determinan-te de orden n - 1 obtenido al suprimir la fila y la columna a las que pertenece el elemento en cuestión.

Por ejemplo, el menor de b3 en el determinante de cuarto orden

se obtiene suprimiendo la fila y la columna a las que pertenece b3, resultando el determinante detercer orden

El menor complementario de un elemento se representa por una letra mayúscula. Por ejem-plo, el menor correspondiente al elemento b3 se representa por E3.

DESARROLLO DE UN DETERMINANTE POR LOS ELEMENTOS DE UNA LINEA. Seobtiene en función de los menores complementarios como sigue:

1) Se elige una línea (fila o columna) cualquiera.

2) Se multiplica cada elemento de la línea por su menor complementario precedido del signomás o menos, según que la suma de los números de la fila y columna a que pertenezca el ele-mento sea par o impar, respectivamente. El menor complementario de un elemento afectadode este signo se llama adjunto de dicho elemento.

3) Se suman algebraicamente los productos obtenidos en 2).

Por ejemplo, desarrollemos el determinante

al bl el dia2 b2 e2 d2a3 b3 e3 d3a4 b4 e4 d4

por los elementos de la tercera fila. Los menores complementarios de a3, b3, e3 Y d3 son A3' E3' C3YD3' respectivamente. El signo correspondiente al elemento a3 es +, ya que pertenece a la l.a colum-na y 3.a fila y 1 + 3 = 4 es un número par. Análogamente, los correspondientes a los elementosb3, e3 Y d3 son -, +, -, respectivamente. Por tanto, el valor del determinante es

qnSI

LA Rel

L

R(¡

S1

s

S(

Ir

ECW

y

UN S

2

http://carlos2524.jimdo.com/

Page 271: Algebra Superior Murray R Spiegel

DETERMINANTES DE ORDEN n

La propiedad VII se utiliza mucho para obtener ceros en una línea de un determinante cual-quiera. Esta propiedad, junto con el desarrollo por los elementos de una línea en función de losmenores complementarios, facílita el cálculo del valor de un determinante cualquiera de ordensuperior al tercero.

LA REGLA DE CRAMER para la resolución de un sistema de n ecuaciones lineales con n in-cógnitas es totalmente análoga a la dada en el capítulo anterior en los casos n = 2 Y n = 3.

Sea el sistema de n ecuaciones lineales con n incógnitas, XI' X2, X3, ... , x.,

a¡¡x¡ + a12x2 + al3x3 + + al.x. = rla21xI + a22x2 + a23x3 + + a2.x. = r2

Llamemos A al determinante de los coeficientes de XI' X2, X3, ... , X., es decir,

all a12 al3

a21 a22 a2)

Representando por Al el determinante A en el que la columna k (que corresponde a los coeficien-tes de la incógnita Xt) se ha remplazado por la columna de los términos independientes de (1) pa-sados al segundo miembro, se tiene

siempre que A i= OXI

Si A i= O, existe solución y ésta es única.

Si A = O, el sistema de ecuaciones puede o no tener solución.

Los sistemas de ecuaciones que carecen de solución se llaman incompatibles; en caso contrarioson compatibles. Si A = O Y por lo menos uno de los determinantes Al, A2, ... , A. i= O, el siste-ma es incompatible. Si A = Al = A2 = ... = A., el sistema puede o no ser compatible.

Los sistemas que tienen un número infinito de soluciones reciben el nombre de indetermina-dos. Si un sistema de ecuaciones es indeterminado, A = O Ytodos los determinantes Al' A2, A3, ... ,

A. = O. El recíproco, sin embargo, no siempre es cierto.

ECUACIONES LINEALES HOMOGENEAS. Si todos los términos independientes, '1' '2' ... ,r.,de lasecuaciones (l)son nulos, el sistema se llama homogéneo. En este caso, Al = A2 = ... = A. = OY se verifica:

Teorema. La condición necesaria y suficiente para que un sistema de n ecuaciones linealeshomogéneas con n incógnitas tenga solución distinta de la trivial (todas las incógnitas iguales acero) es que el determinante de los coeficientes sea nulo, es decir, A = O.

UN SISTEMA DE m ECUACIONES CON n INCOGNITAS puede o no tener solución.

1) Si m > n, se puede obtener el valor de n de las incógnitas dadas. Si estos valores satisfacena las m-n ecuaciones restantes, el sistema es compatible, y en caso contrario es incompatible.

2) Si m < n, se pueden determinar m de las incógnitas en función de las n-m restantes.

263

(1)

DETERMINANTES DE ORDEN n 263

La propiedad VII se utiliza mucho para obtener ceros en una línea de un determinante cual­quiera. Esta propiedad, junto con el desarrollo por los elementos de una línea en función de los menores complementarios, facílita el cálculo del valor de un determinante cualquiera de orden superior al tercero.

LA REGLA DE CRAMER para la resolución de un sistema de n ecuaciones lineales con n in­cógnitas es totalmente análoga a la dada en el capítulo anterior en los casos n = 2 Y n = 3.

Sea el sistema de n ecuaciones lineales con n incógnitas, XI' X 2 , X 3 , ... , x. ,

a¡¡x¡ + a12x2 + a13x3 + ... + al.x. = r l

a21 x I + a22x2 + a23x3 + ... + a2.x. = r2

Llamemos A al determinante de los coeficientes de XI' X 2 , X3, ... , X., es decir,

all a12 al3 ... al. a21 a22 an .. , a2•

(1)

Representando por Al el determinante A en el que la columna k (que corresponde a los coeficien­tes de la incógnita Xt) se ha remplazado por la columna de los términos independientes de (1) pa­sados al segundo miembro, se tiene

XI siempre que A i= O

Si A i= O, existe solución y ésta es única.

Si A = O, el sistema de ecuaciones puede o no tener solución.

Los sistemas de ecuaciones que carecen de solución se llaman incompatibles; en caso contrario son compatibles. Si A = O Y por lo menos uno de los determinantes Al, A2 , ... , A. i= O, el siste­ma es incompatible. Si A = Al = A2 = . .. = A., el sistema puede o no ser compatible.

Los sistemas que tienen un número infinito de soluciones reciben el nombre de indetermina­dos. Si un sistema de ecuaciones es indeterminado, A = O Y todos los determinantes Al ' A2 , A3 , ... ,

A. = O. El recíproco, sin embargo, no siempre es cierto.

ECUACIONES LINEALES HOMOGENEAS. Si todos los términos independientes, '1 ' '2 ' . .. , r., de las ecuaciones (1 )son nulos, el sistema se llama homogéneo. En este caso, Al = A2 = ... = A. = O Y se verifica:

Teorema. La condición necesaria y suficiente para que un sistema de n ecuaciones lineales homogéneas con n incógnitas tenga solución distinta de la trivial (todas las incógnitas iguales a cero) es que el determinante de los coeficientes sea nulo, es decir, A = O.

UN SISTEMA DE m ECUACIONES CON n INCOGNITAS puede o no tener solución.

1) Si m > n, se puede obtener el valor de n de las incógnitas dadas. Si estos valores satisfacen a las m-n ecuaciones restantes, el sistema es compatible, y en caso contrario es incompatible.

2) Si m < n, se pueden determinar m de las incógnitas en función de las n-m restantes.

http://carlos2524.jimdo.com/

Page 272: Algebra Superior Murray R Spiegel

264 DETERMINANTES DE ORDEN 11

PROBLEMAS RESUELTOS

INVERSIONES

l. Hallar el número de inversiones en los casos siguientes:

a) 3, 1, 2

b) 4, 2, 3,

e) 5, 1, 4, 3, 2

d) b, a, C

e) b, a, e, d, C

2. a) ¿Cuál sería elfabético?

3 precede a l y 2. Hay 2 inversiones.

4 precede a 2, 3, 1; 2 precede al; 3 precede a l. Hay 5 inversiones.

5 precede a 1, 4, 3, 2; 4 precede a 3, 2; 3 precede a 2. Hay 7 inversiones.

b precede a a. Hay 1 inversión.

b precede a a; e precede a d, c; d precede a c. Hay 4 inversiones.

número de inversiones de los subíndices de b¡d3c2a4 si las letras se colocaran en orden al-

b) ¿Cuál sería el número de inversiones de las letras b¡dJc2a4 si los subíndices se colocaran en orden na-tural?

a) Escribiremos a4blc2d3• La disposición de los subíndices 4, 1, 2, 3 presenta 3 inversiones.

b) Escribiremos b¡c2d3a4. La disposición de las letras bcda presenta 3 inversiones.

El desarrollo consta de términos de la forma abc con todas las permutaciones posibles de los subíndices, siendomero de inversiones con respecto a los subíndices cuando se colocan las letras en orden alfabético que el de inver-siones con respecto a las letras cuando los subíndices se colocan en orden natural.

3. Desarrollar el determinante 1:: :: ~:I haciendo uso de las inversiones.a3 bJ CJ

El desarrollo consta de términos de la forma abc con todas las permutaciones posibles de los subíndices, siendoel signo de cada producto positivo o negativo según que el número de inversiones de los subíndices sea par o im-par, respectivamente.

El desarrollo consta de 6 términos que son:

a¡b2cJ

a¡b3c2a2b¡ C3

a2b3c¡

a3b2c¡a3b¡ c2

2 inversiones, signo +3 inversiones, signo -2 inversiones, signo +

o inversiones, signo +1 inversión, signo-1 inversión, signo-

4. Determinar los signos de los términos d¡a3c2b4 y a3c2d4b¡ del desarrollo del determinante

a¡ b, cI d¡a2 b2 c2 d2a3 b3 e3 d3a4 b¿ C4 d¿

El término d¡a3c2b4 escrito en la forma a3b4c2d¡ presenta 5 inversiones en los subíndices. Su signo es -.

El término a3c2d4b¡ escrito en la forma a3blc2d4 presenta 2 inversiones en los subíndices. Su signo es +.

PRO5. 1

6. I

7.

8.

9.

http://carlos2524.jimdo.com/

Page 273: Algebra Superior Murray R Spiegel

DETERMINANTES DE ORDEN n 265

PROPIEDADES DE LOS DETERMINANTES

5. Demostrar la Propiedad Ill. Si se permutan dos líneas el determinante cambia de signo.

Caso J. Las líneas son contiguas.

Si se intercambian dos líneas contiguas, también se intercambian dos subíndices en cada uno de los términosdel desarrollo. El número de inversiones de los subíndices aumentará o disminuirá en una unidad. Por tanto, lostérminos cambian de signo y, en consecuencia, cambiará el signo del determinante.

Caso 2. Las líneas no son contiguas.

al-

Supongamos que entre las dos líneas que se van a permutar existen k líneas. Para pasar una de las líneas allugar de la otra habrá que intercambiar dicha línea k veces con las contiguas hasta llegar a la inmediata an-terior, una más para permutarla con ésta y, luego, habrá que efectuar k intercambios de la otra línea para llevar-la hasta el lugar que ocupaba la primera. Esto significa un total de k + 1 + k = 2k + 1 intercambios. Como2k + 1 es siempre un número impar, hay un número impar de cambios de signo y, por tanto, el determinante cam-bia de signo.

na- 6. Demostrar la Propiedad IV: Si dos líneas son iguales el determinante es nulo.

Sea D el valor del determinante. Si se permutan las dos líneas iguales, el determinante continúa siendo el mismoy, sin embargo, por la Propiedad Hl, cambia de signo; por tanto, D = -D, es decir, D = O.

ndover-

7. Demostrar la Propiedad V: Si se multiplican los elementos de una línea por un mismo número p, el valor del de-terminante queda multiplicado por p.

Cada término del desarrollo contiene un solo elemento de la línea que se ha multiplicado por p y, por tanto,en cada término figurará el factor p. Al ser este factor común a todos los términos, el determinante queda multipli-cado por p.

8. Demostrar la Propiedad VI: Si cada elemento de una línea de un determinante son suma de dos (o más) térmi-nos, el determinante es igual a la suma de dos (o más) determinantes.

ndoirn- En el caso de determinantes de tercer orden, tendremos que demostrar que

a, + a,' b, e, a, b, e, a,' b, e,

a2 + a2' b2 e2 a2 b2 e2 + a2' b2 e2

a3 + a3' b3 e3 a3 b3 e3 a3' b3 e3

-Cada término del desarrollo del determinante del primer miembro es igual a la suma de los términos corres-pondientes de los determinantes del segundo miembro, por ejemplo, (a2 + a2')b3e, = a2b3e, + a2'b3e,. Obser-vamos, pues, que la propiedad se cumple en los determinantes de tercer orden. Este método de demostración se puedeaplicar también al caso general.

9. Demostrar la Propiedad VII: Si todos los elementos de una línea de un determinante se suman con los elementoscorrespondientes de otra multiplicados por un número m, el valor del determinante no varía.

En el caso de un determinante de tercer orden, tendremos que demostrar que

la, + mb ; b, e,a2 + mb2 b2 e2a3 + mb, b, e3

http://carlos2524.jimdo.com/

Page 274: Algebra Superior Murray R Spiegel

266 DETERMINANTES DE ORDEN n

Según la Propiedad VI,

Este último determinante se puede escribir m que es igual a cero, según la Propiedad IV.

10. Demostrar que = O.

369

12

25-3

2

2468

1-2-5

7

El número 3 se puede sacar factor común de los elementos de la primera columna, y el 2 de los elementosde la tercera columna, con lo cual,

25-3

2

1 12 -23 -54 7

1

(3)(2) ;

4

que es igual a cero, ya que la primera y tercera columnas son iguales.

11. Aplicando la Propiedad VII, transformar el determinante I ;-2

-2-1

3!I en otro de igual valor que ten-1 ,

ga nulos 10s elementos de la segunda y tercera columnas pertenecientes a la primera fila.

Multiplicando por 2 los elementos de la primera columna y sumándoles los correspondientes de la segun-da columna se obtiene,

12-2

(2) (1) - 2(2) (2) - 1(2) (-2) + 3

!1= \ ;1 -2

o3

-1

Multiplicando los elementos de la primera columna del nuevo determinante por - 3 y sumándoles los co-rrespondientes de la tercera columna se obtiene.

\j O (-3) (1)+3\ \ 1 O O3 (- 3) (2) + 4 = 2 3 -2

-1 (-3) (-2) + 1 -2 -1 7

El resultado se podria haber obtenido, directamente, escribiendo

\j (2) (1) - 2 (-3) (1)+3\ \ 1 O

-~ \(2) (2) - I (-3) (2)+ 4 = 2 3(2)(-2)+3 (-3) (-2) + 1 -2 -1

Se han elegido los números 2 y - 3 para obtener los ceros en los lugares deseados.

12. Aplicando la Propiedad VII transformar el determinante en otro de igual valor que

tenga tres ceros en la cuarta fila.

13.

14.

DES!

15.

http://carlos2524.jimdo.com/

Page 275: Algebra Superior Murray R Spiegel

DETERMINANTES DE ORDEN n 267

Multiplicando cada elemento de la primera columna (la columna base es la sombreada) por -3, -4, +2,Y sumándoles, respectivamente, los correspondientes de las segunda, tercera y cuarta columnas, se obtiene

1,(-3) (3) + 6~"~ (-3) (-2) + 1

(-3) (4) - 5(-3) (1)+-3

(-4) (3) + 2(-4) (-2) - 2(-4) (4) + 1(-4) (1)+4

(2) (3) + 3(2) (-2) + 2(2) (4) + 4(2) (1) - 2

3 -3-2 7

4 -171 O

-10 96 -2

-15 12O O

IV.Obsérvese que es más conveniente elegir una línea base que contenga al elemento l.

13. Obtener 4 ceros en una línea del determinante de quinto orden :~;IJ.~J.f{~11Iffli~¡~lIllil~!fll6 -3 2 4 32 2 5 3-2

3 5-2 3

462234

tos

Elegimos como línea base la fila sombreada y los ceros en la segunda columna. Multiplicando los elemen-tos de esta fila base por - 5, - 3, 3, - 2 Ysumándoles, respectivamente, los elementos correspondientes de la pri-mera, segunda, cuarta y quinta, se obtiene

ten-

-17 O -11 16 17-14 O -7 9 13

.tf@~~ltnf¡_!ltlii_.llí18 O 11 -2 -6-6 O -1 7 4

14. Obtener 3 ceros en una línea del determinante

3 4-2 2

2 -34 5

2 33 -23 4

-2 -2

sin que se modifique su valor.un-

Es conveniente utilizar la Propiedad VIl para obtener un elemento igual a l en una línea. Por ejemplo. mul-tiplicando cada elemento de la columna 2 por - 1 Ysumándoles los correspondientes de la columna 3, se obtiene

co-3 4

-2 22 -34 5

3-24-2

-1O

14-10

8 -2O 1

-15 619 -7

-1O

16-16

que es igual al determinante dado.

Tomando la tercera columna como base y multiplicando sus elementos por 2, -2,2, sumándoles, respec-tivamente, los de la primera, segunda y cuarta columnas, se obtiene

DESARROLLO DE DETERMINANTES POR LOS ELEMENTOS DE UNA LINEA

15. Escribir el menor complementario y adjunto del elemento de la segunda fila y tercera columna del de-terminante

1 -2 -12 1 -2

que 2 - 2

DETERMINANTES DE ORDEN n 267

Multiplicando cada elemento de la primera columna (la columna base es la sombreada) por -3, -4, +2, Y sumándoles, respectivamente, los correspondientes de las segunda, tercera y cuarta columnas, se obtiene

I (-3) (3)+6

~ . (-3) (-2)+ 1 (-3) (4) - 5 (-3) (1) +-3

(-4) (3) + 2 (-4) (-2) - 2 (-4) (4) + 1 (-4) (1)+4

(2) (3) + 3 (2) (-2) + 2 (2) (4) + 4 (2) (1) - 2

3 -3 -2 7

4 -17 O

-JO 9 6 -2

-15 12 O O

Obsérvese que es más conveniente elegir una línea base que contenga al elemento l .

13, Obtener 4 ceros en una línea del determinante de quinto orden

3 5 -2 3

462 2 3 4

:~;IJ.~J.f~\I{~It.~li~1¡I~.il~¡~.1 6 -3 2 4 3 2 2 5 3-2

Elegimos como línea base la fila sombreada y los ceros en la segunda columna. Multiplicando los elemen­tos de esta fila base por - 5, - 3, 3, - 2 Y sumándoles, respectivamente, los elementos correspondientes de la pri­mera, segunda, cuarta y quinta, se obtiene

-17 O -11 16 17 -14 O -7 9 13

1i,¡rl~~l,n~tl¡_illl&~G_Ñ 18 O 11 -2 -6 -6 O -1 7 4

342 3

14. Obtener 3 ceros en una línea del determinante -2 2 3-2

2 -3 3 4 4 5 -2 -2

sin que se modifique su valor.

Es conveniente utilizar la Propiedad Vil para obtener un elemento igual a 1 en una línea. Por ejemplo. mul­tiplicando cada elemento de la columna 2 por - 1 Y sumándoles los correspondientes de la columna 3, se obtiene

3 4 -2 2

2 -3 4 5

3 -2

4 -2

Tomando la tercera columna como base y multiplicando sus elementos por 2, -2,2, sumándoles. respec­tivamente, los de la primera, segunda y cuarta columnas, se obtiene

-1 O

14 -10

8 -2 O 1

-15 6 19 -7

-1 O

16 -16

que es igual al determinante dado.

DESARROLLO DE DETERMINANTES POR LOS ELEMENTOS DE UNA LINEA

15. Escribir el menor complemen~ario y adjunto del elemento de la segunda fila y tercera columna del de­terminante

2 - 2

1 -2 - 1 2 -2

http://carlos2524.jimdo.com/

Page 276: Algebra Superior Murray R Spiegel

268 DETERMINANTES DE ORDEN n

Suprimiendo la fila y columna que contienen al elemento, el menor complementario es

1 ~

-2-2

1

Como el elemento pertenece a la segunda fila y tercera columna, y 2 + 3 = 5 es un número impar, el signoasociado a él será menos. El adjunto es, pues,

-1 ~-2-2

1

16. Escribir los menores complementarios y adjuntos de los elementos de la cuarta fila del determinante

3 -22 11 53 -2

4 25 -3

-2 2-4 1

Los elementos de la cuarta fila son - 3, -2, -4, 1.

Menor del elemento -3,= I-! -1 -~ Adjunto = - Menor

18,

I3 4 2

Menor del elemento - 2 = 2 5 -3 Adjunto = + Menor 19.

1 -2 2

I3 -2 -tiMenor del elemento -4 = 2 1 Adjunto = - Menor1 5

=13 -2 -11Menor del elemento J 2 1 Adjunto = + Menor1 5

17. Desarrollar el determinante del Problema 16 por los elementos de la cuarta fila.

Valor del determinante = suma de los elementos de una línea multiplicados por sus adjuntos

~ (-3) ¡-¡-: _~-ll ¡+ (-2) H¡j -:1¡+ (-4) ¡-1¡ -¡ -:1l(1) H¡ -¡ J ¡

Después de desarrollar cada uno de los determinantes de tercer orden, el resultado obtenido es - 53.

El método de cálculo aquí indicado es largo. Se puede, sin embargo, simplificar mucho transformando pre-viamente el determinante dado en otro equivalente que tenga ceros en una línea (fila o columna) aplicando laPropiedad VII, como se hace en el problema siguiente.

http://carlos2524.jimdo.com/

Page 277: Algebra Superior Murray R Spiegel

DETERMINANTES DE ORDEN n 269

18. Desarrollar el determinante del Problema 16 por los elementos de una línea, transformándole previamente enotro de igual valor que tenga tres ceros en una línea.

Eligiendo la columna base indicada.

2-3

2 •

1

multiplicando sus elementos por - 2. - 5. 3

o

y sumándoles, respectivamente, a las correspondientes de la primera, tercera y cuarta columnas, se obtiene

7 -2O 1

-9 51 -2

14O

-276

-4O

17-5

Desarrollando por los elementos de la segunda fila se obtiene,

(O)(su adjunto) + (l)(su adjunto) + (O)(su adjunto) + (O)(su adjunto)

¡ I9 14 -41 )= (I)(su adjunto) = 1 + -~ -2~ ~~

Desarrollando este determinante, resulta el valor -53, que coincida con el obtenido en el Problema 16.

Obsérvese que por este método se puede desarrollar un determinante de tercer orden en función de deter-minantes de segundo orden.

19. Hallar el valor de los determinantes siguientes

a)2 1 4

3 -1 3 42 3 -3 2

4 1 -2 3-9 O 5 -2

7 O 1 7-10 O 3 -7

Multiplicando los elementos de la base indicada por -2, 1, -3 Ysumándoles, respectivamente, los correspondientes de la segunda, terce-ra y cuarta filas, se obtiene

{ /

-9 51 - 7 1

-10 3

-8 -3 -4 -6 13 ¡ l5 2 5 4 II -8 -4 -6 13 -8 -4 -6 13-pre- O 1 O O O 1 5 5 4 - 1 1 5 5 4 -11= - = - fltI.liwllt\lillo la -7 -3 -3 1 I1 -7 -3 1 1 1

-3 -2 -8 O 9 -3 -8 O 9 -3 -8 O 9:

1

-9- 7

-10-217 •-7

Multiplicando los elementos de la base indicadapor -7 y sumándoles los correspondientes de la pri-mera y tercera columnas, se obtiene

\

-44- O

-31

513

-37

1 {I0=-(1) + I =~-28

-37

1 }= -85-28

1 2 -3 1-1 1 2 -3

h) -3 -2 -1 42 3 4 -13 -4 2 1

Multiplicando los elementos de la columna base indicada por 3, 2, 1,-4, respectivamente, y sumándoles los correspondientes de la primera,tercera, cuarta y quinta columnas, se obtiene

http://carlos2524.jimdo.com/

Page 278: Algebra Superior Murray R Spiegel

270

24.

DETERMINANTES DE ORD,:lN n

Multiplicando en el último determinante los elementos de la fila base indicada por 6, -4 Ysumándoles, res-pectivamente, los elementos de la primera y segunda filas, se obtiene

-50 -22 O 79

{ 1-50 -22 79

1 }= \-SO -22 79\33 17 O -55-(1) + ~; 17 -5~ - 33 17 -55 22.

-3 =-7 1 11 -8 ' %\"'" .".mI-3 -8 O 9

-.MI;¡lI

Multiplicando por 2 los elementos de la fila indicada del último determinante y sumándoles, respectivamen-te, los de la segunda fila, se obtiene

\

-50 -22 79\,Wi"" ,,<,- ~,:~.~ .. ".~,.H~-3 -8 9

Multiplicando los elementos de la fila indicada del último determinante por 22 y 8 Y sumándoles, respecti-vamente, los correspondientes de la primera y tercera filas, se obtiene

1

54427

213-735

1-37 = -(1)-287

SIST

O1

O { 1544 - 7351 }

+ 213 -287 -427

20. Descomponer en factores el determinante siguiente.

Sacando del determinante los factores x e yde la primera y segunda columnas, respectivamente,

23.

= xy x y

Xl yl

=XYIX~1O

: Iy - 1Xl - 1 yl _ 1

Sumando los elementos de la tercera colum-na multiplicados por - 1 con los correspondientesde la primera y segunda columnas,

Ix - 1

=xYxl_ly - 1 Iyl- 1

= xy(x _ 1)(y _ 1)\ 1x + 1

= xy(x - 1)(y - 1)(y - x).

1 Iy + 11

Sacando del determinante los factores(x - 1) e (y - 1) de la primera y segundacolumnas, respectivamente,

SISTEMAS DE ECUACIONES LINEALES. REGLA DE CRAMER

2x+ y- z+ w = -4 2 1 -1 121. Resolver el sistema x + 2y + 2z - 3w = 6 ~= 1 2 2 -3

= 863x - y- z+2w=0 3 -1 -1 22x + 3y + z+4w=-5 2 3 1 4

-4 1 -1 1 2 -4 -1 1~1=

6 2 2 -3= 86 ~l = 1 6 2 -3 -172

O -1 -1 2 3 O -1 2-5 3 1 4 2 -5 1 4

2 1 -4 1 2 1 -1 -4 25. 1

~3'=1 2 6 -3

= 258 ~••=1 2 2 6 -86

3 -1 O 2 3 -1 -'-1 O2 3 -5 4 2 3 1 -5

http://carlos2524.jimdo.com/

Page 279: Algebra Superior Murray R Spiegel

DETERMINANTES DE ORDEN n 271

Luego x = ~, = 1~ ,~2

Y = ~ = -2,~3

Z = -¡; = 3, w=~4=_1~

22. Las intensidades, en amperios, i" i2, i3, i4, is, i6, de las corrie-i.es en un circuito de seis mallas, satisfacen las ecua-ciones siguientes. Hallar el valor de i3.

i, - 2i2 + i3 = 3i2 + 3i4 - is = -5t, + i2 + i3 - is = 12i2 + i3 - 2i4 - 2is = Oi, + i3 + 2i4 + is = 3

1 -2 3 O O 1 -2 1 O OO 1 -5 3 -1 O 1 O 3 -1

~3 = 1 l 1 O -1 = 38, ~= 1 1 1 O -1 = 19,O 2 O -2 -2 O 2 1 -2 -2l O 3 2 1 1 O 1 2 1

SISTEMAS DE ECUACIONES LINEALES COMPATIBLES E INCOMPATIBLES

yx - 3y + 2z = 4

23. Determinar el sistema 2x + y - 3z = -2 es compatible.4x - 5y + z = 5e,

~ = I~-2

~ 1=0 ~,=I ~

-2

~ 1 = O-1 -1-1 -1

~2 = I~

8

~ 1=0 ~3 = I~-2 ~I= O5 -14 -1

Como por lo menos uno de los determinantes, ~" ~2' ~3 -+ O, el sistema es incompatible.

Esto se puede comprobar multiplicando la primera ecuación por 2 y sumándola a la segunda, con que seobtiene 4x - 5y + z = 6 que es incompatible con la última ecuación.

sda

4x - 2y + 6z = 824. Determinar si el sistema 2x - y + 3z = 5 es compatible.

2x - y + 3z = 4

Con los resultados obtenidos hasta ahora, nada se puede decir sobre la compatíbilidad del sistema; sin em-bargo, observando el sistema con más detenimiento, se deduce que las dos últimas ecuaciones son incompatibles.Por tanto, el sistema es incompatible.

2x+ y-2z=425. Determinar si el sistema x - 2y + z = - 2 es compatible.

5x - 5y + z = -2

~ = ~, = ~2 = ~3 = O. Nada se puede decir, por ahora, sobre la compatibilidad del sistema.

http://carlos2524.jimdo.com/

Page 280: Algebra Superior Murray R Spiegel

272

38.

DETERMINANTES DE ORDEN 11

Despejando x e y en función de z en las dos primeras ecuaciones, x = ~(;:+ 2), Y = ~(= + 2).5 5Estos valores, sustituidos en la tercera ecuación, la satisfacen. Si no fuera así, el sistema sería incompatible.

29.3 4

Por tanto. los valores x = 5(z + 2), Y = 5(z + 2). satisfacen al sistema, con lo que existen infinitas so-

luciones. obtenidas todas ellas dando valores a z: Por ejemplo. si z = 3, se tiene x = 3, Y = 4; si z = -2. re-sultan x = O, Y = O. etc.

30.

En este caso, las ecuaciones dadas son dependientes. Se puede comprobar multiplicando la segunda ecuaciónpor 3 y sumándola a la primera; se obtiene 5x - 5y + z = -2, que es la tercera ecuación. 31.

32.SISTEMAS DE ECUACIONES LINEALES HOMOGENEAS

2x - 3y + 4z = O26. Determinar si el sistema x + y - 2z = O tiene soluciones distintas de la trivial x = y = z = O.

3x + 2y - 3z = O

-3I2

-~ 1= -17-3

33.

Como ~ f. O Y ~l = ~2 = ~3 = O, el sistema tiene solamente la solución trivial.34.

x + 3y - 2z = O27. Resolver el sistema 2x - 4y + z = O

x+ y- z=O

~l = ~2 = ~3 = O35.

Por tanto, existen soluciones distintas de la trivial.

Para hallar las soluciones despejamos x e y, entre las dos últimas ecuaciones, en función de z (esto no siem-pre es posible) y se obtiene x = z/2, y = z/2. Estos valores satisfacen la tercera ecuación, con lo cual, hay infini-tas soluciones que resultan al dar valores a z, Por ejemplo, si z = 6, x = 3 e y = 3; si z = -4, x = -2 ey = -2; etc.

36.

x + 2y + kz = O28. Hallar los valores de k para los cuales el sistema 2x + ky + 2z = O tiene soluciones distintas de la trivial.

3x + y + z = O37.

Las soluciones no triviales se tienen cuando ~ = I i ~ 1= O

2kI

Luego tl = - 3k2 + 3k + 6 = O de donde k = - 1, 2.

272 DETERMINANTES DE ORDEN 11

Despejando x e y en función de z en las dos primeras ecuaciones, x = ~(;: + 2), Ji = ~(= + 2). 5 5

Estos valores, sustituidos en la tercera ecuación, la satisfacen. Si no fuera así, el sistema sería incompatible.

3 4 Por tanto. los valores x = 5(;: + 2), Y = 5(z + 2), satisfacen al sistema, con lo que existen infinitas so-

luciones, obtenidas todas ellas dando valores a z. Por ejemplo, si z = 3, se tiene x = 3, Y = 4; si z = -2, re­sultan x = O, Y = O, etc.

En este caso, las ecuaciones dadas son dependientes. Se puede comprobar multiplicando la segunda ecuación por 3 y sumándola a la primera; se obtiene 5x - 5y + z = -2, que es la tercera ecuación.

SISTEMAS DE ECUACIONES LINEALES HOMOGENEAS

2x - 3y + 4z = O 26. Determinar si el sistema x + y - 2z = O tiene soluciones dislintas de la trivial x = y = ;: = O.

3x + 2y - 3z = O

-3 1 2

-~ 1= -17 -3

~1 = ~2 = ~3 = O

Como ~ +- O Y ~1 = ~2 = ~3 = O, el sistema tiene solamente la solución trivial.

x + 3y - 2z = O 27. Resolver el sistema 2x - 4y + z = O

x+y-z=O

Por tanto, existen soluciones distintas de la trivial.

~1 = ~2 = ~3 = O

Para hallar las soluciones despejamos x e y, entre las dos últimas ecuaciones, en función de z (esto no siem­pre es posible) y se obtiene x = z/2, y = z/2. Estos valores satisfacen la tercera ecuación. con lo cual , hay infini­tas soluciones que resultan al dar valores a z. Por ejemplo, si z = 6, x = 3 e y = 3; si z = -4, x = -2 e y = -2; etc.

x + 2y + kz = O 28. Hallar los valores de k para los cuales el sistema 2x + ky + 2: = O tiene soluciones distintas de la trivial.

3x + y + ;: = O

Las soluciones no triviales se tienen cuando ~ = I i 2 k 1

Luego ~ = - 3k2 + 3k + 6 = O de donde k = - 1, 2.

~ 1= O

http://carlos2524.jimdo.com/

Page 281: Algebra Superior Murray R Spiegel

DETERMINANTES DE ORDEN n 273

y a continuación hallar su valor. desarrollándolo por los elementos de dicha fila.

38. Calcular el valor de los determinantes:

3 2 -1 3 22 -1 3 2 3 -1 2 1 1 2 -1 1 -2 O 3 4 3-3 1 2 4 4 2 O -3 -2 3 2 -1 d) 1 -3 -2 1 Oa) h) ell -3 -1 3 -2 l -3 2 3 -1 1 -42 4 1 O 1-1 2 -2 -3 1 3 -1 4 -1 4 -3 2 -1 -1 2 I O

PROBLEMAS PROPUESTOS

29. Hallar el número de inversiones que presentan cada una de las ordenaciones siguientes:a) 4, 3, 1, 2 b) 3, 1, 5, 4, 2 e) e, a, d, b, e

n

30. Dada la ordenación d3, b4, el> e2' a5, determinar.a) el número de inversiones de los subíndices colocadas las letras en orden alfabético.b) el número de inversiones de las letras colocados los subíndices en orden natural.

al b, el d,a2 b2 e2 d2

a3 b3 e3 d3

a4 b¿ e4 d¿

31. En el desarrollo de

32. a) Demostrar la Propiedad 1: Si se intercambian las filas por las columnas en un determinante, su valor nose modifica.

b) Demostrar la Propiedad 11: Si todos los elementos de una línea son nulos, el determinante valecero.

1 2 3 4

33. Demostrar que el determinante 2 4 6 3es igual a cero.3 8 12 2

4 16 24 1

-2 4 1 3

34. Transformar el determinante 1 -2 2 4en otro equivalente que tenga tres ceros en la ter-3 1 -3 2

4 3 -2 -1cera columna.

4 -2 1 3 1-2 1 -3 -2 -2

35. Transformar el determinante 3 4 2 1 3 en otro equivalente que tenga cuatro ceros en1 -3 4 -1 -12 -1 2 4 2

la cuarta columna.

-1 2 3 -236. Dado el determinante 4 -1 -2 2

1- -3 1 2 -12 4 -1 3

a) escribir los menores complementarios y los adjuntos de los elementos de la tercera fila.b) desarrollar el determinante por los elementos de una línea,el hallar el valor del determinante.

1.

37. Transformar el determinante

-2 l 2 33 -2 -3 21 2 1 24 3 -1 -3

en otro equivalente que tenga tres ceros en una fila

http://carlos2524.jimdo.com/

Page 282: Algebra Superior Murray R Spiegel

274 DETERMINANTES DE ORDEN n

1

b)x y Z

x2 y2 Z2

x3 y3 Z3

39. Descomponer en factores los determinantes: a) \ ~2 ~2 ~2 \

a3 h3 e3

40. Resolver los sistemas: a) !x - 2.1" + 2 - 3w = 42x + 3~'- z - 2w = -43x - 4)' + 2= - 4w = 122x - y - 3= + 2w = - 2 !2x ~ y - 3z = - 5

3y + 42 + u: = 5b) 2= _ w - 4x = O

w + 3x - y = 4

{;;: ~ ;;3_-2::: ~44+ 2i5 = O

41. Hallar t, e i4 en ,,1 sistema -:-il -3i2 + 2i4 + 3i5 = 211 + 2/3 - 15 = 92il + i2 = 5

42. Determinar si son compatibles. incompatibles o indeterminados los sistemas:

f2x - 3v + z = 1

a) x + 2.1' - z = 1,3x - y + 2= = 6

{

2X-V+==2b) 3x + 2)' - 4= = 1

x - 4)' + 6= = 3{

X + 3y - 2:: = 2e) 3x - y - z = 1

2x + 6y - 42 = 3{

2u + v - 3w = 1d) u - 2v - w = 2

u + 3v - 2w = -2

{

3x - 2;- + 4= = O43. Resolver el sistema 2x + y - 3z = O

x + 3y - 22 = O

!2x + ky + 2 + w = O3x + (k - I)y - 2= - w = Ox - 2y + 42 + 2w = O2x + y + Z + 2w = O

tiene soluciones distirÍ-44. Hallar los valores de k para los cuales el sistema

tas de la trivial.

SOLUCIONES DE LOS PROBLEMAS PROPUESTOS

29. a) 5

30. a) 8

b) 5

b) 8

e) 3

31. Y +. respectivamente

36. e) -38

37. 28

38. a) 38 b) -143 e) -108 d) 88

39. a) ahe(a - b)(b - e)le - a) b) (x - I)(y - 1)(z - l)(x - y)(}. - 2)(Z "" x)

40. a) x = 2, Y = -1. z = 3, u' = 1 b) x = 1. Y = -1, Z = 2, w = O

41. il = 3. i4 = -242. a) compatible b) indeterminado e) incompatible

43. solo tiene ·Ia solución trivial x = y = z = O

d) incompatible

44. k=-I

FRA

FRA

FRAI

TEO]~

http://carlos2524.jimdo.com/

Page 283: Algebra Superior Murray R Spiegel

CAPITULO 28

Fracciones simples

FRACCION POLINOMICA. Es el cociente ~~:~ de los polinomios de x.

· 3x2 - 1Por ejemplo, 3 7 2 4 es una fracción polinómica.x + x -

FRACCION PROPIA. Es aquella en la que el grado del polinomio numerador es inferior alcorrespondiente del polinomio denominador.

· 2x - 3 4x2 + 1Por ejemplo, 2 5 4 Y 4 3 son fracciones propiasx + x+ x - x

Fracción impropia es aquella en la que el grado del polinomio numerador es igualo superioral correspondiente del denominador

P . l 2X3 + 6x2- 9 fracci .. .or ejemp o, x2 _ 3x + 2 es una raCCIOnImpropia

Efectuando la división, toda fracción impropia siempre es posible expresarla como suma deun polinomio y una fracción propia.

P . l 2X3 + 6x2 - 9_ 2 12+ 32x - 33or eJemp o, x2 _ 3x + 2 - x + x2 _ 3x + 2

FRACCIONES SIMPLES. Toda fracción propia se puede, en general, expresar como suma deotras fracciones (fracciones simples) cuyos polinomios denominadores sean de grado inferior aldel denominador de la fracción dada.

· 3x - 5 3x - 5 . 2 1Por ejemplo, 2 3 2= (x 1)( 2)= --1 + --2x-x+ x- x- x- x-

TEOREMA FUNDAMENTAL. Para descomponer una fracción propia en fracciones simples seprocede de las formas siguientes, según los casos.

1) Divisores lineales distintos.

A cada divisor simple del polinomio denominador de la fracción dada del tipo ax + b, le co-

rresponde una fracción simple de la forma ~b ' siendo A una constante distinta de cero.ax +

x + 4 A BEjemplo. (x + 7)(2x - 1) = x + 7 + 2x - 1

275

CAPITULO 28

Fracciones simples

FRACCION POLINOMICA. Es el cociente ~~:~ de los polinomios de x .

3X2 - 1 Por ejemplo, 3 7 2 4 es una fracción polinómica.

x + x -

FRACCION PROPIA. Es aquella en la que el grado del polinomio numerador es inferior al correspondiente del polinomio denominador.

· 2x - 3 4X2 + 1 Por ejemplo, X2 + 5x + 4 Y x4 _ 3x son fracciones propias

Fracción impropia es aquella en la que el grado del polinomio numerador es igualo superior al correspondiente del denominador

· 1 2X 3 + 6X2 - 9 f' , . . Por eJemp o , 2 3 2 es una raCCIOn Impropia

x - x +

EfectUando la división, toda fracción impropia siempre es posible expresarla como suma de un polinomio y una fracción propia.

Por eJ'emplo, 2X 3 + 6X2 - 9 _ 2 + 12 + 32x - 33 X2 _ 3x + 2 - X X2 - 3x + 2

FRACCIONES SIMPLES. Toda fracción propia se puede, en general, expresar como suma de otras fracciones (fracciones simples) cuyos polinomios denominadores sean de grado inferior al del denominador de la fracción dada.

· 3x - 5 Por eJemplo,.2 3 2

.~ - x + 3x - 5 ' 2 1

-:---;-:-.,---- = = - - + --(x - l)(x - 2) x - 1 x - 2

TEOREMA FUNDAMENTAL. Para descomponer una fracción propia en fracciones simples se procede de las formas siguientes, segÚn los casos.

1) Divisores lineales distintos.

A cada divisor simple del polinomio denominador de la fracción dada del tipo ax + b, le co­

rresponde una fracción simple de la forma ~b ' siendo A una constante distinta de cero. ax +

x + 4 A B Ejemplo. (x + 7)(2x - 1) = x + 7 + 2x - 1

275

http://carlos2524.jimdo.com/

Page 284: Algebra Superior Murray R Spiegel

276 FRACCIONES SIMPLES

2) Divisores lineales múltiples.

A cada' divisor múltiple del polinomio denominador de la fracción dada del tipo ax + b ele-vado a la potencia p, le corresponden p fracciones simples de la forma

Al Az Apax + b + (ax + b)Z + ... + (ax + bY'

siendo Al' Az, ... , Ap constante y Ap '" O.

3x - 1 A BEjemplos. (x + 4)Z = x + 4 + (x + 4)Z

3) Divisores euadrátieos distintos.

A cada divisor simple del polinomio denominador de la fracción dada del tipo ax' + bx + e,le corresponde una fracción simple de la forma

Ax + Bax? + bx + e

siendo A Y B constantes que no son nulas simultáneamente.

Nota. Se supone que ax' + bx + e no se puede descomponer en producto de dos factoreslineales reales de coeficientes enteros

XZ - 3 A Bx + eEjemplos. = -- + ---

(x-2)(x2+4) x-2 xZ+4

2X3 - 6 A Bx + e' Dx + Ex(2x2 + 3x + 8)(x2 + X + 1) = x + 2X2 + 3x + 8 + -X;:'2..:.+~x..:.+=--

4) Divisores euadrátieos múltiples.

A cada divisor del polinomio denominador de la fracción dada del tipo ax? + bx + e eleva-do a la potencia p, le corresponden p fracciones simples de la forma

Alx + BI + A2x + B2 Apx + Bpax' + bx + e (axZ + bx + e)2 + .. , + (a.xz + bx + e)P

siendo Al' BI, Az, Bz, ... , AP, Bp constante y Ap, Bp no nulas simultáneamente.

Ejemplo.x2 - 4x + 1 Ax + B ex + D Ex + F

-:-(.\~.2¡--+-\:-:)T2),,-;(.~\z;--+-.-\-+---;-l)= -x-z-+-\ = (XZ + \)2 + 7-+-.-\ -+-\

1. •

2.

3. -(

Res

276 FRACCIONES SIMPLES

2) Divisores lineales múltiples.

A cada ' divisor múltiple del polinomio denominador de la fracción dada del tipo ax + b ele­vado a la potencia p, le corresponden p fracciones simples de la forma

Al A 2 Ap ax + b + (ax + b)2 + .. . + (ax + bY'

siendo Al' A 2 , • .. , Ap constante y Ap =1= o.

Ejemplos. 3x - 1 A B

(x + 4)2 = X + 4 + (x + 4)2

5X2 - 2 A B e D E 3 2=3+2"+-+ 2+ - -x (x + 1) x x x (x + 1) x + 1

3) Divisores cuadráticos distintos.

A cada divisor simple del polinomio denominador de la fracción dada del tipo ax 2 + bx + e, le corresponde una fracción simple de la forma

Ax + B

ax2 + bx + e

siendo A Y B constantes que no son nulas simultáneamente.

Nota . Se supone que ax 2 + bx + e no se puede descomponer en producto de dos factores lineales reales de coeficientes enteros

X2 - 3 A Bx + e Ejemplos. (x _ 2)(x2 + 4) = x - 2 + X2 + 4

2X 3 - 6 A Bx + e' Dx + E x(2x2 + 3x + 8 )(x2 + X + 1) = x + 2X2 + 3x + 8 + X2 + X +

4) Divisores cuadráticos múltiples.

A cada divisor del polinomio denominador de la fracción dada del tipo ax 2 + bx + e eleva­do a la potencia p , le corresponden p fracciones simples de la forma

Alx + BI A 2x + B 2 Apx+Bp ax2 + bx + e + (ax2 + bx + C)2 + . . . + (a.x2 + bx + c)P

siendo Al ' B I , A 2 , B 2 • .. . , AP, Bp constante y Ap, Bp no nulas simultáneamente.

Ejemplo. X2 - 4x + 1 Ax + B ex + D Ex + F

-:-(x1.2-+---CI;-:)"2)-"(x~2¡--+-.-x -+---:-I)= -x-2-+-1 = (x 2 + 1)2 + 7 -+- .-x-+- 1

http://carlos2524.jimdo.com/

Page 285: Algebra Superior Murray R Spiegel

FRACCIONES SIMPLES 277

PROBLEMAS RESUELTOS

. x+2l. Recomponer en fracciones 2 2 o

X - 7x - 15x +2

(2x + 3)(x - 5)

Tendremosx + 2 A B A(x - 5) + B(2x + 3) (A + 2B)x +- 3B - 5A------ = -- + -- = = ~--~----

(2x+3)(x-5) 2x+3 x-S (2x+3)(x-5) (2x+3)(x-5)

Hay que determinadas constantes A y B de forma que

x + 2 (A + 2B)x + 3B - 5A

(2x + 3)(x - 5) (2x + 3)(x - 5)

x + 2 = (A + 2B)x + 3B - 5A

sea una identidad

o

Igualando los coeficientes de las potencias de x, se obtiene 1 = A + 2B Y 2 = 3B - 5A. Resolviendo estesistema resulta A = -1/13 Y B = 7/13 .

.\'+2 -1/13 7/13 -1 7Luego 2X2 _ Tx - 15 = 2x + 3 + x-S = 13(2x + 3) + l3(x - 5)

Otro método. x + 2 = A(x - 5) + B(2x + 3)

Para hallar B, hacemos x = 5: 5 + 2 = A(O) + B(IO + 3). 7 = I3B. B = 7/13.

Para hallar A, hacemos x = -3/2: -3/2 + 2 = A(-3/2 - 5) + B(O), 1/2 = -13A/2, A = -1/13.

2X2 + IOx - 3 A B e2. 2 =--+--+--

(x + 1)(x - 9) x + 1 x + 3 x - 3

2X2 + 10x - 3 = A(x2 - 9) + B(x + I)(x - 3) + e(x + I)(x + 3)

Para hallar A, hacemos x = - 1 :

Para hallar B, hacemos x = - 3:

Para hallar C. hacemos x = 3:

2 - 10 - 3 = A(I - 9),

18 - 30 - 3 = B( - 3 + 1)( - 3 - 3),

18 + 30 - 3 = e(3 + 1)(3 + 3).

A ~ 11/8.

B = -5/4.

e = 15/8.

2X2 + 10x - 3 11 5 15Luego = --- - --- + ---

(x+l)(x2-9) 8(x+l) 4(x+3) 8(x-3)

\

2X2 + 7x + 23 A B e3. ----- = -- + --- + --

(x - I)(x + 3)2 X - 1 (x + 3)2 X + 3

2X2 + 7.• + 23 = A (x + 3)2 + B( .• - 1) + e(x - I)(x + 3)

= A(x2 + 6x + 9) + B(x - 1) + e( .•2 + 2x - 3)

= A.\·2 + 6Ax + 9A + Bx - B + ex2 + 2e.>: - 3e

= (A + e)x' + (6A + B + 2C).>: + 9A - 8 - :;e

Identificando los coeficientes de potencias x, A + e = 2. 6A + B + 2e = 7 Y 9..1 - B - 3e = 23.

2X2 + 7x -i- 23 2 5Resolviendo el sistema. A = 2. B = - 5. e = O. Luego ---- ...--- = --- - ---

(x - l)(x + 3)2 X - I (x + 3)2

FRACCIONES SIMPLES

PROBLEMAS RESUELTOS

x + 2 l. Recomponer en fracciones 2 o

2x - 7x - 15

x + 2 (2x + 3)(x - 5)

o

Tendremos x + 2 A B A(x - 5) + B(2x + 3) (A + 2B)x +- 3B - 5A

----- = -- + - - = = -'------~----(2x + 3)(x - 5) 2x + 3 x-S (2x + 3)(x - 5) (2x + 3)(x - 5)

Hay que determinarlas constantes A y B de forma que

x+2

(2x + 3)(x - 5)

(A + 2B)x + 3B - 5A

(2x + 3)(x - 5)

x + 2 = (A + 2B)x + 3B - 5A

sea una identidad

277

Igualando los coeficientes de las potencias de x, se obtiene 1 = A + 2B Y 2 = 3B - 5A. Resolviendo este sistema resulta A = -1 / 13 Y B = 7/ 13.

x + 2 -1 / 13 7/ 13 -1 _,...,-7_-::-Luego = -- + -- = +

2X2 - 7x - 15 2x + 3 x - 5 13(2x + 3) l3(x - 5)

Otro método. x + 2 = A(x - 5) + B(2x + 3)

Para hallar B, hacemos x = 5 : 5 + 2 = A(O) + B(IO + 3), 7 = 13B, B = 7/13.

Para hallar A, hacemos x = -3/2: -3/2 + 2 = A(-3/2 - 5) + B(O), 1/2 = -13A/2, A = -1 / 13.

2X2 + IOx - 3 A B e 2. 2 =--+ -- +--

(x + I)(x - 9) x + 1 x + 3 x - 3

2X2 + 10x - 3 = A(x2 - 9) + B(x + I)(x - 3) + e(x + I)(x + 3)

Para hallar A, hacemos x = -1: 2 - 10 - 3 = A(I - 9), A:: 11 /8.

Para hallar B, hacemos x = -3: 18 - 30 - 3 = B(-3 + 1)(-3 - 3), B = -5/4.

Para hallar C. hacemos x = 3: 18 + 30 - 3 = e(3 + 1)(3 + 3), e = 15/8.

2X2 + 10x - 3 1\ 5 15 Luego = --- - --- + ---

(x+l)(x2 -9) 8(x+l) 4(x+3) 8(x-3)

2X2 + 7x + 23 A B e 3. ----- = -- + --- + - -

(x - I)(x + 3)2 X - 1 (x + 3)2 X + 3

2X2 + 7x + 23 = A (x + W + B(x - 1) + e(x - I)(x + 3)

= A(x2 + 6x + 9) + B(x - 1) + e(,,2 + 2x - 3)

= Ax2 + 6Ax + 9A + Bx - B + Cx' + 2Cx - 3C

= (A + C)x 2 + (6A + B + 2C)x + 9A - 8 - :;C

Identificando los coeficientes de potencias x. A + C = 2, 6A + B + 2C = 7 Y 9..1 - B - 3C = 23.

2X2 + 7x -i- 23 2 5 Resolviendo el sistema, A = 2, B = - 5, e = O. Luego ----.. - - = ---- - - --

(x - I)(x + 3)2 X - I (x + 3)2

http://carlos2524.jimdo.com/

Page 286: Algebra Superior Murray R Spiegel

x2 - 4x - 15(x + 2)3

y2 _ 8y - 3y3

7.

278 FRACCIONES SIMPLES

Otro método. 2X2 + 7x + 23 = A(x + 3)2 + B(.' - 1) + C(x - I)(x + 3).

Para hallar A, hacemos x = 1: 2 + 7 + 23 = A(1 + 3)2, A = 2.

Para hallar B, hacemos x = -3: 18 - 21 + 23 = B(-3 - 1), B = -5.

Para hallar e, hacemos x = O: 23 = 2(W - 5( -1) + C( -1 )(3), e = o.

x2-6x+2 A B e D4. =-+-+---+--

x2(x - 2)2 x2 X (x - 2)2 X - 2

x2 _ 6x + 2 = A(x - 2)2 + Bx(x - 2)2 + Cx? + DX2 (x - 2)

= A(x2 ~ 4x + 4) + Bx(x2 - 4x + 4) + Cx" + DX2 (x - 2)

= (B + D)x3 + (A - 4B + e - 2D)x2 + (-4A + 4B)x + 4A

Identificando los coeficientes de iguales potencias de x, B + D = O, A - 4B + e - 2D = 1, -4A + 4B =- 6, 4A = 2. Resolviendo el sistema se obtiene A = 1/2, B = - 1, e = - 3/2, D = 1.

x2-6x+21 1 3 1Luego = - - - - + --

x2 (x - 2)2 2X2 X 2(x - 2)2 X - 2

Deseo

8. -

Otro método. x2 - 6x + 2 = A(x - 2)2 + Bx(x - 2j2 t ex2 + DX2 (x - 2).

Para hallarA, hacemos x = 0:2 = 4A,A = 1/2. Para hallar e, hacemos x = 2:4 - 12 + 2 = 4e,e = -3/2.

Para hallar By D, hacemos x = dos valores cualesquiera excepto Oy 2 (por ejemplo, x = 1, x = - 1).

Para x = 1: 1 - 6 + 2 = A(I - 2)2 + B(I - 2j2 + e + D(I - 2) Y 1) B - D = -2.

Para x = -1: 1 + 6 + 2 = A( -1 - 2)2 - B( -1 - 2)2 + e + D( -1 - 2) Y 2) 9B + 3D = -6.

La solución del sistema formado por 1) y 2), B = -1, D = 1.

9. -

10.

11. -

x2 - 4x - 15S. 3 Sea y = x + 2; tendremos x = y - 2.

(x + 2)

SOLú

(y - 2)2 - 4(y - 2) - 15y3

8. -.\

9. -.\

1 8 3 8 3- Y - y2 - y3 - X + 2 - (x + 2)2 - (x '1- 2)3

10.2

11. -.\

7x2 - 25x + 6 Ax + B e6. (x2 _ 2x _ 1)(3x _ 2) = x2 - 2x - 1 + 3x - 2 12. -

x

7x2 - 25x + 6 = (Ax + B)(3x - 2) + C(x2 - 2x - 1)

= (3Ax2 + 3Bx - 2Ax - 2B) + ex2 - 2ex - e= (3A + e)x2 + (3B - 2A - 2e)x + (-2B - e)

13. -x

14. -.\

Identificando los coeficientes de iguales potencias de x, 3A + e = 7, 3B - 2A - 2e = - 25, - 2B - e = 6.La solución del sistema formado por estas tres ecuaciones es A = 1, B = - 5, e = 4.

15. x

L 7;(2 - 25x + 6. x - 5 4uego (x2 _ 2x _ 1)(3x - 2) = x2 - 2x - 1 + 3x - 2

16. -x

278 FRACCIONES SIMPLES

Otro método. 2X2 + 7x + 23 = A(x + 3)2 + BC-' - [) + C(x - I)(x + 3).

Para hallar A, hacemos x = 1: 2 + 7 + 23 = A(1 + 3j2, A = 2.

Para hallar B, hacemos x = -3: [8 - 21 + 23 = B(-3 - 1), B = -5.

Para hallar C, hacemos x = O: 23 = 2(3)2 - 5(-[) + C(-1)(3), C = O.

x2-6x+2 A BCD 4. =-+ - +---+--

x 2(x - 2)2 X2 X (x - 2)2 X - 2

X2 _ 6x + 2 = A(x - 2)2 + Bx(x - 2)2 + CX2 + DX2 (x - 2)

= A(x2 ~ 4x + 4) + Bx(x2 - 4x + 4) + CX2 + DX2 (x - 2)

= (B + D)x3 + (A - 4B + C - 2D)x2 + (-4A + 4B)x + 4A

Identificando los coeficientes de iguales potencias de x , B + D = O, A - 4B + C - 2D = 1, -4A + 4B = -6, 4A = 2. Resolviendo el sistema se obtiene A = 1/2, B = -1, C = -3/2, D = 1.

x2-6x+21 1 3 1 Luego = - - - - + --

x 2 (x - 2)2 2X2 X 2(x - 2)2 X - 2

Otro método. X2 - 6x + 2 = A(x - 2)2 + Bx(x - 2)2 t- CX2 + DX2 (x - 2).

Para hallarA, hacemos x = 0:2 = 4A,A = 1/2. Para hallar C, hacemos x = 2:4 - 12 + 2 = 4C,C = -3/2.

Para hallar By D, hacemos x = dos valores cualesquiera excepto O y 2 (por ejemplo, x = 1, x = - 1).

Para x = 1: 1 - 6 + 2 = A(1 - 2)2 + B(1 - 2j2 + C + D(I - 2) y I)B - D = -2.

Para x = -1 : 1 + 6 + 2 = A( -1 - 2)2 - B( -1 - 2)2 + C + D( -1 - 2) Y 2) 9B + 3D = -6.

La solución del sistema formado por 1) y 2), B = -1, D = 1.

X2 - 4x - 15 S. 3 Sea y = x + 2; tendremos x = y - 2.

(x + 2)

X2 - 4x - 15 (y - 2)2 - 4(y - 2) - 15 y2 - 8y - 3

(x + 2)3 y3 y3

8 3 8 3

7X2 - 25x + 6 Ax + B C 6. (x2 _ 2x _ 1)(3x _ 2) = X2 - 2x - 1 + 3x - 2

7X2 - 25x + 6 = (Ax + B)(3x - 2) + C(x2 - 2x - 1)

= (3Ax2 + 3Bx - 2Ax - 2B) + CX2 - 2Cx - C

= (3A + C)x2 + (3B - 2A - 2C)x + (-2B - C)

Identificando los coeficientes de iguales potencias de x, 3A + C = 7, 3B - 2A - 2C = - 25, - 2B - C = 6. La solución del sistema formado por estas tres ecuaciones es A = 1, B = - 5, C = 4.

L 7.>:2 - 25x + 6 . x - 5 4

uego 2 = 2 +--(x - 2x - 1)(3x - 2) x - 2x - 1 3x - 2

http://carlos2524.jimdo.com/

Page 287: Algebra Superior Murray R Spiegel

FRACCIONES SIMPLES 279

4x2 - 28 4x2 - 28 Ax + 8 Cx + D7. . = = --- + ---

X4+x2_6 (x2+3)(x2-2) x2+3 x2-2

4x2- 28 = (Ax + 8)(x2 - 2) + (Cx + D)(x2 + 3)

= (Ax3 + 8x2 - 2Ax - 28) + (Cx3 + DX2 + 3Cx +3D)

= (A + C~\"3 + (8 + D)x2 + (3C - 2A)x - 28 + 3D

Identificando los coeficientes de iguales potencias de x,A + C = O, 8 + D = 4. 3C - 2A = O. - 28 + 3D = - 28

4x2 - 28La solución del sistema es A = O. 8 = 8. C = O. D = -4. Luego --,-----=--

x4 + x2 - 68 4

x2+3 -7=2

PROBLEMAS PROPUESTOS

Descomponer en fracciones simples:

x+2 x 3x2-8x+9 5x' + 4x2 + 7x + 38.

x2 - 7x + 1212.

x2 - 3x - 1816. (x ,.:.,.2)3 19.

(x2 + 2x + 2 )(x2 - X - 1)

9.12x + 11

13.IOx2 + 9x - 7

x2+x-6 (x + 2)(x2 - 1) 17.3x3 + IOx2 + 27x + 27

20.3x

x2 (x + 3)2 x' - 18 - X x2 - 9x - 6

10.2X2 + 3x - 2

14.x3 + x2

- 6x 5x2 + 8x + 21 7x' + 16x2 + 20x + 518.

(x2 + x + 6)(x + 1)21.

(x2 +.2x + 2)25x + 4 x'11.x2 + 2x

15.x2 - 4

SOLUCIONES DE LOS PR08LEMAS PROPUESTOS

6 5 1 3 2 58 ----- 17. -+-+------

• x - 4 x - 3 X x2 X + 3 (x + 3)2

7 59. x _ 2 + x + 3

3 210. 2x _ 1 - x + 2

2 311. ~ + x + 2

2/3 1/312. x-6+x+3

32513. x+l+x-l+x+2

I 2 214. - - -- +--

x x-2 x+3

2x + 3 318. +--

x2 + X + 6 x + 1

2x - 1 3x + I19. + -2----

x2 + 2x + 2 x - x - 1

2 215. x + x _ 2 + x + 2

3 4 516. -- + --- + ---

x - 2 (x - 2)2 (x - 2)'

7x + 2 2>:+ I21. + ---------.x2 + 2x + 2 (x2 + 2x + 2)2

http://carlos2524.jimdo.com/

Page 288: Algebra Superior Murray R Spiegel

CAPITULO 29

TEC

w

Series infinitas

SUCESIONES. Una sucesión es un conjunto de números, U¡, U2, U3, ••. , dispuestos en un ordendefinido y que guardan una determinada ley de formación. Cada número de la sucesión se llamatérmino. Si el número de términos es finito, la sucesión se denomina sucesión finita y en caso con-trario, sucesión infinita.

Por ejemplo, el conjunto de números, 2, 5, 8, ... , 20, es una sucesión finita, mientras que 1,1/3, 1/5, 1/7, ... , es una sucesión infinita.

Mientras no se advierta lo contrario, en este capítulo solo trataremos de las sucesiones infinitas.

TERMINO ENESIMO DE UNA SUCESION. Es la ley de formación mediante la cual se ob-tiene un término cualquiera de la sucesión en función de otros anteriores. Esta ley, en general, vienedada por una expresión en la variable n de forma que dándolelos sucesivos valores 1, 2, 3, ... ,se obtienen el primero, segundo, tercero, ... , términos. El término enésimo es el término generalde la sucesión.

1Por ejemplo, si el término enésimo de una sucesión es Un = n2 + 1 ' el primero, segundo

1 1 1 1 1 1Yquinto términos son, respectivamente, U¡ = tT+1 = 2" ' U2 = 22 + 1 = "5 ' Us = 52 + 1 = 26 .

SERIE. Es una suma indicada, U¡ + U2 + U3 + ... , de los números de una sucesión. Los núme-ros U¡, U2, U3' ••• , se denominan primero, segundo, tercero, ... , términos de la serie; Un es el enési-mo término de la misma. Si la serie consta de un número finito de términos, se llama serie finita, y,en caso contrario. serie infinita.

Por ejemplo, si el término enésimo de una serie es1

Un = l'n.1 1

la sene es TI + 2! +

p

El símbolo L Un representa U¡ + u~ + U3 + ... + up

n=1

El símbolo f Un representa U¡ + U2 + U3 + ..."=1

co

Por ejemplo, L"=1

1 1 1 1--=--+--+--+n2 + 1 12 + 1 22 + 1 32 + 1

280

CAPITULO 29

Series infinitas

SUCESIONES. Una sucesión es un conjunto de números, u¡ , U2' U3, ... , dispuestos en un orden definido y que guardan una determinada ley de formación . Cada número de la sucesión se llama término. Si el número de términos es finito, la sucesión se denomina sucesión finita y en caso con­trario, sucesión infinita.

Por ejemplo, el conjunto de números, 2, 5, 8, .. . , 20, es una sucesión finita , mientras que 1, 1/3, 1/5, 1/7, . .. , es una sucesión infinita.

Mientras no se advierta lo contrario, en este capítulo solo trataremos de las sucesiones infinitas.

TERMINO ENESIMO DE UNA SUCESION. Es la ley de formación mediante la cual se ob­tiene un término cualquiera de la sucesión en función de otros anteriores. Esta ley, en general , viene dada por una expresión en la variable n de forma que dándole. los sucesivos valores 1, 2, 3, . . . , se obtienen el primero, segundo, tercero, .. . , términos. El término enésimo es el término general de la sucesión.

1 Por ejemplo, si el término enésimo de una sucesión es Un = n2 + 1 ' el primero, segundo

1 1 1 1 1 1 Y quinto términos son, respectivamente, U¡ = -¡-r::¡:-¡ = 2" ' U2 = 22 + 1 = "5 ' Us = 52 + 1 = 26 .

SERIE. Es una suma indicada, U¡ + U2 + U3 + ... , de los números de una sucesión. Los núme­ros U¡ , U2, U3' . .. , se denominan primero, segundo, tercero, ... , términos de la serie; Un es el enési­mo término de la misma. Si la serie consta de un número finito de términos, se llama serie finita, y, en caso contrario, serie infinita.

Por ejemplo, si el término enésimo de una serie es 1 1

la sene es TI + 2 I +

p

El símbolo L Un representa U¡ + u2 + U3 + . .. + up n=1

El símbolo f Un representa U¡ + U2 + U3 + . .. "=1

<Xl

Por ejemplo, L "=1

1 1 1 1 --=-- + - -+ - -+ n2 + 1 12 + 1 22 + 1 32 + 1

280

http://carlos2524.jimdo.com/

Page 289: Algebra Superior Murray R Spiegel

SERIES INFINITAS 281

LIMITE DE UNA SUCESION. Un número L es el límite de una sucesion infinita, U¡, U2,

U3, ... , si dado un número E, tan pequeño como se quiera, existe un número N tal que IUn - LI < E

para todos los enteros n > N.

P . I d da la sucesi d ,. I 2 1 2n + 1 3or ejemp o, a a a sucesion e termino genera Un = + - = ---, O sea, , 5/2, 7/3,n n

9/4, ... , se puede demostrar que su límite L es igual a 2.

Si existe el límite L de una sucesión U¡, U2, U3, •.. , se escribe Iim Un = L Y se lee «el límiten- 00

de Un cuando n tiende a infinito es igual a L. La expresión «n tiende a infinito» es sinónima de «nes cada vez mayor» o de «n crece indefinidamente».

2n + 1u" = --n-'En la sucesión de término general se escribe Iim 2n + 1 = 2.

n

Si no existe un número L tal que IUn - LI < E para todo n > N, la sucesión carece de lími-te. Si los sucesivos términos de una sucesión aumentan constantemente, no existe lim Un Y

para indicarlo se emplea la notación Iim Un = oo.n-e co

Por ejemplo, si Un = 2n, Iim 2n = 00, ya que los sucesivos términos, 2, 4, 6, 8, 10, ... , den-ceo

la sucesión aumentan constantemente sin ninguna barrera.

Una forma más intuitiva, aunque menos rigurosa que la anterior, de expresar el conceptode límite, es decir, que el límite de la sucesión U¡, U2, U3, .•. es L, si los términos sucesivos se apro-ximan cada vez más al valor L. Esta manera de interpretar el límite sirve, con frecuencia, para «adi-vinar» el valor de L, si bien es cierto que para determinar correctamente el valor de un límite, hayque recurrir al concepto primario a pesar de que la demostración resulte a veces muy dificil.

P . I I ., d ,. I 2n + 1 I ,. .or ejernp o, en a sucesion e termino genera Un = ---1' os terminas sucesivosn +

3 579('2' 3 ' 4 ' 5' ... ) se aproximan a 2, con lo que éste número puede ser su límite. Para la

demostración, véase el Problema 7.

TEOREMAS DE LOS LIMITES. Si existen Iim a; y Iim b., se verifica:a-e co "-'Xl

1. Iim (an ± bn) = Iim a; ± lim b;n-e co n-eco e -e co

2. Iim (an bn) = Iim an Iim bnn-e co a-e co a-e co

lim a"3. Iim a; a-e co siempre que Iim b; =F O

n-e co TJí Iim b; e-e eon-e ec

Si Iim b; = O y Iim a; =F O, Iim ~ no existea-e co a-e co e-e co bn

Si Iim b; = O y Iim an = O, Iiman puede o no existir

a-ceo a-e co a-e co t;

http://carlos2524.jimdo.com/

Page 290: Algebra Superior Murray R Spiegel

282 SERIES INFINITAS

4. Iim (an)P = (lim an)P, p = un número real

SUCESIONES MONOTONAS ACOTADAS. Una sucesión está acotada si existe un número po-sitivo M, independiente de n, tal que Iunl ~ M para n = 1, 2, 3, ...

Por ejemplo, 3, 5/2, 7/3, 9/4 es una sucesión acotada, ya que el valor absoluto de cada térmi-no nunca excede a 3. La sucesión 3/4, - 4/5, 5/6, - 6/7, 7/8 está acotada, ya que el valor absolutode cada término nunca excede a 1.

La sucesión 2, 4, 6, 8, ... , no está acotada.

U na sucesión es monótona creciente si Un + ¡ :?; Un' Y monótona decreciente si Un + ¡ ~ Un' paran = 1, 2, 3, ... Por ejemplo, la sucesión 1/2, 3/4, 4/5, 5/6, ... , es monótona creciente y la suce-sión 3, 2, 1, O, - 1, - 2, - 3, ... , es monótona decreciente.

Toda sucesión monótona, creciente o decreciente, y acotada tiene límite. Por ejemplo, lassucesiones

1/2, 3/4, 4/5, 5/6, .. . y 1/4, 1/8, 1/16, 1/32, ...

están acotadas y son monótonas creciente y decreciente, respectivamente. Por tanto, ambas su-cesiones tienen límite.

Sin embargo, para que una sucesión tenga límite no es necesario que sea monótona crecienteo decreciente. Por ejemplo, la sucesión 2/3,5/4,3/4,6/5,4/5,7/6,5/6, ... , está acotada y no es mo-nótona creciente ni decreciente. Su límite es 1.

CONVERGENCIA Y DIVERGENCIA DE SERIES INFINITAS

Sea S; = U¡ + U2 + U3 + ... + Un la suma de los n primeros términos de la serie U¡ + U2 +U3 + ... Los términos de la sucesión SI' S2, S3, ..• , se llaman sumas parciales de la serie SiIim S; = S es un número finito, la serie U¡ + U2 + U3 + ... es convergente y S es la suma de lan-+ (fJ

serie infinita. Una serie que no sea convergente se llama divergente.

P· 11 dI .. ' dl·1 111 1or ejernp o, a SUma e os n pnmeros terrnmos e a sene "2 + 22 + 23 + 24 + ... es a

1 1suma de los n primeros términos de una progresión geométrica de primer término "2 y razón "2;

esta suma vale S; = 1 - }.. Como lim (1 - }.) = 1, la serie es convergente y su sumaS = 1.a-e co

En la serie 1 - 1 + 1 - 1 + ... , la suma de los n primeros términos es O ó 1 según que elnúmero de términos que se tomen sea par o impar. Por tanto, no existe lim S; y la serie es di-vergente. n- <X)

Si una serie es convergente el límite de su término general cuando n -+ 00 es cero. Sin embar-go, si el límite del término general de una serie, cuando n -+ 00, es igual a cero, la serie puede o noser convergente. Una serie es divergente si el límite de su término general cuando n -+ 00 es dis-tinto de cero.

CR

CR

SEF

COI

282 SERIES INFINITAS

4. Iim (an)P = (lim an)P, p = un número real

SUCESIONES MONOTONAS ACOTADAS. Una sucesión está acotada si existe un número po­sitivo M , independiente de n, tal que Iunl :;::; M para n = 1, 2, 3, ...

Por ejemplo, 3, 5/2, 7/3, 9/4 es una sucesión acotada, ya que el valor absoluto de cada térmi­no nunca excede a 3. La sucesión 3/4, - 4/5, 5/6, - 6/7, 7/8 está acotada, ya que el valor absoluto de cada término nunca excede a 1.

La sucesión 2, 4, 6, 8, ... , no está acotada.

U na sucesión es monótona creciente si Un + I :?; Un' Y monótona decreciente si Un + I :;::; Un' para n = 1, 2, 3, . . . Por ejemplo, la sucesión 1/2, 3/4,4/5, 5/6, .. . , es monótona creciente y la suce­sión 3, 2, 1, O, - 1, - 2, - 3, . . . , es monótona decreciente.

Toda sucesión monótona, creciente o decreciente, y acotada tiene límite. Por ejemplo, las sucesiones

1/2, 3/4, 4/5, 5/6, . . . y 1/4, 1/8, 1/ 16, 1/32, .. .

están acotadas y son monótonas creciente y decreciente, respectivamente. Por tanto, ambas su­cesiones tienen límite.

Sin embargo, para que una sucesión tenga límite no es necesario que sea monótona creciente o decreciente. Por ejemplo, la sucesión 2/3, 5/4, 3/4, 6/5, 4/5, 7/6, 5/6, . . . , está acotada y ho es mo­nótona creciente ni decreciente. Su límite es 1.

CONVERGENCIA Y DIVERGENCIA DE SERIES INFINITAS

Sea Sn = U I + U2 + U3 + .. . + Un la suma de los n primeros términos de la serie U I + U2 + U3 + . .. Los términos de la sucesión SI' S2, S3' ... , se lI'aman sumas parciales de la serie Si Iim Sn = S es un número finito, la serie U I + U2 + U3 + ... es convergente y S es la suma de la n-+ (fJ

serie infinita. Una serie que no sea convergente se llama divergente.

P · II dI . .. dl· 1 111 I or eJemp o, a suma e os n pnmeros termmos e a sene 2" + 22 + 23 + 24 + es a

1 1 suma de los n primeros términos de una progresión geométrica de primer término 2" y razón 2" ;

esta suma vale Sn = 1 - ~. Como lim (1 - ~) = 1, la serie es convergente y su sumaS = 1. n- oc.

En la serie 1 - 1 + 1 - 1 + .. . , la suma de los n primeros términos es O ó 1 según que el número d·e términos que se tomen sea par o impar. Por tanto, no existe lim Sn y la serie es di-vergente.

n- ce:

Si una serie es convergente el límite de su término general cuando n -+ 00 es cero. Sin embar­go, si el límite del término general de una serie, cuando n -+ 00 , es igual a cero, la serie puede o no ser convergente. Una serie es divergente si el límite de su término general cuando n -+ 00 es dis­tinto de cero .

http://carlos2524.jimdo.com/

Page 291: Algebra Superior Murray R Spiegel

SERIES INFINITAS 283

CRITERIOS DE COMPARACION PARA LA CONVERGENCIA de series de términos po-sitivos.

1 1 11) - "2 + 22 - 23 +es convergente.

1 1 12) 1 - "2 + "3 - 4" +1 lid'"2+ "3+ 4"+ .... iverge.

es absolutamente convergente, ya que

a

Si una serie de términos positivos es tal que todos ellos, a partir de uno y todos los siguien-tes, son menores o iguales que los correspondientes de otra serie convergente conocida, la seriedada es convergente.

Si a partir de un término, y todos los siguientes, de una serie dada de términos positivos sonmayores o iguales que los correspondientes de una serie divergente conocida, la serie dada es di-vergente.

Como series conocidas en la aplicación práctica de estos criterios de comparación, son muyútiles las siguientes:

1) La serie geométrica, a + ar + ar2 + ... + ar" - I + ... , siendo a y r constantes, es con-vergente si Irl < 1 y divergente si Irl ~ l.

2 .111 l'd .) La sene TP + 2P + 3P + ... + nP + ... , sien o p una constante, es convergente SI p > 1

di . 1 S' 1 b . I . ,. d' 1 1 1 1y ivergente SI p ~ . I P = se o tiene a sene armontca ivergente +"2 + '3 + 4 +

o

te0-

CRITERIO DEL COCIENTE PARA LA CONVERGENCIA de series de términos positivos,negativos o positivos y negativos.

Sea la serie U¡ + U2 + U3 + ... cuyos términos pueden ser positivos y negativos y llamemos

lim I Un + ¡ I = Rn- 00 Un

En estas condiciones, la serie es:

a) Convergente si R < 1, b) divergente si R> 1, e) nada se puede afirmar si R = 1.

SERIE ALTERNADA. Es aquella cuyos términos son, alternativamente, positivos y negativos.. 1 1 1

Por ejemplo, 1 - "2 + "3 - 4" ++Sila

la

Una serie alternada es convergente si:

a) A partir de un cierto término, y para todos los siguientes, el valor absoluto de cada uno deellos es menor que el del anterior, es decir, si lun+¡1 < lunl, y sib) El límite del término general, cuando n crece indefinidamente, es igual a cero, es decir, silim Un = O.n-e co

CONVERGENCIA ABSOLUTA Y CONDICIONAL

2'

1.

Una serie es absolutamente convergente si es convergente la serie formada con los valores ab-solutos de sus términos. Una serie convergente que no sea absolutamente convergente, se llamacondicionalmente convergente. Por ejemplo,

es convergente, pero no absolutamente convergente, ya que 1 +

Luego la serie alternada es condicionalmente convergente.

SERIES INFINITAS 283

CRITERIOS DE COMPARACION PARA LA CONVERGENCIA de series de términos po­sitivos.

Si una serie de términos positivos es tal que todos ellos, a partir de uno y todos los siguien­tes, son menores o iguales que los correspondientes de otra serie convergente conocida, la serie dada es convergente.

Si a partir de un término, y todos los siguientes, de una serie dada de términos positivos son mayores o iguales que los correspondientes de una serie divergente conocida, la serie dada es di­vergente.

Como series conocidas en la aplicación práctica de estos criterios de comparación, son muy útiles las siguientes:

1) La serie geométrica, a + ar + ar2 + ... + ar" - l + ... , siendo a y r constantes, es con­vergente si Irl < 1 Y divergente si Irl ~ 1.

2) L . 1 1 1 1 . d . a serIe TP + 2P + 3P + ... + nP + ... , sIen o p una constante, es convergente SI p > 1

d · . 1 S' 1 b . l . ,. d ' 1 1 1 1 Y Ivergente SI p ~ . I P = se o tIene a serIe armon/ca Ivergente + '2 + '3 + 4' +

CRITERIO DEL COCIENTE PARA LA CONVERGENCIA de series de términos positivos, negativos o positivos y negativos.

Sea la serie U 1 + U2 + U3 + . .. cuyos términos pueden ser positivos y negativos y llamemos

Iim I Un + 1 I = R n- 00 Un

En estas condi.;:iones, la serie es:

a) Convergente si R < 1, b) divergente si R > 1, c) nada se puede afirmar si R = 1.

SERIE ALTERNADA. Es aquella cuyos términos son, alternativamente, positivos y negativos. . 1 1 1

Por ejemplo, 1 - '2 + '3 - ¡ +

Una serie alternada es convergente si:

a) A partir de un cierto término, y para todos los siguientes, el valor absoluto de cada uno de ellos es menor que el del anterior, es decir, si lun+ 11 < Iunl, y si b) El límite del término general, cuando n crece indefinidamente, es igual a cero, es decir, si Iim Un = O. n~ oo

CONVERGENCIA ABSOLUTA Y CONDICIONAL

Una serie es absolutamente convergente si es convergente la serie formada con los valores ab­solutos de sus términos. Una serie convergente que no sea absolutamente convergente, se llama condicionalmente convergente. Por ejemplo,

1) es absolutamente convergente, ya que

es convergente. 1 1 1

2) 1 - 2' + '3 - ¡ + es convergente, pero no absolutamente convergente, ya que 1 +

1 lid' 2' + '3 + ¡ + .. . ,Iverge. Luego la serie alternada es condicionalmente convergente.

http://carlos2524.jimdo.com/

Page 292: Algebra Superior Murray R Spiegel

284 SERIES INf=INfTAS

3.Los términos de una serie absolutamente convergente se pueden ordenar de cualquier formasin que deje de ser convergente. Sin embargo, si la serie es condicionalmente convergente, orde-nando convenientemente sus términos se puede obtener una serie divergente o convergente cuyasuma se haya prefijado de antemano.

SERIES DF POTENCIAS. Una serie de la forma Co + Clx + C2X2 + C3X3 + ... + c"x" + ... ,en la que los coeficientes Co, Cl, C2' ... , son constantes, se llama serie de potencias de x.

Análogamente, Co + cl(x - a) + c2(x - a)2 + ... + c"(x - a)" + ... , siendo a una cons-tante, es una serie de potencias de (x - a).

, x2 x3

ASI, pues, I + x + 2' + 3" + . . . es una serie de potencias de x.

El conjunto de valores de x para los cuales una serie de potencias es convergente se llama campode convergencia y se determina mediante el criterio del cociente junto con otros aplicados al ex-tremo del intervalo.

4.

PROBLEMAS RESUELTOS

TERMINO GENERAL

1. Escribir los cuatro primeros de la sucesión cuyo término general es el que se indica:

n l· 2 3 4 I 2 3 4a) Los términos son

2(1) <+ I ' 2(2) +. I ' 2(3) + I ' 2(4) + Io sea 3' s' 7'92n + 1

2n - I 2(1) - I 2(2) - I 2(3) - I 2(4) - 1 1 3 5 7b)

, (n + 1)2' Los términos son (T+T¡2' (2 + 1)2 ' (3 + 1)2 ' (4 + I fo sea 4' 9' J6' 25'

5.2.-1 2° 21 22 23 I 2 4 8

-e)n2 + 1

Los términos son J2+1' 22 + 1 ' 32 + 1 ' 42+ o sea 2' s' 10' 17

(-Ir (_1)1 (-If (_1)3 (_1)· 1 1 I Id) Los términos son 1(i+I)' 2(2 + 1)' 3(3 + 1)' 4(4 + 1)

o sea -}, 6' -12'20'n(n + 1)

xn+ 1 I x2 x3 x· x' x2 x3 x· x'e) Los términos son - - - - o sea - - -

120 '(n + I)! 2! ' 3' ' 4!' 5! 2 ' 6 . 24'

2. Escribir los cuatro primeros términos y el término (n + 1) de las series cuyo término enésimo es el que se indica:

n 2 I 4 n+1al .f=I' 1+3+3+'27 término (n + 1): 6. I3"

hl2n +'1 3 5 7 9 2(n + 1) + 1 2n + 3 a-+-+-+- término (n + 1):4h - 2 2 6 10 14 4(n + 1) - 2 4n + 2

el(-I).-'Jn 1 fififl

término (n + 1):( - 1r J;;-:¡:-¡-

---+---n + I 2 3 '4 5 n + 2

http://carlos2524.jimdo.com/

Page 293: Algebra Superior Murray R Spiegel

SERIES INFINIT¡\S 285

1 1 1 1h) - + - + - + - + , , ,

3 6 9 121 1

311' 3(11 + 1) = 3n + 3

a 3, Escribir el término enésimo de las sucesiones siguientes:

a I 2 3 4 1/ 4 5 6 7 ti + 3a) 2' )' - s' d) M' 4:(;' :5:-7'(;8'4 11+ I (11 + 2)(11 + 4)

I 3 5 7 2/1 - I 2 4 6 8 211h) 2' 4' - s' e)1+3'2+3' 3+3'4+3'6 2n 11 + 3

2 4 8 16 2·e) 3' s' 7' 9 211 + 1

4, Hallar el término enésimo y el término (/1 + 1) de las series siguientes:

o1 1 I 1

a) 3 + S + 7 + 9 + , ,1 1

211 + l ' 2(11 + 1) + 1 211 + 3

1 1 1 1,e) I!- 2! + 3! - 4! +

(-Ir-' (-Ir11! (11 + I)!

d) - 14 + 19 - ~/¡6+ ' , ,

3'4'5 4'5'6 5'6,7 6'7'Se) --I!- + -3-'-, - + -5-,-' - + -7-!- + .. ,

(11 + 2)(n + 3)(11 + 4)(2n - I)!

(n + 3)(n + 4)(n + 5)(2/1 + I)!

(2n - 1)(2n + 1)' (2n + 1)(2n + 3)

(-1)"-'-\"'-'

2·- 'n2

(-Ir,\'"2· (n + 1)2

S, Se pide a dos alumnos que escriban el término enésimo, o general, de la sucesión, 1, 16, SI, 256, ' , , , y luego queescriban el quinto término, Uno de ellos contesta que el término enésimo es u. = /14 yel otro no está de acuerdocon el anterior y responde que es u. = IOn3

- 35/12 + 5011 - 24, ¿Cuál de las respuestas es correcta?

Si U. = 114

, se tiene u, = 1, U2 = 24 = 16, U3 = 34 = SI Y U4 = 44 = 256, que coinciden con las de la su-cesión, Por tanto, según el primer alumno, el quinto término es Us = 54 = 625,

Si u. = 10113- 35n

2 + 5011 - 24, se tiene u, = 1. U2 = 16, U3 = 81 Y U4 = 256, que también coincidencon los de la sucesión, Por tanto, para el segundo alumno, el quinto término es Us = 601,

a: Ambos alumnos han respondido, pues, correctamente, Dando solamente un número finito de términos deuna sucesión o serie, no queda definido su término general. Es decir, son posibles infinitos términos enésimos.

6, Escribir las series representadas por los símbolos siguientes:

32

a)

http://carlos2524.jimdo.com/

Page 294: Algebra Superior Murray R Spiegel

286

10.

SERIES INFINITAS

8,

7 I I 1 1 1d) I -=-+-+-+-.=4.Jn j4 j5 J6 J7

f)cc (-I).(x+2f+l (X+2)2 (X+2)3 (X+2)4.1;0 -'-------'----n'---+-I-'--= (X + 2)- --2- + --3- - --4- + ...

LIMITE DE UNA SUCESION

2n + 17. Sea la sucesión de término general u. = ~ .

a) Escribir en forma decimal los términos números 1,2, 5, 10, 100, 1000 Y 10000 de la misma, ¿A qué límiteparece tender?

b) Aplicando la definición de límite, comprobar que el límite anterior es correcto.

c) Hallar el límite teniendo en cuenta los teoremas correspondientes.

a) n = 11,50000

n = 21,66667

n = 101,90909

n = 1001,99010

n=IOoo1,99900

n = 100001,99990

n = 51,83333

El límite parece ser 2,

b) Hay que demostrar que dado un número positivo e, tan pequeño como se quiera, existe un valor de n. porejemplo N (función de E), de forma que lu. - 21 < e para todo n > N.

SUC

9.

Ahora bien, \2n + 1 _ 2\ < f si \_-1-\ < f, es decir. si -1- < f. n + 1 > ~ o sea n > ~ - l.n+1 n+1 n+1 e e

Por ejemplo, si e = 0,01, n > 99. Esto quiere decir que todos los términos de la sucesión posteriores al 99difieren de 2 en menos de 0,01.

Si e = 0,0001, n > 9999. Esto quiere decir que todos los términos de la sucesión posteriores al 9999 difierende 2 en menos de 0,0001.

e) Se tiene

12 + -

2n + 1 nu =----=--• n + 1 t '

1+-n

dividiendo el numerador y el denominador por n.

2 + ~ lim (2 + ~)Luego lim u. = lim __ n = .-x n

e-e cc I + ~ lim (1 + ~)n n

2 + lim -._" n 2 + O I

---'-'--'-- = -1 + O = 2,ya que lim - = O.+ lim ~ a-s co n

n-x. n

286 SERIES INFINITAS

7 I 1 1 1 1 d) I - = - +-+ - +-

. ~ 4 Jn j4 Js .fi .fi

f) '" (- Ir (X + 2r+ I (X + 2)2 (X + 2)3 (x + 2)4

.1;0 -'-------'---n-'--+- I----'-- = (X + 2) - --2- + --3- - - -4- + ...

LIMITE DE UNA SUCESION

2n + 1 7. Sea la sucesión de término general u. = -;;-:¡:¡- .

a) Escribir en forma decimal los términos números 1,2, 5, lO, lOO, 1000 Y 10 000 de la misma. ¿A qué límite parece tender?

b) Aplicando la definición de límite, comprobar que el límite anterior es correcto.

c) Hallar el límite teniendo en cuenta los teoremas correspondientes.

a) n = I 1,50000

n = 2 1,66667

El límite parece ser 2.

n = 5 1,83333

n = 10 1,90909

n = lOO 1,99010

n=IOOO 1,99900

n = lO 000 1,99990

b) Hay que demostrar que dado un número positivo E, tan pequeño como se quiera , existe un valor de n. por ejemplo N (función de E), de forma que lu. - 21 < E para todo n > N .

Ahora bien. 12n + l _ 21 < E si 1 __ 1_ 1 < E, es decir. si _ 1_ < E. n + 1 > ~ o sea n > ~ - l.

n + 1 n+1 n+1 E E

Por ejemplo. si E = 0.01 , n > 99. Esto quiere decir que todos los términos de la sucesión posteriores al 99 difieren de 2 en menos de 0,01.

Si E = 0,0001 . n > 9999 . Esto quiere decir que todos los términos de la sucesión posteriores al 9 999 difieren de 2 en menos de 0.0001.

1 2 + -

e) Se tiene 2n + 1 n

u = --- = - -• n + 1 1 '

dividiendo el numerador y el denominador por n .

1+ ­n

1 1 2 + - lim (2 + -)

Luego Iim 11. = Iim __ n = .-x n l. 1

. - '" 1 + - 11m (1 + - ) n n

2 + lim -.-oc, n 2 + O . 1 - --.---- '-:- = -- = 2 ya que 11m - = O.

+ lim ~ 1 + O ' ._0> n

"- 'XI n

http://carlos2524.jimdo.com/

Page 295: Algebra Superior Murray R Spiegel

SERIES INFINITAS 2878. Hallar los límites siguientes:

. I2n2 + n 2 + - 11m (2 + -)

a) lim = lim __ n_ = ..'C:.~ n_ ='-00 5n2

- I ._'" l. I5 - -, 11m (5 --,)n n ...•cc n

lirn 2 + lim -n-'X., n 2 + O

5 --'O25

lim 5 - lim -~n2

b)3n2 + 4n + 5

lim 7n2 _ 4 lim

4 53 +- +-

n n2

47 --

n2

3 + O + O7 - O

37

2 32n2 + 3 ;; +;¡J O + O

e) 11m --- = lim --- = -- = O'-00 4n3

- 1 '-00 4 _ -'-- 4 - On3

2n2 + 2 . + ;> 1

d) 11m --- = 11m --- = - = co , es decir, no existe límite'-00 3n + 2 '-00 3 2 O

;; + ;>

e)(n - 2)! __ ::..-(n_-_2~)_!_11m ---= 11m

n! '-00 n(n - I)(n - 2)!1

lim ---=0._'" n(n - 1)

4n - 2 4n - 2 4 - 2/n 41) lirn (__ )4 = (lim __ )4 = (Iirn )4 = (_)4 = 16'-00 2n + 3 '-00 2n + 3 '-00 2 + 3/n 2

rz=:'; r: ~ r: (Jn+\ + Jn) 1g) ILm (yn + 1 - yn) = ILm (yn + 1 - yn) ~ r.: = lim ~ r=. = O

'00 .00 (yn+l+yn) '-""yn+l+yn

SUCESIONES MONOTONAS ACOTADAS

9. Examinar la tabla siguiente:

Sucesión Monótona Monótona ExisteAcotada creciente decreciente límite

2; 2,5; 3; 3,5; 4; 4,5; ... No Sí No No

1, -1, 1, -1, 1, -1, ... Sí No No No

1; 1,1; 1,11; 1,111; 1,1111; ... Sí Sí No Sí

1/10, 1/11, 1/12, 1/13, 1/14, ... Sí No Sí Sí

1, 3/4, 1, 4/5, 1, 5/6, 1, 6/7, ... Sí No No Sí

10. DI' . .. I 3n + 1 ) d b)' .emostrar que a sucesión cuyo termmo genera es u. = -;+2 es a acota a y monotona creciente Y. por

consiguiente, tiene límite.

". 3n + 1 .'a) Una cota de la sucesion es 3 (o un numero mayor que 3). ya que --- ~ 3 urncamente cuando 3n + ~n + 2

3n + 6, o sea 1 ~ 6 que es cierto para cualquier valor de n,

. . .. 3n + 4 3n + lb) La sucesion es monotona creciente SI U.+1 ;::; un- Para que ~;::; -;+2 ha de ser (3n + 4)(n + 2);::;(3n + 1)(n + 3), 3n2 + IOn + 8;::; 3n2 + 101\ + 3. o sea 8 ;::; 3. que es cierto para cualquier valor de n.

http://carlos2524.jimdo.com/

Page 296: Algebra Superior Murray R Spiegel

288 SERIES INFINITAS

15.

CONVERGENCIA Y DIVERGENCIA DE SERIES INFINITAS 13

I11. Sea la serie de término general "" = y-i'

a) Escribir los cuatro primeros términos de la serie.

h) Llamando S. a la suma de los 11 primeros términos. hallar. en forma decimal. SI' S2' S, ..... S•. i.A quélimite parece tender la sucesión S. cuando ti ---> x:?

e) Comprobar el limite anterior efectuando la suma de los 11 primeros términos.

1 1 1 1al 3" + 31 + J2 + 33 + o

h) "1 = 1.000000112 = 0.333333", = 0.111111"4 = 0.037037liS = 0.012346"6 = 0.004115117 = 0.001372"8 = 0.000457

SI = "1 = 1.000000S2 = "1 + "2 = 1.333333S, = 111 + "2 + 11, = 1.444444S. = 1/1 + 1/2 + 1/, + 1/. = 1.481481Ss = 1.493827S6 = 1.497942S7 = 1.499314S8 = 1.499771

14.

CF

El limite parece ser lim S. = 1.500000 = 3/2.

1 1 1el La serie 1 + "3 + "9 + 27 + es una serie geométrica de primer término a = 1 Y razón r

suma de los 11 primeros términos es

S. = a(1 - ''''.l = ~ (1/3)" = ~[I _ (~)"]1 - r 1 - 113 2 3

3lim S. ="2y

Por tanto. el limite propuesto en h) es correcto.

1 1 1 3Así. pues. la serie 1 + "3 + "9 + n + ... es convergente y su suma vale "2

. 1 1 1 112. Sea la serie "3 - "3 + "3 - "3 +

a) Calcular S•. suma de sus 11 primeros términos.

b¡ Hallar lim S •.

e) Determinar si la serie es convergente.

1 1 1 1 1 1 1 1 1 1 1a) S, = "3' S2 = "3 - "3 = O. S, = "3 - "3 + "3 = "3' S. = "3 - "3 + "3 - "3 = O. etc.

1 . .Luego S. = -3 SI n es Impar y S. = O si 11 es par.

h) La sucesión SI' S2' S,.1

. .. o bien3

no tiene limite.1 1

O. O.3 3

e) La serie es divergente ya que no existe lim S •.

Esto es evidente. por otro lado. pues el término enésimo no tiende a cero cuando n ---> zc .

288 SERIES INF INITAS

CONVERGENCIA Y DIVERGENCIA DE SERIES INFINITAS

I J. Sea la serie de termino general 11" = y_l'

a) Escribir los cuatro primeros terminos de la se rie .

h) Llamando S. a la suma de los 11 primeros términos. hallar. en forma decimal. S,. S2' Sj . . ... S • . i,A qué lim ite parece tender la sucesión S. cuando 11 ..... x. '?

el Comprobar el limite anterior efectuando la suma de los 11 primeros términos,

1 1 1 1 a) JO + 3' + 3i + 3l + o

h) 11, = 1.000000 " 2 = 0.333333 II j =O.1I1I1I 11. = 0.037037 liS = 0.012346 "b = 0.004 11 5 "7 = 0.001372 "s = 0.000457

S, = lI, = 1.000000 S2 = 1/, + 1/2 = 1.333333 Sj = ", + "2 + "j = 1.444444 S. = 1/, + ", + " j + 1/4 = 1.48 148 1 Ss = 1.493827 S6 = 1.497942 S7 = 1.499314 Ss = 1.49977 1

El limite parece se r lim S. = 1.500000 = 3/2.

1 1 I e) La serie 1 + '3 + '9 + 27 + es una serie geometrica de primer término a = 1 Y razón ,.

suma de los 11 primeros términos es

a( 1 - ,..) - (1 /3)· 3 1 S. = --¡-=-;:- = 1 _ 113 = '2 [1 - (3)·J

Por tanto. el limite propuesto en h) es correcto.

y 3

lim S. = '2

1 1 1 Asi. pues. la serie 1 + - + - + - + .

3 9 27

3 es convergente y su suma vale '2

. 1 1 1 1 12. Sea la sen e '3 - '3 + '3 - '3 +

a) Calcular S •. suma de sus 11 primeros terminos.

h) Ha llar lim S • .

e ) Determina r si la serie es convergente .

1 1 1 1 1 1 I 1 I 1 1 a) S , = -3' S, = -3 - -3 = O. S3 = - - - + - = - . S4 = - - - + - - - = O. etc.

333 3 33 3 3 1

Luego S. = - 3 si 11 es impar y S. = O si 11 es par.

h) La sucesión S,. S,. S3' .... o bien 3' o. 3 O. 3 .. . no tiene lim ite.

e ) La se rie es divergente ya que no ex iste lim S •.

Esto es evidente. por otro lado. pues el te rmi no encsimo no tiende a cero cuando 11 ..... xc .

1 '3. La

http://carlos2524.jimdo.com/

Page 297: Algebra Superior Murray R Spiegel

SERIESINFINITAS 289

13. Hallar el carácter de las series siguientes:

a)2 3 4 n

1+-+-+-+ ... +--+3 5 7 2n-1

El límite del término enésimo es lim __ n_ = ~ .._'" 2n - 1 2

La serie es divergente, ya que el término enésimo no tiende a cero cuando n -+ 0Ci.

b)

1 1 1 1a) JT+l + 22 + 1 + 32 + I + 42 + 1 +

I 1El término general es u. = -2-- < -,

n + 1 n

1El límite del término enésimo es lim - = O; sin embargo, esto no quiere decir que la serie sea convergente.

"-00 nEn realidad la serie es divergente, como se demuestra en el Problema 14d).

1 2 3 4 (-1r-1ne) 2 - 3 + 4 - "5 + . .. n + 1 + ...

La serie es divergente, ya que el término enésimo no tiende a cero cuando n -+ 0Ci.

CRITERIO DEL COCIENTE

14. Hallar el carácter de las series siguientes:

a)1 I I I 1 1 I 1 I

1+-+-+-+ ... e) +2"3+"33+43"+'" e) + 2'72 + 31/2 + 41/2 + ...4 16 64

3 9 27 I I 1 5 5 51+2+4+8+", d) 1+2+3+4+ f) 5+22+32+42"+'"b)

a) Es una serie geométrica de r = 1/4 < 1, luego es convergente.b) Es una serie geométrica de r = 3/2 > 1, luego es divergente.e) Es una serie en la que p = 3 > 1, luego es convergente.d) Es una serie en la que p = 1 (serie armónica), luego es divergente.e) Es una serie en la que p = 1/2 < 1, luego es divergente.f) Es igual a 5 veces una serie en la que p = 2 > 1, luego es convergente.

15. Hallar el carácter de las series siguientes, aplicando el criterio de comparación.

La serie es convergente, ya que es término a término menor que la serie convergente (p = 2 > 1)1 1

+22+32+

3 4 5 6b) N + M + M + s:7+ ...

n + 2El término general es > --

(n + l)(n + 3) n + 3

1 I ILa serie dada es divergente, ya que es término a término mayor que la serie divergente 4 + "5 + (; +

I 1 I Ie) N + 2' 22 + 3' 23 + 4' 24 + ...

I IEl término general es u. = n2' ;¡; 2n

http://carlos2524.jimdo.com/

Page 298: Algebra Superior Murray R Spiegel

1'2 2'3 3·4 4'5e) 32 + 33 + Y + Y + ...

n(n + 1)U"=~'

(n + I)(n + 2)U"+l =

d

290 SERIES INFINITAS

La. serie dada es término a término menor o igual a la serie geométrica convergente (r = 1/2 < 1)1 1 1

"2 + '4 + "8 + . .. luego es convergente.

Jn+I Jn 1El término general es Un = --- > -- = ---.n

n n n

La serie dada es término a término mayor que la serie divergente (p = 1/2 < 1)

luego es divergente.

eJ

1 1 1e) 1 + '4 + "7 + 10 + ...

. . 1 1El termmo general es Un = --- > -

3n - 2 3n

La serie dada es término a término mayor que un tercio de la serie armónica divergente (p = 1)

1 11 + "2 + '3 + . .. luego es divergente.

es

. . n + 1 1El termmo general es Un = 3 < n3(n + 2)n

f)

1 1La serie es término a término menor que la serie convergente (p = 3> 1) 1 + 23 + 33 + . .. luego es

convergente.

CRITERIO DEL COCIENTE

16. Determinar el carácter de las series siguientes aplicando el criterio del cociente. Si con este criterio se llega a uncaso dudoso, aplicar el criterio de comparación.

lo:

n + 2U =--" 2"+ 1 '

n + 3Un+1 = 2n+2 y

Un+ 1 n + 3 2n+ 1 n + 3-;;,:-= 2n+2.· n + 2 = 2(n + 2)

SERIE~

17. H¡

IU.+ 11 1 n + 3 1Luego lim -- = - lim -- = - = R < 1 y la serie es convergente .•_., u. 2 n-., n + 2 2 a)

3'u,. = ;;¡,

3"+ 1

U.+1 = (n + 1)4 yb)

Luego lim IU'+II = 3 lim (_~)4 = 3 = R> 1 y la serie es divergente."-00 u,. "_a:> n + 1

e)

. Iu'+ 11 1. n + 2 1LuegoIim -- = - hm -- = - = R < 1 y la serie es con~ergente."-a:> ~ 3,,_«> n 3

290 SERIES INFINITAS

La serie dada es término a término menor o igual a la serie geométrica convergente (r = 1/2 < 1) 1 1 1 2 + 4 + "8 + . .. luego es convergente.

Jn+t Jn 1 El término general es Un = --- > - = --.¡2

n n n

1 1 La serie dada es término a término mayor que la serie divergente (p = 1/2 < 1) 1 + 21 /2 + 31/2 + .. .

luego es divergente.

1 1 1 e) 1 + 4 + "7 + 10 + ...

. . 1 1 El termmo general es Un = --- > -

3n - 2 3n

La serie dada es término a término mayor que 1m tercio de la serie armónica divergente (p = 1) 1 1

1 + 2 + 3' + . .. luego es divergente.

. . n + 1 1 El termmo general es Un = 3 < -

(n + 2)n n3

1 1 La serie es término a término menor que la serie convergente (p = 3 > 1) 1 + 23 + 33 + . .. luego es

convergente.

CRITERIO DEL COCIENTE

16. Determinar el carácter de las series siguientes aplicando el criterio del cociente. Si con este criterio se llega a un caso dudoso, aplicar el criterio de comparación.

3 4 5 6 a) 2I + 23 + 24 + 2S + . ..

n + 2 u" = 2"+1 '

n + 3 uo +! = 20 + 2 Y

uo +1 _n+3 . 20+1 -~ un - 2n+2 . n + 2 - 2(n + 2)

IU'+ 111 n+31 Luego lim - - = - lim -- = - = R < 1 y la serie es convergente.

o-oc> Uo 2 n_ oc> n + 2 2

30

u" = ;;¡, 3n+ 1

Uo + 1 = (n + 1)4 y

. IUo + 11 . n 4 Luego hm -- = 3 hm (--) = 3 = R> " ..... 00 UII "-00 n + 1

1·2 2·3 3·4 4·5 e) 31+]3+7+3'+ . . .

Luego hm -- = - hm -- = - = R < 1 . Iuo + 11 l . n + 2 1 "- 00 ~ 3"""' 00 n 3

y la serie es divergente.

n(n + 1) (n + I)(n + 2) u"=~ ' U"+l =

y la serie es con~ergente.

http://carlos2524.jimdo.com/

Page 299: Algebra Superior Murray R Spiegel

SERIES INFINITAS 291

22n+2

Un+1 = 32"+3

. IU'+11 4Luego hm -- = - = R < 1

11-00 Un 9y la serie es convergente.

1 1 1e) l' 22 + 2' 32 + 3' 42 + ...

1 1 U'+1 n(n + 1)Un = n(n + 1)2' U. + 1 = (n + 1)(n + 2)2' ~ = (n + 2f

n(n + 1) n2 + nLuego lim ---2 = lim -,.2-----,

a-e co (n + 2) a-e co n + 4n + 4lim 1 + l/n = 1 Y estamos en caso de dudaa-e co 1 + 4/n + 4/n2

1 1 1 1 1Ahora bien, 2~ ---2 para todo n ~ 1. Como ., + ., + ., + ... converge, la serie dada

n(n + 1) - (n + 1) - 2 3 4es convergente.

1u. = ;¡- para todo n ~ O

1 1(O! = 1), u.u = (n + I)! = n!.(n + 1) y Un+1 =--

Un n + 1

. 1Luego hm -- = O Y la serie es convergente.

n-';' n + 1

La suma de la serie dada es igual al número e cuyo valor es 2,71828, aproximadamente, y es la base delos logaritmos naturales o neperianos.

SERIES ALTERNADAS

17. Hallar el carácter de las series siguientes:

1 1 1a) 1 - 22 + 32 - 42 + ...

Como Iu.+ d < Iunl y lim un = O, la serie es convergente.

1 1Se tiene Iunl = (n + 1)(n + 2) Y lu.+11 = (n + 2)(n + 3)

Como lu.+ 11 < lu.1 y lim u. = O, la serie es convergente.

1 234e) '3 - '5 + '1 - 9 + ... n n + 1

Se tiene lunl = -2-- Y lu.+ 11= -2n 3n+l +2

n 1Como lim -2n 1 = "2 + O, la serie es divergente.

"-10 +

. IU'+II 4 Luego 11m -- = - = R < 1 n"'ce Un 9

SERIES INFINITAS

y la serie es convergente.

22n+2

Un+ 1 = 3211 + 3

291

1 1 1 e) 1.22 + 2 . 32 + 3 . 42 + ...

1 1 u. + 1 n(n + 1) u. = n(n + 1)2' U.+ 1 = (n + I)(n + 2)2 ' --;;,;- = (n + 2)2·

n(n + 1) n2 + n 1 + l /n Luego lim ---2 = lim = lim = 1 Y estamos en caso de duda

._'" (n + 2) ._'" n2 + 4n + 4 ._'" 1 + 4/n + 4/n2

1 1 1 1 1 . Ahora bien, 2 :;; ---2 para todo n ~ 1. Como 2" + 2" + 2" + . . . converge, la sene dada

n(n + 1) - (n + 1) - 2 3 4 es convergente.

1 1 1 u. = -;;! para todo n ~ O (O! = 1), U.+ 1 = (n + I)! = n!. (n + 1) Y

UII + 1 =--

u. n + 1

. 1 Luego 11m - - = O Y la serie es convergente .

• _~ n + 1

La suma de la serie dada es igual al número e cuyo valor es 2,71828, aproximadamente, y es la base de los logaritmos naturales o neperianos.

SERIES ALTERNADAS

17. Hallar el carácter de las series siguientes :

. 1 1 Se tiene lu.1 = n2 y lu.+11 = (n + 1)2

Como Iu.+ 11 < lu.1 y lim u. = O, la serie es convergente.

1 1 1 1 b) M - N + M - s:6 + ...

1 1 Se tiene lu.1 = (n + I)(n + 2) Y lu.+11 = (n + 2)(n + 3)

Como lu.+ 11 < lu.1 y lim u. = O, la serie es convergente.

1 234 e) "3 - "5 + "1 - "9 + .. .

n 1 Como lim -2n = -2 + O,

• _'" + 1

n n + 1 Se tiene lu.1 = -2-- Y lu.+11 = -2n 3

n + 1 +

la serie es divergente .

http://carlos2524.jimdo.com/

Page 300: Algebra Superior Murray R Spiegel

292 SERIES INFINITAS

1 1La serie dada es convergente, ya que --- < - y2n + 2 2n lim ~=O.

11....• 00 2n

e

CONVERGENCIA ABSOLUTA Y CONDICIONAL

18. Determinar si las series siguientes son absoluta o condicionalmente convergentes.

1 1Se tiene Iunl = -'/2 Y lun+,1 = ---

n (n + 1)'/2

1 1 1La serie de valores absolutos 1 + .JÍ + J3 + J4 + ... es divergente, ya que en ella es p = ~ < 1.

La serie dada es convergente porque Iun+ ,1 < Iunl y lim Un = O.

Luego la serie dada es condicionalmente convergente.

1 1 1La serie de valores absolutos 1+ 3' + 3' + 3' + ... es convergente, ya que en ella es p = 3. Luego la

234serie dada es absolutamente convergente (y, por tanto, convergente).

e) ..fifi - .j4í3 + .j5i4 - J6i5 + ...n + 2

Se tiene Iunl = (--1)'/2n+ SERIE:

Como lim (n + ~'/2 = 1 -J O, la serie es divergente.a-e ec n + 1

21. H.

1 1Se tiene lunl = 2n y lun+,1 = 2n + 2 a)

1 1 1 1 1 1 1 1La serie de valores absolutos 2 + '4 + 6 + 8' + ... = 2(1 + 2 + '3 + '4 + ... ) es un medio de la serie

armónica (o sea, un medio de la serie con p = 1) Y es divergente

tarr

Luego la serie dada es condicionalmente convergente.

. I I 1 I I 119. Demostrar que la sene - + - - - - - + - + - - es convergente.12 22 31 42 52 62 ...

. 1 1 1 1La sene de valores absolutos 12 + 22 + 32 + 42 + ... es convergente, ya que en ella es p = 2 > l.

Luego la serie dada es absolutamente convergente y, por tanto, convergente.

20. Hallar el carácter de las series siguientes aplicando el criterio del cociente:

1 2 3 4a) 2 - '4 + 8' - 16 + ...

. n n + 1Se tiene lunl = 2n Y lun+ ,1 = 2n+ ,

en dserieen e

292 SERIES INFINITAS

CONVERGENCIA ABSOLUTA Y CONDICIONAL

18. Determinar si las series siguientes son absoluta o condicionalmente convergentes.

1 1 Se tiene lu.1 = -'/2 Y lu.+,1 = ---

n (n + 1)' /2

1 1 1 La serie de valores absolutos 1 + J2 + J3 + .j4 + ... es divergente, ya que en ella es p = ~ < l.

La serie dada es convergente porque lu.+ 11 < lu.1 y lim u. = O.

Luego la serie dada es condicionalmente convergente.

1 1 1 La serie de valores absolutos 1 + 3" + 3" + 3" + .. . es convergente, ya que en ella es p = 3. Luego la

234

serie dada es absolutamente convergente (y, por tanto, convergente).

e) foi - fti3 + .j5i4 - .j6j5 + .. . n+2

Se tiene lu.1 = (_-1)' /2 n+

. n + 2. Como hm ( __ :)' /2 = 1 +- O, • _ 0) n + 1

la serie es divergente .

1 1 1 1 d) 2 - ¡ + "6 - "8 + . . .

. 1 1 1 1 1 1 1 1 La sene de valores absolutos 2 + ¡ + "6 + "8 + ... = 2(1 + 2 + "3 + ¡ + . . . ) es un medio de la serie

armónica (o sea, un medio de la serie con p = 1) y es divergente

1 1 1 La serie dada es convergente, ya que - -- < - y lim - = O.

2n + 2 2n ._ 0) 2n

Luego la serie dada es condicionalmente convergente .

. 1 1 1 l l l 19. Demostrar que la serIe - + - - - - - + - + - - es convergente. 12 22 Y 42 52 62 . .•

. l l l l La serIe de valores absolutos 12 + 2I + 32 + ;jI + . .. es convergente, ya que en ella es p = 2 > l .

Luego la serie dada es absolutamente convergente y, por tanto , convergente.

20. Hallar el carácter de las series siguientes aplicando el criterio del cociente:

1 2 3 4 a) 2 - ¡ + "8 - 16 + . ..

. n n + 1 Se tiene lu.1 = 2- y lu.+'¡ = y+T

http://carlos2524.jimdo.com/

Page 301: Algebra Superior Murray R Spiegel

SERIES INFINITAS 293

IUn+ 11 n + 1 2n n + l llim -- = lim --' - = lim -- = - = R < ln-a;> Un ,. ....•00 2"+ 1 n n-oo 2n 2 y la serie alternada es convergente.

l 2 3 4Como el criterio del cociente también demuestra que la serie de valores absolutos "2 + "4 + 8" + 16 +es convergente, la serie dada es absolutamente convergente.

xnSe tiene Un = --;; y Un + 1

,x" + 1

n + 1

3n

Se tiene /u / - -- yn - (2n)!

)n+ 1

/Un+1/ = (2n + 2)!

. Ilun+11 . 3(2n)!, 3(2n)!hm -- = hm = hmn-<Xl Un a-e co (2n + 2)! n-<Xl (2n + 2){2n + 1 )(2n)!

lim 3 = O = R < 1n-<Xl 4n2 + 6n + 2

Luego la serie es absolutamente convergente.

e)l l l

- 22/3 + y/3 - 42/3 + . lSe tiene /un/ = 2/3 Y

nl

/un+" = (n + 1)2/3

l· IUn+ll u ( n )2/3 l1m -- = 1m -- =n ....•cc Un "-00 n + 1 y estamos en caso de duda.

Pero, como /un+ 1/ < /un/ y lim Un = O. la serie alternada dada es condicionalmente convergente,

SERIES POTENCIALES

21. Hallar el campo de convergencia de las series siguientes:

lim I~I= lim 1,x"+ 1,·..;1 = /x/lim n/x/n-s co un a-s co n + 1.\ "-OCI n + I

La serie es convergente para /x/ < 1, o sea -1 < x < 1,Y divergente para /x/ > l o x > l Y x < - l. Es-tamos en caso de duda para /x/ = l o x= l y .v = _l.

l l lPara x = l. la serie es l + "2 + "3 + "4 + que es divergente.

l l lPara .v = - l. la serie es - l + "2 - "3 + "4 - . .. que es convergente.

Luego el campo de convergencia de la serie dada es - l ~ .r < l. Se representa gráficamente por

----~·~------------------~Or--------1 1

en donde la línea gruesa representa el intervalo de convergencia y la de trazo fino los intervalos en los cuales laserie diverge. El círculo negro indica que la serie es convergente en el extremo - l y el blanco que es divergenteen el extremo 1.

SERIES INFI NITAS 293

r .+ 1 lim -- ' - = r R I \

u \ n+12· 11 + 1 I "~"! 7. = "- 00 2" + 1 n 1m 2;- = 2: = <

y la serie a lternada es convergente.

I 2 3 4 Como el criterio del cociente también demuestra que la serie de valores absolutos "2 + "4 + "8 + 16 +

es convergente, la serie dada es absolutamente convergente.

e)

3· Se tiene lu I - -- y • - (2n)!

3" + t lu. +d = (2n + 2)!

. I\u.+ 1\ . 3(2n)! . 3(2n)! 3 = O = ~~: 7. = ~~: (2n + 2)! = ~~: (2n + 2)(2n + 1)(2n)! = Iim 4n2 + 6n + 2 R < I

Luego la serie es absolutamente convergente.

I I I - 22/3 + y 13 - 4213 +

l· \U. +l\ l· ( n )213 l 1m -- = 1m -- = ti ..... a:: Un ti ..... ce n + 1

. 1 Se tIene lu.1 = 2/3 Y

11

y estamos en caso de duda.

Pero. como lu. + 11 < lu.1 y Iim u. = O. la serie a lternada dada es condiciona lmente convergente.

SERIES POTENCIALES

21. Hallar el campo de convergencia de las series siguientes :

x· Se tiene u. = - y u. + 1

n

lim \u.+ l\ = lim \.\""+1 .. ~\ = Ixllim n Ixl

n ..... cc Un " ..... :;0 n + l .~ n- oo n + 1

.\""+ 1

n+1

La serie es convergente para Ixl < 1, o sea -1 < x < l . Y divergente para Ixl > l o x > l Y x < - l . Es­tamos en caso de duda para Ix l = l o x = I Y x = - l .

l I I Para x = 1, la se rie es I + "2 + "3 + "4 + que es divergente.

I I I Para x = - l. la serie es - l + "2 - "3 + "4 - que es conve rgente.

Luego el campo de convergencia de la serie dada es - l ~ x < l. Se representa gráficamente po r

---~.~---------------------<o~-----1 1

en donde la línea gruesa representa el interva lo de convergencia y la de trazo fi no los intcr\"alos en los cuales la serie diverge. El circulo negro indica que la serie es convergente en el extremo - l yel hlal!l:o que es divergente en el extremo l.

http://carlos2524.jimdo.com/

Page 302: Algebra Superior Murray R Spiegel

294 SERIES INFINITAS

28.

. x"Se tiene u, = 2n

.x"+ I

Y Un+1 = y;+l

lu 1 I·x"+ I 2"1 IXI Ilim ~ = lim ---;;+¡' - = lim - = -Ixl11 ....• 00 Un n-oo 2 x" "-00 2 2

-0-----0--2 2 22.

La serie es convergente para Ii Ixl < l o - 2 < x < 2.

Para x = - 2, la serie es - l + I I + I que es divergente.Para x = 2, la serie es I + I + l + l + . .. que es divergente.

23.

Luego la serie dada es convergente en el intervalo.

e)x"- I

Se tiene u" = --- y(n - I)! 24.

Ix" (n - 1)!1 Ix(n - I)!I 1lim l' -::-¡- = lim I = Ixl lim - = O para todo x.

"_00 n. x" "_00 n(n - 1). "_00 n

Luego la serie es convergente para todos los valores de x, o sea - 00 < x < co,

25.

lim I x"+ 1 1/2 • n1

/

2

1 = Ixllim (_n_)1/2 = IxI-'_00 (n + 1) x" "_00 n + 1

~<:~--- ••..--1 1

La serie es convergente para Ixl < 1 o bien -1 < x < 1.

1 1 1Para x = 1, la serie es 1 - ----= + ----= - ----= + ... que es convergente.

J2 J3 J41 1 1

Para x = -1, la serie es -1 - - - - - - - que es divergente.fifiJ4···Luego el campo de convergencia es - 1 < x ;;¡; 1.

26.

e) x + 2!x2+ 3!x3+ 4!x4+ ...

I(n + 1)1x"+ 11lim I . = Ixl lim (n + 1) = 00 excepto para x ';= O.

"-00 n.x" 11-00

La serie converge solamente para x = O. 27. l

(x - W (x - W (x - 3)4f) (x-3)--2-+-3---4-+ ...

lim I(X - 3r+1

• __ n_, I = Ix _ 311im _n_ = [x - 31._00 n + 1 (x - 3) '_00 n + 1

La serie converge para Ix - 31 < 1 Ó 2 < x < 4.

Para x = 2, la serie es - 1 - 1/2 - 1/3 - 1/4 - . .. que es divergente.Para x = 4, la serie es 1 - 1/2 + 1/3 - 1/4 + . .. que es convergente.

Luego la serie dada es convergente en el intervalo.

o •2 4

294

e)

SERIES INFINITAS

X" X,,+I

Se tiene u" = Tn y U"+I = F'

lim IU"+II= lim I·~::'~I= lim 1~1=~lxl ~ " ..... 00 Un ,. ..... IX) 2 X II "-- 00 2 2

La serie es convergente para Yz Ixl < 1 o - 2 < x < 2.

Para x = - 2, la serie es - l + + que es divergente. Para x = 2, la serie es 1 + l + 1 + 1 + . .. que es divergente .

Luego la serie dada es convergente en el intervalo.

x" - 1

Se tiene u" = --- y (n - I)!

lim I~'~~I = lim Ix (n - I)!I = Ixllim ~ = O para todo x. "- a> n! x" - 1 "- a> n(n - I)! "-a> n

Luego la serie es convergente para todos los valores de x, o sea - 00 < x < oo .

Se tiene lu"1 = I~I y IU"+11 = I x" +1 I n1/2 (n + 1 )1 /2

I x"+ 1 nl /21 n

lim 1/2 . - = Ixl lim ( __ )1 /2 = Ixl . "-a> (n + 1) x" "- a> n + 1 ~<:~---.... -

-1 1

La serie es convergente para Ixl < 1 o bien -1 < x < 1.

Para x = 1, la serie es 1 1 1

1 - fi + J3 - J¡ + ... que es convergente.

1 1 1 Para x = - 1, la serie es - l - - - - - - - que es divergente. fifij4 · ··

Luego el campo de convergencia es - 1 < x ~ 1.

e) x + 2!x2 + 3!x3 + 4!x4 + . .. Se tiene lu"1 = In!x"1 y lu"+aI = I(n + 1)!x"+11

I

(n + I)Ix"+11 lim 1 . = Ixl lim (n + 1) = 00 excepto para x '7' O. "-00 n . ~ "-00

La serie converge solamente para x = O.

(X -3)2 (X_3)3 (X-3)4 f) (x - 3) - --2- + --3- - --4- + ... 1 1-I(X- 3f

l I 1_I(X-3f +1

1 Un - Y U,,+ 1 -n n + 1

I(X - 3f+ 1 n I n

lim . --o = IX - 31 1im -- = IX - 31 "-a> n + 1 (x - 3) "-a> n + 1 o • 2 4 La serie converge para Ix - 31 < 1 Ó 2 < x < 4.

Para x = 2, la serie es - 1 - 1/2 - 1/3 - 1/4 - . .. que es divergente. Para x = 4, la serie es 1 - 1/2 + 1/3 - 1/4 + . . . que es convergente.

Luego la serie dada es convergente en el intervalo.

http://carlos2524.jimdo.com/

Page 303: Algebra Superior Murray R Spiegel

1 1 1 1 1 1a) 1+2:+4+8+'" d) 1-3+5-7+",

1 3 5 7 x2 x3 x4b) 2:+4+8+16+'" e) x-2+3--¡+···

2 4 6 8f) X x2 x3

e) -+--+-+-+ ... 1 + 2'3! + 22'5! + 23'7! + ...1·3 3'5 5'7 7'9

SERIES INFINITAS 295

PROBLEMAS PROPUESTOS

22. Escribir los cuatro primeros términos de la sucesión cuyo término enésimo es el que se indica:

2na) --

n + 23n - 2

b) -2"-(-Ir

e) --n!

nd) 2n2 _ 1

x2"e) --

2n + 1

23. Escribir los cuatro primeros términos y el término (n + 1) de las series cuyo término enésimo es el dado.

na) 3" + 1

b) Jnn + 1

(_1)"-1e)~

2n - 1d) 2n + 1 e)

n(n + 1)(n + 2)

x"-I

24. Hallar un término enésimo de las sucesiones siguientes:

234a) 3'"5' 7' "9'

1 1 1 1b) 2+1'22+1'23+1'24+1'

345 6e) M'H'~'M'

246 8d) 22-1'23-1'24_1'2'_1'

25. Hallar un término enésimo y el término (n + 1) de las series siguientes:

26. Escribir las series representadas por los símbolos siguientes:

s 1 "" 1 co (_ 1)"- 1 (x - 1)"a) ¿

2n + 2e) ¿

(2n + 1)2e) r n!n=1 n= 1 n= 1

3 (-1)" co x"-Ib) ¿ d) ¿

(n - 1)3,.=1 n n=2

. 2 5 8 11 3n - 127. El término general de la sucesion 3' 4' 5' 6 viene dado por u" = ~ .

a) Escribir los términos números 10, 20, 100, I 000, !O 000.¿A qué límite parece tender cuando n __ co'!

b) Aplicando la definición de límite, comprobar que el límite anterior es correcto.

e) Hallar este límite aplicando los teoremas de los límites.

28. Hallar el límite de las sucesiones siguientes cuando n --> 00, siendo el término enésimo (o término general) el quese indica.

5n - 2a) 3n + 1

ne) n2 + 1 d)

30nV;-:;¡

http://carlos2524.jimdo.com/

Page 304: Algebra Superior Murray R Spiegel

296 SERIES INFINITAS

29. Determinar si las sucesiones siguientes están acotadas. son monótonas crecientes o decrecientes. y si tienenlímite.

a) 3, 3,3, 3,33, 3.333,h) 1.9. 2.1, 1,99,2,01. 1,999,2,001,e) 1. -2, 3, -4. 5. -6 ....

d) 1, 2, 1, 3, 1, 4, 1, 5,2 345

e) 3'"5' -::,' 9' ...

2n- 1

30. El término general de una serie es u. = y'36.

a) Escribir sus cuatro primeros términos.

h) Llamando S. a la suma de sus n primeros términos. hallar. en forma decimal, S" S2, S3, ... , S,o. ¿Cuál pa-rece ser el límite cuando n -+ oo?

24.

e) Comprobar el límite anterior efectuando la suma de los n primeros términos. ¿Es convergente?37.

31. Sea la serie l - 3 + 5 - 7'+ ...

a) Hallar S., suma de sus n primeros términos.b) ¿Cuál es el lim S.?

.-oce) ¿Es convergente')

32. Determinar si son correctas o no las conclusiones siguientes:l l 1 l

a) La serie "3 + "5 + -::,+ 9 + ... es convergente porque los términos sucesivos van disminuyendo constan-

temente. 38.

. l 2 3 4b) La sene 3 + "5 + -::,+ 9 + ... es divergente porque el término enésimo no tiende a cero .

. l l 1 le) La sene 12 + '22+ )"2+ 4"2+ ... es convergente porque el término enésimo tiende a cero. 39.

33. Hallar el carácter de las -series

2 2 2 l l l la) 2+3+9+2"7+'" d) 12+22+)"2+4"2+ ...

b)4 42 43 l l l l

l + - + (-) + (-) + ... e) -+-+-+-+3 3 3 ~ 12 13 14

3 3 3 3 l l l le) 2+'22+2'+24+ f) 16+20+30+40+'"

SOL34. Hallar el carácter de las series siguientes aplicando el criterio de comparación.

22.1 l l la) 2+1 + 22 + l + 23 + l + 24 + l + ..

l l l lh) 1 + 3 + "5 + -::,+ ..

3 4 5 6e) --- + -- + --- + --- + ...

33 + l 43 + l 53 + 1 63 + 1

l 1 1 ld) 2' 14 + 4' 24 + 6· 34 + 8· 44 + ...

2 3 4 5e) 22--=,,--¡+ 32'::'1 + 42 _ l + 52 _ l +

23.l 1 l l.n ---- + --- + -- + -- +

"/¡-'2 '.IN fr4 Fs ...

35. Hallar el carácter de las series siguientes aplicando el criterio del cociente. Si se llega a un caso dudoso, aplíqueseel criterio de comparación.

1 l l la) ---+--+--+--+

2 . 3' 3 . 33 4 . 34 5 . 35

http://carlos2524.jimdo.com/

Page 305: Algebra Superior Murray R Spiegel

SERIES INFINITAS 297

X X2 X3 x'b) -+--+--+--+ ...

3 . 1 32. 2 33. 3 3' . 4:e + 2 (x + 2)2 (X + 2)3 (x + 2)'

e) .fi + .fi. + .fi + J4 + ...

3 4 5 6e) 22 + 32 + 42 + 52 + ...

1 1 I Ie) N + M+ N + ¡;:-:¡ + ...

13 132 133 13,f) 2(2) + 4(2) + 6(2) + S(2) + ...

2 3 4 5d) 1! + 2T + 3T + 4T + ...

36. Hallar el carácter de las series siguientes:

I I Ia) I - "3 + "5 - "7 + ...

I I 1 Ib) "3 - 9 + 27 - 81 + ...

1 234e) 2 - "3 + 4 - "5 + ...

2 3 4 5d) 32 - 42 + 52 - 62 + ...

37. Determinar si las series siguientes son absoluta o condicionalmente convergentes.

1 I I 1a) "3- 32 + 33 - 34 + ...

1 1 1 1b) 4 - S + 12 - T6 + ...

1 1 1 1e) 2 + I - 22 + 1 + 23 + 1 - 24 + 1 + ...

2 345d) -¡-:-¡--¡:S+}:6-¡-:-:¡+'"

1 1 1 1e) 3' 22 - 4' 32 + 5 . 42 - 6 . 52 + .. ,

1 1 1 If) 1! -3T + 5! - 7! + .. ,

n-

38. Determinar si son ciertas o no las siguientes conclusiones:a) Una serie alternada en la que cada término, en valor absoluto, es menor que el anterior, es convergente,b) Una serie alternada absolutamente convergente es convergente.

39. Determinar el campo de convergencia de las series siguientes:

d) 2x - 4x2 + 8x3 - 16x' + ...

x3 :es x7

e) :e - "3 + 5" - 7' + . , ,

SOLUCIONES DE LOS PROBLEMAS PROPUESTOS

2 6 422. a) "3' 1, "5' "3

l 7 5b) 2' 1, s' S

1 I Ie) -1, 2' -6' 24

2 3 4d) 1'"7' 17' 3i

l 2 3 4 n+1 l l 1 1 (-1)"23. a) 4 + \O + 28 + 82 ; 3"+I + I

e) 2T-4T+6T-8T; (2n + 2)!.fi .fi. .fi J4 . Jn+I I 3 5 7 2n + 1

b) 2+-3-+4+-5-'~ d) -+-+-+-'--3 5 7 9 '2n + 3

eseI x ~ ~ ~

e) --- + --- + --- + ---' --------I . 2 . 3 2 . 3 . 4 3 . 4 . 5 4 . 5 . 6' (n + l)(n + 2)(n + 3)

n24. a) 2n + 1 b)

2" + 1n + 2

e)(n + 3)(n + 4)

2nd) 2"+I _

http://carlos2524.jimdo.com/

Page 306: Algebra Superior Murray R Spiegel

n

(-lrx"+1Un+1 = n + 1

298 SERIES INFINITAS

25. a)(-Irl (-Ir

d) u. = z;;-=-¡; u.+ I = 2n + 1

2n-1 2n+1b) u. = -2-'-; U.+I = ~

2n 2n + 2e) u. = (2n _ 1)(2n + 1); U.+I = (2n + 1)(2n + 3)

x"-I x"f) U. = 2' I (2n _ 1)'; u. + I = 2' (2n + 1)'

1 1 1 1e) 32 + 52 + 72 + 92 + ...

(x - 1) (x - 1)2 (x - 1)3 (x - 1)4e) --1'- - --2-'- + --3-'- - --4-'- + ...

27. a) UIO = 29/12 = 2,4166 ... , U20 = 59/22 = 2,6818 ... , UIOO = 299/102 = 2,9313 ... ,UIOOO = 2,9930 ... , UIO 000 = 2,9993. . . Tiende a 3.

28. a) 5/3 b) 4 e) ° d) 12 e) 1/2

29. a) acotada, monótona creciente, tiene límite.b) acotada, no es ni monótona creciente ni decreciente, tiene límite.e), d) no está acotada, no es ni monótona creciente ni decreciente, no tiene límite.e) acotada, monótona decreciente, tiene límite.

1 2 4 830. a) "3 + "9 + 27 + 8J

b) SI = 0,33333 , S2 = 0,55555 , S3 = 0,70370 , S4 = 0,80246 , Ss = 0,86831. ..S6 = 0,91220 , S7 = 0,94147 , S. = 0,96098 , S9 = 0,97398 , SIO = 0,98265 ...Tiende a l.

e) Suma de n términos, S. = 1 - (2/3)0. lim So = 1, la serie converge.

31. a) SI = 1, S2 = -2, S3 = 3, S4 = -4, b) lim So no existe. e) La serie diverge.

32. a) No b) Sí e) No

33. a) convergente b) divergente e) convergente d) convergente e) divergente f) divergente

34. a) convergente b) divergente e) convergente d) convergente e) divergente f) divergente

35. a) convergente b) divergente e) divergente d) convergente e) convergente f) divergente

36. a) convergente b) convergente e) divergente d) convergente

37. a) absolutamente convergente d) condicionalmente convergenteb) condicionalmente convergente e) absolutamente convergentee) absolutamente convergente f) absolutamente convergente

38. a) falsa b) cierta\

39. a) -1 :;'i;x:;'i; 1 e) -1 :;'i;x ~ 1 e) -3:;'i;x<-1

b) -3 :;'i;x < 3 d) --! < x < t f) -oo<x<oo

http://carlos2524.jimdo.com/

Page 307: Algebra Superior Murray R Spiegel

APENDICEAPENDICE

http://carlos2524.jimdo.com/

Page 308: Algebra Superior Murray R Spiegel

Tabla de logaritmos decimales

300

N o 1 2 3 4 5 6 7 8 910 0000 0043 0086 0128 0170 0212 0253 0294 0334 037411 0414 0453 0492 0531 0569 0607 0645 0682 0719 075512 0792 0828 0864 0899 0934 0969 1004 1038 1072 110613 1139 1173 1206 1239 1271 1303 1335 1367 1399 143014 1461 1492 1523 1553 1584 1614 1644 1673 1703 173215 1761 1790 1818 1847 1875 1903 1931 1959 1987 201416 2041 2068 2095 2122 2148 2175 2201 2227 2253 227917 2304 2330 2355 2380 2405 2430 2455 2480 2504 252918 2553 2577 2601 2625 2648 2672 2695 2718 2742 276519 2788 2810 2833 2856 2878 2900 2923 2945 2967 298920 3010 3032 3054 3075 3096 3118 3139 3160 3181 320121 3222 3243 3263 3284 3304 3324 3345 3365 3385 340422 3424 3444 3464 3483 3502 3522 3541 3560 3579 359823 3617 3636 3655 3674 3692 3711 3729 3747 3766 378424 3802 3820 3838 3856 3874 3892 3909 3927 3945 396225 3979 3997 4014 4031 4048 4065 4082 4099 4116 413326 4150 4166 4183 4200 4216 4232 4249 4265 4281 429827 4314 4330 4346 4362 4378 4393 4409 4425 4440 445628 4472 4487 4502 4518 4533 4548 4564 4579 4594 460929 4624 46<:9 4654 4669 4683 4698 4713 4728 4742 475730 4771 4786 4800 4814 4829 4843 4857 4871 4886 490031 4914 4928 4942 4955 4969 4983 4997 5011 5024 503832 5051 5065 5079 5092 5105 5119 5132 5145 5159 517233 5185 5198 5211 5224 5237 5250 5263 5276 5289 530234 5315 5328 5340 5353 5366 5378 5391 .5403 5416 542835 5441 5453 5465 5478 5490 5502 5514 5527 5539 555136 5563 5575 5587 5599 5611 5623 5635 5647 5658 567037 5682 5694 5705 5717 5729 5740 5752 5763 5775 578638 5798 5809 5821 5832 5843 5855 5866 5877 5888 589939 5911 5922 5933 5944 5955 5966 5977 5988 5999 601040 6021 6031 6042 6053 6064 6075 6085 6096 6107 611741 6128 6138 6149 6160 6170 6180 6191 6201 6212 622242 6232 6243 6253 6263 6274 6284 6294 6304 6314 632543 6335 6345 6355 6365 6375 6385 6395 6405 6415 642544 6435 6444 6454 6464 6474 6484 6493 6503 6513 652245 6532 6542 6551 6561 6571 6580 6590 6599 6609 661846 6628 6637 6646 6656 6665 6675 6684 6693 6702 671247 6721 6730 6739 6749 6758 6767 6776 6785 6.794 680348 6812 6821 6830 6839 6848 6857 6866 6875 6884 689349 6902 6911 6920 6928 6937 6946 6955 6964 6972 698150 6990 6998 7007 7016 7024 7033 7042 7050 7059 706751 7076 7084 7093 7101 7110 7118 7126 7135 7143 715252 7160 7168 7177 7185 7193 7202 7210 7218 7226 723553 7243 7251 7259 7267 7275 7284 7292 7300 7308 731654 7324 7332 7340 7348 7356 7364 7372 7380 7388 7396N o 1 2 3 4 5 6 7 8 9

No/a: Por razones tipográficas se ha conservado en estas tablas la notación decimalde la edición en inglés.

http://carlos2524.jimdo.com/

Page 309: Algebra Superior Murray R Spiegel

301

Tabla de logaritmos decimales

N o 1 2 3 4 5 6 7 8 955 7404 7412 7419 7427 7435 7443 7451 7459 7466 747456 7482 7490 7497 7505 7513 '1520 7528 7536 7543 755157 7559 7566 7574 7582 7589 7597 7604 7612 7619 762758 7634 7642 7649 7657 7664 7672 7679 7686 7694 770159 7709 7716 7723 7731 7738 7745 7752 7760 7767 777460 7782 7789 7796 7803 7810 7818 7825 7832 7839 784661 7853 7860 7868 7875 7882 7889 7896 7903 7910 791762 7924 7931 7938 7945 7952 7959 7966 7973 7980 798763 7993 8000 8007 8014 8021 8028 8035 8041 8048 805564 8062 8069 8075 8082 8089 8096 8102 8109 8116 812265 8129 8136 8142 8149 8156 8162 8169 8176 8182 818966 8195 8202 8209 8215 8222 8228 8235 8241 8248 825467 8261 8267 8274 8280 8287 8293 8299 8306 8312 831968 8325 8331 8338 8344 8351 8357 8363 8370 8376 838?69 8388 8395 8401 8407 8414 8420 8426 8432 8439 844570 8451 8457 8463 8470 8476 8482 8488 8494 8500 850671 8513 8519 8525 8531 8537 8543 8549 8555 8561 856772 8573 8579 8585 8591 8597 8603 8609 8615 8621 862773 8633 8639 8645 8851 8657 8663 8669 8675 8681 868674 8692 8698 8704 8710 '8716 8722 8727 8733 8739 874575 8751 8756 8762 8768 8774 8779 8785 8791 8797 880276 8808 8814 8820 8825 8831 8837 8842 8848 8854 885977 8865 8871 8876 8882 8887 8893 8899 8904 8910 891578 8921 8927 8932 8938 8943 8949 8954 8960 8965 897179 8976 8982 8987 8993 8998 9004 9009 9015 9020 902580 9031 9036 9042 9047 9053 9058 9063 9069 9074 907981 9085 9090 9096 9101 9106 9112 9117 9122 9128 913382 9138 9143 9149 9154 9159 9165 9170 9175 9180 918683 9191 9196 9201 9206 9212 9217 9222 9227 9232 923884 9243 9248 9253 9258 9263 9269 9274 9279 9284 928985 9294 9299 9304 9309 9315 9320 9325 9330 9335 934086 9345 9350 9355 9360 9365 9370 9375 9380 9385 939087 9395 9400 9405 9410 9415 9420 9425 9430 9435 944088 9445 9450 9455 9460 9465 9469 9474 9479 9484 948989 9494 9499 9504 9509 9513 9518 9523 9528 9533 953890 9542 9547 9552 9557 9562 9566 9571 9576 9581 958691 9590 9595 9600 9605 9609 9614 9619 9624 9628 963392 9638 9643 9647 9652 9657 966} 9666 9671 967;' 968093 9685 9689 9694 9699 9703 9708 9713 9717 9722 972794 9731 9736 9741 9745 9750 9754 9759 9763 9768 977395 9777 9782 9786 9791 9795 9800 9805 9809 9814 9818'96 9823 9827 9832 9836 9841 9845 9850 9854 9859 986397 9868 9872 9877 9881 9886 9890 9894 9899 9903 990898 9912 9917 9921 9926 9930 9934 9939 9943 9948 995299 9956 9961 9965 9969 9974 9978 9983 9987 9991 9996N o 1 2 3 4 5 6 7 8 9

http://carlos2524.jimdo.com/

Page 310: Algebra Superior Murray R Spiegel

302

Tabla de interés compuesto: (1 + itr-. 1% 1~% 1~% 2% 2~% 3% 4% 5% 6%

1 1. 0100 1. 0125 1. 0150 1. 0200 1. 0250 1. 0300 1. 0400 1. 0500 1. 06002 1. 0201 1. 0252 1.0302 1. 0404 1. 0506 1. 0609 1. 0816 1. 1025 1. 12363 1. 0303 1. 0380 1. 0457 1. 0612 1. 0769 1. 0927 1. 1249 1. 1576 1. 19104 1.0406 1. 0509 1. 0614 1. 0824 1. 1038 1. 1255 1. 1699 1. 2155 1. 26255 1. 0510 1.0641 1. 0773 1. 1041 1. 1314 1. 1593 1. 2167 1. 2763 1. 33826 1. 0615 1. 0774 1. 0934 1.1262 1.1597 1. 1941 1. 2653 1. 3401 1. 41857 1.0721 1. 0909 1. 1098 1. 1487 1. 1887 1. 2299 1. 3159 1. 4071 1. 50368 1. 0829 1. 1045 1. 1265 1.1717 1. 2184 1. 2668 1. 3688 1. 4775 1. 59389 1. 0937 1. 1183 1. 1434 1. 1951 1. 2489 1. 3048 1. 4233 1. 5513 1. 6895

10 1. 1046 1. 1323 1. 1605 1. 2190 1. 2801 1. 3439 1.4802 1. 6289 1. 7908

11 1. 1157 1. 1464 1. 1779 1. 2434 1. 3121 1. 3842 1. 5395 1. 7103 1. 898312 1. 1268 1. 1608 1. 1956 1. 2682 1. 3449 1. 4258 1. 6010 1. 7959 2.012213 1. 1381 1. 1753 1. 2136 1. 2936 1. 3785 1. 4685 1. 6651 1. 8856 2. 132914 1. 1495 1. 1900 1. 2318 1.3195 1. 4130 1. 5126 1. 7317 1. 9799 2.26091p 1. 1610 1. 2048 1. 2502 1. 3459 1. 4483 1. 5580 1. 8009 2.0789 2.396616 1. 1726 1. 2199 1. 2690 1. 3728 1. 4845 1. 6047 1.8730 2.1829 2.540417 1. 1843 1. 2351 1. 2880 1. 4002 1.5216 1. 6528 1. 9479 2.2920 2.692818 1. 1961 1. 2506 1. 3073 1. 4282 1. 5597 1. 7024 2.0258 2.4066 2.854319 1. 2081 1. 2662 1. 3270 1. 4568 1. 5987 1. 7535 2. 1068 2.5270 3.0256

20 1. 2202 1. 2820 1. 3469 1. 4859 1. 6386 1.8061 2.1911 2.6533 3.2071

21 1. 2324 1. 2981 1. 3671 1. 5157 1. 6796 1. 8603 I 2.2788 2.7860 3.399622 1. 2447 1.3143 1. 3876 1. 5460 1. 7216 1. 9161 ! 2.3699 2.9253 3.603523 1. 2572 1. 3307 1. 4084 1. 5769 1.7646 1. 9736 2.4647 3.0715 3.819724 1. 2697 1. 3474 1. 4295 1. 6084 1. 8087 2.0328 2.5633 3.2251 4.048925 1. 2824 1. 3642 1.4509 1.6406 1. 8539 2.0938 2.6658 3.3864 4.291926 1.2953 1. 3812 1. 4727 1.6734 1. 9003 2.1566 2.7725 3.5557 4.549427 1.3082 1.3985 1. 4948 1. 7069 1. 9478 2.2213 2.8834 3.7335 4.822328 1. 3213 1. 4160 1.5172 1. 7410 1. 9965 2.2879 2.9987 3.9201 5. 111729 1.3345 1. 4337 1. 5400 1. 7758 2.0464 2.3566 3.1187 4.1161 5.4184

30 1. 3478 1.4516 1.5631 1.8114 2.0976 2.4273 3.2434 4.3219 5.7435

31 1. 3613 1. 4698 1. 5865 1. 8476 2.1;;)0 2.5001 '3.3731 4.5380 6.088132 1. 3749 1. 4881 1. 6103 1.8845 2.2038 2.5751 3.5081 4.7649 6.453433 1.3887 1. 5067 1. 6345 1. 9222 2.2589 2.6523 3.6484 5.0032 6.840634 1. 4026 1. 5256 1. 6590 1. 9607 2.3153 2.7319 3.7943 5.2533 7.251035 1. 4166 1. 5446 1. 6839 1. 9999 2.3732 2.8139 3.9461 5.5160 7.686136 1. 4308 1. 5639 1. 7091 2.0399 2.4325 2.8983 4.1039 5.7918 8.147337 1. 4451 1. 5835 1. 7348 2.0807 2.4933 2.9852 4.2681 6.0814 8. 636138 1. 4595 1. 6033 1. 7603 2. 1223 2.5557 3.0748 4.4388 6.3855 9. 154339 1. 4741 1. 6233 1. 7872 2.1647 2.6196 3. 1670 4.6164 6.7048 9.7035

40 1. 4889 1. 6436 1.8140 2.2080 2.6851 3.2620 4.8010 7.0400 10.2857

41 1. 5038 1. 6642 1. 8412 2.2522 2.7522 3.3599 4.9931 7.3920 10.902942 1. 5188 1. 6850 1. 8688 2.2972 2.8210 3.4607 5.1928 7.7616 11. 557043 1.5340 1. 7060 1. 8969 2.3432 2.8915 3.5645 5.4005 8. 1497 12.250544 1. 5493 1. 7274 1. 9253 2.3901 2.9638 3.6715 5.6165 8.5572 12.985545 1. 5648 1. 7489 1. 9542 2.4379 3.0379

I3.7816 5.8412 8.9850 13.7646

46 1. 5805 1.7708 1. 9835 2.4866 3.1139 3.8950 6.0748 9.4343 14.590547 1. 5963 1.7929 2.0133 2.5363 3.1917 I 4.0119 6.3178 9.9060 15.465948 1. 6122 1. 8154 2.0435 2.5871 3.2715 I 4. 1323 6.5705 10.4013 16.393949 1. 6283 1. 8380 2.0741 2.6388 3.3533 4.2562 6.8333 10.9213 17.3775

50 1. 6446 1.8610 2.1052 2.6916 3.4371 4.3839 7. 1067 11. 4674 18.4202

http://carlos2524.jimdo.com/

Page 311: Algebra Superior Murray R Spiegel

Valor actual después de n periodos: (1 + i) - n

~ 1% la lH;, 2% 2~% 3% 4% 5% 6%1 .99010 .98765 .98522 .98039 .97561 .97087 .96154 .95238 .943402 .98030 .97546 .97066 .96117 .95181 .94260 .92456 .90703 .890003 .97059 .96342 .95632 .94232 .92860 .91514 .88900 .86384 .839624 .96098 .95152 .94218 .92385 .90595 .88849 .85480 .82270 .792095 .95147 .93978 .92826 .90573 .88385 .86261 .82193 .78353 .747266 .94205 .92817 .91454 .88797 .86230 .83748 .79031 .74622 .704967 .93272 .91672 .90103 .87056 .84127 .81309 .75992 .71068 .665068 .92348 .90540 .88771 .85349 .82075 .78941 .73069 .67684 .627419 .91434 .89422 .87459 .83676 .80073 .76642 .70259 .64461 .59190

10 .90529 .88318 .86167 .82035 .78120 .74409 .67556 .61391 .55839

11 .89632 .87228 .84893 .80426 .76214 .72242 .64958 .58468 .5267912 .88745 .86151 .83639 .78849 .74356 .70138 .62460 .55684 .4969713 .87866 .85087 .82403 .77303 .72542 .68095 .60057 .53032 .4688414 .86996 .84037 .81185 .75788 .70773 .66112 .57748 .50507 .4423015 .86135 .82999 .79985 .74301 .69047 .64186 .55526 .48102 .4172716 .85282 .81975 .78803 .72845 .67362 .62317 .53391 .45811 .3936517 .84438 .80963 .77639 .71416 .65720 .60502 .51337 .43630 .3713618 .83602 .79963 .76491 .70016 .64117 .58739 .49363 .41552 .3503419 .82774 .78976 .75361 .68643 .62553 .57029 .47464 .39573 .33051

20 .81954 .78001 .74247 .67297 . 61027 .55368 .45639 .37689 .31180

21 .81143 .77038 .73150 .65978 .59539 .53755 .43883 .35894 .2941622 .80340 .76087 .72069 .64684 .58086 .52189 .42196 .34185 .2775123 .79544 .75147 .71004 .63416 .56670 .50669 .40573 .32557 .2618024 .78757 .74220 .69954 .62172 .55288 .49193 .39012 .31007 .2469825 .77977 .73303 .68921 .60953 .53939 .47761 .37512 .29530 .2330026 .77205 .72398 .67902 .59758 .52623 .46369 .36069 .28124 .2198127 .76440 .71505 .66899 .58586 .51340 .45019 .34682 .26785 .2073728 .75684 .70622 .65910 .57437 .50088 .43708 .33348 .25509 .1956329 .74934 .69750 .64936 .56311 .48866 .42435' .32065 .24295 .18456

30 . 74í92 .68889 .63976 .55207 .47674 .41199 .30832 .23138 .17411

31 .73458 .68038 .63031 .54125 .46511 .39999 .29646 .22036 · 1642532 .72730 .67198 .62099 .53063 ' .45377 .38834 .28506 .20987 · 1549633 .72010 .66369 .61182 .52023 .44270 .37703 .27409 .19987 .1461934 .71297 .65549 .60277 .51003 .43191 .36604 .26355 .19035 .1379135 .70591 .64740 .59387 .511003 .42137 .35538 .25342 .18129 · 1301136 .69892 .63941 .58509 .49022 .41109 .34503 .24367 .17266 .1227437 .69200 .63152 .57644 .48061 .40107 .33498 .23430 .16444 .1157938 .68515 .62372 .56792 .47119 .39128 .32523 .22529 .15661 .1092439 .67837 .61602 .55953 .46195 .38174 .31575 .21662 · 14915 · ¡0306

40 .67165 .60841 .55126 .45289 .37243 .30656 .20829 · 14205 .09722

41 .66500 .60090 .54312 .44401 .36335 .29763 .20028 · 13528 .0917242 .65842 .59348 .53509 .43530 .35448 .28896 . H1257 .12884 .0865343 .65190 .58616 .52718 .42677 .34584 .28054 . 18517 · 12270 .0816344 .64545 .57892 .51939 .41840 .33740 .27237 .17805 · 11686 .0770145 .63905 .57177 .51171 .41020 .32917 .26444 .17120 .11130 .0726546 .63273 .56471 .50415 .40215 .32115 .25674 .16461 .106ÓO .0685447 .62646 .55774 .49670 .39427 .31331 .24926 .15828 .10095 .0646648 .62026 .55086 .48936 .38654 .30567 .24200 .15219 .09614 .0610049 .61412 .54406 .48213 .37896 .29822 .23495 .14634 .09156 .05755

50 .60804 .53734 .47500 .37153 .29094 .22811 .14071 .08720 .05429

303

http://carlos2524.jimdo.com/

Page 312: Algebra Superior Murray R Spiegel

304

Capital de una anualidad Sñ1 i(l+i)n-1

i

rx 1% 1-4% 1~% 2% 2,% 3% 4% 5% 6%1 1. 0000 1. 0000 1. 0000 1. 0000 1. 0000 1.0000 1. 0000 1. 0000 1. 00002 2.0100 2.0125 2.0150 2.0,200 2.0250 2.0300 2.0400 2.0500 2.06003 3.0301 3.0377 3.0452 3.0604 3.0756 3.0909 3.1216 3. 1525 3.18364 4.0604 4.0756 4.0909 4. 1216 4.1525 4.1836 4.2465 4.3101 4.37465 5.1010 5.1266 5.1523 5.2040 5.2563 5.3091 5.4163 5.5256 5.63716 6.1520 6.1907 6.2296 6.3081 6.3877 6.4684 6.6330 6.8019 6.97537 7.2135 7.2680 7.3230 7.4343 7.5474 7.6625 7.8983 8.1420 8.39388 8.285~ 8.3589 8.4328 8.5830 8.7361 8.8923 9.2142 9.5491 9.89759 9.3685 9.4634 9.5593 9.7546 9.9545 10. 1591 10.5828 11. 0266 11. 4913

10 10.4622 10.5817 10.7027 10.9497 11.2034 11. 4639 12.0061 12.5779 13.1808

11 11.5668 11. 7139 11.8633 12.1687 12.4835 12.8078 13.4864 14.2068 14.971612 12.6825 12.8604 13.0412 13.4121 13.7956 14. 1920 15.0258 15.9171 16.869913 13.8093 14.0211 14.2368 14.6803 15. 1404 15.6178 16. 6268 17.7130 18.882114 14.9474 15.1964 15.4504 15.9739 16.5190 17.0863 18.2919 19.5986 21. 015115 16.0969 16.3863 16.6821 17.2934 17.9319 18.5989 20.0236 21. 5786 23.276016 17.2579 17.5912 17.9324 18.6393 19.3802 20.1569 21. 8245 23.6575 25.672517 18.4304 18.8111 19.2014 20.0121 20.8647 21. 7616 23.6975 25.8404 28.212918 19.6147 20.0462 20.4894 21.4123 22.3863 23.4144 25.6454 28.1324 30.905719 20.8109 21. 29f:8 21. 7967 22.8406 23.9460 25.1169 27.6712 30.5390 33.7600

20 22.0190 22.5630 23. 1237 24.2974 25.5447 26.8704 29.7781 33.0660 36.7856

21 23.2392 23.8450 24.4705 25.7833 27.1833 28.6765 31. 9692 35.7193 39.992722 24.4716 25.1431 25.8376 27.2990 28.8629 30.5368 34.2480 38.5052 43.392323 25.7163 26.4574 27.2251 28.8450 30.5844 32.4529 36.6179 41. 4305 46.995824 26.9735 27.7881 28.6335 30.4219 32.3490 34.4265 39.0826 H.5020 50.815625 28.2432 29.1354 30.0630 32.0303 34.1578 36.4593 41. b459 47.7271 54.8E4526 29.5256 30.4996 31. 5140 33.6709 36.0117 38.5530 44.3117 51. 1135 59.156427 30.8209 31. 8809 32.9867 35.3443 37.9120 40.7096 47.0842 54.6691 63.705828 32.1291 33.2794 34.4815 37.0512 39.8598 42.9309 49.9676 58.4026 68.528129 33.4504 34.6954 35.9987 38.7922 41.8563 45.2189 52.9663 62.3227 73.6398

30 34.7849 36. 1291 37.5387 40.5681 43.9027 47.5754 . 56.0849 66.4388 79.0582

31 36.1327 37.5807 39.1018 42.3794 46.0003 50.0027 59.3283 70.7608 84.801732 37.4941 39.0504 40.6883 44.2270 48.1503 52.5028 62.7015 75.2988 90.889833 38.8690 40.5386 42.2986 46.1116 50.3540 55.0778 66.2095 80.0638 97.343234 40.2577 42.0453 43.9331 48.0338 52.6129 57.7302 69.8579 85.0670 104.183835 41. 6603 43.5709 45.5921 49.9945 54.9282 60.4621 73.6522 90.3203 111. 434836 43.0769 45.1155 47.2760 51. 9944 57.3014 63.2759 77.5983 95.8363 119.120937 44.5076 46.6794 48.9851 54.0343 59.7339 66. 1742 81. 7022 101. 6281 127.268138 45.9527 48.2629 50.7199 56.1149 62.2273 69. 1594 85.9703 107.7095 135.904239 47.4123 49.8662 52.4807 58.2372 64.7830 72.2342 90.4091 114.0950 145.0585

40 48.8864 51. 4896 54.2679 60.4020 67.4026 75.4013 95.0255 120.7998 154.7620

41 50.3752 53.1332 56.0819 62.6100 70.0876 78.6633 99.8265 127.8398 165.047742 51. 8790 54.7973 57.9231 64.8622 72.8398 82.0232 104.8196 135.2318 175.950543 53.3978 56.4823 59.7920 67. 1595 75.6608 85.4839 110.0124 142.9933 187.507644 54.9318 58.1883 61.6889 69.5027 78.5523 89.0484 115.4129 151. 1430 199.758045 5"6.4811 59.9157 63.6142 - 71. 8927 81. 5161 92.7199 121. 0294 159.7002 212.743546 58.0459 6.1.6646 65.5684 74.3306 84.5540 96.5015 126.8706 168.6852 226.508147 59.6263 63.4354 67.5519 76.8172 87.6679 100.3965 132.9454 178.1194 241. 098648 61. 2226 65.2284 69.5652 79.3535 90.8596 104.4084 139.2632 188.0254 256.564549 62.8348 67.0437 -71. 6087 81. 9406 94.1311 108.5406 145.8337 198.4267 272.9584

50 64.4632 68.8818 73.6828 84.5794 97.4843 112.7969 152.6671 209.3480 290.3359

http://carlos2524.jimdo.com/

Page 313: Algebra Superior Murray R Spiegel

Valor actual de una anualidad 1 - (1 + i)- n

~ 1% 1-4% 1~% 2% 2~% 3% 4% 5% 6%1 0.9901 0.9877 0.9852 0.9804 0.9756 0.9709 0.9615 0.9524 0.94342 1. 9704 1. 9631 1. 9559 1. 9416 1. 9274 1. 9135 1. 8861 1. 8594 1. 83343 2.9410 2.9265 2.9122 2.8839 2.8560 2.8286 2.7751 2.7232 2.67304 3.9020 3.8781 3.8544 3.8077 3.7620 3.7171 3.6299 3.5460 3.46515 4.8534 4.8178 4.7826 4.7135 4. 6458 4.5797 4.4518 4.3295 4.21246 5.7955 5.7460 5.6972 5.6014 5.5081 5.4172 5.2421 5.0757 4.91737 6.7282 6.6627 6.5982 6.4720 6.3494 6.2303 6.0021 5.7864 . 5.58248 7.6517 7.5681 7.4859 7.3255 7.1701 7.0197 6.7327 6.4632 6.20989 8.5660 8.4623 8.3605 8.1622 7.9709 7.7861 7.4353 7.1078 6.8017

10 9.4713 9.3455 9.2222 8.9826 8.7521 8.5302 8.1109 7.7217 7.360111 10.3676 10.2178 10.0711 9.7868 9.5142 9.2526 8.7605 8.3064 7.886912 11.2551 11. 0793 10.9075 10.5753 10.2578 9.9540 9.3851 8.8633 8.383813 12.1337 11. 9302 11. 7315 11. 3484 10.9832 10.6350 9.9856 9.3936 8.852714 13.0037 12.7706 12.5434 12. 1062 11. 6909 11. 2961 10.5631 9.8986 9.295015 13.8651 13.6005 13.3432 12.8493 12.3814 11. 9379 11. 1184 10.3797 9.712216 14.7179 14.4203 14.1313 13.5777 13.0550 12.5611 11. 6523 10.8378 10.105917 15.5623 15.2299 14.9076 14.2919 13.7122 13.1661 12.1657 11.2741 10.477318 16.3983 16.0295 15.6726 14.9920 14.3534 13.7535 12. 6593 11.6896 10.827619 17.2260 16.8193 16.4262 15.6785 14.9789 14.3238 13.1339 12.0853 11.158120 18.0456 17.5993 17.1686 16.3514 15.5892 14.8775 13.5903 12.4622 11.469921 18.8570 18.3697 17.9001 17.0112 16. 1845 15.4150 14.0292 12.8212 11.764122 19.6604 19. 1306 18.6208 17.6580 16.7654 15.9369 14.4511 13.1630 12.041623 20.4558 19.8820 19.3309 18.2922 17.3321 16.4436 14.8568 13.4886 12.303424 21. 2434 20.6242 20.0304 18.9139 17.8850 16.9355 15.2470 13.7986 12.550425 22.0232 21. 3573 20.7196 19.5235 18.4244 17.4131 15.6221 14.0939 12. 783426 22.7952 22.0813 21. 3986 20. 1210 18.9506 17.8768 15.9828 14.3752 13.003227 23.5596 22.7963 22.0676 20.7069 19.4640 18.3270 16.3296 14.6430 13.210528 24.3164 23.5025 22.7267 21.2813 19.9649 18.7641 16.6631 14.8981 13.406229 25.0658 24.2000 23.3761 21. 8444 20.4535 19.1885 16.9837 15.1411 13.5907'30 25.8077 24.8889 24.01-58 22.3965 20.9303 19.6004 17.2920 15.3725 13.764831 26.5423 25.5693 24.6461 22.9377 21. 3954 20.0004 17.5885 15.5928 13.929132 27.2696 26.2413 25.2671 23.4683 21. 8492 20.3888 17.8736 15.8027 14.084033 27.9897 26.9050 25.8790 23.9886 22.2919 20.7658 18.1476 16.0025 14.230234 28.7027 27.5605 26.4817 24.4986 22.7238 21. 1318 18.4112 16.1929 14.368135 29.4086 28.2079 27.0756 24.9986 23.1452 21. 4872 18.6646 16.3742 14.498236 30. 1075 28.8473 27.6607 25.4888 23.5563 21. 8323 18.9083 16.5469 14.621037 30.7995 29.4788 28.2371 25.9695 23.9573 22. 1672 19. 1426 16.7113 14.736838 31. 4847 30.1025 28.8051 26.4406 24.3486 22.4925 19.3679 16.8679 14.846039 32. 1630 30.7185 29.3646 26.9026 24.7303 22. 8082 19.5845 17.0170 14.9491

40 32.8347 31.32C9 29.9158 27.3555 25. 1028 23.1148 19.7928 17.1591 15.0463

41 33.4997 31. 9278 30.4590 27.7995 25.4661 23.4124 19.9931 17.2944 15.138042 34.1581 32.5213 30.9941 28.2348 25.8206 23.7014 20.1856 17.4232 15.22~543 34.8100 33. 1075 31. 5212 28.6616 26.1664 23.9819 20.3708 17.5459 15.306244 35.4555 33.6864 32.0406 29.0800 26.5038 24.2543 20.5488 17.6628

1

15.383245 36.0945 34.2582 32.5523 29.4902 26.8330 24.5187 20.7200 17.7741 15.~55846 36.7272 34.8229 33.0565 29.8923 27. 1542 2.•. 7754 20. 8847 17.8801 15.52-1447 37.3537 35.3806 33.5532 30.2866 27.4675 2C,.0247 21. 0429 17.9810 15.589048 37.9740 35.9315 3-1.0426 30.6731 27.7732 25.2667 21. 1951 18.0772 15.650049 38.5881 36.4755 34.5247 31.0521 28.0714 25.5017 21.3415 18.1587 15.7076

50 39.1961 37.0129 34.9997 31. 4236 28.3623 25. 7~98 21. ~822 18.2559 15.7619

305

http://carlos2524.jimdo.com/

Page 314: Algebra Superior Murray R Spiegel

AA

A

AA

AAA

AAAAAA

AA

A

AA

AA

B,

Bi

http://carlos2524.jimdo.com/

Page 315: Algebra Superior Murray R Spiegel

INDICE

Abscisa, 77Absoluta, convergencia, 283, 292

desigualdad, 167Absoluto, valor, 2, 6, 7

de un número complejo, 172, 174Acotada, sucesión monótona, 282, 287Actual, valor, 222, 223

de una anualidad, 223, 226, 227Adición (véase Suma)Adjunto, 262, 267-269Algebra, operaciones fundamentales,

teorema fundamental, 183Algebraica, expresión, 11Alternadas, series, 283, 291Amortización, 227Antilogaritmo, 211, 215Anual, renta, 222Anualidad, 222, 223, 226, 227

tablas, 304, 305Argumento de un número complejo, 172, 174Aritmética, media, 144, 170

medias, 141, 144, 145progresión, 140, 142-145

Armónica, media, 149, 170progresión, 141, 149, 150serie, 283, 289, 290, 292

Armónicas, medias, 141, 149, 150Asociación, de términos, factores comunes, 27, 31

símbolos, 12Asociativa, propiedad, 3, 5Axiomas de la igualdad, 67, 68, 70, 71

Base, de las potencias, 3, 42de los logaritmos, 209, 210

Binomio, 11coeficientes, 156demostración, exponentes enteros y positivos, 165,

166desarrollo, fórmula o teorema, 155-161

Briggs, sistema de logaritmos, 210

Cálculo, 81, 83Campo de variación de una variable, 76Características de un logaritmo, 210, 211, 215Cero, 1, 2

división por, 1, 5exponente, 43, 45grado, 12multiplicación por, 2

Circulares, permutaciones, 230, 235Circunferencia, 81, 127, \30Cociente, 1 (véase también División)Coeficientes, 11

del binomio, 156relaciones con las raíces, 185, 198-200

Cologaritmo, 212, 217, 218Combinaciones, 229-231, 235-238Comparación, criterio, 283, 289, 290

series, 283Compatibles, ecuaciones, 101, 263, 271Complejas, fracciones, 37, 39, 40

raíces de ecuaciones, 111, 116, 183, 190Complejo, número, 63-65, 172-179

amplitud, 172, 174, 175argumento, 172, 174, 175conjugado, 63división en forma polar, 173forma polar, 172, 174, 175forma rectangular, 172, 176igualdad, 63imaginario puro, 63módulo, 172, 174, 175multiplicación en forma polar, 173operaciones algebraicas, 63-65parte imaginaria, 63parte real, 63potenciación, 63, 65, 173, 177radicación en forma polar, 173, 177-179representación gráfica, 172suma y resta gráficamente, 174teorema de Moivre, 173valor absoluto, 172, 174, 175

307

INDICE

Abscisa, 77 Absoluta, convergencia, 283, 292

desigualdad, 167 Absoluto, valor, 2, 6, 7

de un número complejo, 172, 174 Acotada, sucesión monótona, 282, 287 Actual, valor, 222, 223

de una anualidad, 223, 226, 227 Adición (véase Suma) Adjunto, 262, 267-269 Algebra, operaciones fundamentales,

teorema fundamental, 183 Algebraica, expresión, 11 Alternadas, series, 283, 291 Amortización, 227 Antilogaritmo, 211, 215 Anual, renta, 222 Anualidad, 222, 223, 226, 227

tablas, 304, 305 Argumento de un número complejo, 172, 174 Aritmética, media, 144, 170

medias, 141 , 144, 145 progresión, 140, 142-145

Armónica, media, 149, 170 progresión, 141, 149, 150 serie, 283, 289, 290, 292

Armónicas, medias, 141, 149, 150 Asociación, de términos, factores comunes, 27, 31

símbolos, 12 Asociativa, propiedad, 3, 5 Axiomas de la igualdad, 67, 68, 70, 71

Base, de las potencias, 3, 42 de los logaritmos, 209, 210

Binomio, 11 coeficientes, 156 demostración, exponentes enteros y positivos, 165,

166 desarrollo , fórmula o teorema, 155-161

Briggs, sistema de logaritmos, 210

307

Cálculo, 81 , 83 Campo de variación de una variable, 76 Características de un logaritmo, 210, 211, 215 Cero, 1, 2

división por, 1, 5 exponente, 43, 45 grado, 12 multiplicación por, 2

Circulares, permutaciones, 230, 235 Circunferencia, 81, 127, \30 Cociente, 1 (véase también División) Coeficientes, 11

del binomio, 156 relaciones con las raíces, 185, 198-200

Cologaritmo, 212, 217, 218 Combinaciones, 229-231, 235-238 Comparación, criterio, 283, 289, 290

series, 283 Compatibles, ecuaciones, 101 , 263, 271 Complejas, fracciones, 37, 39, 40

raíces de ecuaciones, 111 , 116, 183, 190 Complejo, número, 63-65, 172-179

amplitud, 172, 174, 175 argumento, 172, 174, 175 conjugado, 63 división en forma polar, 173 forma polar, 172, 174, 175 forma rectangular, 172, 176 igualdad, 63 imaginario puro, 63 módulo, 172, 174, 175 multiplicación en forma polar, 173 operaciones algebraicas, 63-65 parte imaginaria, 63 parte real, 63 potenciación, 63, 65, 173, 177 radicación en forma polar, 173, 177-179 representación gráfica, 172 suma y resta gráficamente, 174 teorema de Moivre, 173 valor absoluto, 172, 174, 175

http://carlos2524.jimdo.com/

Page 316: Algebra Superior Murray R Spiegel

308

Compuesto, capital, 221, 222, 225interés, 221, 222, 225tablas, 303

Condicional, convergencia, 283, 292desigualdad, 167ecuación 67, 70

Cónicas, secciones, 127Conjugado de número complejo, 63, 183Conjugados, irracionales, 56Conmutativa, propiedad, 2, 3, 5Constante, 75

de proporcionalidad, 135-138Continuidad, 184Convergencia, 282-284, 288, 294

absoluta, 283, 292condicional, 283, 292criterio de comparación, 283, 289criterio del cociente, 283, 290de series alternadas, 283, 291intervalo, 284, 293

Conversión, periodo, 221Coordenadas cartesianas rectangulares, 76, 80

sistema, 76, 80Corchetes, 12Cotas, de sucesiones, 282, 287

superior e inferior, de raíces, 184, 193Cramer, regla, 253, 254, 256-258, 263, 270Criterio de cociente, 283, 290, 291

series de potencias, 284, 293, 294Criterios de convergencia, 283, 284, 288-294Cuadrado perfecto, trinomio, 27, 29Cuadrantes, 77Cuadrática, carácter de las raíces, 112, 116

de dos incógnitas, 110-122de una incógnita, 110-122discriminante, 111, 112, 116ecuación, 69,70, 110-122, 127-133formación, dadas las raíces. 112. 116función, 81irracional, 54, 56producto de raíces, 111, 116-118sistemas, 127-133suma de raíces. 111, 116-118tipos, 112, 119, 120

Cuadráticas, completando un cuadrado perfecto, 110.111. 113

ecuaciones con una incógnita, soluciones, 110-115por descomposición en factores, 110, 113por la fórmula, 111,114,115por métodos gráficos, 111, 115

Cuarta proporcional, 135-\36Cuarto grado, ecuación, 69, 70Cúbica, ecuación, 69. 70

Decimales periódicos. 148. 149Def~tiva, ecuación. 68, 71Denominador, l. 35Dependiente. variable. 75Dependientes, ecuaciones, 101. 103, 263. 272

sucesos. 242, 243

INDiCE

reSEsusi:sotn

ElenElipEnteEnte

raEqui

fnEspeEspeEstaExpcExpcExtr:Extn

EcuDescartes, regla de los signos, 184, 194Descomposición en factores, teorema, 26Descuento, 222, 224, 225Desigualdades, 2, 6, 167-171

absolutas, 167; condicional, 167principios, 167, 168sentidos, 167; signos, 2, 167solución gráfica, 171

Determinantes, 252-258, 260-274de orden n, 260-274de segundo orden, 252, 255-257de tercer orden, 253-255, 257, 258desarrollo, 253, 260, 262en la solución de sistemas de ecuaciones lineales,

252-254, 256-258, 263, 270, 271propiedades, 261, 262, 265-267

Diferencia, 1de cuadrados, 27, 29de cubos, 27, 30, 31tabular, 211

Discriminante, 111, 112, 116Distributiva, propiedad de la multiplicación, 3Divergencia, 282, 283, 288-294

criterio de comparación, 283, 289del cociente, 283, 290

Dividendo, 1, 14División, 1, 3

de expresiones algebraicas, 14, 17, 18de fracciones, 4, 37, 38de números complejos, 64, 65, 173de radicales, 55, 60por cero, 1, 5regla de Rufini, 183, 187, 188

Divisor, 1, 14, 26máximo común, 28, 33primo, 26teorema, 183, 186, 187

Doble. raíz, 110, 183, 189

Fact,Fact,FacuFóm

deFórnFracFrac

al¡coeqim

e, base de los logaritmos neperianos, 291Ecuaciones, 67-73, 87-94, 100-106, 110-122, 127-133,

182-202, 252-258, 263, 270-272cond icionales, 67, 70cotas de las raíces, 184, 193, 194cuadráticas (véase Cuadráticas, ecuaciones)cúbicas, 69, 70de cuarto grado, 69, 70de quinto grado, 69de raíces conocidas, 112, 116, 117, 189de segundo grado, 112, 119, 120defectivas, 69, 70, 72, 182, 183entera, racional, 69, 182equivalentes. 68exponenciales, 218gráfica (véa.H' Gráficas)identidad. 67, 70irracionales, 68. 112, 118, 119lineales ("éase Lineales, ecuaciones)literales. 88miembros. 67

oppnsigsinsin

Funcdegrilinmtnoun

Func

Gen¡GeOl

pn

GeOlser

308

Compuesto, capital, 221, 222, 225 interés, 221 , 222, 225 tablas, 303

Condicional, convergencia, 283, 292 desigualdad, 167 ecuación 67, 70

Cónicas, secciones, 127 Conjugado de número complejo, 63, 183 Conjugados, irracionales, 56 Conmutativa, propiedad, 2, 3, 5 Constante, 75

de proporcionalidad, 135-138 Continuidad, 184 Convergencia, 282-284, 288, 294

absoluta, 283, 292 condicional, 283, 292 criterio de comparación, 283, 289 criterio del cociente, 283, 290 de series alternadas, 283, 291 intervalo, 284, 293

Conversión, periodo, 221 Coordenadas cartesianas rectangulares, 76, 80

sistema, 76, 80 Corchetes, 12 Cotas, de sucesiones, 282, 287

superior e inferior, de raíces, 184, 193 Cramer, regla, 253, 254, 256-258, 263, 270 Criterio de cociente, 283, 290, 291

series de potencias, 284, 293, 294 Criterios de convergencia, 283, 284, 288-294 Cuadrado perfecto, trinomio, 27, 29 Cuadrantes, 77 Cuadrática, carácter de las raíces, 112, 116

de dos incógnitas, 110-122 de una incógnita, 110-122 discriminante, 111 , 112, 116 ecuación, 69, 70, 110-122, 127-133 formación , dadas las raíces. 112. 116 función , 81 irracion~l, 54, 56 producto de raíces, 111 , 116-118 sistemas, 127-133 suma de raíces, 111 , 116-11 8 tipos, 112, 119, 120

Cuadráticas. completando un cuadrado perfecto, 110. 111. 113

ecuaciones con una incógnita, soluciones, 110-115 por descomposición en factores , 110, 113 por la fórmula, 111,114,115 por métodos gráficos, 111, 115

Cuarta proporcional, 135-136 Cuarto grado, ecuación, 69, 70 Cúbica , ecuación, 69, 70

Decimales periódicos. 148. 149 Def~tiva , ecuación. 68, 71 Denominador, l . 35 Dependiente. variable. 75 Dependientes, ecuaciones. 101 . 103, 263, 272

sucesos, 242, 243

INDiCE

Descartes, regla de los signos, 184, 194 Descomposición en factores, teorema, 26 Descuento, 222, 224, 225 Desigualdades, 2, 6, 167-171

absolutas, 167; condicional, 167 principios, 167, 168 sentidos, 167; signos, 2, 167 solución gráfica, 171

Determinantes, 252-258, 260-274 de orden n, 260-274 de segundo orden, 252, 255-257 de tercer orden, 253-255, 257, 258 desarrollo, 253, 260, 262 en la solución de sistemas de ecuaciones lineales,

252-254, 256-258, 263, 270, 271 propiedades, 261, 262, 265-267

Diferencia, 1 de cuadrados, 27, 29 de cubos, 27, 30, 31 tabular, 211

Discriminante, 111, 112, 116 Distributiva, propiedad de la multiplicación, 3 Divergencia, 282, 283, 288-294

criterio de comparación, 283, 289 del cociente, 283, 290

Dividendo, 1, 14 División, 1, 3

de expresiones algebraicas, 14, 17, 18 de fracciones, 4, 37, 38 de números complejos, 64, 65, 173 de radicales, 55, 60 por cero, 1, 5 regla de Rufini, 183, 187, 188

Divisor, 1, 14, 26 máximo común, 28, 33 primo, 26 teorema, 183, 186, 187

Doble, raíz, 110, 183, 189

e, base de los logaritmos neperianos, 291 Ecuaciones, 67-73, 87-94, 100-106, 110-122, 127-133,

182-202, 252-258, 263, 270-272 cond icionales, 67, 70 cotas de las raíces, 184, 193, 194 cuadráticas (véase Cuadráticas, ecuaciones) cúbicas, 69, 70 de cuarto grado, 69, 70 de quinto grado, 69 de raíces conocidas, 112, 116, 117, 189 de segundo grado, 112, 119, 120 defectivas, 69, 70, 72, 182, 183 entera, racional, 69, 182 equivalentes, 68 exponenciales, 218 gráfica (véa.H' Gráficas) identidad. 67, 70 irracionales, 68. 112, 118, 119 lineales (réase Lineales, ecuaciones) literales. 88 miembros. 67

http://carlos2524.jimdo.com/

Page 317: Algebra Superior Murray R Spiegel

Ecuaciones, número de raíces, 183racionales, enteras, 69, 182raíces (véase Raíces)raíces complejas, 183, 190raíces irracionales, 183, 190, 191redundantes, 68, 71simétricas, 128, 132, 133simultáneas (véase Sistemas de ecuaciones)sistemas de (véase Sistemas de ecuaciones)soluciones (véase Soluciones)transformación, 185, 196, .198

Elemento de un determinante, 252, 260Elipse, 127, 129, 130Enteras, raíces de una ecuación, 184Entero, número, 1, 2

racional, 11, 12, 69, 182Equivalentes, ecuaciones, 68

fracciones, 7, 35Especiales, productos, 21-24Esperanza matemática, 243, 244, 248Estadísticas, gráficas, 77, 84Exponencial, forma, 209, 212Exponenciales, ecuaciones, 21f>Extrañas, raíces o soluciones, 68, 71, 112, 118Extremos, 135

Factor (véase Divisor)Factores, de composición, 26-33Factorial, notación, 155, 156, 229Fórmula de resolución de una ecuación cuadrática,

111, 114, 115demostración, 114

Fórmulas, 68-72Fraccionarios, exponentes, 43-45Fracciones, 4, 7, 8, 35-40

algebraicas racionales, 35, 275complejas, 37, 39, 40equivalentes, 7, 35impropias', 275operaciones, 4, 8, 36-39propias, 275signos, 4, 35, 36simples, 275-279simplificadas, 35, 37

Función, 75-81de segundo grado, 81gráfica, 77, 80, 81lineal, 80multiforme, 75notación, 76, 79uniforme, 75

Fundamental. teorema de álgebra, 183

General, término enésimo, 280, 284, 285Geométrica, medida, 147, 170

progresión, 140, 145-149de infinitos términos. 141, 148. 149

Geométricas, medias, 141, 147. 148series infinitas, 141. 148. 283

INDlCE

Grado, 12, 15, 69, 70, 72, 182, 183cero o nulo, 12de un monomio, 12de un polinomio, 12, 15, 69, 70, 182de un término, 69de una ecuación racional entera, 69, 72de una expresión. 69

Gráfica, solución de ecuaciones, 83,100-102,111, 115,127, 130, 184, 192, 193

Gráficas. 76, 77, 80-84de ecuaciones, de funciones, 81, 83, 100-102. 111,

115, 127, 129, 130, 184, 192, 193de ecuaciones de segundo grado con dos incógni-

tas, 127, 129, 130de ecuaciones lineales de dos incógnitas, 100

Hipérbola, 127, 129, 130Homogéneas, ecuaciones lineales, 263. 272Homer, método, 185, 200-203

i, p, 63, 172Identidad, 67, 70

de polinomio, 183, 189Imaginaria, parte de un número complejo. 63

unidad, 63Imaginarias, raíces (véase Complejas raíces)Imaginarios, números, '2,63 (véase también Complejos)Impropia, fracción, 275Incógnitas, 67, 71, 86, 100, 110, 127Incompatibles, ecuaciones, 101, 106. 263. 271Independiente, variable, 75Independientes, sucesos, 242Indice, 42, 53

de una raíz. 53reducción, 54, 57

Inducción matemática, 163-166Inferior, cota o límite de raíces, 184, 193Infinita, progresión geométrica o serie, 141, 148, 149,

283series, 280-298 (véase también Series)

convergencia absoluta y condicional, 283, 284,292, 293

convergencia y divergencia, 282-294criterios de convergencia, 283, 284, 288-294suma, 141, 148, 282. 288sumas parciales, 282, 288

sucesión. 280 (véase también Series)límite, 281, 286, 287

Infinito. 141, 281Interés, 221-225

compuesto. 221. 222. 225periodo de conversión. 221rédito. 221. 225simple. 221. 223. 224

Interpelación. de logarumos, 211lineal. 193

Intervalo de convergencia. 284. 293. 294Inversiones. 260. 264Irracional. numero. 1. 54

309

Ecuaciones, número de raíces, 183 racionales, enteras, 69, 182 raíces (véase Raíces) raíces complejas, 183, 190 raíces irracionales, 183, 190, 191 redundantes, 68, 71 simétricas, 128, 132, 133 simultáneas (véase Sistemas de ecuaciones) sistemas de (véase Sistemas de ecuaciones) soluciones (véase Soluciones) transformación, 185, 196, ,198

Elemento de un determinante, 252, 260 Elipse, 127, 129, 130 Enteras, raíces de una ecuación, 184 Entero, número, 1, 2

racional, 11 , 12, 69, 182 Equivalentes, ecuaciones, 68

fracciones, 7, 35 Especiales, productos, 21-24 Esperanza matemática, 243, 244, 148 Estadísticas, gráficas, 77, 84 Exponencial, forma, 209, 212 Exponenciales, ecuaciones, 21!i Extrañas, raíces o soluciones, 68, 71 , 112, 118 Extremos, 135

Factor (véase Divisor) Factores, de composición, 26-33 Factorial, notación, 155, 156, 229 Fórmula de resolución de una ecuación cuadrática,

111, 114, 115 demostración, 114

Fórmulas, 68-72 Fraccionarios, exponentes, 43-45 Fracciones, 4, 7, 8, 35-40

algebraicas racionales, 35, 275 complejas, 37, 39, 40 equivalentes, 7, 35 impropias', 275 operaciones, 4, 8, 36-39 propias. 275 signos, 4, 35, 36 simples, 275-279 simplificadas, 35, 37

Función, 75-81 de segundo grado, 81 gráfica, 77, 80, 81 lineal, 80 multiforme, 75 notación, 76, 79 uniforme. 75

Fundamental. teorema de álgebra, 183

General . término enésimo, 280, 284. 285 Geométrica, medida, 147. 170

progresión, 140, 145-149 de infinitos términos. 141 , 148. 149

Geométricas, medills, 141, 147. 148 series infinitas. 141. 148. 283

INDlCE

Grado, 12, 15, 69, 70, 72, 182, 183 cero o nulo, 12 de un monomio. 12 de un polinomio. 12, 15. 69, 70, 182 de un término, 69 de una ecuación racional entera, 69, 72 de una expresión. 69

309

Gráfica, solución de ecuaciones, 83,100-102,111,115. 127, 130, 184, 192, 193

Gráficas, 76, 77, 80-84 de ecuaciones, de funciones, 81, 83, 100- \02. 111 ,

115, 127, 129, 130, 184, 192, 193 de ecuaciones de segundo grado con dos incógni­

tas, 127, 129, 130 de ecuaciones lineales de dos incógnitas, 100

Hipérbola, 127, 129, 130 Homogéneas, ecuaciones lineales, 263. 272 Homer, método, 185, 200-203

i , p, 63, 172 Identidad, 67, 70

de polinomio, 183, 189 Imaginaria, parte de un número complejo. 63

unidad, 63 Imaginarias, raíces (véase Complejas raíces) Imaginarios, números, '2,63 (véase también Complejos) Impropia, fracción , 275 Incógnitas, 67, 71 , 86, 100, 110, 127 Incompatibles. ecuaciones, \01, \06. 263. 271 Independiente, variable, 75 Independientes. sucesos, 242 Indice, 42, 53

de una raíz. 53 reducción, 54, 57

Inducción matemática, 163-166 Inferior, cota o límite de raíces, 184, 193 Infinita, progresión geométrica o serie, 141. 148. 149,

283 series, 280-298 (véase también Series)

convergencia absoluta y condicional, 283. 284, 292, 293

convergencia y divergencia, 282-294 criterios de convergencia, 283, 284, 288-294 suma, 141 , 148, 282. 288 sumas parciales, 282, 288

sucesión. 280 (véase también Series) límite, 281 , 286, 287

Infinito. 141 , 281 Interés. 221-225

compuesto. 221. 222. 225 periodo de conversión . 221 rédito. 22\. 225 simple. 221. 223. 224

Interpolación . de loga ritmos. 211 lineal. 193

Interva lo de convergencia . 284. 293. 294 Inversiones. 260. 264 Irracional. numero. 1. 54

http://carlos2524.jimdo.com/

Page 318: Algebra Superior Murray R Spiegel

310

Irracionales, 54, 56binomios cuadráticos. 56ecuaciones, 68, 112, 118, 119

Irracionales. raíces, 183, 184método de Horncr. 185, 200-203obtención gráfica, 83, 184, 192, 193

Irracionalidad, demostraciones, 73, 192

Límite de una sucesión, 281, 286, 287Lineal, función, 80

interpolación, 193Lineales, ecuaciones, 69, 70, 100-106, 252-259, 263,

270-272compatibles, 101, 263, 271. 272de una incógnita, 87-94dependientes, 101, 103, 263, 272homogéneas, 263, 272incompatibles, 101, 106, 263, 271, 272sistemas, 100-106, 252-254, 256-258, 263, 270, 271solución de sistemas, por determinantes, 252-254,256-258, 263, 270, 271

Li terales, I 1Logaritmos, 209-218

base de los neperianos, 291base del sistema, 209, 210característica de los vulgares o decimales, 210, 211,

215decimales o vulgares, 210-212, 215-218mantisa de los vulgares o decimales, 210, 211propiedades, 209, 213, 214sistema de Briggs, 210

Llaves, 12

Mantisa, 210, 211Matemática, esperanza, 243, 244, 248

inducción, 163-166Máximo común divisor, 28, 33Máximos relativos, 81

aplicaciones, 82, 83Mayor que, 2, 167Media proporcional, 135-137Medias, de una progresión, 141, 144, 147-150

de una proporción, 135Menor que, 2, 167Menores complementarios, 262, 267-269Miembros de una ecuación, 67Mínimo, común múltiple, 28, 33, 36

denominador común, 8, 36Mínimos, relativos, 80, 81

aplicaciones, 83Minuendo, 13Módulo de un número complejo, 172, 174, 175Moivre, teorema. 173Monomio, II (I'éase también Término]Monomio, factor común, 27, 28Monótona, sucesión, 282. 287Multiforme, función, 75

lNDlCE

Multinomio, 11, 12Multiplicación, 1

conmutativa, 3, 5de expresiones algebraicas, 13, 16, 17de fracciones, 4, 36; de' radicales, 55, 59de números complejos, 64, 65, 173distributiva, 3, 5por cero, 2propiedad asociativa, 3, 5regla de los signos, 3

PoI

Po

f1

POI

PriPri

Natural, número, 1Negativo, número, 2Nominal, rédito, 221, 225Notación de sumación en series, 280, 285, 286Numerador, 1, 35Numérico, coeficiente, 2Número, complejo (véase Complejo)

imaginario, 2, 63 (véase Complejo)irracional, 1, 54natural, 1negativo, 2operaciones en el campo real, 1-8positivo, 1, 2primo, 26racional, 1, 2real, 1, 2real representación gráfica, 2, 6valor absoluto, 2, 6, 7

rPrc

ee

oPrc

dd

ProPro

aag

ProProProPUl

PUl

PUl

Operaciones fundamentales, 1-3, 11-18Orden, de los números reales, 2

de un determinante, 252, 260de una raíz (véase Indice de una raíz)

Ordenada, 77Origen, 2

de un sistema cartesiano rectangular, 76

Qui

Ra(n

RacRa(Ra(

eee:e:Ít

nr

Pago, periodo o intervalo, 222Parábola, 80, 81, 127, 129, 130

vértice, 80, 81Parciales, sumas de series, 282, 288Paréntesis, 1, 12Pascal, triángulo, 156Perfectas, potencias enésimas, 54Periódicos, números, 148, 149Permutaciones, 229-235Polar, forma de un complejo, 172, 174, 175Polinomios, 12-15, 26, 35, 69, 182, 183

factores o divisores, 26-33grado, 12, 15, 69, 70, 182idénticamente iguales, 183, 189operaciones, 12, 14, 16-18primos, 26

entre sí, 28Positivos, números, 1, 2Potencia, 3, 42 .

de un binomio, 21, 23, 155-161

RaRa

310

Irracionales, 54, 56 binomios cuadráticos, 56 ecuaciones, 68, 112, 118, 119

Irracionales, raíces, 183 , 184 método de Horncr. 185, 200-203 obtención gráfica, 83, 184, 192, 193

Irracionalidad, demostraciones, 73, 192

Límite de una sucesión, 281 , 286, 287 Lineal, función , 80

interpolación, 193 Lineales, ecuaciones, 69, 70, 100-106, 252-259, 263,

270-272 compatibles, 101, 263, 271. 272 de una incógnita, 87-94 dependientes, 101, 103, 263, 272 homogéneas, 263, 272 incompatibles, 101 , 106, 263, 271 , 272 sistemas, 100-106, 252-254, 256-258, 263, 270, 271 solución de sistemas, por determinantes, 252-254, 256-258, 263, 270, 271

Li terales, 1 1 Logaritmos, 209-218

base de los neperianos, 291 base del sistema, 209, 210 característica de los vulgares o decimales, 210, 211,

215 decimales o vulgares, 210-212, 215-218 mantisa de los vulgares o decimales, 210, 211 propiedades, 209, 213, 214 sistema de Briggs, 210

Llaves, 12

Mantisa, 210, 211 Matemática, esperanza, 243, 244, 248

inducción, 163-166 Máximo común divisor, 28, 33 Máximos relativos, 81

aplicaciones, 82, 83 Mayor que, 2, 167 Media rroporcional, 135-137 Medias, de una progresión, 141 , 144, 147-150

de una proporción, 135 Menor que, 2, 167 Menores complementarios, 262, 267-269 Miembros de una ecuación, 67 Mínimo, común múltiplo, 28, 33, 36

denominador común, 8, 36 Mínimos, relativos, 80, 81

aplicaciones, 83 Minuendo, 13 Módulo de un número complejo, 172, 174, 175 Moivre, teorema. 173 Monomio, 11 (I'éase tamhién Término) Monomio, factor común, 27, 28 Monótona, sucesión, 282, 287 Multiforme, función, 75

lNDlCE

Multinomio, 11, 12 Multiplicación, 1

conmutativa, 3, 5 de expresiones algebraicas, 13, 16, 17 de fracciones, 4, 36; de' radicales, 55, 59 de números complejos, 64, 65, 173 distributiva, 3, 5 por cero, 2 propíedad asocíativa, 3, 5 regla de los signos, 3

Natural, número, Negativo, número, 2 Nominal , rédito, 221, 225 Notación de sumación en series, 280, 285, 286 Numerador, 1, 35 Numérico, coeficiente, 2 Número, complejo (véase Complejo)

imaginario, 2, 63 (véase Complejo) irracional , 1, 54 natural , 1 negativo, 2 operaciones en el campo real, 1-8 positivo, 1, 2 primo, 26 racional, 1, 2 real, 1, 2 real representación gráfica, 2, 6 valor absoluto, 2, 6, 7

Operaciones fundamentales, 1-3, 11-18 Orden, de los números reales, 2

de un determinante, 252, 260 de una raíz (véase Indice de una raíz)

Ordenada, 77 Origen, 2

de un sistema cartesiano rectangular, 76

Pago, periodo o intervalo, 222 Pará\Jola, 80, 81 , 127, 129, 130

vértice, 80, 81 Parciales, sumas de series, 282, 288 Paréntesis, 1, 12 Pascal , triángulo, 156 Perfectas, potencias enésimas, 54 Periódicos, números, 148, 149 Permutaciones, 229-235 Polar, forma de un complejo, 172, 174, 175 Polinomios, 12-15, 26, 35, 69, 182, 183

factores o divisores, 26-33 grado, 12, 15,69,70, ' 182 idénticamente iguales, 183, 189 operaciones, 12, 14, 16-18 primos, 26

entre sí, 28 Positivos, números, 1, 2 Potencia, 3, 42 -

de un binomio, 21 , 23, 155-161

http://carlos2524.jimdo.com/

Page 319: Algebra Superior Murray R Spiegel

INDICE

Potencia, de un complejo, 63, 65, 173, 177logaritmo, 209

Potenciación, cero, 43, 45exponente, 3, 4, 42-50fraccionario, 43-45; entero, 42-44propiedades, 4, 43-47

Potencias, serie, 284, 293, 294Primo, factor o divisor, número, polinomio, 26Principal, 221, 223-225

raíz, 42Probabilidad, 242-250

de intentos repetidos, 243, 249, 250de sucesos, dependientes, 242, 243

independientes, 242que se excluyen mutuamente, 243, 244

Producto, 1de las raíces de una ecuación, 111, 116, 185, 198de segundo grado, 111, 116

Productos especiales, 21-24Progresiones, 140-150

aritmética, 140, 142-145armónicas, 141, 149, 150geométricas, 140, 141, 145-149

Propia, fracción, 275Proporción, 135-138Proporcionalidad, constante, 135-138Punto, coordenadas, 77, 80Pura, ecuación cuadrática, 11O, 113Puro, número imaginario, 63

Raíces, número de, 183racionales, 184, 191, 192, 194relación entre los coeficientes y, 185. 198transformación de ecuaciones por cambio de, 185,

196-198Razón. 135-137

de una progresión, aritmética, 140geométrica, 140

Real, número, 1, 2parte, de un complejo, 63representación gráfica, 2, 6

Recíproco, 4Recta, línea, 80, 100-102Rectangulares, coordenadas, 76, 77. 80, 81Rédito, 221, 225Redundantes, ecuaciones, 68, 71Regla de Rufini, 183, 187, 188Renta, anual, 221Resta, 1 (véase también Diferencia)

de complejos, 63-65de expresiones, algebraicas, 13. 16de fracciones, 4. 36, 39de radicales. 55, 58

Resto, 14. 182, 186-188teorema, 182, 186, 187

Semejantes. términos, 11Sentido de una desigualdad, 167Series, 280-298

alternadas, 283, 291de potencias, 284, 293, 294finitas, 280geométricas. 141, 148, 149, 283infinitas (véase Infinita, serie)notación, 280. 285, 286término enésimo o general, 280, 284, 285

Signos, 2, 3en una fracción, 4, 35, 36regla de Descartes, 184, 185. 194, 195reglas, 3

Símbolos de asociación, 12Simétricas, ecuaciones, 128, 132. 133Simple, interés, 221, 223, 224Simples, fracciones, 275, 279Simplificación, 35Simultáneas, cuadráticas, ecuaciones. 127-133

ecuaciones (véase Sistema de ecuaciones)lineales (véase Sistemas de ecuaciones lineales)

Sistemas de ecuaciones, 100-106, 127-133. 252-254.256-258. 263, 270-272

Sistemas de m ecuacione, con n incógnitas. 263Soluciones. 67. 68, 70-72 (véase también Raíces)

de un sistema de ecuaciones (réase Sistemas de ecua-ciones)

gráficas (l'éase Gráfica. solución de ecuaciones)extrañas. 68,112.118.119triviales. no triviales, 263: 272

Subradical, 53, 54Sucesión. 140. 280

acotadas, monótonas. 282. 2M7

Quinto grado, ecuación. 69

Racional, entero, 11. 12,69, 182número, 53

Racionales, raíces, 184, 191, 192, 194Racionalización, 54, 60Radicales, 53-60

cambio de forma, 54. 57, 58ecuación con, 68, 112, 118, 119en su forma más simple, 55-57extracción de potencias perfectas, 54índice u orden, 53multiplicación y división, 55, 59. 60racionalización, 54, 60reducción del índice, 54. 57semejantes, 55, 58suma algebraica, 55, 58

Radicando (véase Subradical)Raíces, 42, 67, 182 (véase también Soluciones)

carácter de, de una ecuación cuadrática, 112de complejos, 173, 177-179de ecuaciones cuadráticas, 110-122de una ecuación, 67dobles, 110. 183, 189enésimas, 42enésimas principales. 42enteras, 184extrañas, 68. 112, 118, 119irracionales (véase Irracionales, raíces)

311

Potencia, de un complejo, 63, 65, 173, 177 logaritmo, 209

Potenciación, cero, 43, 45 exponente, 3, 4, 42-50 fraccionario, 43-45; entero, 42-44 propiedades, 4, 43-47

Potencias, serie, 284, 293, 294 Primo, factor o divisor, número, polinomio, 26 Principal, 221, 223-225

raíz, 42 Probabilidad, 242-250

de intentos repetidos, 243, 249, 250 de sucesos, dependientes, 242, 243

independientes, 242 que se excluyen mutuamente, 243, 244

Producto, 1 de las raíces de una ecuación, 111, 116, 185, 198 de segundo grado, 111, 116

Productos especiales, 21-24 Progresiones, 140-150

aritmética, 140, 142-145 armónicas, 141, 149, 150 geométricas, 140, 141, 145-149

Propia, fracción, 275 Proporción, 135-138 Proporcionalidad, constante, 135-138 Punto, coordenadas, 77, 80 Pura, ecuación cuadrática, 11 O, 113 Puro, número imaginario, 63

Quinto grado, ecuación, 69

Racional, entero, 11 , 12,69, 182 número, 53

Racionales, raíces, 184, 191 , 192, 194 Racionalización, 54, 60 Radicales, 53-60

cambio de forma, 54, 57, 58 ecuación con, 68, 112, 118, 119 en su forma más simple, 55-57 extracción de potencias perfectas, 54 índice u orden, 53 multiplicación y división, 55, 59, 60 racionalización, 54, 60 reducción del índice, 54, 57 semejantes, 55, 58 suma algebraica, 55, 58

Radicando (véase Subradical) Raíces, 42, 67, 182 (véase también Soluciones)

carácter de, de una ecuación cuadrática, 112 de complejos, 173, 177-179 de ecuaciones cuadrá ticas, 110-122 de una ecuación, 67 dobles, 110, 183, 189 enésimas, 42 enésimas principales, 42 enteras, 184 extrañas, 68, 112, 118, 119 irracionales (véase Irracionales, raíces)

IN DICE

Raíces, número de, 183 racionales, 184, 191 , 192, 194

311

relación entre los coeficientes y, 185, 198 transformación de ecuaciones por cambio de, 185,

196-198 Razón, 135-137

de una progresión, aritmética, 140 geométrica, 140

Real, número, 1, 2 parte, de un complejo, 63 representación gráfica, 2, 6

Recíproco, 4 Recta, línea, 80, 100-102 Rectangulares, coordenadas, 76, 77, 80, 81 Rédito, 221 , 225 Redundantes, ecuaciones, 68, 71 Regla de Rufini, 183, 187, 188 Renta, anual, 221 Resta, 1 (véase también Diferencia)

de complejos, 63-65 de expresiones, algebraicas, 13 , 16 de fracciones, 4, 36, 39 de radicales, 55, 58

Resto, 14, 182, 186-188 teorema, 182, 186, 187

Semejantes, términos, 11 Sentido de una desigualdad, 167 Series, 280-298

alternadas, 283, 291 de potencias, 284, 293, 294 finitas, 280 geométricas, 141 , 148, 149, 283 infinitas (véase Infinita, serie) notación, 280, 285, 286 término enésimo o general, 280, 284, 285

Signos, 2, 3 en una fracción, 4, 35, 36 regla de Descartes, 184, 185, 194, 195 reglas, 3

Símbolos de asociación, 12 Simétricas, ecuaciones, 128, 132, 133 Simple, interés, 221, 223, 224 Simples, fracciones, 275, 279 Simplificación, 35 Simultáneas, cuadráticas, ecuaciones, 127-133

ecuaciones (véase Sistema de ecuaciones) lineales (véase Sistemas de ecuaciones lineales)

Sistemas de ecuaciones, 100-106, 127-133, 252-254, 256-258, 263, 270-272

Sistemas de m ecuacione. con n incógnitas, 263 Soluciones, 67, 68, 70-72 (veiase también Raíces)

de un sistema de ecuaciones (réase Sistemas de ecua-ciones)

gráficas (réase Gráfica. solución de ecuaciones) extrañas. 68, 112, 11 8, 119 triviales, no triviales, 263; 272

Subradical . 53 , 54 Sucesión. 140. 280

acotadas, monótonas. 282. 287

http://carlos2524.jimdo.com/

Page 320: Algebra Superior Murray R Spiegel

312

Sucesión. finitas. 280infinitas, 280 (réase también Infinita sucesión)monótonas creciente o decreciente. 280. 284, 285término enésimo o general. 280. 284, 285

Sucesos que se excluyen mutuamente. 243. 244Suma. I

asociativa, 3, 5conmutativa, 2, 5cuadrática o de segundo orden, 111, 116de dos cubos, 27, 30, 31de expresiones algebraicas, 12, 16de fracciones, 4, 8, 36, 39de las raíces de una ecuación, 111, 116, 185, 198de números complejos, 63-65de radicales, 55, 58de series infinitas, 141, 148, 282, 288de una progresión aritmética, 140, 142-144de una progresión geométrica, 141, 145-147infinita, 141, 148parcial, í82, 288propiedades asociativas, 3, 5reglas de los signos, 3

Sustraendo, 13

INDICE

Tablas, de anualidades, 304de interés compuesto, 302de logaritmos decimales o vulgares, 300, 301del valor actual, 303del valor actual de una anualidad, 305

Tabular, diferencia. 211Término, 1I

entero y racional, 11, 12, 69grado, 12, 69

Términos, de una progresión, 140de una serie, 280de una sucesión, 140, 280semejantes, 11

Transposición, 67, 68, 167Trinomio, II

cuadrado, 21, 23factores o divisores, 27, 29, 30

Trivial solución, 263, 272

Uniforme, función, 75

Variable, 75, 100, 127Variación, 135, 137, 138

312

Sucesión. finitas . 280 infinita~. 280 (t'éase tamhién Infinita sucesión) monótonas creciente o decreciente. 280. 284. 285 término enésimo o general. 280. 284, 285

Sucesos que se excluyen mutuamente. 243. 244 Suma. I

asociativa. 3, 5 conmutativa. 2. 5 cuadrática o de segundo orden, 111 , 116 de dos cubos. 27, 30, 31 de expresiones algebraicas, 12, 16 de fracciones. 4, 8, 36, 39 de las raíces de una ecuación, 111, 116, 185, 198 de números complejos, 63-65 de radicales, 55, 58 de series infinitas, 141, 148, 282, 288 de una progresión aritmética, 140, 142-144 de una progresión geométrica, 141, 145-147 infinita, 141 , 148 parcial, . i82, 288 propiedades asociativas, 3, 5 reglas de los signos, 3

Sustraendo, 13

INDICE

Tablas, de anualidades, 304 de interés compuesto, 302 de logaritmos decimales o vulgares, 300, 301 del valor actual , 303 del valor actual de una anualidad, 305

Tabular, diferencia. 211 Término, 11

entero y racional, 11 , 12, 69 grado, 12, 69

Términos, de una progresión, 140 de una serie. 280 de una sucesión, 140, 280 semejantes. 11

Transposición, 67, 68, 167 Trinomio, 11

cuadrado, 21 , 23 factores o divisores, 27, 29, 30

Trivial solución, 263, 272

Uniforme, función , 75

Variable, 75, 100, 127 Variación, 135, 137, 138

http://carlos2524.jimdo.com/

Page 321: Algebra Superior Murray R Spiegel

http://carlos2524.jimdo.com/

Page 322: Algebra Superior Murray R Spiegel

http://carlos2524.jimdo.com/

Page 323: Algebra Superior Murray R Spiegel

/.. http://carlos2524.jimdo.com/

Page 324: Algebra Superior Murray R Spiegel

• El álgebra constituye la base sobre la que se apoya laalta matemática y es el lenguaje en que se expresanla ciencia y técnica modernas. Problemas de difícil solu-ción a partir de un planteamiento aritmético se resuel-ven mucho más fácilmente si se plantean en términosalgebraicos.

• El propósito de este libro es, en esencia, proporcionaral alumno los conocimientos necesarios para llegar a do-minar este campo fundamental de la matemática. Ade-mas, puede ser de considerable utilidad para aquellosotros que deseen repasar sus principios fundamentales yaplicaciones como introducción a ulteriores estudios dematemáticas, ciencias o ingeniería.

• En este libro el estudio que se hace de muchas de lasmaterias tratadas es más profundo y completo que el quese encuentra en la mayoría de los libros de texto; su expo-sición incluye el número complejo, la teoría de ecua-ciones, la combinatoria, el cálculo de probabilidades, lasdeterminantes y las series finitas.

ISBN 970-10-2172-X

McGraw-Hill InteramericanaEditores, S.A. de c.v. r:::fJ .~--A Subsidiary ofThe McGrawHill Companies '. ~

'¿. .9 789701021729

• El álgebra constituye la base sobre la que se apoya la alta matemática y es el lenguaje en que se expresan la ciencia y técnica modernas. Problemas de difícil solu­ción a partir de un planteamiento aritmético se resuel­ven mucho más fácilmente si se plantean en términos algebraicos.

• El propósito de este libro es, en esencia, proporcionar al alumno los conocimientos necesarios para llegar a do­minar este campo fundamental de la matemática. Ade­mas, puede ser de considerable utilidad para aquellos otros que deseen repasar sus principios fundamentales y aplicaciones como introducción a ulteriores estudios de matemáticas, ciencias o ingeniería.

• En este libro el estudio que se hace de muchas de las materias tratadas es más profundo y completo que el que se encuentra en la mayoría de los libros de texto; su expo­sición incluye el número complejo, la teoría de ecua­ciones, la combinatoria, el cálculo de probabilidades, las determinantes y las series finitas .

I S BN 97 0-1 0 - 2172 - X

.. .. .... ..

McGraw-Hill Interamericana Editores, S.A. de c.v. g-z. .. ::

A 5ubsidiary ofTheMcGrawHill Companies . ~("Í.~:{ .J ..

'ft.,

http://carlos2524.jimdo.com/